You are on page 1of 513

1.

Even though it is not common, a new pleural effusion in a patient already on anti-tuberculosis treatment may occur and must be studied, because this can progress despite the clinical improvement of the patient. There is no need to change therapy, unless there is proof of a new infection or drug resistance. Some authors advocate the use of steroids in this setting, but this is not a universally accepted practice. There have been reported cases of new pleural effusions or lung infiltrates during the course of treatment of tuberculosis in patients who are recovering from the disease. The new effusions are usually exudates with lymphocytic predominance. Granulomatous lesions can be found on biopsies, but AFBs are not usually present. Thoracentesis is usually sufficient for diagnostic and therapeutic purposes. It is extremely rare to find new evidence of tuberculosis, as this new effusion is usually an enhanced immunologic response. It is not an indicator of failure of therapy or drug toxicity. If the patient is having systemic symptoms (such as ongoing fever, weight loss, etc.), a bacterial infection or empyema could be suspected, prompting the need for broad-spectrum antibiotic coverage, and maybe even chest tube placement 2. The consumption of undercooked meat during pregnancy may be associated with congenital toxoplasmosis, which can manifest as microcephaly or other abnormalities, such as chorioretinitis, MR, deafness, & seizures Domestic cats are definite hosts for T. gondii Humans can acquire the infection by: (1) the consumption of raw or undercooked meat of infected animals (including lamb, beef, or game), or (2) contact with cat feces Other congenital infections of the TORCH group can also result in microcephaly Asymmetric growth retardation (normal weight, decreased head circumference) is not characteristic for poor infant feeding Maternal smoking during pregnancy is associated with low-birth-weight infants, not microcephaly Excessive maternal caffeine consumption during pregnancy seems to be associated with an increased risk of spontaneous abortion and stillbirth Vast experience with the use of methyldopa & magnesium sulfate (for the treatment of PIH and preeclampsia) indicates that these drugs are unlikely to be associated with long-term adverse outcomes 3. Rupture of chordae tendineae should be suspected in healthy individuals who develop flash pulmonary edema (heart failure) associated with an acute mitral regurgitation. The patient presents with signs and symptoms of acute heart failure (sudden onset of shortness of breath, diaphoresis, Pallor). His EKG findings reveal occasional PVC; there are no signs of ischemia or ventricular hypertrophy. His pathologic murmur (systolic murmur that is heard in the apex, radiates to the axilla, increases with the grip maneuver, and decreases with Valsalva) is characteristic of mitral regurgitation, which may lead to acute heart failure. Acute MR is usually characterized by a soft, decrescendo systolic murmur (can be early, midsystolic or holosystolic), a decreased first heart sound, and the presence of a fourth heart sound. The 4 common causes of acute heart failure are papillary muscle rupture secondary to ischemia, infective endocarditis, rupture of chordae tendineae, and chest wall trauma secondary to mitral valve rupture. The most common cause of isolated, severe acute mitral regurgitation in adults is rupture of chordae tendineae with or without associated myxomatous disease. The diagnosis is confirmed by echocardiography. Myocardial infarction can be complicated by acute mitral regurgitation when there is rupture of the papillary muscle; however, this patient does not have evidence of ischemia in the EKG. Patients with pulmonary embolism can develop acute cor pulmonale; however, in such a setting, the EKG will show right axis deviation, right bundle branch block or both. The above patient clearly has acute pulmonary edema. Spontaneous papillary muscle rupture usually presents in elderly people who have acute chest pain or as a complication of MI. None of these are present here. The patient has features compatible with Ehlers-Danlos syndrome. This condition can cause a myxomatous degeneration of the mitral valve, leading to acute rupture of chordae tendineae. Pes planus and scoliosis are frequent, though not specific findings. The skin can be velvety or thin, and is usually covered with multiple characteristic "cigarette-paper" scars, due to its frailty and easy

bruisability. A past history of hernias and mitral valve prolapse is not uncommon. Joint hypermobility and skin hyperextensibility ("rubber man syndrome") can be dramatic in severe cases. Thyrotoxicosis can lead to acute, high-output, heart failure; however, this will not explain the MR. Skin changes of thyrotoxicosis can be confounded with those found in Ehlers-Danlos, but other characteristic signs & symptoms are lacking. Rheumatic fever is a common cause of mitral regurgitation in young individuals; however, the course of the disease is chronic, and the patients usually have evidence of left ventricular and atrial hypertrophy in the EKG. Marfan syndrome can affect the valvular apparatus, causing degeneration of the mitral & aortic valves. The presentation is usually chronic, progressive MR, and rarely, acute regurgitation due to ruptured chordae tendineae. This patient does not have arachnodactyly, loose joints or increased arm span, which are characteristic of Marfan syndrome. Educational Objective: Marfan or Ehlers-Danlos syndrome must be suspected in patients with connective tissue abnormalities and an acute MR secondary to chordae tendineae rupture, although a primary, pre-existing mitral valve prolapse (MVP) is the most common cause. Some cases may be idiopathic (individuals who experience rupture of the chordae tendineae without previous MVP or connective tissue disease), but an etiology can be found in most occasions. 4. Organ transplantation from cadaveric donors has become increasingly common in the past few years. Most of the organs are obtained from brain dead donors. The successful recovery of viable organs for transplantation depends on appropriate identification and medical care of brain dead patients. It also depends on the speed of recovery of the organs from the potential donors, with a shorter time interval between brain death and organ recovery leading to better outcomes. Any delay in organ procurement leads to an increase in the number and severity of complications; therefore, this patient should be immediately transferred to a specialized transplant center. Brain dead organ donors should ideally be managed in an intensive care setting. The goal of intensive medical care is to achieve hemodynamic stability and maintain physiologic homeostasis to improve the viability of the organs. Particular attention should be paid to maintain a normotensive and euvolemic state. One of the goals is to provide optimal ventilator support to prevent hypoxia and hypercapnia. Completely discontinuing or changing the ventilator setting is not appropriate when the patient is hemodynamically stable. Hypotension is very common in brain dead patients. It is related to the loss of sympathetic tone, systemic infections, volume depletion secondary to losses and diabetes insipidus. Adequate volume resuscitation is therefore an important step in ensuring donor organ viability. IV hydration should be continued to maintain euvolemia and an adequate urine output. Waiting for a cardiac arrest before procuring the organs is not the standard of care, and is not recommended. Educational Objective: Coordination of care between the different teams involved in organ procurement is a crucial step in the management of brain dead organ donors. 5. This patient is deeply demented, completely dependent on others for her care, and is most likely developing pneumonia. Her current condition has made her incapable of making any decisions regarding her health. She needs a surrogate decision-maker to speak on her behalf, and to preserve her right of autonomy. Her son seems to be the most appropriate person for this position, as he is the only direct relative who is constantly visiting her. The physician must always consider the patients clinical picture and prognosis, and respect the opinion of the surrogate decision-maker. Although it is very unusual, dementia can be an indicator of a terminal condition, and aspiration pneumonia commonly presents towards the last episodes of the disease. If the patient or surrogate feels that no further therapy should be given since the disease is non-reversible, and because the patients quality of life is poor, the physician has no right to give further treatment. In this case, the physician has to respect the sons request that his mother receive only oxygen therapy Studies have shown that it is safe to treat elderly patients with pneumonia in a nursing home if they are not critically ill. However, this patient should be transferred to the hospital if her son wants her to be treated with antibiotics and full supportive care. Pain treatment may eventually be needed if the shortness of breath does not improve with oxygen therapy; however, it is not indicated at this point. It is the physicians duty to always act in the patients best interest. In this case, he may at least alleviate the patients pain and give comfort, despite the seriousness of the disease. It is unethical to

do nothing even when the patients prognosis or quality of life is poor. Giving oxygen therapy is the most appropriate next step in management because aside from this step being requested by the surrogate, doing so may control the patients symptoms. Giving antibiotics to the patient may indicate the physicians inattention and lack of regard for the family (or surrogates) decision Educational Objective: Severely demented patients in nursing homes have a poor quality of life and can be viewed as individuals who are in the terminal phase of a prolonged and debilitating illness. The physician has no right to give or withhold potential life-saving measures based on only his own values, personal evaluation, or opinion of what the acceptable quality of life is. Decisions regarding the treatment of acute and life-threatening conditions have to be made with the surrogate, who acts to preserve the patients right to autonomy. On the other hand, the physician cannot be forced to provide unnecessary therapy only because the surrogate of relatives request so. 6. Studies have shown that adolescents in a private practice setting are very concerned about the prospect of gaining weight secondary to OCP use. However, available data has demonstrated that OCPs do not cause an increase in body weight or percent body fat. Occasionally adolescents voice other concerns about OCP use, including fears of developing blood clots, birth defects, & infertility, but these concerns are more common in the lower socioeconomic groups. Confidentiality concerns, cost, desire to have children, and partners opposition are rarely cited by adolescents as reasons not to use OCPs 7. Physicians are often faced with difficult scenarios involving the withdrawal of life-sustaining treatment. The situation may arise when a patient is rendered incompetent or is unable to participate in decision-making, and does not have any advance directives. In such situations, the physician must still recognize and respect the patients autonomy and right to make healthcare decisions. It is the physicians responsibility to act in the patients best interest by identifying a surrogate who must make healthcare decisions for the patient based on substituted judgment. The surrogate speaks on the patient's behalf, and must have the most knowledge on what the patient would have done or wanted if he were able to make his own healthcare decisions. The patients spouse or next of kin usually acts as the surrogate decision-maker in the absence of a formally or legally designated surrogate. The patients son appears to have some insight into his fathers wishes when he claimed that "he would have never wanted to live like this." The patient may have previously expressed his wishes regarding life-sustaining treatment to his son. The son should therefore be asked to provide more information and reasoning behind the decision to withdraw mechanical ventilation at this point. Telling the son that mechanical ventilation cannot be discontinued without an advance directive is incorrect. The son can assume the role of surrogate decision-maker as long as the physician believes that he is acting in the patients best interest. The physician should discuss the situation with the son in greater detail before involving the family. Understanding the reason and thoughts behind the son's opinion should be attempted to ascertain if he can act as the patient's surrogate decision maker. Once a surrogate is identified, the decision to withdraw or maintain ventilatory support can then be made. Important: Before making any decisions regarding the withdrawal of life support measures, it is important for a physician to act in the patient's best interest by identifying a surrogate, with whom he must effectively communicate and discuss all issues and concerns. Quite frequently, a physician is faced with a situation wherein multiple first-degree relatives cannot agree on the approach to medical care, despite appropriate and adequate counseling. In such cases of conflict, the hospitals ethics committee should be involved to act as a mediator between the different family members. In extreme cases, the case may need to be taken to court, where a guardian is appointed to assist in the medical decision-making. All the family members should be involved in the decision-making process regarding the withdrawal of mechanical ventilation. Important: The hospital ethics committee should be involved if a conflict exists between multiple family members regarding the appropriate approach to the patient's medical care despite adequate mediation by the physician.

8. Analgesic nephropathy is seen in patients with heavy, long-term use of aspirin, phenacetin, acetaminophen or other nonsteroidal antiinflammatory drugs. Cumulative amounts of analgesics play a very significant role in the pathophysiology of analgesic nephropathy. Chronic necrosis of the renal papilla with calcification is typically seen. Acute papillary necrosis can also occur and lead to acute renal colic. Urine examination reveals hematuria, proteinuria, and pyuria. The urine culture is usually sterile. The associated anemia is out of proportion to the renal failure, & this is possibly due to GI blood loss. Discontinuation of the analgesic causes stabilization or even improvement in renal function Patients with pyelonephritis have renal colic, but the presentation is not sudden. Fever & dysuria are also commonly observed with this disease A unilateral ureteric calculus will not lead to an increase in the BUN & Creatinine levels Prerenal azotemia is associated with a decrease in renal perfusion and an increase in the BUN level > creatinine; the urine specific gravity is high. Allergic interstitial nephritis characteristically produces azotemia, skin rashes, and eosinophilia. Urine eosinophils can be detected by using Hansels stain 9. Annual screening for prostate cancer should be performed between the ages of 50 and 70 using digital rectal examination and measurement of serum PSA levels; however, those with a higher risk for prostate cancer (i.e., African-Americans, those with a significant family history) should begin annual screening at 45 years of age

10. This patient presents with polyarticular joint pain of acute onset (< 6 weeks). The two important differential diagnoses that should be considered in patients with such a scenario are viral arthritis and early systemic rheumatic disease. Since the duration of the patient's symptoms is relatively short, and there exists a temporal relationship with an upper respiratory infection, viral arthritis should be the primary consideration. Rheumatoid arthritis (RA)-like symptoms with small joint involvement and a weakly positive RF test are especially characteristic for parvovirus infection. In this case, appropriate diagnostic tests should be ordered and the patient should be carefully followed-up RA is unlikely if there is no evidence of synovitis and the symptoms are of < 6 weeks duration (The hallmarks of synovitis include soft tissue swelling, warmth over a joint and joint effusion) SLE may be a possibility, but a more widespread symptomatology is usually more typical of this diagnosis. Furthermore, this patient does not fulfill the diagnostic criteria for SLE. Rheumatic fever occurs after streptococcal pharyngitis and may initially manifest with inflammatory joint disease, but involvement of the small joints is not common and joint involvement is migratory in nature. Sarcoidosis is a systemic disease that may have joint manifestations, but accompanying symptoms (e.g., pulmonary symptoms, erythema nodosum, neurological symptoms) are usually present. Viral arthritis is typically self-limited and of short duration. Therapy is generally directed at the relief of symptoms and maintenance of function; therefore, patients are treated with simple analgesic and anti-inflammatory drugs. Physical and occupational therapy may be employed if required to maintain or improve function Joint deformity is a very uncommon complication in these patients. Rheumatoid arthritis can cause significant joint deformities without appropriate long-term treatment. Joint deformities are less characteristic for other systemic diseases such as SLE and rheumatic fever, but involvement of other organs may be significant in these diseases 11. Assessment of immune status is important in patients with HIV infection in terms of susceptibility to various opportunistic agents and available options for treatment and prophylaxis. Currently, two indicators of disease progression in HIV patients are emphasized as important tools for the assessment of disease progression: viral load and CD4 count. CD4 lymphocyte count is an indicator of the current level of immunosuppression, and is referred to by some authors as, the immunologic damage that has already occurred.' Viral load is a good marker of disease activity, or the potential for future damage to the immune system. It is referred to as the damage that is about to occur.

Therefore, plasma viral load has prognostic significance at any level of CD4 count in patients with HIV infection. Other than absolute CD4 lymphocyte count, CD4 percentage and CD4/CD8 ratio are sometimes used to assess immune status in patients with HIV infection, but these are less useful indicators. p24 and p41 antibodies are used as serological markers to diagnose HIV infection, but are not employed to assess the severity of the disease. 12. The patient in the vignette has a clinical presentation consistent with a partial small bowel obstruction. The presence of air in the distal colon makes the diagnosis of complete obstruction less likely. Partial small bowel obstruction should be initially managed with observation and supportive treatment (e.g., intravenous hydration, nasogastric suctioning and correction of electrolyte abnormalities). If the patient fails to improve in the next 12 to 24 hours, early surgical intervention is recommended. Patients with partial small bowel obstruction should be managed by conservative therapy initially before proceeding to invasive interventions; however, patients with signs of impending strangulation (incarcerated hernias) or mesenteric ischemia should undergo urgent surgical intervention to prevent further deterioration of the clinical status. Colonoscopy has no role in the management of patients with small bowel obstruction. A rectal tube is not indicated in patients with small bowel obstruction. 13. Lesbian women often conceal their sexual orientation from their primary care physicians. Recent developments and legislations regarding same sex marriages have encouraged more homosexual couples to "come forward" or reveal their sexual orientation. It is important for primary care physicians to be sensitive and knowledgeable to their special needs and concerns. Human papilloma virus (HPV) infection has been strongly linked with the development of cervical intraepithelial neoplasia (CIN) and cervical cancer. Sexual intercourse, especially with multiple new partners, is the main risk factor for the acquisition of HPV infection. The risk of acquiring HPV infection (and, hence, CIN/cervical cancer) is much lower in lesbian women if they do not engage in sexual intercourse with men. Transmission of HIV infection, though rare, can occur via exposure to cervical and vaginal secretions of an HIV-infected patient. Lesbian women have a lower risk of contracting syphilis and Chlamydia infections than heterosexual (women who have sex with men) or bisexual women (women who have sex with both men and women). Lesbian women should be given Hepatitis B vaccination. Although they are at much lower risk of acquiring hepatitis B infection than gay men, they can still acquire the infection via vaginal and cervical secretions, especially if they have multiple or new partners. 14. Central venous catheters are very commonly used in hospitalized patients and in outpatient settings. Catheter-related infections are a major complication of their prolonged use. Common clinical features include sudden onset of fever with chills, hypotension, altered mental status, and nonspecific GI symptoms. Local infection at the insertion site, exit site, or subcutaneous tunnel manifests as erythema, induration, and tenderness at the site, with pus coming out of the exit site. The general approach in treatment depends on a variety of factors - local vs. systemic infection, type of device used (tunneled vs. non-tunneled), the infecting organism, and the status of the host (immunocompromised or neutropenic). In the above vignette, suspicion of a systemic infection (fever, chills, with altered mental status) in an immunocompromised patient with a tunneled catheter (Hickman, Broviac, Groshong, or Quinton) as a suspected source warrants the removal of the catheter. The patient should also be started on empiric therapy with vancomycin (for gram positive organisms) and gentamicin (for gram negative bacilli). This should be continued until final microbiologic identification of the causative organism is obtained. The outcomes with central catheter-related infections are significantly affected by the speed of initiation of anti-microbial therapy. Waiting for laboratory data and culture results can lead to an inappropriate delay in treatment and can adversely affect clinical outcomes. The catheter tip should be sent for cultures after removal of the catheter; however, initiating empiric antibiotics remains as the priority. Rifampin with a fluoroquinolone has been used in some preliminary studies for Staphylococcus aureus infections, and there have been good results; however, these drugs should not be used as an empiric therapy for suspected infections.

Metastatic infections and complications are common in patients with Staphylococcus aureus infections. Examples include septic thrombophlebitis, infective endocarditis, osteomyelitis, and rarely, retinitis. The diagnosis is suspected by persistent bacteremia or unchanged clinical status despite adequate treatment. This patient appears to have developed vertebral osteomyelitis or diskitis (infection of the intervertebral disk space), which is a well-recognized complication of catheter-related systemic infections. It usually presents with an insidious onset of low back pain, local tenderness to spinal percussion, reduced back mobility, and spasm of nearby muscles. MRI is the investigative procedure of choice for vertebral osteomyelitis (highly sensitive in detecting vertebral osteomyelitis and/or diskitis). Typical MRI findings in vertebral osteomyelitis include decreased signal intensity in the disk and adjacent vertebral bodies, loss of endplate definition, contrast enhancement of the disk, adjacent vertebral bodies and involved paraspinal and paravertebral soft tissues. Finally, a CT-guided needle biopsy is generally necessary to confirm the clinical and/or radiographic suspicion of the presence of vertebral osteomyelitis or diskitis. Typical radiographic changes of vertebral osteomyelitis consist of destruction of the vertebral bodies with collapse of the intervening disk space. Plain radiographs are often normal in the early phases of infection. The appropriate therapy for vertebral osteomyelitis is to continue vancomycin alone for at least six weeks. Prolonged therapy (12 weeks) is usually required for patients with extensive bone destruction, adjacent soft tissue or paravertebral infection. Switching to linezolid or adding gentamicin is not recommended as long as the bacteria are susceptible to the drug. 15. An upright chest x-ray is the initial test of choice to confirm the diagnosis of pneumothorax. The accumulation of air occurs primarily in the apical and lateral regions when the patient is upright, and is usually seen as a convex white visceral pleural line on x-ray. As little as 50 ml of pleural gas can be visible on upright x-ray. It has been stated that an expiratory x-ray may significantly improve the rate of detection of pneumothorax; however, it has been shown that this statement is incorrect. As an example, one study of 85 patients with pneumothoraces and 93 controls found that inspiratory and expiratory upright chest radiographs have equal sensitivity for pneumothorax detection. Furthermore, considering the limitations of expiratory radiographs, only inspiratory films are recommended as the initial examination of choice for pneumothorax detection. A lateral decubitus chest x-ray may be used to look for a small pneumothorax, but it is not routinely employed as the initial test of choice. CT scan is not recommended for routine use, but may be helpful in selected cases (e.g., to distinguish between a large bulla and a pneumothorax) 16. Ampicillin-associated maculopapular rash is a well-known phenomenon in patients with infectious mononucleosis. The reported incidence of this reaction is as high as 80%. It is believed that this vasculitic rash is immune-mediated, and is caused by circulating IgG and IgM antibodies toward penicillin derivatives. Such antibodies have actually been demonstrated in patients with EBVassociated infectious mononucleosis. The rash does not represent immediate or delayed hypersensitivity to ampicillin, which can be used safely when the infection subsides. Supportive treatment and observation are the mainstays of treatment for individuals with infectious mononucleosis. Supportive treatment includes acetaminophen and NSAIDs for fever, throat pain and malaise, as well as adequate nutrition, fluids, and rest. The antibiotic should be discontinued. Corticosteroids are reserved for patients with severe complications such as impeding airway obstruction, liver failure, or aplastic anemia. Although acyclovir is effective in inhibiting EBV replication, it has not been shown to have significant clinical benefits in patients with infectious mononucleosis. 17. Most often occurring in middle-aged women, Sjogrens syndrome (SS) is a chronic and progressive autoimmune disorder characterized by lymphocytic infiltration of the exocrine glands. Primary symptoms include dry eyes and dry mouth secondary to lacrimal and salivary gland involvement. The syndrome is commonly associated with other autoimmune conditions, such as SLE or scleroderma. To establish that the dryness experienced by the patient is secondary to SS, laboratory evaluation for characteristic autoantibodies (i.e., anti-Ro/SSA or anti-La/SSB) should be undertaken. Quantifying breakup time after fluorescein staining of the cornea is a means of measuring tear film instability. Since this test requires slit lamp examination, it is usually performed by an ophthalmologist. Although

fluorescein staining is a useful means of objectifying patient complaints about dry eyes, it is not a definitive method of diagnosing Sjogrens syndrome. The instillation of Rose Bengal into the eye will stain areas of devitalized tissue. This staining allows for the objective measurement of damage to conjunctival and corneal epithelial cells. Since the Rose Bengal test requires slit lamp examination, it is usually performed by an ophthalmologist. Although Rose Bengal staining is a useful means of objectifying patient complaints about dry eyes, it is not a definitive method of diagnosing Sjogrens syndrome. Parotid gland biopsy is not commonly performed when evaluating patients for Sjogrens syndrome. The Schirmer test is used to measure tear production. The test involves inserting a small piece of filter paper into the lower eyelid to quantify the extent of wetting within a certain time frame. Although the Schirmer test is a useful means of objectifying patient complaints about dry eyes, it is not a definitive method of diagnosing Sjogrens syndrome. Patients with suspected Sjogrens syndrome (SS) should have the diagnosis confirmed with a labial salivary gland biopsy, which is considered the "gold standard." The biopsy sample is obtained from a normal-appearing portion of the lower lip, and the classic histologic finding in SS is focal collections of lymphocytes. Anti-centromere antibodies are suggestive of CREST syndrome, not Sjogrens syndrome. Although an elevated erythrocyte sedimentation rate is found in up to 70% of patients with SS, this finding is not specific and cannot be used to confirm the diagnosis. Malignant lymphoproliferative disorders are more common in patients with Sjogrens syndrome (SS). The lymphocytic infiltration of exocrine glands seen in SS is typically accompanied by polyclonal B-cell activation, as evidenced by the presence of autoantibodies anti-Ro/SSA and anti-La/SSB. The resulting chronic, excessive B-cell stimulation contributes to the increased incidence of non-Hodgkin's lymphoma (extranodal marginal zone B-cell lymphoma) in this patient population. Studies indicate that the time between the onset of SS and the diagnosis of non-Hodgkins lymphoma varies from 4 to 12 years. Fortunately, this increased incidence of lymphoma is not associated with an elevated risk of death in the SS patient population. T-cell lymphoma is associated with infection with human Tlymphotrophic virus, type 1 (HTLV-1). There is no known association between this condition and Sjogrens syndrome. Acute pancreatitis is associated with mumps infection, which can cause parotid gland inflammation. Pancreatitis has not been associated with Sjogrens syndrome. The risk factors for development of carcinoma of the salivary glands are radiation exposure, Epstein-Barr virus infection, genetic factors (e.g., tumor gene inactivation), environmental factors (e.g., exposure to silica dust or kerosene), and dietary factors (e.g., reduced fruit and vegetable intake). 18. Most antibiotics are not neurotoxic, but some are associated with an increased risk of seizures. Conditions that would predispose an individual to antibiotic-induced seizure include renal insufficiency, older age, pre-existing CNS disease, and concomitant use of proconvulsant drugs. Of all antibiotics, beta-lactams are the most commonly associated with adverse CNS events. Specifically, penicillins, cephalosporins, monobactams, carbapenems (including imipenem), and fluoroquinolones are the antibiotics most likely to trigger seizures 19. The patient's history, physical examination findings, thyroid function test results, and thyroid scan results (focal uptake) are very characteristic of a toxic nodule. Graves' disease is the most common cause of hyperthyroidism; however, focal uptake in the thyroid scan rules out this disease. Subacute and painless thyroiditis are both uncommon in elderly patients. These diseases demonstrate a diffuse reduction in radioiodine uptake.

This patient most likely has Marfan syndrome (MFS), a disease characterized by arachnodactyly, increased arm span relative to height, and valvular (mitral or aortic) insufficiency. Dural ectasia is the most common finding (present in more than 90% of patients), and usually requires an MRI of the lumbar spine for the confirmation of the diagnosis. Ectopia lentis can be seen in 50-80% of the patients with MFS, and is characterized by an upward displacement of the lens. Aortic dilatation can

be seen in 50% of the children with MFS and 70-80% of the adults. Some of these patients will eventually develop aortic insufficiency. 20. Diffuse esophageal spasm manifests with chest pain and dysphagia. The etiology is unclear, although in many patients it is associated with emotional factors and functional gastrointestinal disorders. Manometric studies demonstrate high amplitude peristaltic contractions. In contrast to achalasia, the lower esophageal sphincter usually has a normal relaxation response. Manometric findings may be intermittent, thus making the diagnosis difficult. The esophagogram is frequently normal, although the classic corkscrew esophagus is seen occasionally. Treatment is with antispasmodics, dietary modulation, and psychiatric counseling. Surgery is very rarely required for this disorder. Zenkers diverticulum is a disorder of the proximal esophagus generally seen in females. The diverticulum may vary in size and is generally asymptomatic in presentation. The occasional patient may present with complaints of food sticking in the throat, halitosis, and regurgitation. There is no pain associated with the diverticulum. Treatment is surgery. In achalasia, the lower esophageal sphincter does not relax (high tone). Histopathology reveals hypertrophied, inner circular muscle with the absence or degeneration of ganglia in Auerbachs plexus. Manometry will show the absence of peristalsis. The cause is not known, but a similar condition in South America is caused by the parasite, Trypanosoma cruzi. An esophagogram typically reveals a dilated esophagus with a birds beak narrowing of the distal esophagus. Therapy is balloon dilation of the narrowed esophagus or surgery. Scleroderma is a collagen vascular disorder which can present with loss of distal peristalsis of the esophagus. There is complete atrophy of the esophageal smooth muscle and fibrosis. The lower esophageal sphincter becomes incompetent (low tone) with time, leading to reflux esophagitis and a stricture. The condition is progressive and difficult to treat. Esophagitis can be due to several causes, the most common being Herpes, Candida, or a Cytomegalovirus. Esophagitis generally occurs in immunocompromised individuals (e.g., those with AIDS, malignancy, diabetes) and may present with dysphagia, oral thrush, or odynophagia. Endoscopy with washings, culture, and biopsy may reveal the cause. **Extremely high yield question for USMLE. Understand the pathophysiology, the presence of and the absence of peristalsis, and LES tone in all of the above conditions. 21. Patients with borderline personality disorder suffer from considerable instability in self-image, moods, impulse control, and relationships. Relatively minor events or disagreements are often interpreted as threatening a relationship, causing many borderline patients to respond with dramatic displays of anger or self-harm. Marked changes in mood can occur throughout the day. When primitive idealization occurs, the patient views another individual as perfect and without flaw, and is unable to tolerate any evidence to the contrary. This often happens when the borderline patient interacts with a "savior," someone (such as the physician, in this case) who has cared for her in a time of crisis. Compensation results when an individual overemphasizes achievements in one sphere because of failure in another. One example would be an unattractive individual focusing on scholastic success. Projection occurs when an individual attributes his thoughts or desires especially those that are socially unacceptable to another person. One example would be a woman who resents a coworkers success and says, "He doesnt like me." Reaction formation is the overcompensation for uncomfortable impulses. One example is a man who is attracted to other men but behaves in a homophobic manner. The hallmark of borderline personality disorder is splitting, a phenomenon in which all external objects are classified as wholly good or wholly bad (a categorization that may abruptly change based on one positive or negative encounter). Patients with borderline personality disorder tend to use a psychological defense mechanism known as splitting, in which all external objects are classified as wholly good or wholly bad. Primitive idealization is one aspect of splitting in which another individual is viewed as perfect. *Extremely important question for USMLE step-3 Dialectical behavior therapy is one of the most successful means of treating borderline personality disorder. It is an intensive process and may take more than a year of therapy before improvement is seen. Unlike traditional psychotherapy (which analyzes unconscious motives), dialectical behavior

therapy focuses on behavior modification and the building of skills. Important issues to address during therapy sessions include the establishment of appropriate boundaries, validation of the patients experience, assumption of responsibility for one's own actions, management of feelings on both sides, promotion of reflecting before acting rather than being impulsive, reduction of tendency to engage in splitting, and the setting of limits on self-destructive behaviors. Selective serotonin reuptake inhibitors can help reduce mood lability and temper outbursts in borderline patients; however, medications are not first-line in the treatment of this disorder, and should only be used as a supplement to psychotherapy. Tricyclic antidepressants are contraindicated because of the high risk of suicide attempts. 22. A 32-year-old white female with no significant past medical history comes to the office because she has noticed a small lump in her left breast. She is married, has no children, and works as an attorney in a very busy local bureau. She does not use tobacco, alcohol, or drugs. There is no history of breast cancer in the family. Her last menstrual period was five days ago. On physical examination, you find a 1.5 cm round, smooth, soft, mobile, mildly tender mass in the left breast. No axillary nodes, skin lesions, or nipple discharge are found. Which of the following is the most appropriate next step in her management? A. Fine needle aspiration of the lesion B. Breast ultrasound C. Mammogram D. Reevaluation three weeks from now E. Order serum BRCA1 and 2 Explanation: Female patients younger than 35 years have a decreased risk of breast cancer, especially if there is no positive family history. The patient is a busy woman who needs a fast answer to her problem. At the same time, the characteristics of her breast mass - smooth, soft, mobile, and round - usually correspond to a breast cyst, which may easily be assessed through fine needle aspiration (FNA). (Choice C) A mammogram is not usually recommended in patients less than 35 years of age because the breast tissue at this age is too dense to allow good imaging. (Choice B) If the mass has no cystic characteristics, or if the patient refuses FNA, a breast ultrasound is indicated. (Choice D) Benign lesions are supposed to decrease in size 3 to 10 days after menstruation, not three weeks later. Since the patient came five days after her menstrual period, reevaluation of the lesion will not be useful. (Choice E) BRCA tests are not advisable. Educational Objective: Even though breast cancer is rare in women younger than 35 years, every breast lesion must be taken seriously. Patients can be reevaluated 3 to 10 days after the menstrual period to look for regression. If the lesion appears cystic, FNA should be done unless the patient declines. If the mass looks solid, is too small, or cannot be felt, ultrasound is the next step to determine if biopsy is needed. 40% of people answered this question correctly. Case 26 The following vignette applies to the next 2 items

A 56-year-old African American male presents with several months history of a swallowing difficulty. He says, It started with meat sticking in my throat and then got worse. He now has to drink a lot of fluids with his meals. He denies choking episodes, shortness of breath, voice change and heartburns, but notes that sometimes he coughs during meals. He has lost several pounds during the last two months. His past medical history is insignificant. He smokes 2 packs of cigarettes daily and consumes 6-8 bottles of beer on weekends. He is not sexually active. His blood pressure is 120/70 mmHg and heart rate is 80/min. His lungs are clear on auscultation. Neck palpation reveals no lymph node enlargement. Item 1 of 2 What is the best initial step in the management of this patient? A. Chest x-ray B. CT scan of the chest C. Barium swallow D. Upper GI Endoscopy E. Bronchoscopy Explanation: Dysphagia is an alarm symptom that warrants immediate evaluation. A thorough history and physical examination can provide important clues to the correct diagnosis and appropriate tests to be run. In this case, the progressive dysphagia to solids indicates a high probability of mechanical obstruction vs motility disorder. Considering this patient's risk factors (e.g., age, smoking, alcohol, weight loss), esophageal cancer is likely. In patients with suspected upper esophageal lesions, it is always safer to proceed first with barium swallow than with endoscopy. (Choice D) Although many physicians consider endoscopy as the initial test of choice, barium swallow can be run initially in certain patients. Intubation of the proximal esophagus during endoscopy is done relatively blindly, thereby risking perforation in patients with an upper esophageal pathology. (Choice B) CT scan of the chest can be a part of the work-up after the diagnosis is established. It is useful in assessing the extent of the disease and helps to choose an appropriate treatment strategy. (Choice E) Bronchoscopy is sometimes employed to detect tracheal and bronchial involvement in patients with upper esophageal lesions. (Choice A) A chest x-ray will convey little diagnostic information in this situation. Educational Objective: If an upper esophageal lesion is being suspected, it is always safer to proceed with barium swallow first before doing an endoscopy. 71% of people answered this question correctly. Item 2 of 2 Endoscopy reveals an upper esophageal mass with irregular contours that partially obstruct the lumen of the esophagus. What is the most likely histological type of the tumor? A. Mucoepidermoid B. Adenoid cystic C. Squamous cell D. Small cell E. Adenocarcinoma

Explanation: The two most common histological types of esophageal cancer are squamous cell carcinoma and adenocarcinoma. Several decades ago, squamous cell carcinoma was clearly predominant, accounting for up to 90% of esophageal cancer. Nowadays, the incidences of these two types are almost equal. The two differ in their pathophysiology and patient risk profile. Squamous cell carcinoma is more common in African Americans and shows a significant association with smoking, alcohol consumption, and some dietary factors. It usually affects the upper and middle parts of the esophagus. (Choice E) Adenocarcinoma is more common in Caucasians and usually arises from Barretts esophagus, a well-known metaplastic complication of GERD. It is associated with obesity, but shows little association with alcohol consumption. It typically affects the distal esophagus, gastroesophageal junction and gastric cardia region. (Choices A and B) Other histological types of esophageal cancer, including mucoepidermoid carcinoma and adenoid cystic carcinoma, are rare. Educational objective: Squamous cell carcinoma usually affects the upper and middle parts of the esophagus. It is more common in African Americans and shows a significant association with smoking, alcohol consumption, and some dietary factors. Adenocarcinoma is more common in Caucasians and usually arises from Barretts esophagus. 88% of people answered this question correctly.

Case 27 The following vignette applies to the next 2 items A 65-year-old male presents to the emergency department with an ulcer on his left foot. The lesion started as an erythematous patch on the medial aspect of his left forefoot three weeks ago. Two weeks ago, a small ulcer formed, enlarged, and eventually developed some purulent discharge. The amount of purulent discharge increased during the past week. He denies pain in his left foot. He complains of mild fever and chills. He has had type-2 diabetes mellitus for the past fourteen years. He is currently on a mixed split regimen of insulin, with suboptimal glycemic control. He has hypertension, for which he is on captopril and hydrochlorothiazide. His hypercholesterolemia is being managed with pravastatin. He has a 20 pack-year history of smoking. On physical examination, he is 60" (180 cm) tall, and weighs 152 lb. (71 kg). His blood pressure is 165/94 mm Hg, heart rate is 96/min regular, and temperature is 100 F (37.7 C). Neck examination reveals a left carotid bruit. Examination of the left foot revealed a 3x2-cm ulcer with a necrotic base on the medial aspect of the first metatarsophalangeal joint. There is some purulent foul-smelling discharge oozing from this ulcer. The surrounding skin is erythematous and warm. There is no significant tenderness in the left foot. Probing of the ulcer is suspicious for entering into the first metatarsal head. Peripheral pulsations are not felt in both feet. There is a significant sensory impairment up to the knees in both lower extremities. Ankle jerks are absent. Other systems are unremarkable. Item 1 of 2 Which of the following non-invasive modality has the highest accuracy for diagnosing osteomyelitis in his foot? A. MRI scan B. Technetium bone scan C. WBC scan D. Plain radiograph E. CT scan Explanation: The patients clinical features are highly suggestive of osteomyelitis. Probing into the bone is highly suggestive of osteomyelitis, which is generally confirmed using a non-invasive test. Of all the noninvasive tests, MRI is most accurate, with a sensitivity of 99% and specificity of 88%. The gold standard for diagnosing osteomyelitis is bone biopsy. It is important to diagnose osteomyelitis because the treatment may vary once the diagnosis is established. Majority of patients require long-term parental antibiotic treatment and resection of the affected bone. In this case, because there is a suspicion of significant ischemia, it might be prudent to consult a vascular surgeon, particularly if the response to antibiotic(s) and surgical debridement is poor. (Choices B, C, D and E) The specificity and sensitivity of other non-invasive tests is much lower. Educational objective: MRI is the most accurate test in diagnosing osteomyelitis in the vertebrae and in diabetic foot. 48% of people answered this question correctly. Item 2 of 2 Which of the following is the most useful to obtain a microbiologic diagnosis in this patient? A. Blood culture B. Culture swab from ulcer base C. Aspiration from surrounding erythematous skin D. Culturing pus coming out from ulcer E. Culture of deep tissue obtained by curettage

Explanation: Establishing a correct microbiological diagnosis will help in choosing the proper antibiotic(s) treatment. The majority of foot infections in diabetics are caused by mixed aerobic and anaerobic organisms (80%). The most commonly involved organisms are S. aureus, group B Streptococci, Proteus, Pseudomonas, E. coli, Candida, Bacteroides, Peptococcus and Clostridium. Organisms isolated from cultures of specimens from deep curettage correlate closely with cultures obtained from surgical resection of the deep tissue. Antimicrobial therapy for diabetic foot infections should be individualized. Most superficial skin infections require outpatient oral antimicrobial therapy against S. aureus, while extensive, deep infections require parental antibiotic therapy with a broad range for microbes, including anaerobes. Major surgical debridement may be necessary. (Choiced B, C, and D) A superficial swab from the ulcer, culture of the purulent discharge, and needle aspiration do not always reflect the correct etiologic organism. (Choice A) Blood cultures are positive in about 40% of patients with foot infection. Educational Objective: Organisms isolated from cultures of specimens from deep curettage correlate closely with cultures obtained from surgical resection of the deep tissue in patients with infected foot ulcers. 51% of people answered this question correctly. Case 28 A 75-year-old female is brought to the emergency room after she fell in her bathroom. She is unable to walk or bear weight on her right leg. Her past medical history is significant for hypertension, coronary artery disease, myocardial infarction, congestive heart failure and osteoporosis. On examination, you note that her right leg is externally rotated, and all the movements involving the right hip joint are very painful. The x-ray of the right hip shows a fracture in the neck of the femur. She is scheduled for right hip replacement. Which of the following is the most appropriate course of action to prevent deep venous thrombosis in this patient? A. Aspirin alone B. Warfarin alone C. Pressure stockings D. Low molecular weight heparin E. Inferior vena cava filter Explanation: Approximately 250,000 hip fractures occur every year in the United States. Development of venous thromboembolism in these patients is one of the major causes of postoperative morbidity and mortality. 4-7% of DVT patients suffer from fatal pulmonary embolism. Because of this, thromboembolic prophylaxis has been recommended for these patients. Studies comparing the prophylactic effects of aspirin, coumadin and unfractionated or low molecular weight heparin (LMWH) have shown that the LMWH group has superior results when compared with the other two groups. LMWH is therefore considered to be the therapy of choice, and should be given unless contraindications for its use are present. (Choice B) Coumadin is considered a second line prophylactic agent. It can be used as first line if any contraindication to the use of LMWH exists. Administration of coumadin should be started on the day of admission, and the target INR of 2 to 3 is achieved for adequate prophylaxis. (Choice C) Pressure stockings and intermittent pneumatic compression are used as adjuvants to other therapies. They are not sufficient to prevent DVT as sole agents because they are only effective in preventing DVT formation in calf veins, and have no prophylactic effect on pelvic vein thrombosis.

(Choice A) Aspirin is used as a third line agent after heparin and coumadin. It is recommended as a sole prophylactic agent against thromboembolism only if both heparin and coumadin (warfarin) are contraindicated. (Choice E) Inferior vena cava filter placement is indicated in patients who have contraindications to anti-coagulation. It is also used in patients who develop DVT despite ongoing anticoagulation treatment. Educational Objective: LMWH is considered to be the prophylactic therapy of choice for preventing deep vein thrombosis in patients at high risk. Case 29 The following vignette applies to the next 3 items It is estimated that the prevalence of smoking in a population is 50%. A cohort study conducted using a random sample from this population showed that the five-year risk of ischemic stroke is 1:1,000 in smokers and 0.5:1,000 in non-smokers. Item 1 of 3 What is the relative risk (RR) of ischemic stroke for smokers compared to non-smokers? A. RR = 1.0 B. RR = 1.5 C. RR = 2.0 D. RR = 2.5 E. RR = 3.0 Explanation: Relative risk represents a measure of outcome in follow-up studies. It is the risk ratio which compares the risk among the exposed to the risk among the unexposed. In this case, the five-year risk of the exposed (smokers) is 0.1% (1:1000), and the risk of the unexposed is 0.05 (0.5:1000); therefore, the relative risk is 0.1/0.05 = 2.0. The following interpretation is valid in this case: A five-year risk of stroke in smokers is twice that of non-smokers. Educational Objective: Relative risk is a measure of outcome in follow-up studies. It is the risk of the exposed, divided by the risk of the unexposed. 78% of people answered this question correctly. Item 2 of 3 What percentage of the strokes observed in smokers is attributed to their smoking status? A. 10% B. 25% C. 33% D. 50% E. 75% Explanation: In this scenario, you should calculate the attributable risk percent (ARP), which is also called etiologic fraction. ARP is a measure of excess risk. It estimates the proportion of the disease in exposed subjects that is attributed to exposure status. Two approaches can be used to calculate ARP. The first approach uses the following formula: ARP = (Risk in exposed Risk in unexposed)/Risk in exposed = (0.1 0.05)/0.1 = 0.5 (50%). The other approach uses relative risk (RR) to calculate ARP:

ARP = (RR 1)/RR = (2 1)/2 = 0.5 (50%). The following interpretation is valid in this case: 50% of the ischemic strokes observed in smokers can be attributed to their smoking status, and could therefore have been eliminated if they had not smoked. Educational Objective: ARP is a measure of excess risk. It estimates the proportion of the disease in exposed subjects that is attributed to exposure status. Item 3 of 3 What percentage of the strokes observed in the population is attributed to smoking? A. 10% B. 25% C. 33% D. 50% E. 75% Explanation: In this scenario, you should calculate the population attributable risk percent (PARP). PARP estimates the proportion of the disease in the population that is attributed to the exposure. Unlike attributable risk percent, PARP is the measure of excess risk in the total population, not only in exposed subjects. PARP can be calculated using the following approach: PARP = (Risk in the total population Risk in unexposed)/Risk in the total population Knowing the risk of stroke in exposed (0.1%), the risk in unexposed (0.05%), and the prevalence of exposure in the population (0.5, or 50%), it is possible to calculate the risk in the total population: Risk in the total population = 0.1 x 0.5 + 0.05 x 0.5 = 0.075 The risk in the total population is 0.075%. Now we can calculate PARP: PARP = (0.075 0.05)/0.075 = 0.33 (33%) The following interpretation is valid in this case: 33% of the ischemic strokes observed in the population can be attributed to smoking. Educational Objective: PARP estimates the proportion of the disease in the population that is attributed to the exposure. Unlike attributable risk percent, PARP is the measure of excess risk in the total population, not only in the exposed subjects. Case 30 The following vignette applies to the next 3 items A 27-year-old woman with no medical history presents to your office complaining of recent onset chest pain. Her description of the chest pain is vague and nonspecific even when pressed for details, although she is quite dramatic in her presentation and at one point becomes teary-eyed. She is a very attractive woman dressed in revealing clothing and mentions that she is one of your neighbors. Physical examination, including vital signs, is unremarkable. Laboratory evaluation and an electrocardiogram are normal and suggest no etiology for her chest pain. You reassure her that the more serious causes of chest pain are highly unlikely, but she insists upon scheduling a follow-up appointment for next week. On the day of the second appointment, she arrives after your office is closed and intercepts you in the parking lot. She asks that you re-examine her now. Item 1 of 3 What is the most appropriate next step?

A. Refuse care and refer her to another physician B. Briefly examine her in the parking lot C. Perform full examination in the office D. Inform her that an examination is not possible now and suggest she return to the office tomorrow during business hours E. Send her to the emergency room by ambulance Explanation: Frequently encountered in the primary care setting, difficult patients tend to elicit strong reactions from their physicians. The task of properly diagnosing and managing the medical and psychiatric disorders of these patients is therefore quite challenging and invariably requires patience and insight on the behalf of the physician. When a patient behaves seductively or makes sexual overtures, the best response is to calmly establish definitive boundaries that allow for appropriate medical treatment while also maintaining a professional doctor-patient relationship. If a patient in stable condition insists upon being examined outside of the office, it is best to be pleasant but firm in reiterating that the patient must schedule an appointment to be seen during normal business hours (Choice D). Transferring this patient now (Choice A) certainly avoids the difficulties inherent in dealing with her, but is likely premature at this stage. Generally, termination of the doctor-patient relationship is done as a last resort in extreme cases (eg, with a verbally abusive or physically threatening patient) after all attempts to establish a successful relationship have failed and fair warning has been given. It is important to document the termination in writing and to ensure that the patient is not abandoned before she establishes care with a new physician. Performing a brief examination in the parking lot (Choice B) is highly inadvisable. It is important to maintain professional environs and boundaries with all patients, especially those who are behaving in an inappropriate or seductive manner. Performing a full examination in the office now (Choice C) despite the late hour is not ideal because it suggests a willingness to accommodate even unreasonable requests made by the patient. Since she is stable and failed to arrive at the appointed time, she should be advised it is necessary to return the next day. Sending her to the emergency department by ambulance (Choice E) is inappropriate since she is stable and her condition is not urgent. Educational Objective: When a patient behaves seductively or makes sexual overtures, the best response is to calmly establish definitive boundaries that allow for appropriate medical treatment while also maintaining a professional doctor-patient relationship. 91% of people answered this question correctly. Item 2 of 3 Appropriate action is taken. The patient responds by insisting that you comment on her appearance, as she is dressed in a sexually provocative manner. When you refuse, she becomes extremely upset. Which of the following conditions is she most likely suffering from? A. Antisocial personality disorder B. Borderline personality disorder C. Dependent personality disorder D. Histrionic personality disorder E. Narcissistic personality disorder Explanation:

According to the DSM-IV, patients with histrionic personality disorder (Choice D) display a persistent pattern of gratuitous emotionality and attention seeking as evidenced by five or more of the following criteria: 1) discomfort when not the center of attention; 2) behaves in a sexually seductive or provocative manner; 3) rapid shifting and shallowness of emotion; 4) usage of physical appearance to draw attention; 5) speech is vague and impressionistic; 6) dramatic, theatrical, and exaggerated behavior and emotion; 7) easily influenced by others; 8) exaggerates intimacy of relationships. Patients with antisocial personality disorder (Choice A) frequently violate the rights of others and the rules of society. Physical aggressiveness, lack of remorse, and consistent irresponsibility are also hallmark features. Patients with borderline personality disorder (Choice B) have unstable and intense interpersonal relationships, moods, and self-image. Impulse control is poor and suicidal behaviors are common. "Splitting" may be observed. Patients with dependent personality disorder (Choice C) crave being cared for by others and are often excessively clingy or submissive. Patients with narcissistic personality disorder (Choice E) are grandiose and desire adulation and admiration from other people. They may envy and exploit others, and are sometimes known to exaggerate their own accomplishments. Educational Objective: Histrionic personality disorder is characterized by gratuitous emotionality and attention seeking. Behaving in a dramatic, sexually provocative manner is common in patients with this personality disorder. 75% of people answered this question correctly. Item 3 of 3 As your neighbor, this patient is aware that you are single. She invites you to have dinner with her at a nearby restaurant later this week. You find her attractive. What is the most appropriate response? A. Accept her offer and document all personal interactions in her chart B. Accept her offer but do not document any personal interactions in her chart C. Refuse her offer and report the incident to her boyfriend, who is also your patient D. Refuse her offer and admit her to the acute-care psychiatric ward for stabilization E. Refuse her offer and inform her that you are prohibited from dating patients Explanation: It is the American Medical Associations formal position that any sexual interaction occurring between a doctor and patient constitutes professional misconduct. Because of the inherent power structure of the doctor-patient relationship and the commonality of transference and counter-transference, sexual contact would allow for the potential exploitation of patients or the clouding of a physicians judgment. It is expected that at the least, the physician will terminate the doctor-patient relationship prior to entering a sexual or romantic relationship with a patient. Many ethicists, however, argue that it is never appropriate to date a patient even years after the doctor-patient relationship ended (Choice E). Whether or not any personal interactions are documented, it is always considered inappropriate to date a current patient (Choices A and B). Reporting the incident to the patients boyfriend (Choice C) would be a violation of the confidential nature of the doctor-patient relationship and is therefore inappropriate.

Admission to the psychiatric ward for stabilization (Choice D) is appropriate when the patient is at imminent risk of harming herself or others. Although she appears to be dysfunctional and to have some serious psychiatric issues, there is no evidence of imminent danger. Educational Objective: At the least, a physician should terminate the doctor-patient relationship prior to entering a sexual or romantic relationship with a patient. Many ethicists, however, argue that it is always inappropriate to date a patient even years after the doctor-patient relationship ended. 95% of people answered this question correctly. Case 31 An 84-year-old Caucasian woman is brought to the emergency department because of severe pain in her left hip and leg. She fell from the bed while sleeping. She has been bedridden for the past two years due to morbid obesity and advanced Alzheimers dementia. Her other medical problems include hypertension, hypercholesterolemia, diabetes mellitus type 2, and coronary artery disease. She lives at home with her granddaughter. Her medications include aspirin, hydrochlorothiazide, enalapril, donepezil, glimepiride and atorvastatin. Physical examination shows an obese, alert, mildly dehydrated elderly woman. Her temperature is 36.1 C (97 F), blood pressure is 170/85 mmHg, pulse is 100 /min and respirations are 18/min. The heart sounds are normal, and the lungs are clear. The abdomen is soft, non-tender and non-distended. Bowel sounds are normal. There is no rebound tenderness or rigidity. An ecchymotic area is present on the lateral aspect of the left thigh. Her left leg is rotated outwards, and looks shorter than the right one. Passive movement of the left lower extremity is extremely painful. An x-ray of the hip reveals a trochanteric fracture of the left femur. Her granddaughter is concerned about the fracture, and says that she is willing to sign consent forms so that her grandmother can undergo hip surgery to alleviate her pain. Which of the following is the most appropriate statement about the patients condition? A. If surgery is not done, the pain will persist B. Surgery will not offer benefits in her case and can be dangerous C. If surgery is not performed, she may become more disabled and need nursing home placement D. Because of her advanced age & dementia, surgery will be an unnecessary risk E. Instead of surgery, external traction will be preferred Explanation: The patient is not a good candidate for surgery. The type of surgery, the patients age, multiple comorbidities, and severe disability places her at an extremely high risk for perioperative complications. The ideal candidates who may benefit from a surgical intervention are elderly patients who were ambulatory prior to their hip fracture. Since the patient is already bedridden, performing the surgical procedure will make no difference. The possibility of a successful rehabilitation and recovery is difficult to predict or evaluate before surgery because this heavily relies on the surgical outcome and the patients response to the procedure. (Choice C) Although the decision to send the patient to a nursing home may be affected by her disability or need for surgical intervention, one of the main considerations is her current available social support. Since this patients granddaughter seems to provide ample social support, there is no need to send her to a nursing home. (Choice D) Advanced age and dementia should not preclude surgical intervention if the procedure offers the opportunity to improve or at least partially restore function and the quality of life of the patient. (Choices A and E) Several studies have evaluated and discussed the role of conservative therapy in patients who are unable to undergo hip surgery. These have shown that hip pain due to a fracture is not an indication for surgery because conservative measures can effectively control pain symptoms.

On the other hand, systematic reviews have concluded that other forms of conservative therapy such as external traction may cause more pain. Educational Objective: Even in cases where the risk is high, only the absence of benefits may prompt the physician to refuse a procedure; therefore, a physician must always look at the entire clinical picture and prognosis before suggesting or refusing a surgical intervention. The patients age, presence of comorbidities, functional status, quality of life, and personal values must all be considered to determine if the patient will benefit from hip surgery. If the patient is already bedridden, the benefit of surgery is minimal. If the patient has other serious medical problems, the risks may outweigh the benefits. Pain alone is not an indication for surgery, as conservative therapy can also effectively control this symptom. Case 32 A 7-year-old Caucasian boy is brought to the emergency room by his mother because of abdominal pain of acute onset. He had a mild, upper respiratory tract infection several days ago, but, otherwise, his past medical history is insignificant. His temperature is 36.7 C (98 F), blood pressure is 110/65 mmHg, pulse is 105/min., and respirations are 20/min. Physical examination reveals a symmetric, erythematous, macular rash on his lower extremities that progresses to papules after several hours. The abdomen is mildly tender on physical examination. Which of the following is the most likely diagnosis in this patient? A. Hemolytic uremic syndrome B. Thrombotic thrombocytopenic purpura C. Henoch-Schonlein purpura D. Acute intermittent porphyria E. Chickenpox Explanation: The clinical scenario described is highly suggestive of Henoch-Scholein purpura. An antecedent upper respiratory infection is present in 50% of patients. Abdominal pain is a presenting symptom in 1015% of patients. The skin lesions are symmetric, involve dependent parts of the body, and classically progress from an erythematous, macular rash to papular purpura. The joints and kidneys are also commonly involved. (Choice A) Hemolytic uremic syndrome typically develops in patients younger than two years old, and presents as abdominal pain and diarrhea (usually bloody) that progresses to acute renal failure. (Choice B) Thrombotic thrombocytopenic purpura (TTP) is a serious disorder characterized by the following classical pentad: 1. Severe thrombocytopenia 2. Microangiopathic hemolytic anemia (RBC fragments) 3. Fluctuating neurological signs 4. Renal failure 5. Fever Patients with TTP generally present with fever, pallor, petechiae, and confusion. The peripheral smear shows RBC fragments. PT and PTT are usually normal. The LDH is elevated due to hemolysis. Hemolytic uremic syndrome (HUS) and TTP comes under a spectrum of diseases. If a patient has more neurologic symptoms and less renal failure, the disease is considered TTP. On the other hand, if a patient has significant renal failure and less neurologic symptoms, the disease is considered HUS. Both TTP and HUS are very serious conditions, and require emergent plasmapheresis. (Choice D) Acute abdominal pain is common in patients with acute intermittent porphyria; however, it is unusual before 18 years of age, and skin lesions are absent.

(Choice E) Chickenpox may present as erythematous lesions early in the course of the disease; however, the rash appears on the scalp, face, trunk, and proximal limbs, and then progresses sequentially to form vesicles. Educational Objective: Classical clinical manifestations of Henoch-Scholein purpura include abdominal pain, arthralgias, skin lesions, and renal involvement. 85% of people answered this question correctly. Case 33 The following vignette applies to the next 2 items A 19-year-old man comes to the student health center, complaining of dysuria and a watery urethral discharge. He has had these symptoms for five days. He has no other medical problems. The patient remembers having sexual intercourse with a prostitute almost two weeks ago. He started smoking when he was 14, and has been smoking one pack of cigarettes daily ever since. He uses marijuana and "crack" almost daily, and drinks alcohol on weekends. He is unemployed and does not have any medications. Examination shows no abnormalities, except for a clear watery urethral discharge. A urethral swab is done and sent to the laboratory. Item 1 of 2 Which of the following is the most appropriate pharmacotherapy? A. Doxycycline B. Ceftriaxone C. Azithromycin D. Ofloxacin E. Metronidazole Explanation: The patients history and physical examination findings are highly suggestive of non-gonococcal urethritis (NGU). The incubation period is usually 5 to 10 days post-exposure, compared to 2 to 7 days for gonococcal infection. (According to the patients information, exposure may have occurred 9 to 10 days before the initiation of symptoms.) The urethral discharge is typically mucoid or watery in NGU, while in gonococcal urethritis, the secretion is purulent and abundant. Therapy is the same as for cervicitis in women: azithromycin or doxycycline. Azithromycin (1 g) is given as a single oral dose, and is preferred in this case to assure patient adherence. (Choice A) Doxycycline is given for 7 days, and the 100 mg tablets should be taken twice daily. The patient might have difficulties adhering to this type of therapy, which is therefore less preferred. (Choice B) Ceftriaxone is indicated for gonococcal urethritis. (Choice D) Ofloxacin has not been shown to be effective against Chlamydia trachomatis, Ureaplasma urealyticum or Mycoplasma genitalium, which are responsible for the majority of cases of NGU. Ofloxacin is effective against gonococcal infections. Educational Objective: NGU usually presents 5 to 10 days post-exposure, and is characterized by dysuria and a mucoid or watery urethral discharge. Therapy consists of a single dose of azithromycin or a 7-day course of doxycycline. Rates of success with either regimen are around 90%. Item 2 of 2 The patient returns ten days later with his elder brother. He states that he continues to have dysuria and some watery urethral discharge. His brother confirms that the patient has not been re-exposed to

any sexual partner, and that he took the medication as indicated. The results of the urethral swab are negative for Chlamydia and Neisseria gonorrhoeae. Which will be the most appropriate next step in management? A. Refer the patient to the urologist. B. Start metronidazole. C. Start doxycycline. D. Start ofloxacin. E. Start azithromycin. Explanation: The patient has non-gonococcal urethritis (NGU), which, in 90% of the cases, responds to therapy with azithromycin or doxycycline. If re-exposure and non-adherence are ruled out, the Center for Disease Control (CDC) guidelines do not recommend repeating the treatment with doxycycline or azithromycin (Choices C and E). Treatment is instead aimed at covering Trichomonas and resistant NGU pathogens. Metronidazole is given as a single dose (2 grams), followed by 7 days of erythromycin (500 mg every six hours). (Choice A) There are no current indications to refer the patient to the urologist. (Choice D) Ofloxacin will be effective for enteric bacteria and gonococcus, and not for Trichomonas, Chlamydia or Ureaplasma. Educational Objective: NGU that is refractory to antibiotic therapy must be treated with metronidazole and erythromycin, according to CDC recommendations. The objective is to treat Trichomonas and resistant NGU agents. Another alternative is erythromycin monotherapy, which requires high doses of the drug (800 mg six times a day). 36% of people answered this question correctly. Case34 The following vignette applies to the next 2 items A 26-year-old Caucasian female comes to see you in the office with complaints of fatigue, headache, generalized body aches, and low-grade fever for the past four days. She has just returned from a hiking and camping trip in Long Island, New York, two days ago. She saw a local physician there on the first day of her symptoms. He sent her for serologic testing for Rocky Mountain spotted fever, the results of which were inconclusive at that time. She goes to the same place in Long Island with her friends during spring break every year, and has never had any problems in the past. She is otherwise in good health, and does not take any medications regularly. On examination, her temperature is 37.8 C (100F), heart rate is 92/min, blood pressure is 122/74 mmHg, and respiratory rate is 16/min. There is evidence of a fine petechial rash over both wrists and the left ankle. The rest of her physical examination is unremarkable. Item 1 of 2 Which of the following is the most appropriate next step in her management? A. Repeat the serologic testing for Rocky Mountain spotted fever in one week. B. Perform Weil-Felix test now. C. Obtain a skin biopsy of the petechial lesions. D. Follow her platelet counts serially. E. Start the patient on treatment for Rocky Mountain spotted fever. Explanation: Rocky Mountain spotted fever is a tick-borne rickettsial infection caused by an intracellular gramnegative organism, Rickettsia rickettsii. It is seen throughout the United States, and has a major

prevalence in the southeastern and central states. It is usually seen in spring and early summer, when outdoor activities are at their peak. Most of the patients become symptomatic five to seven days after the tick bite. Early symptoms of the infection are nonspecific and can be misleading. Such symptoms include: low-grade fever, lethargy, myalgias and headaches. The rash of Rocky Mountain spotted fever is typically seen on the third to fifth day of illness. It is a petechial rash which usually begins on the ankles and wrists, and spreads to the palms, soles, and to the central body. In severe, fulminant cases, the patient may develop changes in mental status (i.e. confusion), focal neurological signs, seizures, and multiorgan dysfunction, leading to death. There is no single, reliable, diagnostic test during the early phase of the illness. The diagnosis and the decision to treat patients should be based clinically - on the patients presentation in the right setting (endemic area in spring or early summer). Patients should be started on treatment early, without waiting for confirmatory tests, since the delay in treatment is associated with a higher mortality rate. (Choice A) Although serologic testing with indirect fluorescent-antibody testing, enzyme immunoassay, or complement fixation test can be used to confirm the diagnosis of Rocky Mountain spotted fever, it is not useful during the first five days of illness because the antibodies are typically seen 7-10 days after the onset of illness. Repeating the serology in this patient after one week would delay the initiation of treatment, and worsen her prognosis. (Choice B) Weil-Felix test was used in the past to detect cross-reacting antibodies. It is a nonspecific test, and is not recommended for the diagnosis of Rocky Mountain spotted fever. (Choice C) Biopsy of the skin lesions for Rickettsia can confirm the diagnosis of Rocky Mountain spotted fever; however, skin lesions do not typically appear before the third to fifth day of illness. The results of biopsies also take some time; therefore, this can cause a potentially fatal delay in the management of the patient. (Choice D) Thrombocytopenia is one of the many fatal complications of this disease. The patient should be started on treatment for Rocky Mountain spotted fever once the diagnosis is suspected, based on her symptoms and epidemiology. Simply monitoring the platelet counts serially is not recommended. Educational Objective: Patients with suspected Rocky Mountain spotted fever should be treated empirically (without waiting for confirmation of the diagnosis). Item 2 of 2 Which of the following is the most appropriate treatment for patients diagnosed with Rocky Mountain spotted fever? A. Doxycycline B. Erythromycin C. Cephalexin D. Chloramphenicol E. Levofloxacin Explanation: Doxycycline is the treatment of choice in both children and adult patients with proven or suspected Rocky Mountain spotted fever. It is typically continued for at least three days after defervescence. Early initiation of therapy is indicated for all patients suspected of having Rocky Mountain spotted fever, and is based on the clinical picture and epidemiologic setting. Any delays in treatment can lead to potentially fatal complications and higher mortality rates. (Choice D) Chloramphenicol is an alternative option for the treatment of patients with suspected Rocky Mountain spotted fever. It has a higher incidence of side effects, and is only reserved for pregnant females and for patients who are unable to tolerate tetracycline.

(Choices B, C, and E) Erythromycin, cephalexin and levofloxacin are not recommended for the treatment of Rocky Mountain spotted fever. Educational Objective: Doxycycline is the treatment of choice for patients with Rocky Mountain spotted fever. Case 35 The following vignette applies to the next 2 items You are called to see a 32-year-old African American man with no past medical history who sustained a mid-shaft fracture of his left tibia during a motor vehicle accident. Open reduction and internal fixation were performed immediately after the patient arrived in the emergency department. The leg was stabilized in a plaster cast. Continuous morphine was provided for analgesic effect. At present, the patient complains of significant pain in his left calf. He appears very uncomfortable and is sweating and shifting restlessly in bed. He pleads for better analgesia. Pulses are present bilaterally in his lower extremities. Compartment syndrome seems likely. Item 1 of 2 Which of the following is an early sign of compartment syndrome in a limb? A. Erythema B. Homans sign C. Loss of deep tendon reflexes D. Pain with passive muscle motion E. Tissue ulceration and necrosis Explanation: The events most commonly responsible for compartment syndrome include crush injuries, fractures, prolonged external compression, burns, and snake bites. Such events cause either a decrease in compartment size or an increase in compartment pressure. As the intracompartmental pressure rises, the capillary blood perfusion is reduced until it can no longer maintain tissue viability. Early physical signs of acute compartment syndrome include tightness, weakness, and pain with passive muscle motion (Choice D). One of the most suggestive signs of compartment syndrome is pain out of proportion to the injury. Hypesthesia and paresthesia may be documented as well. Erythema (Choice A) is typically seen with venous congestion. Acute compartment syndrome is more commonly associated with cyanosis, pallor, or mottling of the extremity. Pain or increased resistance with dorsiflexion of the foot is known as Homans sign (Choice B). It is an unreliable indication of deep venous thrombosis. The loss of deep tendon reflexes (Choice C) can be seen in association with acute compartment syndrome, but is typically a later finding. Tissue ulceration and necrosis (Choice E) would be a very late finding of compartment syndrome. Educational Objective: Early physical signs of acute compartment syndrome include tightness, weakness, and pain with passive muscle motion. One of the most suggestive signs of compartment syndrome is pain out of proportion to the injury. Item 2 of 2 Which of the following is considered the most ominous sign of compartment syndrome? A. Paresthesia B. Loss of arterial pulse C. Mottling of the extremity

D. Pallor of the extremity E. Excruciating pain that persists after cast removal Explanation: The most ominous sign of compartment syndrome is the loss of the arterial pulse, as that signifies a cessation of blood flow to the extremity (Choice B). Muscle tissue has impairment of function after 2-4 hours of ischemia and irreversible loss of function after 4-12 hours, while nerve tissue has impairment of function after 30 minutes of ischemia and irreversible loss of function after 12-24 hours. Treatment includes splitting of the cast and underlying padding, which can decrease the compartment pressure by 50-85%. Paresthesia (Choice A) is an early finding and can be associated with arterial pulses that are diminished and not absent. It is therefore not considered the most ominous finding of compartment syndrome. Mottling (Choice C) and pallor (Choice D) of the extremity are physical findings occasionally associated with acute compartment syndrome. However, they can be seen in conjunction with arterial pulses that are diminished and not absent, and are therefore not considered the most ominous findings of compartment syndrome. Excruciating pain does sometimes persist after cast removal (Choice E) and is not always cause for increased concern. However, if any symptoms do not resolve within an hour after cast removal and the pressure measurement remains elevated, fasciotomy is indicated. Normal function will be regained in approximately two-thirds of patients if fasciotomy is performed within 12 hours of the onset of compartment syndrome. Educational Objective: The most ominous sign of compartment syndrome is the loss of the arterial pulse, as that signifies a cessation of blood flow to the extremity. Treatment includes splitting of the cast and underlying padding, which can decrease the compartment pressure by 50-85%. Case 36 A 36-year-old Caucasian woman comes to the emergency department and complains of severe left calf pain. While she was running on a racetrack, she heard a loud snap, after which she felt an "excruciating pain" in her left calf area. She was a marathon runner in the past, but she stopped running two years ago, after she had her first baby. She just started retraining a week ago for an upcoming race. The physical findings are highly suggestive of a complete Achilles tendon rupture. Which of the following is the best clinical sign that will support this diagnosis? A. Increased plantar flexion of the ankle on calf muscle compression B. Decreased plantar flexion of the ankle on calf muscle compression C. Increased dorsiflexion of the ankle on calf muscle compression D. Decreased dorsiflexion of the ankle on calf muscle compression E. No foot movement on calf muscle compression. Explanation: Rupture of the Achilles tendon may occur after abrupt calf muscle contraction. This typically occurs in men over the age of 40 who do not perform a regular leg-conditioning program. The most common symptoms are severe pain in the calf and the inability to stand up on the toes. The patient may note an audible snap at the time of injury. A positive Thompson test is further evidence of an Achilles tendon rupture. This test is performed with the patient kneeling on a chair, or lying prone on an examination table with his feet hanging over the edge. When the examiner squeezes the calf muscle on the normal side, the foot responds with plantar flexion. On the affected side, there is no foot response. Treatment consists of immediate immobilization of the lower leg and surgical repair of the tendon as soon as possible.

Educational Objective: A complete Achilles tendon rupture leads to severe pain in the calf and the inability to stand up on the toes. A positive Thompson test further supports the diagnosis.

Case 37 A female infant born at term to a 28-year-old Caucasian woman has dorsal feet and hands edema, short webbed neck, and a cardiac murmur. Buccal smear reveals no Barr body. The mother is concerned about the recurrence risk of such an anomaly in subsequent pregnancies. Which of the following is the best response? A. The recurrence risk is approximately 50% B. The risk increases with maternal age C. The recurrence risk is close to 25% D. The recurrence risk is close to 10% E. The recurrence risk is close to that of the general population Explanation: The clinical scenario described (dorsal feet and hands edema, short webbed neck, and a cardiac murmur) is typical for Turner syndrome. It is characterized by monosomy of the X chromosome (45, X); this explains why there is no Barr body on the buccal smear. Less common chromosomal abnormalities that can be present in patients with Turner syndrome include X chromosome mosaicism and Xp deletion. No increased recurrence risk is present after having an infant with Turner syndrome. (Choice B) Interestingly, the risk of having an infant with monosomy for the X chromosome does not increase with advance maternal age, unlike Downs syndrome and Klinefelters syndrome (47, XXY). Educational Objective: No increased recurrence risk is present after having an infant with Turner syndrome. The risk of 45, X does not increase with increased maternal age. Case 38 A 20-year-old Caucasian woman comes to see you in the office for a regular follow-up visit. She has had three heterosexual partners in the past four years. Her last Pap smear two years ago was negative. She informs you that she is a lesbian now, and does not have any heterosexual partners. She wants to discontinue getting regular Pap smears now that she is not having sexual intercourse with men. Which of the following is the most appropriate response? A. I agree with you, there is no need for further Pap smears. B. You should still get a Pap smear every now and then. C. You should get a regular Pap smear if your other lesbian partner is HPV (human papilloma virus) positive. D. You should still get regular Pap smears every year. E. You should have a Pap smear done every 3 - 5 years. Explanation: Lesbian women often conceal their sexual orientation from their primary care physicians. With more recent developments and legislations regarding same sex marriages, more homosexual couples are now coming forward with their sexual orientation. It is important for a primary care physician to be knowledgeable and sensitive to their special needs and concerns. Human papilloma virus (HPV) infection has been strongly linked with the development of cervical intraepithelial neoplasia (CIN) and invasive cervical cancer. Sexual intercourse, especially with multiple partners, is the main risk factor for the acquisition of HPV infection. Lesbian women still need

to undergo routine screening for cervical cancer. Annual screening by Papanicolaou smear is recommended for all women approximately three years after the onset of vaginal intercourse, or at the age of 21, whichever is earlier. The risk of acquiring HPV infection (and CIN/cervical cancer) is lower in lesbian women if they do not engage in sexual intercourse with men. The risk of cervical neoplasia is highest in lesbian women who have had sex with more than one male sexual partner, have an early age at first coitus with men, have been infected with HPV, and have been treated for an abnormal cervical cytology test in the past. The risk is also increased with cigarette smoking. (Choices A, B and E) Increasing the screening interval or not screening lesbian women at all can lead to the delayed diagnosis and treatment of high-grade cervical intraepithelial neoplasia and cervical cancer. (Choice C) The patient should have an annual Pap smear even if her partner is HPV negative. Her partners HPV status does not change her risk of cervical cancer due to her previous sexual history. Educational Objective: Annual screening by Papanicolaou smear is recommended for all women (including lesbian women) approximately three years after the onset of vaginal intercourse, or at the age of 21, whichever is earlier. 64% of people answered this question correctly. Case 39 The following vignette applies to the next 3 items A previously healthy 62-year-old African-American woman is hospitalized because of shortness of breath and chest pain. She has had these symptoms for the past two days. She has no other medical problems, and takes no medications. She has smoked one pack of cigarettes daily for the last 20 years. Her temperature is 36.7 C (98 F), blood pressure is 130/80 mmHg, pulse is 98/min., and respirations are 22/min. The patient's pulse oximetry showed 94% on 4-liters of oxygen. V/Q scan of the chest reveals a high probability for pulmonary embolism. Her baseline labs reveal the following: CBC Hb 13 g/dL Hct 38% Platelet count 240,000/cmm Leukocyte count 8,000/cmm Coags Prothrombin time 14 sec INR 1.06 You start her on anticoagulation with unfractionated heparin and warfarin. Her symptoms gradually resolve over the next five days. On day six of her hospitalization, she complains of pain and pallor in her left arm. Physical examination reveals a pale and tender distal left arm with diminished pulses. The patients labs reveal: CBC Hb 12.6 g/dL Hct 37.2% Platelet count 40,000/cmm Leukocyte count 10,000/cmm Coagulation Tests Prothrombin time 19 sec INR 1.78 PTT 60 sec

Item 1 of 3 Which of the following is the most likely cause of her condition? A. Subtherapeutic anticoagulation B. Heparin-induced thrombocytopenia C. Warfarin induced skin necrosis D. Heparin-induced skin necrosis E. Warfarin-induced thrombocytopenia Explanation: This patient had an arterial thrombosis of the left arm following the administration of unfractionated heparin. She has developed heparin-induced thrombocytopenia (HIT), which is a well-known complication of heparin therapy. Two forms of HIT have been recognized, depending upon the onset, clinical course, and severity of the disease. Type I HIT is seen within two days of initiation of heparin therapy, and is usually associated with a lesser degree of fall in the platelet count (nadir platelet count of 100,000/microliter). The platelet count usually returns to normal with discontinuation of heparin, and there are no clinical consequences. Type II HIT is a more serious immune-mediated disorder characterized by the formation of antibodies against heparin-platelet factor 4 complex. The heparin-platelet factor 4 antibody complex then binds to the platelet surface, causing platelet activation and aggregation, leading to thrombocytopenia and platelet-rich clots. It typically develops 4 to 10 days after the initiation of heparin therapy. Patients have the platelet count in the range of 30,000 to 60,000/microliter. Spontaneous bleeding is unusual. Immune-mediated HIT is associated with both venous and arterial thrombosis. The major manifestations of venous thrombosis are deep venous thrombosis, pulmonary embolism, venous limb gangrene, and cerebral sinus thrombosis. Arterial thrombosis can lead to strokes, myocardial infarction, and limb and organ (kidneys, mesenteric) ischemia. (Choice C) Warfarin-induced skin necrosis has been reported in patients within the first few days of taking high doses of warfarin. High-dose warfarin induces a transient hypercoagulable state by causing a rapid reduction in protein C levels on the first day of therapy. The lesions are seen over the extremities, trunk, and breasts. It is not associated with thrombocytopenia. (Choice D) Heparin-induced skin necrosis is also a well-known complication of unfractionated heparin use. It usually involves areas rich in fat, such as the abdomen, although distal extremities can also be involved. It presents as an area of erythema, which quickly progresses to purpura, hemorrhage, and necrosis. Most of these patients do not develop thrombocytopenia. (Choice A) Even though his PT/INR is subtherapeutic, his PTT is therapeutic. This patient is well anticoagulated. means that the

Educational Objective: HIT should be suspected in patients receiving heparin anticoagulation if they present with thrombocytopenia, thrombosis with thrombocytopenia, or a > 50% fall in the platelet count, 4- 10 days after the initiation of treatment. 74% of people answered this question correctly. Item 2 of 3 Which of the following is the most appropriate next step in the management of this patient? A. Discontinue warfarin

B. Discontinue unfractionated heparin C. Discontinue warfarin and unfractionated heparin D. Discontinue unfractionated heparin and warfarin; initiate low molecular weight heparin E. Discontinue heparin and warfarin; initiate argatroban Explanation: The first and the most important intervention in a patient with suspected or documented HIT is the immediate cessation of exposure to all heparin products. The patients with HIT still remain at risk for thrombosis, even after the discontinuation of heparin. All such patients should be anticoagulated with direct thrombin inhibitors such as lepirudin or argatroban. They should be used for the prophylaxis and treatment of patients with HIT with or without thrombosis. The choice of agent depends on the coexisting medical conditions. (Lepirudin should be used with caution in patients with renal insufficiency, while the dose of argatroban should be adjusted in patients with hepatic dysfunction.) (Choice B) Warfarin therapy alone may increase the risk of venous gangrene in patients with deep vein thrombosis. Its use should be avoided in the absence of other anticoagulants until the platelet count rises above 100,000/microliter. (Choice D) Low molecular weight heparin should not be substituted for unfractionated heparin because it can also cross react with heparin-induced antibodies, and induce more antibody formation. Educational Objective: Exposure to both heparin and warfarin should be discontinued in patients with HIT. They should be anticoagulated with direct thrombin inhibitors for the prevention or treatment of thromboembolic complications. Item 3 of 3 Which of the following would have been the most useful strategy in preventing the above condition? A. Monitoring prothrombin time frequently B. Monitoring platelet counts frequently C. Substitution of unfractionated heparin with low molecular weight heparin D. Initiating warfarin after 3 5 days of heparin therapy E. Using higher doses of heparin early in the course of treatment Explanation: The best way to prevent HIT is to use low molecular weight heparin (LMWH) instead of unfractionated heparin whenever possible. LMWH and heparinoids, such as danaparoid, are associated with a much lower incidence of HIT compared with unfractionated heparin. Another option is to limit heparin use to less than five days in order to prevent an antibody response, as well as the development of HIT. (Choice A) Prothrombin time is used to monitor the effects and adjust the dose of warfarin, not heparin. It does not affect the development of HIT. (Choice B) Monitoring the platelet counts will lead to early detection and treatment of HIT. It will not prevent the formation of heparin-platelet factor 4 antibodies and their clinical consequences. (Choice D) Initiating warfarin early can minimize the duration of heparin use, and prevent antibody formation. It should be started at the same time as (or within 24 hours of) heparin administration. (Choices E and F) Development of HIT is independent of the dose used, and has been reported to occur even with heparin flushes and the use of heparin-coated catheters. Educational Objective:

The best way to prevent heparin-induced thrombocytopenia is by substituting unfractionated heparin with low molecular weight heparin. Case 40 The following vignette applies to the next 3 items A 29-year-old male comes to the emergency department (ED) because of palpitations and a severe headache. By the time you see him, he is more or less asymptomatic. He has had at least two similar episodes for the past one and a half months. He is requesting for "a presciption for some pills to prevent the recurrence of the symptoms." He currently has no medications. Physical examination reveals a thin young male who appears anxious and diaphoretic. His blood pressure is 126/84 mmHg, pulse is 86/min, temperature is 36.7C (98F) and respirations are 16/min. During his prior visit to the ED for an identical episode, his blood pressure was 149/98 mmHg. The thyroid is normal to palpation and without any obvious nodules. Item 1 of 3 Which of the following is the most appropriate next step? A. Measure blood pressure after two weeks in the office B. Start diuretics C. A 24-hour urinary metanephrine and free catecholamines D. MRI of the abdomen E. Alpha blockade Explanation: The patient has clinical features suggestive of pheochromocytoma. Biochemical confirmation of the diagnosis is required before imaging is performed to localize the tumor. Measurement of urinary metanephrine and catecholamine levels is a good screening test that is used for biochemical diagnosis. Both tests are ordered simultaneously. A 24-hour urine collection is superior to spot urinary collection; multiple 24-hour urine collections may be required to make a diagnosis of pheochromocytoma. Urinary vanillylmandelic acid (VMA) can also be used with other tests as a screening test. VMA has low sensitivity but higher specificity. Urine levels can be altered by a number of drugs and foods. For this reason, some centers use measurement of plasma free metanephrine levels as the initial screening test. (Choice A) This patient has features that are highly suggestive of pheochromocytoma. Investigations should be started without further delay. (Choice B) Diuretics are not good initial therapeutic agents for controlling the hypertension of patients with pheochromocytoma. Many of these patients have a lower intravascular volume, which can be further aggravated by giving diuretics. (Choice D) MRI/CT is performed to localize the tumor only after biochemical confirmation of pheochromocytoma has been achieved. (Choice E) Treatment with alpha-blockers is not started before the biochemical diagnosis of pheochromocytoma is confirmed because these increase serum catecholamine and metanephrine levels. If these drugs are immediately started, it would be extremely difficult to interpret some of the biochemical test results. Educational Objective: A biochemical diagnosis of pheochromocytoma is typically made by measurement of plasma free metanephrine levels or a 24-hour urine collection for measurement of catecholamine, metanephrine, and vanillylmandelic acid levels. Confirmation of the diagnosis with these biochemical tests is required

before imaging is performed for tumor localization. A number of drugs (e.g., alpha-blockers) can interfere with the biochemical test results. *Extremely important question for USMLE step-3 Item 2 of 3 The diagnosis is confirmed. What is the next best step in the management of this patient? A. CT/MRI of adrenals B. Alpha-blocking agent C. Beta-blocking agent D. Removal of tumor E. Metaiodobenzylguanidine (MIBG) scan Explanation: Alpha blockade is started after biochemical confirmation of the diagnosis of the pheochromocytoma is made. Alpha blockade, along with liberal salt and fluid intake, is required preoperatively to restore the intravascular volume. While the patient is started on a preoperative surgical regimen, investigations to localize his tumor can be performed. The most common agent used for preoperative preparation is the long-acting noncompetitive alpha-blocker phenoxybenzamine. Majority of patients require 40-80 mg of phenoxybenzamine daily. Patients should be treated for 10-14 days preoperatively before surgical resection can be performed. Adequate alpha blockade preoperatively reduces intraoperative complications. (Choice A) CT/MRI is typically done while the patient is on alpha blockade. CT and MRI have equal sensitivities for tumor localization, although MRI can be more sensitive in localizing extraadrenal pheochromocytoma. In addition, MRI can sometimes differentiate benign from malignant pheochromocytoma. (Choice C) Beta-blocking agents started before alpha blockade can lead to paradoxical increase in the blood pressure. Beta blockade is only preformed after alpha blockade is complete.

(Choice D) Removal of the tumor (after localization) is performed after adequate alpha blockade. The chances of intraoperative complications are much higher with inadequate preoperative alpha blockade. Approximately 10% of patients have bilateral adrenal pheochromocytomas, and 10% are extraadrenal in location. (Choice E) MIBG resembles norepinephrine and is taken up by pheochromocytomas. The sensitivity of MIBG scan is 70%. MIBG scan is performed in patients who have borderline biochemical values, but with the CT showing an adrenal mass. MIBG can also be performed in patients with biochemically confirmed pheochromocytoma, but the CT or MRI are unable to localize the tumor. Educational Objective: Alpha blockade is done for 10-14 days preoperatively to control hypertension and restore intravascular volume. Beta-blockers are only given to patients who are adequately alpha blocked. *Extremely important question for USMLE step-3 Item 3 of 3 The appropriate steps are taken, and the patient is taken for surgical removal of the tumor. During the procedure, he rapidly becomes hypotensive. His blood pressure falls from 110/89 mmHg to 80/50 mmHg. Which of the following is the most appropriate therapy for this patients hypotension?

A. Bolus of normal saline followed by continuous normal saline infusion B. Intravenous colloid bolus C. Intravenous phentolamine bolus D. Dopamine infusion E. Dobutamine infusion Explanation: Even with adequate preoperative alpha blockade, some patients with pheochromocytoma have intraoperative complications. One of the important intraoperative complications is hypotension. This generally occurs after the removal of the tumor, which is followed by a decrease in circulatory catecholamine levels and alpha blockade, thereby leading to a marked decrease in the vascular tone. The resulting intraoperative hypotension responds nicely to a normal saline bolus followed by infusion. (Choice B) Intravenous colloid may have theoretical benefits compared to crystalloid normal saline. Possible benefits of colloid are rapid restoration of intravascular volume and decreased chances of pulmonary edema. Studies have not shown that colloids are superior to the use of crystalloid solution in the management of acute hypotension. Finally, colloids are more expensive than normal saline. (Choice C) Intravenous phentolamine is used for acute hypertension intraoperatively. (Choices D & E) Dopamine and dobutamine are less effective in patients with pheochromocytoma and intraoperative hypotension because majority of these patients are on preoperative long-acting alpha blockade; therefore, the vascular response to vasoconstrictors is reduced. Educational Objective: Intraoperative hypotension in patients with pheochromocytoma responds to an intravenous bolus of normal saline. The use of colloid is not superior to the use of normal saline bolus for the management of intraoperative hypotension. Vasopressors are less effective than a bolus of normal saline. Intravenous bolus of phentolamine is used for acute severe hypertension (not hypotension) during surgery for pheochromocytoma. *Extremely important question for USMLE step-3 Case 41 Several tests have been developed to measure the serologic markers of breast cancer. These tests have different specificities and sensitivities for the early stage of breast cancer. If positive, which of the following tests will have the highest predictive value for the disease? A. Sensitivity B. Sensitivity C. Sensitivity D. Sensitivity E. Sensitivity Explanation: A high specificity increases the positive predictive value (PPV) of the test: PPV = True Positives / (True positives + False positives) Confirmatory tests must have a high specificity. In this case, the test with the highest specificity is the best choice, since this high specificity decreases the number of false-positive results and helps RULE OUT the disease. (Choice E) Increasing the sensitivity of the test will increase the negative predictive value. Educational Objective: 80%, specificity 65%, specificity 70%, specificity 75%, specificity 85%, specificity 90% 97% 94% 92% 90%

Confirmatory tests must have a high specificity. This high specificity helps RULE OUT the disease by decreasing the number of false-positive results, and by increasing the positive predictive value. 35% of people answered this question correctly. Case 42 A 70-year-old widowed African-American woman is admitted late one night to the hospital for an acute exacerbation of her congestive heart failure. During morning rounds, the medical team responsible for this womans care meets with her for the first time. The senior resident begins to ask the patient some questions about her medical history and addresses her by first name. What is the appropriate response on the part of the attending physician? A. Address the patient by first name as well B. Address the patient with the salutation Madam followed by her surname C. Address the patient with the salutation Mrs. followed by her surname D. Address the patient with the salutation Ms. followed by her surname E. Reprimand the resident and send him from the room Explanation: New patients should always be addressed as "Ms. Smith" or "Mr. Smith," as that properly demonstrates the respect due the patient (Choice D). Keep in mind as well that older patients or patients with a different cultural heritage (such as the woman in this question) are often very sensitive to being spoken to in a manner that they perceive as disrespectful. The doctor-patient relationship may needlessly suffer if the doctor speaks informally without direct permission to do so. There is some debate regarding whether a patient should ever be called by first name, even if they so request it. Specifically, some ethicists insist that the level of formality in address must be applied mutually; therefore, if a patient is called by her first name, then the physician too must be called by his first name. Ultimately, it is best to err on the side of formality rather than informality. Addressing the patient by the first name (Choice A) is presumptive and risks alienating the patient. The salutation of "Madam" (Choice B) is typically used for individuals of high standing (e.g., "Madam President" or "Madam Ambassador"). Using this form of address with all patients may be interpreted as sarcasm. The salutation of "Mrs." (Choice C) should be used if the patient has indicated it to be her preference. Keep in mind that some women do not want reference made to their marital status, which is why the salutation "Ms." is more often used. Openly reprimanding the resident (Choice E) is excessively harsh if the resident otherwise appeared to have good intentions in speaking with the patient. It also creates an awkward situation for the patient, who may feel obligated to defend the resident. If an attending wants to discuss the matter of proper salutation with the resident, it is best to do so outside of the patients presence. Educational Objective: New patients should be addressed as "Ms. Smith" or "Mr. Smith" and not by first name. 45% of people answered this question correctly. Case 43 The following vignette applies to the next 2 items A 55-year-old Caucasian woman is admitted to the hospital with symptoms of left leg pain and swelling of two weeks duration. She denies any chest pain or difficulty in breathing. She has a past medical history of hypothyroidism and hypertension. Her daily medications include L-thyroxine and hydrochlorothiazide. In addition, she has been on hormone replacement therapy (HRT) with estrogenprogestin combination for the last two years for intractable postmenopausal symptoms. She smokes a

pack of cigarettes a day. On physical examination, she has pitting edema and warmth over the left lower leg. Lower extremities Doppler reveal the presence of left proximal femoral deep vein thrombosis. Item 1 of 2 Which of the following is the most appropriate next step in the management of this patient? A. Discontinue hormone replacement therapy immediately B. Start her on warfarin 5 mg a day C. Start her on warfarin 10 mg a day D. Start her on weight-based unfractionated heparin E. Repeat the ultrasound in a week Explanation: The patient has developed a deep vein thrombosis (DVT) of the left femoral vein. Anticoagulation is indicated for all patients with symptomatic proximal DVT. The primary goal of therapy is to prevent clot extension and prevent acute pulmonary embolism. Patients with DVT or pulmonary embolism should be treated with unfractionated intravenous heparin or low molecular weight (LMW) heparin. Therapy with heparin should be continued for at least five days. Oral anticoagulation with warfarin can be started simultaneously, and should be overlapped for at least four to five days. Treatment with intravenous unfractionated or LMW heparin should be continued until the INR (International normalized ratio) has been therapeutic (2.0 - 3.0) for at least two days. LMW heparin is generally recommended over unfractionated heparin. (Choice A) HRT has been associated with an increased risk of venous thromboembolism (VTE), especially in the first year of use. Observational studies, the HERS trial, and the Women's health initiative have suggested a two-fold increase in the risk of VTE with HRT use. HRT should therefore be discontinued in patients with an increased risk of DVT or a documented DVT; however, HRT should be tapered gradually because an abrupt discontinuation can precipitate a flare of postmenopausal symptoms. (Choices B and C) Warfarin causes a rapid reduction in the protein C levels on the first day of therapy. This induces a transient hypercoagulable state, which can rarely lead to warfarin-induced skin necrosis. Furthermore, it takes time to have a therapeutic effect. For these reasons, warfarin should always be started after the patient has been under the cover of unfractionated or LMW heparin. (Choice E) DVT documented by an initial ultrasound should be treated with full anticoagulation. There is no need to repeat the ultrasound. (Choice F) The patient has no clinical features of an acute pulmonary embolization Educational Objective: Anticoagulation with heparin followed by warfarin is indicated in all patients with symptomatic deep vein thrombosis. Item 2 of 2 You manage the patient appropriately, and she is ready for discharge on the fifth hospital day. You counsel her regarding the side effects of continuing hormone replacement therapy and the duration of anticoagulation. She wishes to continue taking HRT as it makes her look and feel good. Which of the following is the most appropriate response? A. She can continue HRT without any additional risks, as long as she promises to quit smoking B. She can continue HRT, if she takes warfarin for at least six months C. She should stop HRT and continue warfarin for at least three months D. She can continue HRT, if she agrees to daily subcutaneous, low-molecular eight heparin therapy E. She should stop HRT and switch to tamoxifen for hot flashes

Explanation: It is important to consider the presence or absence of risk factors before deciding on the duration of anticoagulation therapy. There are guidelines based on the evidence-based recommendations of the sixth ACCP Consensus Conference on Antithrombotic therapy. According to these guidelines, patients with a first thromboembolic event in the presence of a reversible risk factor should be treated with anticoagulation for at least three months. In contrast, patients with an idiopathic first event, continuing risk factor, or a recurrent thromboembolic event should be treated for a prolonged period of time. (Choices A, B, D, and F) All patients on HRT with a documented DVT should be strongly urged to stop hormone replacement therapy. It is not recommended to increase the duration of anticoagulation or to switch to LMW heparin in order to facilitate the continuing use of HRT. Smoking cessation alone will not prevent the future episodes of thromboembolism. Estrogen-only preparations carry the same risk of causing venous thromboembolism. In contrast, these may increase the risk of endometrial hyperplasia and cancer due to the loss of the protective effects of progesterone. (Choice E) Tamoxifen has no role in the prevention of hot flashes in postmenopausal women, and should not be recommended. Educational Objective: A first episode of thromboembolism in the presence of reversible or time-limited risk factors should be treated for at least three months to prevent recurrence. Case 44 A 38-year-old Caucasian woman is brought to the emergency department by her coworkers, who noticed that she had been "breathing heavily" and had been restless for the last couple of hours, when she started to become drowsy. She complains of nausea, vomiting, dizziness and tinnitus; these have been present for the past few hours. She denies shortness of breath, cough or chest pain. Her other medical problems include rheumatoid arthritis and depression. She has smoked one pack of cigarettes daily for the past twenty years. She uses marijuana occasionally and drinks one glass of beer daily. Her medications include aspirin, fluoxetine and low-dose prednisone. She has no known drug allergies. Her temperature is 38 C (100.4 F), blood pressure is 120/80 mm Hg, pulse is 100/min, respirations are 26/min, and oxygen saturation is 99% at room air. Examination reveals an awake, but mildly drowsy young woman with dry mucus membranes. Heart sounds are normal. Lungs are clear. The abdomen is soft, non-tender, and non-distended; bowel sounds are present. There is no rebound tenderness or rigidity. No motor or sensory focal deficit is present. Meningeal signs are absent. A 12lead electrocardiogram (EKG) shows sinus tachycardia. A portable chest x-ray reveals no abnormalities: the lung fields are clear; there is no cardiomegaly or effusions. The patient is given intravenous fluids and 1-liter oxygen per nasal cannula. She is placed on continuous EKG monitoring. Laboratory tests are ordered. Item 1 of 2 Which of the following is the most likely cause of her condition? A. Anxiety B. Diastolic heart failure C. Pulmonary embolism D. Drug overdose E. Acute bacterial pneumonia Explanation: The patients symptoms, along with her history of illicit drug intake and aspirin use, highly suggest an overdose of aspirin or another drug that causes metabolic acidosis. The initial manifestations of salicylate intoxication are tinnitus, restlessness, nausea, vomiting, and mild gastrointestinal discomfort. After a few hours, a depressed level of consciousness will ensue. Fever, metabolic acidosis, and hyperventilation eventually develop. Hyperventilation without dyspnea is suggestive of

respiratory alkalosis, which may be a compensatory response to the metabolic acidosis. Aside from tachycardia and tachypnea, there are no other indications of cardiac or pulmonary compromise. (Choice B) Although diastolic dysfunction can be suspected in a patient with signs and symptoms of heart failure with a normal ejection fraction, there is no clinical evidence of heart failure in this case. (Choice C) Pulmonary embolism can present with sinus tachycardia, normal chest x-ray and mild fever; however, this diagnosis cannot fully explain the patients other presenting characteristics (i.e., oxygen saturation is 99%, drowsy state without being hypoxic, nausea, vomiting and tinnitus). (Choice A) The presence of fever, drowsiness, tachycardia and tachypnea, as well as the patients denial of dyspnea, points to an organic cause of her disease. (Choice E) The patient does not have a history of cough, dyspnea or sputum production. The chest xray is clear, and her oxygen desaturation is 99%. For these reasons, the possibility of pneumonia is very unlikely. Educational Objective: Dyspnea is the subjective sensation of labored or difficult breathing. Athletes and well-trained individuals develop dyspnea only when doing hard work or exercise, while sedentary subjects may experience it even after minor efforts. Hyperventilation associated with metabolic acidosis is hardly ever accompanied by dyspnea. Metabolic acidosis must be suspected as the cause of hyperventilation (compensatory respiratory alkalosis) without dyspnea if there is no evidence of cardiac or pulmonary disease, the patient is a potential drug abuser, and there are signs and symptoms compatible with compromise of other organs. Item 2 of 2 The patient continues to be drowsy, but she is now easily arousable. Her temperature is 37.8 C (100 F), blood pressure is 120/70 mm Hg, pulse is 104/min, respirations are 28/min, and oxygen saturation is 100% with one-liter nasal cannula. Her laboratory tests reveal the following: CBC Hb 13.8 g/dL Ht 41% MCV 91fl Platelet count 320,000/cmm Leukocyte count 6,000/cmm Segmented neutrophils 70% Bands 2% Eosinophils 2% Lymphocytes 18% Monocytes 8% Serum chemistry Serum Na 145 mEq/L Chloride 108 mEq/L BUN 30 mg/dL Calcium 9.9 mg/dL Serum K 4.3 mEq/L Bicarbonate 17 mEq/L Serum creatinine 1.6 mg/dL Blood glucose 75 mg/dL Direct bilirubin 0.8 mg/dL Aspartate aminotransferase 150 U/L

LFT Total bilirubin 1.3 mg/dL Alkaline phosphatase 200 U/L Alanine aminotransferase 110 U/L Gases, arterial blood pH 7.25 pO2 110 mm Hg

pCO2 20 mm Hg

Which of the following is the most appropriate next step in management?

A. Intravenous antibiotics B. Oral lactulose and metronidazole C. Intravenous furosemide D. Non invasive mechanical ventilation E. Gastric lavage and alkalinization of the urine Explanation: The patient has moderate metabolic acidosis, with a pH of 7.25, a bicarbonate level of 17 mEq/L and an anion gap of 20 (obtained by subtracting the values of bicarbonate and chloride from the sodium). Her anion gap is high (normal anion gap: 12, ranging from 8-16). Her slightly elevated BUN and creatinine levels indicate mild renal failure, which can be due to dehydration or acute interstitial inflammation. Other abnormal findings are the moderately elevated aminotransferases, while the white blood cell count is normal. The clinical vignette is consistent with aspirin overdose, which is associated with the development of acute renal failure and transient hepatotoxicity, which can lead to coagulopathy, severe encephalopathy and non-cardiogenic pulmonary edema. Therapy consists of gastric lavage, administration of activated charcoal, and alkalinization of the urine to enhance secretion. (Choice C) The patient is dehydrated. Giving furosemide will aggravate her dehydrated status and renal failure. (Choice B) The encephalopathy in this case is due to intoxication and metabolic acidosis, not hepatic disease. Lactulose and metronidazole will not be useful here. (Choice D) Non-invasive mechanical ventilation is not needed as the patient does not have pCO2 retention and is not in ventilatory failure. (Choice A) Even though the patient is febrile, antibiotics are not needed because the cause of her hyperthermia is drug overdose, and there is no evidence of infection. Aspirin causes uncoupling of the oxidative phosphorylation, resulting in hyperthermia. Educational Objective: Salicylate overdose presents with high anion gap metabolic acidosis, compensatory respiratory alkalosis, transient hepatotoxicity, acute renal failure and depressed mental status. Therapy is aimed at eliminating the offending agent through gastric lavage and administration of activated charcoal. If the mental status is optimal, vomiting can be induced. It is important to determine salicylate serum levels. Values greater than 35 mg/dL indicate significant acidosis, and alkalinization of the urine is indicated to enhance aspirin excretion. Case 45 A 36-year-old Caucasian nulligravida presents to your office for a routine check-up. Her past medical history is insignificant. She is concerned about the possibility of cervical cancer, because a friend of hers was recently diagnosed with invasive cervical cancer. You perform a Pap smear, and the results indicate that the sample is satisfactory, and a high-grade squamous intraepithelial lesion (HGSIL) is present. Which of the following is the next best step in the management of this patient? A. Repeat cytology in 6 months B. Proceed with colposcopy C. Do HPV testing D. Do excisional biopsy E. Reassure and repeat Pap smear in 12 months Explanation:

HGSIL revealed on Pap smear indicates a 1-2% probability of already having invasive cervical cancer and a 20% probability of acquiring invasive cervical cancer if left untreated. Immediate referral for colposcopy and endocervical curettage is indicated. (Choice D) If colposcopy suggests HGSIL, a diagnostic excisional procedure should be performed. (Choice C) HPV testing may be indicated if cytologic examination reveals atypical squamous cells of undetermined significance (ASCUS). The results of HPV testing may influence the decision to proceed with a colposcopy in such cases. (Choices A and E) Repeating the cytology in 6 or 12 months is not correct because invasive cervical cancer can be missed. Educational Objective: If a Pap smear reveals a high-grade squamous intraepithelial lesion (HGSIL), immediate referral for a colposcopy is indicated. 79% of people answered this question correctly. Case 46 An 18-month-old girl is brought to the office by her mother because of bilateral breast enlargement for the last three months. She has no known medical problems. There is no history of headaches, visual problems, growth of axillary or pubic hair, change in genitalia, rapid increase in height, or change in body habitus. On physical examination, she appears to be normal and playful. Her height and weight are at the 75th percentile on the growth curve for normal females (her weight and weight has remained at the 75th percentile on this growth chart for the last year). She has 3 cm of breast tissue on both sides. She has stage 1 pubic hair and no axillary hair or odor. Her genitalia are normal. The rest of the physical examination is within normal limits. Her LH, FSH, and estradiol levels are within normal range for her age group. TSH and prolactin levels are within normal limits as well. What is the most likely cause of this patient's breast enlargement? A. Benign premature thelarche B. Hypothalamic hamartoma C. Adrenal tumor D. Ovarian tumor E. McCune-Albright syndrome Explanation: The clinical features in this patient are consistent with benign premature thelarche, which is characterized by bilateral breast enlargement not accompanied by other signs of isosexual precocious puberty. These other signs of precocious puberty include rapid increase in height, increase in bone maturity, appearance of axillary and pubic hair, and menstrual bleeding. The treatment for benign premature thelarche is expectant because majority of the patients remain stable or have reversal of the breast enlargement in a few months. Patients with benign premature thelarche have a normal hormone profile. Their final height is generally not compromised. (Choice B) Hypothalamic hamartomas secrete GnRH and cause central isosexual precocious puberty in both males and females. Central precocious puberty is characterized by rapid acceleration of height, increase in bone age, thelarche, adrenarche, pubarche, and menarche. Lab investigations in patients with hypothalamic hamartoma reveal gonadotropin levels in the pubertal range and elevated estrogen levels. Majority of these patients will require treatment with a GnRH analog. (Choice C) Adrenal tumors which lead to the excessive production of estrogen are exceedingly uncommon. Adrenal tumors can produce cortisol and androgen. Excessive androgen production can

cause virilization in females (heterosexual precocious puberty) and peripheral isosexual precocious puberty in males. Excessive cortisol secretion leads to Cushings syndrome.

(Choice D) Estrogen production from an ovarian tumor can lead to peripheral precocious puberty. This syndrome has similar clinical features as hypothalamic hamartoma, and is characterized by accelerated height and bone age, and menstrual bleeding. Hormonal profile reveals elevated estradiol in the presence suppressed LH and FSH. Treatment is usually surgical. (Choice E) McCune-Albright syndrome consists of "caf-au-lait" spots, fibrous dysplasia of the bone, and precocious puberty. This patient does not have any features to suggest McCune-Albright syndrome. The cause of precocious puberty in McCune-Albright syndrome is excessive production of estrogen from ovarian cysts. Educational Objective: Benign premature thelarche occurs at 18 to 24 months of age and is characterized by breast enlargement without other features of precocious puberty. The treatment of benign premature thelarche is expectant because majority of patients remain stable or improve in a few months. 58% of people answered this question correctly. Case 47 A 46-year-old male with type-2 diabetes mellitus is scheduled to undergo cardiac catheterization in two days for an undiagnosed episodic chest pain. The patient has been having retrosternal chest pain radiating to his arm for the past 15 days. The pain is gradually becoming more frequent. He went to the emergency department about 7 days ago following an episode of chest pain. He was admitted overnight, and he left against medical advice when three sets of cardiac enzymes were negative. His EKG revealed nonspecific ST-T changes. The patient has had diabetes for the past six years, and has been on metformin (1,000 mg twice daily) for the past several years. He does not perform selfmonitoring of his blood sugar. His chemistry profile and CBC from his last emergency department visit were normal. He also has a history of hypertension that is controlled with atenolol (50 mg once daily) and hydrochlorothiazide (12.5 mg once daily). Baby aspirin (81 mg once daily) was started on his last emergency room visit. He smokes one pack a day, and drinks alcohol socially. He has a strong family history of premature coronary artery disease. On examination, he is 5?6" (165 cm) tall, and weighs 180 lbs. (82 kg). His blood pressure is 136/70 mmHg and pulse rate is 66/min. The rest of the clinical examination is normal. What is the next best step in this patient?s care? A. Stop metformin B. Stop hydrochlorothiazide C. Increase atenolol D. Discontinue aspirin E. No change in therapy until cardiac catheterization Explanation: Metformin was approved in the United States for the treatment of type-2 diabetes after its use in the rest of the world showed that the chances of lactic acidosis are minimal when used in patients without any contraindications. Major contraindications include renal insufficiency (creatinine more than 1.5 mg/dl in males, 1.4 mg/dl in females or creatinine clearance less than 60 ml/min), hepatic dysfunction, alcoholics, sepsis, and congestive heart failure. Because of its potential to cause renal dysfunction when a large amount of contrast is being infused, metformin is typically held before or at the time of procedure. It should not be restarted unless normal renal function is documented following the procedure.

(Choice B) Hydrochlorothiazide is usually not catheterization, especially when used in low doses.

associated

with

any

problems with

cardiac

(Choice C) There is no need to increase atenolol dosage, because the patient appears to be adequately beta-blocked. (Choice D) Aspirin can be continued safely during cardiac catheterization. In procedures where an increased risk for bleeding is expected, aspirin should be discontinued at least seven days before the procedure, because it may cause platelet dysfunction that can last for more than a week. (Choice E) Metformin needs to be discontinued before cardiac catheterization. Educational Objective: Metformin use is contraindicated in patients with renal failure, sepsis, hepatic dysfunction, and severe heart failure. It should also be stopped in patients who are at risk to develop renal failure, such as those who will undergo angiography, a procedure that involves infusion of a high load of contrast agents. 37% of people answered this question correctly. Case 48 The following vignette applies to the next 3 items A 46-year-old Hispanic man is brought by the police to the emergency department (ED). The police found him unresponsive on a bench in a nearby park. His clothes are dirty and torn, and there is evidence of vomitus on his lips and clothes. His breath smells of alcohol. His temperature is 36 C (97 F), blood pressure is 96/75 mmHg, heart rate is 110/min, and respiratory rate is 12/min. He is extremely drowsy, disoriented and unresponsive to any of your questions. He has a disheveled appearance, and has cool extremities. Spider angiomas and palmar erythema can be identified on closer inspection. He has decreased breath sounds at the lower base of the right lung. The cardiovascular examination is within normal limits. His abdomen is soft; there is no tenderness or guarding. There is evidence of mild hepatomegaly. While in the ED, he has a large emesis with gross bright, red colored blood. Item 1 of 3 What is the next best step in the management of this patient? A. Perform the endotracheal intubation B. Initiate aggressive fluid resuscitation C. Start the patient on IV octreotide D. Send him for stat endoscopy E. Give the patient intravenous propranolol Explanation: Based on the above presentation, the patient is most likely suffering from an acute variceal hemorrhage secondary to chronic alcoholic liver disease. Acute variceal hemorrhage is a major cause of death in patients with chronic liver disease or cirrhosis. Apart from the acute blood loss, the complications of bleeding and its treatment are responsible for a significant amount of morbidity and mortality in these patients. The primary goal of management of patients with variceal hemorrhage is to maintain hemodynamic stability, prevent the complications of bleeding and its treatment, and control active bleeding. Adequate control and protection of the airway should always be a priority in the initial management of all patients with active or recent variceal bleeding. This can be initially done by keeping the patient in a left decubitus position or by gastric decompression via the insertion of a nasogastric tube. In patients who are disoriented and unresponsive, the airway can be secured via endotracheal intubation to prevent the risk of aspiration and its complications during active bleeding or its treatment. If the

patient is hemodynamically unstable at the initial presentation, aggressive fluid resuscitation and airway management should be performed simultaneously. The patient in the above vignette has marked disorientation and can deteriorate rapidly with further recurrent episodes of bleeding; therefore, it is important to secure his airway with endotracheal intubation at this point before instituting any specific therapy. (Choice B) Although the patient has had an acute bleeding episode, he appears to be hemodynamically stable. Airway protection should be the priority at this time. If there is any evidence of hemodynamic compromise, aggressive resuscitation and airway protection should be done simultaneously. (Choices C and D) The patient should not be sent for any specific treatment before securing his airway. (Choice E) Intravenous propranolol has no role in the management of active variceal bleeding. Educational Objective: Airway protection, followed closely or simultaneously by hemodynamic resuscitation, should be a priority in all patients with active variceal hemorrhage. Item 2 of 3 The initial appropriate steps were taken for the patient. His current temperature is 36.2 C (97.4 F), blood pressure is 112/66 mmHg, heart rate is 96/min, and respiratory rate is 13/min. The upper GI endoscopy shows large esophageal varices with evidence of recent bleeding, which is treated with injection sclerotherapy. He is admitted to the hospital for further observation and management. On the second day of hospitalization, he becomes extremely disoriented and belligerent. You suspect that he is having acute alcohol withdrawal. He is started on treatment for alcohol withdrawal, and his condition improves over the next day. On his fourth day of hospitalization, he has another episode of large bloody emesis. What is the most appropriate next step in the management of this patient? A. Repeat endoscopy and band ligation B. Perform a Transjugular intrahepatic portosystemic shunt C. High-dose intravenous octreotide D. Sengstaken-Blakemore tube insertion E. Give the patient intravenous propranolol Explanation: Urgent upper GI endoscopy with either injection sclerotherapy or band ligation is the definite treatment of choice for patients with active variceal hemorrhage. It is generally successful in controlling acute bleeding in 80-90% of the patients. All patients with a recent variceal hemorrhage are at a high risk of recurrent hemorrhage or early re-bleeding. This risk of re-bleeding is greatest within the first 48-72 hours, and remains high for the first six weeks following the cessation of active bleeding. Some of the known risk factors for early re-bleeding include an age greater than 60 years, severe initial bleeding, large varices, and the presence of renal failure. In all patients who have another episode of variceal bleeding more than 48 hours after the initial endoscopic treatment, as in the above vignette, a repeat endoscopy with band ligation should be attempted initially to achieve hemostasis. If the patient continues to bleed despite endoscopic treatment, more definitive treatment with either balloon tamponade or portosystemic shunt should be used to control the active bleeding. (Choices B and D) Endoscopic treatment with band ligation or sclerotherapy is the treatment of choice for active variceal bleeding. Sengstaken-Blakemore tube insertion for balloon tamponade or TIPS procedure should be employed to achieve hemostasis only if the endoscopic treatment fails to control the bleeding.

(Choice C) Somatostatin or intravenous octreotide can initially be used in patients with suspected acute variceal hemorrhage. It should not replace endoscopic treatment as a more definite procedure of choice for acute variceal hemorrhage. Educational Objective: Endoscopic therapy with either band ligation or sclerotherapy is the definite procedure of choice for early re-bleeding in patients with recent variceal hemorrhage. Item 3 of 3 The appropriate step is taken. One week later, the patient is ready for discharge. In addition to providing counseling to completely stop alcohol consumption, which of the following is the treatment of choice to reduce recurrent bleeding? A. Proton pump inhibitors B. Propranolol C. Cimetidine D. Prostaglandin analogs E. Follow-up endoscopy every six months Explanation: All patients with a history of variceal hemorrhage are at high risk for re-bleeding or recurrent variceal hemorrhage. This risk is greatest in the first few days, and returns to baseline by six weeks; however, if left untreated, approximately 70% of the patients suffer from recurrent variceal bleeding within the first year from the initial bleeding episode. Recurrent bleeding is one of the important causes of death in such patients. Therefore, it is important to institute specific measures to prevent recurrent variceal hemorrhage. Of all the above available options, propranolol (non-selective beta-blockers) has been shown to reduce the risk of recurrent variceal hemorrhage. This is likely due to reduction in the portal pressure or hepatic vein pressure gradient seen with the use of non-selective beta-blockers. (Choices A, C, and D) H2 blockers (Cimetidine), proton pump inhibitors or prostaglandin analogs have not been shown to reduce the risk of recurrent variceal hemorrhage. (Choice E) Regular endoscopic treatment with band ligation is the treatment of choice for long-term management of variceal hemorrhage. Follow-up endoscopy alone has no role in the prevention of recurrent bleeding. Educational Objective: Non-selective beta-blockers (propranolol or nadolol) have been shown to reduce the risk of re-bleeding in patients with a history of variceal hemorrhage. 74% of people answered this question correctly. Case 49 A 12-year-old Caucasian female develops an episode of severe diarrhea that requires hospitalization. Her past medical history is insignificant, and she has no history of recent travel. Her stool sample is positive for Giardia lamblia trophozoites. After hearing about her condition, the parents of her classmates insisted on having their children screened for giardiasis. Twenty percent of the class turned out to be positive for Giardia cysts on stool ova and parasite test. Which of the following is the best strategy to manage giardiasis in this case? A. Isolate all carriers B. Treat all carriers, but not isolate them C. Treat all carriers and their family members D. Treat only symptomatic carriers E. Treat all carriers and people at risk

Explanation: Giardiasis has several routes of transmission: person-to-person, food-borne and waterborne. Personto-person transmission occurs in two settings: (1) in institutions where there is fecal incontinence and poor hygiene (e.g., some daycare centers), and (2) in male homosexuals. Symptomatic patients with giardiasis should receive appropriate treatment. (Choice A) Asymptomatic infection occurs in approximately 60 percent of people exposed to Giardia. Asymptomatic cyst carriage can last over six months. There is no need to isolate asymptomatic carriers once appropriate hygienic measures are undertaken. (Choices B, C and E) Asymptomatic carriers are not usually treated, except in specific instances such as outbreak control and for prevention of household transmission by toddlers to pregnant women and patients with hypogammaglobulinemia or cystic fibrosis. Educational Objective: Asymptomatic carriers of Giardia lamblia are not usually treated, except in specific instances such as in outbreak control and for prevention of household transmission by toddlers to pregnant women and patients with hypogammaglobulinemia or cystic fibrosis. 38% of people answered this question correctly.

Case 50 A 6-year-old boy is brought to the pediatrician after a sudden onset of fever and difficulty in breathing. According to his mother, he was complaining of a sore throat and some trouble swallowing yesterday, but these symptoms resolved after he received some over-the-counter analgesics. On physical examination, he appears toxic. His voice is muffled. He is leaning forward and moving around restlessly. His mouth is open wide and his tongue is protruding. What is the most appropriate course of action? A. Obtain lateral neck radiograph B. Perform laryngoscopy in office C. Refer to local pediatric hospital for laryngoscopy D. Administer corticosteroids and racemic epinephrine E. Call ambulance and send to emergency department Explanation: With the introduction of the Haemophilus influenzae type b vaccine in 1985, epiglottitis has become much less common in recent years. When it does occur, it causes an inflammatory edema of the epiglottis that impinges upon the airway. Respiratory arrest is easily provoked at this stage, and treatment should be focused on relieving any airway obstruction and treating the infection. The threshold for performing intubation should be very low. Since the outpatient setting is inadequate for such procedures, it is imperative that an ambulance be called so the childs condition can be properly addressed by emergency personnel. (Choice A) Frequently, epiglottitis is diagnosed solely by the clinical presentation. However, if a lateral neck radiograph is deemed necessary, it will reveal a swollen epiglottis (the "thumbprint sign") in classic epiglottitis. In the child in this case, an endotracheal intubation should be performed before the radiograph is obtained, because he is at great risk for respiratory arrest. (Choices B and C) Laryngoscopy can be used to exclude other causes of airway obstruction in a cooperative child; however, it is not recommended for cases of suspected epiglottitis as it may worsen the patient's condition.

(Choice D) Corticosteroids and racemic epinephrine have not been shown to be helpful in the treatment of epiglottitis. Educational Objective: Epiglottitis is a serious, life-threatening infection. Treatment should be focused on relieving any airway obstruction and treating the infection. The threshold for performing intubation should be very low. Case 51 The following vignette applies to the next 3 items A 9-month-old Caucasian male is brought to the emergency department on one winter night with oneday history of poor appetite, nasal discharge and wheezing. His past medical history is insignificant. He has no known allergies, and all his vaccinations are up-to-date. He has been meeting all developmental milestones appropriately. His temperature is 38.3 C (101 F), blood pressure is 90/60 mm Hg, pulse is 150/min, and respirations are 40/min. Oxygen saturation on room air by pulse oximetry is 91%. Lung auscultation reveals expiratory wheezing and prolonged expiration. The liver edge is palpable. Chest-x ray shows hyperinflation. The patient is admitted to the hospital. Item 1 of 3 Which of the following is the best management for this patient? A. Respiratory isolation and trial of bronchodilators B. Short course of corticosteroids C. Respiratory isolation and ribavirin therapy D. Administer decongestants and expectorants E. Respiratory isolation and erythromycin for 7-10 days Explanation: This patient presents with a clinical picture that is most consistent with bronchiolitis: young age (less than 2 years), mild upper respiratory symptoms, mild-to-moderate fever and wheezing are typical. The liver and spleen are palpable because of hyperinflation of the lungs, but these are not enlarged. In healthy infants and young children, bronchiolitis is usually a self-limited disease; however, patients who are hypoxic or cannot feed because of distress should be hospitalized. These children should be kept in respiratory isolation. Therapy in most cases consists of supportive measures. Humidified oxygen and tube or intravenous feedings are indicated. Although there is no strong evidence that inhaled bronchodilators are effective in patients with bronchiolitis, it is a routine practice to administer these (nebulized albuterol or epinephrine) and observe the patient for any effect. If no prompt clinical response is seen, most clinicians discontinue these drugs. (Choice B) Corticosteroids are not routinely recommended, but may be beneficial in patients with an underlying chronic lung disease and in those with prior wheezing episodes. (Choice C) Although ribavirin is a nucleoside analogue with good in vitro activity against RSV, studies examining its effect in children have been conflicting, and the cost for a course of therapy is substantial. It is usually reserved for patients with severe disease. (Choices D and E) Antibiotics, decongestants, and expectorants are of no value in the treatment of patients with bronchiolitis. Educational Objective: Although there is no strong evidence that inhaled bronchodilators are effective in patients with bronchiolitis, it is a routine practice to administer these and observe the patient's clinical response. 55% of people answered this question correctly.

Item 2 of 3 Which of the following is the most appropriate next step in the diagnosis of his condition? A. Rapid tissue culture B. Rapid detection of antigen in nasal secretions C. Serology for antibody detection D. Sputum induction and Gram stain E. Urinary antigen assays Explanation: Rapid detection of RSV antigen in nasal or pulmonary secretions utilizing antigen capture technology is now available and can be performed in less than 30 minutes. The sensitivity and specificity of such tests exceed 90 percent. (Choice B) Even though the definitive diagnosis is made by tissue culture, it takes 4 days to 2 weeks for the results to be reported. (Choice C) Diagnostic serology is not helpful in infants because this also detects maternal antibodies. Educational Objective: Diagnosis of RSV is quickly made by detection of RSV antigen in nasal or pulmonary secretions by ELISA. 63% of people answered this question correctly. Item 3 of 3 While you are giving the appropriate management to this patient, his mother asks about what she should expect in the future. Which of the following diseases is the child at risk of developing? A. Pneumonia B. Lung abscess C. Cystic fibrosis D. Asthma E. Aspergillosis Explanation: Although mild RSV infection does not produce apparent long-term pulmonary sequelae in most individuals, up to 30% of patients hospitalized with severe infection will subsequently develop reactive airway disease later in childhood (Choice A) A secondary bacterial infection of the middle ear is the most common complication (1020%) of RSV bronchiolitis, but bacterial pneumonia is very rare (0.5-1%). (Choice B) Lung abscess is not seen with respiratory syncytial virus. (Choice C) These children are not prone to cystic fibrosis (CF). CF is a congenital condition and is not acquired from any infection. (Choice E) Aspergillosis infections generally occur in immunocompromised patients. The disease may present as an allergic reaction involving the lung, or may present with a cavitary lesion in the lung. Disseminated Aspergillus infections are only seen in immunocompromised individuals. Aspergillus infections are not related to prior respiratory syncytial infections. Educational Objective:

A secondary bacterial infection of the middle ear and future risk of bronchial hyperreactivity are the most common complications of RSV bronchiolitis. 68% of people answered this question correctly. Case 52 The following vignette applies to the next 2 items A 50-year-old obese man comes to the emergency department with a painful left great toe. He was sleeping peacefully at night, when he woke up with sudden severe pain in his left great toe. He never had this kind of problem in the past. He denies trauma to the affected part. He smokes 1 pack of cigarettes daily and drinks alcohol regularly. Physical examination shows normal vital signs. His left great toe is inflamed and swollen. The patient does not allow further examination due to intense pain. There is no skin rash. Item 1 of 2 What is the next step in making the diagnosis? A. Allopurinol B. Arthrocentesis C. X-ray of the great toe D. MRI E. Aspergillosis Explanation: The patient has features suggestive of acute monoarticular gout. The diagnosis should be confirmed by needle aspiration of the joint fluid. The joint aspirate in gout shows characteristic negatively birefringent crystals of monosodium urate. An infection can coexist with acute gout; therefore, the joint fluid should be cultured if there is any suspicion for an infectious process. (Choice A) Serum uric acid is not used for making a diagnosis of acute gout because the uric acid levels can be normal or even low during an acute attack. (Choices C & D) Radiologic imaging plays no role in the diagnosis of acute gout. Educational Objective: A diagnosis of acute gout is confirmed by arthrocentesis by the demonstration of negatively birefringent crystals. Serum uric acid can be normal during acute gout and is not useful in making the diagnosis. 82% of people answered this question correctly. Item 2 of 2 The diagnosis of acute gout is made. Which of the following is the treatment of choice at this time? A. Allopurinol B. Colchicine C. Indomethacin D. Probenecid E. Methotrexate Explanation: NSAIDS are used as the first line agents in the treatment of acute gout in most patients. NSAIDS are effective in more than 90% of patients with acute gout. Agents with a short half-life (e.g.,

indomethacin and ibuprofen) are most effective. Majority of the patients become asymptomatic in 5-7 days. Other anti-inflammatory agents such as colchicine or glucocorticoids can be used. Intra-articular glucocorticoids may be used in elderly patients with renal failure. (Choices A and D) Hypouricemic therapy with allopurinol or probenecid should not be started during acute gout. When hypouricemic therapy is started during an acute episode, there are high chances of flare-up of the disease. (Choice B) Colchicine can be used in the treatment of acute gout. Typically, 0.6 mg is given every hour until there is relief of symptoms or GI toxicity occurs. A total of 2 to 3 mg can be given. High doses of colchicine are associated with serious toxicity. Lower doses are advocated in elderly patients. Due to its potential serious toxicity, colchicine is not the first line agent in most patients with acute gout. (Choice E) Methotrexate is not indicated in the treatment of acute or chronic gout. Educational Objective: Acute gouty arthritis is treated with NSAIDs as first line therapy. Colchicine and glucocorticoids can also be used. Hypouricemic therapy is never used during an acute episode of gout. 79% of people answered this question correctly. Case 53 A large-scale clinical trial was conducted to assess the effect of carvedilol (a mixed alpha- and betablocker on the clinical course of chronic heart failure), NYHA class III-IV. The study showed that serum sodium level correlated with serum noradrenaline level, a marker of the degree of neurohumoral activation: correlation coefficient, r = - 0.39 and p = 0.005. Which of the following statements correctly describes the association between serum sodium level and serum noradrenaline level observed in this study? A. Strong correlation was observed B. As serum sodium level increases serum noradrenaline level also increases C. Serum sodium changes cause serum noradrenaline changes D. The association does not reach statistical significance E. Negative correlation is observed Explanation: A negative correlation is present between serum sodium level and serum noradrenaline level, which is indicated by the negative sign of the correlation coefficient. The null value for the correlation coefficient is 0 (no association), and the range of plausible values is from -1 to 1. The sign of the correlation coefficient indicates positive or negative association. The closer the value is to its margins (-1 or 1), the stronger the association. (Choice B) Negative correlation means that as serum sodium level decreases, serum noradrenaline level increases. (Choices A and D) In this scenario, the association is weak because the value of the correlation coefficient is close to 0, although it is statistically significant: p = 0.005. (Choice C) It is very important to know that the correlation coefficient shows the strength of association, but does NOT necessarily imply causality. Educational Objective: The correlation coefficient shows the strength and the direction (positive, negative) of linear association between two variables. It does not necessarily imply causality.

48% of people answered this question correctly. Case 54 A 49-year-old blue-eyed, blonde-haired Caucasian woman presents to clinic requesting treatment for "this unattractive redness of my facial skin." She reports that in the past three months, she has developed new blemishes on her nose, cheeks, and chin. She has also noticed that she now flushes intensely in response to extreme temperatures, emotion, and the consumption of spicy foods or alcohol. Based on recommendations from her friends, she modified her diet and tried various herbal preparations and skin creams, but nothing has alleviated her symptoms. She has no history of dermatological problems and did not have acne as a teenager. She has already undergone menopause. Physical examination reveals a reddening of the central face, erythematous papules on the chin, and multiple telangiectasias on the nose and cheeks. Her skin is very fair otherwise. Item 1 of 2 Which of the following is the most appropriate treatment of this condition? A. Oral griseofulvin B. Topical clindamycin C. Topical isotretinoin D. Topical metronidazole E. Topical permethrin Explanation: Rosacea is a chronic acneiform condition characterized by vascular dilation in the central face. It is most common in adults ages 30 to 60 years and occurs more often in individuals with light skin, hair, and eye color. Periods of exacerbation and remission are expected. Symptoms generally include facial erythema (both transient and nontransient), telangiectasias, and papules and pustules. The inflammatory lesions seen generally appear identical to those found with acne, although comedones are not present. Therapy is usually successful in reducing the number and severity of the inflammatory lesions, but facial erythema may persist despite treatment. It is important that patients with rosacea use mild cleansers and sunscreens regularly while also avoiding irritants. One of the most frequently used initial treatments for this condition is topical metronidazole (Choice D), with or without a course of oral antibiotics. The metronidazole may need to be applied daily on a long-term basis to maintain remission. Oral griseofulvin (Choice A) is not prescribed for the treatment of rosacea. The oral antibiotics commonly used include tetracycline, doxycycline, minocycline, and erythromycin. Topical clindamycin (Choice B) can be used in the form of a solution, gel, or lotion, but is generally considered less effective than topical metronidazole. Topical isotretinoin (Choice C) is appropriate for those patients with papular or pustular lesions unresponsive to initial therapies. It may also be used in conjunction with topical antibiotics, especially in those patients with refractory symptoms. Some clinical data suggests that topical permethrin (Choice E) improves symptoms of rosacea, a not unexpected finding as Demodex mites are frequently found in increased numbers in the skin scrapings of patients with rosacea. However, further evaluation is indicated before this treatment can be recommended as appropriate first-line therapy. Educational Objective: Rosacea is a chronic acneiform condition characterized by vascular dilation in the central face. An excellent initial treatment for this condition is topical metronidazole, with or without a course of oral antibiotics. Item 2 of 2 The appropriate action was taken. Which of the following complications is most frequently associated with this condition?

A. Chalazion B. Glaucoma C. Endophthalmitis D. Sinusitis E. Squamous cell carcinoma Explanation: Patients with rosacea often report experiencing ocular symptoms. Complaints frequently include burning or foreign body sensations, blepharitis, keratitis, conjunctivitis, episcleritis, and chalazion (Choice A). The chalazion is characterized by a granulomatous inflammation of a meibomian gland and presents as a painless, pea-sized nodule within the eyelid. Acute angle-closure glaucoma (Choice B) is manifested as a painful, red eye. Accompanying symptoms may include nausea, vomiting, and headache. Glaucoma is not typically associated with rosacea. Endophthalmitis (Choice C) is classically associated with corneal ulcers and ocular infections with organisms such as Pseudomonas, Yersinia, and Mycobacterium. Rosacea, in contrast, usually causes a more focal and superficial ocular irritation. Sinus infections (Choice D) can arise when mucosal edema obstructs the ostia, often subsequent to viral upper respiratory tract infections. Sinusitis is not typically associated with rosacea. Classically, squamous cell carcinoma (Choice E) presents as an ulcerated nodule or superficial erosion of sun-damaged skin or lip. Although it is more common in the fair-skinned, is not typically associated with rosacea. Educational Objective: Patients with rosacea may also experience ocular symptoms. Complaints frequently include burning, foreign body sensation, blepharitis, keratitis, conjunctivitis, episcleritis, and chalazion (dysfunction of the meibomian gland). Case 55 A 66-year-old postmenopausal female comes to the emergency department and complains of fever and left flank pain for the past four days. She also complains of chills, increased frequency of micturition, and dysuria. The pain in her left flank is described as agonizing, non-radiating, and aggravated by motion. She denies any hematuria, cough, dyspnea, nausea, vomitings, and skin rash. Her past medical history is significant for type-2 diabetes for the past ten years, hypertension, dyslipidemia and osteoporosis. Her fasting blood glucose levels have been ranging between 100-140 mg/dl. Her HBA1c, performed 2 weeks ago, was 8.2 % (normal 4-6%). Physical examination reveals a middle-aged woman in distress, with a temperature of 102 F(39 C), blood pressure of 100/70 mm Hg, and heart rate of 100/min. There is marked tenderness in her left flank. CBC reveals polymorphonuclear leukocytosis. The basic metabolic panel is normal, except for a BUN level of 33mg/dl (normal 10-20 mg/dl) and glucose level of 320 mg/dl. Urine examination shows 1+ protein, 3 + glucose, many WBCs, WBC casts and positive leucocyte esterase. Blood cultures are drawn. CT scan of the abdomen shows a fluid collection in the left flank, with extensive perinephric stranding. Air is seen in the left kidney and perinephric space. Besides IV hydration and glycemic control, what is the best next step in this patient's care? A. Parental antibiotics B. Parental antibiotics and percutaneous drainage C. Parental antibiotics and immediate nephrectomy D. Parental antibiotics and surgical drainage E. Oral antibiotics Explanation: There are a number of infections which predominantly occur in diabetic individuals. Emphysematous pyelonephritis is one of them. E. coli and K. pneumoniae are the most common organisms involved.

Treatment is guided by the extent of involvement of the perinephric space. If abscess and gas extend into the perinephric space, an immediate nephrectomy is necessary. (Choice B) Mild pyelonephritis is managed with a combination of parental antibiotics and percutaneous drainage. (Choices A and E) Parental antibiotics alone are not generally sufficient, and oral antibiotics are usually ineffective. (Choice D) Surgical drainage does not offer a significant advantage over percutaneous drainage. Emphysematous cholecystitis also occurs predominantly in diabetics. As compared to emphysematous pyelonephritis, which occurs more commonly in females, emphysematous cholecystitis is seen commonly in males. Clostridia and E. coli are the most common organisms responsible for emphysematous cholecystitis. The most common symptoms are fever, right upper quadrant pain, nausea and vomiting. Crepitus on abdominal examination is very suggestive of emphysematous cholecystitis. The diagnosis is made on CT scan, which depicts gas within the gall bladder. All patients require urgent removal of the infected gall bladder and parental antibiotics. Open surgical removal of the infected gall bladder is preferred over laparoscopic removal. Educational Objective: Parenteral antibiotics and immediate nephrectomy is indicated in patients with advanced emphysematous pyelonephritis. Case 57 A 45-year-old female patient of yours comes to see you in the office. She is accompanied by her 14year-old daughter. She has been your patient for the last 20 years, and you have an excellent rapport with her. Her daughter became sexually active three months ago. She is aware of the risk of cervical cancer with sexual activity, and wants you to convince her daughter to have a cervical Papanicolaou smear done. What is the most appropriate response in this setting? A. Reassure the mother & tell her that a Pap smear is not necessary at this time. B. Reassure the mother and tell her that you will do the Pap smear at this time. C. Tell the mother that you will do the Pap smear if she has been exposed to multiple partners. D. Tell the mother that you will do the Pap smear if she has engaged in unprotected intercourse. E. Tell her that she does not need a Pap smear until she is 18 years of age. Explanation: Human papilloma virus (HPV) infection is a sexually transmitted infection strongly linked with the development of cervical intraepithelial neoplasia (CIN) and cervical cancer. Early initiation of sexual activity, multiple new sexual partners, and high-risk partners (partners with HPV infection), are the main risk factors for the acquisition of HPV infection and cervical cancer. Screening for cervical cancer by cytologic examination/Pap smear is an effective way of detecting early pre-invasive and invasive carcinoma. According to the guidelines, screening for cervical intraepithelial neoplasia (CIN) or cancer should be started three years after the onset of sexual intercourse, or at the age of 18 years (recently changed to 21 years) whichever is earlier. This is due to the fact that high-grade cervical cytologic abnormalities due to HPV usually do not occur until three to five years after HPV exposure. The daughter in the above vignette started engaging in sexual activities only three months ago. There is no reason to start screening her for cervical cancer by Pap smear at this time. The mother should be assured and informed of the facts regarding the timing of exposure and initiation of screening. (Choices C and D) The initiation of cervical cancer screening does not depend on the history of multiple partners or unprotected intercourse. Educational Objective: Cervical cancer screening should be started three years after initiation of sexual intercourse or at the age of 18 years (some say 21 years). Case 58 A 30-year-old female presents with tingling and numbness in both of her distal lower extremities for the last three months. Her symptoms are gradually worsening. She also complains of intense fatigue. She denies weight change, constipation, skin dryness, headaches or visual changes. She was

diagnosed with primary hypothyroidism secondary to Hashimotos thyroiditis when she was 10 years old. She is currently on 100 mcg per day of levothyroxine orally. Her mother also has hypothyroidism. She does not smoke or drink alcohol. She denies the use of recreational drugs. Her menstrual cycles have been irregular for the last six months. Her last menstrual period was two months ago. She is sexually active with one partner. Her blood pressure is 114/78 mm Hg and heart rate is 72/min. She weighs 152 pounds and is 54" tall. Her thyroid is nonpalpable. Her mucous membranes are moist and pale. Her tongue appears to be bald. She does not have any scleral icterus. Vibration and proprioception are decreased in both of her lower extremities distally. Pain and temperature sensations appear normal. Her ankle jerks are absent, but other reflexes are brisk. Babinski and Rombergs signs are positive. The rest of the physical examination is unremarkable. Stool obtained from the rectal examination is negative for occult blood. Lab investigations reveal a hemoglobin of 8 g/dL and hematocrit of 24%. Her total WBC count is 3,200 per cubic millimeter. Her platelet count is 300,000 per cubic millimeter. Basic serum chemistries are within normal limits. Her Total T4 level is 8.2 mcg/dL (normal 4-12 mcg/dL) and TSH level is 2.4 micro IU/mL (normal 0.5 to 5 micro IU/mL). Item 1 of 2 What is the next best step in the management of this patient? A. Increase her levothyroxine dose to improve her fatigue B. Perform MRI of the spine to look for spinal cord compression C. Serum protein electrophoresis D. Measure serum B12 levels E. Measure folate levels Explanation: The patient's primary hypothyroidism predisposed her to other autoimmune disorders such as pernicious anemia. Involvement of the posterior column (loss of proprioception), lateral column (brisk reflexes) and peripheral nerves (loss of ankle jerks) indicate neurologic involvement due to Vitamin B12 deficiency, which resulted from atrophic gastritis (pernicious anemia). Dietary vitamin B12 binds to the intrinsic factor secreted by the parietal cells in the gastric mucosa. The intrinsic factor-vitamin B12 complex is then carried to the terminal ileum for receptor-mediated absorption. Pernicious anemia results from the autoimmune destruction of parietal cells, leading to achlorhydria and decreased production of the intrinsic factor. The lack of intrinsic factor leads to vitamin B12 deficiency. The red blood cells are macrocytic due to ineffective erythropoiesis secondary to defective nucleic acid synthesis. Neurologic involvement is characterized by involvement of posterior and lateral columns in the spinal cord known as 'subacute combined degeneration' and leads to ataxia, loss of proprioceptive and vibratory sensations. Severe spasticity, weakness, and peripheral nerve involvement can occur. Symptoms and signs are more prominent in the lower than the upper extremities. (Choice A) Increasing the levothyroxine dose is not the appropriate choice for this patient because her thyroid function tests are within normal limits. The fatigue in this patient is most likely due to anemia. (Choice B) Although neurologic involvement can indicate cord compression, the history and other features of this patient suggest otherwise; therefore, MRI is unnecessary. (Choice C) Endocrinopathy with polyneuropathy can be seen with POEMS (polyneuropathy, organomegaly, endocrinopathy, M-proteins and skin changes) syndrome. SPEP is done to look for an M-spike. This patient does not have organomegaly or skin changes. (Choice E) Folate levels may be obtained to determine the cause of marocytic anemia; however, the patient's history of autoimmune hypothyroidism and characteristic neurological picture is more suggestive of vitamin B12 deficiency. Educational Objective: Patients with primary hypothyroidism are predisposed to get other autoimmune diseases such as pernicious anemia. Vitamin B12 deficiency in pernicious anemia is due to a deficiency of intrinsic factor secreted by the stomach. Neurologic involvement in Vitamin B12 deficiency is characterized by subacute combined degeneration of the spinal cord and peripheral neuropathy. 87% of people answered this question correctly.

Item 2 of 2 Which of the following is most crucial to monitor during the first few days of treatment in the above patient? A. TSH level B. Hemoglobin level C. Serum potassium level D. Platelet count E. White blood cell count Explanation: Treatment with vitamin B12 in patients with moderate to severe megaloblastic anemia can cause hypokalemia, which can sometimes be very severe and life threatening. Hypokalemia results following the uptake of potassium by newly forming red blood cells. The patients' serum potassium levels should therefore be monitored during the first 48 hours. Potassium is replaced depending on the measured serum potassium levels. Some physicians transfuse packed red blood cells in patients with severe megaloblastic anemia before Vitamin B12 supplementation to prevent hypokalemia. (Choices A, B, D & E) These parameters are not necessary to monitor during the first few days of B12 therapy. Educational Objective: Patients with moderate to severe megaloblastic anemia can have severe hypokalemia during the first 48 hours of treatment with vitamin B12. Close monitoring and supplementation of potassium is required during this period. Case 59 A 36-year-old female comes to see you in the office for smoking cessation counseling. She has been struggling to quit cigarette smoking for the past year. She has a 50-pack-year history of smoking, and currently smokes two to three packs of cigarettes daily. She works as a legal assistant in a very reputable law firm in the city. She recently got married, and her husband does not smoke at all. They are planning to have a child next year, and she is really interested in quitting before that time comes. She has multiple failed attempts to quit smoking in the past. She says, "Every time I try to quit smoking, I start shaking really bad. I can't sleep, and I get very anxious and frustrated. I can't seem to work, which is bad because my job is very demanding. Eventually, I can't take it anymore, and I start smoking again. But this time, I'm really determined to quit. I need your help, doctor." Which of the following is the most appropriate next step in her management? A. Prescribe a low-dose nicotine spray. B. Prescribe bupropion. C. Encourage her to continue her attempts at smoking cessation. D. Ask her to change her job. E. Prescribe a high-dose nicotine patch. Explanation: Cigarette smoking causes significant physiological and psychological dependence. Withdrawal symptoms are extremely common in patients attempting to quit, and are especially pronounced in patients with a long history of heavy cigarette smoking. Some of the common symptoms include restlessness, anxiety, irritability, frustration, decreased concentration, insomnia, or depressed mood. These generally peak in the first three to four days after smoking cessation, and usually resolve in the next three to four weeks. These symptoms and the associated intense craving for cigarettes are responsible for the majority of relapses seen in patients. Most smokers typically make several attempts to quit before they are able to quit successfully. Current recommendations for the management of smoking cessation involve a combination of behavioral therapy and a pharmacologic intervention, such as buproprion or nicotine replacement. Nicotine replacement therapy (gum, patch or inhaler) acts by maintaining a low level of nicotine in the

blood. It is preferred for the amelioration of withdrawal symptoms during the early stages of smoking cessation. (Choice A) A low-dose nicotine inhaler is unlikely to ameliorate the withdrawal symptoms of a patient with a history of heavy smoking. She should initially receive high-dose nicotine replacement therapy. (Choice B) Nicotine replacement therapy is preferred to buproprion for the management of withdrawal symptoms in the early stages of smoking cessation. (Choices C and D) The patient needs a specific intervention to ameliorate her withdrawal symptoms in addition to moral support and behavioral therapy. Educational Objective: Nicotine replacement therapy is extremely helpful in ameliorating the symptoms of nicotine withdrawal in the early stages of smoking cessation.

Case 60 A 7-year-old Caucasian boy is brought to your office by his mother because of a sudden acceleration of growth and a very annoying rash. His past medical history is insignificant. Physical examination reveals severe cystic acne involving his face and shoulders, enlarged testicles and penis, and coarse pubic and axillary hair. His height corresponds to 98 percentile and his weight to 85 percentile for his age. Which of the following is the most likely diagnosis in this patient? A. Hypothalamic dysfunction B. 21-hydroxylase deficiency C. Severe hypothyroidism D. Abnormal karyotype 47, XXY E. Abnormal karyotype 47, XYY Explanation: It is very important to distinguish between the two most common presentations and causes of precocious pubarche in order to facilitate the proper treatment. Precocious puberty is caused by premature activation of the hypothalamus-pituitary-gonad (HPG) axis, while precocious pseudopuberty is caused by a gonadotropin-independent process, typically an excess of sex steroids. This patient presents with signs of severe androgen excess (e.g. severe cystic acne, significant growth acceleration) which suggests precocious pseudo-puberty. It can be caused by late-onset congenital adrenal hyperplasia. (Choice A) Hypothalamic dysfunction leading to precocious puberty is usually less dramatic in presentation. Sequential development of the following is typically present: testicular enlargement, penis enlargement, pubic hair growth, and lastly, a growth spurt. (Choice C) Severe hypothyroidism is a rare cause of precocious puberty that is characterized by the slowing, not acceleration, of growth. (Choice D) Klinefelters syndrome may present with a height that is higher than normal, but signs of androgen excess are not typical. (Choice E) 47, XYY karyotype may manifest as severe acne, but precocious puberty is not characteristic. Educational Objective: Precocious pubarche with signs of severe androgen excess is suggestive of precocious pseudopuberty that is caused by a gonadotropin-independent process, typically an excess of sex steroids.

Case 61 A 74-year-old Chinese-American woman is brought to the emergency department. She has had fever, chills, productive cough, and shortness of breath for the past 24 hours. Her other medical problems include hypertension and rheumatoid arthritis. Her medications include enalapril and naproxen. Her temperature is 38.9 C (102 F), blood pressure is 150/90 mmHg, pulse is 106/min., and respirations are 22/min. Examination showed decreased breath sounds and crackles at the left base. Fecal occult blood per rectum was negative. Her laboratory tests reveal the following: CBC Hb 9.4 g/dL Platelet count 450,000/cmm Segmented neutrophils 84% MCV 84 fl Leukocyte count 10,000/cmm Lymphocytes 16%

The chest x-ray showed an alveolar infiltrate in the left base. The patient was started on intravenous ceftriaxone and oral azithromycin. On the third day of treatment, additional hematologic tests are done, and these show the following: Ferritin 300 ng/mL (NV 15-120) Iron 44 mcg/dL (NV 50-170) TIBC 200 mcg/dL (NV 250-450) Which of the following is the most appropriate diagnostic test for this patients anemia? A. Erythropoietin levels B. Bone marrow biopsy C. Serum Transferrin D. Serum Transferrin saturation E. Erythrocyte sedimentation rate (ESR) Explanation: This patient has rheumatoid arthritis (RA), left lower lobe pneumonia, and normocytic, normochromic, anemia. Although RA usually causes anemia of chronic disease, a reliable diagnosis in this setting can only be established by performing a bone marrow biopsy. (Choices C and E) Because of the patient's acute infection, the ESR and ferritin levels will be abnormally increased, while the serum transferrin level and TIBC will be decreased. (Choice D) The serum transferrin saturation (serum iron/TIBC) and iron levels will not distinguish between anemia of chronic disease and iron deficiency anemia because both will present with diminished levels. (Choice A) Measurement of erythropoietin levels can help with decisions regarding the most adequate treatment of anemia of chronic disease in a patient with RA. This is not used to diagnose the anemia itself. Educational Objective: Anemia of chronic disease (ACD) can be difficult to distinguish from iron deficiency anemia in the setting of an acute bacterial infection. In this setting, the best test to confirm the diagnosis and rule out iron deficiency anemia is a bone marrow biopsy. Case 62 A 32-year-old Caucasian female comes to the emergency department (ED) and complains of a generalized, throbbing, headache that began during the night and got worse in the morning. She has also brought in her 8-year-old son, who has a similar type of headache and nausea. She says that her husband went to work today, although he was not feeling well. She denies any previous illness, and

has no idea what might have brought these symptoms on. Her temperature is 36.7C(98F), blood pressure is 120/76 mm Hg, pulse is 90/min, and respirations are 16/min. No neck rigidity or meningeal signs are present on physical examination. Which of the following tests can confirm the most probable diagnosis in this patient? A. Non-contrast CT scan of the head B. Lumbar puncture C. CBC with differential D. Carboxyhemoglobin level E. Serum acetaminophen level Explanation: The clinical scenario described is highly suggestive of carbon monoxide (CO) poisoning. The most important feature that should be emphasized is the involvement of several people that have similar symptoms. Interestingly, accidental CO poisoning has seasonal and regional variations, and are more common during cold winters in cold climates. The potential causes of the problem may be poorly functioning heating systems or improperly vented fuel-burning devices (e.g., kerosene heater). Clinical symptoms of CO poisoning include throbbing headaches, nausea, malaise and dizziness. Severe poisoning may result in seizures, syncope and coma. Delayed neuropsychiatric syndrome develops in up to 40% of patients with significant CO poisoning. The diagnosis is confirmed by carboxyhemoglobin level measurement. (Choicees A and B) Other diagnoses such as subarachnoid hemorrhage and acute meningitis are unlikely in this case. (Choice C) CBC with differential would not assist in the differential diagnosis of this case. (Choice E) Acetaminophen intoxication may initially manifest as nausea and vomiting, but eventually leads to liver damage. The absence of acetaminophen intake in the history also makes the diagnosis unlikely. Educational Objective: Clinical symptoms of CO poisoning include throbbing headache, nausea, malaise and dizziness. Several people simultaneously presenting with a headache is an important clue. The diagnosis is confirmed by carboxyhemoglobin level measurement. Case 63 A randomized, double-blinded clinical trial was conducted to assess the role of multidrug chemotherapy in the treatment of patients with stage III and IV stomach cancer. 150 patients in the treatment group and 100 patients in the placebo group were followed for 24 months. 120 patients in the treatment group (80%) and 80 patients in the placebo group (80%) died during the follow-up period. The investigators conclude that the treatment is effective. Which of the following is the most likely explanation of such a conclusion? A. Observers bias may be present B. Selective survival may be an issue C. The results are confounded D. Time-to-event data were analyzed E. Two-year risk was calculated Explanation: Time-to-event data analysis is becoming more popular in the analysis of follow-up studies and clinical trials. This type of analysis is called survival analysis. It accounts not only for the number of events in both groups, but also for the timing of the events. (Choice E) Interestingly, patients in the treatment group may live longer than the patients in the placebo group despite the fact that a two-year mortality risk is the same for both groups. For example, the median survival time may be three months for the placebo group and nine months in

the treatment group; therefore, time-to-event analysis in the study presented may explain the conclusion that the treatment group did better. (Choice A) Observers bias is reduced by the double-blinding technique. (Choice C) Randomization is an effective tool to control for confounding. (Choice B) Selective survival is not a likely explanation of the conclusion given by the investigators. Educational Objective: Survival analysis is used to analyze follow-up studies and clinical trials. It accounts not only for the number of events in both groups, but also for the timing of the events. Case 64 The following Vignette applies to the next 2 items A 50-year-old male comes in to the emergency department complaining of unremitting pain in an area of his right buttock near the anus. Although he is uncertain when the discomfort first began, he notes that the pain has increased significantly over the past 24 hours. He finds he is unable to pass stool because of the pain. He has not experienced symptoms like this before. Ten years ago, he was diagnosed with diabetes mellitus. He is currently on insulin. He smokes half a pack of cigarettes per day but would like to quit. He denies drinking alcohol or using recreational drugs. His temperature is 38.1C (100.6F), blood pressure is 128/86 mm Hg, and pulse is 90/min. There is a hot, tender, mass located between the anus and the left ischial tuberosity. The mass fluctuates upon palpation.

Item 1 of 2 What is the most appropriate next step in the management of this patient? A. Increase the insulin dose and discharge home B. Prescribe bupropion and discharge home C. Prescribe oral antibiotics and discharge home D. Admit to the hospital and start intravenous antibiotics E. Incise and drain the mass Explanation: Anal abscesses arise when one or more of the several glands that encircle the anus become blocked and the bacteria within grow unchecked. Patients with anal abscesses typically present with severe, constant pain that may be accompanied by fever or malaise. Physical examination commonly reveals erythematous, indurated skin or a fluctuant mass over the perianal or ischiorectal space. Purulent material may be seen if the abscess has begun to drain spontaneously. It is widely agreed that the most important aspect of treatment is prompt incision and drainage of the abscess (Choice E). Perianal and small ischiorectal abscesses are often drained in the office, but larger ischiorectal abscesses typically require surgical intervention. Increasing the insulin dose (Choice A) or prescribing bupropion (Choice B) are inappropriate because they do not treat the abscess. However, strict control of diabetes is very important in reducing the complications associated with wound healing. Oral antibiotics (Choice C) should also be prescribed to those patients who have diabetes mellitus, immunosuppression, extensive cellulitis, or valvular heart disease. However, antibiotics are an adjunct to and not a substitute for incision and drainage of the abscess. Admission to the hospital for intravenous antibiotics (Choice D) is unnecessary in a patient with localized infection.

Educational Objective: Anal abscesses should be treated with prompt incision and drainage. Antibiotics should also be prescribed in those patients who have diabetes mellitus, immunosuppression, extensive cellulitis, or valvular heart disease. Item 2 of 2 The patient is treated appropriately. To minimize risk of complications, he is advised to promptly schedule a follow-up visit with his primary care physician. Patients with anal abscesses are at greatest risk of developing which of the following conditions? A. Soft tissue infection B. Fistula C. Fecal impaction D. Incontinence E. Hemorrhoids Explanation: While an anal abscess is an infection within one or more of the anal spaces, an anal fistula (Choice B) is a tunneling between the anus or rectum and another epithelial lined space (eg, the skin overlying the drainage site). Fifty percent of patients with anal abscesses will go on to develop a chronic fistula from the involved anal gland to the overlying skin. Patients with fistulas typically present with an anal abscess that persists after incision and drainage, or with a pustule-like lesion in the perianal or ischiorectal area that continually drains. Surgical repair is usually necessary to eliminate the fistula while preserving fecal continence. Soft tissue infection (Choice A) is rarely a concern when the initial mass has been properly incised and drained. Fecal impaction (Choice C) could be of concern in a patient with an anal abscess who had been unable to pass stool for a long period before presenting for treatment. It does not commonly occur subsequent to treatment of anal abscess, however. Incontinence (Choice D) and hemorrhoids (Choice E) are not commonly associated with anal abscesses. Educational Objective: Fifty percent of patients with anal abscesses will go on to develop a chronic fistula from the involved anal gland to the overlying skin. Such fistulas require surgical repair. Case 66 The following vignette applies to the next 2 items A 45-year-old Caucasian man is brought to the emergency department by his wife. She states that he has been having fever, loss of appetite, nausea, vomiting, abdominal pain with distention, and hallucinations. He has chronic hepatitis C; he was diagnosed eight years ago. Treatment with ribavirin and pegylated interferon failed two years ago. He does not drink alcohol. His temperature is 38.3 C (101 F), blood pressure is 100/50 mmHg, pulse is 108/min, and respirations are 20/min. He is currently awake, but irritable. Physical examination reveals dry, icteric mucus membranes and clear lung fields. His abdomen is tender and distended; bowel sounds are present; shifting dullness is observed. There is no rebound tenderness and hepatomegaly. Fecal occult blood is negative. There are no focal motor or sensory deficits. Mild intention tremors are present. The patients laboratory tests reveal the following: CBC Hb 8.8 g/dL Ht 26% Platelet count 80,000/cmm Leukocyte count 8,000/cmm Segmented neutrophils 80%

Bands 3% Lymphocytes 17% LFT Total bilirubin 5.2 mg/dL Direct bilirubin 2.5 mg/dL Alkaline phosphatase 520 U/L Aspartate aminotransferase 98 U/L Alanine aminotransferase 85 U/L Albumin 2.8 g/dL Coagulation Profile: Prothrombin time 20 sec PTT 38 sec

A paracentesis is performed. The results are the following: Leukocytes 900/mm3 Neutrophils 65% RBC 3/cmm Item 1 of 2 Which of the following is the most appropriate pharmacotherapy? A. Therapeutic paracentesis B. Blood transfusion and antibiotics C. Mannitol and lactulose D. Peritoneo-venous shunt E. Lactulose and antibiotics Explanation: This patient is developing hepatic encephalopathy (HE). The presence of tremors is suggestive of stage 2 HE. This is most likely due to spontaneous bacterial peritonitis (SBP), which is diagnosed when there are more than 250 neutrophils/mm3 in the ascitic fluid of a patient with nephrotic syndrome or cirrhosis, as in this case. Management involves the administration of lactulose. Cultures are not always positive in this condition; however, antibiotic therapy must not be delayed while the results are pending. (Choice C) Mannitol is used with lactulose only in the more advanced stages of HE (Choices A and D) Repeated paracentesis and peritoneo-venous shunt are treatments for refractory ascites, not for SBP. (Choice B) Blood transfusion is not indicated now, as there is no evidence of ongoing blood loss. Blood transfusion is usually reserved for patients with hemoglobin levels less than 8 g/dL. Educational Objective: SBP is a serious condition and is diagnosed when > 250 neutrophils/mm3 is found in the peritoneal fluid. Empiric antibiotic therapy must be started immediately while waiting for the culture results, especially if there are other clinical indicators of infection. Item 2 of 2 The patient responds well to the treatment. He is currently less agitated and more oriented. He is discharged from the hospital. Four weeks later, he returns with his wife for a follow-up visit. His new laboratory tests show the following: Total bilirubin 2.8 mg/dL Direct bilirubin 1.3 mg/dL Alkaline phosphatase 380 U/L Aspartate aminotransferase 78 U/L Alanine aminotransferase 43 U/L Albumin 2.6 g/dL Prothrombin time 19 sec The patients wife is very concerned and asks about his prognosis. Which of the following is the most appropriate response?

A. His prognosis is good B. He has a very high risk of hepatocellular carcinoma C. He needs a liver transplant D. There is nothing much we can do E. We have to observe him for a couple of months for an accurate prognosis Explanation: The patient has possible cirrhosis secondary to chronic hepatitis-C infection. His prothrombin time remains prolonged, hepatic enzymes are mildly elevated and AST is higher than ALT, which points towards profound liver damage. In addition, he has hyperbilirubinemia and hypoalbuminemia. With the laboratory data available, the Child-Pugh score of liver damage can be calculated. Child-Turcotte-Pugh (CTP) score components are used to select the candidates for liver transplantation. Points 1 2 3 . For cholestatic disorders the bilirubin categories are < 4, 4-10, and > 10 Class A: CTP score 6 Class B: CTP score 7-9 Class C: CTP score > 10 Liver transplantation should be considered in all cirrhotic patients with CTP > 7 (Choice A) The patient's prognosis is not good. He needs a liver transplant. (Choice E) Because the acute event has been controlled, the score can be calculated and a prognosis given without an observation period. (Choice D) His disease is not terminal at this point. (Choice B) Studies have shown that approximately 7% (over a period of five years) of patients with hepatitis-C and cirrhosis die from hepatocellular cancer. Furthermore, sharing this information rather than advising the patient to undergo a liver transplantation is inappropriate. Educational Objective: A clinical and biochemical evaluation of a patient with cirrhosis and chronic hepatitis is important to determine if he/she is a candidate for surgery. Liver transplantation is a treatment option when the disease has progressed and there is no other treatment available, and when the patient is young and is not currently using alcohol. Case 67 A 23-year-old Caucasian college student comes to the student health center for the evaluation of lowgrade fever, malaise, and dry cough for the past two weeks. She also complains about a recurrent dry and itchy rash over her face and axilla. She has had recent problems with maintaining her weight. She has lost 15 pounds in the past two months, despite her attempts to gain weight to stay in the college weightlifting team. She admits that she has been under a lot of stress recently, and was working three jobs at one time to keep up with her finances. Which of the following is the most important in the medical history, and should be obtained in detail to help diagnose this patient? A. The episodes will diminish with age, but treatment is not usually effective. B. Sexual history C. History of allergies Bilirubin (mg/dl) Encephalopathy <2 2-3 Stage 1-2 >3 Ascites Absent Slight/responsive Moderate-severe Prothrombin time <15 sec 15-17 sec Albumin >3.5 2.8-3.5 <2.8 (g%) None Stage 3-5

>17 sec

D. Family history E. We have to observe him for a couple of months for an accurate prognosis Explanation: The index of suspicion for HIV infection should be high in any young patient who presents with nonspecific or vague symptoms and a significant, unintentional weight loss in the past few weeks. This patient should be screened for risk factors for HIV infection, which includes a history of sexual exposure and the use of intravenous drugs in the past. A detailed sexual history must be obtained, and this should include the patient's sexual orientation and past history of unprotected sexual intercourse. The physical examination should also be focused, and the search for clues of HIV infection or Acquired Immunodeficiency Syndrome (AIDS) must be exhaustive. New-onset seborrheic dermatitis (erythematous and pruritic rash on the face and axilla) is commonly seen in HIV-infected patients, and can be one of the presenting complaints of the disease. (Choices A, C, and D) The occupational history, family history, and previous history of allergies are not as important as a detailed sexual history in this patient. Educational Objective: Obtain a detailed sexual history in all young patients who present with weight loss and nonspecific complaints. Case 68 A 4-year-old Caucasian boy is brought to your office by his mother because of frequent staring spells that last about 10-20 seconds. His past medical history is significant for three episodes of otitis media and one episode of severe diarrhea that required hospitalization. The neurologic examination is normal. A staring spell is provoked at the EEG lab, and a generalized 3/second spike and wave activity is observed. No myoclonic activity is noted. The mother is concerned about the childs condition, and asks you about the treatment and prognosis. Which of the following is the best response to her concern? A. The episodes will diminish with age, but treatment is not usually effective. B. The episodes will diminish with age, and treatment is usually effective. C. The episodes will not change with age, but treatment is usually effective. D. The episodes will not change with age, and treatment is not usually effective. E. The episodes will increase with age, and treatment is not usually effective. Explanation: The clinical scenario described is consistent with childhood absence epilepsy (CAE): age of onset (4-8 years), no neurologic signs, typical EEG pattern, and no myoclonic activity. This condition is usually responsive to ethosuximide or valproate, although high doses may be required to control it effectively. Interestingly, many traditional anti-epileptic drugs are not effective in absence epilepsy, and can even exacerbate the condition (e.g. gabapentin). The prognosis in patients with CAE, especially if generalized tonic-clonic seizures are absent, is good. Staring spells will disappear in the teenage years. The risk of persistence of the condition is higher in patients who develop generalized tonicclonic seizures. Unlike CAE, juvenile myoclonic epilepsy (JME) is characterized by a late onset of absence seizures with myoclonic activity, and is associated with life-long seizures. Educational Objective: The prognosis in patients with childhood absence epilepsy, especially if generalized tonic-clonic seizures are absent, is good. Case 69 A 42-year-old Caucasian woman comes to the emergency department because of right-sided neck pain, persistent cough, and dyspnea for the past three days. Her past medical problems include hypertension and obesity. She takes enalapril and hydrochlorothiazide. She has smoked one pack of cigarettes daily for the past 25 years. She does not drink alcohol. She works as an executive secretary. Her temperature is 37.8C (100 F), blood pressure is 130/80 mm Hg, pulse is 92/min and

respirations are 22/min. Examination shows decreased breath sounds in the upper part of the right lung. There is moderate swelling and erythema of the neck, moderate right jugular vein dilatation, and increased venous marks in the right arm. A chest x-ray reveals a right apical lung mass. Which of the following is the most appropriate immediate step in the management of this patient? A. Start broad-spectrum antibiotic therapy B. Order a neck and chest computerized tomography (CT) scan with contrast C. Prepare for endotracheal intubation D. Order thyroid tests, CBC and chemistry panel, and reevaluate with the results E. Order magnetic resonance imaging (MRI) of the neck and chest Explanation: The patients history, physical findings, and radiologic findings suggest superior vena cava syndrome (SVCS) secondary to a lung malignancy. At least 80% of the cases of SVCS are due to bronchogenic carcinoma. The classic presentation of SVCS begins with dyspnea, persistent cough, fascial fullness and neck pain, and progresses into hoarseness, dysphagia, chest pain and syncope. Pertinent physical findings are edema and erythema of the neck (which may sometimes compromise the face), and dilated veins of the arms and neck. Advanced disease is manifested by cyanosis, collateral veins in the thorax, ocular proptosis and lingual edema. The best diagnostic test for SVCS is a contrast CT scan of the chest and neck, which will reveal an obstruction of the superior vena cave due to the pulmonary mass, the metastatic nodes, or an intravenous thrombus. CT scan is very useful because it can reveal the extent of obstruction and provide a histopathologic diagnosis (via percutaneous biopsy), which will determine the particular therapeutic regimen required to manage the underlying malignancy. (Choice E) Magnetic resonance imaging (MRI) is a more expensive procedure, and does not offer any visual advantage over computed tomography. It is only used when a patient is allergic to the contrast dye used in CT scan, or when venous access cannot be obtained for contrast-enhanced studies. (Choice C) Immediate endotracheal intubation may be performed if there exists a risk of sudden airway occlusion, such as in patients with mediastinitis and deep neck infection; however, this patient does not have a history of esophageal disease, upper respiratory tract infection, fever, or chills, to make us suspect mediastinitis. (Choice A) The erythema of the patient is due to venous occlusion and possible thrombosis, not to facial or neck cellulitis. Antibiotic therapy may be subsequently needed if the patient is found to have an infection; however, it is not necessary at this point. (Choice D) Biochemical and hematological tests may be obtained after CT scan evaluation. Educational Objective: Superior vena cava syndrome must be suspected in any patient who has a high risk of malignancy (e.g., family history or smokers) and presents with dyspnea, orthopnea, neck pain and swelling, and has cervical and upper extremities venous dilatation. The best diagnostic test to perform is computerized tomography of the neck and chest. 63% of people answered this question correctly.

Case 70 The following vignette applies to the next 2 items A 24-year-old Italian man is brought to the emergency department by his girlfriend for the evaluation of difficulty in breathing. He has been vacationing in the United States along with his girlfriend for the past two months. His shortness of breath started four days ago and is progressively getting worse. He has to pause several times during your interview to breathe. His girlfriend tells you that he had suffered from an episode of gastroenteritis two weeks ago. He also had bloody bowel moments at that time. The diarrhea resolved spontaneously, but he felt very weak and fatigued afterwards. One week later, he felt a fine tingling sensation in his toes and feet. Three days later, he was extremely weak and could not get up from his bed and started to have some difficulty in breathing. The breathing difficulty got much worse when he woke up this morning, and he decided to come to the emergency room. He has no significant past medical history or family history. He denies any history of intravenous drug abuse, alcohol or tobacco use, or risk factors for HIV disease. His temperature is 37.6C (99.8F), blood pressure is 142/62 mmHg, respirations are 28/min, and heart rate is 96/min. His lungs and cardiovascular examination are unremarkable. His neurological examination reveals a flaccid paralysis of the bilateral lower extremities. Deep tendon reflexes are also absent bilaterally. The rest of the neurological examination, including a detailed cranial nerve examination, is normal. Item 1 of 2 What is the next best step in the management of this patient? A. Start intravenous antibiotics B. Send blood cultures C. Give botulinum antitoxin D. Provide respiratory support E. Obtain stat serum chemistry and electrolytes Explanation: The patient in the above vignette has a classic presentation of acute Guillain-Barre syndrome (GBS), which is an acute inflammatory demyelinating ascending polyneuropathy characterized by progressive flaccid paralysis and areflexia. The pathogenesis is immunologically mediated, with approximately 2/3 of the patients presenting with a history of upper respiratory or gastrointestinal infection in the proceeding 2 to 4 weeks (Campylobacter jejuni, cytomegalovirus, Epstein-Barr virus, or herpes simplex viruses infections). The patient usually presents with progressive lower extremity weakness that may ascend rapidly over days to involve the upper extremities, cranial nerves and eventually, the muscles of respiration. The physical examination reveals bilateral symmetric weakness or flaccid paralysis and absent deep tendon reflexes in the extremities. Approximately 25 to 30% of patients with GBS will eventually develop neuromuscular respiratory failure and need respiratory support with mechanical ventilation. All patients with suspected Guillain-Barre syndrome and clinical signs and symptoms of progressive respiratory compromise should be hospitalized immediately and closely monitored for signs of respiratory failure. This includes continuous pulse oximetry monitoring, bedside spirometry, arterial blood gas measurement and frequent clinical evaluation for signs of respiratory muscle fatigue. Measurements of bedside vital capacity provide the most useful information regarding the degree of respiratory impairment. Respiratory failure is an important and common cause of death in these patients. Urgent intubation and mechanical ventilation should be considered for the patients with rapidly progressive respiratory compromise to decrease the risk of complication and mortality. Most patients with GBS have spontaneous remission; they either recover completely or have only minor neurological deficits. Some of the factors associated with a poor outcome include older age, rapidly progressive disease, prolonged mechanical ventilation, and the presence of associated comorbidity. (Choices A and B) Blood culture and intravenous antibiotics have no role in the management of patients with Guillain-Barre syndrome.

(Choice C) Therapy with botulinum antitoxin is indicated in patients with progressive symptoms of suspected botulism. These patients classically present with an acute onset of bilateral cranial nerve neuropathies with symmetric descending flaccid paralysis. (Choice E) There is no indication for obtaining urgent electrolytes and serum chemistry at this point. Educational Objective: Campylobacter jejuni infection is the most frequent precipitant of Guillain-Barre syndrome (GBS). All patients with suspected GBS should be hospitalized, regardless of the severity of the illness. Measurements of bedside vital capacity provide the most useful information regarding the degree of respiratory impairment. Patients with signs of respiratory failure should be promptly intubated and supported with mechanical ventilation until the resolution of respiratory muscle weakness. Item 2 of 2 What is the therapy of choice for the patient's condition? A. Plasmapheresis B. High dose intravenous steroids with a gradual taper C. Antitoxin alone is enough D. Antitoxin and immunoglobulin E. Clindamycin and Vancomycin Explanation: The first step in the management of patients with Guillain-Barre syndrome (GBS) is to triage the severity and provide appropriate respiratory support for patients with signs of respiratory failure. After the airway is secured, specific therapy should be instituted as early as possible. Plasmapheresis and/or administration of intravenous immunoglobulin (IVIG) are the mainstays of therapy for patients with severe and progressive GBS. The indications for plasmapheresis are: severe flaccid paralysis, bulbar palsy, progressive respiratory failure, and patients on mechanical ventilation. It is most effective when it is started within seven days of the onset of symptoms; however, some improvement still occurs even when it started late in the course of disease. Plasmapheresis is not usually indicated for ambulatory patients with mild or non-progressive disease. Daily administration of IVIG for two weeks may also be used to treat patients with GBS because it has been shown to be as effective as plasmapheresis. (Choices B and E) Antibiotics and corticosteroids have no role in the management of patients with GBS. (Choices C and D) Botulinum antitoxin is extremely useful in patients with progressive weakness and suspected botulism. It has no role in the management of patients with GBS. Educational Objective: Plasmapheresis or intravenous immunoglobulin is the mainstay of therapy for patients with severe and progressive GBS. Corticosteroids have no role in the management of patients with GBS 72% of people answered this question correctly. Case 71 The following vignette applies to the next 2 items A follow-up study was conducted to assess the role of different treatment regimens on the risk of cardiovascular outcomes. As a subset of the study, two treatment regimens were evaluated: highdose hydrochlorothiazide (100 mg) and low-dose hydrochlorothiazide (25 mg). According to the study results, the mean systolic blood pressure (BP) in the high-dose group was 139 mmHg, and the mean diastolic BP was 88 mmHg; in the low-dose group, the mean values were 143 mmHg and 92 mmHg, respectively. A two-sample t test gave a p value of 0.03 for systolic BP and 0.04 for diastolic blood

pressure differences between the two groups. The relative risk (RR) of sudden cardiac death comparing the low-dose group to the high-dose group was 0.4 (95% Confidence Interval 0.25 - 0.55) Item 1 of 2 Which of the following is the best statement concerning the results of the study? A. High-dose hydrochlorothiazide has no additional benefit in lowering BP. B. Low-dose hydrochlorothiazide reduces the risk of sudden death in the cohort by 40%. C. High-dose hydrochlorothiazide doubles the risk of sudden death in the cohort. D. The risk of sudden death decreases by 60% with a lower dose of hydrochlorothiazide. E. The dose of the drug is not significantly related to the risk of sudden death. Explanation: The scenario described emphasizes the pitfalls in interpreting the measures of effect. Be very careful in determining which groups are being compared, as the resulting interpretation will differ strikingly. Relative risk is a measure of effect that can be calculated in follow-up studies. It compares the risks between two groups; in this case, a low-dose group is compared to a high-dose group. Note that in this computation, the high-dose hydrochlorothiazide group is set as a reference group! Compared to high-dose hydrochlorothiazide treatment, low-dose hydrochlorothiazide treatment appears to protect from the occurrence of sudden death. The relative risk of 0.4 therefore indicates a protective effect of a lower dose of the drug, which reduced the risk of sudden death by 60%. (Choice E) The effect is statistically significant because the confidence interval does not contain the null value (1.0). (Choices B and C) The results presented have no information on the baseline risk of sudden death in the cohort; therefore, no conclusions about risk reduction in the cohort (no treatment) would be valid. (Choice A) High-dose hydrochlorothiazide treatment seems to be more effective in reducing BP than the low-dose regimen (Note the p values for the two-sample t test). Educational Objective: During the interpretation of the measures of effect, be careful in determining which groups are being compared, as the resulting interpretation will differ strikingly. Item 2 of 2 Another subset of patients in this cohort were given placebo, and their risk of sudden death was assessed. This group's RR of sudden death is very close to 1.0 when compared to the low-dose hydrochlorothiazide group, although the mean systolic and diastolic BP are higher when compared to the low-dose hydrochlorothiazide group. Which of the following is the best statement concerning the risk of sudden death in this cohort? A. The risk of sudden death in the cohort is high. B. High-dose hydrochlorothiazide increases the risk of sudden death approximately 2.5 times. C. High-dose hydrochlorothiazide increases the risk of sudden death by 60%. D. Low-dose hydrochlorothiazide decreases the risk of sudden death almost 3 times. E. Hydrochlorothiazide is a safe drug regardless of the dose. Explanation: The scenario assesses your understanding of risk and relative risk (RR) concepts. It mentions that the RR of sudden death when comparing low-dose hydrochlorothiazide treatment to placebo is close to 1 (null value). This means that the patients taking a low dose of the drug have an almost equal risk of sudden death with the patients taking placebo. On the other hand, the RR of sudden death in the lowdose drug group is 0.4, when compared to the high-dose group. To switch the groups when computing for the RR, the inverse of the estimate is obtained. For instance, the risk of sudden death for the highdose group is 2.5 times higher, when compared to the low-dose group and placebo group (2.5 is the inverse value of 0.4).

RR (low-dose group vs. high-dose group) = 0.4 RR (high-dose group vs. low-dose group) = 1/0.4 = 2.5 RR (low-dose group vs. placebo) = 1.0 Summarizing the results of the study, high-dose hydrochlorothiazide treatment seems to increase the risk of sudden death by 2.5 times, when compared to low-dose treatment and placebo. Educational Objective: Switching the groups when computing for the RR means obtaining the inverse of the estimate

Case 72 A 54-year-old Caucasian male is hospitalized for an episode of life-threatening angioedema. He was treated with corticosteroids and fresh frozen plasma infusion. He never had such an episode before. His past medical history is significant for diabetes diagnosed two years ago and hypertension diagnosed two weeks ago. His family history is significant for diabetes mellitus and stroke in his mother, and prostate cancer in his father. His blood pressure is 150/95 mmHg and heart rate is 80/min. Which of the following is the best treatment option for hypertension in this patient? A. Captopril B. Clonidine C. Terazosin D. Losartan E. Metoprolol Explanation: The episode of angioedema is most likely related to the treatment of recently diagnosed hypertension with an ACE inhibitor. Although the exact mechanism of ACE inhibitor-induced angioedema is unclear, it is likely that both kinin-dependent and kinin-independent mechanisms have a role. The incidence of this complication is 0.2 to 0.7%. Selective beta-blockers have been shown to be well-tolerated and equally beneficial in diabetic patients as in non-diabetics; therefore, metoprolol is the most appropriate treatment option in this patient. (Choice A) Re-trial of an ACE inhibitor is not recommended, although switching to an angiotensin receptor blocker (ARB) may be considered in selected patients. (Choice D) Many cases of angioedema have been described with ARBs including losartan, although the incidence is much lower than with ACE inhibitors. This patient experienced a life-threatening episode of angioedema, and has no compelling indication for ARB therapy (e.g., heart failure, diabetic nephropathy). (Choices B and C) Clonidine and terazosin are not considered first-line agents for the treatment of hypertension. Educational Objective: ARBs can be used very cautiously in selected patients with ACE inhibitor-induced angioedema; however, it is best to use alternative drugs, if possible.

15% of people answered this question correctly. Case 73 The following vignette applies to the next 4 items A 24-year-old female is admitted to the hospital after a motor vehicle accident, where she sustained a right humerus fracture, multiple rib fractures and a contusion over her right lower leg. The next morning, a nurse calls to inform you that she is complaining of severe pain in her right lower leg. Her temperature is 37.2 C (99 F), heart rate is 96/min, respiratory rate is 18/min, and blood pressure is 140/82 mmHg. Physical examination reveals a tense swelling around the right calf region. The pain is worsened on palpation and passive movements of the foot. Neurological examination reveals motor weakness and hypoesthesia of the distal right leg. Item 1 of 4 Which of the following is the most likely diagnosis? A. Fat embolism B. Deep venous thrombosis C. Acute compartment syndrome D. Acute vascular occlusion E. Neural compression Explanation: Acute compartment syndrome refers to ischemic tissue damage secondary to elevated pressures in the enclosed compartments of the lower legs or forearm. When the tissue pressure in an enclosed compartment exceeds the perfusion pressure, the resulting diminished tissue perfusion and compromised blood flow to the muscles and nerves inevitably lead to ischemic tissue necrosis. Majority of the cases involving the lower extremities are due to a traumatic event, most commonly tibial fractures. Other causes include a crush injury or other long bone fractures in a motor vehicle accident, a tight cast or dressing after trauma, and drug overdose. Patients usually present with severe pain which is out of proportion to the extent of injury. The pain is typically worsened by passive movements of the involved muscles. Sensory nerves are usually affected earlier than the motor nerves, and the neurologic deficit presents as decreased vibration sense, decreased two-point discrimination, numbness or hypoesthesia. Late features include extremity paralysis and absent distal pulsation (pulseless paralysis). (Choice A) Fat embolism is infrequently seen in patients with long bone or pelvic fractures. Patients usually present with a triad of hypoxemia, neurological abnormalities (i.e., confusion), and a petechial rash (involving the head, neck, anterior chest wall, or axilla). (Choice B) Deep venous thrombosis usually does not present acutely in a setting of motor vehicle accident in an otherwise healthy young patient. It is a rare cause of compartment syndrome and does not compromise blood circulation and neuromuscular function. (Choice D) Vascular occlusion secondary to a motor vehicle accident usually presents more suddenly and dramatically. Acute compartment syndrome usually has a lag period of a few hours before irreversible nerve injury and muscle necrosis occurs. (Choice E) Nerve compression may occur in a patient after a motor vehicle accident and possible bone fracture; however, it does not lead to blood flow compromise and muscle necrosis (pain with passive movements of the involved muscles). Educational Objective: Acute compartment syndrome usually occurs after a traumatic event and causes pain, paresis, hypoesthesia, and diminished to absent pulses in the involved limb. 87% of people answered this question correctly.

Item 2 of 4 Which of the following is the most common life-threatening complication of the above condition? A. Disseminated intravascular coagulation B. Rhabdomyolysis and renal failure C. Pulmonary embolism D. Gangrene of the limb E. Thrombocytopenia Explanation: Acute compartment syndrome results in markedly diminished to absent tissue perfusion within hours of the inciting event, causing tissue necrosis, muscle infarction, and rhabdomyolysis, which releases myoglobin into the peripheral circulation. Myoglobin is directly toxic to the renal tubules, and subsequently causes acute tubular necrosis and acute renal failure. Acute renal failure and its complications (electrolyte disturbances) are one of the most common life-threatening complications of acute compartment syndrome. Laboratory studies typically reveal markedly elevated creatinine kinase levels and the presence of myoglobin in the urine (positive dipstick for blood in the absence of RBCs in the urine). (Choices A, C, D, E) Thrombocytopenia, disseminated intravascular coagulation, pulmonary embolism, and gangrene of the limb are not usually seen in patients with acute compartment syndrome.

Educational Objective: Rhabdomyolysis and subsequent development of acute renal failure is one of the most common and severe life-threatening complications of acute compartment syndrome. 24% of people answered this question correctly. Item 3 of 4 Which of the following is the most appropriate next step in the management of this patient? A. Administer oxygen B. Start the patient on anticoagulation C. Order venous Doppler ultrasonography D. Check the tissue pressure E. Order a nerve conduction study Explanation: Compartment syndrome is characterized by an increase in the tissue pressure in the enclosed myofascial compartments of the extremities. When the elevated tissue or compartment pressure reaches its threshold level, the capillaries collapse, and this eventually leads to tissue and muscle necrosis. The exact value for the tissue pressure at which blood flows to the muscle and nerve tissue stops is controversial. The current general consensus for the threshold value is greater than 30 mmHg. It is therefore important to measure the tissue or compartment pressure early in the course of management, especially if the diagnosis is in question. (Choice A) Oxygen is usually used in the supportive treatment of patients with fat or cholesterol emboli. (Choice B) Anticoagulation is usually required for patients with deep venous thrombosis. (Choice C) Venous Doppler ultrasonography is useful for the diagnosis of deep venous thrombosis. It is not helpful for the diagnosis of compartment syndrome. (Choice E) Nerve conduction studies are helpful in the diagnosis and localization of the site of nerve damage.

Educational Objective: Direct measurement of the compartment or tissue pressure is the diagnostic procedure of choice for patients with suspected acute compartment syndrome. 80% of people answered this question correctly. Item 4 of 4 The appropriate step was taken for the patient. Which of the following is the best next step in management? A. Continue with oxygen therapy B. Obtain a hypercoagulable panel C. Consult a vascular surgeon D. Perform urgent fasciotomy E. Review the results of venous Doppler ultrasonography Explanation: Acute compartment syndrome is a surgical emergency. Any delay in treatment leads to irreversible muscle and nerve damage. A compartment pressure of 30 mmHg or greater warrants an emergent fasciotomy (also known as compartment release). Surgical decompression aims to relieve the pressure within the enclosed compartment and to restore the blood flow to muscles and other tissues within 6-10 hours of the initial symptoms. Some patients may develop a persistent sensory or motor deficit after an episode of acute compartment syndrome despite early fasciotomy. (Choice A) Oxygen therapy has no role in the management of patients with acute compartment syndrome. (Choice B) Obtaining a hypercoagulable panel may be useful in patients with idiopathic deep venous thrombosis. It is not indicated in patients with acute compartment syndrome. (Choice C) A consultation with a vascular surgeon is necessary in patients with vascular injury or occlusion. (Choice E) Venous Doppler ultrasonography is useful to diagnose deep venous thrombosis. It has no role in the diagnosis or management of patients with acute compartment syndrome. Educational Objective: An emergent fasciotomy is the definitive treatment in patients with acute compartment syndrome. Surgical decompression aims to restore the capillary blood flow and tissue perfusion. Case 74 The following vignette applies to the next 3 items A six-month-old well-nourished African-American boy is brought to the pediatrician by his mother. She is concerned because on multiple occasions in the past 48 hours, she has discovered evidence of rectal bleeding in her sons diaper. The mother reports that her son has no history of vomiting, excessive crying, or demonstration of discomfort when passing stool. She denies any history of trauma or foreign body ingestion. The boy was born at term via normal spontaneous vaginal delivery and has no pertinent medical history. He is now sitting very quietly in his mothers lap, and appears to be experiencing mild discomfort. His temperature is 37.0C (98.6F), blood pressure is 84/56 mm Hg, pulse is 132/min, and respirations are 28/min. His anterior fontanelle is slightly depressed. His abdomen is soft and nontender. There is no hepatosplenomegaly. The rectal examination is unremarkable. Immediately after being examined, the child passes stool that looks like currant jelly, and contains gross amounts of blood and mucous. Item 1 of 3

Which of the following diagnostic tests would be most useful in the management of this patients condition? A. Abdominal plain films B. Barium enema C. CT scan of abdomen D. Technetium 99m scan E. Ultrasound of abdomen Explanation: The second most common cause of small bowel obstruction in infants and toddlers, intussusception is the telescoping of a proximal portion of intestine into a more distal portion of intestine. The resulting venous and lymphatic congestion can cause significant intestinal edema and may ultimately result in ischemia, perforation, and peritonitis. The most common presentation involves intermittent, severe, crampy abdominal pain accompanied by intense crying and vomiting. A sausage-shaped mass may be palpated on the right side of the abdomen. Interestingly, the classic triad of pain, a palpable mass, and "currant-jelly" stool (containing blood and mucous) is observed in less than 15% of patients, and up to 20% of patients may have no evident pain. The diagnosis is most commonly confirmed with a barium enema contrast study. This study is preferred because it is both diagnostic and therapeutic. Reduction of the intussusception through this method is successful 75-90% of the time. (Choice A) Abdominal plain films occasionally demonstrate a soft-tissue density extending into the gaseous pockets within the large bowel, a finding known as the "crescent sign." Frank intestinal obstruction and pneumoperitoneum may also be found. (Choice C) CT scans of the abdomen may allow for the diagnosis of intussusception, but are less preferred because they require sedation of young children and are time-consuming. (Choice D) A Meckel scan with technetium 99m is often used to highlight ectopic gastric mucosa. It is not commonly used in the diagnosis of intussusception. (Choice E) Ultrasonography is also an excellent choice as its sensitivity and specificity near 100%. The classic image seen on ultrasound is that of a "bull?s eye," signifying layers of intestine nestled within intestine. Although there have been some reports of reducing the intussusception under ultrasound guidance, this is an unusual and not well-studied technique; therefore, barium enema remains as the preferred means of diagnosis and therapy. Educational Objective: The barium enema (not barium swallow) contrast is a preferred means of diagnosing intussusception because it is also therapeutic; reduction of the intussusception through this method is successful 7590% of the time. Ultrasonography is also an excellent choice as its sensitivity and specificity near 100%. * If you dont find barium enema in the choices choose ultrasonography. *Extremely important question for USMLE step-3 Item 2 of 3 The appropriate diagnostic test is selected and the mother agrees to the proposed treatment. The boys father is not present. Which of the following is the most appropriate next step? A. Document the mothers informed consent and proceed with diagnostic testing and treatment B. Have the father brought in for written consent C. Have the father telephoned for oral consent D. Have another adult relative brought in for written consent E. Request a nurse to witness the informed consent

Explanation: Obtaining informed consent formally ensures that the patient is an informed participant in his health care decisions. To properly equip the patient with adequate knowledge to make such decisions, several concepts must be conveyed, including the nature, alternatives, risks, and benefits of the procedure. For a patients consent to be deemed valid, he must be considered competent, and his consent must be voluntary. The same standards apply if the patient is a minor, with the exception that the custodial parent or legal guardian retains the legal authority to provide consent. It is generally considered sufficient to document the informed consent of one parent and then proceed with diagnostic testing and treatment. (Choices B and C) It is not necessary to contact the childs father for written or oral consent, unless the mother does not have the legal authority to act on the childs behalf. (Choice D) Contacting another adult relative to provide consent is only appropriate if that individual has the legal authority to act on the childs behalf while the mother does not. (Choice E) If the procedure is a simple one (eg, a blood draw), it may be appropriate to have the nurse obtain informed consent from the patients mother. However, more complex procedures warrant an in-depth discussion between the physician and patient. Once such a discussion occurs, the physician should document the consent and include it in the medical chart. It is rarely necessary to have support staff witness the informed consent if the physician is already present. Educational Objective: When providing health care to minors, it is generally sufficient to document the informed consent of one parent who has legal custody of the child and then proceed with diagnostic testing and treatment. 88% of people answered this question correctly. Item 3 of 3 The risks and benefits of the preferred curative procedure are discussed, and the childs mother asks about potential complications. Which of the following is most likely? A. Adhesions B. Fistula C. Perforation D. Renal failure E. Stricture Explanation: The complication of greatest concern when reducing an intussusception with a barium or air contrast study is perforation of the bowel, which occurs in <1% of patients (Choice C). The perforation usually occurs on the distal aspect of the intussusception. The risk of this complication occurring is greatest in two groups: infants younger than six months old who have demonstrated symptoms for at least three days and those patients who appear to have small bowel obstruction. The risk of small bowel obstruction secondary to postoperative adhesions (Choice A) is of concern after any abdominal surgery and is thought to occur in up to 5% of patients who undergo operative reduction of an intussusception. It is not a likely complication if the reduction is accomplished with barium contrast enema. Fistulas (Choice B) are not common complications of intussusception reduction, whether accomplished operatively or nonoperatively. Renal failure (Choice D) is of greatest concern when performing radiological studies that use intravenous contrast. It is unlikely to occur secondary to usage of barium contrast enemas. Stricture (Choice E) occurring 4-8 weeks after the reduction -- whether accomplished by operative or nonoperative means -- has been known to occur in the portion of intestine involved. However, this is a rare occurrence.

Educational Objective: The complication of greatest concern when reducing an intussusception with a barium or air contrast study is perforation of the bowel, which occurs in <1% of patients. Case 75 A 45-year-old immigrant from Eastern Europe is brought to the emergency department because of several weeks history of progressive back pain. His past medical history is insignificant. He is not taking any medications and denies drug abuse. The patients work-up reveals bilateral cavitary lesions in the pulmonary apices. Sputum analysis is positive for acid-fast bacilli. Immediately after obtaining this information, the patient was isolated, and the three staff members that were in close contact with him are tested with PPD. PPD test results turn out to be negative in all of them. Which of the following is the best next step in infection prevention among the exposed staff? A. Repeat PPD test in three months. B. Perform chest X-ray. C. Start treatment with isoniazid (INH). D. Start treatment with at least three drugs. E. No further measures are necessary. Explanation: If tuberculosis precautions were not promptly instituted upon admission of a patient with an active infection, an investigation of the people at risk should be undertaken, and appropriate preventive measures should be initiated. Identifying health care workers and other individuals who were exposed is very important. PPD testing should be promptly performed on exposed individuals. If the results are negative, the PPD test should be repeated in three months. (Choice C) If PPD conversion occurs, monotherapy with INH is necessary. (Choices B and D) In people with positive PPD test results, a chest x-ray should be taken to check for signs of infection; if positive, anti-tuberculosis treatment should be instituted. Public health authorities should be notified if active infectious tuberculosis is revealed. Educational Objective: PPD testing should be promptly performed on individuals exposed to active infectious tuberculosis. If the results are negative, the PPD test should be repeated in three months. Case 76 A 25-year-old Caucasian male presents to your office for pre-travel counseling. He plans to fly his private airplane and engage in scuba diving in the Caribbean islands. His past medical history is significant for allergic rhinitis since childhood, and he takes inhalational steroids as needed to treat it. He does not smoke or consume alcohol, and denies any recreational drug use. He regularly works out at a gym twice a week. All his vaccinations are up-to-date. The physical examination is within normal limits. Which of the following is the best statement concerning this patients recreational activities? A. He should use steroids before diving if he has symptoms of allergy. B. Antibiotics can be used to prevent diving-associated medical conditions. C. Decongestants (e.g., pseudoephedrine) can be used before diving. D. Consider pulmonary function tests before diving activities. E. Consider otoscopic examination and audiometry before diving. Explanation: Scuba diving is associated with a range of pressure-related injuries. It is important to inform the patient about the possibility of such injuries, and to educate him about the basic measures that help prevent them (e.g., discourage flying in a plane within 24 hours of diving). The most common condition associated with scuba diving is ear barotrauma. This patient has an increased susceptibility

to ear and sinus barotraumas because of his history of allergic rhinitis, which can block drainage due to mucosal edema and discharge. Studies have shown that the use of non-sedating decongestants (specifically pseudoephedrine) before diving reduces the incidence of ear and sinus barotraumas by 75%. It is therefore reasonable to suggest the use of such drugs to the patient. (Choice A) The patient should be instructed to avoid diving activities if he has any symptoms of allergy. (Choice B) Antibiotic prophylaxis is not recommended for scuba divers. (Choices D and E) There is no indication to perform pulmonary function tests or audiometry in this patient. Educational Objective: Studies have shown that the use of non-sedating decongestants (specifically pseudoephedrine) before diving reduces the incidence of ear and sinus barotraumas by 75%. 20% of people answered this question correctly. Case 77 A male infant is born at term to a 30-year-old Caucasian primigravida. The delivery was complicated by mild shoulder dystocia, and the infant demonstrates tachypnea, cyanosis, and a weak cry shortly after birth. The prenatal course was complicated by two episodes of mild viral infection during the second trimester. The mother has had diabetes mellitus, type-1 since she was 11 years of age. Physical examination reveals decreased movement of the rib cage on the right side and abdominal flattening. Which of the following favors most the diagnosis of diaphragmatic paralysis over diaphragmatic hernia in this case? A. Hypoplastic right lung B. Mediastinal shift to the left C. Erbs palsy D. Sacral agenesis E. Spina bifida Explanation: Diaphragmatic paralysis of the newborn usually results from a phrenic nerve injury. The two most common causes of phrenic nerve injury are birth injury and cardiothoracic injury. Shoulder traction during delivery is the most probable cause of diaphragmatic paralysis in this case. It is typically accompanied by the signs of brachial plexus injury, such as Erbs palsy. (Choice E) The other causes of diaphragmatic paralysis (e.g., diaphragmatic hypoplasia, neural and neuromuscular disorders) are very rare. (Choice A) Ipsilateral and less frequently contralateral hypoplastic lung is characteristic for diaphragmatic hernia. (Choice B) Mediastinal shift to the unaffected side typically occurs in a diaphragmatic hernia, but may occur in diaphragmatic paralysis. (Choice D) Sacral agenesis is a rare anomaly observed in children of diabetic mothers. It is not related to any of these conditions. Educational Objective: Diaphragmatic paralysis in a newborn is typically caused by birth trauma or cardiothoracic surgery. It is typically accompanied by the signs of brachial plexus injury, such as Erbs palsy. 61% of people answered this question correctly.

Case 78 A mother brings her 1-year-old son to the emergency department with complaints of a brief episode of jerky movements all over his body. The jerking episode lasted for about one minute. The mother adds that the patient has had a cough with fever for the past three days. The patient now appears comfortable. Vital signs are as follows: temperature 104 F (40 C), pulse 120/min and respiratory rate 22/min. Chest auscultation reveals bilateral rhonchi and wheezing. Chest x-ray shows bilateral pulmonary infiltrates. Basic metabolic panel results are all normal. While giving the necessary treatment to the patient, the mother expresses her concern about the possibility of her child having epilepsy. What do you tell her? A. There is no increased risk compared to the general population. B. There is a slightly increased risk compared to the general population. C. There is a significantly increased risk compared to the general population. D. There is no way to predict the prognosis of his condition. E. There is currently a high risk compared to the general population, but this will decline as the patient grows older. Explanation: The generally accepted criteria for febrile seizures are: Age less than six years No past history of afebrile seizures Temperature greater than 38 C No evidence of CNS infection / inflammation No metabolic disturbances present which may produce seizures Febrile seizures may be subdivided into 2 forms: simple and complex. A simple febrile seizure is characterized by the absence of focal features, a duration of less than 15 minutes for an isolated event, and for seizures occurring in series, a cumulative duration of less than 30 minutes. This form of seizures is more common, and is associated with only a mild elevation of the risk for subsequent epilepsy (i.e. afebrile seizures) in latter life, compared to the general population. Complex febrile seizures, on the other hand, are characterized by focal features (i.e. postictal paresis), a duration of more than 15 minutes, and if occurring in series, a cumulative duration of greater than 30 minutes. The patients history is classic for a simple febrile seizure; therefore, he has a slightly increased risk compared to the general population. It is important to note that the question specifically asks about prognosis, and not recurrence. Had the question particularly asked about the recurrence of febrile seizures during childhood, then the answer would be significantly elevated risk, regardless if this case was a simple or complex febrile seizure for the first time. (Choice A, C) Contrary to previous thinking, it is now accepted that patients with simple febrile seizures have a slightly increased risk of subsequent epilepsy in latter life compared to the general population. (Choice D, E) Multiple prospective studies have established a slightly increased risk for subsequent epilepsy in latter life of patients with childhood febrile seizures, as mentioned above. Educational Objective: Febrile seizures are divided into simple and complex febrile seizures. Contrary to previous thinking, it is now accepted that patients with simple febrile seizures have a slightly increased risk of subsequent epilepsy in latter life compared to the general population.

Case 79

Several tests have been developed to measure the serologic markers of breast cancer. These tests have different specificities and sensitivities for the early stage of breast cancer. Which of the following tests could be the best screening test for the early detection of breast carcinoma if applied to a population with a stable incidence of the disease? A. Sensitivity ? 80%, specificity ? 90% B. Sensitivity ? 65%, specificity ? 97% C. Sensitivity ? 70%, specificity ? 94% D. Sensitivity ? 75%, specificity ? 92% E. Sensitivity ? 85%, specificity ? 90% Explanation: A screening test must have a high sensitivity. This high sensitivity helps to RULE IN the disease, giving as few false-negative results as possible. Furthermore, the high sensitivity increases the negative predictive value (NPV) of the test: NPV = True negatives / (True negatives + False Negatives) In this case, the test with the highest sensitivity and good specificity is the best choice. (Choice B) Choosing a highly specific test with low sensitivity would give more false-negative results; thus, many diseased people would be labeled healthy. Educational Objective: A screening test must have a high sensitivity. This high sensitivity helps to RULE IN the disease by decreasing the number of false-negative results, and by increasing the negative predictive value. 81% of people answered this question correctly. Case 80 The following vignette applies to the next 2 items A 58-year-old Caucasian woman comes to see you in the office with complaints of right wrist pain for the past 2 months. The pain is located in the thumb and first 2 fingers, and is associated with numbness and a tingling sensation in the same distribution. The symptoms get worse at night, and with activities requiring prolonged flexion or extension of her wrist. Her other medical problems include osteoporosis, a history of wrist fracture, diabetes, hypertension, coronary artery disease, hypothyroidism and end stage renal disease. The Phalen maneuver was performed in the office, and similar symptoms were reproduced after 30 seconds. You make a clinical diagnosis of carpal tunnel syndrome. Item 1 of 2 Which of the following conditions is least associated with the diagnosis of carpal tunnel syndrome? A. Diabetes mellitus B. Osteoporosis C. End stage renal disease D. Hypothyroidism E. Wrist trauma Explanation: Carpal tunnel syndrome (CTS) is due to an entrapment of the median nerve on the volar surface of the wrist. A variety of local or systemic conditions have been associated with carpal tunnel syndrome because they can cause external compression of the median nerve. Some of these conditions are: a history of wrist trauma or fracture, diabetes mellitus, rheumatoid arthritis, hypothyroidism, acromegaly, pregnancy, menopause, end stage renal disease and dialysis, obesity, and fibromyalgia. Osteoporosis is characterized by a progressive loss of bone mineral density and an increased risk of bone fractures. It is not associated with an increased incidence of carpal tunnel syndrome.

(Choice A) Evidence from clinical studies support the association between the presence of diabetes and the development of carpal tunnel syndrome. (Choice C) Carpal tunnel syndrome is commonly seen in patients with end stage renal disease (ESRD) and dialysis. The etiology appears to be multifactorial in these patients. Direct compression, ischemic damage to the median nerve, and infiltration of the carpal tunnel with amyloid fibrils all seem to play a role in its pathogenesis. (Choice D) CTS should be suspected in patients with hypothyroidism and wrist pain / paresthesias. It is believed to be due to the deposition of mucopolysaccharides in the carpal tunnel. (Choice E) Direct trauma or compression by callus formation can cause median nerve entrapment, which can lead to carpal tunnel syndrome. Educational Objective: Diabetes mellitus, hypothyroidism, ESRD / dialysis and direct trauma are associated with the development of carpal tunnel syndrome. Item 2 of 2 Which of the following is the most appropriate initial approach to manage her symptoms? A. Observation and tight blood sugar control B. A trial of a short course of oral prednisone C. Continuous wrist splint D. Surgical release of volar carpal ligament E. Corticosteroid injection into the carpal tunnel Explanation: Carpal tunnel syndrome (CTS) occurs due to a compression of the median nerve in the carpal tunnel. The initial symptoms are pain and paresthesias in the median nerve distribution (the thumb, the first 2 fingers and the lateral half of the ring finger). Symptoms usually get worse at night, and with prolonged use of the wrist, particularly in activities involving repeated flexion or extension of the wrist. The initial treatment is directed at measures which can help reduce the pressure on the median nerve. In the majority of the patients, conservative treatment is usually enough to relieve the symptoms and prevent the progression of the disease and its complications. Continuous wrist splinting is a simple and effective way to keep the wrist in a neutral position, and to reduce the pressure on the nerve in the carpal tunnel. It is especially useful to reduce the nighttime symptoms. (Choice A) Although diabetes has been associated with the development of CTS, tight blood sugar control alone will not improve the symptoms of carpal tunnel syndrome. (Choice B and E) A short course of oral glucocorticoids may relieve the pain, but this effect is short lived. Furthermore, these treatment regimens are associated with a higher incidence of complications. A trial of carpal tunnel injection with glucocorticoids may be helpful in patients with persistent pain despite conservative management; however, steroid injection is not a benign procedure. In the worstcase scenario, it can cause permanent nerve injury with a claw hand deformity; therefore, this has to be performed by highly-skilled physicians only. (Choice D) The majority of patients usually respond to conservative measures (wrist splints) and local injections of glucocorticoids. Surgical treatment is indicated in patients with persistent pain despite the abovementioned measures, motor weakness, or atrophy of the thenar eminence. Educational Objective: Wrist splints are an effective initial way of improving the symptoms of patients with carpal tunnel syndrome. 72% of people answered this question correctly. Case 81

A 35-year-old male has been hospitalized for two days after being involved in a motor vehicle accident where he sustained multiple severe injuries, including rib fractures. He is developing progressive shortness of breath, and is being maintained on 100% oxygen. He has no previous medical history or any known lung abnormalities. His family history is not significant. He does not smoke, drink alcohol, or use any illicit drugs. A chest x-ray shows diffuse bilateral pulmonary infiltrates. You strongly suspect ARDS. Which among the following is the most useful strategy to decrease mortality in ARDS patients? A. Administration of surfactant B. Early administration of methyl prednisone C. Inhaled nitric oxide (NO) D. Prostacyclins E. Mechanical ventilation that delivers lower tidal volumes & limits plateau pressure Explanation: Development of pulmonary contusions secondary to chest wall trauma is an important etiological factor of ARDS. ARDS is associated with a mortality rate of 35-40%. Among the different strategies, mechanical ventilation that delivers lower tidal volumes (< 6 ml/kg) and limits plateau pressure (< 30 CmH2O) has been shown to be relatively more effective. Studies comparing the effects of low tidal volume and limited plateau pressure to that of high tidal volume and plateau pressure have shown a significant reduction in mortality in the first group (32% vs. 40%). (Choice A) Theoretically, surfactant decreases the alveolar surface tension and reduces the chances of atelectasis. It facilitates the mucous clearance and helps to suppress inflammation. Unfortunately, studies have failed to demonstrate any significant positive outcome on 30 days mortality in these patients. (Choice B) Although steroids are frequently used in clinical practice in such situations, studies have not shown any beneficial effect on the acute phase. Some studies have even demonstrated increased infections in patients using steroids. Steroids are helpful, however, in reducing the intensity of the fibro-proliferative phase of ARDS in later stages. (Choice C) NO is a vasodilator which should theoretically improve V/Q mismatch; however, large multicenter, double blind, controlled studies have shown that the effect of NO compared to placebo, was modest and inconsistent. There was no significant difference found in 28 days mortality in both arms of these studies. (Choice D) Prostacyclins have similar effects as NO since it increases oxygenation and decreases pulmonary pressures; however, studies have also failed to demonstrate significant effects of prostacyclins on mortality. Educational Objective: ARDS is associated with a mortality rate of 35-40%. Among the different strategies, mechanical ventilation that delivers lower tidal volumes (< 6 ml/kg) and limits plateau pressure (< 30 CmH2O) has been shown to be relatively more effective than other strategies such as NO inhalation, use of prostacyclins, exogenous surfactant, and steroids. Case 82 A 60-year-old man comes to the emergency department for evaluation of left foot pain. While waiting in the triage area, his 56-year-old wife suddenly complains of shortness of breath and generalized itching. You bring her into the examination room. Her husband tells you that she was eating a cookie in the waiting area when her symptoms started. She has a history of allergies to peanuts, and he believes that the cookie had peanuts in it. On physical examination, her vital signs reveal a temperature of 36.7 C (98 F), blood pressure is 88/60 mmHg, heart rate is 124/min, and respiratory rate is 26/min. She is using her accessory muscles of respiration and has an audible wheeze. Her skin is flushed and has urticarial rash all over her body. Lung examination reveals diffuse wheezing on both sides. Which of the following is the most appropriate next step in the management of this patient? A. Administer 1 mg of intravenous Glucagon B. Intravenous hydrocortisone 100 mg every six hours C. Administer 0.5 mL of 1:1000 epinephrine subcutaneously

D. Administer 0.25 mg of terbutaline subcutaneously E. Start albuterol nebulization Explanation: The patient has a clinical syndrome consistent with an anaphylactic reaction. Anaphylaxis is a lifethreatening immediate IgE-mediated hypersensitivity reaction to exogenous stimuli. A few common exogenous causes of anaphylaxis are foods (peanuts, fish, or other sea foods), drugs (beta-lactam antibiotics, angiotensin-converting enzyme inhibitors), intravenous contrast dye, and insect stings. The symptoms of anaphylaxis typically develop within 5-60 minutes of exposure to the allergen. Patients typically have a feeling of flushing, generalized pruritus or urticaria, shortness of breath, and wheezing secondary to bronchospasm. In severe cases, patients can develop laryngeal edema, angioedema, or severe bronchospasm leading to respiratory failure. Severe hypotension and cardiovascular collapse can occur secondary to peripheral vasodilation and hypovolemia, causing anaphylactic shock. The initial management consists of rapid assessment of patients airway and cardiopulmonary status, removal of the offending agent and obtaining adequate intravenous access. Patients with bronchospasm and hypotension should be administered 0.3 to 0.5 mL of 1:1000 epinephrine subcutaneously or intramuscularly. This may be repeated for every 10-15 minutes up to a total of 3 doses. Epinephrine is the preferred drug in patients with anaphylaxis as it can reverse both bronchospasm and hypotension induced by massive histamine release. (Choice A) Glucagon should be administered to patients with anaphylaxis if they have a history of concurrent beta-blocker use. Glucagon acts through receptors other than beta receptors and causes positive inotropic and chronotropic effects. (Choice B) Hydrocortisone or other intravenous corticosteroids are only effective in preventing the late phase reaction. They should not be used as an initial drug of choice in patients with anaphylaxis. (Choice D) Terbutaline is a beta 2-receptor agonist and is effective in reversing the bronchospasm. It does not reverse the hypotension and should not be used alone in the initial stages. Educational Objective: Subcutaneous or intramuscular administration of 0.3-0.5 mL of 1:1000 epinephrine should be used as an initial pharmacologic therapy for patients with symptoms of anaphylaxis. Case 83 A healthy 6-year-old African-American girl is brought to the physician for a routine follow-up visit. She has sickle cell disease. She is asymptomatic, and has never had a sickle cell crisis. She is not taking any medications. Her mother also has sickle cell disease. Her physical examination is normal. Which of the following complications is most likely to occur in this patient? A. Splenic infarction B. Bone infarction C. Splenic sequestration D. Stroke E. Acute coronary ischemia Explanation: Among the above complications, splenic sequestration is the most common. It occurs in about 20% of the patients with sickle cell disease. If the disease presents before 2 years of age, one-third of these patients will be affected. (Choice A) The falciform red blood cells in sickle cell disease may cause vascular occlusion, which could lead to a series of ischemic events, such as splenic infarction; however, such ischemic events are less common complications than splenic sequestration.

(Choice B) Bone infarction is a complication that can lead to osteomyelitis in the affected bone. It is not as common as the other complications of sickle cell disease. (Choices D and E) Stroke and acute coronary ischemia are rare complications, and are especially uncommon in the pediatric population. The latter complication is also a differential diagnosis in a patient with sickle cell acute chest syndrome. Educational Objective: In children, the most common initial symptom of sickle cell disease is dactylitis, which develops in 40% of patients. Splenic sequestration is the second most common, and occurs in about 20% of patients. Other complications, such as ischemic events, are not as frequent, and are seen mainly in the adult population.

Case 84 A 43-year-old Chinese-American man comes to the office. He is complaining of diarrhea, cramping abdominal pain, postprandial nausea and vomiting for the past five days. He also has dizziness, generalized sweating, and shortness of breath. He had a partial gastrectomy for a bleeding gastric ulcer one month ago. He has smoked one pack of cigarettes a day for twenty years, and quit two months ago. He does not use alcohol or drugs. The patient is a food handler. His medications are omeprazole and antacids. Examination reveals a mildly distended abdomen with increased bowel sounds. The epigastrium is mildly tender, and the postsurgical scar is healing well. Which of the following will be the most appropriate therapeutic intervention? A. Start metoclopramide. B. Start a high protein diet. C. Start a low-fiber diet. D. Start a high carbohydrate diet. E. Start alprazolam. Explanation: The patient has dumping syndrome, a complication of gastrectomy. Liquid and food passage through the stomach into the jejunum is faster. This leads to abdominal pain, diarrhea, nausea and vomiting, as well as some neurovegetative symptoms such as dizziness, generalized sweating, and dyspnea. A change in the diet usually relieves the symptoms. A high protein diet, and fractionated, smaller, but more frequent food portions are advised. (Choice D) The diet should be low (not rich) in carbohydrates. (Choice A) Metoclopramide stimulates the contraction of the lower esophageal sphincter and increases gastric emptying. It may worsen the symptoms. (Choice C) A low-fiber diet may improve the diarrhea, but will not control the other features of dumping syndrome. (Choice E) Alprazolam is helpful for neurovegetative (not digestive) symptoms. Educational Objective: Dumping syndrome is a common complication of gastrectomy. Treatment is aimed at decreasing the speed of the passage of fluids and food into the small gut. A high-protein and low-carbohydrate diet is advised, as well as smaller but more frequent meals throughout the day. Case 85

A 28-year-old man with acquired immune deficiency syndrome (AIDS) is hospitalized for the treatment of Pneumocystosis carinii pneumonia and esophageal candidiasis. He was diagnosed with AIDS fourteen months ago. His current medications include zidovudine, lamivudine and indinavir. In the hospital, he is being treated with trimethoprim - sulfamethoxazole (Bactrim) and intravenous fluconazole (Diflucan). There is an apparent symptomatic improvement with therapy. On his fourth day in the hospital, he complains of decreased vision on the right side, and weakness and a tingling sensation in the right upper and lower extremities. Physical examination reveals visual field defects in both eyes, but the pupillary reflexes are intact. The right upper and lower extremities exhibit exaggerated deep tendon reflexes and diminished sensations of touch and motor strength. MRI of the brain reveals non-enhancing demyelinating lesions in the parietal-occipital region of the left cerebral hemisphere with no mass effect. Which of the following is the most likely diagnosis? A. Progressive multifocal leukoencephalopathy B. Toxoplasmosis C. CNS lymphoma D. Brain abscess E. HIV encephalopathy Explanation: Progressive multifocal leukoencephalopathy (PML) is an opportunistic infection seen in immunocompromised patients. It is caused by the JC virus, a human polyomavirus. The exact mode of transmission is unknown. PML predominantly involves the cortical white matter, but the brainstem and cerebellum may also be involved. The lesions typically do not produce a mass effect, and the onset of symptoms is usually gradual. The most common presenting symptoms are hemiparesis and disturbances in speech, vision and gait. Cranial nerve deficits may occasionally develop. An immunocompromised patient with focal neurological deficits should raise the suspicion for PML, and this diagnosis is best confirmed with MRI. Classic MRI findings in PML consist of multiple demyelinating, non-enhancing lesions with no mass effects. There is no effective treatment for PML, and the mean duration of survival from the time of diagnosis is six months. (Choices B, C and D) Although toxoplasmosis, CNS lymphoma and brain abscess are all common in AIDS patients, these are associated with ring-enhancing lesions and show a mass effect on MRI. (Choice E) HIV encephalopathy usually presents with dementia as the predominant symptom. Focal neurological deficits are not characteristic of this disease. MRI findings may be similar to those of PML, but these usually tend to be symmetrical. Educational Objective: Multiple focal neurological deficits in an immunocompromised patient (i.e., patient with AIDS) suggest a diagnosis of PML. The usual findings on MRI are non-enhancing, demyelinating lesions with no mass effects. Case 87 A 10-year-old girl is brought to your office by her mother, who is your close friend. Your friend is extremely concerned because the girl has had a recent change in behavior. She has become irritable and cranky. Her school grades have dropped significantly. She has been sleeping poorly at night, and has started wetting her bed. She also refuses to sleep at night until her father returns home and goes to bed. You are aware that the girl's father works as a taxi driver and is an alcoholic. You attempt to talk to the young girl, but she just bursts into tears. Prior to this office visit, you have known her to be a very cheerful and lively girl. Which of the following diagnoses should you consider at this point? A. Major depression with melancholic features B. Adjustment disorder C. Anxiety disorder D. Physical abuse E. Acute stress disorder Explanation: Although it may not necessarily be the correct diagnosis, physical or sexual abuse should be highly suspected in children (especially females) with sudden behavioral problems, especially if the family

has an unstable economic background or if the child's parents have a history of drug/alcohol abuse. In order to possibly protect the child from further harm, it is imperative to rule out the diagnosis of physical abuse before any other disorder or disease is considered. (Choice A) Major depression is certainly an important differential diagnosis because poor sleep and irritability are symptoms of depression in the pediatric age group; however, this diagnosis should be considered only after physical abuse has been ruled out. (Choice B) Adjustment disorder is usually an abnormal behavioral response to an identifiable stressor. The response is usually disproportionate to what would be normally expected with the given stressor. In this case, there is no mention of any particular stressful event; however, a detailed history may reveal some stressors. Nevertheless, this diagnosis should be considered after ruling out physical abuse. (Choice C) A diagnosis of anxiety disorder requires the presence of a pattern of nervousness about several different issues. A detailed history could possibly detect an underlying anxiety disorder, but physical abuse needs to be ruled out first. (Choice E) Acute stress disorder always develops in response to a traumatic event and presents as recurrent thoughts or flashbacks of the event associated with hypervigilance and avoidance of stimuli associated with the event. Educational Objective: Always maintain a high index of suspicion for physical/sexual abuse in children (especially females) with sudden behavioral problems, especially if the family has an unstable economic background or if the child's parents have a history of drug/alcohol abuse. 62% of people answered this question correctly. Case 88 A 32-year-old African-American female calls you in the office and complains of a moderate amount of vaginal discharge and intense pruritus. Approximately one week ago, she noticed the presence of a whitish, malodorous vaginal discharge. She has been using an over-the-counter (OTC) antifungal cream for a week now. She was diagnosed with insulin-dependent diabetes 20 years ago. She has been in a monogamous relationship for the last two years. She gets similar vaginal infections approximately twice or thrice a year, which usually resolve with OTC antifungal creams. She is concerned, and asks you what she should use this time. What is the most appropriate response? A. Continue the antifungal cream for another two weeks B. Schedule an appointment to see you in the office C. Prescribe a single 400 mg dose of fluconazole D. Ask her about her blood sugar control in the last month E. Obtain a more detailed sexual history Explanation: Vulvovaginitis is an extremely common condition in the sexually active age group. The etiology includes infections of the vulva, vagina, and cervix, and the noninfectious causes are chemicals, irritants, hormonal disorders, and rarely, some systemic disorders. Common infectious causes are vulvovaginal candidiasis, bacterial vaginosis, and trichomoniasis. Most of the symptoms of vaginitis are nonspecific, and self-diagnosis and treatment by the patient is unreliable. Even a diagnosis by history and physical examination by a physician is not reliable and should always be confirmed in the office by microscopy. This is important to avoid frequent misdiagnosis and inappropriate treatment. The most commonly used techniques are saline and KOH preparation for microscopy. The vaginal secretions have characteristic pseudohyphae in candidiasis. Bacterial vaginosis is confirmed by the presence of clue cells in the smear. Motile trichomonads and an abundance of polymorphonuclear cells are found in patients with trichomoniasis.

(Choices A and C) Continuing the antifungal cream or prescribing fluconazole is inappropriate in the absence of a specific diagnosis. (Choice D) Tight blood sugar control is important to prevent recurrences of vulvovaginal candidiasis. This may give a clue to the diagnosis, but should not replace microscopy for confirmation of the diagnosis. (Choice E) A detailed sexual history can provide further evidence of a specific infection, but laboratory evidence is required for appropriate therapy. Educational Objective: Clinical features of vaginitis are very nonspecific, and the diagnosis should always be confirmed by microscopy to avoid inappropriate treatment. 44% of people answered this question correctly. Case 89 The following vignette applies to the next 2 items An African-American male newborn is evaluated immediately after an uncomplicated vaginal delivery. His mother has no medical problems. The physical examination is unremarkable; however, the baby appears plethoric, to some extent. Blood samples from the heel through simple prick method reveal a hematocrit of 70%. Item 1 of 2 What is the most appropriate next step in management? A. Recheck the hematocrit by performing a new heel prick in one hour B. Recheck the hematocrit by performing a new heel prick immediately C. Recheck the hematocrit by performing a new heel prick in 12 hours D. Recheck the hematocrit from a sample obtained from a central line E. Recheck the hematocrit from a sample of peripheral blood Explanation: Neonatal polycythemia is defined as a hematocrit higher than 65%, or a hemoglobin level greater than 22 g/dL. If the hematocrit is greater than 65% with a heel prick, it should be remeasured using a venous blood sample. Peripheral venous hematocrit is usually 5 to15% lower than hematocrit taken from a capillary sample. (Choices A and B) The hematocrit can be falsely elevated in capillary samples. For this reason, it is paramount to obtain a sample of venous blood. (Choice C) The hematocrit reaches its maximum value when the infant is approximately two hours of age. For this reason, it is important to recheck the hematocrit levels 12 to 24 hours after delivery if the hematocrit value is borderline high. The sample must be venous (not capillary) blood. (Choice D) There is no need to obtain a venous sample from a central line because the procedure is invasive, and the result does not have any diagnostic or therapeutic significance. Educational Objective: Neonatal polycythemia is diagnosed when the peripheral venous hematocrit is higher than 65%. High hematocrit levels that are obtained from capillary samples (usually from the heel) are only screening values that need to be confirmed through venous blood sampling. Item 2 of 2 One day later, the patient is found to be highly irritable, sleepy and with no appetite. The nurse observes one episode of apnea. His temperature is 37.4 C (99.3 F), pulse is 170/min and respirations

are 60/min. Examination reveals a drowsy, plethoric newborn who is hypotonic and poorly responsive. Cyanosis of the fingertips is observed. There are no focal neurologic deficits. Laboratory tests reveal the following: CBC Hb 22.0 g/dL Platelet count 320,000/cmm Segmented neutrophils 70% Bands 3% Lymphocytes 25% Monocytes 2% Serum Chemistry Serum Na 146 mEq/L Chloride 110 mEq/L BUN 16 mg/dL Calcium 9.6 mg/dL Ht 68% Leukocyte count 10,000/cmm

Serum K 4.0 mEq/L Bicarbonate 26 mEq/L Serum Creatinine 0.6 mg/dL Blood Glucose 40 mg/dL

The patient receives intravenous glucose and is transferred to the neonatal intensive care unit. What is the most appropriate next step in management? A. Amantadine and symptomatic support B. Order a chest x-ray C. Start antibiotics D. Prepare the patient for partial exchange transfusion E. Recheck the hematocrit from a sample of peripheral blood Explanation: The patient has signs and symptoms of polycythemia. Newborns with this condition develop irritability, drowsiness, poor feeding, abdominal distention and hypotonia. Peripheral cyanosis may occur. Infants usually appear plethoric and lethargic. Hypoglycemia, jaundice, and apnea are common presentations of neonatal polycythemia. These signs and symptoms are due to blood hyperviscosity, which decreases blood flow to different tissues, including the brain and gut. Treatment consists of intravenous hydration and partial exchange transfusion. (Choice A) The patients neurologic signs and symptoms are non-focal, and are caused by a metabolic, not by a structural, pathology. (Choices B and C) Apnea and hypoglycemia can be markers of neonatal sepsis; however, this patient has the classical clinical picture and laboratory findings (i.e., hematocrit of 68%) of symptomatic neonatal polycythemia. Educational Objective: Neonatal polycythemia can be a life-threatening disease that causes apnea, hypoglycemia, hyperbilirrubinemia, cardiac and respiratory compromise. Treatment consists of adequate hydration and partial exchange transfusion. Case 90 A 24-year-old man is brought to the emergency department (ED) by the police after he was arrested for fighting in the streets. During the fight, he was struck with a knife on his head, and the knife was firmly implanted in his skull. The estimated length of the knife is 14 cm, of which 3 cm has penetrated his skull. His vital signs are within normal limits. An initial physical examination reveals that he is minimally responsive to verbal or tactile stimuli. There is no other evidence of injury on his body. While in the ED, which of the following is the most important next step in the management of this patient? A. Remove the knife from his skull B. Irrigate the wound with normal saline C. Obtain a stat PT, PTT, blood group, and crossmatch

D. Obtain a CT scan of the head to assess the damage E. Recheck the hematocrit from a sample of peripheral blood Explanation: The patient has presented to the emergency department with a penetrating injury to his skull. Maintenance of an adequate airway and hemodynamic stability are the first steps in acute management. (This is true for all patients with a penetrating injury or trauma to any body part.) Even if he has normal hemodynamic parameters at this point, he is still at high risk for subsequent deterioration secondary to acute blood loss within the next few hours, especially during the process of removal of the knife. This is further compounded by the lack of availability of any pertinent medical history due to his mental status. In conclusion, all such patients should receive immediate laboratory studies to identify any reversible coagulopathy that may increase the risk of bleeding. This also includes blood typing and crossmatching, in case there is an urgent need for a blood transfusion during the course of his treatment and hospitalization. (Choice A) Even though immediate removal of the knife seems appropriate, it should not be attempted without obtaining any prior information. Removal of the knife may lead to an increase in the bleeding due to opening up of the dural venous sinuses. This should be attempted in the operating room. (Choice B) Local wound management is not an emergency, and can be done at a later stage. (Choice D) It is important to ensure the patient's hemodynamic stability before proceeding with any further radiographic imaging. Educational Objective: Maintenance of adequate hemodynamic stability is the most important step in the management of a patient with a penetrating injury. Always attempt to identify and correct any reversible cause of coagulopathy, and anticipate the need for blood transfusions in such patients. Case 91 It is influenza season, and a 53-year-old Caucasian female with a history of asthma, epilepsy, and obesity comes to your clinic because of a four-day history of fever, headaches, myalgia, and weakness. Her sister was recently sick. Ten days ago, she was vaccinated against the flu. Her throat swab for influenza viral antigen was positive. The patient is concerned that she got the flu despite being vaccinated against it. Her physical examination is unremarkable, except for oropharyngeal hyperemia and mild cervical lymphadenopathy. Which of the following is the most appropriate next step in the management of this patient? A. Amantadine and symptomatic support B. Zanamivir and symptomatic support C. Acetaminophen and symptomatic support D. Rimantadine and symptomatic support E. Aspirin and symptomatic support Explanation: It takes about two weeks to mount an adequate immunologic response against the influenza virus after proper vaccination. Before this time, the patient is vulnerable to the infection. Therapy with drugs such as zanamivir, amantadine and rimantadine should be started within the first 30-48 hours of the onset of sypmtoms. Beyond this period, symptomatic treatment with acetaminophen (Tylenol) is preferred. (Choice E) Acetaminophen use is preferred over aspirin use to avoid the risk of Reyes syndrome (associated with aspirin use), especially in the pediatric population. (Choices B, A and D) Therapy with zanamivir must be started within the first 30 hours of symptoms to be effective, while amantadine and rimantadine should be given at least within the first 48 hours. Furthermore, zanamivir can exacerbate asthma by inducing bronchospasm, while amantadine and rimantadine can decrease the seizure threshold in epileptic patients.

Educational Objective: Treatment for influenza must be started immediately, especially in vulnerable patients, within the first 30 to 36 hours when using zanamivir and oseltamivir (neuraminidase inhibitors), or within the first 48 hours when using rimantadine or amantadine. If the patient is seen after this period, only symptomatic therapy with acetaminophen is suggested. Case 92 You are an on-call physician in your community hospital. At about 6:00 p.m., you receive a phone call from one of your patients saying that her 4-year-old son, while playing with his elder brothers toys, started coughing all of a sudden, became short of breath, and appeared cyanotic. Based on her description of the incident, you strongly suspect foreign body aspiration. In addition to advising her to call 911 immediately for help, what else should you recommend? A. To hold her son upside down and blow on his back B. Perform the Heimlich maneuver C. He needs immediate rigid bronchoscopy D. He needs immediate laryngoscopy E. Give him something to drink Explanation: No matter what the nature of the emergency is, never forget your ABCs of resuscitation.The history in this case is very suggestive of foreign body aspiration, and the child has become cyanotic, which indicates that he has choked his major airway. It is very important to immediately evacuate his airway before doing anything else. For children less than one year of age, it is strongly recommended to hold the child upside down, and give a series of five blows on the back and five chest thrusts. If the child is older than one year, give a series of five abdominal thrusts (the Heimlich maneuver) with the child standing or sitting. If the child is unconscious, do it while the patient is lying down. After the abdominal thrusts, examine the airway for a foreign body. If visualized, it should be removed. (Choice A) As explained above, this is recommended in a child less than one year of age. (Choices C and D) Rigid bronchoscopy (procedure of choice for foreign body aspiration) or laryngoscopy is appropriate to consider after the child has been brought to the hospital. Educational Objective: Never forget your ABCs of resuscitation no matter what the nature of emergency is. The Heimlich maneuver is recommended in children older than one year of age. Below this age, give blows on the back with chest thrusts. 62% of people answered this question correctly. Case 93 A 20-year-old college student presents to the student health center with a five day history of dysuria and mucopurulent urethral discharge. He is sexually active, but denies any recent new sexual partners. Grams stain of the discharge shows multiple white blood cells per high power field, with intracellular gram-negative diplococci. You treat the patient with a single dose of ceftriaxone (125 mg intramuscularly). A week later, he returns to the clinic because of persistent urethral discharge. What is the most likely cause of his persistent symptoms? A. Antibiotic resistance of the organism B. Inadequate dose of the antibiotic C. Repeat exposure to the infectious agent D. Coinfection with another organism E. Wrong choice of the antibiotic Explanation:

The patient in the above scenario is suffering from gonococcal urethritis. Patients with gonococcal infection are frequently co-infected with Chlamydia trachomatis. The most common symptom of urethritis is painful urethral discharge. It is diagnosed by the presence of > 5 WBCs per high power field in urethral secretions, presence of leukocyte esterase, or >10 WBCs on microscopy of the first voided urine. The diagnosis is confirmed with a Gram stain smear and culture of a urethral swab specimen. All the patients diagnosed with gonococcal urethritis should also be treated empirically with an agent active against C. trachomatis, such as doxycycline 100 mg orally twice a day for 7 days, or azithromycin 1 gm orally as a single dose. (Choices A, B, and E) Ceftriaxone (125 mg intramuscularly) is one of the preferred and most commonly used agents against gonococcal infection. Neisseria gonorrhoeae is extremely susceptible to broad-spectrum cephalosporins, including ceftriaxone. The current recommended dose is 125 mg intramuscularly, which is effective in more than 99% of the cases. (Choice C) Repeat exposure to N. gonorrhoeae is possible, but it is too early to have symptomatic disease from reexposure after adequate treatment. Coinfection with Chlamydia is more likely. Educational Objective: All the patients diagnosed with gonococcal urethritis should also be treated empirically with an agent active against C. trachomatis, such as doxycycline 100 mg orally twice a day for 7 days, or azithromycin 1 gm orally as a single dose. 75% of people answered this question correctly. Case 94 Two Caucasian children are brought to the physician by their mother. The older child is an eight-yearold boy and the younger is his 6-year-old sister. Ever since the family dog died, the boy has been crying uncontrollably, while his sister looks calm and believes that everything will be fine. Both children do not have any medical problems, and are not on any medication. Their developmental and growth milestones are normal. What is the best way to approach this situation? A. Refer the 8-year-old child to psychotherapy B. Refer the 6-year-old child to psychotherapy C. Refer both children to psychotherapy D. Start both children on sedative medications E. Both children are normal Explanation: Bereavement in children can present in different ways, according to the childs age. Children who are between 3 and 7 years old usually react to the loss of a loved person or pet with disbelief. At this age, these children think that death is only temporary (e.g., sometimes they think that the dead person or animal is only sleeping and will later wake up or come back to life). This reaction usually lasts for some days, after which the child gradually accepts the fact that death is not reversible. On the other hand, older children (greater than 7 years) are already aware that death is final. They can react in different manners: with sadness, uncontrollable tearing, inability to concentrate, rage, guilt, nightmares, regressive behavior or social isolation. (Choices A, B, and C) Each of the two siblings has reacted appropriately, according to their ages, and do not need to be referred to psychotherapy. (Choice D) Sedative medications are indicated only for those children who develop unmanageable hysteric crisis or panic attacks. Educational Objective: Bereavement in children can present in different ways, according to the childs age. Those who are younger than 7 years tend to react with disbelief, and those younger than 5 years have magical

thoughts about death and can feel guilty or responsible. Children older than 7 years accept death as final and can experience depression, anxiety or have regressive behavior. Case 95 A father brings his 4-year-old girl to the emergency department of the local childrens hospital because she has developed scattered ecchymoses and petechiae all over her body within the last few hours. Laboratory evaluation reveals a platelet count of 4,000/?L. Her hemoglobin and WBC counts are normal. The father believes he had a similar episode as a child. What is the best means of managing this girl?s condition? A. Emergency splenectomy B. Administration of prednisone C. Administration of packed red blood cells D. Administration of platelets E. Administration of cryoprecipitate Explanation: Platelet-specific autoantibodies are the presumed pathogenesis of idiopathic thrombocytopenic purpura. In children, the condition is typically characterized by a sudden onset of bleeding, manifested as petechiae, purpura, epistaxis, and gingival bleeding. More severe bleeding is rare. Commonly, there is a history of infection in the several weeks prior to presentation. Symptomatic patients with moderate to severe thrombocytopenia (<30,000/?L) should be treated with corticosteroids (e.g., prednisone or methylprednisolone) and intravenous immunoglobulin (Choice B). Splenectomy (Choice A) may be indicated in children with chronic ITP who persistently experience hemorrhagic symptoms. It would not be appropriate in this child. Administration of packed red blood cells, platelets, or cryoprecipitate (Choices C, D, and E) have not been demonstrated to improve patient symptoms or outcome, and are therefore not indicated. Educational Objective: Symptomatic, severely thrombocytopenic children with ITP should be treated with corticosteroids and/or intravenous immunoglobulin. 64% of people answered this question correctly. Case 96 A 3-year-old Caucasian boy is brought to the emergency department by his mother because she saw him putting coins in his mouth. The child is asymptomatic. A plain radiograph reveals a coin located in the childs stomach. The patients mother insists that something should be done immediately. What is the next best step in the management of this patient? A. Ipecac syrup B. Laparotomy C. Fluoroscopy-guided removal D. Endoscopic removal E. No intervention Explanation: Gastrointestinal foreign bodies typically occur in toddlers, because exploring objects by placing them in the mouth is a characteristic behavior pattern of this group. Coins are the most common GI foreign bodies. In the clinical scenario described, no intervention is the best choice because the coin is already in the stomach. Up to 90% of foreign bodies that have made it into the stomach will be passed without difficulty. Pyloric obstruction is rare and manifests as persistent vomiting.

(Choices C and D) Foreign bodies can become lodged in any of the areas of esophageal physiologic narrowing. Such cases require immediate attention and removal. (Choice A) Ipecac syrup is not indicated in the management of patients with a GI foreign body. (Choice B) Laparotomy is indicated only when serious complications (e.g., perforation) have developed. Educational Objective: Up to 90% of foreign bodies that have made it into the stomach will be passed without difficulty; therefore, no intervention is usually necessary. Case 97 The following Vignette applies to the next 2 items A 22-year-old Caucasian man comes to clinic for routine follow-up of his mild asthma. He has a history of keeping his appointments but has missed the last three visits. Today, he seems distracted and uninterested, answering various questions with very brief replies. At one point he states he would like to hurry the visit along so he can get home sooner. After inquiries about his social life are made, he becomes a little more animated and says that lately he spends time with a new group of friends that "really know how to party." Further questioning reveals that he has been using cocaine for the past six months. He says that he began using drugs mostly because he had been feeling anxious about some college courses. After "shooting up" though, he feels more happy and confident. He dropped out of college last quarter and was fired from his job as a waiter for absenteeism. Several weeks ago, his parents asked him to move out of their house, so he is currently staying with some of the new friends. His temperature is 37.7C (99.9F), blood pressure is 120/82 mm Hg, pulse is 86/min and respirations are 15/min. Physical examination reveals septal perforations. There are blue-black lines on his arms bilaterally. Item 1 of 2 What is the next best step in the management of this patient? A. Obtain a toxicology screen B. Refer him to a detoxification program C. Admit him to the hospital D. Prescribe methadone E. Prescribe naloxone Explanation: Drug abuse is a common occurrence; accordingly, it is thought that 10-16% of people seen in an outpatient general medicine practice suffer from problems related to addiction. It is important to screen everyone for addiction because a significant portion of patients will relay such information only when specifically asked. The best evidence of long-term drug use is obtained from a thorough history and a positive toxicology screen (Choice A). The toxicology screen is also beneficial in that it reveals exactly which drugs are currently used by the patient, knowledge that is of help when weighing different treatment options. Some of the widely accepted indications for toxicology screening include a history of alcohol or other drug use, evidence of drug use on physical examination (eg, septal perforation), signs of intoxication or withdrawal, or an altered mental status. It is important to seek the patients permission before testing, however, so as to not jeopardize his trust. Referral to a detoxification program (Choice B) is certainly of help in addressing drug addiction but should not be the next step taken. Admission to the hospital (Choice C) would be warranted if there was evidence of drug withdrawal or if the patient was deemed to be a danger to himself or others.

Methadone (Choice D) is used as maintenance for those patients addicted to heroin. It satisfies the craving for opioids without causing the euphoria associated with heroin. Naloxone (Choice E) is a narcotic antagonist and can be used to reverse the effects of opiates. It is helpful in cases of opioid overdose. Educational Objective: The best evidence of long-term recreational drug use is obtained from a thorough history and a positive toxicology screening. Item 2 of 2 Appropriate action was taken. Which of the following treatment modalities would be the most appropriate means of initiating his recovery from drug addiction? A. Psychotherapy B. Group therapy C. Drug rehabilitation program D. Inpatient hospitalization E. Providing alternative medication Explanation: Drug addiction may involve physical and/or psychological dependence. Much like other chronic problems, it is also frequently associated with feelings of ambivalence and denial on the part of the patient. Effective drug treatment must take these issues into account and motivate the drug user to change his behaviors and modes of thinking. While the supreme goal of drug abuse treatment is the achievement of long-term abstinence, the short-term goals include the reduction of drug use, an improvement in the patient?s functionality, and the minimization of the medical and social consequences of drug abuse. It would therefore be most appropriate to recommend that this patient enter a drug rehabilitation program (Choice C) as a means of initiating his recovery from drug addiction. Enrollment in drug rehabilitation programs provides patients with a highly structured, drugfree environment in conjunction with intensive individual and group therapy. Psychotherapy (Choice A) can be of benefit for some addicted individuals, and is often incorporated into drug rehabilitation programs. It is rarely sufficient when used in isolation, however. Group therapy (Choice B) is an extremely powerful tool for most people in recovery, as it provides social support and accountability while also offering addicts insight into their illness. Attendance in twelve-step programs such as Alcoholics Anonymous is often credited as essential in the maintenance of long-term abstinence from drug or alcohol use. Inpatient hospitalization (Choice D) would be indicated in a patient who was experiencing physical withdrawal symptoms or who posed a danger to himself or others. Once such a patient is medically stable, however, he should be placed in a drug rehabilitation program that can better accomplish the short- and long-term goals of recovery. Medication (Choice E) is not normally indicated in the treatment of cocaine addiction. Disulfiram is one drug occasionally prescribed to discourage drinking in alcoholics, but it is not considered to be a primary first-line treatment of alcoholism. Educational Objective: Drug rehabilitation programs are the preferred means of initiating recovery from drug addiction, while group therapy programs (eg, Alcoholics Anonymous) are the preferred means of maintaining drug and alcohol abstinence. Case 98 You are the primary care physician for an obese 42-year-old Caucasian man who was diagnosed with diabetes mellitus five years ago. He was recently begun on an insulin regimen that allowed for excellent glycemic control. His medical history is also significant for hypercholesterolemia, which is well managed with atorvastatin. Recently, however, this patient has been missing his follow-up

appointments despite frequent reminders from your office staff. After four months elapse, the patient presents to the office. At this visit, his fasting blood glucose is 142 mg/dL and his glycosylated hemoglobin level is 8.1%. Both tests were within normal limits previously. After thorough inquiry, the patient admits that he began to use the herbal preparation gingko biloba. He is proud to use herbal remedies because "it,s always better to use natural medicines." Which of the following is a wellknown side effect of ginkgo biloba? A. Psychosis B. Hepatotoxicity C. Cardiac arrhythmias D. Stevens-Johnson syndrome E. Bleeding and platelet dysfunction Explanation: Gingko biloba leaf extract is an increasingly popular herbal supplement that many patients use as a "memory booster" because of its suggested propensity for increasing cerebral blood flow. Some studies have suggested that gingko is at least somewhat effective in the treatment of intermittent claudication and Alzheimer?s disease. Gingko has been associated with a number of side effects, including seizures, headaches, irritability, restlessness, diarrhea, nausea, and vomiting. It is most notorious for increasing the risk of bleeding and its potentiation of the effects of anticoagulant therapy through various mechanisms, including the inhibition of platelet-activating factor (Choice E). Several cases of serious intracerebral bleeding associated with gingko use have been reported; in most cases, patients were concurrently taking anticoagulant medications. Aconite is an ingredient in some Chinese herbal medicines used to treat pain or heart failure. It is known to cause serious and sometimes fatal arrhythmias (Choice C), including ventricular or supraventricular tachycardia, bidirectional tachycardia, sinus bradycardia with first-degree heart block, bundle branch block with junctional escape rhythm, and torsade de pointes. Hepatotoxicity (Choice B) has been associated with usage of unsaturated pyrrolizidine alkaloids. These compounds are found in several herbal supplements, including comfrey, borage leaf, and coltsfoot. Other herbal supplements known to cause liver toxicity include ephedra, chaparral, germander, and a Chinese medicine called jin bu huan. Psychosis (Choice A) may arise as a side effect of multiple prescription medications. Some examples include corticosteroids, appetite suppressants, quinacrine, antidyskinetics (eg, levodopa, carbidopa, amantadine), and isotretinoin. Stevens-Johnson syndrome (Choice D) may appear on presentation as a more severe form of erythema multiforme. It is characterized by erosion of mucous membranes, small blisters on purpuric macules, and atypical target lesions. Drugs that are known to cause this condition include antibiotics (eg, sulfonamides, aminopenicillins, quinolones, cephalosporins) and anticonvulsants (eg, lamotrignine, phenytoin, carbamazepine). Educational Objective: Gingko biloba leaf extract is used by many patients as a "memory booster" because of its suggested propensity for increasing cerebral blood flow. It is most notorious for an increased risk of bleeding and a potentiation of the effects of anticoagulant therapy through various mechanisms, including the inhibition of platelet-activating factor. 56% of people answered this question correctly. Case 99 A healthy 42-year-old Caucasian woman comes to the clinic with fever, chills, loss of appetite, sore throat, hoarseness, dry cough, headaches, and nausea. She is a paramedic. Her temperature is 38.3 C (101 F), blood pressure is 120/70 mmHg, pulse is 120/min, and respirations are 20/min. Examination shows pharyngeal erythema and exudates that are forming membranes and tender cervical lymphadenopathy. Examination of the lung is normal. The patient is treated with penicillin G

and admitted because of poor oral intake. You receive a call from the laboratory staff, who reports that the throat culture is positive for Corynebacterium diphtheriae. A decision is made to start the patient on diphtheria antitoxin to avoid further cardiac or neurologic complications. This treatment puts the patient in a greater risk for which of the following complications? A. Neurotoxicity B. Erythema multiforme C. Anaphylaxis D. Hepatotoxicity E. Bleeding diathesis Explanation: Diphtheria is a serious condition that can be life threatening. Its prompt recognition in patients with pharyngitis, cervical adenopathy, and low-grade fever is extremely important. If there is a high suspicion, diphtheria antitoxin should be administered as soon as possible to avoid complications such as myocarditis, neuritis, or, rarely, nephritis. Diphtheria antitoxin is made with horse serum; thus, the risk of hypersensitivity or serum sickness is approximately 10%. There is also a lesser risk of anaphylaxis. For these reasons, epinephrine must be always available. (Choices A, B, D, and E) There are no reports of neurotoxicity, erythema multiforme, hepatotoxicity, or bleeding complications . Educational Objective: Diphtheria antitoxin is made with horse serum; thus, the risk of hypersensitivity or serum sickness is approximately 10%. There is also a lesser risk of anaphylaxis. Case 101 Researchers want to further investigate the association between a new hypolipidemic drug and the occurrence of severe acute myositis. They noted that although several other studies reported this side effect, none of these studies statistically obtained a significant difference in the occurrence of severe acute myositis between the treatment and placebo groups. Which of the following is the best method to further investigate the above mentioned association? A. Conduct a new large-scale clinical trial. B. Review the medical charts to re-ascertain the events. C. Do stratified analysis on multiple risk factors. D. Pool the data from several trials. E. Ignore the possible association between the drug and acute myositis. Explanation: Pooling the data from several studies to perform an analysis is called meta-analysis. Meta-analysis is a useful epidemiologic tool that is employed to increase the power (i.e., the ability to detect the difference in the outcome of interest between groups) of a study. If the outcome is rare or the difference between the groups is small, it is difficult for a single, even large-scale study to detect the difference and reach statistical significance. In this case, meta-analysis can be used to increase the sample size, which thereby increases the power of the analysis. The major disadvantage of metaanalysis includes concomitant pooling of the biases and limitations of individual studies into one analysis. (Choice A) Conducting a new large-scale clinical trial is not ethical at this point because a serious complication (e.g., acute severe myositis) can be induced by the drug. (Choice C) Stratified analysis is used to control confounding factors, and is of little value in this case. (Choice B) Reviewing the medical charts may be used if misclassification of the events is suspected.

(Choice E) Ignoring the possible association between the drug and acute myositis is not a good choice. This side effect is severe; therefore, drug safety should be investigated. Educational Objective: Meta-analysis is conducted by pooling the data from several studies to increase the statistical power. The scenario described is a good example of the usefulness of meta-analysis. 36% of people answered this question correctly.

Case 102 A 42-year-old female comes to see you in the office for a follow-up visit regarding her hypertension. She has been your patient for the last eight years, during which her blood pressure has been stable on hydrochlorothiazide and amlodipine. She tells you that she recently started taking some "diet supplements" due to the advice of one of her friends. After starting the supplements, her home blood pressure readings became high. She wants your advice on this matter. Her blood pressure in the office today is 152/88 mmHg. Which of the following herbal preparations is associated with hypertension? A. Licorice B. Ginkgo C. Kava D. Black cohosh E. Horse chestnut Explanation: The use of complimentary and alternative medicine therapies (e.g., dietary supplements, herbal medicines) has been steadily increasing in the United States for the last several years. A lot of these substances are taken without any specific diagnosis, due to the recommendations of family, friends, and media without prior consultation with the patients primary care physician. In the absence of any known clinical information on their safety and efficacy, these substances have the potential to cause adverse effects. It is therefore extremely important to ask patients in detail about their current and recent over-the-counter medication use, including the use of alternative remedies. Chronic ingestion of licorice can cause or aggravate hypertension in patients. Licorice inhibits the enzyme 11-beta hydroxysteroid dehydrogenase, thereby preventing the conversion of cortisol to cortisone. The available cortisol binds to mineralocorticoid receptors and causes hypertension, hypokalemia, and metabolic alkalosis. (Choice B) Ginkgo has been marketed as an antioxidant. It is used for the treatment of mild memory loss, dementia, macular degeneration, and peripheral vascular disease. It does not have any significant adverse effects; however, it can interact with aspirin or warfarin, leading to a potential risk of spontaneous bleeding. (Choice C) Kava has been used for the treatment of various disorders. Its concomitant use with alcohol, benzodiazepines, or other prescription sedatives can potentiate their effect and cause excessive drowsiness or disorientation. (Choice D) Black cohosh has been used for the treatment of premenstrual syndrome and menopausal symptoms. Its excessive use can cause hypotension. It should be used with caution in patients already on antihypertensive medications.

(Choice D) Horse chestnut has been used in patients with venous insufficiency or chronic venous stasis. It can inhibit platelet aggregation and can cause bleeding in patients already on aspirin, aspirin-containing products, or warfarin. Educational Objective: The use of licorice can cause or aggravate hypertension in patients, and should not be used in patients with a history of hypertension. Ginkgo does not have any significant adverse effects; however, it can interact with aspirin or warfarin, leading to a potential risk of spontaneous bleeding. 41% of people answered this question correctly. Case 103 The following vignette applies to the next 2 items A 25-year-old Hispanic college student comes to the emergency department with complaints of sudden onset of left foot pain for the past hour. The pain started suddenly when he was sitting at his desk at college and has progressed gradually. He denies any history of similar episodes in the past or recent trauma to his left leg. His temperature is 37.2 C (99 F), blood pressure is 120/80 mmHg, heart rate is 86/min, and respiratory rate is 14/min. The lungs are clear on auscultation. On cardiovascular examination, the heart rhythm is regular, and a diastolic murmur is heard over the cardiac apex. The abdomen is soft, nontender and without any palpable masses. The neurological examination is within normal limits. Examination of the extremities reveal adequate, bilateral, femoral pulses. On the left side, the popliteal and dorsalis pedis pulses are absent. The left foot also appears pale and cooler than the right foot. The sensations on the left foot are intact and he is able to make voluntary movements. There is no tenderness on palpation of the lower left leg or foot. The vascular surgeons are immediately called, and the patient is taken to the operating room for urgent embolectomy, where an embolus obstructing the entire lumen of the left popliteal artery is removed. Item 1 of 2 Which of the following is the most appropriate next step in the management of this patient? A. Doppler examination of bilateral lower extremities B. Angiographic examination of the left lower extremity C. Echocardiogram D. Histologic examination of the embolus E. Hypercoagulation workup Explanation: The patient has a classic presentation of acute arterial occlusion. Acute ischemia of the limb due to acute arterial occlusion usually presents with five Ps; pain, pallor, pulselessness, paresthesias and paralysis. Almost all the patients present with a gradually progressive pain, usually in the distal extremity. The skin of the lower extremities is cool and pale, and the pulses are greatly reduced to absent. Acute arterial occlusion is usually the result of (1) an embolus from a distal source, (2) acute thrombosis due to a previously diseased vessel, or (3) direct trauma to the involved artery. The onset of symptoms can help in differentiating the etiology of arterial occlusion. The sudden onset of symptoms in a previously asymptomatic patient is most likely due to an embolus, while a history of gradually progressive symptoms in a previously symptomatic patient is consistent with thrombosis. The sudden onset of dramatic symptoms in this patient is most consistent with an embolic source of acute arterial occlusion. Most of the emboli are from a cardiac source, with a few coming from the arterial aneurysms or atherosclerotic plaques. Some of the well-recognized causes of cardiac emboli include atrial fibrillation, a prior history of myocardial infarction and ventricular dysfunction, endocarditis, valvular disease, atrial myxoma, and the presence of a prosthetic aortic valve. It is important to find the exact cause of cardiac emboli to prevent future recurrences. After an embolectomy, the surgical specimen should be sent for histopathologic examination to ascertain the exact source of the emboli.

(Choices A and B) Doppler or angiographic examination of the lower extremities is used for the diagnosis of acute arterial occlusion. These are not helpful in locating the source of the emboli. (Choice D) An echocardiogram is an important part of the diagnostic workup since this may identify a source of emboli; however, approximately 20% of the emboli can arise from a peripheral arterial source. Histologic examination is therefore more important since this can differentiate a cardiac source from a peripheral one. (Choice E) Hypercoagulation workup is important to look for inherited thrombophilias causing venous thrombosis. Since this patient appears to have acute arterial occlusion from a cardiac embolic source, the hypercoagulation workup is not indicated at this point. Educational Objective: Histopathologic examination of the embolectomy specimen is extremely useful in locating the origin of the embolus which caused the acute arterial occlusion. 5% of people answered this question correctly. Item 2 of 2 Which of the following is the most likely cause of the patients condition? A. Prolonged immobilization B. Left atrial myxoma C. Atherosclerosis of the abdominal aorta D. Thromboangiitis obliterans E. Left ventricular thrombus Explanation: Based on the clinical presentation, the most likely cause of acute embolic arterial occlusion in this 25year-old previously asymptomatic male is left atrial myxoma. Atrial myxomas are the most common primary cardiac tumors. Most of these arise from the left atrium, and the remaining arise from the right atrium and left ventricle. The tumors are typically pedunculated, with a stalk arising from the atrial septum. These can be extremely friable, resulting in embolization of the part of the myxoma to the systemic circulation. Some large tumors may initially present with signs and symptoms of mitral valve obstruction (diastolic murmur or "tumor plop"), rapidly worsening heart failure in otherwise young healthy individuals, or new onset atrial fibrillation. The diagnosis is usually made by echocardiography (either transthoracic or transesophageal echocardiography). Once the diagnosis of atrial myxoma is made, it should be excised as soon as possible to reduce the risk of recurrent embolization. (Choice A) Prolonged immobilization, along with other underlying risk factors, can cause deep venous thrombosis. It does not cause acute arterial occlusion. (Choices C & D) Thromboangiitis obliterans or atheroemboli is an unlikely cause of acute arterial occlusion in a young and otherwise previously healthy patient. (Choice E) A left ventricular thrombus is usually seen in patients with a prior history of myocardial infarction or severely reduced LV function. There is no such history in this patient. Educational Objective: Left atrial myxomas can present with signs and symptoms of mitral valve obstruction (diastolic murmur or "tumor plop"), rapidly worsening heart failure in otherwise young healthy individuals, or new onset atrial fibrillation. These can also present with systemic embolization, thereby causing acute arterial occlusion in otherwise healthy patients. 54% of people answered this question correctly.

Case 104 The following vignette applies to the next 2 items You are working in the emergency department of a small community hospital, when the paramedics bring in a 24-year-old man after being involved in a motor vehicle accident. At the scene of the accident, the medics noted that he had sustained a traumatic amputation of his right index finger. They were able to retrieve the severed finger and brought it to the emergency room. You provided the patient with initial wound care and achieved hemostasis with a compression dressing. Your hospital does not have a facility for digit replantation and reconstructive surgery, and you decided to transfer the patient to a nearby tertiary care center. Five minutes later, the nurse approaches you and asks, "How do you want me to preserve his severed finger?" Item 1 of 2 Which of the following is the most appropriate response? A. Fill it with sterile saline solution B. Wrap it in sterile gauze, moisten it with sterile water and place it in a sterile sealed plastic bag C. Wrap it in sterile gauze, moisten with sterile saline and place it in a sterile sealed plastic bag D. Wrap it in sterile gauze, moisten it with Ringers lactate and antibiotic solution and place it in a sterile sealed plastic bag E. Left ventricular thrombus Explanation: Recent technologic advances have led to successful replantations of various body parts including fingers, thumbs, ear, genitalia, and arms severed in various accidents. This has led to the preservation of the function and appearance of most body parts, as well as an improved quality of life for the patient. As a general rule, all amputated body parts should be retrieved and brought to the emergency room. Since the thumb or finger amputation can especially compromise the function of the hand, all attempts should be made to retrieve and replant the digit in the hope to preserve function. For transportation purposes, the amputated part should be wrapped in saline-moistened sterile gauze and sealed in a sterile plastic bag. (Choices A and D) It is not recommended to keep the amputated thumb or body part in Ringers lactate or an antibiotic solution. Educational objective: As a general rule, all amputated body parts should be retrieved and brought to the emergency room. For transportation purposes, the amputated part should be wrapped in saline-moistened sterile gauze and sealed in a sterile plastic bag. 65% of people answered this question correctly. Item 2 of 2 The appropriate step is taken. At your request, the nurse brings an insulated cooling container to transport the severed thumb. Before putting the plastic bag in the container, she asks you "What do you want to use to fill up the container?" Which of the following is the most appropriate response? A. Fill it with sterile saline solution B. Fill it with sterile water C. Fill it with ice only D. Fill it with saline moisten sterile gauze E. Leave it empty and just put the plastic bag in it Explanation:

After adequate steps have been taken to preserve the severed finger, it should be sealed in a plastic bag and placed on ice in a container. The severed part should never be placed directly on ice, as this could lead to injury to the vessels and other tissues. It should also not be immersed in water as this may make the replantation technically more difficult. Educational Objective: The severed part should be sealed in a sterile plastic bag and placed on ice for transportation purposes. 83% of people answered this question correctly. Case 105 The following vignette applies to the next 2 items A healthy 24-year-old Japanese woman comes to the physician because of a lesion on her left cheek. Two years ago, she already noticed a small mole on her left cheek, but this lesion has recently grown in size. She does not have any other complaints. Her social history is not significant. Vital signs are within normal limits. Examination shows a 4 cm dark brown, slightly elevated macule with slightly irregular borders on the left cheek. Item 1 of 2 Which of the following is the most likely diagnosis? A. Keratoacanthoma B. Pigmented basal cell carcinoma C. Seborrheic keratosis D. Malignant melanoma E. Verruca vulgaris Explanation: Malignant melanoma is a concern if a nevus increases in size and develops irregular borders. It can be confused with an atypical melanocytic nevus; however, this disease is infrequent in the Asian population. These lesions can appear spontaneously or develop over a previously atypical nevus. (Choice C) Seborrheic keratosis is rare before the age of 30. It also has a "stuck on" appearance, with a "warty" form. (Choice A) Keratoacanthoma is common on the cheek. It is a low-grade malignancy that pathologically resembles squamous cell carcinoma. It appears as a solitary, firm, round, skin-colored or reddish plaque that develops into a nodule with a central keratin plug. (Choice E) Verruca vulgaris has the same warty appearance as seborrheic keratosis, and is a welldemarcated lesion. (Choice B) Basal cell carcinoma is usually pink or red in color. It is more frequent in males, and it rarely appears before the age of 40. Educational Objective: It is important to recognize the presence of a melanoma, because excision can be curative if the lesion is identified early. Melanoma lesions are usually asymmetric, with color variegation, and usually more than 5 mm in diameter. 69% of people answered this question correctly. Item 2 of 2 Which of the following is the most appropriate statement in this patients management? A. A biopsy is not required since the clinical presentation is so classic B. An excisional biopsy is the next best step. C. Administer intralesional corticosteroids.

D. Observation and repeat follow-up in six weeks. E. Start therapy with interferon alfa-2b. Explanation: The sensitivity of a melanoma diagnosis by a dermatologist based on the history and physical examination is approximately 85%; however, false negatives have been reported in 15% of the time. Due to the significant amount of false negatives reported, an excisional biopsy is essential for confirmation of the diagnosis and staging of the lesion. Complete excision is the treatment of choice. (Choice E) Interferon alfa-2b is used as an adjuvant therapy in patients with a high risk of developing metastatic disease. Educational Objective: An excisional biopsy is the next best step in patients with a suspected malignant melanoma. 86% of people answered this question correctly. Case 106 A 27-year-old Caucasian male presents to the office with several days history of joint pain. He has never had such symptoms before. He denies any fever or chills. He has tried over-the-counter acetaminophen and ibuprofen, but these gave only little pain relief. His past medical history is insignificant. He smokes one pack of cigarettes daily and consumes 2-3 bottles of beer on weekends. He admits smoking marijuana occasionally but denies any injectable drug use. He has no known allergies. He has not traveled outside the country recently. He has been sexually active with two partners over the last six months, and he uses condoms occasionally. Physical examination reveals swelling and mild hyperemia of the right knee and left ankle. The range of movements is restricted in these joints due to pain. No rash is present. Arthrocentesis shows turbid yellow fluid and a leukocyte count of 9,000/mm3. Item 1 of 2 What is the best test to confirm the diagnosis? A. Joint fluid culture B. Joint fluid Gram stain C. Culture from joint fluid, rectum, urethra and oral cavity D. Microscopic examination of the joint fluid E. Chemistry studies of the joint fluid Explanation: Gonococcal arthritis is probably the most common cause of acute non-traumatic mono- and oligoarthritis in young healthy adults. It should be strongly suspected in patients with symptoms like those described in this scenario, especially if a history of unprotected sexual relationships is present. The typical synovial fluid leukocyte count in gonococcal arthritis is around 50,000 cells/mm3; however, cell counts below 10,000 cells/mm3 can be present in occasional cases. Confirmation of the suspected diagnosis is usually done by culturing the joint fluid and the mucosal surfaces, including the urethral, cervical, rectal and oral mucosa. More than 80% of patients with disseminated gonococcal infection have positive cultures from at least one of the mucosal sites. (Choice A) Synovial fluid culture alone yields positive results in less than 50% of patients with gonococcal arthritis. (Choice B) Joint fluid Gram stain is a reasonable initial approach in patients with suspected infectious arthritis, but it lacks sufficient sensitivity and specificity to establish the microbiological diagnosis in most cases. (Choice E) Chemistry studies of the joint fluid include the concentrations of glucose, lactate dehydrogenase, and protein. These have only limited value; a reduction in glucose concentration and elevation of LDH are consistent with a bacterial infection, but are not diagnostic or highly sensitive.

Educational Objective: In patients with suspected disseminated gonococcal infection, culturing the joint fluid and the mucosal surfaces, including the urethral, cervical, rectal and oral mucosa, should be done. Item 2 of 2 After running all the appropriate tests, you suspect an infectious cause of arthritis. Which of the following additional findings is most helpful in determining the cause of the infection? A. Sausage digits B. Back pain and restriction of movements C. Painful tendons along the ankle and toe joints D. Subcutaneous nodules E. Urticarial skin rash Explanation: Although disseminated gonococcal infection may manifest only as mono- or oligoarthritis, a number of unique, characteristic clinical features is sometimes present; these allow gonococcal infection to be distinguished from other forms of infectious arthritis. For example, tenosynovitis is a unique finding in patients with disseminated gonococcal infection; it is very unusual for other forms of infection. Several tendons are usually simultaneously inflamed, particularly at the wrist, fingers, ankle, and toes. Another characteristic feature is pustular or vesiculo-pustular skin rash that is often transient and disappears spontaneously in several days. (Choices A and B) Sausage digits and back pain may be present in patients with reactive arthritis (e.g., Reiters syndrome). (Choice D) Subcutaneous nodules are typical for patients with rheumatoid arthritis. (Choice E) Urticarial skin rash is not specific; it has been described in patients with hepatitis B who may present with symmetric polyarthritis-like symptoms but have non-inflammatory synovial fluid changes. Educational Objective: Tenosynovitis is a unique finding in patients with disseminated gonococcal infection; it is very unusual for other forms of infection. 35% of people answered this question correctly.

Case 107 A newborn baby girl is found to have a bulging sac covered with membranes in her lower back. She is diagnosed with myelomeningocele. The mother is concerned about the future development of complications secondary to her baby's disease. What is the most common extraneural complication of myelomeningocele? A. Involvement of the genitourinary system B. Involvement of the GI tract C. Involvement of the respiratory system D. Involvement of the cardiovascular system E. Fractures of the lower extremities Explanation:

In 80% of myelomeningocele cases, the lumbar region is involved. Almost all these patients will have bladder dysfunction, which can ultimately lead to upper urinary tract involvement and renal dysfunction. (Choice B) Children with S2-S3 involvement can have external anal sphincter dysfunction that can lead to fecal incontinence. This is a much less common complication than bladder involvement, and is specifically associated with S2-S3 involvement. (Choices C and D) Involvement of the respiratory and cardiovascular system is not associated with lumbosacral myelomeningocele. (Choice E) Lower extremity fractures occur in approximately 30% of myelomeningocele patients without a known history of trauma. These fractures may be associated with vigorous physical therapy. Educational Objective: Bladder dysfunction is a universal complication of lumbosacral myelomeningocele (excluding neurological deficits), followed by lower GI tract dysfunction and fractures of the lower extremities. Case 108 A 77-year-old Caucasian male is rescued from a house fire and is brought to the emergency department with altered nausea, headache, and altered mental status. His past medical history is significant for hypertension, hypercholesterolemia, and hypothyroidism. He is a non-smoker and nonalcoholic. His temperature is 36.7 C (98 F), blood pressure is 110/70 mmHg, pulse is 80/min, and respirations are 16/min. Which of the following is most helpful in diagnosing his underlying condition? A. Pulse oximetry B. Measurement of methemoglobin levels C. Measurement of arterial PO2 D. Measurement of venous PO2 E. Co-oximetry Explanation: The above patient is most likely suffering from carbon monoxide (CO) poisoning. The diagnosis is based on the history and increased carboxyhemoglobin levels. Carboxyhemoglobin levels are measured by co-oximetry of a blood gas sample. This spectrophotometric laboratory method can distinguish normal hemoglobin from carboxyhemoglobin. (Choice B) Carbon monoxide poisoning does not cause an increase in methemoglobin levels. (Choices C and D) In carbon monoxide poisoning, there is no effect on the amount of oxygen dissolved in the blood; therefore the PO2 of arterial or venous blood is unaffected. Furthermore, an arterial blood gas is not useful for diagnosing CO poisoning, except for identifying the metabolic acidosis from hypoxia. (Choice A) Pulse oximetry reveals normal oxygen saturation levels in patients with carbon monoxide poisoning, as it does not differentiate oxyhemoglobin from carboxyhemoglobin. Educational Objective: Co-oximetry is used for the diagnosis of carbon monoxide poisoning.

Case 109 A mother brings her healthy 5-year-old Caucasian son to your clinic because "he frequently wets his bed at night." She is asking you to comment about an article she had read recently on the internet stating that alarms are more effective than medication. What should you tell her? A. Relapse rates might be higher after stopping alarms than medications. B. Studies have shown that some types of alarms are better than others. C. Alarms are more effective if augmented by other behavioral approaches. D. Desmopressin and tricyclics are as effective as alarms. E. Both alarms and medications are equally effective. Explanation: First line management for primary nocturnal enuresis for children less than seven years of age is to reassure the patient parents that the child usually outgrows this phase and spontaneously recovers. Other options for treatment, however, are the use of alarms, along with behavioral therapy, such as limiting the childs fluid intake before bedtime. In the alarm method, a sensor is placed in the childs underwear or in the bed padding. Once the child voids and moisture is detected, the alarm is activated, waking up the child so that he could go to the toilet before he continues to empty his bladder. Although alarms have been shown to be less immediately effective than desmopressin use, the former is still more effective in preventing relapses. Alarms are more effective than treatment with tricyclics during and after treatment. (Choice A) The relapse rate of enuresis after treatment with alarms is lower than treatment with desmopressin and tricyclics. (Choice B) Alarms are equally effective. (Choice D) Desmopressin and tricyclics are less effective than alarms. Educational Objective: First line management for primary nocturnal enuresis for children less than seven years of age is reassurance the patients parents. Among different treatment modalities, alarms are most effective in inducing remission and preventing relapses. The success rate is higher if this method is combined with complex behavioral intervention. Although desmopressin has a fast induction, it is less effective in preventing relapses. Case 110 An 81-year-old Caucasian man is brought to the physician because he had a recent fall. He fell to the floor in the living room two days ago. He denies any dizziness or loss of consciousness prior to the event. He thinks that he only slipped, but his wife is concerned about the possibility of him falling again. His other medical problems include hypertension and a history of stroke ten years ago. He lives with his wife, who is five years younger and has no major medical problems. His vital signs are within normal limits. Examination shows no abnormalities. There is no motor deficit due to the stroke, and there are no bruises or other superficial lesions. Which of the following is the most appropriate screening test for this patient? A. Head computerized tomography (CT) scan with contrast B. Head CT scan without contrast C. Ambulatory cardiac monitoring (Holter monitor) D. Blood electrolytes, chemistry panel and complete blood count (CBC) E. Get up and go test Explanation: According to the guidelines published by the American Geriatric Society (AGS), British Geriatric Society (BGS) and American Academy of Orthopedic Surgeons, all elderly patients must be asked annually about any episode of falls. If a patient reports a single episode of fall, he should be evaluated with at least one postural stability test. Assessment of gait and balance is important because this determines the need for further evaluation. The "Get up and go" test is most commonly used to assess postural stability. In this test, the physician instructs the patient to stand up from a

chair without assistance, walk a short distance, turn around, return, and sit down again. If the patient is unsteady or has difficulties during the test, further evaluation is necessary. (Choice C) Holter monitoring, electrocardiogram or echocardiogram are only recommended if a cardiac cause (e.g., arrhythmias, possible cardiac syncope, myocardial ischemia) is suspected. (Choices A and B) Head MRI or CT scans are only needed when a neurologic problem is suggested by the history or physical exam. If there is evidence of significant head trauma, a head CT scan without contrast should be done. (Choice D) Blood chemistry or CBC are only indicated when a systemic condition is suspected. Educational Objective: All elderly patients must be asked annually if they had any history of falls. If a patient reports a fall, his gait and balance must be initially evaluated using the "get up and go test." If the patient successfully performs the test, no further evaluation is needed, unless there is evidence of a specific problem. Those who have recurrent falls may need a more extensive work-up. Case 111 A 30-year-old male with seizure-like activities is admitted to the intensive care unit. He is hypercalcemic, and his serum parathyroid hormone levels are also elevated. After the initial resuscitation, the patient is stabilized and is taken to the OR for parathyroidectomy. A single parathyroid adenoma is removed. Eight hours after the surgery, the nurse reports that he has developed facial asymmetry. Physical examination reveals facial asymmetry. He had a history of upper respiratory infection two weeks ago. What is the likely cause of his new symptoms? A. Metoprolol B. Hypercalcemia C. Hungry bone syndrome D. Complication of the anesthetic agent E. Get up and go test Explanation: This patient had hypercalcemia secondary to primary hyperparathyroidism, and subsequently underwent removal of the parathyroid adenoma. Following parathyroidectomy, serum calcium levels can fall (see below) and produce symptoms of hypocalcemia such as perioral numbness, muscle cramps, carpopedal spasm, positive Chvostek sign (ipsilateral contraction of facial muscles on tapping the angle of the jaw) and positive Trousseau's sign (rapid development of carpopedal spasm on occlusion of blood supply to the upper extremity). Severe hypocalcemia can cause mental status changes and seizures. Typically, the signs of hypocalcemia are bilaterally symmetrical. Hemifacial involvement is not typically seen; therefore, with a preceding history of upper respiratory infection and facial asymmetry, the most likely diagnosis of this patient is Bells palsy (lower motor nuclear involvement of the facial nerve). (Choice B) Typical features of hypercalcemia include nausea, vomiting, constipation, polyuria, and pleuritis. Mental status changes can occur with severely high calcium levels. (Choice C) Following parathyroidectomy (adenoma removal or removal of 3? parathyroid glands), serum calcium can fall by relative hypoparathyroidism or hungry bone syndrome. Relative hypoparathyroidism results from the suppression of normal parathyroid glands by high calcium levels, which fail to respond to low calcium levels following surgery. The suppression is transient and most patients recover in few days. Hungry bone syndrome is caused by the sudden withdrawal of parathyroid hormone in patients with severe hyperparathyroidism, causing an increased influx of calcium from the circulation into the bone. Preoperative risk factors for hungry bone syndrome are severe hyperparathyroidism, severe bone disease and vitamin D deficiency. All these conditions have a very high bone turnover. Postoperatively, when parathyroid hormone levels fall, the dynamics of bone turnover shifts from net efflux of calcium from bone to net influx of calcium into the bone. In hungry bone syndrome, the serum calcium typically falls to its nadir between 2 to 4 days following surgery. Eight hours following surgery is too early to get symptomatic hungry bone syndrome. Educational Objective:

Hypocalcemia following parathyroidectomy is caused by relative hypoparathyroidism or hungry bone syndrome. The signs of hypocalcemia are typically bilateral. Unilateral signs may suggest an alternative diagnosis. Hungry bone syndrome usually develops 2 to 4 days following surgery. *Extremely important question for USMLE step-3

Case 112 A 53-year-old Caucasian man comes to the emergency department because of a sudden onset of nausea, vomiting, and severe epigastric abdominal pain radiating to the back. He denies constipation, diarrhea and black stools. His other medical problems include hypertension, hypercholesterolemia and gastroesophageal reflux disease. He denies the use of tobacco, alcohol, or illicit drugs. His temperature is 37.8C (100F), blood pressure is 130/80mm Hg, pulse is 118/min and respirations are 20/min. The patients labs reveal: Hb 13.5g/dL Platelet count 180,000/cmm Leukocyte count 12,500/cmm Segmented neutrophils 79% Bands 1% Lymphocytes 20% Alkaline phosphatase 150 U/L Amylase 355 U/L Lipase 523 U/L (N=1-160) Which of the following is the most likely cause of his symptoms? A. Metoprolol B. Thiazide C. Prazosin D. Ramipril E. Amlodipine Explanation: Drug- induced pancreatitis accounts for 5% of cases of pancreatitis. Most cases of drug-induced pancreatitis are mild. The common conditions which involve the use of drugs (important for USMLE) that can cause pancreatitis are: Patient on diuretics - furosemide, thiazides Patient with inflammatory bowel disease - sulfasalazine, 5-ASA Patient on immunosuppressive agents - azathioprine, L-asparaginase Patient with history of seizures or bipolar disorder - valproic acid AIDS patient - think about didanosine, pentamidine Patient on antibiotics - metronidazole, tetracycline Educational Objective: Furosemide and thiazide diuretics can cause acute pancreatitis. 65% of people answered this question correctly. Case 113 A 15-year-old female presents to clinic for a routine examination. Her medical history is significant for a diagnosis of attention deficit hyperactivity disorder, which is reasonably well managed with methylphenidate, and an elective abortion performed one year ago. She is currently a sophomore in high school and has won many awards for her athletic accomplishments. As the visit comes to a close, she informs you that she has recently become sexually active with a male classmate and would like a prescription for oral contraception. She asks that the physician not inform her mother about her use of birth control. What is the most appropriate response on the behalf of the physician? A. Inform the girl that her parent(s) will not be informed because she is an emancipated minor B. Inform the girl that her parent(s) will not be informed because the visit is regarding contraception C. Inform the girl that her parent(s) will be informed because she is a minor D. Inform the girl that her parent(s) will be informed because she has had an abortion E. Inform the girl that her parent(s) will be informed because they pay for her health insurance

Explanation: As a physician, it can be difficult to balance the adolescent?s right to confidentiality against the parental right to oversee the child?s health. To address this conflict, most states have implemented laws that allow for a physician to provide care to adolescents without parental consent. These laws are typically limited to the management of certain issues, including pregnancy, contraception, sexually transmitted diseases, substance use, and emotional illness. Therefore, a teenaged patient is normally entitled to confidentiality regarding his or her use of contraceptives (Choice B). Most often, emancipated minors are teenagers who have successfully presented evidence to the court that they deserve emancipation, a special independent status legally equivalent to adulthood. Common examples include teenagers who are completely self-sufficient and no longer living with their parents and teenagers who are parents to one or more children. If the teenager in this situation were an emancipated minor, she would not need to obtain parental consent for any aspect of her health care. However, no evidence was presented to suggest that she is an emancipated minor (Choice A). If a teenager requires non-urgent treatment of any condition that is not exempt from parental consent laws (eg, surgical repair of a torn ligament), then her parents would be contacted because she is a minor (Choice C). However, if the visit is regarding contraception, she can be prescribed oral contraceptives without parental involvement. Since elective abortion is usually exempt from parental notification/consent laws, it would be illogical to presume that a history of abortion has any bearing upon a request for contraceptives (Choice D). In most circumstances, parents pay for the health care insurance for their children. However, this does not automatically entitle them to all information regarding the child?s medical care. Since it is possible that health insurance bills will include descriptions of the services rendered, physicians should make every effort to preserve confidentiality when submitting insurance claims while still charting and billing correctly (Choice E). Educational Objective: Most states have implemented laws that allow for a physician to provide certain types of medical care to adolescents without parental consent. Exempted areas typically include pregnancy, contraception, sexually transmitted disease, substance use, and emotional illness. Case 114 A 60-year-old male comes to see you in the office for a physical evaluation prior to undergoing right total knee replacement. He does not have any complaints, except for the pain in his right knee. He has a past medical history of hypertension, degenerative joint disease, idiopathic thrombocytopenic purpura, and chronic obstructive pulmonary disease (COPD). His medications include hydrochlorothiazide, celecoxib, Advair Diskus (fluticasone and salmeterol), and albuterol inhaler, as needed. His lung disease has been under good control, and he has not had any flare-ups of COPD in the last nine months. He has a 40-pack-year history of smoking, and continues to smoke one pack of cigarettes a day. His temperature is 36.7C(98F), blood pressure is 120/76 mm Hg, pulse is 80/min, and respirations are 16/min. Oxygen saturation is 94% on room air. His general physical examination, including the lung exam, is unremarkable. Which of the following should be done next before clearing the patient for the planned surgery? A. Obtain a pulmonary consultation. B. Obtain an arterial blood gas analysis. C. No other tests are indicated. D. Obtain a pulmonary function test. E. Obtain a pulmonary function test and an arterial blood gas analysis. Explanation:

It is important to perform preoperative pulmonary risk assessment in all patients who will be undergoing major surgical procedures. Risk stratification identifies potential moderate to high risk patients, and helps in reducing postoperative pulmonary complications and length of stay in the hospital. A detailed history and physical examination is the first step in the evaluation of these patients. It is done to specifically look for the known risk factors for postoperative pulmonary complications. In a patient with a known, stable, chronic obstructive pulmonary disease and no evidence of exercise intolerance, routine testing with arterial blood gas analysis, pulmonary function testing, or spirometry is not indicated. (Choice A) Pulmonary consultation is not indicated in a patient with stable chronic obstructive pulmonary disease. (Choice B) A preoperative arterial blood gas analysis is recommended in patients who will be undergoing lung resection, and in patients with a history of tobacco use or unexplained dyspnea who will be undergoing coronary artery bypass surgery or upper abdominal surgery. An arterial blood gas analysis in other situations does not provide any additional information than what is already available from the clinical evaluation. (Choices D and E) Pulmonary function testing should not be done routinely in all patients with chronic obstructive pulmonary disease who will be undergoing abdominal or lower extremity surgeries. They should only be obtained in patients with chronic obstructive pulmonary disease or asthma if the clinical examination reveals airflow limitation, or suggests that the patients are not at their normal baseline lung functions. Educational Objective: A detailed history and physical examination aimed at detecting known risk factors for postoperative pulmonary complications is the first step in the preoperative evaluation of all patients who will be undergoing major surgical procedures. 22% of people answered this question correctly. Case 115 The following vignette applies to the next 2 items A 32-year-old gravida 3, para 2 Caucasian woman at 40 weeks of gestation was admitted to the labor and delivery unit after the spontaneous rupture of membranes. She had regular prenatal care and a normal ultrasonogram at 20 weeks of gestation. After an unremarkable labor she vaginally delivered a son weighing 3440 grams (7 lbs, 9 oz). Apgar scores were 8 and 9, respectively. Physical examination of the child reveals a palpably distended bladder and is otherwise unremarkable. The boy?s urinary stream is very weak, but he is able to pass stool. Ultrasound evaluation demonstrates bladder distention and bladder wall thickening. Item 1 of 2 What is the most likely diagnosis? A. Dehydration B. Hypospadias C. Nephrolithiasis D. Posterior urethral valves E. Prune belly syndrome Explanation:

Posterior urethral valves (Choice D) are predominantly found in males and are the most common cause of severe obstructive uropathy in children. The abnormal development of the valves in utero can obstruct urinary flow, leading to detrusor hypertrophy and, eventually, vesicoureteral reflux and hydronephrosis. Hallmarks of PUV include a distended bladder and a weak urine stream. Dehydration (Choice A) in children is characterized by a prolonged capillary refill time, poor skin turgor, tachycardia and tachypnea. This child?s physical examination was normal aside from the distended bladder. Hypospadias (Choice B) is an abnormality of urethral development in which the opening of the urethra is located on the ventrum of the penis proximal to the tip of the glans. The urinary stream may be deflected downward, but the quantity of urine output is usually normal. Nephrolithiasis (Choice C) occurs most often young and middle-aged adult males. Symptoms may include severe pain, cramping, nausea, and hematuria. Prune belly syndrome (Choice E) is characterized by a multitude of renal, ureteral, and urethral abnormalities in the neonate. Obstruction and upper urinary tract dilatation are often observed. Abdominal musculature is underdeveloped, which leads to constipation and a weak cough. Educational Objective: Posterior urethral valves are predominantly found in males and are the most common cause of severe obstructive uropathy in children. Hallmarks of PUV include a distended bladder and a weak urine stream. 90% of people answered this question correctly. Item 2 of 2 Which of the following postnatal procedures is most commonly used to definitively diagnose this condition? A. Voiding cystourethrogram B. Radiograph of kidneys, ureter, bladder C. Abdominal CT scan D. Laboratory evaluation of electrolytes E. Karyotyping Explanation: Voiding cystourethrogram (VCUG) (Choice A) is a radiographic examination of the bladder and lower urinary tract. The bladder is filled with contrast material by catheter and multiple radiographic images of the bladder and urethra are obtained as the patient empties the bladder. VCUG is especially helpful in evaluating young children for vesicoureteral reflux and posterior urethral valves. Radiograph of the kidneys, ureter, and bladder (Choice B) are often used to make the diagnosis of prune belly syndrome based on the classic appearance of the bowels hanging over the lateral edge of the abdominal wall. Evaluation of electrolytes (Choice C) could provide some evidence of dehydration if that was a concern in this child. Karyotyping (Choice D) evaluates for conditions such as Klinefelter or Turner syndrome. Abdominal CT scan (Choice E) would be the appropriate imaging modality for evaluating nephrolithiasis.

Educational Objective: Voiding cystourethrogram (VCUG) is a radiographic examination of the bladder and lower urinary tract that is especially helpful in evaluating young children for vesicoureteral reflux and posterior urethral valves. 91% of people answered this question correctly. Case 116 The following vignette applies to the next 2 items You are working in the emergency department (ED) on New Year?s Eve, when the paramedics bring in a homeless man. They found him lying on the side of a street. He was minimally responsive in the ambulance. In the ED, his core body temperature is 32?C(90F), systolic blood pressure is 70 mmHg, heart rate is 40/min and respiratory rate is 6/min. His oxygen saturation is 90% on 5 liters of oxygen. On examination, he remains unresponsive to all verbal stimuli, but he withdraws his limbs on painful stimuli. His breath smells of alcohol. His pupils are equal, round, normal in size, and have a sluggish reaction to light. His lung examination is unremarkable. Cardiovascular examination reveals bradycardia with an irregular pulse rate. Neurological examination is difficult to perform, but reveals marked hyporeflexia throughout. An initial electrocardiogram reveals sinus bradycardia with frequent premature ventricular complexes. Item 1 of 2 Which of the following is the most appropriate initial step in the management of this patient? A. Lidocaine B. Atropine C. Bretylium D. Intravenous fluids E. Endotracheal intubation Explanation: The patient in the above vignette is suffering from moderate hypothermia. Some factors associated with a higher risk of hypothermia include an older age, homelessness, alcohol abuse, and certain medical conditions such as hypothyroidism, adrenal insufficiency, sepsis, and malnutrition. Most patients with mild hypothermia (core body temperature between 32?C to 35?C) present with confusion, ataxia, slurred speech, tachycardia, and tachypnea. Patients with moderate hypothermia (core temperature between 28?C to 32?C) have marked central nervous system depression, hypoventilation, hypotension, bradycardia, hyporeflexia, and cardiac conduction abnormalities. Severe hypothermia (core temperature less than 28?C) can cause marked hypotension, areflexia, coma, malignant ventricular arrhythmias (ventricular fibrillation), and asystole. The first step in the resuscitation of such patients is to protect and secure the airway with immediate or early endotracheal intubation. The patient in the above vignette has marked hypoventilation and altered mental status. An early endotracheal intubation should be performed to prevent aspiration and respiratory arrest. (Choices A and C) Antiarrhythmic drugs are not indicated in patients with frequent PVCs. They occur as a response to hypothermia and usually resolve with active rewarming of the patient. (Choice B) Bradycardia is also due to a physiologic response to hypothermia and resolves with active rewarming. Atropine or cardiac pacing is not required unless the bradycardia persists despite specific treatment. (Choice D) An adequate airway should be established in all patients before performing any other resuscitative measures.

Educational Objective: Airway control should remain the first priority in all patients during resuscitative efforts. remember the "ABC's" (Airway, Breathing and Circulation) in any resuscitation effort. 71% of people answered this question correctly. Item 2 of 2

Always

The appropriate step was taken for the patient. His clinical status remains unchanged. His blood pressure is 72/42 mmHg and heart rate is 42/min. An EKG monitor reveals persistent sinus bradycardia with frequent PVCs. Which of the following is the most appropriate next step in the management of this patient? A. Lidocaine B. Intravenous fluids C. Active rewarming D. Bretylium E. Endotracheal intubation Explanation: The next important step in the management of this patient is to support and improve his blood pressure with aggressive intravenous hydration. Administration of intravenous fluids (and inotropic agents, if required) will help improve his cardiac output, blood pressure, and perfusion to various organs. After the initial aggressive intravenous hydration, warm intravenous fluids can be used to support blood pressure and provide active internal rewarming. (Choices A and D) There is no evidence of any malignant ventricular arrhythmia (ventricular fibrillation) in this patient. Antiarrhythmic drugs are not indicated for the treatment of frequent PVCs. (Choice C) Active rewarming reverses the physiologic and metabolic responses to hypothermia, and is the specific treatment for patients with moderate to severe hypothermia; however, the initial management of patients should be aimed towards resuscitation and support of airway, breathing, and circulation. Educational Objective: Aggressive intravenous hydration should be used to support low blood pressure in patients with hypothermia and hypotension. 37% of people answered this question correctly. Case 117 A 56-year-old female was found to have a TSH level of 0.2 mU/ml on routine lab testing. She denies symptoms suggestive of thyrotoxicosis. She reports good energy level and no change in body weight. She has mild hypertension, for which she is on hydrochlorothiazide. Her family history is negative for any thyroid disorder. She denies smoking or alcohol use. She does not have allergies to any medications. She had her menopause about three years ago. She has never received hormone replacement therapy. She takes adequate amounts of calcium and vitamin D supplementation. Bone mineral density using dual photon absorptiometry was within normal range about six months ago. Examination is unremarkable, including examination of the thyroid gland. Free T4 levels and free T3 are well within normal limits. What is the next best step in this patient?s care? A. Radioactive iodine uptake B. Start methimazole C. Start propylthiouracil

D. Recheck thyroid functions test in 6-8 weeks E. Subtotal thyroidectomy Explanation: The patient has subclinical thyrotoxicosis, which is defined as suppressed TSH levels along with normal thyroid hormone levels. The most common causes of subclinical thyrotoxicosis are treatment with levothyroxine, nodular thyroid disease, Graves' disease, and thyroiditis. Subclinical thyrotoxicosis induced by levothyroxine is simply treated by reducing the dose. In some cases, the etiology cannot be determined, and TSH becomes normal if repeated in a few weeks. Patients who have mildly suppressed TSH, no symptoms, normal heart rhythm, and normal bone density are not intensively investigated because no treatment is necessary, and there is a high chance of normalization of TSH levels. Repeating TSH after 6-8 weeks is generally performed. (Choices A, B, C, and E) The above patient did not have any indication for aggressive intervention. In symptomatic patients (e.g. persistent fatigue), an antithyroid drug in a small dose (methimazole 5-10 mg/d) is generally started after performing a radioactive iodine uptake and scan. If the symptoms improve with antithyroid drugs, radioactive iodine ablation is usually performed. Patients with multinodular goiter have a 5-10%/year chance of becoming overtly thyrotoxic; therefore, treatment in such patients is warranted. Patients with atrial fibrillation and low bone densities also require treatment. Educational Objective: Patients who have mildly suppressed TSH but normal T4 and T3, no symptoms, normal heart rhythm, and normal bone density are not intensively investigated because no treatment is necessary and there is a high chance of normalization of TSH levels. 79% of people answered this question correctly.

Case 118 A relatively healthy 71-year-old Asian man comes to the office for a routine follow-up visit after a recent discharge from the hospital. During the last visit, he was diagnosed with benign prostatic hypertrophy with a slight elevation of PSA. He subsequently underwent TURP (transurethral resection of the prostate), and his prostatic specimen histopathology results came back as prostate adenocarcinoma in situ. He has a history of mild COPD and hypertension. He has smoked one pack of cigarettes daily for 38 years. Which of the following is the most appropriate next step in his care? A. Radical, suprapubic prostatic resection with lymph node exploration B. Pelvic radiation and bilateral orchiectomy C. Pelvic radiation and estrogen therapy D. Chemotherapy E. Anti-androgen therapy Explanation: Suprapubic resection of the prostate is the currently accepted therapy for patients whose prostate cancer was diagnosed through TURP, needle biopsy, or cytology. This procedure is accompanied by lymph node resection, which can be preceded by sentinel lymph node identification through technetium radio labeling for a higher yield. (Choices B and C) Pelvic radiation can cause serious local lesions (such as strictures) and bowel or rectal damage. It is reserved for more advanced stages of the disease.

(Choices D and E) Anti-androgen agents and chemotherapy have not increased survival when used as adjuvants in this type of patient. Hormonal therapy with bicalutamide is promising (and is being used already in Europe) but is not yet FDA-approved. These are usually reserved for advanced stages of the disease. Educational Objective: Patients with an incidental finding of prostate cancer in situ through TURP or biopsy need further surgical resection of the gland through a suprapubic approach. Radiotherapy, estrogens, and chemotherapy are measures reserved for more advanced stages of the disease. 42% of people answered this question correctly. Case 119 A 68-year-old Caucasian man is brought to the emergency department by her daughter because of altered mental status. The daughter reports that he lives alone and has a history of diabetes mellitus type 2 and hypertension. He takes aspirin, enalapril and glipizide. His blood pressure is 100/60 mm Hg, pulse is 100/min and respirations are 20/min. He is not febrile. Physical examination reveals dry mucus membranes, no jugular venous distension, clear lung fields and normal first and second heart sounds. The abdomen is soft, non-tender, and non-distended. Neurologic examination reveals a drowsy patient who is disoriented to time and space. He is barely communicative. There are no meningeal or focal signs. His laboratory tests reveal: CBC Ht: 44% Platelet count: 300,000/cmm Segmented neutrophils: 70% Monocytes: 8% Serum chemistry Serum Na: 150 mEq/L Chloride: 120 mEq/L BUN: 36 mg/dL Calcium: 9.7 mg/dL Serum ketones: Negative MCV: 90fl Leukocyte count: 10,000/cmm Lymphocytes: 22%

Serum K: 4.6 mEq/L Bicarbonate: 20 mEq/L Serum creatinine: 1.5 mg/dL Blood glucose: 800 mg/dL

The electrocardiogram reveals sinus tachycardia. He is started on insulin therapy. Which of the following is the most appropriate fluid solution to provide adequate hydration to this patient? A. 0.9% sodium chloride solution (normal saline) B. 0.45% sodium chloride solution (half normal saline) C. Hypertonic solution D. Ringer?s lactate E. Free water Explanation: Hyperosmolar, hyperglycemic state (HHS) is a complication usually experienced by poorly controlled type 2 diabetics. It is characterized by altered mental status, glycemic levels over 800 mg/dL, bicarbonate levels higher than 15 mEq/L, effective osmolality higher than 320 mOsm/kg, and the presence of minimal amount of serum ketones. Hydration and insulin therapy aim to correct the electrolyte imbalances and hyperglycemia. Hyperglycemia can cause either hyponatremia or hypernatremia in uncontrolled diabetes mellitus. Hyperglycemia increases serum osmolality, which results in osmotic water movement out of the cells; therefore dilutional hyponatremia (pseudohyponatremia) is usually seen. On the other hand, glucosuria-induced osmotic diuresis results in water loss in excess of sodium and potassium; this may raise the plasma sodium concentration and plasma osmolality, unless there is a compensatory increase in water intake.

In order to adequately correct the hypo/hypernatremia in HHS, the corrected value of serum sodium must be calculated. This is done by adding 1.6 mEq/L for every 100 mg/dL of glucose over the baseline (100 mg/dL) to the total sodium value. The patient?s sodium level is 150 mEq/L, while his glucose level is 800 mg/dL. Using the abovementioned concepts, the calculation for the patient?s corrected serum sodium value is as follows: (1.6 x 7) + 150 = 161.2 mEq/L The corrected sodium level indicates severe hypernatremia, which should be treated with hydration therapy using half-normal saline solution. (Choices A and C) Hyponatremia is treated with normal saline solution or hypertonic solution (for severe cases). (Choice D) Ringers lactate has a similar osmolality and sodium content as normal saline solution. (Choice E) Free (oral) water may be used in the treatment of mild hypernatremia. This patient?s severe hypernatremia warrants urgent treatment with intravenous half-normal saline solution. In addition, although intravenous D5W (free water with dextrose) can be used in patients who need correction of severe hypernatremia, it cannot be used in hyperglycemic patients (such as in this case). Educational Objective: The treatment of hyperosmolar hyperglycemic state involves hydration and insulin therapy. To adequately hydrate the patient, the corrected level of serum sodium must be calculated. Only those patients with hyponatremia or hypovolemic shock should receive normal saline. Those with normal sodium levels or hypernatremia must be hydrated with half-saline solution. Most studies show that the usual water deficit is approximately 9 liters. 35% of people answered this question correctly. Case 120 The following vignette applies to the next 2 items A 47-year-old Caucasian man is brought to the emergency department of a small rural clinic. He was hunting in the woods with a group of friends 45 minutes ago, when he was accidentally shot by another hunter. He complains of abdominal pain, shortness of breath and palpitations. The symptoms are progressively getting worse. He does not use tobacco, alcohol, or illicit drugs. His temperature is 36.1 C (97 F), blood pressure is 90/60 mm Hg, pulse is 118/min and respirations are 26/min. His pulse oximetry reading is 94% at room air. The abdomen is mildly distended. Bowel sounds are absent. Rigidity and rebound tenderness are present. There is a gunshot wound in the left upper quadrant, with no exit orifice. A bedside chest x-ray reveals clear lung fields. Item 1 of 2 Which of the following is the most appropriate course of action? A. Perform a Diagnostic Peritoneal lavage (DPL) B. Perform a Focused abdominal sonogram (FAST) C. Order an Abdominal CT Scan with oral contrast D. Call the nearest surgical reference center to prepare for laparotomy E. Perform laparoscopy Explanation: Approximately 85% of abdominal wall gunshot wounds (GSWs) penetrate the abdominal cavity, and approximately 95% will require surgery. This patient was injured with a long arm bullet (probably a

rifle). The force of the impact is enough to assume penetration through the abdominal wall, unless there is evidence of only a superficial or tangential laceration during the physical examination. Furthermore, the presence of peritoneal signs in the patient is an absolute indication for an urgent laparotomy. Every effort must be made to transfer him immediately to the operating room. Other absolute indications for an urgent laparotomy in patients with acute abdominal trauma are: major vascular injury, hollow viscus perforation, hemodynamic compromise, hemo- or pneumoperitoneum, diaphragmatic lesions, and spinal cord injury. (Choices A, B, and C) There is no need to perform further ancillary procedures to investigate the extent of intra abdominal injuries in this patient because he already has an absolute indication for an urgent laparotomy. (Choice E) Laparoscopy is used to assess tangential abdominal GSWs. It is not recommended in the management of patients with abdominal GSWs with absolute indications for an urgent laparotomy. Educational Objective: Most abdominal gunshot wounds will require laparotomy for further exploration and immediate management of intra abdominal injuries, unless the wounds are only superficial or tangential to the skin. The absolute indications for an urgent laparotomy are: presence of peritoneal signs, spinal cord damage, intra abdominal vessels or bladder damage, and hollow viscus perforation. If the patient is hemodynamically unstable or has lost a great amount of blood, the procedure must be done immediately. 67% of people answered this question correctly. Item 2 of 2 The patient is reevaluated 20 minutes later. His abdominal symptoms are stable. His shortness of breath has improved after nebulizations, but he continues to have palpitations. He has received one and a half liters of normal saline solution. His blood pressure remains at 90/60 mm Hg, pulse is 100/min and respirations are 22/min. His pulse oximetry reading is 97% at room air. Examination reveals clear lung fields. Abdominal findings have not changed. Which of the following is the most appropriate immediate step in the management of this patient? A. Cross type at least 2 units of packed red blood cells B. Perform an abdominal CT scan with oral contrast C. Transfer the patient to the nearest surgical facility, which is 20 minutes apart by helicopter D. Observe and reevaluate clinically every two to four hours E. Explore the abdominal wound under local anesthesia Explanation: Even though the patient remains stable, he still needs an urgent laparotomy. If the procedure cannot be done because of lack of resources or an experienced surgeon, the patient must be transferred immediately to another facility. Most rural areas accomplish immediate patient transfers through air transportation. (Choice A) Although the patient may need a blood transfusion as indicated by the failure of his blood pressure to increase despite adequate intravenous hydration, the transfusion should not delay his transfer. Furthermore, most surgical centers consider blood transfusion in trauma patients when the systolic blood pressure is less or equal to 70 mmHg, because transfusing at higher levels of blood pressure has been shown to increase the rate of bleeding. For these reasons, the transfusion may be given to the patient in the operating room. (Choices B and D) There is no need to periodically observe the patient or perform further ancillary tests/procedures. He requires an urgent laparotomy.

(Choice E) Exploration of the abdominal wound under local anesthesia is done when a superficial injury is suspected, and when the case of penetrating abdominal trauma is due to stabbing. Educational Objective: Early transportation can be life-saving for patients with penetrating abdominal trauma secondary to gunshots. Transfer to a regional trauma center is required if there is no surgeon or operating room available, if the patient has multiple injuries, or if he needs procedures that are not available (e.g., hemodialysis, organ transplantation, neurosurgery, etc.). Air transport (life flight) is recommended for patients who are potentially unstable, or who are in rural areas or places where ground transportation may take more than 20 minutes. 48% of people answered this question correctly. Case 121 A healthy 26-year-old Caucasian male comes to the office for a routine health maintenance examination. He recently got married, and he is currently planning to have children. His younger brother was recently diagnosed with cystic fibrosis, and he wants to know his chances of carrying the abnormal allele. Which of the following is the best response? A. Proceed with treatment B. He has 1/4 chance of being a carrier C. He has 3/4 chance of being a carrier D. He has 4/4 chance of being a carrier E. Explore the abdominal wound under local anesthesia Explanation: Cystic fibrosis is an autosomal recessive disease, and it is manifested when a person carries two mutant alleles. Parents of affected children are heterozygous carriers. The chance of being a carrier (i.e., the chance of carrying one mutant allele) is 1/2, the chance of having the disease (i.e., the chance of having two mutant alleles) is 1/4, and the chance of having both normal alleles is 1/4. Overall, 1/4 offspring will be phenotypically abnormal and 3/4 will be phenotypically normal. Educational Objective: A sibling of a cystic fibrosis patient has a 1/2 (50%) chance of being a carrier. 54% of people answered this question correctly. Case 122 An 8-year-old boy is brought to the pediatrician because he is complaining of a subjective fever, sore throat, headache, and stomach upset. His symptoms began yesterday. His grandmother, who cares for him after school while his mother works, accompanies him. Physical examination reveals temperature of 39.3C (102.8F), erythema and edema of the pharynx and uvula, pharyngeal exudate, and anterior cervical lymphadenopathy. Rapid strep test is positive. The grandmother indicates that the boy has great difficulty swallowing pills, and that an injection of penicillin would be preferred. When asked if she has the authority to permit treatment, the grandmother appears confused and replies that she has never been asked such a question before. What is the most appropriate means of handling this situation? A. Proceed with treatment B. Ask boy for consent to treat C. Ask grandmother for consent to treat D. Call mother at work for consent to treat

E. Petition court for consent to treat Explanation: Unless the grandmother has legally adopted the child or has been designated as his legal guardian, she does not automatically have the authority to permit his treatment. Therefore the mother must be called at work (Choice D) to obtain permission to treat. In the future, the mother can submit a signed statement to the pediatrician authorizing the grandmother to consent to medical care. This statement should be kept with the child?s medical records. Although some physicians would simply proceed with the injection (Choice A), this course of action is risky since consent has not been given. Failing to obtain informed consent before performing a test or procedure is technically considered battery, a form of assault. Since the boy is a minor, he is not able to authorize treatment (Choice B). Were he an adolescent, he could give consent for treatment of sexually transmitted diseases, substance use, and emotional illness. Since there is no paperwork stating that grandmother has permission to treat, her consent (Choice C) would be of no legal benefit. Petitioning the court for consent to treat (Choice E) is only appropriate in serious situations in which the child?s parent or legal guardian refuses to allow a treatment (e.g., blood transfusions) that the physician considers medically necessary. Educational Objective: Permission to treat a child can only be granted by the parent or legal guardian. As an exception to this rule, adolescents are typically allowed to give consent for their own care in regards to pregnancy, contraception, sexually transmitted diseases, substance use, and emotional illness. 58% of people answered this question correctly. Case 123 A 45-year-old Caucasian man comes to the office with complaints of low-grade fever and abdominal distention for the past five days. He denies any history of cough, shortness of breath, nausea, vomiting or any change in his bowel habits. He denies any past medical history. He is an IV drug abuser. He has a history of multiple sexual partners with unprotected sexual intercourse in the past. His temperature is 38.5?C (101.4F), blood pressure is 126/82 mmHg, pulse rate is 96/min, and respiratory rate is 16/min. The general physical examination reveals a thin cachectic man with a distended abdomen and mild icteric sclerae. His lungs are clear on auscultation, with slightly decreased breath sounds at both lungs bases. The cardiovascular examination is within normal limits. He has a distended abdomen with a vague, generalized tenderness and hepatosplenomegaly. Shifting dullness and fluid thrill are elicited during the abdominal examination. There is no guarding or rebound tenderness; the bowel sounds are normoactive on auscultation. Further workup of the patient in the hospital reveals a chronic hepatitis C infection. What is the next best step in the management of this patient? A. Interferon therapy B. Hepatitis B vaccination C. HIV testing D. Abdominal paracentesis E. Ultrasound of the abdomen

Explanation: The patient in the above vignette has ascites due to chronic hepatitis C and cirrhosis of the liver. The presence of fever, generalized abdominal tenderness and distention in such a patient is highly suspicious for the presence of spontaneous bacterial peritonitis. Spontaneous bacterial peritonitis (SBP) is defined as an ascitic fluid infection in the absence of an evident intra-abdominal source. It is almost always seen in patients with advanced cirrhosis and clinically apparent ascites. The most common symptom in patients with suspected SBP includes fever, abdominal pain and/or tenderness, altered mental status or worsening of the underlying hepatic encephalopathy; however, the clinical presentation of SBP can be very subtle, and early diagnosis and treatment is important to prevent major morbidity and mortality in such patients. Abdominal paracentesis and ascitic fluid analysis is required in all patients for the diagnosis of SBP. Ascitic fluid should be sent for cell count and differential, protein and albumin concentration, glucose and LDH concentration, Gram staining and cultures. The diagnosis of SBP is usually made by a polymorphonuclear leukocyte count of greater than 250 cells/cubic mm or with positive ascitic fluid cultures. (Choice A) Treatment with interferon can be used to prevent the development of cirrhosis and its complications in patients with chronic hepatitis C. The patient appears to have advanced cirrhosis from underlying hepatitis C infection at presentation. Therapy with interferon would probably not be helpful at this point. (Choices B and C) The patient should eventually receive hepatitis B vaccination and also should be tested for co-infection with HIV; however, his clinical presentation warrants an initial abdominal paracentesis to rule out spontaneous bacterial peritonitis. (Choice E) Ultrasound of the abdomen is not necessary for the management of this patient at this point. Educational Objective: Abdominal paracentesis should be performed in all patients with new-onset ascites, or with signs of clinical deterioration in patients with ascites and liver cirrhosis. 57% of people answered this question correctly. Case 124 A 66-year-old male comes to the emergency department because of bright red blood per rectum. He says he feels weak and dizzy. The bleeding started this morning and was painless. He denies the use of any medications, trauma, or recent surgery. The physical exam is unremarkable, except for an ejection systolic murmur radiating to the carotids in the second right intercostal space. His blood work reveals a hemoglobin level of 9.7 g/dL. His hemoglobin level three months ago was 13 g/dL. What is the most likely diagnosis? A. Colon cancer B. Diverticulitis C. Ulcerative colitis D. Angiodysplasia E. Hemorrhoids Explanation: Massive colonic bleeding classically has been attributed to diverticulosis, but recent evidence suggests that angiodysplasia, also known as vascular ectasia, is also common. These two entities

frequently coexist, and exact identification of the bleeding source may require a combination of endoscopic and radiographic methods. Before the advent of angiography, angiodysplasia was not recognized as a source of colonic bleeding. The cause of angiodysplasia is not known, but may be related to degenerative changes associated with aging and to intramuscular hypertrophy that obstructs submucosal veins. There has been an association between aortic stenosis and angiodysplasia. (Choice B) Diverticulosis is also a common cause of massive colonic bleeding, and this has been attributed to ruptured vasa recta either at the apex or neck of a diverticulum. The bleeding is generally painless and can be massive. Diagnosis can be made by colonoscopy. However, diverticulitis is an inflammatory condition of the left colon and is usually seen in the elderly. It usually presents with a fever, abdominal pain, and mild bleeding. Diverticulitis can also cause bleeding as a result of superficial mucosal ulcerations, but usually the bleeding is mild. The condition is diagnosed with a CT scan and is readily treated with antibiotics. A change to a diet rich in fiber is recommended to prevent recurrences. A colonoscopy is contraindicated in the acute condition. (Choice A) Cancer of the colon usually causes occult rather than massive gastrointestinal bleeding. Colon cancers of both the right and left side can cause a trace amount of bleeding and are often present with anemia. The bleeding is generally painless. A guaiac stool test may be the first indication of an underlying colon cancer. (Choice C) Ulcerative colitis is an inflammatory bowel disorder which can present with lower gastrointestinal bleeding. The bloody diarrhea may be associated with mucus and mild abdominal cramping. The diagnosis is made by endoscopy and barium enema. Surgery is the definitive treatment for ulcerative colitis. Complications of untreated ulcerative colitis include toxic megacolon, colonic perforation, and cancer. (Choice E) Hemorrhoids can cause mild bleeding per rectum. In the elderly population who present with a lower gastrointestinal bleed, even if hemorrhoids are present, colon cancer must be ruled out. Hemorrhoids may be visible externally and may present with painless or painful rectal bleeding. Educational Objective: The most common cause of massive lower gastrointestinal bleeding in elderly patients is angiodysplasia or diverticulosis. Remember the association between aortic stenosis and angiodysplasia. 68% of people answered this question correctly. Case 125 A 4-year-old boy complaining of an earache was brought by his mother to your clinic three days ago. According to the mother, the child has had a cold for a week, fever for 24 hours, and pain in the left ear for eight hours. The boy was extremely irritable and difficult to examine during that visit. His temperature was 38.8C (102F). On otoscopy, you observed a bulging, red, tympanic membrane (TM), which did not move with insufflation. The right TM and the rest of the physical examination was normal. The patient had not received any antibiotics for 30 days. At that time, you prescribed amoxicillin 40-mg/kg po qd for 7 days. The mother returns with her son today, and states that he still has an earache and fever. His temperature is 38.5C (101.3F). The left TM is now pink, but still bulging and immobile. The rest of the boy's examination is unchanged. What is the best next step in this patient's management? A. Discontinue amoxicillin, begin IM ceftriaxone, and refer the boy to an otolaryngologist for urgent tympanocentesis. B. Discontinue amoxicillin and start amoxicillin/clavulanate for 7 days. C. Continue the current treatment and reassure the mother that her son will improve within 7 days. D. Continue amoxicillin and refer the patient to an otolaryngologist for urgent tympanostomy and tubing.

E. Discontinue amoxicillin 40-mg/kg po qd and start amoxicillin 90-mg/kg po qd for 7 days. Explanation: This patient has been appropriately treated for acute otitis media (AOM) with the usual dose of amoxicillin; however, he currently presents with an initial clinical treatment failure, characterized by: persistent ear pain, fever, bulging TM, or otorrhea three days after therapy. Current recommendations for treatment failure in an AOM patient who has not received antibiotics in the month prior to the initiation of treatment are high doses of amoxicillin/clavulanate or certain second or third generation cephalosporins. The rationale is to enhance activity against penicillin-resistant S. pneumoniae (drug resistant S. pneumoniae or DRSP) found in an increasing percentage (30-60%) of cases of pneumococcal AOM in the US. (Choice A) This would be an appropriate choice if the patient has an initial clinical treatment failure and has received antibiotics in the month prior to beginning the current treatment. (Prior antibiotics in the month preceding AOM increases the likelihood that AOM is due to DRSP.) Under these conditions, the patient would also have been initially treated with high dose amoxicillin, high dose amoxicillin/clavulanate, or certain second or third generation cephalosporins. These regimens are reported to be more effective against DRSP than the usual dose of amoxicillin. IM ceftriaxone is even more effective against DRSP. Moreover, immediate tympanocentesis allows culture and sensitivity testing which can provide invaluable guidance in selected difficult cases not responding to empiric treatment. (Choice C) This approach might be successful, since acute otitis media has a favorable natural history regardless of antibiotic use. A meta-analysis of studies conducted from 1966 to 1992 concluded that the overall rate of spontaneous resolution of acute otitis media was close to 80 percent; however, this approach is not recommended because of the more recent rising prevalence of DRSP. (Choice D) Tympanostomy and tubing is generally reserved for chronic otitis media (COM) with effusion persisting for more than 3 months, or recurrent AOM (greater than six episodes in 6 months) which is not prevented by prophylactic antibiotics (half of normal dose amoxicillin or sulfisoxazole). It might be considered in AOM if the TM bulging, earache, fever, vomiting, and/or diarrhea were unusually severe or persistent. (Choice E) Initial acute otitis media clinical treatment failures are in large part attributable to DRSP, which may not be eradicated by simply increasing the dose of amoxicillin. Educational Objective: AOM patients with an initial clinical treatment failure (failure of acute otitis media to respond clinically to amoxicillin by day 3 of treatment) and who have not received antibiotics in the preceding month require a change to high dose amoxicillin/clavulanate or to other antimicrobials active against drug resistant Streptococcus Pneumoniae strains. 42% of people answered this question correctly. Case 126 A 76-year-old female is seen in the clinic with numerous medical complaints. She is scheduled to undergo elective surgery for right hip replacement. She says she is tired, has a very poor appetite, and does not sleep well at night. She denies any fevers, chills, chest pain, or shortness of breath. Her ECG and chest x-ray are normal. She cannot remember which medications she takes. What is the most important factor in perioperative adverse drug reaction in the elderly population? A. Age B. Multiple medications C. Reduced renal function D. Gastric alkalinity E. Reduced total body water

Explanation: Multiple medications are the most common factor in predisposing elderly patients to an increased incidence of adverse drug reactions. Patients over age 65 take an average of two to six prescribed drugs and one to three non-prescription drugs daily. These patients are at an increased risk preoperatively for anesthetic and operative drug reactions. The most common medications associated with adverse drug reactions include anti-psychotics, anti-hypertensives, sedatives, diuretics, NSAIDs, corticosteroids, and digoxin. (Choice D) Most elderly patients have an increased gastric pH, which tends to increase the absorption of drugs, thereby adding to the circulating levels of antagonistic compounds. (Choices C and E) The gradual reduction in the glomerular filtration rate in the elderly also adds to the circulating drug levels. This is further compounded by the fact that elderly patients have reduced levels of total body water. (Choice A) Age, itself, does not increase the risk of a perioperative adverse drug reaction. Educational Objective: Multiple medications are a major cause of adverse drug reactions in the elderly. 50% of people answered this question correctly. Case 127 A 28-year-old male with a known history of bipolar disorder is admitted to the floor due to an acute manic episode. You are the senior resident on-call, and during your interview, he reports a weight loss of more then 30 lbs in the past three months, with associated fever and night sweats. He admits to having multiple sexual partners, and states he never uses protection. There is a strong suspicion for HIV, but the patient refuses to have his blood drawn for the test. With concerns about the potential risk to his sexual partners, which of the following is the most appropriate step at this point? A. Go ahead and get the patient tested B. Contact the hospital?s ethics committee C. Obtain a court order to have the test done D. Do not test the patient?s HIV status E. Discuss with the attending and assess the patient?s competency to make a decision Explanation: A patient's right to refuse treatment can be overruled if there are potential risks to others due to the patient's refusal to treatment. However, patients have a right to refuse HIV testing. While other lab tests do not require any formal consent, blood test for HIV requires a formal consent from the patient. In this case, the physician must respect the patient's decision to not proceed with HIV testing. (Choice A) It is unethical and inappropriate to conduct HIV testing without obtaining formal consent from the patient, even if there are concerns for the health risks and welfare of his sexual partners. (Choice B) The hospital?s ethics committee need not be involved in this case, since the patient is clearly refusing the HIV test. (Choice E) Even with poor decision-making capacity, patients cannot be tested for HIV without obtaining their consent. Educational Objective: Do not test a patient?s HIV status without obtaining a formal consent. 39% of people answered this question correctly.

Case 128 A 16-year-old Caucasian boy with Down syndrome is brought into the emergency department by ambulance after being in a serious motor vehicle accident. He is unconscious, has unstable vital signs, and has sustained multiple lacerations to his face, arms, and abdomen. His adult sister is present, having witnessed the accident. She mentions that their parents are on a cruise and cannot be contacted. What is the next best step in managing this boy?s care? A. Provide treatment without seeking consent B. Seek consent from the sister C. Seek consent from the grandparents D. Petition the court for consent E. Withhold treatment until consent cannot be obtained from parents Explanation: Normally, the parents or legal guardian of a minor must provide voluntary, informed consent for treatment and most medical tests or procedures. However, there are exceptions to this rule, including the allowance that is made in case of emergency. If immediate medical care is necessary to prevent serious harm or death, consent is always assumed. Therefore the emergency department personnel should provide treatment for this boy without seeking consent (Choice A). Consent from the sister (Choice B) or grandparents (Choice C) is unnecessary and not of legal benefit, since they are not the legal guardians of the boy. Petitioning the court for consent (Choice D) would take far too long in an emergency setting. Moreover, the court is asked to intervene primarily when the parents are refusing treatment recommended by physicians. In this case, the parents have not refused treatment. They are simply unavailable to provide explicit consent. Withholding treatment until consent cannot be obtained from parents (Choice E) may lead to the death of the boy. Since an exception to the informed consent rule is provided in case of emergency, treatment should proceed. Educational Objective: Normally, the parents or legal guardian of a minor must provide voluntary, informed consent for treatment and most medical tests or procedures. However, if immediate medical care is necessary to prevent serious harm or death, consent is always assumed. 70% of people answered this question correctly. Case 129 A large-scale clinical trial is being planned to evaluate the effect of a non-selective beta-blocker, propranolol, on the clinical course of portal hypertension. The primary outcomes of the study are allcause mortality and major gastrointestinal hemorrhage. Secondary outcomes are minor gastrointestinal hemorrhage and the number of hospitalizations. The investigators are concerned about the possibility that major gastrointestinal hemorrhage events could be over-reported in the placebo group. Which of the following is the most useful technique to reduce this possibility? A. Randomization B. Blinding C. Matching D. Restriction E. Stratified analysis

Explanation: Observer?s bias is defined as a misclassification of events that can result from knowing the exposure status of a patient. In this clinical scenario, physicians may tend to classify hemorrhagic episodes as ? major? if a patient belongs to placebo group, and ?minor? if a patient belongs to the treatment group. Blinding the physician to the treatment status of a patient can balance this effect. The other listed techniques are used to control confounding variables. (Choice A) Randomization helps to balance numerous known and unknown extraneous factors (confounders) evenly between the treatment and placebo groups. It is commonly used in clinical trials. (Choices C and D) Matching and restriction are typically employed in other study designs (e.g., cohort studies). (Choice E) Stratified analysis is a statistical tool used to control confounding on the analysis stage of a study. Educational Objective: Observer?s bias can be effectively reduced by using the blinding technique. 60% of people answered this question correctly.

Case 130 A 56-year-old Caucasian female comes to the emergency department with complaints of a sudden onset of severe pain in the left side of her abdomen. It is accompanied by nausea and two episodes of vomiting. The pain waxes and wanes in intensity, and she has been unable to find a comfortable position. An initial CT scan of the abdomen reveals a 5 mm stone present in the left upper ureter. She is admitted to the hospital and is treated with intravenous hydration and pain control. After 30 hours of conservative management, she continues to have severe pain. Her kidney function is within normal limits. Which of the following is the most appropriate next step in the management of this patient? A. Continue observation and treatment. B. Start her on intravenous antibiotics. C. Refer her for extracorporeal shockwave lithotripsy. D. Refer her for flexible ureteroscopic removal of the stone. E. Refer her for percutaneous ureterolithotomy. Explanation: Renal or ureteral stones are a common presenting problem in the emergency department. Most of the patients present with a sudden onset of pain and hematuria. The pain is usually colicky in nature (waxing and waning) and can vary from a mild ache to severe discomfort requiring narcotic analgesics. The location of the pain can sometimes give clues to the site of the stone. Upper ureteral or renal stones usually cause pain in the flank, whereas a lower or distal ureteral stone causes pain which radiates to the ipsilateral groin area. Initial management is usually conservative, and includes intravenous hydration and pain control. Stones that are less than 5 mm in size usually pass spontaneously. On the other hand, stones greater

than 8 to 10 mm are unlikely to pass spontaneously and require removal. Stone removal is also indicated in patients with persistent pain, acute renal failure, or signs of urosepsis. There are three common techniques that have been employed to facilitate stone removal or passage from the ureters. These include extracorporeal shockwave lithotripsy (ESWL), flexible ureteroscopy, and percutaneous ureterolithotomy. The choice of procedure usually depends on the location of the ureteral stone. Shockwave lithotripsy is the treatment of choice for small symptomatic proximal ureteral calculi (less than 5 ? 10 mm in size). For large (more than 10 mm) proximal ureteral stones, flexible ureteroscopy combined with laser lithotripsy is the preferred initial therapy. (Choice A) Constant or persistent pain after conservative treatment is an indication for stone removal. (Choice B) There is no indication for using intravenous antibiotics in this patient with an uncomplicated ureteral stone. (Choice D) Ureteroscopic removable of calculi is performed in patients with large and proximal (more than 10 mm) ureteral calculi and/or distal ureteral stones. It is also useful in the management of ureteral calculi after failed therapy with shockwave lithotripsy. (Choice E) A percutaneous approach for stone removal is only used when shockwave lithotripsy and ureteroscopic removal fail to remove the ureteral stones. Educational Objective: Stones that are less than 5 mm in size usually pass spontaneously. Shockwave lithotripsy is the preferred initial therapy for small symptomatic proximal ureteral calculi (less than 5 to 10 mm). Stone removal is indicated in patients with stones greater than 8 to 10 mm (since these are unlikely to pass spontaneously), persistent pain, acute renal failure, or signs of urosepsis. 49% of people answered this question correctly.

Case 131 A healthy 15-year-old African-American woman is brought to the office by her mother for a routine evaluation. The patient has no complaints. Her father died of an aortic dissection when he was 39. Examination shows a diastolic murmur at the left third intercostal space. Lungs are clear to auscultation. The abdomen is soft, and nontender. Bowel sounds are present. The patient has particularly long and slender fingers. The girl?s mental and physical development is adequate for her age. She is at greatest risk for which of the following complications? A. Stroke B. Sudden cardiac death C. Acute aortic dissection D. Pulmonary embolism E. Acute pulmonary edema Explanation: This patient probably has Marfan syndrome (MFS). This is a genetic disease that is characterized by arachnodactyly, an arm span greater than the height, aortic root dilation, and ectopia lentis. The patient?s murmur is characteristic of aortic insufficiency due to aortic root dilation. This clinical finding needs to be confirmed by echocardiography, since it could potentially increase the risk for developing an acute aortic dissection at a young age.

(Choice E) About 80% of MFS patients have mitral valve prolapse, and will develop mitral valve insufficiency. These patients have a higher risk of acute pulmonary edema and congestive heart failure (CHF). (Choice B) MFS patients with CHF have a higher risk of sudden cardiac death . (Choice A) Because of the aortic insufficiency and dilatation of intracranial arteries, there can be an increased risk of stroke in patients with MFS, but it is much lower than the risk of aortic dissection or cardiac disease . (Choice D) There is no increased risk of PE in patients with Marfan syndrome. Educational Objective: The risk of aortic dissection is high in patients with MFS. For this reason, corrective surgery is recommended when the aortic root reaches 45 mm. About 80% of the patients will have mitral insufficiency, which can lead to CHF. These patients will benefit from mitral valve replacement. 82% of people answered this question correctly. Case 132 A 46-year-old Caucasian female comes to see you in the office to follow-up her breast biopsy results. She does not have any family history of breast cancer. She had a screening mammography done about a month ago, which identified an area of microcalcifications that was suspicious for malignancy in her left breast. She subsequently underwent fine needle aspiration biopsy of the lesion. The biopsy results revealed the presence of lobular carcinoma in situ in the left breast. Which of the following is the most appropriate next step in the management of this patient? A. Regular examination and mammogram B. Wide surgical excision of the lesion C. Excisional biopsy with radiation therapy D. Perform an open biopsy of the lesion E. Wide surgical excision of the lesion with chemotherapy Explanation: Lobular carcinoma in situ (LCIS) is examinations of breast tissue. It arises a multicentric (present at multiple sites in younger premenopausal women, the African-American women. usually detected as an incidental finding on microscopic from the lobules and terminal ducts of the breast. It is usually in the breast) and bilateral disease. It is more commonly seen incidence being much higher in white women, as compared to

Lobular carcinoma in situ is not considered to be the direct precursor of invasive breast cancer; however, its presence is one of the most important risk factors for the subsequent development of invasive breast cancer. The risk is increased in both the ipsilateral and contralateral breast, regardless of the side where LCIS is present. The various available options to manage patients with LCIS include close observation, tamoxifen therapy, and prophylactic bilateral mastectomy. Close observation with regular examinations and annual mammograms is appropriate for most women with LCIS. Studies have shown that there is no significant statistical difference in the mortality of women managed by close observation as compared to those managed by a surgical intervention or mastectomy. Patients can be followed regularly, just as any other patient with an increased risk of developing breast cancer. (Choice B) Surgical excision of the lesion with or without radiation therapy is generally not recommended because LCIS is usually a multicentric and bilateral disease. The only surgical modality

that can be considered after extensive discussions with the patient is bilateral prophylactic mastectomy. (Choice D) Open breast biopsy of the lesion is usually not necessary. (Choice E) Chemotherapy has no present role in the management of patients with LCIS; however, tamoxifen has been shown to reduce the risk of development of invasive cancer and should be strongly considered in these patients. Educational Objective: The three possible options for the management of patient with LCIS include close observation, tamoxifen therapy, and bilateral prophylactic mastectomy. Surgical excision of the lesion, chemotherapy and radiation therapy have no role in the management of patients with LCIS. 11% of people answered this question correctly. Case 133 The following vignette applies to the next 4 items A 24-year-old female is admitted to the hospital after a motor vehicle accident, where she sustained a right humerus fracture, multiple rib fractures and a contusion over her right lower leg. The next morning, a nurse calls to inform you that she is complaining of severe pain in her right lower leg. Her temperature is 37.2?C(99F), heart rate is 96/min, respiratory rate is 18/min, and blood pressure is 140/82 mmHg. Physical examination reveals a tense swelling around the right calf region. The pain is worsened on palpation and passive movements of the foot. Neurological examination reveals motor weakness and hypoesthesia of the distal right leg. Item 1 of 4 Which of the following is the most likely diagnosis? A. Fat embolism B. Deep venous thrombosis C. Acute compartment syndrome D. Acute vascular occlusion E. Neural compression Explanation: Acute compartment syndrome refers to ischemic tissue damage secondary to elevated pressures in the enclosed compartments of the lower legs or forearm. When the tissue pressure in an enclosed compartment exceeds the perfusion pressure, the resulting diminished tissue perfusion and compromised blood flow to the muscles and nerves inevitably lead to ischemic tissue necrosis. Majority of the cases involving the lower extremities are due to a traumatic event, most commonly tibial fractures. Other causes include a crush injury or other long bone fractures in a motor vehicle accident, a tight cast or dressing after trauma, and drug overdose. Patients usually present with severe pain which is out of proportion to the extent of injury. The pain is typically worsened by passive movements of the involved muscles. Sensory nerves are usually affected earlier than the motor nerves, and the neurologic deficit presents as decreased vibration sense, decreased two-point discrimination, numbness or hypoesthesia. Late features include extremity paralysis and absent distal pulsation (pulseless paralysis). (Choice A) Fat embolism is infrequently seen in patients with long bone or pelvic fractures. Patients usually present with a triad of hypoxemia, neurological abnormalities (i.e., confusion), and a petechial rash (involving the head, neck, anterior chest wall, or axilla).

(Choice B) Deep venous thrombosis usually does not present acutely in a setting of motor vehicle accident in an otherwise healthy young patient. It is a rare cause of compartment syndrome and does not compromise blood circulation and neuromuscular function. (Choice D) Vascular occlusion secondary to a motor vehicle accident usually presents more suddenly and dramatically. Acute compartment syndrome usually has a lag period of a few hours before irreversible nerve injury and muscle necrosis occurs. (Choice E) Nerve compression may occur in a patient after a motor vehicle accident and possible bone fracture; however, it does not lead to blood flow compromise and muscle necrosis (pain with passive movements of the involved muscles). Educational Objective: Acute compartment syndrome usually occurs after a traumatic event and causes pain, paresis, hypoesthesia, and diminished to absent pulses in the involved limb. 87% of people answered this question correctly. Item 2 of 4 Which of the following is the most common life-threatening complication of the above condition? A. Disseminated intravascular coagulation B. Rhabdomyolysis and renal failure C. Pulmonary embolism D. Gangrene of the limb E. Thrombocytopenia Explanation: Acute compartment syndrome results in markedly diminished to absent tissue perfusion within hours of the inciting event, causing tissue necrosis, muscle infarction, and rhabdomyolysis, which releases myoglobin into the peripheral circulation. Myoglobin is directly toxic to the renal tubules, and subsequently causes acute tubular necrosis and acute renal failure. Acute renal failure and its complications (electrolyte disturbances) are one of the most common life-threatening complications of acute compartment syndrome. Laboratory studies typically reveal markedly elevated creatinine kinase levels and the presence of myoglobin in the urine (positive dipstick for blood in the absence of RBC?s in the urine). (Choices A, C, D, E) Thrombocytopenia, disseminated intravascular coagulation, pulmonary embolism, and gangrene of the limb are not usually seen in patients with acute compartment syndrome. Educational Objective: Rhabdomyolysis and subsequent development of acute renal failure is one of the most common and severe life-threatening complications of acute compartment syndrome. 24% of people answered this question correctly.

Item 3 of 4 Which of the following is the most appropriate next step in the management of this patient? A. Administer oxygen B. Start the patient on anticoagulation C. Order venous Doppler ultrasonography D. Check the tissue pressure E. Order a nerve conduction study

Explanation: Compartment syndrome is characterized by an increase in the tissue pressure in the enclosed myofascial compartments of the extremities. When the elevated tissue or compartment pressure reaches its threshold level, the capillaries collapse, and this eventually leads to tissue and muscle necrosis. The exact value for the tissue pressure at which blood flows to the muscle and nerve tissue stops is controversial. The current general consensus for the threshold value is greater than 30 mmHg. It is therefore important to measure the tissue or compartment pressure early in the course of management, especially if the diagnosis is in question. (Choice A) Oxygen is usually used in the supportive treatment of patients with fat or cholesterol emboli. (Choice B) Anticoagulation is usually required for patients with deep venous thrombosis. (Choice C) Venous Doppler ultrasonography is useful for the diagnosis of deep venous thrombosis. It is not helpful for the diagnosis of compartment syndrome. (Choice E) Nerve conduction studies are helpful in the diagnosis and localization of the site of nerve damage. Educational Objective: Direct measurement of the compartment or tissue pressure is the diagnostic procedure of choice for patients with suspected acute compartment syndrome. 80% of people answered this question correctly. Item 4 of 4 The appropriate step was taken for the patient. Which of the following is the best next step in management? A. Continue with oxygen therapy B. Obtain a hypercoagulable panel C. Consult a vascular surgeon D. Perform urgent fasciotomy E. Review the results of venous Doppler ultrasonography Explanation: Acute compartment syndrome is a surgical emergency. Any delay in treatment leads to irreversible muscle and nerve damage. A compartment pressure of 30 mmHg or greater warrants an emergent fasciotomy (also known as compartment release). Surgical decompression aims to relieve the pressure within the enclosed compartment and to restore the blood flow to muscles and other tissues within 6-10 hours of the initial symptoms. Some patients may develop a persistent sensory or motor deficit after an episode of acute compartment syndrome despite early fasciotomy. (Choice A) Oxygen therapy has no role in the management of patients with acute compartment syndrome. (Choice B) Obtaining a hypercoagulable panel may be useful in patients with idiopathic deep venous thrombosis. It is not indicated in patients with acute compartment syndrome. (Choice C) A consultation with a vascular surgeon is necessary in patients with vascular injury or occlusion.

(Choice E) Venous Doppler ultrasonography is useful to diagnose deep venous thrombosis. It has no role in the diagnosis or management of patients with acute compartment syndrome. Educational Objective: An emergent fasciotomy is the definitive treatment in patients with acute compartment syndrome. Surgical decompression aims to restore the capillary blood flow and tissue perfusion. 86% of people answered this question correctly. Case 134 A 74-year-old Caucasian woman comes to the emergency department because of fever, nausea, dysuria, and frequency. She also complains of loss of appetite and abdominal pain. She has not been eating or drinking well for the past two weeks. She lives alone at home. She does not use tobacco, alcohol, or drugs. Her medications include aspirin, lisinopril, and ibuprofen. Her blood pressure is 130/80 mmHg and pulse is 98/min. Physical examination shows dry mucus membranes, clear lung fields, and normal first and second heart sounds. Her abdomen is soft, mildly tender, and nondistended. Her laboratory tests reveal the following: CBC Hb: 12.8 g/dL Ht: 38% Leukocyte count: 13,000/cmm Segmented neutrophils: 90% Lymphocytes: 10% Serum Chemistry Serum Na:148 mEq/L Serum K: 7.1 mEq/L Chloride: 112 mEq/L Bicarbonate: 17 mEq/L BUN: 78 mg/dL Serum Creatinine: 2.8 mg/dL Urine Specific gravity: 1.020 Blood: Trace Leukocyte esterase: Positive Nitrites : Positive WBC: 20-30/hpf RBC: 1-2/hpf The electrocardiogram shows sinus tachycardia, peaked T-waves in all leads, and no changes in the ST segments. Which of the following is the most immediate step in the management of this patient? A. Intravenous antibiotics B. Intravenous hydration C. Intravenous insulin D. Intravenous calcium gluconate E. Intravenous bicarbonate Explanation: The patient has a urinary tract infection and is developing high anion gap metabolic acidosis, dehydration, acute renal failure, and hyperkalemia. This latter condition is especially serious because there are EKG changes, potassium is higher than 7 mEq/L, and the process seems to be acute. In such a situation, the first thing to do is to administer intravenous calcium gluconate to stabilize the

membrane of the cardiac conduction tissue and prevent the development of life-threatening arrhythmias. (Choices C and E) Insulin and bicarbonate are also part of the therapy, but these will take some time to act. (Choices A and B) Hydration and antibiotic treatment are not as urgent as the management of significant hyperkalemia. Educational Objective: Significant hyperkalemia can be life threatening and needs to be managed immediately. Hyperkalemia is considered a medical emergency when there is an acute increase in the serum potassium level, the potassium is higher than 7, or there is simultaneous metabolic acidosis. In such situations, administration of calcium gluconate is the first thing to do. 77% of people answered this question correctly. Case 135 A 15-year-old female presents to clinic for a routine examination. Her medical history is significant for a diagnosis of attention deficit hyperactivity disorder, which is reasonably well managed with methylphenidate, and an elective abortion performed one year ago. She is currently a sophomore in high school and has won many awards for her athletic accomplishments. As the visit comes to a close, she informs you that she has recently become sexually active with a male classmate and would like a prescription for oral contraception. She asks that the physician not inform her mother about her use of birth control. What is the most appropriate response on the behalf of the physician? A. Inform the girl that her parent(s) will not be informed because she is an emancipated minor B. Inform the girl that her parent(s) will not be informed because the visit is regarding contraception C. Inform the girl that her parent(s) will be informed because she is a minor D. Inform the girl that her parent(s) will be informed because she has had an abortion E. Inform the girl that her parent(s) will be informed because they pay for her health insurance Explanation: As a physician, it can be difficult to balance the adolescent?s right to confidentiality against the parental right to oversee the child?s health. To address this conflict, most states have implemented laws that allow for a physician to provide care to adolescents without parental consent. These laws are typically limited to the management of certain issues, including pregnancy, contraception, sexually transmitted diseases, substance use, and emotional illness. Therefore, a teenaged patient is normally entitled to confidentiality regarding his or her use of contraceptives (Choice B). Most often, emancipated minors are teenagers who have successfully presented evidence to the court that they deserve emancipation, a special independent status legally equivalent to adulthood. Common examples include teenagers who are completely self-sufficient and no longer living with their parents and teenagers who are parents to one or more children. If the teenager in this situation were an emancipated minor, she would not need to obtain parental consent for any aspect of her health care. However, no evidence was presented to suggest that she is an emancipated minor (Choice A). If a teenager requires non-urgent treatment of any condition that is not exempt from parental consent laws (eg, surgical repair of a torn ligament), then her parents would be contacted because she is a minor (Choice C). However, if the visit is regarding contraception, she can be prescribed oral contraceptives without parental involvement. Since elective abortion is usually exempt from parental notification/consent laws, it would be illogical to presume that a history of abortion has any bearing upon a request for contraceptives (Choice D).

In most circumstances, parents pay for the health care insurance for their children. However, this does not automatically entitle them to all information regarding the child?s medical care. Since it is possible that health insurance bills will include descriptions of the services rendered, physicians should make every effort to preserve confidentiality when submitting insurance claims while still charting and billing correctly (Choice E). Educational Objective: Most states have implemented laws that allow for a physician to provide certain types of medical care to adolescents without parental consent. Exempted areas typically include pregnancy, contraception, sexually transmitted disease, substance use, and emotional illness. 84% of people answered this question correctly. Case 136 The following vignette applies to the next 2 items A 53-year-old African American woman presents to clinic at the insistence of her husband. Her husband accompanies her and reports that for quite some time, his wife has been extremely restless when she is sitting, resting, or lying down at night. She has trouble falling asleep in bed, which in turn keeps him awake. The woman adds that the restlessness comes about every night when she feels a strange sensation that she compares to an "internal itch that creeps up the lower legs." The sensation compels her to get up and pace the floor, and is worse when she is feeling particularly anxious or fatigued. Her medical history is significant for well-controlled hypertension, gastroesophageal reflux disease, allergic rhinitis, and tension headaches. She is currently taking hydrochlorothiazide, omeprazole, and ibuprofen. She does not smoke or use recreational drugs, but consumes 1-2 alcoholic drinks on the weekends. There is no family history of similar problems. Physical examination is unremarkable. Item 1 of 2 Which of the following studies is most appropriate to order? A. Urinalysis B. Serum electrolytes C. Liver function tests D. Iron studies E. Erythrocyte sedimentation rate Explanation: Restless legs syndrome (RLS) occurs in 5-15% of the population and is more common in older people. The diagnosis of RLS is based on the clinical history. It is characterized by spontaneous, repeated leg movements in association with unpleasant sensations that occur at rest. Movement provides relief. RLS may be a symptom of iron deficiency; since iron deficiency is frequently present in the absence of an anemia the iron studies should be performed. No other laboratory tests are routinely indicated. In this woman, iron studies (Choice D) should be checked. Urinalysis, serum electrolytes, liver function tests, and erythrocyte sedimentation rate (Choices A, B, C, and E) should be ordered if there is any indication of their necessity in the history or physical examination. Educational Objective: RLS may be a symptom of iron deficiency; since iron deficiency is frequently present in the absence of an anemia the iron studies should be performed. 10% of people answered this question correctly.

Item 2 of 2 The appropriate study was ordered and its results were within normal limits. Which of the following medications should be prescribed at this time? A. Clonazepam B. Codeine C. Gabapentin D. Levodopa E. Pramipexole Explanation: Restless legs syndrome (RLS) can be classified as intermittent, daily, and refractory. Intermittent or daily RLS is sometimes treated with nonpharmacologic methods, including iron replacement therapy, mentally stimulating activities (eg, crossword puzzles), and the avoidance of exacerbating factors (eg, nicotine, alcohol, caffeine). When prescription medication is warranted for treatment of daily RLS, dopamine agonists such as pramipexole (Choice E) or ropinirole are usually the most effective. Clonazepam (Choice A) may be of help in milder cases of RLS, especially in younger patients. Benzodiazepines are recommended for intermittent RLS, and short-acting agents are helpful in patients with sleep-onset insomnia secondary to RLS. Although opioid abuse potential is thought to be low in patients with RLS, prescription of narcotics is typically restricted to those patients with severe symptoms who have failed therapy with dopaminergic drugs or benzodiazepines. Codeine (Choice B) is one such possibility, and may work especially well in patients with intermittent RLS. Gabapentin (Choice C) is more commonly used in patients who have less intense or painful symptoms. It is also a good choice when used in the setting of neurodegenerative disorders such as Parkinson?s disease or dementia. Levodopa, also called L-dopa (Choice D), has been shown to cause augmentation in RLS patients, which is the worsening of symptoms the morning after a nighttime dose. It can also cause rebound, which is the recurrence of RLS symptoms early in the morning. L-dopa is therefore primarily recommended for usage with intermittent RLS because it is thought that sporadic usage of the drug may decrease the likelihood a patient will develop augmentation or rebound. Educational Objective: First-line treatment for daily restless legs syndrome includes dopamine agonists such as pramipexole or ropinirole. 9% of people answered this question correctly. Case 136 A 35-year-old male has been hospitalized for two days after being involved in a motor vehicle accident where he sustained multiple severe injuries, including rib fractures. He is developing progressive shortness of breath, and is being maintained on 100% oxygen. He has no previous medical history or any known lung abnormalities. His family history is not significant. He does not smoke, drink alcohol, or use any illicit drugs. A chest x-ray shows diffuse bilateral pulmonary infiltrates. You strongly suspect ARDS. Which among the following is the most useful strategy to decrease mortality in ARDS patients? A. Administration of surfactant B. Early administration of methyl prednisone C. Inhaled nitric oxide (NO) D. Prostacyclins E. Mechanical ventilation that delivers lower tidal volumes and limits plateau pressure

Explanation: Development of pulmonary contusions secondary to chest wall trauma is an important etiological factor of ARDS. ARDS is associated with a mortality rate of 35-40%. Among the different strategies, mechanical ventilation that delivers lower tidal volumes (< 6 ml/kg) and limits plateau pressure (< 30 CmH2O) has been shown to be relatively more effective. Studies comparing the effects of low tidal volume and limited plateau pressure to that of high tidal volume and plateau pressure have shown a significant reduction in mortality in the first group (32% vs. 40%). (Choice A) Theoretically, surfactant decreases the alveolar surface tension and reduces the chances of atelectasis. It facilitates the mucous clearance and helps to suppress inflammation. Unfortunately, studies have failed to demonstrate any significant positive outcome on 30 days mortality in these patients. (Choice B) Although steroids are frequently used in clinical practice in such situations, studies have not shown any beneficial effect on the acute phase. Some studies have even demonstrated increased infections in patients using steroids. Steroids are helpful, however, in reducing the intensity of the fibro-proliferative phase of ARDS in later stages. (Choice C) NO is a vasodilator which should theoretically improve V/Q mismatch; however, large multicenter, double blind, controlled studies have shown that the effect of NO compared to placebo, was modest and inconsistent. There was no significant difference found in 28 days mortality in both arms of these studies. (Choice D) Prostacyclins have similar effects as NO since it increases oxygenation and decreases pulmonary pressures; however, studies have also failed to demonstrate significant effects of prostacyclins on mortality. Educational Objective: ARDS is associated with a mortality rate of 35-40%. Among the different strategies, mechanical ventilation that delivers lower tidal volumes (< 6 ml/kg) and limits plateau pressure (< 30 CmH2O) has been shown to be relatively more effective than other strategies such as NO inhalation, use of prostacyclins, exogenous surfactant, and steroids. 79% of people answered this question correctly. Case 137 The following vignette applies to the next 2 items A 14-year-old girl comes to the emergency department with her mother for the evaluation of left knee pain. The girl was playing soccer when she fell and injured her left knee after being tackled by one of the players. She immediately felt a tearing sensation and excruciating pain in the front of her left knee. She is presently having a great deal of pain and difficulty in rising and bearing weight on her left leg. On physical examination, there is diffuse swelling and tenderness on the anterior aspect of her left knee. She has a significant joint effusion. Passive extension and flexion of her left knee are normal; however, she is unable to maintain extension of the left knee against gravity. The anterior and posterior drawer signs are normal. The valgus and varus stress to the left knee does not reveal an increased laxity or instability on the medial or lateral side. McMurray?s maneuver on the left knee does not reveal a popping sensation with external rotation and passive extension of the lower leg. Item 1 of 2 Which of the following is the most likely explanation for her findings? A. Anterior cruciate ligament tear B. Medial collateral ligament tear

C. Lateral collateral ligament tear D. Patellar tendon tear E. Medial meniscal tear Explanation: Soft tissue knee injuries are extremely common in adolescent and young adults who engage in various contact sports. Most patients complain of an immediate, profound pain and difficulty or inability to bear weight on the same leg. It is important to establish the correct anatomic diagnosis of these injuries in order to institute an early and specific intervention. The most common vulnerable structures in the knee include the anterior and posterior cruciate ligaments, medial and lateral collateral ligaments, medial and lateral meniscus, and joint capsule. A detailed history and physical examination (including specific maneuvers) can help in differentiating between the damage to various possible knee structures. The patient in the above vignette has a patellar tendon tear. The patellar tendon is the distal extension of the quadriceps insertion. Its rupture is usually due to traumatic athletic injury. The most common mechanism is a sudden and unusual quadriceps contraction with the foot firmly planted, and with the knee in partial flexion during the injury. Patellar tendon tear or rupture usually occurs at the osseotendinous junction and results in excruciating pain, swelling, and difficulty in bearing weight. Physical examination generally reveals swelling and tenderness in the anterior part of the knee. With a complete tendon rupture, the patients are unable to perform active extension of the leg and are unable to maintain the passively extended knee against gravity. (Choice A) Anterior cruciate ligament tears are one of the most serious forms of knee injuries. Most patients complain of profound pain, inability to ambulate, and a popping sensation or sound at the time of injury. The anterior drawer test is commonly used to evaluate the integrity of the anterior cruciate ligament. The patient is placed in the supine position with the hip flexed at 45 degrees, the knees bent at 90 degrees, and the patient?s foot firmly planted on the examination table. With both hands behind the affected knee, the examiner tries to displace the lower leg anteriorly. A difference of 1 cm compared with the opposite side suggests a complete tear of the anterior cruciate ligament. (Choices B and C) The valgus and varus stress tests are used to determine the integrity of the medial and lateral collateral ligaments, respectively. A history of knee injury, local tenderness on the medial or lateral joint lines, and an abnormal opening of the knee as compared to the opposite side on valgus or varus stress is suggestive of ligament injury or disruption. (Choice E) Menisci are the fibrocartilaginous pads located between the femoral condyles and tibial surfaces. The common mechanism of meniscal injury is a twisting force with the foot fixed on the ground. This is commonly seen during football and basketball games. McMurray?s maneuver is used to detect the presence of meniscal tears. To detect a medial meniscal injury, the patient is put in a supine position with the knee in maximum flexion. One hand of the examiner is placed on the posteromedial margin of the involved knee and the other hand supports the foot. The tibia is then externally rotated, and the knee is extended slowly. The test is positive if there is an audible or palpable click or popping sensation during extension of the involved knee. Educational Objective: Patellar tendon rupture presents with excruciating pain, swelling in the anterior part of the knee, and an inability to maintain passive extension of the knee against gravity. 73% of people answered this question correctly. Item 2 of 2 Which of the following is the most appropriate next step in the management of this patient? A. Inquire if there is a family history of seizures B. Complete bed rest for two weeks with gradual return to baseline activity in six weeks

C. Consult orthopedic surgery for early surgical intervention D. Brace immobilization and ice packs in the local area for four weeks E. Medial meniscal tear Explanation: Based on the clinical presentation, the patient appears to have a complete disruption of the patellar tendon and dysfunction of the knee extensor mechanism. In all such patients, early surgical intervention and repair of the patellar tendon is the treatment of choice. Complete rupture of the patellar tendon causes significant damage to the knee extensor mechanism. Delayed treatment can lead to quadriceps muscle atrophy, contracture formation, and limited range of motion of the knee, thereby causing early fatigue, pain, and significant disability. Early diagnosis and surgical repair usually lead to excellent recovery of knee function and prevent long-term disability. (Choices A and B) Non-operative treatment (conservative treatment with bed rest and pain control) has a very limited role in the management of patients with acute patellar tendon rupture. It can lead to a significant dysfunction of the knee extensor mechanism and long-term disability. (Choice D) A cast or brace immobilization in full extension is used very occasionally in patients with partial patellar tendon tear. Complete patellar tendon tears are ideally managed by early surgical intervention. Educational Objective: Early surgical repair of the ruptured patellar tendon is recommended to promote the recovery of normal knee motion and prevent long-term disability. Case 138 A 5-year-old boy with an unremarkable medical history is brought to the office by his mother for his annual wellness exam. He is scheduled to receive the measles-mumps-rubella (MMR) vaccine today. His mother mentions that the last time he was given this vaccine, he developed a fever of 39.1C (102.4F). She asks if he should still receive the MMR vaccine. Which of the following is the most appropriate response? A. Inquire if there is a family history of seizures B. Inquire if there is a family history of anaphylaxis C. Inquire if the boy has a history of allergy to mercury D. Inquire if the boy has a history of allergy to eggs E. Assure her the boy appears to have no contraindications to the MMR vaccine Explanation: Vaccination against measles-mumps-rubella in children/adults is contraindicated if any of the following situations apply: Current moderate or severe febrile illness Anaphylaxis to neomycin or gelatin Severe immunodeficiency Thrombocytopenia after the first dose of MMR Recent administration of immunoglobulins Pregnancy Current moderate or severe febrile illness: In patients with moderate to severe febrile illnesses vaccine administration should be delayed until the acute phase of the illness is resolved. Allergies: A history of anaphylaxis after either ingestion of gelatin or contact with neomycin is considered a contraindication; however, a history of anaphylaxis to egg products is not a contraindication to measles immunization.

Thrombocytopenia: A patient with a history of thrombocytopenia is at risk of developing severe thrombocytopenia following measles vaccination. It is advisable not to give a second dose of MMR in patients who developed thrombocytopenia following the first dose. Recent administration of immunoglobulins can diminish the efficacy of MMR; therefore, vaccination should be delayed for a period of time (varies from 3-11 months) depending on the strength of antibody in the preparation. Immunocompromised state: An immunocompromised state is considered a contraindication to administer live vaccines because of the risk of vaccine associated fatal infections (eg; MMR). On the other hand, the immunocompromised patients are also at high risk of developing natural measles infection; therefore, whether or not to vaccinate these patients depends on the degree of immunosuppression and the risk of natural measles infection. The consensus is that patients who are severely immunocompromised (congenital immunodeficiency, severe HIV infection, hematologic or solid tumors and long term immunosuppressive therapy) should not receive MMR. Asymptomatic HIVinfected patients without severe immunosuppression should be vaccinated with MMR. The following are not considered as contraindications for MMR: Tuberculosis or positive PPD Breastfeeding Immunodeficient family member or household contact Asymptomatic HIV-infected patients without severe immunosuppression The boy?s history of a mild febrile reaction to the MMR vaccine is not considered a contraindication. (Choice A) A family history of seizures was formerly considered a contraindication to receipt of DTP vaccine, but recent studies have suggested that this is an unnecessary precaution. (Choice B) A family history of anaphylaxis is not considered relevant in this situation. The only allergic contraindication would be a personal history of anaphylaxis associated with ingestion of gelatin or exposure to neomycin, as these are ingredients in the MMR vaccine. (Choice C) An allergy to mercury is extremely rare, but this does occur. Since the MMR vaccine does not contain thimerosal (mercury), this issue is not of importance in this child. (Choice D) A history of anaphylaxis after egg ingestion is not considered a contraindication to MMR vaccination, as the current MMR vaccine does not contain a significant amount of egg proteins. Educational Objective: MMR vaccination is contraindicated in patients with: Current moderate or severe febrile illness Anaphylaxis to neomycin or gelatin Severe immunodeficiency Thrombocytopenia after the first dose of MMR Recent administration of immunoglobulins Pregnancy The following are not considered as contraindications for MMR: Tuberculosis or positive PPD Breastfeeding Immunodeficient family member or household contact Asymptomatic HIV-infected patients without severe immunosuppression Anaphylaxis to eggs *Extremely important question for USMLE step-3. Please read the explanation at least twice. 59% of people answered this question correctly.

Case 139 A 59-year-old African-American man comes to the office and complains of a vague upper abdominal discomfort. The pain has been present most of the time for the past three weeks, but has never been severe enough to make him seek immediate medical attention. The discomfort is not associated with eating or performing any activities. He denies any other medical problems. His vital signs are unremarkable. Abdominal examination reveals the presence of multiple scars from previous surgical procedures around the epigastric area. A 3 x 5 cm oblong-shaped mass is palpable in the epigastric area. There is no tenderness on direct palpation of the mass and the surrounding epigastric area. The patient tells you that he previously had a similar mass in the same area "but it was taken out". Which of the following is the most likely explanation for the mass? A. Tell mother to push with the next contraction B. Ventral hernia C. Muscle growth D. Abdominal wall neuroma E. Assure her the boy appears to have no contraindications to the MMR vaccine Explanation: The patient's clinical presentation is consistent with a diagnosis of desmoid tumor. Desmoid tumors are locally aggressive neoplasms arising from fibroplastic elements within the muscle or fascial planes. Since these are locally invasive and slowly infiltrate the surrounding tissues and structures, these only cause local complications. These usually present as painless or minimally painful, slowgrowing masses over the extremities, shoulder girdle, and hip-buttock area, although these may occur at all body sites. The diagnosis is generally confirmed with a tissue biopsy. Surgical excision with a wide margin of resection is the treatment of choice for patients with an easily approachable and resectable mass. There is a high rate of local recurrence, even after adequate resection. (Choice A) Abdominal aortic aneurysms usually present as a painless pulsatile mass. does not have any risk factors for the development of aortic aneurysm. This patient

(Choice B) Ventral or incisional hernia can develop at the site of previous surgical scars; however, this is generally reducible, and pain is an unusual symptom of ventral hernia, unless it is strangulated. Educational Objective: Desmoid tumors are slow growing and locally aggressive benign neoplasms with a high rate of local recurrence, even after surgical excision. 6% of people answered this question correctly. Case 140 A 36-year-old gravida 6, para 5 Caucasian woman at 41 weeks of gestation is admitted to the labor and delivery unit after experiencing regular uterine contractions every 10 minutes and the spontaneous rupture of membranes. Her previous pregnancies were unremarkable except for asymptomatic bacteriuria that developed when she was pregnant with her first and third children and a cesarean delivery for her fourth child because of breech positioning. This current pregnancy had been uncomplicated except for obesity, and she weighed 272 lbs at admission. An oral glucose tolerance test was administered at 29 weeks of gestation and while her fasting plasma glucose was elevated, the other glucose levels were considered normal. The woman?s fundal height was 42 cm the day prior to admission. As her labor progresses, the cervix thins and completely dilates. The vertex of the fetus appears after 15 minutes of second stage labor. However, the fetal anterior shoulder is now proving undeliverable by gentle traction. What is the most appropriate next step? A. Tell mother to push with the next contraction

B. Tell mother not to push C. Perform McRoberts maneuver D. Perform Zavanelli maneuver E. Break fetal clavicle to deliver anterior shoulder Explanation: Shoulder dystocia is commonly defined as a failure of the fetal shoulders to pass through the maternal pelvis once the fetal head has been delivered. It is diagnosed when the anterior shoulder cannot be delivered with mild, downward pressure. Risk factors for the development of shoulder dystocia include macrosomia, maternal diabetes mellitus, operative vaginal delivery, shoulder dystocias in previous deliveries, postdate pregnancies, male fetal gender, advanced maternal age, obesity and excessive weight gain, and disproportion between the fetal shoulders and maternal pelvis. However, more than 50% of cases of shoulder dystocia are not associated with any known risk factors. When shoulder dystocia occurs, appropriate support staff (eg, nursing, anesthesia, obstetrics, pediatrics) should be summoned. The mother should be told not to push (Choice B) while attempts are made to reposition the fetus. Suprapubic pressure directed downward and laterally should then be applied by an assistant. If that fails to deliver the anterior shoulder, then typically the McRoberts maneuver is attempted, though many other techniques work as well (eg, Rubin maneuver, Woods screw maneuver, delivery of posterior arm). Telling the mother to push with the next contraction (Choice A) is incorrect, as it will only exacerbate the problem. The McRoberts maneuver (Choice C) requires that two assistants grasp both of the mother?s legs and flex the thighs back against her abdomen. This maneuver has been shown to relieve the shoulder dystocia in 42% of patients. Before the maneuver is implemented, however, the mother should be told to stop pushing until everything is in place. The Zavanelli maneuver (Choice D) replaces the fetal head in the pelvis before performing a cesarean section. It is generally accepted that the physician has up to seven minutes to deliver a previously well-oxygenated infant before there is an increase in the risk of damage due to asphyxia. Therefore, the Zavanelli maneuver is normally employed when other methods have failed and the "safe" period of seven minutes is dwindling. Breaking the fetal clavicle (Choice E) is occasionally done, though it is difficult to perform and obviously not a preferred means of resolving the problem unless other alternatives are unsuccessful. Educational Objective: When shoulder dystocia occurs, appropriate support staff should be summoned. The mother should be told not to push while attempts are made to reposition the fetus. If suprapubic pressure fails to deliver the anterior shoulder, then typically the McRoberts maneuver is attempted, though many other techniques work as well (eg, Rubin maneuver, Woods screw maneuver, delivery of posterior arm). 9% of people answered this question correctly. Case 140 A 26-year-old anthropology student presents to the emergency department 10 hours after his right arm was scratched by a bat. He says, ?I frequently explore caves and bats do not always seem to like me.? He had a similar scratch one year ago and received complete anti-rabies vaccination. His past medical history is otherwise insignificant. He denies smoking, alcohol consumption, and any illicit drug use. All his vaccinations are up-to-date. He is currently not taking any medications and has no known allergies. What is the best management strategy for this patient? A. Rabies vaccine

B. Rabies immune globulin C. Rabies immune globulin and vaccine D. Hospitalization and observation E. Reassurance and routine discharge Explanation: Rabies is an invariably fatal viral disease primarily acquired from a bite or exposure to saliva of a rabid animal. This can be prevented with proper postexposure prophylaxis. Bats represent a potential source of the infection; therefore, prophylaxis is standardly given to patients who have bat bites. There are two types of rabies prophylaxis products: rabies immune globulin for passive immunization and rabies vaccines (several are available) for active immunization. Previously vaccinated persons who are potentially re-exposed to rabies should receive only active immunization with a rabies vaccine. (Choice C) Previously unvaccinated individuals who are exposed to a potential source of rabies should receive both active and passive immunization. Educational Objective: Previously vaccinated persons who are potentially re-exposed to rabies should receive only active immunization with a rabies vaccine. 9% of people answered this question correctly. Case 141 An asymptomatic 60-year-old Caucasian man comes to the physician for a routine health maintenance examination. He has no other medical problems. He is a software engineer. He does not drink alcohol. He has smoked one pack of cigarettes daily for 10 years. His family history is not significant. He has no medications. His vital signs are within normal limits. The physical examination shows no abnormalities. The digital rectal examination is normal. His serum PSA level is 6 ng/mL. Which of the following is the most appropriate next step in the management of this patient? A. Suggest it is unlikely that the daughter will quit smoking unless her mother quits first B. Refer to urologist for biopsy C. Reassurance D. Radical prostatectomy E. Reassurance and routine discharge Explanation: The normal range of serum PSA is 0 to 4 ng/mL. Patients with PSA levels greater than 4 ng/mL should be referred to a urologist for a biopsy. (Choices A and C) Only a biopsy can confirm or rule out a diagnosis of prostate cancer. Reassurance or repeating the measurement of PSA levels in six months is not the appropriate choice because more than 20% of patients with PSA levels greater than 20 ng/mL have cancer on biopsy. The cutoff value has been reduced to 4 ng/dL to increase the sensitivity for diagnosing early prostate cancer. (Choice D) Radical prostatectomy is a treatment option for early prostate cancer. Pretreatment tests include biopsy of the tumor and a CT scan. Educational Objective: Patients with PSA levels greater than 4 ng/ml should be referred to a urologist for biopsy. 42% of people answered this question correctly.

Case 142 A 14-year-old girl presents to the pediatrician for an annual wellness exam. She is accompanied by her distraught mother, who expresses dismay that the daughter started smoking six months ago. Apparently, the daughter now regularly takes cigarettes from her mother?s purse to smoke with a best friend at school. In addressing this issue, which of the following is the most appropriate next step? A. Suggest it is unlikely that the daughter will quit smoking unless her mother quits first B. Suggest that the mother lock her cigarettes away C. Suggest that the daughter be forbidden to socialize with her best friend D. Suggest that the daughter?s friend?s mother be informed of the situation E. Suggest that the school principal be asked to punish the daughter for smoking on campus Explanation: Nicotine is a highly addictive drug. It is extremely difficult for even the highly motivated to stop smoking. Numerous factors may elicit the urge to smoke. Studies have shown that smokers who attempt to quit while working or living with people who continue to smoke are much more likely to relapse. If the mother is genuinely interested in helping her daughter stop smoking, she should quit smoking herself. Prescriptions for nicotine replacement (e.g., nicotine patch, nicotine gum) should be offered to those smokers interested in quitting. (Choice B) Locking up the cigarettes may prevent the daughter from using her mother?s cigarettes; however, the daughter may still continue to smoke by obtaining cigarettes from other avenues, including friends and unscrupulous merchants. (Choice C) Forbidding the daughter to socialize with her best friend may backfire and strain the daughter?s relationship with her mother, without accomplishing the goal of smoking cessation. It is difficult to control the daughter?s actions while she is away from the house and family. She may secretly continue to see her best friend and smoke cigarettes. Even if the daughter were to stop spending time with this particular friend, she may well have other friends who also smoke. (Choices D and E) Placing the responsibility onto another adult is inappropriate. If the mother is concerned about the issue of her daughter?s smoking, she should take on the responsibility of addressing the problem in its entirety herself. Educational Objective: Smoking cessation is difficult to accomplish if the smoker remains subject to environmental triggers, including friends and family who continue to smoke. Parents who want their children to stop smoking should quit themselves. Case 143 A 16-year-old gravida 1, para 0 Caucasian girl at 36 weeks of gestation is admitted to the hospital immediately after developing severe hypertension, edema, and proteinuria. She complains of headache and nausea. Accompanying her is her mother and her boyfriend, the father of the baby. The decision is made to induce labor, and pitocin is infused intravenously. After five hours of labor, the patient?s cervix dilates to four centimeters. Her uterine contraction pattern varies from 200-220 Montevideo units. After another two hours, the patient?s cervix remains at four centimeters. What is the next best step in managing this girl?s care? A. Recommend cesarean section and obtain consent from patient?s mother B. Recommend cesarean section and obtain consent from patient?s boyfriend C. Recommend cesarean section and obtain consent from patient D. Recommend cesarean section and obtain consent from hospital ethics committee E. Increase Pitocin drip and re-evaluate cervix in one hour Explanation:

The girl in this question is suffering from preeclampsia, a condition occurring after 20 weeks of gestation that is characterized by hypertension, edema, and proteinuria. Those women who develop severe preeclampsia should be delivered once they reach 32-34 weeks of gestation. Since this girl is at 36 weeks of gestation, her fetus should be sufficiently mature to warrant delivery. In such circumstances, vaginal delivery is normally successful and results in a good outcome for both mother and child. However, since this patient's cervix has not dilated beyond four centimeters in the past two hours, she is diagnosed with a failure to progress. Failure to progress is an indication to perform a cesarean section. Although this girl is a minor at age 16, most states have legislated that a physician can provide care for adolescents without parental consent when it comes to certain issues, including pregnancy, contraception, sexually transmitted diseases, substance use, and emotional illness. Therefore, the appropriate management of this situation entails recommending a cesarean section and seeking consent from the patient (Choice C). The patient?s mother (Choice A) would need to give consent for surgical procedures unrelated to this pregnancy. The patient?s boyfriend (Choice B) does not have any legal authority in this circumstance and should not be consulted. The hospital ethics committee (Choice D) does not need to be consulted in this case because consent can be obtained from the appropriate individual. Increasing the Pitocin and re-evaluating the cervix in one hour (Choice E) does not address the key issue of the girl?s failure to progress. Educational Objective: Expectant teenage mothers are legally entitled to give consent for procedures related to the management of their pregnancies, including the performing of cesarean sections. 75% of people answered this question correctly. Case 144 A 63-year-old Caucasian female is brought to the emergency department after a car accident. She is complaining of a severe headache, and had two episodes of vomiting in the ambulance. She hit her head over the steering wheel, but did not lose consciousness. Her past medical history is significant for chronic atrial fibrillation and hypertension. Her current medications are warfarin, enalapril and metoprolol. Her most recent INR value was 2.1. Her heart rate is 70/min, irregular; her blood pressure is 160/100 mmHg. No focal neurologic deficit is evident on physical examination. She is lethargic, but oriented to place and time. Head CT scan findings are consistent with subarachnoid hemorrhage. Her hemoglobin is 11.6 gm/dL. Which of the following is the best treatment option for this patient? A. Infuse fresh frozen plasma. B. Administer parenteral vitamin K . C. Stop warfarin and provide routine care. D. Continue warfarin and provide routine care. E. Consider whole blood transfusion. Explanation: This patient presents with traumatic subarachnoid hemorrhage associated with chronic intake of warfarin. In this life-threatening situation, it is important to implement immediate measures to stop the hemorrhage. Her coagulation status should be improved as quickly as possible. Fresh frozen

plasma infusion is the best therapeutic choice because it rapidly provides the necessary clotting factors for coagulation to proceed. (Choice B) Although warfarin is an antagonist of vitamin K, supplementation of the vitamin will not work quickly because it takes some time for clotting factors to be synthesized in the liver. (Choice C) Simply discontinuing warfarin will not rapidly and adequately correct the coagulation abnormality. (Choice E) There is no indication for whole blood transfusion in this patient. Educational Objective: Life-threatening hemorrhage in patients on warfarin therapy should be treated with fresh frozen plasma transfusion. 46% of people answered this question correctly. Case 145 A 70-year-old man with moderately severe dementia who is placed in a nursing home is noted to soil his bed with urine. He also has frequency, hesitancy, and terminal dribbling while voiding. Physical examination reveals an enlarged rubbery prostate. Mild pressure on the abdomen results in urine dripping. Which of the following is the best statement about this patient?s condition? A. Urinary incontinence indicates that normal pressure hydrocephalus (NPH) is present B. No epidemiologic association is present between dementia and urinary incontinence C. Bladder atony results from the damage of subcortical structures D. Enlarged prostate seems to be the main cause of this patient?s condition E. Urinary incontinence associated with dementia is untreatable Explanation: Dementia-associated urinary incontinence is a multi-factorial disorder that may not be etiologically related to dementia itself. Therefore, careful assessment of an individual patient and revealing predisposing urinary tract and non-urinary tract conditions are important because many of these conditions may be treatable. In this case, overflow incontinence due to bladder outflow obstruction is likely. (Choice C) Until recently, it was a common belief that Alzheimer?s and multi-infarct dementia damage cortical and subcortical inhibitory structures, thereby causing an uninhibited bladder and creating a pathophysiological basis for urinary incontinence; however, several studies have proven that even severely demented patients may stay continent. (Choice B) Urinary incontinence has a strong epidemiologic association with dementia. (Choice A) The presence of urinary incontinence does not always imply that NPH is present, because urinary incontinence is also strongly associated with other types of dementia.

Educational Objective: Dementia-associated urinary incontinence is a multi-factorial disorder that may not be etiologically related to dementia itself. Careful assessment of an individual patient and revealing predisposing urinary tract and non-urinary tract conditions are important because many of these conditions may be treatable.

82% of people answered this question correctly. Case 146 A 35-year-old Caucasian male with no significant past medical history presents to the emergency department complaining of severe pain in his left eye. Earlier that afternoon he was working on his boat when he dropped a heavy metal crate onto the boat?s wooden deck. Immediately after he dropped the crate, he felt one or more small foreign objects enter his left eye. The eye began to water profusely and he was unable to open it sufficiently to examine the cornea. He states that he does not wear contact lenses. Physical examination of the left eye reveals edema of the upper eyelid, conjunctival hyperemia, diffuse corneal inflammation, and corneal abrasion. After a topical anesthetic was applied to the injured eye, irrigation with saline was successful in removing all debris. What organism is most commonly cultured from corneal foreign bodies? A. Pseudomonas B. Staphylococcus C. Neisseria D. Sporothrix E. Candida Explanation: Although rarely sent for culture in practice, corneal foreign bodies are known to frequently test positive for pathogens. Studies have shown that the most common pathogen isolated from these cultures is coagulase negative Staphylococcus (Choice B). Other common pathogens include Streptococcus, Haemophilus, and Pseudomonas. Therefore, the treatment for all foreign-body associated corneal abrasions should include empiric broad-spectrum antibiotic eye drops or ointments (e.g., erythromycin, sulfacetamide, ciprofloxacin, ofloxacin). Pseudomonas (Choice A) is a common cause of bacterial infection in contact lens wearers. Neisseria gonorrhoeae (Choice C) can penetrate intact corneal epithelium and cause bacterial keratitis. This is one of many possible manifestations of gonorrheal infection resulting from sexual transmission or passage through the birth canal. Sporotrichosis (Choice D) is a fungal infection most often found in persons handling thorny plants, sphagnum moss, or baled hay. It typically manifests as nodules on the skin that ulcerate and are slow to heal. Candidal (Choice E) infection of the eye is most often found in immunosuppressed patients or those with chronic diseases of the ocular surface. Educational Objective: The most common pathogen isolated from cultures of corneal foreign bodies is coagulase negative Staphylococcus. Other common pathogens include Streptococcus, Haemophilus, and Pseudomonas. 46% of people answered this question correctly.

Case 147 The following vignette applies to the next 2 items A 38-year-old healthy Caucasian man is brought to the emergency department because of sudden onset of shortness of breath and diaphoresis. He denies fever, chills, cough or abdominal pain. He has no other medical problems. He had surgery for bilateral inguinal hernia when he was 16. He does not use tobacco, alcohol or illicit drugs. He takes no medication and has no known drug allergies. His blood pressure is 110/60 mm Hg, pulse is 116/min and respirations are 28/min. He is in marked respiratory distress. Pallor and diaphoresis are noted. His skin is velvety and has multiple scars. On auscultation of the heart, an early, decrescendo, systolic murmur at the cardiac apex is heard; the murmur decreases with Valsalva maneuver, and increases with the grip maneuver, radiating to the axilla. The first sound is barely audible; the second heart sound is normal. A fourth heart sound is also present. There are bilateral crackles in both lungs. Jugular venous distention and hepatojugular reflux are present. The abdomen is soft, non-tender and non-distended. The neurologic examination reveals no abnormalities. The initial EKG shows sinus tachycardia with occasional premature ventricular complexes. The chest x-ray reveals no cardiomegaly, but bilateral alveolar infiltrates and hilar prominence are present. Item 1 of 2 Which of the following is the most likely cause of his condition? A. Acute myocardial infarction B. Rupture of chordae tendineae C. Pulmonary embolism D. Infective endocarditis E. Papillary muscle rupture Explanation: The patient presents with signs and symptoms of acute heart failure. His EKG findings reveal occasional premature ventricular complexes (PVC); there are no signs of ischemia or ventricular hypertrophy. His pathologic murmur (systolic murmur that is heard in the apex, radiates to the axilla, increases with the grip maneuver, and decreases with Valsalva) is characteristic of mitral regurgitation, which may lead to acute heart failure. Acute mitral regurgitation is usually characterized by a soft, decrescendo systolic murmur (can be early, midsystolic or holosystolic), a decreased first heart sound, and the presence of a fourth heart sound. Due to the acute nature of this condition, there is no evidence of left ventricular hypertrophy in the electrocardiogram or the chest xray. The four common causes of acute heart failure are papillary muscle rupture, infective endocarditis, rupture of chordae tendineae, and chest wall trauma with compromise of the valvular apparatus. The most common cause of isolated, severe acute mitral regurgitation in adults is rupture of chordae tendineae with or without associated myxomatous disease. The diagnosis is confirmed by echocardiography. (Choice A) Myocardial infarction can be complicated by acute mitral regurgitation when there is rupture of the papillary muscle; however, this patient does not have evidence of ischemia in the EKG. (Choice C) Patients with pulmonary embolism can develop acute cor pulmonale; however, in such a setting, the EKG will show right axis deviation, right bundle branch block or both. The above patient clearly has acute pulmonary edema. (Choice D) The patient does not have fever or risk factors for bacterial endocarditis. (Choice E) Spontaneous papillary muscle rupture usually presents in elderly people who have acute chest pain or as a complication of myocardial infarction. None of these are present here.

Educational Objective: Rupture of chordae tendineae should be suspected in healthy individuals who develop flash pulmonary edema (heart failure) associated with an acute mitral regurgitation. The differential diagnosis of this condition includes infective endocarditis, papillary muscle rupture secondary to ischemia, and mitral valve rupture secondary to trauma. 46% of people answered this question correctly. Item 2 of 2 The patient is feeling better after receiving the appropriate therapy. A more detailed physical examination reveals the presence of scoliosis and pes planus. The cardiac enzymes were negative. Blood gas analysis reveals respiratory alkalosis. Complete blood cell count and serum chemistry results are unremarkable. The patient is scheduled for echocardiography. Which of the following is the most probable diagnosis? A. Coronary artery disease B. Rheumatic fever C. Marfan syndrome D. Ehlers-Danlos syndrome E. Thyrotoxicosis Explanation: The patient has features compatible with Ehlers-Danlos syndrome. This condition can cause a myxomatous degeneration of the mitral valve, leading to acute rupture of chordae tendineae. Pes planus and scoliosis are frequent, though not specific findings. The skin can be velvety or thin, and is usually covered with multiple characteristic "cigarette-paper" scars, due to its frailty and easy bruisability. A past history of hernias and mitral valve prolapse is not uncommon. Joint hypermobility and skin hyperextensibility ("rubber man syndrome") can be dramatic in severe cases. (Choice A) There is no evidence of ischemic heart disease. Troponins and CK-MB are normal. The EKG shows no ischemic changes. (Choice E) Thyrotoxicosis can lead to acute, high-output, heart failure; however, this will not explain the mitral regurgitation. Skin changes of thyrotoxicosis can be confounded with those found in EhlersDanlos, but other characteristic signs and symptoms are lacking. (Choice B) Rheumatic fever is a common cause of mitral regurgitation in young individuals; however, the course of the disease is chronic, and the patients usually have evidence of left ventricular and atrial hypertrophy in the EKG. (Choice C) Marfan syndrome can affect the valvular apparatus, causing degeneration of the mitral and aortic valves. The presentation is usually chronic, progressive mitral regurgitation, and rarely, acute regurgitation due to ruptured chordae tendineae. This patient does not have arachnodactyly, loose joints or increased arm span, which are characteristic of Marfan syndrome. Educational Objective: Marfan or Ehlers-Danlos syndrome must be suspected in patients with connective tissue abnormalities and an acute mitral regurgitation secondary to chordae tendineae rupture, although a primary, preexisting mitral valve prolapse (MVP) is the most common cause. Some cases may be idiopathic (individuals who experience rupture of the chordae tendineae without previous MVP or connective tissue disease), but an etiology can be found in most occasions. 41% of people answered this question correctly. Case 148

A 64-year-old healthy male comes to your office after a local emergency department (ED) visit for an episode of paroxysmal atrial fibrillation, which ceased spontaneously three weeks ago. The ED doctor gave him a prescription for metoprolol, and advised a follow-up appointment with you, the primary care physician. He currently has no complaints. He takes no other medications. Physical examination revealed no abnormalities. EKG showed normal sinus rhythm. A 2D-Echocardiogram is ordered, which revealed cardiac dilatation and an ejection fraction of 45%. Which of the following is the most appropriate next step in diagnosis? A. Refer the patient for ambulatory monitoring of blood pressure (AMBP). B. Refer the patient for cardiac stress test. C. Refer the patient for renal artery Doppler ultrasound. D. Refer the patient for polysomnography. E. Check BNP (B-Natriuretic peptide) and troponins. Explanation: Ischemic heart disease is the most common cause of congestive heart failure (CHF), especially dilated cardiomyopathy, in the United States. Approximately 50 to 75% of the patients with heart failure (HF) have coronary disease as the etiology. Other known etiologies are: hypertension (13%), valvular disease (10-to12%), renovascular disease, and very rare causes, such as obstructive sleep apnea, myocarditis, alcohol or cocaine abuse, etc. Although this patient is asymptomatic, his echocardiogram results (cardiac dilatation and low ejection fraction) are very suggestive of CHF. The next step in the management of this patient is to identify the etiology of his CHF. Since ischemic heart disease is the most common etiology, a cardiac stress test should be done first. The objective here is to quickly identify the presence of ischemia to address the need for coronary angiography (for possible revascularization). (Choices A, C and D) Hypertension, renovascular disease, and sleep apnea are less common causes of CHF. (Choice E) Troponin and BNP level determination is not useful in this setting. Troponins can be elevated in any type of heart failure or coronary ischemia. This is also true for BNP (b-natriuretic peptide), which is mainly used to distinguish cardiogenic pulmonary edema from primary pulmonary conditions. Educational Objective: The most common cause of heart failure (CHF) is ischemic heart disease. In a new case of CHF with unknown etiology, efforts must first be made to rule out the presence of coronary lesions which may be corrected by an angioplasty. Other important causes of CHF are hypertension, valvular and renovascular disease. 53% of people answered this question correctly. Case 149 The following vignette applies to the next 2 items A healthy 24-year-old Japanese woman comes to the physician because of a lesion on her left cheek. Two years ago, she already noticed a small mole on her left cheek, but this lesion has recently grown in size. She does not have any other complaints. Her social history is not significant. Vital signs are within normal limits. Examination shows a 4 cm dark brown, slightly elevated macule with slightly irregular borders on the left cheek. Item 1 of 2 Which of the following is the most likely diagnosis? A. Keratoacanthoma B. Pigmented basal cell carcinoma

C. Seborrheic keratosis D. Malignant melanoma E. Verruca vulgaris Explanation: Malignant melanoma is a concern if a nevus increases in size and develops irregular borders. It can be confused with an atypical melanocytic nevus; however, this disease is infrequent in the Asian population. These lesions can appear spontaneously or develop over a previously atypical nevus. (Choice C) Seborrheic keratosis is rare before the age of 30. It also has a "stuck on" appearance, with a "warty" form. (Choice A) Keratoacanthoma is common on the cheek. It is a low-grade malignancy that pathologically resembles squamous cell carcinoma. It appears as a solitary, firm, round, skin-colored or reddish plaque that develops into a nodule with a central keratin plug. (Choice E) Verruca vulgaris has the same warty appearance as seborrheic keratosis, and is a welldemarcated lesion. (Choice B) Basal cell carcinoma is usually pink or red in color. It is more frequent in males, and it rarely appears before the age of 40. Educational Objective: It is important to recognize the presence of a melanoma, because excision can be curative if the lesion is identified early. Melanoma lesions are usually asymmetric, with color variegation, and usually more than 5 mm in diameter. 69% of people answered this question correctly. Item 2 of 2 Which of the following is the most appropriate statement in this patient?s management? A. A biopsy is not required since the clinical presentation is so classic. B. An excisional biopsy is the next best step. C. Administer intralesional corticosteroids. D. Observation and repeat follow-up in six weeks. E. Start therapy with interferon alfa-2b. Explanation: The sensitivity of a melanoma diagnosis by a dermatologist based on the history and physical examination is approximately 85%; however, false negatives have been reported in 15% of the time. Due to the significant amount of false negatives reported, an excisional biopsy is essential for confirmation of the diagnosis and staging of the lesion. Complete excision is the treatment of choice. (Choice E) Interferon alfa-2b is used as an adjuvant therapy in patients with a high risk of developing metastatic disease. Educational Objective: An excisional biopsy is the next best step in patients with a suspected malignant melanoma. 86% of people answered this question correctly. Case 150

A 24-year-old female who is on methimazole 10 mg twice daily for the treatment of Graves' disease comes to your office for a follow-up. She was diagnosed with Graves' disease approximately six months ago. Her clinical symptoms have markedly improved after she started taking methimazole. She complains of nausea for the past two weeks, but does not have any other symptoms. She opted for treatment with antithyroid drugs instead of radioactive iodine ablation because she thought of becoming pregnant in the near future. She is married and has no children. She does not smoke or drink alcohol. Her mother has hypothyroidism. Besides methimazole, she does not take any other medications. She denies allergies to medications. She is sexually active, and does not use any form of contraception. Her last menstrual period was two months ago. Her prior menstrual cycles were regular. Physical examination is unremarkable. Thyroid function tests performed five days before this visit revealed a TSH of 0.7 mU/ml (normal 0.35 mU/ml-5.0 mU/ml) and total T4 of 15.2 mg/dl (normal 4 to 12 mg/dl). Aside from ordering a pregnancy test, what is the next best step in this patient?s management? A. Increase the dose of methimazole B. Change to propylthiouracil C. Check free T4 D. Check total T3 E. Continue same dose of methimazole Explanation: The patient is sexually active, does not use any form of contraception, and has missed her period. She has nausea, and her total T4 is elevated. It is very likely that she is pregnant. (During pregnancy, thyroid binding globulin and total T4 are increased.) Methimazole should be discontinued, and propylthiouracil should be started. Thyrotoxicosis during pregnancy can also be treated with a thyroidectomy during the second trimester (safest) if the patient is intolerant to propylthiouracil. Radioactive iodine is contraindicated during pregnancy. (Choice A) Methimazole is known to cause a scalp defect (aplasia cutis) in the fetus. Even if the patient is not pregnant, increasing methimazole is still not indicated because her TSH level is normal and she is euthyroid . (Choice E) Continuing methimazole in the same dose is a reasonable option if the patient is not pregnant; however, since pregnancy is highly suspected in this case, the patient's medication should be changed to PTU. (Choices C and D) Total T4 is elevated during pregnancy. Free T4, on the other hand, may be normal to high, but usually normal in the majority of cases. TSH is the most sensitive indicator of primary thyroid dysfunction; therefore, free T4 and total T3 will not add to existing information. Educational Objective: Methimazole is discontinued during pregnancy because it is known to cause a scalp defect (aplasia cutis) in the fetus. Propylthiouracil is the drug of choice. 49% of people answered this question correctly.

A 78-year-old Caucasian woman with a history of metastatic colon cancer, malignant ascites, and congestive heart failure collapses at her son?s home. She is escorted by ambulance to the hospital, where she is admitted to the intensive care unit. In the days leading up to her admission she had complained of fever, productive cough, shortness of breath, and malaise. Shortly after admission she develops respiratory distress that requires sedation, intubation, and ventilatory support. Widespectrum intravenous antibiotics are begun to treat suspected pneumonia, and her diuretic dosage is increased. Responsible for her care are a number of specialists, including an oncologist, pulmonologist, cardiologist, and critical care hospitalist. Each physician meets separately with the patient?s adult children every day to discuss his assessment and treatment recommendations. The patient?s condition moderately improves but all attempts to wean her from the ventilator fail. As each day passes, the family members become increasingly irritable when discussing their mother?s health problems, and finally accuse you of suggesting unnecessary procedures and treatments. Which of the following is most likely responsible for the family?s negative attitude? A. Inadequate pain management B. Violation of patient?s living will directive against intubation C. Poor nursing care D. Concern that health care insurance will not cover cost of hospital admission E. Lack of communication between physicians involved in patient care Explanation: The issues of greatest concern for families of patients with cancer include pain management and the establishment of excellent communication with the health professionals involved. If multiple physicians are responsible for a patient?s medical care, it is crucial that they communicate regularly about key treatment objectives and the preferred means of accomplishing those objectives. When such communication is present, all physicians involved are well equipped to provide family members with a clear and cohesive picture of the medical team?s assessment and plan. If such communication is absent, however, family members are left feeling that care is being provided in a haphazard or ineffective fashion (Choice E). Since the patient is sedated and there is no reference made to her experiencing any pain or discomfort since her arrival at the hospital, it is unlikely that her family feels her pain management is inadequate (Choice A). No mention was made of a living will, and it is unlikely that the family members would express concern about intubation several days after it was performed (Choice B). There is no evidence that nursing care or the cost of hospital admission is of concern for the family members (Choices C and D). Educational Objective: If multiple physicians are responsible for a patient?s medical care, it is crucial that they communicate regularly so that family members are provided with a clear and cohesive picture of the medical team? s assessment and plan. 80% of people answered this question correctly. The following Vignette applies to the next 2 items A 29-year-old female who emigrated from India about five years ago comes to your office for a routine visit. The patient has no medical problems, and is on the 26th week of her first pregnancy. She is asymptomatic and takes prenatal vitamins. Her family history is positive for type-2 diabetes on her father's side. She does not smoke or drink. She lives with her husband and works as a research assistant in a pharmaceutical company. She denies any over-the-counter medication use. On physical examination, she is five feet (150 cm) tall and weighs 180 lbs. (81 kg). The rest of the

examination is unremarkable. Her one-hour blood glucose level after ingesting 50-gm glucose is 160 mg/dL.

Item 1 of 2 What is the next best step in the management of this patient? A. Perform 100-gm glucose tolerance test with blood glucose measurements for 3 hours. B. Perform 75-gm oral glucose tolerance test with blood glucose measurements for 3 hours. C. Repeat 50-gm glucose tolerance test in 3 months. D. The patient has a normal 50-gm glucose tolerance test; there is no need to repeat a glucose tolerance test. E. Measure HBA1c. Explanation: All pregnant patients are generally screened for gestational diabetes between the 24th and 28th weeks of pregnancy using the 50 gm glucose tolerance test. This test does not require fasting. Blood glucose levels are checked one hour after the ingestion of 50 gm of glucose. Patients with blood glucose values of 140 mg/dL or higher should be subjected to a 3-hour glucose tolerance test after the ingestion of 100 gm of glucose on a fasting state. Two or more blood glucose values greater than 105, 190, 165 and 145 mg/dL at 0, 1, 2 and 3 hours, respectively, are diagnostic of gestational diabetes. Some workers have proposed lower cutoff values for the diagnosis of gestational diabetes. The American Diabetes Association is recommending the use of a 75-gm glucose tolerance test, with different cut-off values, for use in non-pregnant women and for diagnosing gestational diabetes. (Choice D) The patient is at a high risk for developing gestational diabetes since her screening 50-gm glucose tolerance test is abnormal. Normal values for blood glucose levels one hour after the ingestion of 50-gm glucose should be less than 140 mg/dL. (Choice C) The 50-gm glucose tolerance test is used for screening, and is not repeated once the results are abnormal. (Choice E) HbA1c is an excellent test for monitoring glycemic control; however, its role in the diagnosis of diabetes is not very clear. Educational Objective: Understand the screening and diagnostic glucose tolerance test for gestational diabetes mellitus. A 50-gm oral glucose challenge with blood glucose values of 140 mg/dL or higher is generally followed by a 100-gm oral glucose tolerance test with hourly blood glucose measurement for 3 hours. 59% of people answered this question correctly. Item 2 of 2 After the appropriate evaluation, a diagnosis of gestational diabetes is made. A diet and exercise regimen is started. The patient says that she is usually compliant with her diet, but she occasionally overeats. Her weight gain and blood pressure during pregnancy are within normal limits. She checks her fasting and post-lunch blood glucose values daily. On the third week of dietary restriction, her fasting blood glucose values range from 90 and 110 mg/dL, and her 2 hour post-lunch glucose values range from 120-140 mg/dL. What is the most appropriate way to manage this patient? A. Continue dietary restriction B. Start glimepiride C. Start metformin D. Start NPH insulin E. Start Glargine insulin Explanation: The recommended fasting blood glucose values in pregnant diabetic patients should range between 60-90 mg/dL, and postprandial blood glucose should be less than 120 mg/dL. This patient should be

started on NPH insulin (usually as a single dose at bedtime). The insulin dose and frequency is then adjusted, depending upon her glycemic control. Rapid acting insulin (regular or lispro) may be added for persistent postprandial sub-optimal blood glucose levels. (Choice A) The patient?s glycemic control is suboptimal. Simply continuing the dietary restrictions will not control her blood glucose values. (Choices B and C) Glimepiride and metformin are not used during pregnancy to obtain glycemic control. (Choice E) Glargine is a long acting insulin, and is not indicated during pregnancy because of the teratogenic effects observed in experimental animals. Educational Objective: The recommended fasting blood glucose values in pregnant diabetic patients should range between 60-90 mg/dL, and postprandial blood glucose values should be less than 120 mg/dL. NPH in combination with regular or lispro insulin is generally recommended if diet and exercise are not able to adequately control the blood sugar. Oral hypoglycemics are not indicated in pregnant patients. The use of glargine insulin is not considered safe during pregnancy. 25% of people answered this question correctly. The following Vignette applies to the next 3 items You are the current "on call physician" for your group, when you receive a call regarding one of your nursing home patients. The nurse reports that one patient has been acting strangely for the last few hours and is agitated at times. This patient is an 82-year-old female who has been in the nursing home for the last two years, and has always been very pleasant. You know her very well from your recent nursing home visit. She has a past medical history of hypertension, diabetes mellitus, osteoporosis, and Alzheimer?s dementia. Her daily medications are amlodipine, metformin, calcium vitamin D, and donepezil. You are concerned that something is not right with the patient. Item 1 of 3 Which of the following is the most appropriate next step in the management of this patient? A. Transfer the patient to a nearby hospital B. Give a dose of haloperidol C. Ask the nurse to check vital signs and pulse oximetry D. Give diazepam for sedation E. Ask the nurse to use physical restraints as necessary to prevent falls and injury Explanation: This patient is suffering from an acute confusional state, also known as delirium. Delirium is extremely common in old nursing home patients. It manifests as a change in the level of awareness, easy distractibility, cognitive and perceptual disturbances including memory loss, disorientation, and even agitation. It typically develops over hours to days, and this acute onset distinguishes it from dementia. Dementia is a multifactorial disorder. The factors that increase the risk of delirium are subdivided into those that increase the susceptibility to delirium and those that precipitate delirium. The most common factors that increase the patient?s susceptibility to delirium include advanced age and an underlying brain disorder (i.e., dementia, Parkinson?s disease, and history of prior stroke). The factors that precipitate delirium include multiple medication use (polypharmacy), infections (i.e., urinary tract infections and pneumonia), fluid and electrolyte disturbances (dehydration, hyponatremia, or hypernatremia), malnutrition, immobility (including the use of restraints), use of bladder catheters, and a wide variety of other medical conditions. In fact, delirium may be the only presenting complaint

in an older demented patient suffering from an acute medical illness; therefore, checking this patient? s vital signs should be the first step to rule out early infections and fluid and electrolyte disturbances. (Choice A) Transferring this patient to a hospital may be necessary for further evaluation of the cause of the delirium; however, this should not be the first step in the management of this patient. (Choices B and D) Misuse of antipsychotics and benzodiazepines is very common in managing behavioral problems in demented nursing home patients. It leads to frequent adverse effects and worsening of the cognitive status; therefore, it is important to rule out any medical, environmental, or psychosocial causes before prescribing any psychotropic medications. (Choice E) As explained above, the use of physical restraints can precipitate and even worsen the delirium, and should therefore be avoided. Educational Objective: Delirium can be the only finding of an acute medical condition in older demented individuals. Use of psychotropic medications as a first line therapy for behavioral problems is not recommended. 65% of people answered this question correctly. Item 2 of 3 The nurse calls you back in tweny minutes. She states that she gave a dose of prn (as needed) medication which was ordered by a nursing physician before. She arranged an ambulance to transfer the patient to a nearby hospital. Her temperature is 36.7 C (98 F), blood pressure is 116/78 mmHg, pulse rate is 76/min, and respiratory rate is 18/min. She is still agitated, and she is now disoriented to place and person. She gets easily distracted, and she is having visual and auditory hallucinations. In the hospital, her physical examination is unremarkable. What is the most appropriate next step in the management of her condition? A. Give haloperidol to calm the patient B. Order a stat psychiatry consult C. Order urine analysis and complete metabolic panel D. Order a head CT scan E. Order physical restraints to prevent injury to patient and nursing staff Explanation: This patient is still exhibiting the classic signs and symptoms of delirium. Cognitive and perceptual disturbances including hallucinations are very common in delirious patients. As described above, virtually any acute medical condition can precipitate delirium in an older demented patient. It is important to perform a complete physical examination and obtain basic laboratory studies to identify or rule out the potential causes of delirium, especially the reversible causes, which include: dehydration, hypo/hypernatremia, infections, hypoglycemia, hypercalcemia, uremia, and liver failure. (Choices A and B) The use of psychotropic medications to calm this patient is not recommended at this time. It is prudent to rule out the reversible medical causes of delirium before prescribing psychotropic medications or obtaining a psychiatric consultation. (Choice D) There is no history of head trauma or any focal neurological signs and symptoms; therefore, a head CT scan is not indicated at this point. (Choice E) Physical restraints should be used only as a last resort in an agitated and delirious patient. They frequently increase confusion and agitation. Constant observation, preferably by someone familiar to the patient, is more helpful in such situations.

Educational Objective: Focused history, physical examination and basic laboratory work-up should be performed early in the initial evaluation of delirium in any patient. 63% of people answered this question correctly.

Item 3 of 3 A CT scan of the patient's head is normal. Initial lab studies reveal the presence of a urinary tract infection with mild prerenal azotemia. You start the patient on empiric antibiotic therapy and intravenous fluids; however, the patient?s mental status remains unchanged. You provide a bedside sitter for the patient for constant supervision. The sitter reports that the patient is very combative now. She has been trying to get out of bed and has twice attempted to pull her IV line out. She resisted the physical restraints that the nurse was trying to apply. Frequent orientation and reassurance has not improved her behavior. Which of the following is the most appropriate next step in the management of this patient? A. Give low dose haloperidol B. Give low dose lorazepam C. Order physical restraints to prevent injury anyway D. Provide another sitter for the patient E. Perform lumbar puncture Explanation: The patient is exhibiting behavior that could harm herself and the healthcare staff. Initial supportive measures such as reassurance, orienting her to the environment, and having constant supervision has not been successful in managing her disruptive behavior. Low-dose haloperidol may be useful in controlling her symptoms, preventing harm, and allowing for the safe continuation of her treatment. Antipsychotics are associated with a variety of side effects. These include sedation, orthostatic hypotension, anticholinergic side effects, tardive dyskinesia, and extrapyramidal symptoms. Thus, the patient should be started at lower doses and the dose is then titrated upwards to achieve the desired effects. (Choice B) Benzodiazepines have a more rapid onset of action, but these can worsen confusion and sedation. They can be used as an adjunct to antipsychotics to reduce extrapyramidal side effects. (Choice C) Physical restraints should be only used as a last resort in a combative patient. They frequently lead to worsening of confusion. They are used when medications fail to act, and the patient remains a significant threat to his/her and the healthcare staff's safety. (Choice D) Sitters are helpful in providing orientation and constant supervision for a confused patient. Once a patient becomes combative, other modalities including chemical and physical restraints are frequently necessary. (Choice E) Performing a lumbar puncture is only indicated if the patient has unexplained fever, altered mental status, and nuchal rigidity. Change in mental status alone is not an indication for lumbar puncture. Educational Objective: Physical and chemical restraints should be used judiciously in a combative and disruptive patient. These help prevent injuries and falls, and allow for the continuation of medical therapy. Always give haloperidol (antipsychotics) before you apply restraints.

56% of people answered this question correctly.

The following Vignette applies to the next 2 items You are taking care of a 65-year-old male patient in the intensive care unit. The patient had an unwitnessed, out-of-hospital cardiac arrest two days ago. He was intubated and successfully resuscitated after a prolonged CPR in the field. Two days later, he remains intubated, and relies on full ventilator support. He is unresponsive to verbal and tactile stimuli. His temperature is 37.2 C (99 F), blood pressure is 110/70 mmHg, and pulse is 70/min. After a lengthy discussion with the patient?s family, you all agree to have the patient's life-sustaining support withdrawn.

Item 1 of 2 Which of the following will you document to determine that the patient meets the criteria for brain death? A. Absence of respiratory drive for 5 minutes off the ventilator B. Body temperature below 35 C C. EEG with nonspecific waveforms D. Intermittent cerebral circulation on cerebral doppler scanning E. Irreversible absence of cerebral and brainstem reflexes Explanation: Brain death is defined as the cessation of cerebral and brain stem function. A person is considered legally dead in the United States when criteria for brain death have been demonstrated. One of the criteria to determine brain death is the irreversible absence of cerebral and brainstem reflexes including pupillary, oculocephalic, oculovestibular (caloric), corneal, gag, sucking, swallowing, and extensor posturing. Some of the other criteria for determination of brain death include: 1. Absence of respiratory drive (apnea) off the ventilator for a duration that is sufficient to produce hypercarbic drive (usually 10 to 20 minutes to achieve pCO2 of 50 to 60 mmHg) (Choice A). 2. Body temperature above 34 C (93.2 F) (Choice B). 3. EEG isoelectric for 30 minutes at maximal gain (Choice C). 4. Absence of cerebral circulation by Doppler or magnetic resonance angiography (Choice D). 5. At least 24 hours of observation in adults with anoxic-ischemic brain damage with a negative drug screen Educational Objective: Brain death is the irreversible absence of all cerebral and brainstem reflexes. There are no spontaneous breaths regardless of hypercarbia or hypoxemia. 68% of people answered this question correctly. Item 2 of 2 Using the above criteria, you are able to document that the patient is brain dead. You discuss these findings with his family and recommend that mechanical ventilator support be withdrawn. Everyone in the family appears to understand the situation and agrees to proceed with the withdrawal of ventilatory support. One relative then walks into the patient?s room and notices some movements in the patient's legs. He gets angry with you and claims that the patient is not dead. Which of the following is the most appropriate response to the relative?s reaction at this time? A. Tell the family that leg movements are inconsistent with the diagnosis of brain death B. Arrange for further testing to confirm your diagnosis C. Obtain a neurology consultation D. Tell the family that the relative is probably hallucinating E. Explain calmly that limb movements can be seen normally even in a brain dead person Explanation: Brain death is defined as the irreversible absence of cerebral and brainstem reflexes including pupillary, oculocephalic, oculovestibular (caloric), corneal, gag, sucking, swallowing, and extensor posturing. Purely spinal reflexes, including tendon reflexes, plantar reflexes, and limb movements to painful stimuli can be present in these patients. Explaining this in a calm manner should be the appropriate response to dissipate the anger and anxiety among the family members. (Choice A) The presence of limb movements is not inconsistent with the diagnosis of brain death . (Choices B and C) Any further testing or consultations based on limb movements alone is not indicated if the criteria for brain death have been fulfilled earlier. Educational Objective:

It is not unusual to see purely spinal reflexes manifesting as isolated limb movements in a brain dead person. 78% of people answered this question correctly. A healthy, 31-year-old African-American woman is brought to the emergency department because of fever, chills, abdominal pain, and frequency for the past five days. She vomited twice today. Her symptoms are progressively getting worse. She does not use tobacco, alcohol, or drugs. Her last menstrual period was one week ago. She is sexually active and uses oral contraception. She has no known drug allergies. Her temperature is 38.9 C (102 F), blood pressure is 120/70 mmHg, pulse is 100/min, and respirations are 14/min. The abdomen is soft, tender, and nondistended. Bowel sounds are increased. There is no rebound tenderness or rigidity. There is no splenomegaly and no costovertebral angle tenderness. Test of the stool for occult blood is negative. Gynecologic exam revealed some fullness and tenderness on palpation of both adnexa, as well as tenderness on lateralization of the cervix. A mild, yellowish vaginal discharge is noted. There is no tenderness to palpation of the lower back. The patient?s laboratory tests reveal: CBC Hb: 12.6 g/dL Ht: 38% MCV: 90 fl Leukocyte count: 15,000/cmm Segmented neutrophils: 80% Bands: 7% Eosinophils: 0% Lymphocytes: 12% Monocytes: 1% The urinalysis results are unremarkable. Which of the following is the most appropriate pharmacotherapy? A. Intravenous amikacin and intramuscular ceftriaxone B. Intravenous cefoxitin and intravenous doxycycline C. Oral azithromycin and intravenous doxycycline D. Oral doxycycline and intramuscular ceftriaxone E. Intravenous ceftriaxone and intravenous metronidazole Explanation: This patient has acute pelvic inflammatory disease (PID). She has abdominal pain and tenderness on palpation of the adnexae and lateralization of the cervix. The presence of fever, chills, bandemia, leukocytosis, as well as nausea and vomiting, indicates that the infection is severe. PID can be a serious condition that needs to be treated immediately. Intravenous administration of cefoxitin and doxycycline is the best combination of all the given options. (Choices A, C, and D) Intramuscular ceftriaxone, oral azithromycin or oral doxycycline can be used if there is mild or moderate infection, but not in this case. (Choice E) Intravenous ceftriaxone is a good choice, but should not be accompanied by metronidazole. Educational Objective: Treatment of severe PID is aimed at obtaining high blood concentrations of the antibiotics as soon as possible. For this reason, all therapy should be intravenous. Combinations that can be used are: cefoxitin + doxycycline or ceftriaxone + doxycycline. If a tubo-ovarian abscess is diagnosed, surgical evaluation is immediately needed. 46% of people answered this question correctly.

A randomized, double-blinded clinical trial was conducted to evaluate the effect of a new hypolipidemic drug on the survival of patients after PTCA. 1000 patients undergoing PTCA were randomly assigned to the drug or placebo group. 500 patients were in each group. The two groups were then followed for three years for the development of acute coronary syndromes. Severe acute myositis was reported as a rare side effect of the drug therapy, but the difference between the two groups in the occurrence of this side effect was not statistically significant (p = 0.09). The same side effect was reported in several small clinical trials of this drug. Which of the following is the most likely cause for the failure to detect a significant statistical difference in the occurrence of acute myositis between the treatment and placebo groups? A. Ascertainment bias B. Short follow-up period C. Inappropriate selection of the patients D. Small sample size E. Observer?s bias Explanation: Power is the probability to detect a difference in the outcome of interest between two groups, if such a difference exists. Interestingly, acute myositis was reported in several small clinical trials of the drug; in this study, the results are not statistically significant. Because this side effect is rare and few patients experienced it, the p value is marginal approaching, but not reaching statistical significance. A bigger sample size will increase the power of the study, (the ability to detect the difference), and the p value will reach statistical significance. (Choice A) Diagnosing severe acute myositis is not a big clinical challenge using appropriate diagnostic techniques. Well-organized clinical trials have little potential for ascertainment bias for such events. (Choice B) Increasing the follow-up period may not increase the power, because the side effect can occur in susceptible individuals relatively early after starting the therapy, and long follow-up would not increase the number of events. (Choice C) No information is given on how the patients were selected for the study. (Choice E) Double-blinding techniques decrease the potential for observer?s bias. Educational Objective: Power is the ability to detect a difference in the outcome of interest between two groups, if such a difference exists. Increasing the sample size increases the power of a study. 37% of people answered this question correctly. A 26-year-old white female presents ten weeks after delivery with acute, mid-thoracic back pain. Xray of the thoracic spine revealed a compression fracture of the T8 vertebra. Her pregnancy was complicated by extensive deep venous thrombosis (DVT), for which she was on subcutaneous heparin for the last 20 weeks of her pregnancy. Her past medical history, aside from her history of DVT, is unremarkable. Her family history is unremarkable. She has not had any menstrual periods since her delivery. She is currently breast-feeding her baby. She takes a daily supplement of 1200 mg of elemental calcium and 800 international units of vitamin D. Tenderness over the mid-thoracic spine is noted. The rest of the physical examination is unremarkable. The serum calcium, phosphorus, and 25-hydroxyvitamin D levels are normal. What is the next best step in the management of this patient? A. Double the dose of calcium and vitamin D B. Check serum PTH levels

C. Start on hormone replacement therapy D. Discontinue breast-feeding E. Start alendronate Explanation: Normal females can lose up to 4-6% of their bone mass during the first 4-6 months of the postpartum period, but most will regain all this lost mass afterwards. Bone mineral density can also be significantly reduced by long-term heparin use. This patient presents with an exaggerated bone loss, which can be explained by the combined effects of heparin use during pregnancy and increased bone loss during the postpartum state. Discontinuation of breast-feeding in the postpartum state has been shown to preserve or improve bone mineral density, and will most likely be helpful in this patient. The reasons for the improvement of bone density after cessation of breast-feeding are decreased calcium loss and decrease in the levels of parathyroid hormone related peptide (PTHrP). PTHrP is possibly important for calcium transfer into the breast milk, and its levels are increased during nursing. Increased levels of PTHrP can cause increased bone resorption. (Choice A) Unfortunately, increasing calcium intake has not been shown to be effective in the restoration of bone loss during the postpartum period. The patient appears to be vitamin D sufficient; therefore, increasing the dose of vitamin D will not be helpful. (Choice B) The patient has normal serum calcium and phosphorus levels, and her clinical features are not suggestive of hyperparathyroidism. Checking the serum PTH level is not indicated. (Choice C) As the patient has a history of DVT, hormone replacement therapy is contraindicated. (Choice E) Alendronate has not been adequately studied in premenopausal patients with bone loss, and is not indicated as the next best step. Educational Objective: Some patients during the postpartum state may lose significant bone mass, predisposing them to fragility fractures. Discontinuation of breast-feeding helps prevent further loss of bone mass in these patients. 8% of people answered this question correctly. An 80-year-old white male with a history of hypertension, hypercholesterolemia, coronary artery disease, and coronary artery bypass graft comes to the emergency department because of a two-day history of abdominal pain and bloody diarrhea. The patient describes the pain as severe and predominantly at the left upper quadrant. The diarrhea started a few hours after developing the abdominal pain. His blood pressure is 130/80 mmHg, pulse rate is 98/minute, temperature is 37.2?C (98?F), and respirations are 16/min. His initial labs showed slightly elevated amylase levels. Nasogastric tube aspiration showed a clear aspirate. An upright abdominal x-ray showed no free air, but edematous and dilated transverse colon was seen. Sigmoidoscopy showed mucosal edema, but no other lesions. Based on the above presentation, which of the following is the most likely diagnosis? A. Acute pancreatitis B. Acute ischemic colitis C. Acute diverticulitis D. Pseudomembranous colitis E. Gastric ulcer Explanation: This patient has all the risk factors for acute ischemic colitis. He has hypertension, hypercholesterolemia, and coronary artery disease suggestive of atherosclerotic disease. Acute ischemic colitis typically presents with abdominal pain followed by bloody diarrhea. The most

vulnerable areas are watershed areas, which include the splenic flexure and rectosigmoid junction. Patients usually have an elevated white count. X-rays and sigmoidoscopies usually show mucosal edema and mucosal ulcerations. (Choice A) Acute pancreatitis is very unlikely to cause bloody diarrhea. Amylase can be elevated in various conditions, which could cause damage to the bowel wall. (Choice C) Acute diverticulitis usually presents with left lower quadrant abdominal pain, fever, and an elevated white count. Evidence of diverticula is almost always found on sigmoidoscopy. (Choice D) Pseudomembranous colitis is usually seen in elderly patients who reside in nursing homes and other long-term care facilities. Patients usually present with abdominal pain, fever, and nonbloody diarrhea. They can have an elevated white count. Stool Clostridium difficile toxin assay is positive. Prior history of antibiotic use is also present in majority of the patients. (Choice E) In 10-15% of the patients, upper GI bleeding can present with severe bloody diarrhea; however, most of the times, nasogastric tube aspiration will reveal either blood or coffee-ground aspirate. Educational Objective: Acute ischemic colitis is seen in patients who have evidence of atherosclerotic disease, and is usually manifested by acute abdominal pain followed by bloody diarrhea. Acute pancreatitis does not usually cause bloody diarrhea. 84% of people answered this question correctly. A 62-year-old Chinese-American man comes to the physician for a general medical evaluation. He has been having intermittent urinary frequency for the past two months. His other medical problems include hypertension, hypercholesterolemia, and chronic obstructive pulmonary disease (COPD). He quit smoking a few years ago when he was diagnosed with COPD. He does not use alcohol or drugs. His medications include hydrochlorothiazide, lovastatin, and ipratropium metered-dose inhaler. His blood pressure is 160/90 mmHg. Rectal examination shows a diffuse, smooth, and moderately enlarged prostate. Which of the following is the most appropriate pharmacotherapy? A. Add enalapril B. Add metoprolol C. Add amlodipine D. Stop ipratropium bromide E. Add prazosin Explanation: The seventh report of the Joint National Committee on prevention, detection, evaluation, and treatment of high blood pressure (JNC VII) states that alpha-1-blockers (i.e. prazosin, terazosin) are indicated for the management of patients with both hypertension and benign prostatic hypertrophy (BPH). Alpha-1-blockers are also used in diabetic patients because they increase insulin sensitivity, decrease LDL, and increase HDL cholesterol. (Choice D) Stopping ipratropium bromide will probably improve the prostatic symptoms, but will not be of any benefit to the blood pressure control and COPD management of this patient. (Choice B) Metoprolol, a beta-blocker, can worsen prostatic symptomatology, bronchoconstriction, decrease insulin sensitivity, and mask hypoglycemic symptoms. produce

(Choice A) Enalapril can decrease renal and retinal damage, especially in diabetic patients, but does not improve prostatic symptoms. (Choice B) Amlodipine will only help in the blood pressure control of this patient. Educational Objective:

The seventh JNC report recognizes BPH as a clear indication for alpha-1-blocker use in hypertension management, unless the patient has congestive heart failure or develops persistent dizziness. Alpha-1 blockers are also beneficial in patients with type-2 diabetes mellitus because they increase insulin sensitivity, decrease LDL cholesterol, and increase HDL cholesterol. 89% of people answered this question correctly. A 14-month-old Caucasian boy is brought to your office for routine check-up. The child had an upper respiratory infection and otitis media one month ago that was treated with antibiotics and subsided quickly. Otherwise, his past medical history is insignificant. He has been meeting all the developmental milestones appropriately. He does not have any known allergies. His older sister was diagnosed with polycystic kidneys, infantile type and underwent kidney transplantation two months ago. All his vaccinations are up-to-date. Which of the following is the best statement concerning varicella vaccination in this patient? A. Varicella vaccine is a killed vaccine and can be safely administered. B. Varicella vaccine should not be administered to this patient. C. Varicella vaccine can be administered and the child monitored for the appearance of a rash. D. Varicella vaccine can be administered and the child isolated for several weeks. E. Varicella vaccine should be administered with varicella immune globulin (VZIG). Explanation: Varicella vaccination of household contacts of transplant recipients is relatively safe and is currently recommended by the American Association of Pediatrics (AAP). It is advisable for all patients to receive pre-transplantation immunization, although this is not always possible and durable protection is reached in less than 50% of patients. Transmission of vaccine-associated VZV is not typical, except for some cases when post-vaccination rash appears; therefore, this child should be monitored for the appearance of a rash and isolated if a rash appears (Choice D). (Choice A) The varicella vaccine is a live virus vaccine. (Choice E) Varicella immune globulin should be administered to varicella seronegative transplant recipients exposed to varicella or zoster. Educational Objective: Varicella vaccination of household contacts of transplant recipients is relatively safe and is currently recommended. 31% of people answered this question correctly. A study was conducted to assess the relationship between the use of an over-the-counter pain reliever during pregnancy and the development of a neural tube defect in the offspring. Mothers whose children have neural tube defects, as well as age-matched controls (mothers whose children do not have such abnormalities), were interviewed using a standard questionnaire. The study showed that the use of the pain reliever during pregnancy increases the risk of a neural tube defect, even after adjusting for race, other medication use, family history of congenital abnormalities, and serum folate levels. The odds ratio (OR) is 1.5; p is 0.03. Which should be the major concern while interpreting the results of this study? A. Selection bias B. Interviewer bias C. Recall bias D. Observer bias E. Confounding

Explanation: Recall bias should always be considered as a potential problem in case-control studies because it can cause an overestimation of the effect of an exposure. In this scenario, the women whose children have neural tube defects are more likely to report use of the drug than the women whose children are healthy. This over-report is generally due to psychological trauma induced by the birth of the baby with a congenital abnormality and the search for potential explanations of the problem. Other typical scenarios that may illustrate recall bias are: women whose husbands are diagnosed with lung cancer tend to over-report the number of the cigarettes smoked daily by the patient; patients with melanoma tend to over-report low tanning ability. (Choice A) No information is given on how the patients were selected for the study. (Choice B) Using a standard questionnaire minimizes the potential for interviewer bias. (Choice D) Observer bias results in misclassification of the outcome (e.g., labeling a healthy baby as having an abnormality and vice versa). It is explained by the knowledge of the exposure status of the patients by the physician who makes the diagnosis. In this scenario, diagnosis of the congenital abnormality was made at birth; therefore, there is little potential for observer bias. (Choice E) Two techniques were used to control the potential confounders in this study: matching and adjustment during the data analysis. Educational Objective: Recall bias is a potential problem in case-control studies. It results in over-reporting of the exposure by the cases. 57% of people answered this question correctly.

The following vignette applies to the next 3 items A 60-year-old male comes to your office for a regular physical examination. He has mild hypertension, which is being controlled with diuretics. Physical examination shows palpable kidneys. His BUN and serum creatinine levels are mildly elevated. Ultrasonogram of the abdomen shows multiple cysts in both kidneys. A diagnosis of autosomal dominant polycystic kidney disease (ADPKD) is made. Item 1 of 3 What is the ideal follow-up plan for this patient? A. Regular blood pressure checks B. Keep diastolic blood pressure less than 95 mmHg C. Yearly ultrasonography of the kidneys D. Yearly CT scan of the abdomen E. MRI of brain with contrast Explanation: ADPKD has a prevalence rate of approximately 1:500, and accounts for 10% of ESRD patients in the United States. A progressive decline in renal function commonly occurs with this disease, and the course of this decline is variable. Poor prognostic factors for a rapid decline in renal function include severe hypertension, male sex, early age of diagnosis, and early development of renal dysfunction. ACE inhibitors are considered as the drugs of choice for controlling hypertension and preventing progression of renal failure in these patients. In this case, the patient is at low risk for progression of renal failure. Furthermore, since his blood pressure is controlled with diuretics, all he requires are regular follow-up visits to ensure that his blood pressure is maintained within normal limits. (Choice B) The ideal goal for blood pressure control in any patient with chronic kidney disease, including ADPKD, is less than 130/80mm Hg. A diastolic blood pressure less than 95 mmHg appears to be suboptimal. The risk for cardiovascular and cerebrovascular events increase with a blood pressure over 140/90 mmHg. (Choice C & D) Yearly CT scan and ultrasonography of the kidneys are not recommended. However, if a patient presents with acute abdominal pain, imaging is performed to differentiate acute urinary obstruction (by clot or stone) from hemorrhage into a cyst. (Choice E) Approximately 10% of patients with ADPKD have intracranial berry aneurysms. Routine screening of these berry aneurysms is restricted to patients with a positive family history of subarachnoid hemorrhage. Good blood pressure control is also helpful in preventing rupture of the berry aneurysm and subsequent intracranial bleeding. Educational Objective: Most patients with ADPKD have a progressive decline in renal function. The ideal goal for blood pressure control in any patient with chronic kidney disease, including ADPKD, is less than 130/80mm Hg. *Extremely important question for USMLE step-3 37% of people answered this question correctly. Item 2 of 3 What is the most common extrarenal manifestation of ADPKD? A. Splenic cysts B. Colonic diverticula

C. Hepatic cysts D. Hypertension E. Cyst formation in brain Explanation: The most common extrarenal manifestation of ADPKD is hepatic cysts. Their incidence increases in the second through fifth decades of life; by 50 years of age, approximately 70-80% of patients with ADPKD will have hepatic cysts. Females occasionally develop massive enlargement of these cysts. (Choices A and C) Splenic cysts can be found in patients with ADPKD. Other locations where cysts could be present are the pancreas and ovaries. However, these cysts are less common extrarenal manifestations of ADPKD. (Choice B) Patients with ADPKD also have an increased incidence of diverticular disease; however, these usually develop in patients who are undergoing dialysis secondary to end stage renal disease (ESRD). Patients with ADPKD have a higher risk of rupture of the colonic diverticula, when compared to other patients with colonic diverticula. (Choice D) Hypertension is not an extrarenal manifestation of ADPKD. Hypertension is caused by compression of the intrarenal vasculature, thereby leading to renal ischemia and subsequent activation of the renin-angiotensin system. (Choice E) Although berry aneurysms have an increased frequency (10-15%) in patients with ADPKD, cyst formation in the brain is rare. Educational Objective: The most common extrarenal manifestation of ADPKD is hepatic cysts. A higher incidence of colonic diverticula is seen in ESRD patients who are on dialysis. Hypertension is not an extrarenal manifestation of ADPKD. Although berry aneurysms are common (present in approximately10-15% of patients), cyst formation in the brain is rare. *Extremely important question for USMLE step-3; 26% of people answered this question correctly. Item 3 of 3 After learning about the patient's disease, the family members become concerned and request that they be screened for possible kidney disease. Which of the following is the most appropriate next step? A. Regular blood pressure check B. Ultrasonogram of the abdomen C. CT scan of the abdomen D. Yearly measurement of BUN and serum creatinine E. Genetic studies for PKD1 and PKD2 Explanation: Ultrasonography is the most cost-effective screening procedure for asymptomatic family members of a patient with ADPKD. The presence of at least three to five cysts in each kidney is required to make a diagnosis of ADPKD. (Choice C) Although computed tomography (CT) scan is more sensitive than ultrasonography in the detection of small cysts, it is not used for screening purposes due to its high cost.

(Choice E) Genetic linkage analysis is available for the diagnosis of ADPKD; however, it is reserved for cases wherein the ultrasound is negative and the need for definitive diagnosis is critical (e.g., screening an individual who is a potential renal transplant donor). Educational Objective: Ultrasonography is the procedure of choice for screening the asymptomatic family members of a patient with ADPKD. *Extremely important question for USMLE step-3 49% of people answered this question correctly. A 79-year-old Caucasian female is brought to the emergency department (ED) by her daughter because of bloody stools. For the past three days, the patient had been occasionally passing blood with her stools, but this morning, she completely filled up the toilet bowl with bloody stools. She denies any associated symptoms such as nausea, vomiting or abdominal pain. Her temperature is 36.7C (98F), heart rate is 92/min, blood pressure is 106/62 mmHg, and respiratory rate is 16/min. She appears very pale. Her general physical examination, including her abdominal examination, is unremarkable. Rectal examination revealed bright red blood in the rectal vault; there is no tenderness. As you are walking out of the room, she has another episode of large amount of bleeding per rectum. You start the patient on generous intravenous fluids and send out blood samples for laboratory evaluation. Which of the following is the most appropriate next step in the management of this patient? A. Start the patient on intravenous infusion of vasopressin B. Start the patient on octreotide C. Refer the patient for urgent colectomy D. Obtain a radionuclide scan E. Obtain a stat barium enema Explanation: The patient is most likely suffering from lower GI bleeding, which is defined as gastrointestinal blood loss from a site distal to the ligament of Treitz. Although rare, upper GI bleeding in the form of a briskly bleeding peptic ulcer can also present as bright red blood per rectum. Regardless of the origin, the initial management of acute GI blood loss is hemodynamic stabilization. This includes rapid establishment of intravenous access and administration of generous amounts of intravenous fluids. The next step is to search for the cause of bleeding to institute specific therapy. An urgent colonoscopy is the initial procedure of choice due to its diagnostic and therapeutic advantages. If a colonoscopy is unavailable or if there is poor visualization due to active bleeding in the colon, radionuclide imaging with technetium-99 labeled autologous red blood cells is the next procedure of choice for localizing the bleeding site. Radionuclide scan requires that the rate of active bleeding be 0.1 to 0.5 mL/min. It is more sensitive than angiography, but only localizes the bleeding to an area of the abdomen. (Choice A) Intraarterial infusion of vasopressin is mainly used to control the bleeding by intraarterial administration during an angiography, although it has also been used to control active variceal hemorrhage. Vasopressin causes constriction of the mesenteric blood vessels and reduces the blood flow to the bleeding site. Adverse effects are arrhythmias and ischemic damage to the organs, and there is a high incidence of recurrence of bleeding once the infusion is discontinued. (Choice B) Octreotide is a long-acting somatostatin analog that is used to control bleeding esophageal varices. (Choice C) Urgent colectomy is sometimes indicated in patients with life-threatening lower GI bleeding; however, all efforts should be made to localize the bleeding site prior to surgical intervention. Surgical treatment without prior localization of the bleeding site has been associated

with higher mortality rates, when compared to that of patients who undergo surgical treatment after the identification/localization of the bleeding site. (Choice E) Barium enema has no role in the localization of lower GI bleeding. It is an insensitive study for visualization of superficial mucosal lesions and makes subsequent colonoscopy difficult to perform. Educational Objective: Radionuclide scan with technetium-99 labeled red blood cells is a very sensitive study to localize active lower GI bleeding. 47% of people answered this question correctly.

A 36-year-old African American man with a medical history significant for hypertension and hypercholesterolemia presents to his primary care physician for an annual exam. He has no complaints at this time. His physical exam is significant for obesity (BMI=31.4). His current medications include verapamil and simvastatin. He mentions that his father recently died of prostate cancer at age 60. Because of this family history, the patient is now concerned about his own risk for developing prostate cancer. At what age would you commence prostate cancer screening in this patient? A. 40 B. 45 C. 50 D. 55 E. 60 Explanation: Men considered to be at high risk for developing prostate cancer are (1)African-Americans, and (2) those with two or more first-degree relatives with prostate cancer. Although this patient only has one first-degree relative with prostate cancer, his ethnic background places him at high risk for developing the disease. According to the American Cancer Society?s guidelines released in 2001, it is recommended that digital rectal exams and PSA testing be performed annually, beginning at age 45 in those men at high risk for developing prostate cancer, and at age 50 for those men not at high risk and with a life expectancy of at least 10 years (Choice C). (Choices A, D, and E) The American Cancer Society (ACS) guidelines recommend that men at high risk for developing prostate cancer be screened starting at age 45, not ages 40, 55, or 60. Educational Objective: Screening for prostate cancer should be started at age 45 in high-risk males (African-Americans and those with two or more first-degree relatives with the disease) and at age 50 in all other men. 18% of people answered this question correctly. The following vignette applies to the next 2 items A 60-year-old Caucasian man is admitted to the hospital with an episode of bacterial pneumonia. Upon review of his past medical history, you notice that he has had three recent admissions within the past seven months for bacterial pneumonia. All the episodes were successfully treated with intravenous antibiotics. Previous chest x-ray comparisons reveal that the infiltrate always occurs at the right middle lobe and leaves a persistent scar. The scar was not noted on the chest x-ray prior to the episodes of pneumonia. He has a 50-pack year history of smoking. He quit smoking 10 years ago. Item 1 of 2 Which of the following is the most likely cause of the patient's recurrent pneumonia? A. Tuberculosis B. Recurrent alveolar hemorrhage C. Endobronchial obstruction D. Congenital immunodeficiency disorder E. Recurrent pulmonary embolism Explanation: The patient in the above vignette has multiple/recurrent episodes of bacterial pneumonia, which respond to antibiotics. This is most likely due to a partial obstruction of the bronchus or a branch of the bronchus supplying the right middle lobe. The most likely cause of endobronchial obstruction in this elderly male with an extensive smoking history is bronchogenic carcinoma. A carcinoid tumor is

another cause of endobronchial obstruction, especially in younger and non-smoking patients. Endobronchial lesions cause compression of the airway lumen and prevent adequate clearance of secretions. This causes stasis of secretions and recurrent pneumonias, even after successful treatment of previous episodes. Other causes of nonresolving pneumonia or pulmonary infiltrates are bronchoalveolar cell carcinoma, lymphoma, eosinophilic pneumonia, bronchiolitis obliterans organizing pneumonia (BOOP), systemic vasculitis, pulmonary alveolar proteinosis, and drugs (amiodarone). (Choice A) Patients with tuberculosis usually do not respond to antibiotic therapy, unless there is a superimposed bacterial infection. They also present with symptoms of chronic tubercular infections, such as a chronic cough, weight loss and night sweats. (Choice B) Patients with recurrent alveolar hemorrhage usually present with recurrent episodes of hemoptysis. These episodes do not respond to antibiotic therapy. (Choice D) Acquired immunodeficiency (AIDS) is a possibility. immunodeficiency to present at this age. It is unusual for a congenital

(Choice E) Recurrent pulmonary embolism can rarely present as nonresolving pneumonia. Patients classically have symptoms of pleuritic chest pain with hypoxia and do not respond to antibiotics. Educational Objective: Recurrent pneumonia in an elderly smoker may be the first manifestation of bronchogenic carcinoma. 86% of people answered this question correctly. Item 2 of 2 Which of the following is the most useful test to confirm the diagnosis in this patient? A. Sputum for cytology and AFB staining B. Bronchoalveolar lavage for hemosiderin laden macrophages C. Flexible bronchoscopy D. Immunoglobulin levels E. High resolution CT scan Explanation: This patient would benefit the most from a flexible bronchoscopy for the confirmation of the diagnosis. Flexible bronchoscopy is a simple, invasive test that will definitely diagnose the cause of this patient's recurrent pneumonia, since it will enable the physician to visualize the actual endobronchial lesion, and take a tissue biopsy at the same time. It is relatively easy to perform, and has a low risk of complications, thus making it the procedure of choice for obtaining tissue samples and lavage fluid to confirm the diagnosis. It also has a better yield for diagnosing certain other infections, such as fungal infections or tuberculosis. (Choice A) Obtaining sputum samples for AFB staining and cytology has a low yield, and is not very helpful in patients suspected of having endobronchial obstruction. (Choice B) Hemosiderin laden macrophages are seen in diffuse alveolar hemorrhage syndromes (Wegner?s granulomatosis, Goodpasture?s syndrome and other systemic vasculitis). (Choice D) Checking for immunoglobulin deficiencies would not be helpful in this patient. (Choice E) High resolution CT scan (HRCT) can diagnose recurrent pulmonary embolism, and can also help in characterizing the persistent "scar" seen on chest x-ray. It will not provide a definite cause of the lesions and recurrent pneumonia.

Educational Objective: Flexible bronchoscopy is a primary diagnostic tool to evaluate patients with persistent or nonresolving pneumonia or pulmonary infiltrates (HRCT scan is an alternative, but it does not provide confirmatory information). *The best diagnostic test for endobronchial obstructive lesions is flexible bronchoscopy. If the question had asked for the next best step in management, then the answer would have been CT scan. 63% of people answered this question correctly.

Case reports indicate that the exposure to manganese in miners may lead to central nervous system damage, particularly Parkinson-like syndrome. A group of investigators wanted to investigate this problem further to define the potential association between the exposure and increased mortality in miners. They calculated standardized mortality ratio (SMR), which is equal to 1.75. Which of the following is the best statement concerning the result of the study? A. SMR is an unadjusted measure of the overall mortality. B. SMR is not generally applicable to occupational studies. C. The observed number of death is higher than expected. D. The crude rate of death is high. E. The effect of exposure seems to vary by age. Explanation: The standardized mortality ratio (SMR) represents an adjusted measure of the overall mortality (Choice A), and is typically used in occupational epidemiology (Choice B). Mortality is typically adjusted for age (less commonly for gender, race and other factors). The standard population is used for comparison. The SMR is calculated using the following formula: SMR = observed number of deaths/expected number of deaths The expected number of deaths is calculated using age-specific rates of death in the standard population (e.g. total US population). The observed number of deaths in the population of interest (e.g. miners) is then divided by the expected number to obtain the SMR. An SMR of 1.75 indicates that the observed number of death in miners is 1.75 higher than in the standard population (e.g. total US population). (Choice D) The crude (unadjusted) rate of death in miners may be higher than the crude rate of the standard population; however, it may also be lower (if the proportion of younger people is high in the population of interest). (Choice E) SMR is a measure of overall mortality adjusted for age (or other factor); age-specific mortality rates are not mentioned in the scenario. Educational Objective: SMR represents an adjusted measure of overall mortality, and is calculated by dividing the observed number of deaths in the population of interest (e.g. miners) by the expected number derived from the reference population (?standard?). 39% of people answered this question correctly. A 59-year-old African-American man comes to the office and complains of a vague upper abdominal discomfort. The pain has been present most of the time for the past three weeks, but has never been severe enough to make him seek immediate medical attention. The discomfort is not associated with eating or performing any activities. He denies any other medical problems. His vital signs are unremarkable. Abdominal examination reveals the presence of multiple scars from previous surgical procedures around the epigastric area. A 3 x 5 cm oblong-shaped mass is palpable in the epigastric area. There is no tenderness on direct palpation of the mass and the surrounding epigastric area. The patient tells you that he previously had a similar mass in the same area "but it was taken out". Which of the following is the most likely explanation for the mass? A. Primary hyperparathyroidism B. Ventral hernia C. Muscle growth

D. Abdominal wall neuroma E. The effect of exposure seems to vary by age.

Explanation: The patient's clinical presentation is consistent with a diagnosis of desmoid tumor. Desmoid tumors are locally aggressive neoplasms arising from fibroplastic elements within the muscle or fascial planes. Since these are locally invasive and slowly infiltrate the surrounding tissues and structures, these only cause local complications. These usually present as painless or minimally painful, slowgrowing masses over the extremities, shoulder girdle, and hip-buttock area, although these may occur at all body sites. The diagnosis is generally confirmed with a tissue biopsy. Surgical excision with a wide margin of resection is the treatment of choice for patients with an easily approachable and resectable mass. There is a high rate of local recurrence, even after adequate resection. (Choice A) Abdominal aortic aneurysms usually present as a painless pulsatile mass. does not have any risk factors for the development of aortic aneurysm. This patient

(Choice B) Ventral or incisional hernia can develop at the site of previous surgical scars; however, this is generally reducible, and pain is an unusual symptom of ventral hernia, unless it is strangulated. Educational Objective: Desmoid tumors are slow growing and locally aggressive benign neoplasms with a high rate of local recurrence, even after surgical excision. 6% of people answered this question correctly. You receive a call from the surgical ICU to evaluate a 20-year-old male professional football player who was admitted three weeks ago following a motor vehicle accident. The patient sustained a C5-C6 translocation, which resulted in quadriplegia. Presently, he has a tracheostomy tube and is on a ventilator. He is being fed by a PEG tube. Investigations reveal a serum calcium level of 12.8 mg/dL (albumin 2.5 g/dL), a PTH level of 10 pg/mL (normal 10-65 pg/mL), a 25-hydroxyvitamin D level of 20 mcg/L (normal 18-68 mcg/L), and a 12,5-dihydroxyvitamin D level of 15 ng/L (normal 15-60 ng/L). A repeat serum calcium level is 13.1 mg/dL, along with a phosphorus level of 3.2 mg/dL, and ionized calcium level of 6.6 mg/dL (normal 4-5.6 mg/dL). What is the most likely cause of this patient?s hypercalcemia? A. Primary hyperparathyroidism B. Vitamin D toxicity C. Paraproteinemia D. Lab error E. Immobilization Explanation: This patient has a characteristic presentation of hypercalcemia caused by immobilization. This is usually seen in patients with a very high bone turnover (e.g., adolescent patients and older patients with Paget?s disease). When these patients are immobilized, bone resorption is increased and bone formation is decreased (uncoupling of bone turnover). The mechanism of this uncoupling is not clearly known. Excessive bone resorption after immobilization leads to hypercalcemia. It happens in days to weeks following immobilization. High serum calcium levels suppress PTH secretion, as seen in this patient. Vitamin D (25-OH D and 1,25-OH2 D) levels are usually normal, although 1,25dihydroxyvitamin D levels may be lower due to the suppressed PTH secretion (PTH stimulates 1-alphahydroxylase activity in the kidneys, an enzyme responsible for formation of 1,25-dihydroxy vitamin D). Management of hypercalcemia due to immobilization is similar to other hypercalcemic states. Bisphosphonates are particularly useful in decreasing the bone resorption. Their use may preserve the bone mass in patients who are immobilized for very long periods. In this patient, the level of albumin-corrected total calcium is 14.3 mg/dL (For every decrease of 1 g/dL serum albumin level below 4 g/dL, add 0.8 mg to the total calcium level. For instance, if the calcium level is 9.0 mg/dl, and the corresponding albumin level is 3 g/dL then the corrected calcium level is 9.8 mg/dL).

(Choices A and B) PTH levels are suppressed, and vitamin D levels (both 25-OH D and 1,25-OH2 D) are within normal limits; therefore, this patient is unlikely to have primary hyperparathyroidism or vitamin D toxicity. (Choices C and D) The patient has an increase in both total and ionized calcium; therefore, the chances of a lab error or paraproteinemia are unlikely. High protein levels in patients with paraproteinemia can avidly bind to calcium, leading to spuriously elevated total calcium levels. Patients with multiple myeloma can have true hypercalcemia. Educational Objective: Excessive bone resorption after immobilization leads to hypercalcemia. This is usually seen in patients with a very high bone turnover (e.g., adolescent patients and older patients with Paget?s disease). 76% of people answered this question correctly. The following vignette applies to the next 2 items A 62-year-old Caucasian female comes to the emergency department with a complaint of severe upper abdominal pain. She has had episodes of nausea and upper abdominal pain over the past three months, but the pain was tolerable and only lasted for 1 to 2 hours. Last night, she had a similar episode one hour after dinner, but the pain progressed in severity, and was accompanited by three episodes of vomiting. The pain is now maximally located in the right upper quadrant and radiates to her back. She denies any diarrhea or constipation. She is a moderately obese female who is in marked distress due to the pain. Her temperature is 38.3C (101F), blood pressure is 118/87 mmHg, heart rate is 112/min and respirations are 20/min. Her lung and cardiovascular examination is unremarkable. Her abdominal examination reveals the presence of hypoactive bowel sounds. There is marked tenderness on deep palpation of the right upper quadrant. She has voluntary and involuntary abdominal guarding. Murphy?s sign is positive. The initial blood work reveals the presence of marked leukocytosis with a left shift. The results of the liver function tests are pending. Item 1 of 2 What is the next best step in the management of this patient? A. Plain x-ray of the abdomen B. Order a right upper quadrant ultrasound C. Obtain a CT scan of the abdomen with IV contrast D. Wait for the liver function tests E. Consult GI for an endoscopic retrograde cholangiopancreatography Explanation: The clinical presentation of this patient is consistent with acute cholecystitis. Acute cholecystitis is defined as gallbladder inflammation characterized by steady right upper quadrant pain, fever and leukocytosis. It is generally caused by gallstone disease; however, acalculus cholecystitis can also occur especially in elderly and critically ill patients. Patients with acute cholecystitis typically complain of a steady and severe upper abdominal or right upper quadrant pain. The pain may radiate to the back or right shoulder, and is generally associated with fever, nausea, vomiting and anorexia. The patient may give a history of intolerance to fatty foods or history of prior episodes of intermittent pain with fatty food ingestion. The physical examination usually reveals an ill-looking patient with significant right upper quadrant tenderness, voluntary and involuntary abdominal guarding and a positive Murphy?s sign (discomfort and inspiratory arrest on palpation of the gallbladder fossa during deep inspiration).

Although patients have a typical presentation, no clinical or laboratory findings is sufficiently accurate to confirm or refute the diagnosis of acute cholecystitis. Confirmation of the diagnosis is generally made using the physical findings, laboratory studies and radiographic imaging studies. Patients with a typical clinical presentation should undergo imaging studies to confirm the diagnosis. Ultrasonography is the initial best test to establish the diagnosis. The ultrasonographic findings which support the diagnosis include the presence of stones in the gall bladder or cystic duct, gallbladder wall thickening, pericholecystic fluid collection, and positive sonographic Murphy?s sign. (Choices A) Plain abdominal x-rays have no role in the diagnosis of acute cholecystitis. (Choice C) Abdominal CT scan is usually not necessary for the diagnosis of acute cholecystitis. It can easily distinguish the gallbladder wall edema or thickening as the cause of upper abdominal pain; however, right upper quadrant ultrasound remains as the initial test of choice for the diagnosis of acute cholecystitis. (Choice D) Liver function tests are usually normal or slightly elevated in patients with acute cholecystitis; therefore, these will not help to rule in or rule out the diagnosis. (Choice E) ERCP should be done in patients with suspected common bile duct stones. It has no role in the management of patients with acute cholecystitis. Educational Objective: Ultrasonography is the initial test of choice to establish the diagnosis of acute cholecystitis. HIDA scan is useful in excluding cystic duct obstruction in patients with clinical features suggestive of acute cholecystitis. 85% of people answered this question correctly. Item 2 of 2 The appropriate test is ordered and the results are pending at this point. In the meantime, what is the most appropriate next step in her management? A. Emergent exploratory laparotomy B. Emergent T tube placement and biliary tree decompression C. Begin intravenous antibiotics D. Emergent laparoscopic cholecystectomy E. Perform an endoscopic retrograde cholangiopancreatography Explanation: In patients with uncomplicated acute cholecystitis, the symptoms generally subside in 7 to 10 days; however, if left untreated, there is a high risk of complications. The most common complication in patients with acute cholecystitis include gall bladder gangrene and perforation, development of cholecystoenteric fistula and emphysematous cholecystitis. All patients with acute cholecystitis should therefore be admitted to the hospital and managed with supportive care in the first 24 - 48 hours. This includes adequate analgesia, NPO (nil per os; "nothing by mouth"), placement of nasogastric tube if necessary, appropriate intravenous hydration, and correction of any associated electrolyte abnormalities. All patients with acute cholecystitis should be treated with intravenous antibiotics. Empiric antibiotic therapy is usually directed against the most commonly recovered organisms (Escherichia coli, enterococcus, Klebsiella, and Enterobacter). A commonly used empiric antibiotic regimen is intravenous ampicillin + gentamicin (for coverage against gram-negative bacilli and synergistic action with ampicillin against enterococcus). Since anaerobic organisms are rarely recovered from patients with acute cholecystitis, their empiric treatment is not necessary. (Choices A and D) All low-risk patients should receive 24 - 48 hours of supportive therapy prior to cholecystectomy. Emergent laparotomy or laparoscopic cholecystectomy is only considered for

patients with gallbladder gangrene or perforation, and for patients who fail to respond or have progressive symptoms while on supportive therapy. (Choice B) T tube placement (percutaneous cholecystostomy) is generally considered only for high risk surgical patients (advanced pulmonary or cardiac disease) who continue to have severe symptoms and fail to improve with conservative therapy. (Choice E) ERCP has no role in the management of patients with acute cholecystitis. Educational Objective: All hospitalized patients with acute cholecystitis should be treated empirically with intravenous antibiotics to reduce the risk of secondary infection. 81% of people answered this question correctly. Several tests have been developed to measure the serologic markers of breast cancer. These tests have different specificities and sensitivities for the early stage of breast cancer. Which of the following tests could be the best screening test for the early detection of breast carcinoma if applied to a population with a stable incidence of the disease? A. Sensitivity ? 80%, specificity ? 90% B. Sensitivity ? 65%, specificity ? 97% C. Sensitivity ? 70%, specificity ? 94% D. Sensitivity ? 75%, specificity ? 92% E. Sensitivity ? 85%, specificity ? 90% Explanation: A screening test must have a high sensitivity. This high sensitivity helps to ?RULE IN? the disease, giving as few false-negative results as possible. Furthermore, the high sensitivity increases the negative predictive value (NPV) of the test: NPV = True negatives / (True negatives + False Negatives) In this case, the test with the highest sensitivity and good specificity is the best choice. (Choice B) Choosing a highly specific test with low sensitivity would give more false-negative results; thus, many diseased people would be labeled healthy. Educational Objective: A screening test must have a high sensitivity. This high sensitivity helps to ?RULE IN? the disease by decreasing the number of false-negative results, and by increasing the negative predictive value. 81% of people answered this question correctly. The following vignette applies to the next 2 items A 27-year-old male comes to the office with a scrotal mass. He first noticed this mass three months ago while taking a bath. At first, he ignored it, but he became concerned as the mass increased in size over the past few weeks. He describes this mass as "many worms in a bag." His scrotum feels heavy, and is sometimes itchy. His vital signs are within normal limits. The physical examination is normal, except for left scrotal enlargement. The mass is warm, nontender, non-fluctuant, and it does feel like a bag of worms. The left testicle is difficult to palpate. He is diagnosed with varicocele and advised to undergo surgical removal. The patient is anxious and afraid of surgery. Item 1 of 2

Which of the following is the most likely complication of untreated varicocele? A. Surgical removal of varicocele B. Impotence C. Infection D. Development of hydrocele E. Sensitivity ? 85%, specificity ? 90% Explanation: The testes are located outside the abdomen, and venous drainage involves a complex network of venous channels called the pampiniform plexus. This plexus is responsible for maintaining the appropriate temperature for the scrotal sacs (i.e., approximately 2 degrees lower than body temperature). Seminiferous tubules make up most of the testicular mass, and are very sensitive to an increase in temperature. Patients with varicocele have dilatation of the pampiniform plexus, which leads to a rise in intrascrotal temperature, thereby causing the seminiferous tubules to atrophy. (Choice B) Androgen-secreting Leydig cells are more resistant to an increase in temperature caused by varicoceles; therefore, hypoandrogenism does not commonly occur with this disease. (Choice C) Intrascrotal infection is unlikely in patients with varicocele. (Choice D) Hydrocele is a collection of fluid within the potential space in the tunica vaginalis, and can be congenital (patent processus vaginalis) or acquired. Acquired causes include: epididymitis, orchitis, trauma, testicular torsion and tumor. There is no increased incidence of varicocele in patients with hydrocele. Educational Objective: Venous drainage of the testes involves a complex network of veins called the pampiniform plexus, which are responsible for keeping the temperature of the scrotal sacs below the normal body temperature. Dilatation of the pampiniform plexus (varicocele) results in an increased temperature of the scrotal sacs, thereby causing testicular atrophy. 39% of people answered this question correctly. Item 2 of 2 Six months later, the patient comes back to your office with a right scrotal mass. The physical findings suggest a diagnosis of right varicocele. Which of the following is the most appropriate intervention? A. Doppler examination of bilateral lower extremities B. Embolization of right testicle vein C. CT scan of the abdomen D. Scrotal ultrasonography E. Sensitivity ? 85%, specificity ? 90% Explanation: The testes embryologically develop intra-abdominally. Just before birth, the testes descend down to the scrotal sacs. This explains why the testicular arterial supply and venous drainage come from the abdomen. Varicoceles are more common on the left side because the left spermatic vein enters the left renal vein at a 90-degree angle. The right spermatic vein drains at a more obtuse angle directly into the inferior vena cava, thereby facilitating a more continuous flow. Processes that cause inferior vena caval obstruction (e.g., clot, tumor) should be ruled out in patients who have bilateral varicocele, right

varicocele or varicocele that does not disappear in the supine position. This patient has bilateral varicocele; therefore abdominal computed tomography (CT) is warranted to look for causes of inferior venacaval obstruction. (Choices A and B) Surgical removal of the varicocele or embolization is not the most appropriate intervention at this time. (Choice D) Scrotal ultrasonography might show a dilated venous plexus; however, it is not very likely to show a cause of varicocele. Occasionally, testicular masses can cause varicocele by metastasis into the abdominal lymph nodes. Educational Objective: Processes that cause inferior vena caval obstruction (e.g., clot, tumor) should be ruled out in patients who have bilateral varicocele, right varicocele or varicocele that does not disappear in the supine position. 40% of people answered this question correctly. The following vignette applies to the next 2 items A 25-year-old Hispanic college student comes to the emergency department with complaints of sudden onset of left foot pain for the past hour. The pain started suddenly when he was sitting at his desk at college and has progressed gradually. He denies any history of similar episodes in the past or recent trauma to his left leg. His temperature is 37.2?C(99F), blood pressure is 120/80 mmHg, heart rate is 86/min, and respiratory rate is 14/min. The lungs are clear on auscultation. On cardiovascular examination, the heart rhythm is regular, and a diastolic murmur is heard over the cardiac apex. The abdomen is soft, nontender and without any palpable masses. The neurological examination is within normal limits. Examination of the extremities reveal adequate, bilateral, femoral pulses. On the left side, the popliteal and dorsalis pedis pulses are absent. The left foot also appears pale and cooler than the right foot. The sensations on the left foot are intact and he is able to make voluntary movements. There is no tenderness on palpation of the lower left leg or foot. The vascular surgeons are immediately called, and the patient is taken to the operating room for urgent embolectomy, where an embolus obstructing the entire lumen of the left popliteal artery is removed. Item 1 of 2 Which of the following is the most appropriate next step in the management of this patient? A. Doppler examination of bilateral lower extremities B. Angiographic examination of the left lower extremity C. Echocardiogram D. Histologic examination of the embolus E. Hypercoagulation workup Explanation: The patient has a classic presentation of acute arterial occlusion. Acute ischemia of the limb due to acute arterial occlusion usually presents with five Ps; pain, pallor, pulselessness, paresthesias and paralysis. Almost all the patients present with a gradually progressive pain, usually in the distal extremity. The skin of the lower extremities is cool and pale, and the pulses are greatly reduced to absent. Acute arterial occlusion is usually the result of (1) an embolus from a distal source, (2) acute thrombosis due to a previously diseased vessel, or (3) direct trauma to the involved artery. The onset of symptoms can help in differentiating the etiology of arterial occlusion. The sudden onset of symptoms in a previously asymptomatic patient is most likely due to an embolus, while a history of gradually progressive symptoms in a previously symptomatic patient is consistent with thrombosis. The sudden onset of dramatic symptoms in this patient is most consistent with an embolic source of acute arterial occlusion. Most of the emboli are from a cardiac source, with a few coming from the

arterial aneurysms or atherosclerotic plaques. Some of the well-recognized causes of cardiac emboli include atrial fibrillation, a prior history of myocardial infarction and ventricular dysfunction, endocarditis, valvular disease, atrial myxoma, and the presence of a prosthetic aortic valve. It is important to find the exact cause of cardiac emboli to prevent future recurrences. After an embolectomy, the surgical specimen should be sent for histopathologic examination to ascertain the exact source of the emboli. (Choices A and B) Doppler or angiographic examination of the lower extremities is used for the diagnosis of acute arterial occlusion. These are not helpful in locating the source of the emboli. (Choice D) An echocardiogram is an important part of the diagnostic workup since this may identify a source of emboli; however, approximately 20% of the emboli can arise from a peripheral arterial source. Histologic examination is therefore more important since this can differentiate a cardiac source from a peripheral one. (Choice E) Hypercoagulation workup is important to look for inherited thrombophilias causing venous thrombosis. Since this patient appears to have acute arterial occlusion from a cardiac embolic source, the hypercoagulation workup is not indicated at this point. Educational Objective: Histopathologic examination of the embolectomy specimen is extremely useful in locating the origin of the embolus which caused the acute arterial occlusion. 5% of people answered this question correctly.

Item 2 of 2 Which of the following is the most likely cause of the patient?s condition? A. Prolonged immobilization B. Left atrial myxoma C. Atherosclerosis of the abdominal aorta D. Thromboangiitis obliterans E. Left ventricular thrombus Explanation: Based on the clinical presentation, the most likely cause of acute embolic arterial occlusion in this 25year-old previously asymptomatic male is left atrial myxoma. Atrial myxomas are the most common primary cardiac tumors. Most of these arise from the left atrium, and the remaining arise from the right atrium and left ventricle. The tumors are typically pedunculated, with a stalk arising from the atrial septum. These can be extremely friable, resulting in embolization of the part of the myxoma to the systemic circulation. Some large tumors may initially present with signs and symptoms of mitral valve obstruction (diastolic murmur or "tumor plop"), rapidly worsening heart failure in otherwise young healthy individuals, or new onset atrial fibrillation. The diagnosis is usually made by echocardiography (either transthoracic or transesophageal echocardiography). Once the diagnosis of atrial myxoma is made, it should be excised as soon as possible to reduce the risk of recurrent embolization. (Choice A) Prolonged immobilization, along with other underlying risk factors, can cause deep venous thrombosis. It does not cause acute arterial occlusion. (Choices C & D) Thromboangiitis obliterans or atheroemboli is an unlikely cause of acute arterial occlusion in a young and otherwise previously healthy patient. (Choice E) A left ventricular thrombus is usually seen in patients with a prior history of myocardial infarction or severely reduced LV function. There is no such history in this patient. Educational Objective: Left atrial myxomas can present with signs and symptoms of mitral valve obstruction (diastolic murmur or "tumor plop"), rapidly worsening heart failure in otherwise young healthy individuals, or new onset atrial fibrillation. These can also present with systemic embolization, thereby causing acute arterial occlusion in otherwise healthy patients. 54% of people answered this question correctly. The following vignette applies to the next 2 items A healthy 24-year-old Japanese woman comes to the physician because of a lesion on her left cheek. Two years ago, she already noticed a small mole on her left cheek, but this lesion has recently grown in size. She does not have any other complaints. Her social history is not significant. Vital signs are within normal limits. Examination shows a 4 cm dark brown, slightly elevated macule with slightly irregular borders on the left cheek. Item 1 of 2 Which of the following is the most likely diagnosis? A. Keratoacanthoma B. Pigmented basal cell carcinoma C. Seborrheic keratosis D. Malignant melanoma E. Verruca vulgaris

Explanation: Malignant melanoma is a concern if a nevus increases in size and develops irregular borders. It can be confused with an atypical melanocytic nevus; however, this disease is infrequent in the Asian population. These lesions can appear spontaneously or develop over a previously atypical nevus. (Choice C) Seborrheic keratosis is rare before the age of 30. It also has a "stuck on" appearance, with a "warty" form. (Choice A) Keratoacanthoma is common on the cheek. It is a low-grade malignancy that pathologically resembles squamous cell carcinoma. It appears as a solitary, firm, round, skin-colored or reddish plaque that develops into a nodule with a central keratin plug. (Choice E) Verruca vulgaris has the same warty appearance as seborrheic keratosis, and is a welldemarcated lesion. (Choice B) Basal cell carcinoma is usually pink or red in color. It is more frequent in males, and it rarely appears before the age of 40. Educational Objective: It is important to recognize the presence of a melanoma, because excision can be curative if the lesion is identified early. Melanoma lesions are usually asymmetric, with color variegation, and usually more than 5 mm in diameter. 69% of people answered this question correctly. Item 2 of 2 Which of the following is the most appropriate statement in this patient?s management? A. A biopsy is not required since the clinical presentation is so classic. B. An excisional biopsy is the next best step. C. Administer intralesional corticosteroids. D. Observation and repeat follow-up in six weeks. E. Start therapy with interferon alfa-2b. Explanation: The sensitivity of a melanoma diagnosis by a dermatologist based on the history and physical examination is approximately 85%; however, false negatives have been reported in 15% of the time. Due to the significant amount of false negatives reported, an excisional biopsy is essential for confirmation of the diagnosis and staging of the lesion. Complete excision is the treatment of choice. (Choice E) Interferon alfa-2b is used as an adjuvant therapy in patients with a high risk of developing metastatic disease. Educational Objective: An excisional biopsy is the next best step in patients with a suspected malignant melanoma. 86% of people answered this question correctly.

A 24-year-old female with primary hypothyroidism for the last four years comes to you for advice regarding the use of oral contraceptive pills. Her hypothyroidism has been adequately controlled on 125 mcg per day of levothyroxine. Her most recent serum TSH and free T4 levels were well within normal limits. She has no other medical problems and takes no other medications. What would be the most appropriate response to this patient? A. Oral contraceptive pills are contraindicated in hypothyroidism because of the increased risk of side effects. B. The levothyroxine dose might be increased. C. The levothyroxine dose might be decreased. D. The dose of estrogen in the combined oral contraceptive pills should be higher. E. Progesterone-only pills should be prescribed. Explanation: Combination oral contraceptive pills (OCPs) are the most reliable form of reversible contraception; however, the use of these hormonal drugs may affect a person's TBG, total T4, and free T4 levels. The estrogen component of the combination pill increases Thyroxine Binding Globulin (TBG) levels, which may result in an increase in the total T4 levels and decrease in free T4 levels. A person with an intact and normal thyroid gland may compensate for these changes by increasing thyroid hormone production, which normalizes the free T4 levels. On the other hand, a hypothyroid patient cannot compensate for these possible physiologic changes since she is unable to produce sufficient amounts of thyroid hormone and is dependent on levothyroxine therapy. The dosage of levothyroxine may therefore be increased in hypothyroid patients who are taking OCPs to compensate for the mentioned physiologic responses. After starting combination OCPs in well-controlled patients with hypothyroidism, TSH levels should be checked in 12 weeks, and the dose of levothyroxine should be adjusted accordingly. (Choice A) There is no increased risk of side effects with the use of combined oral contraceptive pills in patients with primary hypothyroidism; therefore, these are not contraindicated in such patients. (Choice C) Levothyroxine dosages are either unchanged or increased with concurrent use of combined oral contraceptive pills. (Choice D) The dose of estrogen in the combined OCP has substantially decreased in the recent years. Combination OCPs with ethinyl estradiol less than 50 mcg per tablet are called low-estrogen pills. It is not necessary to use a higher dose of estrogens in the combined contraceptive pills. (Choice E) The use of progesterone-only pills (also called as Mini-Pills) is associated with a high risk of breakthrough bleeding. These pills are mainly used in lactating women. Educational Objective: TSH levels should be checked 12 weeks after patients with primary hypothyroidism are started on combined oral contraceptive pills, and the dosage of levothyroxine should be adjusted accordingly. The dose of levothyroxine may need to be increased. 62% of people answered this question correctly. A healthy 42-year-old Caucasian woman comes to the clinic with fever, chills, loss of appetite, sore throat, hoarseness, dry cough, headaches, and nausea. She is a paramedic. Her temperature is 38.3 C (101 F), blood pressure is 120/70 mmHg, pulse is 120/min, and respirations are 20/min. Examination shows pharyngeal erythema and exudates that are forming membranes and tender cervical lymphadenopathy. Examination of the lung is normal. The patient is treated with penicillin G and admitted because of poor oral intake. You receive a call from the laboratory staff, who reports that the throat culture is positive for Corynebacterium diphtheriae. A decision is made to start the patient on diphtheria antitoxin to avoid further cardiac or neurologic complications. This treatment puts the patient in a greater risk for which of the following complications?

A. Neurotoxicity B. Erythema multiforme C. Anaphylaxis D. Hepatotoxicity E. Bleeding diathesis Explanation: Diphtheria is a serious condition that can be life threatening. Its prompt recognition in patients with pharyngitis, cervical adenopathy, and low-grade fever is extremely important. If there is a high suspicion, diphtheria antitoxin should be administered as soon as possible to avoid complications such as myocarditis, neuritis, or, rarely, nephritis. Diphtheria antitoxin is made with horse serum; thus, the risk of hypersensitivity or serum sickness is approximately 10%. There is also a lesser risk of anaphylaxis. For these reasons, epinephrine must be always available. (Choices A, B, D, and E) There are no reports of neurotoxicity, erythema multiforme, hepatotoxicity, or bleeding complications . Educational Objective: Diphtheria antitoxin is made with horse serum; thus, the risk of hypersensitivity or serum sickness is approximately 10%. There is also a lesser risk of anaphylaxis. 55% of people answered this question correctly. The following Vignette applies to the next 4 items A healthy 55-year-old Caucasian woman was brought to the emergency department because of sudden onset of shortness of breath, chest pain, and palpitations. She denies cough and hemoptysis. She was discharged from the hospital after a prolonged ICU course for sepsis one week ago. She stayed for one month in the hospital, and was subsequently transferred (after discharge) to a rehabilitation facility for physical therapy. She does not drink alcohol. She has smoked one-and-a-half packs of cigarettes daily for 15 years. Her father has lung cancer, and her mother has ovarian cancer. Her blood pressure is 110/70 mmHg, pulse is 112/min, and respirations are 28/min. Her pulse oximetry is 89% at room air. Physical examination shows no abnormalities except bilateral foot drop, which she developed during her previous hospitalization. Her chest x-ray is clear. EKG reveals sinus tachycardia. Item 1 of 4 Which of the following is the most appropriate next step in the management of this patient? A. Ventilation perfusion scan B. Duplex of the lower extremities C. Cardiac enzymes D. Thrombolytic therapy E. Initiate coumadin therapy Explanation: The most likely diagnosis in this case is pulmonary embolus (PE). Sudden onset of shortness of breath with clear lung sounds should always raise the suspicion for PE. Other suggestive clinical findings are tachycardia, tachypnea, and hypoxia. Furthermore, the history of recent hospitalization and prolonged bed rest puts the patient at high risk for PE. The best initial diagnostic test in the above setting is a V/Q scan. (Choice C) Coronary artery disease usually causes shortness of breath by causing pulmonary congestion. This patient?s chest x-ray is clear. Even though cardiac enzymes are needed to rule out myocardial infarction, this is not the next best step.

(Choice D) Thrombolytics are used in patients with hemodynamic instability or if there is worsening after IVC filter placement. It is also indicated in acute ST elevation myocardial infarction. (Choice E) PE is initially treated with heparin. Coumadin is used once good therapeutic anticoagulation is achieved with heparin. The reason for this chronologic order is that coumadin takes time to start acting, and its use has been associated with skin necrosis in some individuals if given without prior heparin infusion. Educational Objective: A V/Q scan is the most helpful investigation in the evaluation of PE. 75% of people answered this question correctly. Item 2 of 4 Further evaluation shows normal cardiac enzymes and intermediate probability ventilation perfusion scan. The patient does not want to be treated until the appropriate diagnosis is made. What is the best next step in the management of this patient? A. Duplex of the lower extremities B. Pulmonary angiography C. Explain and initiate warfarin therapy D. Echocardiography E. Coronary angiography Explanation: Pulmonary thromboembolism is difficult to diagnose, and usually requires the combination of several diagnostic modalities. It is therefore important to know the order in which to order these diagnostic tests. The first test that is usually employed is ventilation/perfusion scanning. If the results reveal the classic pattern of mismatched perfusion, it is advisable to proceed directly with treatment. If the scan is normal, significant pulmonary thromboembolism should be ruled out; however, ventilation/perfusion scanning results are inconclusive in a substantial number of patients. Since the usual source of emboli causing this condition is deep venous thrombosis (DVT) of the lower extremities, the next best step in this case is venous ultrasonography to reveal DVT or CT angiogram of the chest. A diagnosis of DVT makes the probability of pulmonary thromboembolism very high, and warrants immediate treatment. (Choice A) Although pulmonary angiography is the ?gold standard? for diagnosing pulmonary thromboembolism, this invasive procedure is only employed if venous ultrasonography or CT angiogram is negative. Chest CT angiogram is also frequently used in indeterminate cases of pulmonary embolism, but this ancillary procedure can only detect large emboli; small emboli are usually missed. Educational Objective: The next best step in a patient with intermediate probability V/Q scan is venous ultrasonography to reveal DVT or CT angiogram of the chest. 42% of people answered this question correctly. Item 3 of 4 Which of the following is considered the gold standard for the diagnosis of the above patient?s condition? A. Elevated D-dimer levels B. CT angiogram of the chest C. Ventilation perfusion scan D. Pulmonary angiography E. Coronary angiography

Explanation: Pulmonary angiogram is the gold standard test for the diagnosis of pulmonary embolism (PE); however, this procedure is rarely being performed since it is invasive and CT angiogram is readily available. Item 4 of 4 What is the best long-term treatment for this patient? A. Family predisposition B. Low molecular weight heparin C. Warfarin (Coumadin) D. Lorazepam, as needed E. Coronary angiography Explanation: Pulmonary embolism (PE) is generally treated by starting the patient on heparin first, and then adding warfarin (Coumadin) after 24-48 hours. The recommended duration of pharmacotherapy depends on the setting in which the PE occurred, as indicated below: Occurrence of PE in the setting of reversible risk factors (e.g., use of oral contraceptive pills, immobilization, or surgery) should be treated with 3-6 months of warfarin therapy. If the first episode of thromboembolism occurs in a setting that involves an underlying malignancy, anticardiolipin antibody, and antithrombin deficiency, the patient should be treated with at least 12 months of warfarin therapy. Patients with a first episode of idiopathic thromboembolism should be treated for at least six months with warfarin; three months of therapy is inadequate in this patient group. Patients with recurrent thromboembolism or a continuing risk factor should be treated indefinitely. In this case, the patient should continue pharmacotherapy for a minimum of three months. Educational Objective: Pulmonary embolism (PE) is generally treated by starting the patient on heparin first, and then adding warfarin (Coumadin) after 24-48 hours. The recommended duration of pharmacotherapy depends on the setting in which the PE occurred. 80% of people answered this question correctly. The following vignette applies to the next 2 items A 17-year-old Caucasian boy presents to the emergency department (ED) with a generalized rash. He seems scared, and says that he never had such a rash before. He has had fever and throat pain for the past week. He went to the doctor three days ago and got an antibiotic prescription. His past medical history is insignificant. He does not smoke or consume alcohol, and he denies any recreational drug use. He is sexually active with one partner and uses condoms for contraception. His temperature is 37.8C (100F), blood pressure is 110/76 mm Hg, pulse is 88/min, and respirations are 16/min. Physical examination reveals generalized maculopapular rash and posterior cervical lymphadenopathy. Item 1 of 2 Which of the following is the most likely mechanism of rash development in this patient? A. Family predisposition B. Viral exanthema C. Immediate hypersensitivity

D. Circulating immune complexes E. Delayed hypersensitivity Explanation: Ampicillin-associated maculopapular rash is a well-known phenomenon in patients with infectious mononucleosis. The reported incidence of this reaction is as high as 80%. It is believed that this vasculitic rash is immune-mediated, and is caused by circulating IgG and IgM antibodies toward penicillin derivatives. Such antibodies have actually been demonstrated in patients with EBVassociated infectious mononucleosis. (Choices C and E) The rash does not represent immediate or delayed hypersensitivity to ampicillin, which can be used safely when the infection subsides. (Choice B) The pathogenesis of ampicillin-associated rash seems to be different from viral exanthema which are commonly observed during viral infections. Educational Objective: It is believed that ampicillin-associated rash in patients with infectious mononucleosis is immunemediated, and is caused by circulating IgG and IgM antibodies toward penicillin derivatives. 30% of people answered this question correctly. Item 2 of 2 Which of the following is the best treatment for this patient? A. Switch to another antibiotic B. Discontinue the antibiotic and observe C. Discontinue the antibiotic and prescribe corticosteroids D. Discontinue the antibiotic and prescribe acyclovir E. Reassure and continue the antibiotic treatment Explanation: Supportive treatment and observation are the mainstays of treatment for individuals with infectious mononucleosis. Supportive treatment includes acetaminophen and NSAIDs for fever, throat pain and malaise, as well as adequate nutrition, fluids, and rest. (Choice E) The antibiotic should be discontinued. (Choice C) Corticosteroids are reserved for patients with severe complications such as impeding airway obstruction, liver failure, or aplastic anemia. (Choice D) Although acyclovir is effective in inhibiting EBV replication, it has not been shown to have significant clinical benefits in patients with infectious mononucleosis. (Choice A) There is no indication for antibiotic treatment in this patient. Educational Objective: Supportive treatment and observation are the mainstays of treatment for individuals with infectious mononucleosis. 67% of people answered this question correctly. A healthy, 27-year-old Guatemalan-American woman comes to the physician because of weight loss, nausea, abdominal pain, abdominal distention, and diarrhea. She has no other medical problems. The patient is a schoolteacher, and has lived in America for ten years. She does not use tobacco, alcohol, or drugs. She is a vegetarian. Her family history is not significant. She takes no medication. She has no known drug allergies. She has just received a visit from her relatives who came from

Guatemala four months ago, and who are now living in her home. Her vital signs are within normal limits. Examination shows that the abdomen is soft, mildly tender, and distended, with increased bowel sounds. There is no rebound tenderness or rigidity. There is no hepatomegaly or splenomegaly. The patient?s labs reveal: CBC Hb: 10 g/dL Ht: 38% Platelet count: 200,000/cmm Leukocyte count: 8,000/cmm Segmented neutrophils: 70% Bands: 3% Eosinophils: 10% Lymphocytes: 14% Monocytes: 3% Which of the following is the most appropriate pharmacotherapy? A. Trimethoprim-sulfamethoxazole B. Ciprofloxacin C. Metronidazole D. Albendazole E. No antibiotic needed Explanation: Intestinal parasitosis is not common in the United States, but it is highly prevalent in other regions of the world, such as Central America. Eosinophilia associated with diarrhea points to the presence of helminthiasis, which are infections caused by Nematodes such as Ascaris lumbricoides (roundworm), Fasciola hepatica (sheep liver fluke), Trichuris trichiura (whipworm), Necator americanus or Ancylostoma duodenale (hookworms). All of these conditions can be treated with albendazole. (Choice C) Metronidazole is used to manage parasitosis such as Giardia or amoeba infections; however, such infections are not associated with eosinophilia. (Choice A) Ciprofloxacin or TMP-SMX are used in Gram-negative infections or traveler?s diarrhea. (Choice E) Parasitosis must be treated because of the risk of transmission to other persons, and also because hookworms can cause anemia through indolent intestinal bleeding, if left untreated. Educational Objective: The presence of diarrhea and eosinophilia can be seen in certain conditions (such as intestinal parasitosis) due to Helminths, eosinophilic gastroenteritis, or Addison?s disease. Treatment of parasitic infections by roundworms, hookworms, whipworms, or pinworms can be achieved with albendazole or mebendazole. 40% of people answered this question correctly. The following vignette applies to the next 4 items A 24-year-old female is admitted to the hospital after a motor vehicle accident, where she sustained a right humerus fracture, multiple rib fractures and a contusion over her right lower leg. The next morning, a nurse calls to inform you that she is complaining of severe pain in her right lower leg. Her temperature is 37.2?C(99F), heart rate is 96/min, respiratory rate is 18/min, and blood pressure is 140/82 mmHg. Physical examination reveals a tense swelling around the right calf region. The pain is worsened on palpation and passive movements of the foot. Neurological examination reveals motor weakness and hypoesthesia of the distal right leg.

Item 1 of 4 Which of the following is the most likely diagnosis? A. Fat embolism B. Deep venous thrombosis C. Acute compartment syndrome D. Acute vascular occlusion E. Neural compression Explanation: Acute compartment syndrome refers to ischemic tissue damage secondary to elevated pressures in the enclosed compartments of the lower legs or forearm. When the tissue pressure in an enclosed compartment exceeds the perfusion pressure, the resulting diminished tissue perfusion and compromised blood flow to the muscles and nerves inevitably lead to ischemic tissue necrosis. Majority of the cases involving the lower extremities are due to a traumatic event, most commonly tibial fractures. Other causes include a crush injury or other long bone fractures in a motor vehicle accident, a tight cast or dressing after trauma, and drug overdose. Patients usually present with severe pain which is out of proportion to the extent of injury. The pain is typically worsened by passive movements of the involved muscles. Sensory nerves are usually affected earlier than the motor nerves, and the neurologic deficit presents as decreased vibration sense, decreased two-point discrimination, numbness or hypoesthesia. Late features include extremity paralysis and absent distal pulsation (pulseless paralysis). (Choice A) Fat embolism is infrequently seen in patients with long bone or pelvic fractures. Patients usually present with a triad of hypoxemia, neurological abnormalities (i.e., confusion), and a petechial rash (involving the head, neck, anterior chest wall, or axilla). (Choice B) Deep venous thrombosis usually does not present acutely in a setting of motor vehicle accident in an otherwise healthy young patient. It is a rare cause of compartment syndrome and does not compromise blood circulation and neuromuscular function. (Choice D) Vascular occlusion secondary to a motor vehicle accident usually presents more suddenly and dramatically. Acute compartment syndrome usually has a lag period of a few hours before irreversible nerve injury and muscle necrosis occurs. (Choice E) Nerve compression may occur in a patient after a motor vehicle accident and possible bone fracture; however, it does not lead to blood flow compromise and muscle necrosis (pain with passive movements of the involved muscles). Educational Objective: Acute compartment syndrome usually occurs after a traumatic event and causes pain, paresis, hypoesthesia, and diminished to absent pulses in the involved limb. 87% of people answered this question correctly. Item 2 of 4 Which of the following is the most common life-threatening complication of the above condition? A. Disseminated intravascular coagulation B. Rhabdomyolysis and renal failure C. Pulmonary embolism D. Gangrene of the limb E. Thrombocytopenia

Explanation: Acute compartment syndrome results in markedly diminished to absent tissue perfusion within hours of the inciting event, causing tissue necrosis, muscle infarction, and rhabdomyolysis, which releases myoglobin into the peripheral circulation. Myoglobin is directly toxic to the renal tubules, and subsequently causes acute tubular necrosis and acute renal failure. Acute renal failure and its complications (electrolyte disturbances) are one of the most common life-threatening complications of acute compartment syndrome. Laboratory studies typically reveal markedly elevated creatinine kinase levels and the presence of myoglobin in the urine (positive dipstick for blood in the absence of RBC?s in the urine). (Choices A, C, D, E) Thrombocytopenia, disseminated intravascular coagulation, pulmonary embolism, and gangrene of the limb are not usually seen in patients with acute compartment syndrome. Educational Objective: Rhabdomyolysis and subsequent development of acute renal failure is one of the most common and severe life-threatening complications of acute compartment syndrome. 24% of people answered this question correctly. Item 3 of 4 Which of the following is the most appropriate next step in the management of this patient? A. Administer oxygen B. Start the patient on anticoagulation C. Order venous Doppler ultrasonography D. Check the tissue pressure E. Order a nerve conduction study Explanation: Compartment syndrome is characterized by an increase in the tissue pressure in the enclosed myofascial compartments of the extremities. When the elevated tissue or compartment pressure reaches its threshold level, the capillaries collapse, and this eventually leads to tissue and muscle necrosis. The exact value for the tissue pressure at which blood flows to the muscle and nerve tissue stops is controversial. The current general consensus for the threshold value is greater than 30 mmHg. It is therefore important to measure the tissue or compartment pressure early in the course of management, especially if the diagnosis is in question. (Choice A) Oxygen is usually used in the supportive treatment of patients with fat or cholesterol emboli. (Choice B) Anticoagulation is usually required for patients with deep venous thrombosis. (Choice C) Venous Doppler ultrasonography is useful for the diagnosis of deep venous thrombosis. It is not helpful for the diagnosis of compartment syndrome. (Choice E) Nerve conduction studies are helpful in the diagnosis and localization of the site of nerve damage. Educational Objective: Direct measurement of the compartment or tissue pressure is the diagnostic procedure of choice for patients with suspected acute compartment syndrome. 80% of people answered this question correctly. Item 4 of 4

The appropriate step was taken for the patient. Which of the following is the best next step in management? A. Continue with oxygen therapy B. Obtain a hypercoagulable panel C. Consult a vascular surgeon D. Perform urgent fasciotomy E. Review the results of venous Doppler ultrasonography Explanation: Acute compartment syndrome is a surgical emergency. Any delay in treatment leads to irreversible muscle and nerve damage. A compartment pressure of 30 mmHg or greater warrants an emergent fasciotomy (also known as compartment release). Surgical decompression aims to relieve the pressure within the enclosed compartment and to restore the blood flow to muscles and other tissues within 6-10 hours of the initial symptoms. Some patients may develop a persistent sensory or motor deficit after an episode of acute compartment syndrome despite early fasciotomy. (Choice A) Oxygen therapy has no role in the management of patients with acute compartment syndrome. (Choice B) Obtaining a hypercoagulable panel may be useful in patients with idiopathic deep venous thrombosis. It is not indicated in patients with acute compartment syndrome. (Choice C) A consultation with a vascular surgeon is necessary in patients with vascular injury or occlusion. (Choice E) Venous Doppler ultrasonography is useful to diagnose deep venous thrombosis. It has no role in the diagnosis or management of patients with acute compartment syndrome. Educational Objective: An emergent fasciotomy is the definitive treatment in patients with acute compartment syndrome. Surgical decompression aims to restore the capillary blood flow and tissue perfusion. 86% of people answered this question correctly. A 48-year-old Hispanic woman with no past medical history is brought to the emergency department by an ambulance after being rescued from a burning apartment building. The woman was trapped in a smoke-filled back bedroom of the apartment. On physical exam, her vital signs are stable, and while there is some singing of her facial hair, there is no visible evidence of burns. An arterial blood gas is obtained, and the values are as follows: pH: 7.39 PaCO2: 42 mm Hg PaO2: 91 mm Hg HCO3-: 24 mEq/L What potential development is of most concern in this woman over the next 24 hours? A. Night terrors B. Acute stress disorder C. Sepsis D. Supraglottic edema E. Cardiac arrhythmias Explanation: The majority of burn center deaths are due to inhalation injuries. Supraglottic damage is one of the most alarming types of inhalation injuries and stems from inhaling hot air, steam, or smoke. The ensuing inflammatory response results in edema of surrounding soft tissues and significant narrowing of the airway. Fiberoptic laryngoscopy or bronchoscopy may be necessary to properly assess the

extent of airway involvement. In patients such as this woman, the clinician must maintain a high index of suspicion for airway injury and a correspondingly low threshold for intubation. (Choice A) Night terrors are episodes of sleep disturbance that are typically found in children aged 312 years, and are not of concern in this woman. (Choice B) Acute stress disorder is a psychiatric condition that occurs when an individual feels intense fear, horror, or helplessness during a traumatic event that involves actual or threatened death, or serious injury. Subsequently, the individual exhibits dissociative symptoms, but continues to be functionally impaired by re-experiencing the traumatic event. Unlike the more chronic posttraumatic stress disorder, acute stress disorder lasts a maximum of 4 weeks after the traumatic event. While this woman is certainly at risk for developing either acute stress disorder or posttraumatic stress disorder, neither condition would put her in immediate danger within the next 24 hours. (Choice C) Sepsis would be of grave concern if this woman had experienced substantial burns to her body that disrupted the skin?s protective barrier. Since no such burns are evident in her physical examination, the potential consequences of inhalation injury are much more ominous. (Choice E) The development of cardiac arrhythmias is indeed potentially lethal in any individual deprived of oxygen for a lengthy period; however, since this woman has normal vital signs and normal arterial blood gas values, it appears unlikely that her tissues were subjected to hypoxic stress great enough to induce life-threatening arrhythmias in the next 24 hours. Educational Objective: Suspect airway injury and edema in any individual who has been exposed to smoke or superheated air. Remember the ABCs (airway, breathing, circulation) and intubate early if the airway is in danger. 71% of people answered this question correctly.

A 36-year-old Caucasian nulligravida presents to your office for a routine check-up. Her past medical history is insignificant. She is concerned about the possibility of cervical cancer, because a friend of hers was recently diagnosed with invasive cervical cancer. You perform a Pap smear, and the results indicate that the sample is satisfactory, and a high-grade squamous intraepithelial lesion (HGSIL) is present. Which of the following is the next best step in the management of this patient? A. Repeat cytology in 6 months B. Proceed with colposcopy C. Do HPV testing D. Do excisional biopsy E. Reassure and repeat Pap smear in 12 months Explanation: HGSIL revealed on Pap smear indicates a 1-2% probability of already having invasive cervical cancer and a 20% probability of acquiring invasive cervical cancer if left untreated. Immediate referral for colposcopy and endocervical curettage is indicated. (Choice D) If colposcopy suggests HGSIL, a diagnostic excisional procedure should be performed. (Choice C) HPV testing may be indicated if cytologic examination reveals atypical squamous cells of undetermined significance (ASCUS). The results of HPV testing may influence the decision to proceed with a colposcopy in such cases. (Choices A and E) Repeating the cytology in 6 or 12 months is not correct because invasive cervical cancer can be missed. Educational Objective: If a Pap smear reveals a high-grade squamous intraepithelial lesion (HGSIL), immediate referral for a colposcopy is indicated. 79% of people answered this question correctly. The following Vignette applies to the next 3 items A 50-year-old Caucasian bookkeeper presents to clinic with complaints of weakness in the fingers of both hands, with her right hand more symptomatic than the left. She reports that periodically her fingers "feel numb, with some tingling, sort of like they fell asleep." When she shakes her hands, normal sensation is somewhat restored. The symptoms are more often present in the evening and at night but are occasionally experienced during the daytime as well. She has no abnormalities of sensation on the dorsal surfaces of her hands. She cannot recall when she first noticed the tingling sensations, but reports it has become slightly more difficult to sign her name to paperwork over the past several weeks. There is no history of injury to the upper extremities. Her medical history is significant for irritable bowel syndrome, well-controlled hypertension, and mitral valve prolapse. She currently takes atenolol and a fiber supplement. Physical examination is notable for mild atrophy of the thenar eminences bilaterally. Item 1 of 3 What is the next best step in establishing the diagnosis? A. Eliciting a positive Trousseau?s sign B. Eliciting pain or paresthesia upon compressing the wrist joint C. Radiographs of both wrists D. Magnetic resonance imaging of both wrists E. Nerve conduction studies Explanation:

Carpal tunnel syndrome is the most common nerve entrapment disorder and is characterized by numbness, tingling, and pain of the thumb and first three fingers. Eventually, atrophy of the hand musculature can develop. The diagnosis is suspected when: 1) the patient complains of paresthesias, hypesthesia, pain, or numbness in an area of the hand served by the median nerve, and 2) there are physical findings of median nerve compression, such as pain or paresthesias after compression of the median nerve at the wrist (Choice B) or secondary to tapping of the median nerve (Hoffman-Tinel test) or after 30-60 seconds of acute wrist flexion (Phalen maneuver). Although the sensitivity and specificity of these tests is rather low, the combination of suggestive history and physical examination findings indicates the diagnosis of carpal tunnel syndrome is likely. The positive Trousseau?s sign (Choice A) occurs when compression of the forearm produces spasm in the hand and wrist. This neuromuscular hyperexcitability is associated with hypocalcemia, not with carpal tunnel syndrome. Radiographs of the wrists (Choice C) are not indicated unless trauma or bony alterations are suspected. Magnetic resonance imaging (Choice D) should be obtained if there is reason to suspect a lesion in the carpal tunnel or if the patient has failed initial treatment attempts. Nerve conduction studies (Choice E) can establish the diagnosis of carpal tunnel syndrome, as well as localize the area of irritation. Physical examination should be performed first, however. Educational Objective: Obtaining findings of median nerve compression on physical examination is the first step in diagnosing carpal tunnel syndrome. 65% of people answered this question correctly. Item 2 of 3 The appropriate action is taken, and the findings suggest the most likely diagnosis. It is thought that her condition is secondary to repetitive wrist movements at her place of employment. What is the next best step in confirming the diagnosis? A. Obtain thyroid function tests B. Obtain erythrocyte sedimentation rate C. Refer for electrodiagnostic testing D. Refer for biopsy of the median nerve E. Refer for ultrasonography of the carpal tunnel Explanation: Slowed median nerve conduction velocity during electrodiagnostic testing (Choice C) serves as a more definitive method of diagnosing carpal tunnel syndrome. Because the study is somewhat painful and expensive, it is recommended only for three groups of patients: those with an unclear diagnosis after history and physical examination, those who have not responded to therapy, and those with evidence of motor dysfunction or atrophy of the thenar eminence. Since this patient has experienced bilateral atrophy of the thenar eminence (and appears to have developed the condition in her workplace, which often results in litigation), her diagnosis invites further scrutiny. Evaluation of thyroid function (Choice A) and the erythrocyte sedimentation rate (Choice B) would be indicated if carpal tunnel syndrome persists despite therapy or if symptoms recur. Biopsy of the median nerve (Choice D) is not typically recommended as a means of diagnosing carpal tunnel syndrome.

Ultrasonography of the carpal tunnel (Choice E) is highly operator dependent and is less often used, in part due to its inherent inconsistency. It can be of help in detecting synovitis, a finding sometimes indicative of inflammatory processes such as rheumatoid arthritis. Educational Objective: Electrodiagnostic testing, which uses nerve conduction velocity without electromyography, can be used to confirm the clinical diagnosis of carpal tunnel syndrome. Item 3 of 3 Which of the following is the best initial treatment for this condition? A. Surgical carpal tunnel release B. Injected corticosteroids C. Oral corticosteroids D. Splinting the wrist at night E. Physical therapy Explanation: Timely treatment of carpal tunnel syndrome can halt the progression of the condition and prevent the development of permanent disability. Most patients will fare well with only conservative treatment. It is important to first identify and eliminate those factors that appear to exacerbate the condition. Actions that require wrist flexion and extension (eg, making a fist, holding objects, keyboard work) often cause a significant rise in pressure within the carpal tunnel and should be minimized or avoided. One of the most helpful and cost-effective conservative therapies is nighttime usage of a cockup splint (Choice D), which prevents wrist flexion by keeping the wrists in a neutral position. Surgical release (Choice A) is indicated only in those patients who have repeatedly failed conservative measures (approximately one-third of those with carpal tunnel syndrome). It is not normally a firstline treatment. Corticosteroid injection (Choice B) of the carpal tunnel is most often performed in patients whose pain persists despite usage of more conservative measures. However, the procedure is dangerous and can cause nerve atrophy or necrosis if the steroid directly enters the median nerve sheath. A short-term course of oral corticosteroids (Choice C) can provide rapid pain relief. Commonly prescribed is 20 mg of prednisone daily for 7 days, followed by 10 mg daily for 7 days. Unfortunately, the benefits tend to dissipate over several weeks. Physical therapy (Choice E) in the form of wrist strength and mobilization exercises is often prescribed. However, a recent review of studies showed limited evidence of the benefit of exercise for carpal tunnel syndrome. Educational Objective: Nighttime splinting of the wrist is one of the more effective initial treatment options for carpal tunnel syndrome. 72% of people answered this question correctly. A 55-year-old Caucasian male comes to the emergency department with complaints of a sudden onset of upper abdominal pain. He has a history of alcoholism and chronic, recurrent pancreatitis. His physical examination and laboratory evaluation confirm another attack of pancreatitis. He is admitted to the hospital for observation, pain control, intravenous hydration, and bowel rest. Two days later, he complains of nausea and has two episodes of coffee-ground emesis. An upper GI endoscopy reveals the presence of varices in the fundus of the stomach, without any evidence of esophageal varices. Which of the following is the most likely cause of the above findings?

A. Refer her for surgical intervention. B. Portal vein thrombosis C. Hepatic venoocclusive disease D. Budd-Chiari syndrome E. Physical therapy Explanation: The presence of isolated gastric varices in a patient with a history of chronic, recurrent pancreatitis is suggestive of splenic vein thrombosis, which is one of the less frequent complications of chronic pancreatitis. The splenic vein runs along the posterior surface of the pancreas, and can get directly inflamed and thrombosed due to recurrent pancreatic inflammation. Apart from gastric varices, patients with chronic splenic vein thrombosis may develop noncirrhotic portal hypertension, ascites, and massive splenomegaly with associated features of hypersplenism (anemia, thrombocytopenia, and leukopenia). (Choice B) Portal vein thrombosis is one of the causes of prehepatic/noncirrhotic portal hypertension. The clinical consequences of portal vein thrombosis are similar to that of splenic vein thrombosis; however, patients with portal vein thrombosis have both gastric and esophageal varices. (Choice C) Hepatic venoocclusive disease is due to the occlusion of terminal hepatic venules and causes postsinusoidal portal hypertension. It presents as tender hepatomegaly, jaundice, and ascites. (Choice D) Budd-Chiari syndrome is due to the thrombosis of hepatic veins or intra/suprahepatic inferior vena cava. Acutely, it presents with right upper quadrant pain, hepatomegaly, jaundice, and rapidly developing ascites. Chronic Budd-Chiari syndrome usually presents with ascites, cirrhosis, and portal hypertension (gastroesophageal varices and splenomegaly). Educational Objective: Isolated gastric varices (without evidence of esophageal varices) due to splenic vein thrombosis can be seen as a complication of chronic recurrent pancreatitis. 60% of people answered this question correctly. A 58-year-old Hispanic woman comes to the emergency department and complains of vaginal bleeding. She also describes a feeling of heaviness in her lower pelvic area. The sensation is worse in the evening and with prolonged standing, and is relieved by lying flat on her back. Her other medical problems include a history of osteoporosis, degenerative joint disease of both knees, and borderline hypertension. She denies any past history of smoking. She had three full-term vaginal deliveries at 30, 32, and 36 years of age. She had her menopause six years ago. On pelvic examination, you notice an ulcerated, bleeding mass at the introitus. The mass protrudes through the vaginal orifice upon bearing down. Which of the following is the most appropriate next step in the management of this patient? A. Refer her for surgical intervention. B. Obtain an endometrial biopsy at this time. C. Push the mass back into the vaginal cavity manually. D. Insert an estrogen-coated pessary to hold the uterus and cervix. E. Physical therapy Explanation: The presence of isolated gastric varices in a patient with a history of chronic, recurrent pancreatitis is suggestive of splenic vein thrombosis, which is one of the less frequent complications of chronic pancreatitis. The splenic vein runs along the posterior surface of the pancreas, and can get directly inflamed and thrombosed due to recurrent pancreatic inflammation. Apart from gastric varices, patients with chronic splenic vein thrombosis may develop noncirrhotic portal hypertension, ascites,

and massive splenomegaly with associated features of hypersplenism (anemia, thrombocytopenia, and leukopenia). (Choice B) Portal vein thrombosis is one of the causes of prehepatic/noncirrhotic portal hypertension. The clinical consequences of portal vein thrombosis are similar to that of splenic vein thrombosis; however, patients with portal vein thrombosis have both gastric and esophageal varices. (Choice C) Hepatic venoocclusive disease is due to the occlusion of terminal hepatic venules and causes postsinusoidal portal hypertension. It presents as tender hepatomegaly, jaundice, and ascites. (Choice D) Budd-Chiari syndrome is due to the thrombosis of hepatic veins or intra/suprahepatic inferior vena cava. Acutely, it presents with right upper quadrant pain, hepatomegaly, jaundice, and rapidly developing ascites. Chronic Budd-Chiari syndrome usually presents with ascites, cirrhosis, and portal hypertension (gastroesophageal varices and splenomegaly). Educational Objective: Isolated gastric varices (without evidence of esophageal varices) due to splenic vein thrombosis can be seen as a complication of chronic recurrent pancreatitis. A 58-year-old Hispanic woman comes to the emergency department and complains of vaginal bleeding. She also describes a feeling of heaviness in her lower pelvic area. The sensation is worse in the evening and with prolonged standing, and is relieved by lying flat on her back. Her other medical problems include a history of osteoporosis, degenerative joint disease of both knees, and borderline hypertension. She denies any past history of smoking. She had three full-term vaginal deliveries at 30, 32, and 36 years of age. She had her menopause six years ago. On pelvic examination, you notice an ulcerated, bleeding mass at the introitus. The mass protrudes through the vaginal orifice upon bearing down. Which of the following is the most appropriate next step in the management of this patient? A. Refer her for surgical intervention. B. Obtain an endometrial biopsy at this time. C. Push the mass back into the vaginal cavity manually. D. Insert an estrogen-coated pessary to hold the uterus and cervix. E. Physical therapy Explanation: The presence of isolated gastric varices in a patient with a history of chronic, recurrent pancreatitis is suggestive of splenic vein thrombosis, which is one of the less frequent complications of chronic pancreatitis. The splenic vein runs along the posterior surface of the pancreas, and can get directly inflamed and thrombosed due to recurrent pancreatic inflammation. Apart from gastric varices, patients with chronic splenic vein thrombosis may develop noncirrhotic portal hypertension, ascites, and massive splenomegaly with associated features of hypersplenism (anemia, thrombocytopenia, and leukopenia). (Choice B) Portal vein thrombosis is one of the causes of prehepatic/noncirrhotic portal hypertension. The clinical consequences of portal vein thrombosis are similar to that of splenic vein thrombosis; however, patients with portal vein thrombosis have both gastric and esophageal varices. (Choice C) Hepatic venoocclusive disease is due to the occlusion of terminal hepatic venules and causes postsinusoidal portal hypertension. It presents as tender hepatomegaly, jaundice, and ascites. (Choice D) Budd-Chiari syndrome is due to the thrombosis of hepatic veins or intra/suprahepatic inferior vena cava. Acutely, it presents with right upper quadrant pain, hepatomegaly, jaundice, and rapidly developing ascites. Chronic Budd-Chiari syndrome usually presents with ascites, cirrhosis, and portal hypertension (gastroesophageal varices and splenomegaly). Educational Objective:

Isolated gastric varices (without evidence of esophageal varices) due to splenic vein thrombosis can be seen as a complication of chronic recurrent pancreatitis. 60% of people answered this question correctly. The following Vignette applies to the next 2 items A healthy 75-year-old African-American man comes to the physician because of persistent lower back pain for the past six months. The pain disappears when he sits down, improves when he bends over his knees, and gets worse when he extends his back. A couple of times before, when he was walking, he had a sudden onset of intense pain radiating from his back to his thighs. His other medical problems include hypertension, gastroesophageal reflux disease and rheumatoid arthritis. He does not use tobacco, alcohol, or drugs. He lives in a house with his wife. His medications include aspirin, atenolol, lansoprazole, and naproxen. He has no known drug allergies. On physical examination, straight-leg raising to 90 degrees is negative. Hyperextension of the lumbar spine produces lumbar pain radiating to the gluteal region. His pedal pulses are palpable and full.

Item 1 of 2 Which of the following is the most likely diagnosis? A. Spondyloarthrosis B. Disc herniation C. Vertebral fracture D. Lumbar spinal stenosis E. Lumbago Explanation: Lumbar spinal stenosis is a degenerative disease that affects the elderly population. It is characterized by increased lumbar pain on extension of the spine. The pain usually improves when the patient sits down or when he bends forward (e.g., when using a grocery cart). Romberg?s sign can be found on examination. (Choices A and C) Disappearance of the pain when the patient sits is highly specific for lumbar spinal stenosis. This characteristic pain is not found in spondyloarthrosis or vertebral fracture cases. (Choice B) Disc herniation worsens with lumbar flexion. (Choice E) Lumbago is usually self-limited, and the pain lasts for approximately two months. (Choice F) The preservation of pedal pulses helps distinguish the disease from vascular claudication. Educational Objective: Lumbar spinal stenosis is frequently seen in elderly patients. It usually appears during the sixth decade of life, and is very unusual before that age. The associated pain characteristically disappears/decreases upon sitting down, increases with spine extension, and decreases with flexion. Therapy can be conservative or can include a lumbar epidural block. Surgical decompression through a laminectomy is an option when other therapies fail. 62% of people answered this question correctly. Item 2 of 2 Which of the following is the most helpful in establishing the patient's diagnosis? A. X-ray of the lumbar spine B. Duplex arterial study C. MRI of the spine D. Electromyography E. HLA-B27 levels Explanation: The diagnosis of spinal stenosis in a patient with symptoms is best confirmed by using MRI of the spine. The encroaching of osteophytes at the facet joints, hypertrophy of the ligamentum flavum, and protrusion of intervertebral disks results in a narrowing of the spinal canal. In some patients, gait disturbance is so prominent that they complain of having "spaghetti legs" or walking "like a drunken sailor." The preservation of pedal pulses helps distinguish this disease from vascular claudication. (Choice A) X-ray will not be useful for the diagnosis of spinal stenosis. (Choice E) HLA-B27 is positive in more than 90% of patients with ankylosing spondylitis; however, the patient's history is not suggestive of this disease. Educational Objective: MRI is the investigative procedure of choice for suspected lumbar spinal stenosis.

73% of people answered this question correctly. A 41-year-old executive presents to clinic complaining of generalized lethargy and weakness. He says he is "stressed out" at his job. Lately he finds himself drinking four to six cups of coffee per day just to stay focused on the tasks at hand. He claims to eat an otherwise nutritious diet and drinks a glass of wine before retiring to bed at night. Physical examination is normal. Upon further inquiry, he hesitantly reveals that he has frequently visited prostitutes while on business trips to foreign countries. An ELISA test for HIV antibodies returns with a positive result and is confirmed with the Western blot. The patient indicates an interest in beginning antiretroviral therapy. He insists, however, that his wife not be informed about his diagnosis because he suspects the news would compel her to seek a divorce. Which of the following is the most appropriate response to his request? A. Agree to maintain confidentiality regarding his HIV status B. Agree to maintain confidentiality regarding his HIV status, but then contact his wife to inform her once he leaves the office C. State that you must inform his wife about his HIV status because you are her primary care physician too D. Submit information regarding his HIV status to a state agency so that his wife will be contacted by an official third party E. Persuade him to tell his wife about his HIV status Explanation: Patient confidentiality has been a sacred and fundamental principle of medicine since the writing of the Hippocratic Oath. In addressing an issue of such import as HIV status, then, it is extremely important to honor the patient?s right to privacy and confidentiality to the greatest extent possible. However, most states also acknowledge the necessity of notifying individuals (such as the spouse in this case) who are known to be at risk for contracting HIV. The case of Tarasoff v. Regents of the University of California established that when a physician finds an identifiable third party to be foreseeably endangered due to a patient?s conduct, that physician has a duty to warn the third party. Many medical ethicists argue that this legal precedent applies to the potential transmission of HIV as well. Therefore, the physician in this case should make every effort to convince the patient to inform his wife about his HIV status (Choice E), even if such a revelation results in divorce. To ease the patient?s burden, the physician should also offer to help in conveying the difficult news to the patient?s wife. If the patient refuses to inform his wife despite the physician?s best efforts, most states allow for the physician to then convey the information to the at-risk spouse. Agreeing to maintain the patient?s confidentiality (Choice A) puts the wife at great risk of contracting HIV and is therefore inappropriate. While it is ethical to ensure that the wife is informed about her risk of contracting HIV, a physician should never deceive his patient about his intentions (Choice B). The patient should be given sufficient opportunity to tell his wife about his HIV status before a third party intervenes (Choices C and D). Educational Objective: An HIV-positive patient should be persuaded to tell his spouse about his HIV status. If he refuses despite the physician?s best efforts, most states allow for the physician to then convey the information to the at-risk spouse. 84% of people answered this question correctly.

A 22-year-old African-American man comes to the emergency department because he sustained an animal bite in the local zoo. He claims that he was playing with the animal with a few other friends, when the animal suddenly attacked him. He has a small 1 x 1 cm laceration on the dorsum of his right hand; there is no loss or limitation of function. His last tetanus injection was approximately two years ago. Which of the following is the most important information to obtain before proceeding with the management of this patient? A. The exact location of the animal in the zoo B. The kind of animal involved in the incident C. The duration of the bite D. The time lapse between the bite and his presentation to the emergency room E. The presence of any endemic diseases in the zoo Explanation: An animal bite is an extremely common problem encountered in the emergency department. Most of the bites involve an animal known to the victim, and are unprovoked. Sometimes, the bites can be from stray and wild animals as well. It is important to obtain an accurate and complete history from the patient for appropriate management. All the above queries are important in the initial history; however, the most important piece of information is the exact location of the animal in the zoo. The bite in the zoo should not be presumed to be from the animals kept under observation. The involved animal could just have been within the surroundings, and not kept under observation; therefore, the animal may not have been vaccinated. This can change the whole management strategy, since a more aggressive approach will be warranted in such a situation. Educational Objective: The animal involved in the bite should be traced and ideally kept under observation to look for the development of symptoms in the animal. 15% of people answered this question correctly. A healthy 22-year-old Asian man comes to your clinic because of mild, crampy, abdominal pain accompanied by abdominal distention, bloating, flatulence, and diarrhea. He has had these symptoms for the past six years, and has noted that the symptoms usually occur after eating or drinking dairy products. His brother has similar symptoms. He has no other medical problems, and is not taking any medications. He denies the use of tobacco, alcohol, or illicit drugs. He has no known drug allergies. Physical examination reveals no abnormalities. His vital signs are within normal limits. Abdominal x-ray and ultrasound did not reveal any abnormalities. Which of the following is the most appropriate dietary recommendation for the patient? A. Stop amiodarone B. Chocolate milk C. Ice cream D. Yogurt with live activated cultures E. The presence of any endemic diseases in the zoo Explanation: The patient?s history is very typical of lactose intolerance. Yogurt is a good alternative source of calcium for this patient, as studies have shown that the fermented milk and live cultures in yogurt contain beta-galactosidase, which is well tolerated in lactose-intolerant patients. It is necessary, however, to advise the patient to be careful when choosing commercially available yogurt products, since milk or milk products are sometimes added back after the fermentation process of these products.

(Choices A, B, and C) Milk and ice cream have high concentrations of lactose. Complete restriction of these and other lactose-containing products is necessary in order to confirm the diagnosis and rid the patient of his symptoms. Once the patient is symptom-free, the patient may be instructed to gradually add lactose-containing products to his diet as long as he is able to tolerate it. Ice cream has a high concentration of sugar and fat, and can be used initially in small quantities after a lactose-free interval. Educational Objective: Yogurt is a good alternative source of calcium for patients with lactose intolerance, as studies have shown that the fermented milk and live cultures in yogurt contain beta-galactosidase, which is well tolerated in these patients. 73% of people answered this question correctly.

A 28-year-old gravida 1, para 0 African-American woman at 37 weeks of gestation presented to the labor and delivery unit complaining of uterine contractions spaced eight minutes apart. Her prenatal care was regular and unremarkable. Ultrasonogram at 16 weeks revealed twin gestations. An epidural anesthetic is administered in preparation for attempted vaginal delivery of both twins. Twelve hours after admission, the woman delivers a girl weighing 2480 grams (5 lbs, 7 oz). The second twin is in cephalic position and at +1 station with reassuring fetal heart tones and an intact amniotic sac. Labor does not resume. What is the next best step in managing this situation? A. Observation B. Forceps delivery C. Intravenous oxytocin D. Internal podalic version E. Cesarean section Explanation: Recent studies have indicated that if the fetal heart rate is reassuring, the second twin does not have to be delivered within a fixed time frame after the first twin. Expectant management suffices for the spontaneous deliveries of most twins, but electronic fetal monitoring and ultrasound are of significant help in promptly recognizing fetal distress when it does occur. Thus in this case, once the first twin is delivered, the positioning and heart rate of the second twin should be assessed with ultrasound. If labor has halted, oxytocin (Choice C) is then administered. Observation (Choice A) may ultimately be without harm, but accepted practice is to initiate oxytocin administration if labor has halted. Forceps delivery (Choice B) is contraindicated if the amniotic sac is intact. Moreover, since the second twin is in no distress, immediate delivery is not necessary. Internal podalic version (Choice D) is a procedure in which the physician manipulates the fetus inside the uterine cavity from the breech to cephalic position. Since the second twin in this case is in cephalic position, version is not necessary. When vaginal birth is attempted in the delivery of twins, it is important to be prepared for urgent cesarean section (Choice E), as there are a multitude of complications that can arise (e.g., prolapsed umbilical cord, fetal distress). Studies have indicated that emergency cesarean section is necessary in 10-30% of twin deliveries. In this case, the second twin is in cephalic position and fetal heart tones are reassuring, so cesarean section is not indicated at this time. Educational Objective: Once the first twin is delivered, the positioning and heart rate of the second twin must be assessed with ultrasound. If labor has halted, oxytocin should be administered. 55% of people answered this question correctly. A 55-year-old man comes to the clinic for a routine physical examination. His medical history is significant for hypertension controlled with thiazides, diabetes mellitus controlled with metformin, and hypercholesterolemia controlled with atorvastatin. His temperature is 37.2?C (99F), blood pressure is 140/90 mmHg, respirations are 12/min, and pulse is 80/min. Physical examination reveals a palpable abdominal mass. Auscultation reveals a bruit below his umbilicus. Abdominal ultrasound reveals a 5 cm abdominal aortic aneurysm. The sonologist?s note also mentions the possibility of aneurysmal extension to the renal arteries. A CT scan with IV contrast is suggested. The patient agrees to receive further management. What is the most appropriate advice to this patient? A. Stop thiazide one day prior to the procedure B. Stop statin two days prior to the procedure C. Stop metformin one day prior to the procedure

D. Start copious fluid intake the night before the procedure E. No change in present management Explanation: Metformin is a very useful agent for the treatment of type 2 diabetes; however, before starting metformin, careful consideration should be given regarding any contraindications to its use. Important contraindications include renal failure (creatinine 1.5 or more in males and 1.4 or more in females), decreased creatinine clearance (below 50 mL/min), congestive heart failure, significant liver disease, and alcohol abuse. The risk for lactic acidosis is increased with all these conditions. Furthermore, metformin should be discontinued in any situation where a decrease in creatinine clearance is expected (e.g., before the administration of contrast agents for radiologic procedures). Similarly, metformin should also be discontinued in sick, admitted patients. Other drugs which should not be used prior to intravenous contrast administration are NSAIDs. (Choice A) Thiazide need not be discontinued for the procedure, as long as the patient?s hydration is maintained. (Choice B) There is no need to discontinue statin before the administration of iodinated contrast agents. (Choice D) The efficacy of oral hydration in the prevention of contrast-induced nephropathy is unclear. Patients who are at risk for the development of acute renal failure following the administration of contrast agents should be given sodium bicarbonate or normal saline infusion before and after the administration of radio-contrast agents. N-Acetylcysteine (Mucomyst) can also be used. (Choice E) The patient needs discontinuation of metformin before the procedure, and no change in his current therapy is likely to result in significant complications. Educational Objective: Metformin should be held temporarily in patients who will undergo radiologic procedures using radiocontrast agents. High-risk patients should be treated with sodium bicarbonate or normal saline infusion and acetylcysteine to prevent contrast-induced acute renal failure. 57% of people answered this question correctly. A 55-year-old chronic alcoholic Mexican male presents to your office in the early morning with complaints of severe abdominal pain and vomiting after he had been binge drinking last night. He describes the pain as very severe and radiating to the back. His blood pressure is 110/70 mmHg, pulse rate is 110/min, temperature is 38.6 C (101.5 F), and respiratory rate is 26/min. His laboratory test results are as follows: WBC count: 22,000/cmm Total bilirubin: 1.0 mg/dl Serum lipase: 1,500 IU/L Serum amylase: 2,000 IU/L LDH: 360 IU/L AST: 270 IU/L Blood sugar: 200 mg/dl Serum calcium: 10.0 g/dl BUN: 20 mg/dl A CT scan of the abdomen shows a fluid collection around the pancreas. He is started on intravenous fluids, analgesics, nasogastric tube suction and is kept on "nothing by mouth" (NPO) order. Which of the following is the next best step in the management of this patient? A. Start the patient on ranitidine

B. Start the patient on imipenem C. Do an endoscopic pancreaticography D. Do a laparotomy E. Start the patient on pancreatic protease inhibitors

Explanation: This patient has an acute episode of alcoholic pancreatitis with poor prognosis, as assessed with Ranson?s criteria. The next best step in this patient's care would be to start prophylactic antibiotics such as imipenem or cefuroxime. Earlier studies have shown that the use of prophylactic antibiotics had no beneficial effect on morbidity and mortality rates; however, evidence from current experimental studies favors the use of prophylactic antibiotics in patients with severe, acute pancreatitis, large fluid collections, or sterile pancreatic necrosis involving more than 30% of the gland. In these conditions, prophylactic antibiotics should be used for at least 4-6 weeks. The carbapenem group of antibiotics (including imipenem) penetrates well into the pancreatic tissue and has a very broad spectrum including Staphylococcus, Pseudomonas, and Enterococcus. (Choice A) Studies have shown that H2 receptor antagonists have no effect on the course of acute pancreatitis; however, all patients are routinely managed with either H2 blockers or proton pump inhibitors to prevent stress-induced gastritis. (Choice C) ERCP can be used in patients with acute pancreatitis when there is concurrent dilatation of the biliary system and/or elevation of liver function tests. (Choice D) Surgery should be considered in patients with severe pancreatic necrosis, lack of response to nonoperative therapy, biliary pancreatitis, or for the treatment of specific complications. (Choice E) Protease inhibitors can prevent or suppress AP if given before the insult; therefore, these are particularly useful in preventing ERCP-induced acute pancreatitis. Educational Objective: Evidence from current experimental studies favors the use of prophylactic antibiotics in patient with severe acute pancreatitis, large fluid collections, or sterile pancreatic necrosis involving more than 30% of the gland. 49% of people answered this question correctly. A 33-year-old female with a 4-week history of fever and periodic chills is hospitalized. Her fever exceeds 38.3C (101F) during the day and has no consistent pattern. She denies any associated headache, cough, diarrhea, abdominal pain or rash. She is currently not taking any medications, except for acetaminophen, which she has taken several times to relieve the fever. She denies recreational drug use. Her past medical history is significant for an uncomplicated appendectomy performed five years ago. She denies any history of close contact with tuberculosis patients. She visited her parents two months ago in Missouri, and does not remember any mosquito or tick bites. She is sexually active in a monogamous relationship with her husband. Physical examination is insignificant. Preliminary investigations fail to establish the cause of fever. Which of the following is the most likely etiology of this patient?s condition? A. Infectious disease B. Neoplastic disease C. Collagen vascular disease D. Adult Still?s disease E. Factitious disorder Explanation: A thorough history (including a detailed travel history, immunization history, family history, drug and sexual history, occupation, etc.) and comprehensive physical examination is very important in the evaluation of a patient who presents with fever and no localizing signs. Fever of unknown origin (FUO) is said to occur when a patient presents with a temperature exceeding 38.3C (101F) on several occasions, the fever lasts more than 3 weeks, and no diagnosis can be established after one week of

inpatient investigation. According to the literature, infectious conditions are the most common cause of FUO; these account for 30-40% of cases. (Choice B) Neoplastic diseases account for 20-30% of cases of FUO. (Choice C) Collagen vascular diseases account for 10-20% of cases of FUO. (Choices D and E) Other causes, such as adult Still?s disease and factitious disorder, are less common. Educational Objective: Infectious conditions are the most common cause of FUO; these account for 30-40% of cases. 48% of people answered this question correctly. A 42-year-old chronic alcoholic African-American female comes to the emergency department and complains of severe abdominal pain of acute onset (three hours ago). On examination, she is in severe distress and has mild epigastric tenderness. Her laboratory test results reveal the following: Total bilirubin: 1.0 mg/dl AST: 45 IU/L ALT: 40 IU/L Serum lipase: 1,500 IU/L Serum amylase: 1,800 IU/L A diagnosis of acute pancreatitis is made. Which of the following set of laboratory data will help assess the prognosis of her pancreatitis at this point in time? A. Serum calcium and serum albumin B. Blood urea nitrogen and hematocrit C. WBC count and LDH levels D. WBC count and serum calcium levels E. Hematocrit and LDH levels Explanation: A number of scoring systems have been developed for determining the prognosis in patients with acute pancreatitis. Among these, the Ranson?s scoring system is the most widely adopted. Ranson?s criteria are as follows: A. Within first 48 hours: 1. Age > 55 years 2. WBC count > 16,000/cmm 3. LDH levels > 350 IU/L 4. Glucose levels > 200 mg/dl 5. AST > 250 IU/L B. After 48 hours: 1. Po2 < 60 mmHg 2. Calcium < 8 mg/dl 3. BUN increase > 5 mg/dl 4. Hematocrit decrease > 10% 5. Albumin < 3.2 mg/dl 6. Estimated fluid deficit > 4 L The presence of three or more of these criteria is associated with a grave prognosis in acute pancreatitis.

Another scoring system entitled "Acute Physiology and Chronic Health Evaluation (APACHE II) score" is gaining popularity. The APACHE II score is based on the evaluation of clinical data (i.e., blood pressure, pulse, temperature), biochemical data (i.e., urea and electrolytes), and renal and pulmonary function parameters. (Choices A, B, D and E) These are important predictors of prognosis after the first 48 hours of acute pancreatitis. Educational Objective: Know the Ranson?s criteria used to assess the prognosis in patients with acute pancreatitis. 42% of people answered this question correctly. A healthy, 29-year-old African-American woman comes to the physician for a routine health maintenance examination. She has a history of migraine headaches. She has been smoking one pack of cigarettes daily for the last three years. She drinks alcohol only on weekends. Her father died of colon cancer at the age of 50, and her mother is diabetic. She is currently taking no medications. Her vital signs are within normal limits. Examination shows no abnormalities. Her annual pap smear shows high-grade squamous intraepithelial lesion (LSIL). Satisfactory colposcopy examination confirms CIN II. Which of the following is the most appropriate next step in the management of this patient? A. Expectant management B. Cryosurgery C. Laser ablation D. Cold knife conization E. LEEP Explanation: High-grade squamous intraepithelial lesions include CIN II, CIN III, moderate and severe dysplasia, and carcinoma in situ. High-grade squamous intraepithelial lesions are more likely to be progressive than low-grade lesions, and these should always be treated with ablation or excision. Ablation can be done using cryosurgery or laser, and excision can be done using knife conization, laser conization or Loop Electrosurgical Excision Procedure (LEEP). Loop Electrosurgical Excision Procedure (LEEP) is the treatment of choice for high-grade squamous intraepithelial lesion. LEEP is preferred because of its low cost, accuracy, and easiness to perform. It is a very successful procedure and can be performed in an office setting. (Choice A) Expectant management is not the appropriate choice for high-grade lesions. (Choices B, C and D) Ablation is done when a histologic diagnosis is accurate, when there is no evidence of invasion, and glandular lesion, and when colposcopy is satisfactory. Otherwise, excision is preferred. Educational Objective: LEEP is the treatment of choice for high-grade squamous intraepithelial lesions. 50% of people answered this question correctly. A 30-year-old Caucasian man with no significant past medical history presents to his primary care physician complaining of nausea, vomiting, diarrhea, abdominal cramping, and fever to 38.3C (100.9F). The symptoms started 12 hours after the man consumed a beverage that contained pureed fruit, powdered protein, and three raw eggs. Physical examination of the man is unremarkable. Salmonella enteritidis is subsequently isolated from his stool culture. What is the best means of managing this patient?s care? A. Treatment with ampicillin

B. Treatment with ciprofloxacin C. Treatment with trimethoprim/sulfamethoxazole D. Treatment with ceftriaxone E. Supportive therapy and observation Explanation: Symptomatic individuals found to be infected with Salmonella enteritidis should be given replacement fluid and electrolytes, as they are at risk for becoming dehydrated. Since the gastroenteritis is usually self-limited and antibiotic use has not been shown to hasten the resolution of symptoms or improve the rate of Salmonella clearance from stool, antibiotic usage is not recommended for immunocompetent adults or children older than one year of age. Therefore, supportive therapy and observation (Choice E) would be most appropriate. In the United States, Salmonella enteritidis is becoming increasingly resistant to ampicillin (Choice A). Even if antibiotic therapy were indicated, ampicillin would be a poor choice. Preemptive therapy is warranted in patient groups at greater risk for complications. These groups include children younger than twelve months of age (with special attention paid to neonates) and immunocompromised adults. Therapy may be considered for adults at least 50 years old with known atherosclerotic disease, as they are more prone to developing bacteremia and endovascular infection. Effective antibiotics include ciprofloxacin (Choice B), trimethoprim/sulfamethoxazole (Choice C), and ceftriaxone (Choice D). The latter antibiotic is only available intravenously, so its usage is restricted. Educational Objective: Salmonellosis does not need to be treated with antibiotics in immunocompetent individuals age 12 months or older. 48% of people answered this question correctly. A 22-year-old Caucasian female is hospitalized after a car accident. She sustained a hip fracture, fracture of several ribs, and a blunt abdominal injury that required a laparotomy. The laparotomy revealed a liver laceration and extensive hemoperitoneum. In the early postoperative period, the patient is noted to have hyperactive deep tendon reflexes. Which of the following electrolyte abnormalities may be responsible for this condition? A. Hypokalemia B. Hyperkalemia C. Hyponatremia D. Hypocalcemia E. Hypermagnesemia Explanation: Hypocalcemia is the most probable diagnosis in this patient. Hypocalcemia can occur during or immediately after surgery, especially in patients undergoing major surgery and requiring extensive transfusions. Usually, hypocalcemia occurs due to volume expansion and hypoalbuminemia, and is therefore asymptomatic; however, sometimes it may manifest as hyperactive deep tendon reflexes, muscle cramps and, rarely, convulsions. Hypomagnesemia may mimic hypocalcemia, but is associated with alcoholism, prolonged nasogastric suction or diarrhea, and diuretic use. (Choice E) Mild hypermagnesemia results in decreased deep tendon reflexes. A severe form causes loss of the deep tendon reflexes and muscle paralysis, thereby leading to flaccid quadriplegia, decreased respiration, and eventual apnea.

(Choice B) Hyperkalemia typically results in gastrointestinal disturbances (nausea, vomiting), ECG changes, and asystole, if severe. It may be associated with severe burns, crush injuries, and renal insufficiency. (Choices A and C) Hypokalemia and hyponatremia are unlikely to manifest as hyperactive deep tendon reflexes. Educational Objective: Hypocalcemia can occur during or immediately after surgery, especially in patients undergoing major surgery and requiring extensive transfusions. Hyperactive deep tendon reflexes may be the initial manifestation. Hypermagnesemia, on the other hand, results in loss of the deep tendon reflexes. 45% of people answered this question correctly. An 18-year-old high school sophomore is brought to the emergency room after she was involved in a motor vehicle accident. Upon her arrival to the emergency room, she is minimally responsive, and there is evidence of bleeding from the right side of her head. Her vital signs are as follows: temperature 37.6?C (99.6F), blood pressure 182/98 mmHg, heart rate 52 per minute and respiratory rate 6 per minute. Her oxygen saturation is 96% on two liters of oxygen. Her breath smells of alcohol. Physical examination reveals a right pupillary size of 7 mm with minimal response to light. The left pupil is 3 mm with normal pupillary light reflex. The right eye is deviated outwards and downwards. There is bilateral papilledema on funduscopic examination. While in the ER, the patient had an episode of generalized body extension, with adduction and pronation of her upper extremities. There is no evidence of any thoracic or abdominal injuries. Which of the following is the most appropriate next step in the management of this patient? A. Intravenous thiamine B. CT scan of the brain C. Intravenous mannitol D. Hyperventilation to achieve a PaCO2 of 25-30 mmHg E. Endotracheal intubation Explanation: The above patient has the typical clinical features of elevated intracranial pressure (ICP), most likely secondary to an intracranial hemorrhage from head trauma sustained in the motor vehicle accident. Other causes of elevated intracranial pressure include an intracranial hemorrhage from ruptured aneurysms, bleeding AV malformations, CNS infections, CNS neoplasm, and hydrocephalus. Patients with intracranial hypertension have been classically described to have bradycardia, hypertension and respiratory depression (Cushing's triad). In the early stages, patients complain of headaches, vomiting, blurred vision, and have papilledema on funduscopic examination. Further elevation in the intracranial pressure leads to transtentorial herniation of brain tissue, causing altered levels of consciousness (stupor progressing to coma), dilation of the ipsilateral pupil, third cranial nerve palsy, hemiparesis, decerebrate posturing, and eventually, respiratory arrest. The patient in the above vignette has signs of marked respiratory depression and intracranial hypertension. The most important next step in the management is to rapidly intubate the patient to protect and maintain her airway, in case of a respiratory arrest. (Choice A) Intravenous thiamine is useful in alcoholic patients with severe malnutrition to prevent the development of Wernicke's encephalopathy. It has no role in the lowering of an elevated intracranial pressure. (Choice B) CT scan of the brain should be done urgently to look for an intracranial hemorrhage or any other potentially reversible causes of elevated intracranial pressure; however, the patient?s airway should be secured before sending her for neuroimaging.

(Choice C) Intravenous mannitol is an osmotic diuretic which reduces brain volume by drawing water out of the cells. It can be used to acutely lower elevated intracranial pressures in emergent situations; however, securing and maintaining a patent airway should still remain the first priority in the management of such patients. (Choice D) Lowering the PaCO2 by hyperventilating the patient causes cerebral vasoconstriction, which decreases the cerebral blood flow, and reduces intracranial pressure. Hyperventilation is therefore used to decrease the PaCO2 to 25-30 mmHg and to rapidly lower intracranial pressure in emergent situations; however, it is contraindicated in patients with traumatic brain injury and an acute stroke. This is due to the fact that a significant decrease in cerebral blood flow caused by vasoconstriction can lead to worsening of neurological injury in these patients. Educational Objective: Securing and maintaining a patent airway is the first priority in the management of trauma patients with symptomatic intracranial hypertension. 73% of people answered this question correctly. A 41-year-old woman comes to see you in the office for the first time. She has recently moved to your town and wants to establish her medical care with a primary care physician. She is also requesting for an influenza vaccination for this year. She tells you that she had a "really bad" episode of flu last year and had to take a week off from work. She denies any active medical problems. She does not have a history of any chronic medical illness. She smokes approximately one pack of cigarettes a week and drinks alcohol occasionally. Her mother and father both had diabetes mellitus; her father also had significant coronary artery disease. She works as a secretary in the local law firm. She is married and has two sons, who are 8 and 11 years old. Her husband and sons are all in good health, and do not have any medical problems. Which of the following is the most appropriate response to this patient's request for vaccination?

A. You should be fine because you have antibodies from last year's infection. B. You do not have any risk factors, so vaccination is not advisable. C. You should wait for the influenza epidemic to start, and then return for vaccination. D. I agree. Lets give you the vaccine. E. You have two young kids at home. You should get the vaccine now and every year. Explanation: Influenza is an acute viral respiratory illness caused by influenza A and B virus. It occurs in epidemics nearly every year, usually in the winter season. It is responsible for a substantial morbidity in the general population. Mortality occurs due to an associated influenza pneumonia and occurs mainly in patients with chronic underlying cardiac, pulmonary, and other medical illnesses. Vaccination of the high-risk groups is the major means of preventing influenza and its associated mortality. The United States Preventive Health Services and Center for Disease Control and Prevention (CDC) have devised guidelines for the vaccination of people who are at high risk of developing complications from influenza. Certain groups of people at high risk include the following: Persons 50 years of age or older (age limit has been increased to 65 or older for 2004-2005 season secondary to vaccine shortage). All adults and children with a chronic health condition (heart disease, diabetes, kidney disease, asthma, COPD, HIV/AIDS, and malignancy, etc). All residents of nursing homes and long-term care facilities. Children between 6 and 23 months of age. Children and young kids 6 months to 18 years of age on long-term aspirin treatment. Women who will be in second or third trimester of pregnancy in the influenza season. Healthcare workers or workers at long-term care facilities who may transmit influenza to high-risk persons. Household members of persons at high risk of developing influenza and its complications. Out-of-home caregivers and household contacts of children less than 6 months of age. The woman in the above vignette does not have any risk factors or any high-risk contacts at home or work. Although she may contract influenza, she is unlikely to have the associated complications; therefore, it is not advisable to vaccinate her at this point. (Choice A) The influenza virus (influenza A virus in particular) undergoes periodic changes in its antigenic characteristics; therefore, prior infection or vaccinations are not helpful in preventing future infections. (Choices C and D) Vaccination of a patient without any known risk factors is not recommended. (Choice E) Household members of children or adults with chronic medical illnesses should receive an influenza vaccine annually. Since this patient?s husband and two sons do not have any chronic medical illness, vaccination is not recommended. Educational Objective: Influenza vaccination is generally recommended only for those people who are at high risk for influenza-related complications. 38% of people answered this question correctly. A 72-year-old, male, nursing home patient is admitted to the hospital with a three-day history of nausea, vomiting, diarrhea, and confusion. The nurses at the nursing home tell you that he has been drinking a lot of water for the past three days. He has a history of tobacco abuse, diabetes mellitus, hypertension, coronary artery disease, multiple myeloma, and schizophrenia. His medications include aspirin, insulin, metoprolol, and chlorpromazine. His laboratory findings in the hospital reveal a serum sodium level of 120 mEq/L, urine sodium concentration of 80 mEq/L, serum osmolality of 258 milliosmoles/kg, and urine osmolality of 400 milliosmoles/kg. A chest x-ray showed a 3 cm mass

around the right hilar region. Which of the following is most likely cause of the patient's laboratory findings? A. Syndrome of inappropriate ADH secretion B. Chlorpromazine toxicity C. Hyperproteinemia secondary to multiple myeloma D. Volume depletion from excessive vomiting and diarrhea E. Excessive water ingestion Explanation: Hyponatremia is characterized by an excess of water in relation to the total body sodium concentration. It is caused by excessive free water intake (primary polydipsia), endocrine disorders (i.e., adrenal insufficiency and hypothyroidism), or impaired water excretion from advanced renal failure and excessive antidiuretic hormone (ADH) release. Excessive ADH secretion can be due to decreased effective circulating volume (true volume depletion secondary to vomiting and diarrhea, congestive heart failure, cirrhosis or overuse of diuretics) and syndrome of inappropriate ADH secretion (SIADH). SIADH can be seen in a variety of CNS disorders, pulmonary diseases, HIV infection, tumors (i.e., small cell carcinoma of the lung), use of drugs such as carbamazepine, cyclophosphamide and selective serotonin re-uptake inhibitors, and in postoperative patients. Patients with hyponatremia secondary to SIADH usually have a decreased plasma or serum osmolality, elevated urinary osmolality (due to excessive fluid retention), urinary sodium concentration of more than 40 mEq/L and normal renal, adrenal and thyroid functions. The patient in the above vignette appears to have SIADH secretion secondary to lung cancer, most likely small cell carcinoma of the lung. (Choice B) The drugs that can cause excessive ADH release include selective serotonin re-uptake inhibitors (fluoxetine and sertraline), chlorpropamide, carbamazepine, and cyclophosphamide. Chlorpromazine is an anti-psychotic agent and is not associated with SIADH. (Choice C) Pseudohyponatremia secondary to hyperlipidemia or hyperproteinemia from any cause is usually associated with a normal or elevated plasma osmolality. (Choice D) True volume depletion due to gastrointestinal (vomiting, diarrhea) or renal causes can cause hyponatremia with a low plasma osmolality and elevated urine osmolality; however, the urinary sodium concentration is typically less than 20 mEq/L in these patients. (The urinary sodium concentration is usually more than 40 mEq/L in patients with SIADH.) (Choice E) Patients with hyponatremia secondary to primary polydipsia or excessive water ingestion excrete a very dilute urine and have a urine osmolality of less than 100 milliosmoles/kg. Educational Objective: The patients with hyponatremia secondary to SIADH typically have a low plasma osmolality, elevated urine osmolality, and a high urinary sodium concentration. 89% of people answered this question correctly. A 76-year-old male with a history of hypertension, diet-controlled type-2 diabetes mellitus, coronary artery disease, and atrial fibrillation comes to the office for a follow-up. The patient?s medication includes aspirin, metoprolol, hydrochlorothiazide, warfarin, amiodarone, and multivitamins. He has been on all these medications for one year. He complains of tiredness, weight gain, and occasional swelling in his feet for the last three months. He denies chest pain, dyspnea, orthopnea, or dizziness. Physical examination is unremarkable, except for a 10-lbs.-weight gain since his last visit six months ago. Which of the following is the most appropriate next step in the management of this patient?

A. Discontinue hydrochlorothiazide and add furosemide B. Decrease the dose of metoprolol C. Check thyroid-stimulating hormone level D. Discontinue amiodarone E. Add digoxin

Explanation: The next best step in the management of this patient is to check serum TSH levels. The patient is experiencing symptoms of hypothyroidism, which may be attributed to amiodarone use. Amiodarone causes thyroid dysfunction due to its high iodine content. Hypothyroidism (85%) is more common than thyrotoxicosis (10-15%). Two mechanisms by which amiodarone-induced-thyrotoxicosis may occur are: activation of Graves? disease (type-1 thyrotoxicosis) and destructive thyroiditis (type-2 thyrotoxicosis). Graves' disease induced by amiodarone is generally treated with high-dose thioamides (methimazole or propylthiouracil). Perchlorate can be used to decrease further iodine uptake by the thyroid gland. Steroids are generally required for treating type-2 thyrotoxicosis. Thyroid functions are monitored every six months in euthyroid patients on amiodarone. It is prudent to start with a lower dose in elderly patients or those who have significant coronary artery disease. (Choices A and E). The patient does not have congestive heart failure (no orthopnea, no SOB, no JVD, and normal lung exam); therefore, starting a stronger diuretic or adding digoxin would not help. (Choice B) Fatigue could be a side effect of a beta-blocker; however, in this case, fatigue is more likely due to amiodarone-induced hypothyroidism. (Choice D) It is not necessary to discontinue amiodarone if a patient becomes hypothyroid . Generally, patients who have amiodarone-induced hypothyroidism required a higher dose of levothyroxine to bring their TSH within normal range because amiodarone inhibits conversion of T4 to its active form T3. The hypothyroidism is treated with levothyroxine. Educational Objective: Amiodarone can cause thyroid dysfunction, corneal deposits, skin discoloration, pulmonary fibrosis (lipoid pneumonitis), and liver toxicity. 74% of people answered this question correctly. A 29-year-old Caucasian man is brought to the emergency department after he was found lying on the street, unconscious, and bleeding from his right leg. He was involved in a fight, and was stabbed twice in the right thigh. On arrival, his clothes are partially covered with snow. He is extremely pale, and his extremities are cold. His temperature is 35C (95F), blood pressure is 90/60 mm Hg, pulse is 120/min and respirations are 12/min. Examination shows decreased bilateral pulses, superficial breathing effort, clear lungs on auscultation, and decreased heart sounds. There is slow bleeding through one of the two penetrating wounds in his right thigh. The patient is stuporous, but does not have any focal neurological abnormality. He is emergently intubated and infused with warm fluids. His body is covered with warm blankets. The patient?s laboratory tests reveal: CBC Hb: 6.3 g/dL Ht: 19% MCV: 92 fl Platelet count: 150,000/cmm Leukocyte count: 4,500/cmm Segmented neutrophils: 65% Lymphocytes: 29% Monocytes: 6% Serum chemistry Serum Na: 134 mEq/L Serum K: 3.8 mEq/L Chloride: 100 mEq/L Bicarbonate: 14 mEq/L BUN: 15 mg/dL Serum Creatinine: 0.7 mg/dL

Calcium: 9.8 mg/dL Blood Glucose: 52 mg/dL Because of the acute anemia, the patient receives a transfusion of packed red blood cells. He also receives parenteral glucose and thiamine, and his mental status slightly improves. He suddenly develops muscle spasms of the face and upper extremities, diaphoresis, and bilateral hand contracture. Fundoscopy reveals papilledema. He soon develops involuntary, repetitive, generalized tonic-clonic seizures. He is given intravenous lorazepam, which temporarily controls the abnormal movements. He then has another seizure episode. Which of the following is the most appropriate immediate step in the management of this patient? A. Start bicarbonate infusion B. Start potassium replacement C. Start calcium replacement D. Start fresh frozen plasma infusion E. Order a head CT scan Explanation: In general, symptomatic hypocalcemia resulting from transfusion of citrated blood is rare, because normal individuals rapidly metabolize citrate in the liver and kidney; however, patients with renal failure, hepatic failure, hypothermia or shock who receive blood transfusions have a high risk of hypocalcemia. This is due to their inability to metabolize citrate, which is concomitantly transfused with every blood transfusion. The conversion of citrate into lactate is impaired, which leads to an excess amount of citrate in the blood. The excess citrate then binds calcium, and this leads to hypocalcemia. The measured serum calcium levels may remain normal despite this occurrence, because the deficit of ionized calcium is not reflected in the total calcium levels. Prophylactic administration of at least 10 cc of 10% calcium gluconate is therefore recommended for every 500 ml of packed red blood cells transfused. (Choice A) Sodium bicarbonate is only indicated if the patient?s serum bicarbonate levels are extremely low (5 mEq/L or less). As the patient?s circulation improves, there will be a rebound or increase in the concentration of alkali. This may be aggravated by exogenous administration. Some of the excess bicarbonate will also be transformed into carbon dioxide in the brain, which may cause paradoxical acidification of the central nervous system. (Choice B) Patients with hypothermia usually develop hypokalemia. This patient is currently normokalemic; however, as he recovers from the metabolic acidosis induced by hypothermia, his potassium level may become slightly low. At this point, an urgent potassium replacement is not necessary. (Choice D) There is no need for transfusion of fresh frozen plasma because there is no evidence of coagulopathy. (Choice E) The patient?s seizures and new neurologic findings are due to a metabolic problem. Transient papilledema is a usual finding in significant hypocalcemia that leads to seizures. Unless there is strong evidence or suspicion of head trauma, there is no urgent need for a head CT scan. Educational Objective: Patients with hypothermia or shock who receive blood transfusions are predisposed to hypocalcemia because of their impaired ability to metabolize citrate into lactate. In these patients (renal failure, hepatic failure, shock, lactic acidosis) prophylactic administration of at least 10 cc of 10% calcium gluconate is recommended for every 500 ml of packed red blood cells transfused. 33% of people answered this question correctly.

A 23-year-old African-American woman, gravida 2 para 1, presents to the emergency department at 32 weeks gestation with regular menstrual-like cramping and low back pain. Her past medical history is significant for preterm labor at 28 weeks gestation, which resulted in infant death in the early postnatal period. Physical examination shows regular uterine contractions and cervical dilation (3 cm). No evidence of membrane rupture is present. The contraction stress test is negative. You decide to proceed with adequate hydration and tocolysis. Which of the following steroids is preferred for antenatal corticosteroid therapy? A. Prednisone B. Betamethasone C. Hydrocortisone D. Fluticasone E. Fludrocortisone Explanation: Antenatal corticosteroid therapy has been proven to be effective in reducing perinatal morbidity and mortality associated with preterm labor. It reduces the risk of infant respiratory distress syndrome by stimulating phospholipids (surfactant!) synthesis and accelerating morphologic lung development. In addition, antenatal corticosteroid therapy appears to reduce the risk of intraventricular hemorrhage in infants. For these reasons, it should be given to any pregnant woman from 24 to 34 weeks of gestation with intact membranes at high risk for preterm delivery. Two regimens of therapy are available: betamethasone and dexamethasone (some authors believe that betamethasone is preferred over dexamethasone). (Choice C) Hydrocortisone is not considered effective because it is extensively metabolized in the placenta and only a little proportion reaches the fetus. (Choice A) Prednisone, the most popular agent for corticosteroid therapy, is not used for antenatal therapy. (Choice D) Fluticasone is an inhalational agent used for the treatment of allergic and inflammatory disorders. (Choice E) Fludrocortisone is a synthetic mineralocorticoid, not glucocorticoid agent. Educational Objective: Antenatal corticosteroid therapy has been proven to be effective in reducing perinatal morbidity and mortality associated with preterm labor. Betamethasone (and sometimes dexamethasone) is used for antenatal corticosteroid therapy. 79% of people answered this question correctly. The following vignette applies to the next 2 items A 65-year-old female is seen for acute mid-thoracic back pain. Seven days ago, she slipped over ice, and has had the back pain ever since. The pain is described as deep-seated, boring in quality, nonradiating, and without any aggravating or relieving factors. The pain has not improved, despite the use of over-the-counter analgesics. Her past medical history is significant for chronic obstructive pulmonary disease and hypertension. She is currently on fluticasone, salmeterol, and ipratropium inhalations, as well as oral lisinopril and hydrochlorothiazide. She denies prolonged use of oral glucocorticoids. She has a 30-pack-year history of smoking, but quit approximately five years ago, when she was diagnosed with chronic obstructive pulmonary disease. She had menopause when she was 48 years old. Her mammogram and Pap smear were negative six months ago. Her mother and two sisters have osteoporosis. Examination reveals tenderness over the mid-thoracic vertebrae. The rest of the examination is unremarkable. The complete blood count, basic metabolic panel, calcium

and phosphorus are within normal limits. Plain radiographs of the thoracic spine reveal a compression fracture in the 10th and the 11th vertebrae.

Item 1 of 2 What is the next best step in the management of this patient? A. Check bone mineral density of the hip and spine using dual energy X-ray absorptiometry (DEXA) B. Measure the heel bone mineral density by using ultrasound C. Bone biopsy D. Check parathyroid hormone levels E. Perform a technetium bone scan Explanation: The patient has multiple risk factors for bone loss. These factors are: Caucasian race, family history of osteoporosis, poor lifestyle, and postmenopausal status. Measurement of her bone density using a central device (preferably DEXA) is recommended. Although she qualifies for treatment irrespective of her bone mineral status due to her compression fracture, baseline DEXA will still be useful for monitoring her response to osteoporosis treatment. (Choice B) Peripheral devices such as heel ultrasounds can predict fracture risks; however, monitoring the patient's response to treatment using a peripheral device is not advocated. Even if low bone mass is documented by a peripheral device, baseline bone mineral density by DEXA will still be necessary for monitoring. Directly proceeding with measurement of central bone mineral density is therefore advised. (Choice C) Bone biopsy is almost never required for the diagnosis of primary osteoporosis. (Choice D) This patient has normal calcium and phosphorous levels. She does not have any clinical features that suggest primary hyperparathyroidism. Measurement of the PTH levels are not required. (Choice E) Bone scan is a sensitive but very nonspecific test for a variety of bone diseases. It can be useful in differentiating old from new vertebral fractures (an increased uptake is only seen in new compressive fractures.). It can also be useful if metastatic disease is suspected. In this patient, a bone scan is likely to show an increased uptake in the region of the compressive fracture, but this will not be very useful in managing the patient. Educational Objective: The evaluation of patients with suspected osteoporosis is best performed by performing central bone density measurement (hip and lumbar spine) using DEXA. 88% of people answered this question correctly. Item 2 of 2 Her 25-hydroxyvitamin D level is 25 mcg/L (normal 18 to 68 mcg/L), and TSH level is 2.1 microunits/mL (normal 0.35 to 5 microunits/mL). Bone density at the spine and hip shows a T-score of ?2.7 at the lumbar spine and ?2.0 at the hip. What is the best way to treat this patient? A. Start calcium and vitamin D supplementation B. Calcium and vitamin D supplementation with alendronate C. Calcium and vitamin D supplementation with daily nasal calcitonin D. Subcutaneous parathyroid hormone injections E. Calcium and vitamin D supplementation with hormone replacement therapy Explanation: Once osteoporosis is suspected, bone mineral density (BMD) should be measured. T- and Z- scores are routinely used to report BMD. The T-score is the BMD score of a patient in comparison to young healthy adults around 25-30 years of age. The Z-score is the BMD score of a patient compared to age-matched normal controls.

The World Health Organization categorizes BMD in postmenopausal Caucasian women (WHO criteria) according to the T-score (BMD score in comparison to young white females), as follows: normal osteopenic osteoporosis (T-score > 1.0) (T-score between ?1.0 and 2.5) (T-score <-2.5)

As the T-score decreases, the fracture risk increases without any threshold effect. The diagnosis of osteoporosis in men and young premenopausal females by bone densitometry criteria is not clear. In addition to measuring bone mineral density (BMD), some basic chemical workup is also required to look for secondary causes of osteoporosis. This basic workup includes a complete blood count and routine chemistry profile with calcium and phosphorous levels. More thorough investigations may also be done, and these include urinary calcium, serum protein electrophoresis, PTH, TSH, and bone turnover markers (N-telopeptide for bone resorption and alkaline phosphatase for bone formation). These tests are usually decided on the basis of clinical features and initial laboratory values. The National Osteoporosis Foundation recommends bone-specific drug treatment for low bone density in all postmenopausal females with a T-score of ?2.0 or less, and in those with T-scores of ?1.5 or less with risk factors for fragility fractures. The major risk factors for fragility fractures are low bone density, history of fragility fractures, family history of osteoporosis, current smoking, and a body weight less than 127 lbs. Optimal calcium and vitamin D intake is necessary in all patients. Antiresorptive therapy is indicated in all patients with fragility fractures, irrespective of the bone mineral density. Alendronate is a bisphosphonate which improves bone mineral density by decreasing bone resorption. It has been associated with an increase in bone mineral density, particularly at the hip and lumbar spine, thereby significantly reducing the incidence of fragility fractures at these areas. Risedronate is another bisphosphonate which is approved for the treatment of osteoporosis. This agent is similar to alendronate, and its only difference is a slight decrease in its gastrointestinal side effects. This patient has a T-score of -2 and less at the hip and spine, respectively. regimen for her is calcium and vitamin D supplementation with alendronate. The best treatment

(Choice A) Because of the patient?s T-score, treatment with a bisphosphonate (alendronate or risedronate) is necessary. Supplementation of calcium and vitamin D alone would be suboptimal. (Choice C) Calcitonin also inhibits bone resorption, and is similar to bisphosphonates. It causes a mild improvement in vertebral bone mineral density, and reduces the incidence of vertebral fractures. It has a slight analgesic effect which may reduce the pain from a spinal fracture; however, it is a weak anti-resorptive agent, and improvement in bone density and fracture risk has not been not consistently documented. Because of the aforementioned, it is not the preferred first-line agent for osteoporosis. (Choice D) Subcutaneous injections of parathyroid hormone fragment (PTH 1-34, teriparatide) is very effective in increasing bone mineral density at the lumbar spine, and in significantly reducing the incidence of vertebral fractures. The problem lies with the drug being expensive, as well as its required daily injections. It is used in patients with a very high risk for fragility fractures, and possibly in patients who fail therapy with bisphosphonate. (Choice E) Hormone replacement therapy fell out of favor as the treatment for postmenopausal osteoporosis following the report of the Women?s Health Initiative. The report was published in 2002, and showed that patients on a combination of conjugated equine estrogen and medroxyprogesterone have a significantly higher risk of myocardial infarction, stroke, deep venous thrombosis, pulmonary embolism, and breast cancer. Short-term use of estrogen may not be harmful, and can still be used in postmenopausal patients with moderate to several hypoestrogenic symptoms.

Educational Objective: Bone mineral density in postmenopausal Caucasian women is classified (WHO criteria) according to the T-score (BMD score in comparison to young white females) as follows: normal (T score > 1.0) osteopenic (T score between ?1.0 and 2.5) osteoporosis (T score <-2.5) Optimal calcium and vitamin D intake is necessary in all patients. Antiresorptive therapy is indicated in all patients with fragility fractures, irrespective of the bone mineral density. A 62-year-old female collapsed at home after a period of shortness of breath and "foaming at the mouth." An advanced cardiac life support team arrived 30 minutes later, and found no respirations or pulse in the patient. The electrocardiogram revealed asystole. Atropine and epinephrine were then administered, after which, a pulse was restored. The patient was intubated and transferred to the emergency department. She was in a comatose state. Her blood pressure was 98/60 and core temperature was 34C (93.2F). Electrocardiogram revealed inferior Q waves, first-degree atrioventricular block, and right bundle branch block. Her daily medications included levothryoxine, lisinopril, lasix, amlodipine, digoxin, and metoprolol. Abnormal lab values included a TSH level of 67.38 uU/mL. Free T4 level was low at 0.4 ng/dl (normal 0.8 ? 1.8). The patient was admitted in the intensive care unit, and four hours later, she was still comatose, and had no motor response to firm supraorbital pressure. Her pupils were dilated and fixed at 8mm. Corneal, oculocephalic, gag, and cough reflexes were absent. Caloric testing produced no conjugate eye deviation. Seven hours after admission, a brain CT scan demonstrated loss of the gray/white junction, diffuse low density, loss of sulci, and obliterated ventricles. An apnea test at 10 hours after admission did not induce breathing. A second neurological examination again revealed coma, with no response to deep pain stimulation and absent brainstem reflexes. What is the most appropriate next step in the management of this patient? A. Watch for spontaneous movements of the limbs other than decorticate flexion or decerebrate extension. B. Make a clinical diagnosis of brain death and inform the patient's relatives. C. Obtain a technetium radionuclide brain perfusion scan. D. Document whether or not the patient still has deep tendon reflexes and a Babinski reflex. E. Wait six hours and repeat an apnea test. Explanation: This patient has suffered from severe ischemic-hypoxic encephalopathy secondary to cardiac arrest. She meets the three cardinal criteria for a clinical diagnosis of brain death, namely: 1. deep coma with unresponsiveness to deep central pain stimulation (i.e., supraorbital pressure, forceful twisting of nipples) 2. absence of brainstem reflexes 3. a positive apnea test, defined as no spontaneous ventilation in response to an increase in PaCO2 >= 20 mm Hg above baseline in the presence of adequate oxygenation The patient also fulfills the requirement for a final diagnosis of brain death of a second neurological examination, which should replicate the findings (i.e., absent deep central pain response and absent brainstem reflexes) of a neurological examination that was performed approximately six hours earlier; however, the patient does not meet all the prerequisites required for a definite clinical diagnosis of brain death. These prerequisites are: 1. Clinical or neuroimaging evidence of an acute CNS catastrophe that is compatible with brain death 2. Exclusion of complicating medical conditions, such as severe electrolyte, acid-base, or endocrine disorder 3. No drug intoxication or poisoning

4. Core temperature > 32C (90F) This patient failed the second prerequisite because she has evidence of significant hypothyroidism, which, although unlikely to account by itself for her comatose state, does not allow a definite diagnosis of brain death without the perfomance of a confirmatory test. A confirmatory test is required whenever a presumed braindead patient does not meet all the prerequisites or does not meet all of the three cardinal criteria. Confirmatory tests, in approximate order of decreasing sensitivity, include: 1. cerebral angiography (showing no intracerebral filling above the circle of Willis) 2. cortical EEG (no activity during at least 30 minutes) 3. transcranial Doppler ultrasound (showing small systolic peaks in early systole without diastolic or reverberating flow) 4. technetium-99m hexamethylpropyleneamineoxime brain scan (showing no isotope uptake in brain parenchyma) 5. absence of a somatosensory evoked potiential in response to median nerve stimulation This patient had a technetium brain perfusion scan, revealing absent intracranial blood flow, thereby confirming a clinical diagnosis of brain death. (Choice A) Limb movements may be occasionally observed due to spinal mechanisms in a patient who has lost all brain stem function. Such movements provide no reliable evidence for or against a clinical diagnosis of brain death. The same holds true for deep tendon reflexes and the Babinski reflex, which may or may not be observed for some time after brainstem functions have been irreversibly lost. (Choice B) This would be the correct next step if the patient had no simultaneous medical disorders confounding the diagnosis of brain death. Current medicolegal guidelines require performance of an additional confirmatory test to establish brain death if endocrine, metabolic, or toxic factors and/or hypothermia which could be contributing to coma are present in the patient. (Choice E) According to the American Academy of Neurology (AAN) guidelines and the diagnostic criteria for brain death, a second apnea test is not required to establish a clinical diagnosis of brain death after a first apnea test has been positive. Such repetition is optional. In contrast, a repeat neurological examination, which duplicates motor unresponsiveness to deep pain stimulation and absent brainstem reflexes, is recommended before venturing a clinical diagnosis of brain death. An interval of six hours between the first and second completely abnormal examinations is a reasonable standard; however, according to the AAN, this designation of six hours is currently "arbitrary?. Educational Objective: The three cardinal criteria for a clinical diagnosis of brain death are: 1. deep coma with unresponsiveness to deep central pain stimulation 2. absence of brainstem reflexes 3. a positive apnea test If the patient has any concurrent endocrine, metabolic, toxic, or hypothermic (core T < 32C) condition which could cause coma, an additional confirmatory test is necessary, but should be performed once the core T is > 32C. 9% of people answered this question correctly. A 22-year-old man presents to the university health clinic for a wellness exam. He appears quiet and reserved. When pressed, he informs the nurse that he has some nonspecific aches and pains. Once you enter the examination room, he admits to some concerns about his genitalia. He is not sexually active at this time and last had sexual intercourse eight months ago. He has been masturbating twice per day for the past few months. He worries that he recently acquired a disease through masturbation. Physical examination reveals an uncircumcised penis and multiple dome-shaped skincolored papules measuring 1-2 mm in diameter. The papules are located on the sulcus and corona of

the glans penis and are arranged circumferentially in a row. Which of the following is the most likely diagnosis? A. Condyloma acuminata B. Contact dermatitis C. Gonococcal urethritis D. Herpes simplex infection E. Normal variant Explanation: Pearly penile papules are considered a normal variant (Choice E), are not spread by sexual contact or activity, and harbor no malignant potential. These asymptomatic lesions are more common in uncircumcised males and are thought to occur in a substantial proportion of the male population. They typically appear as one or multiple rows of small, flesh-colored, dome-topped or filiform papules positioned circumferentially around the corona or sulcus of the glans penis. Treatment is not necessary. Condyloma acuminata (Choice A), or anogenital warts, is the most common viral sexually transmitted disease in the United States. The condition is caused by infection with human papilloma virus. Patients may be asymptomatic or may complain of pruritus, bleeding, burning, or tenderness. The lesions are typically skin-colored or pink and vary in appearance from smooth flattened papules to verrucous, papilliform growths. Contact dermatitis (Choice B) is an inflammation of the skin that results from exposure to specific exogenous agents (eg, nickel, fragrances, cleaning products). The irritated area is usually clearly demarcated and the lesions range from minimal skin erythema to significant edema, vesicles, and ulcers. Gonococcal urethritis (Choice C) typically causes urethral discharge and dysuria within 2-7 days of exposure to an infected partner. An increase in urinary urgency or frequency is inconsistently reported. The discharge may be scant at symptom onset but typically becomes profuse within the next day or two. In men, herpes simplex virus infection (Choice D) presents with herpetic vesicles on the glans penis, the prepuce, the shaft of the penis, and occasionally on the scrotum or thighs. The lesions may progress to pustules and painful ulcers that crust over after 4-15 days. Urethritis is a common complaint and is associated with severe dysuria and mucoid discharge. Educational Objective: Pearly penile papules are a normal variant and are not spread by sexual contact or activity. These asymptomatic papules are more common in uncircumcised males and typically appear as one or multiple rows of small, flesh-colored, dome-topped or filiform papules positioned circumferentially around the corona or sulcus of the glans penis. 44% of people answered this question correctly. A 19-year-old Caucasian woman without health insurance presents to the gynecologist?s office complaining of mild dysuria, nocturia, and increased urinary frequency. These symptoms began yesterday shortly after she had intercourse with her partner of three months. Past medical history is noncontributory. Current medications include oral contraceptives and she reports an allergy to sulfa. Temperature is 36.7C (98F), blood pressure is 124/78 mm Hg, pulse is 68/min, and respirations are 13/min. Mild suprapubic tenderness is apparent on palpation. Urinalysis is positive for leukocyte esterase and nitrites and demonstrates moderate pyuria. She is requesting the most cost effective treatment. What is the next best step in managing this woman?s condition? A. Prescribe phenazopyridine

B. Prescribe trimethoprim/sulfamethoxazole C. Prescribe trimethoprim D. Prescribe ciprofloxacin E. Prescribe metronidazole Explanation: Urinary tract infections (UTIs) are extremely common in women and may be characterized by dysuria, nocturia, hematuria, increased urinary frequency and urgency, and suprapubic tenderness. Urinalysis demonstrating pyuria and bacteriuria confirms the diagnosis. Sexual intercourse can increase the risk of UTI, as can wiping from posterior to anterior after a bowel movement. Escherichia coli causes over 80% of uncomplicated UTIs, with Staphylococcus saprophyticus causing another 5-15%, and Klebsiella and Proteus mirabilis accounting for the remainder. In a woman with limited financial resources and a sulfa allergy, three-day treatment with trimethoprim (Choice C) is appropriate. Phenazopyridine (Choice A) is a reasonably inexpensive medication that provides topical analgesia of the genitourinary tract. However, it is not an antimicrobial agent and will not cure the infection. Typically, UTIs are treated with a three-day course of trimethoprim/sulfamethoxazole (Choice B), which is contraindicated in a patient with sulfa allergy. Trimethoprim alone has a similar efficacy. Ciprofloxacin (Choice D) is a wide-spectrum antibiotic that is extremely effective in treating UTI. However, it is particularly expensive, and should therefore not be first-line treatment for an uninsured patient with an uncomplicated UTI. Metronidazole (Choice E) is commonly prescribed for the treatment of bacterial vaginosis in women, but is not indicated in this situation. Educational Objective: Uncomplicated UTIs are typically treated with a three-day course of trimethoprim-sulfamethoxazole. Cost-conscious patients with sulfa allergies can be given trimethoprim alone. 12% of people answered this question correctly. A 40-year-old African-American nulligravida is undergoing an evaluation after the cytologic detection of high-grade intraepithelial cervical lesion. The following colposcopy does not confirm the cytologic diagnosis, and you decide to proceed with a cone biopsy using the loop electrosurgical excision procedure (LEEP). Which of the following is the most likely short-term complication of this procedure? A. Bleeding B. Perforation C. Dyspareunia D. Adnexal mass E. Pelvic cellulitis Explanation: Bleeding is the most common short-term complication after cervical conization (cone biopsy), both cold knife conization and LEEP. The reported incidence of bleeding after LEEP is 2 to 10%. The causes of bleeding include inadequate intraoperative hemostasis and vasodilatation after the effect of the local vasoconstrictor wears off. Usually, the bleeding can be easily controlled by conservative measures. Surgical hemostasis is rarely required. (Choice B) Perforation is uncommon. (Choices D and E) Adnexal mass and pelvic cellulitis rarely develop after LEEP.

(Choice C) Dyspareunia may develop after gynecologic surgery or procedures, usually operations leading to distortions of vulvovaginal anatomy or fibrosis with decreased tissue mobility. Educational Objective: Bleeding is the most common short-term complication after cervical conization (cone biopsy). 70% of people answered this question correctly. A 32-year-old homosexual man presents to the emergency department with two days history of fever, chills, cough, and rusty sputum. He is known to be HIV-positive; his CD4 cell count was 600 cells/microL two months ago. He does not smoke or consume alcohol. He took several tablets of acetaminophen today to relieve his fever. He is sexually active with one partner, who is also HIVpositive. His partner was hospitalized one month ago because of pneumonia, and antiretroviral therapy was started. His blood pressure is 120/70 mmHg and heart rate is 90/min. Breath sounds are decreased over the right middle lobe. Chest x-ray shows right middle lobe infiltrate. WBC count is 13,000/cmm, with a left shift. Which of the following is the most likely cause of this patient?s condition? A. Pneumocystis carinii B. Mycoplasma C. S. pneumoniae D. Legionella E. Orthomyxovirus infection Explanation: This patient presents with community-acquired pneumonia that is most likely caused by a pyogenic organism (e.g. S. pneumoniae). The clinical picture of ?typical? pneumonia is described: acute onset of symptoms, purulent sputum, and characteristic physical findings. Although the patient is homosexual and HIV-positive, the diagnosis of PCP infection should not be assumed. The entire clinical picture must be considered. (Choices A, B, and E) ?Atypical? pneumonia can be caused by Pneumocystis carinii, Mycoplasma, Chlamydia, and viral infections. It is characterized by gradual onset, non-productive cough, exertional dyspnea and interstitial infiltrates on chest x-ray. (Choice D) Legionella infection causes community-acquired pneumonia that is commonly accompanied by extra-pulmonary symptoms such as diarrhea, nausea, vomiting, and neurologic findings (headache, confusion). Sputum analysis reveals many neutrophils, but very few, if any, organisms. Educational Objective: Diagnosis of PCP infection should not be assumed in HIV-positive patients. The entire clinical picture must be considered. 84% of people answered this question correctly. An 80-year-old Caucasian female is being discharged from the hospital after surgical repair of the right hip fracture. She fractured her right hip after falling down the steps in her home. She has a past history of osteoporosis, hypertension controlled with thiazide diuretics and metoprolol, diabetes mellitus treated with metformin, and hypercholesterolemia controlled with simvastatin. She suffered from an acute myocardial infarction six years ago, but she has not had any complications or recurrence of symptoms. She has a 50-year-old daughter who lives near her house. You advise the patient to consider an alternate living arrangement; however, she clearly expresses her desire to live by herself in her own home. She does not want to live in her daughter's house or in a nursing home. Which of the following is the most appropriate next step?

A. Inform her that nursing home is the only available option for her B. Ask her to live with her daughter for a few weeks C. Ask the patient to prepare advanced directives D. Get home assessment by a nurse E. Ask the daughter to convince her mother to stay with her Explanation: Falls are a common cause of morbidity and mortality in the elderly population. These are associated with injuries and fractures, significant decline in functional status, and subsequent greater chance of nursing home placement. Falls in the elderly population occur as a result of a mixture of intrinsic and extrinsic factors. The intrinsic factors include any acute illness or an age-related decline in balance, vision, ambulation, proprioception, cognitive impairment, musculoskeletal or cardiovascular function. Some of the extrinsic stressors that can lead to an increased risk of falls include the use of multiple medications, psychotropic medications, and an unsafe home environment. Prevention of recurrent falls and the associated complications is important in patients with a history of prior falls. Some interventions that have been proven to be useful in the prevention of falls include muscle strength and balance training, withdrawal of psychotropic medications, and home hazard evaluation and modification by a trained professional. Therefore, the patient in the above vignette should have a total assessment and modification of her home environment to reduce the risk of subsequent falls. (Choices A, B & E) The patient appears competent and does not wish to live in the nursing home or with her daughter. She should not be forced by the physician or her daughter to move to the nursing home or her daughter?s house. Educational Objective: Some interventions that have been proven to be useful in the prevention of falls in elderly patients include muscle strength and balance training, withdrawal of psychotropic medications, and home hazard evaluation and modification by a trained professional. 89% of people answered this question correctly. The following vignette applies to the next 3 items A concerned mother brings in her 4-week-old Caucasian son to the pediatrician. She says that over the past several days, he has started to "spit up a whole lot real forcefully, right after a feeding and then he?s hungry again!" She describes the vomitus as white milk with little curdling. The mother thinks he may be producing less stool and urine than normal. He has no history of fever, diarrhea, or symptoms suggestive of upper respiratory tract infection. The family has not traveled recently, and there have been no ill contacts. Physical examination shows a mildly dehydrated child in no apparent distress. His abdomen is slightly distended and there are active bowel sounds. No hepatosplenomegaly is present. Serum electrolytes Sodium: 133 mEq/L Potassium: 3.3 mEq/L Bicarbonate: 31 mEq/L Chloride: 90 mEq/L BUN: 20 mg/dL Creatinine: 1.2 mg/dL Item 1 of 3

Which imaging modality should be ordered to confirm the diagnosis? A. Barium enema B. Abdominal CT Scan C. Upper GI endoscopy D. Abdominal Radiograph E. Abdominal ultrasound Explanation: Classically, infantile hypertrophic pyloric stenosis (IHPS) presents in a male infant aged three to six weeks who develops postprandial projectile vomiting. There is usually no blood or bile in the vomit, and the child is immediately hungry after vomiting. Physical examination often reveals a palpable olive-shaped mass in the right upper quadrant of the abdomen, and peristaltic waves are sometimes seen traveling from left to right in the upper abdomen immediately before vomiting. Historically, laboratory evaluation will show hypokalemia and a hypochloremic, metabolic alkalosis secondary to the loss of gastric hydrochloric acid, although electrolyte imbalances are seen less often now that the diagnosis is made earlier. The imaging modality used most commonly to diagnose IHPS is ultrasound (Choice E), though an upper gastrointestinal (UGI) contrast study is preferred in some medical centers. Barium enema (Choice A) is often used to diagnose intussusception. An abdominal CT scan (Choice B) may be used to diagnose tumors, infections, or appendicitis in young children. Upper endoscopy (Choice C) may be used in infants with suspected IHPS when other imaging modalities do not clarify the situation. Abdominal radiograph (Choice D) is helpful in diagnosing conditions such as duodenal atresia or malrotation. Educational Objective: Ultrasound is the most commonly used imaging modality in the diagnosis of infantile hypertrophic pyloric stenosis, though an upper gastrointestinal (UGI) contrast study is preferred in some medical centers. 79% of people answered this question correctly.

Item 2 of 3 Once the diagnosis is confirmed, what is the next best step in the management of this child?s condition? A. Provision of continuous nasoduodenal feedings B. Correction of electrolyte imbalances C. Administration of atropine D. Immediate pyloromyotomy E. Referral for balloon dilation Explanation: Since this child with infantile hypertrophic pyloric stenosis is dehydrated and suffering from a hypokalemic, hypochloremic metabolic alkalosis, his dehydration and electrolyte derangements should be corrected (Choice B) before proceeding to surgery. Studies have shown that children who undergo surgery without first correcting the electrolyte imbalances are at increased risk for postoperative apnea. The provision of continuous nasoduodenal feedings (Choice A) for a lengthy period is a conservative treatment that can be used in infants who are not good surgical candidates. As the infant gains weight, the pyloric stenosis is thought to pose less of an impediment to the digestive process. The oral or intravenous administration of atropine (Choice C) may be used to relax the pyloric musculature. Although rarely used, this is another conservative alternative for infants who are not good surgical candidates. If the child diagnosed with infantile hypertrophic pyloric stenosis is well hydrated and has normal electrolytes, pyloromyotomy (Choice D) may be performed immediately. Endoscopically guided balloon dilation (Choice E) has been studied, but is not often used because of inconsistent success in opening the muscular ring of the pylorus. This approach is typically reserved for patients who are not otherwise good surgical candidates. Educational Objective: Electrolyte derangements and dehydration must be corrected before proceeding with surgical correction of infantile hypertrophic pyloric stenosis. 65% of people answered this question correctly. Item 3 of 3 The mother of this boy mentions that he was given an antibiotic as prophylaxis after an outbreak of a particular respiratory illness. Which of the following antibiotics is associated with the development of his condition? A. Ampicillin B. Cephalexin C. Ciprofloxacin D. Erythromycin E. Tetracycline Explanation: Studies have documented an association between the development of infantile hypertrophic pyloric stenosis and the usage of oral erythromycin (Choice D), which is typically given as postexposure prophylaxis for pertussis. In addition, there is some indication that the usage of macrolides in breastfeeding women is linked to the development of infantile hypertrophic pyloric stenosis, especially in infant girls.

The usage of other antibiotics (Choices A, B, C, and E) is not linked with infantile hypertrophic pyloric stenosis. Educational Objective: The usage of erythromycin is associated with the development of infantile hypertrophic pyloric stenosis. A 37-year-old woman has had left breast discomfort for several months. The breast has also been painful before menses. Yesterday, she noted a lump. Her past medical history and family history are unremarkable for breast and gynecologic diseases. She had a baseline mammogram at the age of 36 that was normal. A smooth, soft, movable mass is palpated in the upper outer quadrant of the left breast. Some diffuse nodularity is present bilaterally. The vital signs and the rest of the exam are normal. Fine-needle aspiration (FNA) of the mass reveals a thin, greenish fluid. The mass disappeared completely after FNA. What should the patient's follow-up therapy include? A. Mammogram B. Oral contraceptives C. Observation D. Ultrasound E. Antibiotics Explanation: Cysts that disappear completely, contain non-bloody fluid, and possess a characteristic diffuse nodularity, are most likely due to fibrocystic disease. Careful observation for recurrence of the mass is the standard follow-up therapy. (Choice A) A mammogram would be indicated if the mass didn?t disappear completely, or if its content was bloody. (Choice D) An ultrasound is indicated if the mass cannot be visualized using a mammogram because of dense, normal breast tissue (seen especially in young women). Ultrasound is also useful for the evaluation of cystic lesions, but this cyst has already been drained. (Choice B) Oral contraceptives are sometimes given to control mastodynia; however, for this patient, the mastodynia is expected to resolve after FNA. (Choice E) The patient doesn?t have signs of infection. Thick, not thin greenish fluid, could be pus. Antibiotics are not necessary. Educational Objective: A breast mass that produces non-bloody aspirate and disappears completely on aspiration, does not need any further evaluation other than observation. 64% of people answered this question correctly. The following vignette applies to the next 2 items A 60-year-old Caucasian woman is intubated in the intensive care unit for respiratory failure. She has a history of chronic asthma and is being treated for an acute exacerbation. On her fourth day of hospitalization, she develops a fever. The chest x-ray is suggestive of new infiltrate in the right lung. The physical examination reveals a well-circumscribed erythematous patch measuring 10 cm x 8 cm on the lower abdomen. One day later, a hemorrhagic bulla develops in the center of this patch, and within hours the bulla evolves into a black necrotic ulcer with an erythematous rim. The ulcer exudes a yellow-green, purulent fluid sticky to the touch. The most recent vital signs include a temperature

of 38.9C (102F), blood pressure of 124/76, pulse of 90/min, and respirations of 21/min. An urgent Gram stain of the fluid is positive for gram-negative rods, and a sample is obtained for culture. Item 1 of 2 Given the clinical presentation, what is the most likely diagnosis? A. Cellulitis B. Impetigo C. Erythema migrans D. Pyoderma gangrenosum E. Ecthyma gangrenosum

Explanation: Pseudomonas aeruginosa is a gram-negative aerobic bacillus commonly responsible for severe nosocomial infections, especially in the immunocompromised. In immunocompetent patients, P. aeruginosa bacteremia is most often seen in patients with indwelling urinary or central venous catheters, or following infection of traumatic or surgical wounds. Patients with P. aeruginosa bacteremia frequently develop ecthyma gangrenosum. This condition is caused by perivascular bacterial invasion of the media and adventitia of arteries and veins, followed by ischemic necrosis. Characteristic findings are lesion(s) of the skin or mucous membrane that rapidly worsen and evolve into nodular patches marked by hemorrhage, ulceration, and necrosis. Although ecthyma gangrenosum is not pathognomonic of P. aeruginosa, its presence increases the likelihood of Pseudomonas as the causative agent. Of particular relevance in this case is the observation that mortality is higher in patients with Pseudomonas bacteremia and concurrent pulmonary infection. (Choice A) Cellulitis is an inflammation of the skin most commonly caused by an infection with Streptococcus pyogenes or Staphylococcus aureus. (Choice B) Impetigo is a highly contagious, pyogenic skin infection most commonly found in children. Bullous impetigo is primarily caused by Staphylococcus aureus, while nonbullous impetigo is caused by Staphylococcus aureus, Streptococcus pyogenes, or a combination of the two. (Choice C) Found in patients with Lyme disease, erythema migrans is an erythematous lesion that gradually increases in size over a few days. Erythema migrans frequently occurs at or near the site of the tick bite. (Choice D) Pyoderma gangrenosum occurs in two forms, classical and atypical. Classical pyoderma gangrenosum is most commonly found on the legs and is characterized by deep ulcers with violaceous borders. Lesions of atypical pyoderma gangrenosum appear on the hands, arms, or face and are superficial ulcerations that have vesiculopustular borders. Educational Objective: Ecthyma gangrenosum is associated with lesion(s) of the skin or mucous membrane that rapidly worsen and evolve into nodular patches marked by hemorrhage, ulceration, and necrosis. Although ecthyma gangrenosum is not pathognomonic of infection with P. aeruginosa, its presence increases the likelihood of Pseudomonas as the causative agent. 35% of people answered this question correctly. Item 2 of 2 The correct diagnosis is made. Which of the following treatments is most appropriate given this woman?s condition? A. Oral antibiotic B. Intravenous antibiotics C. Intravenous antibiotics and surgical debridement D. Surgical debridement E. Intravenous steroids Explanation: Pseudomonas aeruginosa bacteremia is usually treated with a two-drug regimen to limit the emergence of antibiotic resistance, although some experts recommend monotherapy with ceftazidime or ciprofloxacin when susceptibility data are available. Empiric treatment is frequently employed because the timely administration of antibiotics has a significant impact on cure rate; in one study, delays of 1-2 days reduced the cure rate from 74 to 46% in these patients. Popular two-drug regimens include an aminoglycoside (e.g., tobramycin or amikacin) and an extended-spectrum antipseudomonal penicillin (e.g., piperacillin) or an antipseudomonal cephalosporin (e.g., ceftazidime

or cefepime). In neutropenic patients, therapy is continued for 14 days or longer until the neutrophil count returns to baseline. In patients with bacteremia secondary to catheter infection, therapy should be stopped after 7-10 days once the catheter has been removed and the patient has significantly improved. (Choice A) Intravenous antibiotics are preferred to oral antibiotics in particularly ill patients because of their ease of administration and rapidity of distribution. Moreover, some of the newer or more effective antibiotics are available only in parenteral form. (Choices C and D) Surgical debridement is not indicated in the treatment of ecthyma gangrenosum. (Choice E) Intravenous steroids would not be an appropriate means of treating this patient?s Pseudomonas infection, and could in fact worsen her condition by suppressing her immune response. Educational Objective: Pseudomonas aeruginosa bacteremia is usually treated with a two-drug regimen to limit the emergence of antibiotic resistance. Such regimens typically include an aminoglycoside (e.g., tobramycin or amikacin) and an extended-spectrum antipseudomonal penicillin (e.g., piperacillin) or an antipseudomonal cephalosporin (e.g., ceftazidime or cefepime). Surgical debridement is not indicated in the treatment of ecthyma gangrenosum. 16% of people answered this question correctly. A 45-year-old Caucasian woman is brought to the emergency department by her daughter. She has been agitated since yesterday, and is complaining of muscular cramps, profuse sweating, and fever. She does not use alcohol or illicit drugs, but has smoked one pack of cigarettes daily for the past 25 years. She has depression, and has been taking tranylcypromine for the past nine months. She was started on fluoxetine one month ago. Her temperature is 38.3?C (101.0?F), blood pressure is 120/80 mmHg, pulse is 116/min and respirations are 24/min. Examination reveals a diaphoretic and agitated patient. Oropharyngeal, cardiac and pulmonary exams reveal no abnormalities. Her abdomen is soft, non-tender, and non-distended. Deep tendon reflexes are increased. Muscular strength is preserved, but there is some muscular rigidity. The muscular tone is increased. Which of the following is the most probable diagnosis? A. Malignant neuroleptic syndrome B. Viral meningitis C. Serotonin syndrome D. Tetanus E. Acute psychotic reaction Explanation: When monoamine oxidase inhibitors (MAOIs; e.g., tranylcypromine and phenelzine) are used with selective serotonin reuptake inhibitors (SSRIs; e.g., fluoxetine, paroxetine, sertraline, citalopram, etc.) there is a possibility of an excessive increase in serotonin levels in the brain, leading to "serotonin syndrome". This syndrome is characterized by hyperthermia, tachycardia, diaphoresis, agitation and muscular rigidity. The patient is taking an SSRI (fluoxetine) and a MAOI (tranylcypromine), and presents with the characteristic signs and symptoms. The possibility of serotonin syndrome is extremely high. (Choice A) The presentation of malignant neuroleptic syndrome is similar, but the patient has no history of taking any neuroleptics. (Choice E) Psychosis will not explain the profound alteration of vital signs and neuromuscular changes.

(Choice D) Tetanus usually presents with stiff neck, opisthotonus, risus sardonicus (sardonic smile), a board-like rigid abdomen, periods of apnea, and dysphagia. (Choice B) Serotonin syndrome may be mimicked by central nervous system (CNS) infections; however, there is no evidence of infection in this patient. Her fever can be explained by multiple causes. A work-up to rule out an infection may be necessary, but is not a priority at this point, because the most probable diagnosis is serotonin syndrome.

Educational Objective: SSRIs and MAOIs should not be taken together because of the high risk of "serotonin syndrome", which is a potentially lethal condition that is characterized by fever, diaphoresis, tachycardia, agitation and muscular rigidity. 69% of people answered this question correctly. A 45-year-old Caucasian woman is brought to the emergency department by her daughter. She has been agitated since yesterday, and is complaining of muscular cramps, profuse sweating, and fever. She does not use alcohol or illicit drugs, but has smoked one pack of cigarettes daily for the past 25 years. She has depression, and has been taking tranylcypromine for the past nine months. She was started on fluoxetine one month ago. Her temperature is 38.3?C (101.0?F), blood pressure is 120/80 mmHg, pulse is 116/min and respirations are 24/min. Examination reveals a diaphoretic and agitated patient. Oropharyngeal, cardiac and pulmonary exams reveal no abnormalities. Her abdomen is soft, non-tender, and non-distended. Deep tendon reflexes are increased. Muscular strength is preserved, but there is some muscular rigidity. The muscular tone is increased. Which of the following is the most probable diagnosis? A. Malignant neuroleptic syndrome B. Viral meningitis C. Serotonin syndrome D. Tetanus E. Acute psychotic reaction Explanation: When monoamine oxidase inhibitors (MAOIs; e.g., tranylcypromine and phenelzine) are used with selective serotonin reuptake inhibitors (SSRIs; e.g., fluoxetine, paroxetine, sertraline, citalopram, etc.) there is a possibility of an excessive increase in serotonin levels in the brain, leading to "serotonin syndrome". This syndrome is characterized by hyperthermia, tachycardia, diaphoresis, agitation and muscular rigidity. The patient is taking an SSRI (fluoxetine) and a MAOI (tranylcypromine), and presents with the characteristic signs and symptoms. The possibility of serotonin syndrome is extremely high. (Choice A) The presentation of malignant neuroleptic syndrome is similar, but the patient has no history of taking any neuroleptics. (Choice E) Psychosis will not explain the profound alteration of vital signs and neuromuscular changes. (Choice D) Tetanus usually presents with stiff neck, opisthotonus, risus sardonicus (sardonic smile), a board-like rigid abdomen, periods of apnea, and dysphagia. (Choice B) Serotonin syndrome may be mimicked by central nervous system (CNS) infections; however, there is no evidence of infection in this patient. Her fever can be explained by multiple causes. A work-up to rule out an infection may be necessary, but is not a priority at this point, because the most probable diagnosis is serotonin syndrome. Educational Objective: SSRIs and MAOIs should not be taken together because of the high risk of "serotonin syndrome", which is a potentially lethal condition that is characterized by fever, diaphoresis, tachycardia, agitation and muscular rigidity. 69% of people answered this question correctly.

A healthy, 26-year-old Caucasian woman comes to the clinic with her husband for a routine health maintenance examination. You are her primary care physician. She is currently pregnant and is also being regularly seen by a hematologist because of a probable Rh sensitization. She is Rh negative, and her husband is Rh positive. Her Coomb?s test is currently positive. She discloses to you that she has been pregnant in the past from another man and she had an abortion. Her husband does not know this, and she does not want it to be revealed to him. A friend of her husband happens to be a physician, and he got some medical information about cases similar to his wife?s condition. Her husband asks you how this is possible considering that this is their first child. Which of the following is the best answer? A. Tell him that he may need to talk to his wife about this. B. Explain to him that sensitization can sometimes occur during the first pregnancy due to unclear reasons, and this is currently a matter of research. C. Tell him that this happened because his wife had conceived in the past. D. Refer him to the hematologist for a better explanation. E. Tell him that more tests need to be done to address his question. Explanation: Patient confidentiality is paramount in medical practice and has recently been enforced by the new HIPAA (Health Insurance Portability and Accountability Act) regulations. In this case, the best thing to do is to tell the husband to talk to his wife about that particular topic. (Choice C) If the husband?s health or safety is not jeopardized by his wife?s condition, the physician has no obligation to disclose any confidential information about her to him. (Choices B, D and E) The physician must not give false information or refer the patient to another physician to address this delicate topic, as he or she is the primary care physician. Educational Objective: A patient?s confidentiality is paramount in medical practice. It should be honored in most circumstances, unless a situation jeopardizes the third party?s (spouse or relative) health or safety (e.g. homicidal intentions, HIV, or STD infection). If these cases occur, and the patient is mentally competent, the physician should encourage her to disclose the truth to her partner before he/she is obliged to do it him/herself. 61% of people answered this question correctly. A 35-year-old Caucasian female presents to your office with a self-palpated breast mass. After the appropriate work-up, you decide to proceed with fine-needle aspiration (FNA). The results of FNA came back as negative, but the patient is still concerned about the possibility of the breast cancer. You reply that the probability of the breast cancer is low in her case because FNA has a high: A. Sensitivity B. Specificity C. Positive predictive value D. Negative predictive value E. Validity Explanation: In this scenario, the physician is talking about negative predictive value (NPV). NPV is the probability of being free of a disease if the test results are negative. NPV depends on the pretest probability of a disease. For example, if the probability of the breast cancer is high after initial work-up in this patient, and the FNA result is negative, then the NPV is low. On the other hand, if the probability of the breast cancer is low after initial work-up, and the FNA result is negative, then the NPV is high; that is, the probability that the patient is free of disease is high. Another good example is as follows: the NPV is

higher in a woman with a negative HIV test who belongs to the low-risk group than in a woman with a negative HIV test who belongs to the high-risk group, because the pre-test probability of having HIV is higher in the latter. (Choices A and B) NPVdepends on the pretest probability of a disease, unlike sensitivity and specificity (which are fixed values). (Choice C) Positive predictive value follows the same concept, but applies if the test returns positive. (Choice E) Validity represents the appropriateness of the test (a test measures what it is supposed to measure); it does not depend on the pretest probability of the disease. Educational Objective: NPV is the probability of being free of a disease if the test returns negative. It is high if the pre-test probability of the disease is low. 30% of people answered this question correctly. Review of a patient?s clinical course reveals that at approximately 2:00 pm, the patient developed an abrupt temperature elevation to 39 C (102.2 F) and a shaking chill. Which of the following times would be the best to draw blood cultures from this patient? A. During the febrile period B. One hour before the anticipated temperature elevation and chill C. One hour after the return of the temperature to normal D. At random E. In the morning Explanation: Systemic manifestations of bacteremia, which include abrupt temperature elevations and chills, usually occur 30-90 minutes after bacteria enter the blood stream. Because the circulating phagocytes usually are quite effective in removing these bacteria, blood cultures drawn during the chill are frequently negative. If there is a time pattern to the chill and fever pattern, the likelihood of obtaining a positive blood culture increases if it is obtained approximately one hour before the anticipated fever. Extremely high fevers should be aggressively treated, as one may develop heat stroke, delirium, or convulsions. Initially, cooling blankets and antipyretics are often used. Educational Objective: If possible, blood cultures should be obtained one hour prior to the spiking of a temperature. 26% of people answered this question correctly. A 56-year-old Caucasian male is brought to the emergency department because of a 7-hour history of weakness of the left extremities. His past medical history is significant for hypertension and diabetes mellitus, type 2. He smokes 1 ? packs a day, and consumes alcohol occasionally. His current medications include hydrochlorothiazide, enalapril and glyburide. His blood pressure is 190/110 mmHg and heart rate is 90/min. Physical examination reveals left hemiplegia with hemi-sensory loss; no meningeal signs are present. ECG shows left ventricular hypertrophy with secondary repolarization changes. CT scan of the head reveals no blood, but a locus of brain swelling in the right hemisphere is present. Which of the following is the best next step in the management of this patient? A. Proceed with cerebral angiogram B. Administer mannitol C. Administer IV corticosteroids

D. Consider t-PA E. Hypotensive agents are not indicated

Explanation: Hypertension during an acute ischemic stroke should not be treated unless it is very severe (systolic blood pressure > 220 mmHg, diastolic blood pressure > 120 mmHg) or causes end-organ damage (pulmonary edema, unstable angina, etc). Most authors consider the elevated blood pressure during an ischemic stroke to be a protective reaction that is intended to preserve the circulation in the underperfused areas of the brain. Several studies demonstrated that decreasing the blood pressure in the setting of an acute ischemic stoke is associated with clinical deterioration and a worsened prognosis. (Choices B and C) Local brain swelling is present in up to 40% of the patients with acute ischemic stroke, and is associated with a worse prognosis; mannitol and IV corticosteroids are not indicated in this setting. (Choice D) Thrombolytic agents are generally considered in patients who presents within three hours from the onset of the symptoms. Educational Objective: Hypertension during an acute ischemic stroke should not be treated unless it is very severe or causes end-organ damage. 20% of people answered this question correctly. A 35-year-old African-American marathon runner presents to the gynecologist complaining of secondary amenorrhea that developed three months ago. Her cycles are normally 28 days long, and her menses last three to five days with moderate flow. One year ago, the woman adopted a vigorous exercise regimen that lasted between three and five hours every day. Since then, her BMI has declined from 23.4 to 16.5. She has been winning many local races and is considering increasing the difficulty of her exercise regimen, but would like to address the issue of her amenorrhea first. Physical examination reveals a thin woman with well-defined musculature but is otherwise unremarkable. Pregnancy test is negative. What is the most likely etiology of her amenorrhea? A. Kwashiorkor B. Testosterone deficiency C. Estrogen deficiency D. Progesterone deficiency E. Prolactin excess Explanation: Amenorrhea is thought to occur in female athletes when there is a relative caloric deficiency secondary to inadequate nutritional intake as compared to the amount of energy expended. Women athletes with this condition have been shown to have decreased levels of luteinizing hormone (LH) and gonadotropin-releasing hormone (GnRH), resulting in an estrogen deficiency (Choice C). These amenorrheic women are therefore at increased risk for all conditions associated with estrogen deficiency, including infertility, vaginal atrophy, breast atrophy, and osteopenia. Kwashiorkor (Choice A) is a malnutrition disease caused by severe protein deficiency. This condition primarily occurs in children upon weaning from the breast, and is not the cause of amenorrhea in this woman. Testosterone deficiency (Choice B) occurs in disorders such as Klinefelter?s syndrome and cryptorchidism. It is not the cause of amenorrhea in this woman. Progesterone (Choice D) is an important hormone in the middle to late luteal phase of the menstrual cycle and also serves in the maintenance of pregnancy. A deficiency in this hormone is not the cause of amenorrhea in this woman.

High serum levels of prolactin (Choice E) can occur in pregnant or breastfeeding women or as the result of a prolactinoma. It is an extremely unlikely cause of amenorrhea in this woman. Educational Objective: Secondary amenorrhea is relatively common in elite female athletes and results from estrogen deficiency. 67% of people answered this question correctly. A 22-year-old Caucasian woman presents to your office complaining of progressive and bilateral visual problems. Detailed questioning revealed that other family members also have this problem. You draw a diagram showing the following pattern. What is the most probable mode of inheritance of the disorder that is present in this family? A. Autosomal dominant B. Autosomal recessive C. X-linked dominant D. X-linked recessive E. Mitochondrial Explanation: The pattern of transmission is consistent with a mitochondrial disease, which is characterized by a mother-to-offspring inheritance. Males can acquire the disease from their mothers, but do not transmit it further. In the diagram given above, it is clear that the grandmother of the patient had the disease and transmitted it to all her children, but only the daughters transmitted the disease further. This patttern, along with the history, makes Leber hereditary optic neuropathy (LHON) the most probable diagnosis in this case. LHON is a bilateral optic atrophy occurring at age 15-30. Other mitochondrial diseases include mitochondrial encephalopathies and myopathies (e.g., MELAS). (Choices A, B, C and D) Such a pattern is not consistent with either autosomal or X-linked inheritance. Educational Objective: Mitochondrial diseases are characterized by a mother-to-offspring inheritance pattern. acquire the disease from their mothers, but do not transmit it further. 47% of people answered this question correctly. A 23-year-old African American male who presents with fatigue and progressive dyspnea is hospitalized. Physical examination shows pallor. There is no jaundice. The lungs are clear on auscultation. The abdominal examination reveals no abnormalities. His initial lab test results are as follows: CBC Hb: 7.5g/dL MCV: 82 fl Platelet count: 180,000/cmm Leukocyte count: 7,500/cmm Packed red blood cell transfusion is started. He becomes agitated and complains of severe bilateral flank pain while undergoing blood transfusion. He has received several blood transfusions in the past, but all of them were uncomplicated. His temperature is 38.3C(101F), blood pressure is 80/60 mm Hg and pulse rate is 120/min. Bilateral wheezing is heard on lung auscultation. His urine is dark. Which of the following is the best next step in the management of this patient? A. Stop transfusion and hydrate with IV normal saline B. Stop transfusion and give IV methyl prednisolone C. Stop transfusion and give IV furosemide Males can

D. Stop transfusion and administer IV antibiotics E. Continue transfusion and give supplemental oxygen Explanation: Acute hemolytic transfusion reaction is a medical emergency that is usually caused by ABO incompatibility. Patients who require chronic red cell transfusions (e.g., African-American patients with sickle cell anemia) may form multiple antibodies to common Rh, Kell, or other blood group antigens. The presence of such antibodies may increase the time required for a transfusion service to supply serologically compatible red cells. If undetected, these red cell antibodies may cause hemolysis, but severe hemolytic reactions are uncommon. In this case, the transfusion should be stopped immediately, and vigorous hydration should be started to treat the hypotension and to prevent renal failure. Low dose dopamine infusion and osmotic diuresis may also be employed. (Choice B) IV corticosteroids are used to treat allergic reactions. (Choice D) IV antibiotics are used in patients with septic reaction. (Choice C) Loop diuretics are commonly administered to treat volume overload. (Choice E) Continuing the transfusion may cause further complications such as hemolysis. Educational Objective: Patients with acute hemolytic transfusion reaction should be managed by immediately stopping the transfusion and vigorously hydrating the patient with normal saline (not Ringer's or dextrose) to treat the hypotension and to prevent renal failure. A 26-year-old female comes to your office for a routine visit. She has a history of papillary thyroid cancer for which a near total thyroidectomy was performed without complications three years ago. She received an ablative dose of radioactive iodine following her thyroid surgery. She is currently on 125 mg of levothyroxine everyday, which she takes on an empty stomach every morning. She denies any symptoms. Physical examination is unremarkable. Laboratory investigations show undetectable thyroglobulin, TSH of 5.9 mU/mL (normal 0.35-5.0 mU/mL), and free T4 of 1.3 ng/dL (normal 0.8-1.8 ng/dL). What is the next best step in the management of this patient? A. Increase the dose of levothyroxine to suppress the TSH below 0.35 mU/mL B. Continue the same dose of levothyroxine C. Add liothyronine (T3) D. Increase the dose of levothyroxine to bring the TSH to within normal range E. Ask her to take levothyroxine after meals Explanation: The patient has a history of papillary thyroid cancer, which appears to be in remission. In patients with thyroid cancer in remission, the dose of levothyroxine is adjusted to suppress the TSH below normal range, usually between 0.1 and 0.3 mU/mL. The dose of levothyroxine for this patient should therefore be increased to bring the TSH to within the goal range. For patients with distant metastasis, even lower levels of TSH (complete suppression) are required. It is also important to remember that patients being treated with suppressive doses of levothyroxine are at an increased risk for bone loss and atrial fibrillation. (Choice C) T3 is not warranted for chronic treatment of hypothyroidism. T3 can be used for a short period to prevent symptoms of hypothyroidism in preparation for total radioactive iodine uptake and scan in patients with thyroid cancer.

(Choice E). The correct way to administer levothyroxine is to take it on an empty stomach and separately from other medications. Taking levothyroxine with meals or other medication could significantly impair its absorption. The patient is taking levothyroxine correctly. Educational Objective: In patients with thyroid cancer in remission, the dose of levothyroxine is adjusted to suppress the TSH below normal range, usually between 0.1 and 0.3 mU/mL. 18% of people answered this question correctly.

A healthy 27-year-old Caucasian man is brought to the emergency department after being involved in a motor vehicle accident. He complains of right knee pain and swelling. He was sitting in the front seat without a seat belt. His knees hit the front dashboard. His vital signs are stable. On examination, there is mild swelling and stiffness. Which of the following was most likely injured in this patient's knee joint? A. Anterior cruciate ligament B. Posterior cruciate ligament C. Medial meniscus D. Medial collateral ligament E. Lateral collateral ligament Explanation: This patient has a classic ?dashboard? injury, wherein a posteriorly directed force is placed on the anterior aspect of the proximal tibia, with the knee in a flexed position. This results in disruption of the posterior cruciate ligament (PCL). A similar injury can be seen in an athlete who falls on a flexed knee with the foot in plantar flexion. Unlike patients with ACL injury, these patients do not complain of a typical ?popping? sound, and may not have an unstable knee. (Choice A) ACL injuries are common in athletic activities. An isolated injury may occur after hyperextension of the knee. Up to 33% to 90% of patients with an ACL tear experience a ?popping? sensation. The rapid onset of a knee effusion and an unstable knee are common. Lachman's test is a very sensitive physical test for acute anterior cruciate ligament tear. It is done with the patient's knee flexed at 20 degrees; one hand pulls the proximal tibia, while the other hand stabilizes the femur. (Choices C and D) Meniscal injuries principally occur after a twisting injury to the knee with one foot fixed to the ground (e.g. with sudden turning while running). The medial meniscus is more commonly injured, as compared to the lateral meniscus. A bucket handle tear is the most common type of medial meniscus tear. Patients generally complain of pain and swelling of the knee, and a popping sensation at the time of injury. In medial meniscus injuries, tenderness is generally felt along the medial side of the knee. Effusion with meniscus injuries takes about 24 hours to form, and is often not significantly bloody, unlike ACL or osteochondral injuries. Locking of the knee joint on extension is generally seen. McMurray?s sign is snapping felt with tibial torsion and the knee flexed at 90 degrees. (Choices E and F) Medial and lateral collateral ligaments are injured when the line of force strikes from the side of the joint. An abduction injury with a torsional component causes a medial collateral ligament tear in most of the instances. On examination, the knee joint is swollen due to the effusion, with tenderness over the medial aspect of the knee. As the medial collateral ligament resists valgus angulations at the knee, injury to this ligament leads to increased angulation of the affected knee on valgus stress. A valgus stress test is generally performed with the knee flexed at 20-30 degrees. Educational Objective: The classic dashboard injury results from a posteriorly directed force on the anterior aspect of the proximal tibia with the knee in a flexed position. This results in disruption of the posterior cruciate ligament (PCL). 47% of people answered this question correctly. A healthy 43-year-old Caucasian man comes to the physician for a routine health maintenance examination. He has no complaints. He has no medical problems. He has smoked one pack of cigarettes daily for twenty years. He drinks 1-2 beers daily. His father was diagnosed with colon cancer at the age of 63. His vital signs are within normal limits. Examination shows no abnormalities. Which of the following is the most appropriate screening test for this patient? A. Perform colonoscopy now and repeat in 10 years. B. Perform colonoscopy at the age of 50 and repeat in 10 years.

C. Perform colonoscopy now and repeat in 5 years. D. Obtain genetic testing. E. Perform sigmoidoscopy now and repeat in 10 years. Explanation: The above patient has a high risk of colorectal cancer (based on his family history). All such patients should be screened earlier. The age at which screening for colorectal cancer should be started is 50 for the average-risk patient, and 40 for the high-risk patient. There are various options for colorectal cancer screening. These include fecal occult blood testing (FOBT), sigmoidoscopy, colonoscopy, FOBT + sigmoidoscopy, and double contrast barium enema. Colonoscopy is the recommended screening test by the American College of Gastroenterologists. A normal colonoscopy result is followed by a repeat colonoscopy after ten years. (Choice E) Sigmoidoscopy may also be used for screening purposes, but it should be repeated after five years. (Choice D) Genetic testing is indicated in patients with a family history of FAP or HNPCC. Educational Objective: Patients with a history of colorectal cancer in first-generation relatives should start having screening colonoscopy at 40 years of age, and screening should be repeated every 10 years. 23% of people answered this question correctly. A 42-year-old Caucasian woman comes to the office for a routine follow-up. She was diagnosed with multiple sclerosis three months ago, and has had no relapses ever since. She refused to take medications before, but she is now willing to do so. She has no other medical problems. She does not use drugs, alcohol or tobacco. Examination shows no abnormalities. There is a new treatment A available that is apparently highly effective to prevent multiple sclerosis relapses. A medicine journal shows that treatment A reduced the incidence of relapses to 20% after 6 months of treatment. Patients managed with placebo had an incidence of 25% during the same time period. Considering this information, how many patients will need to be treated in order to prevent one relapse during the first six months of therapy? A. Four patients B. Eight patients C. Twenty patients D. Ten patients E. Five patients Explanation: The number needed to treat (NNT) is a useful measure to evaluate the efficacy of a therapy and the risk of adverse events, which can sometimes outweigh the benefits. In order to calculate the NNT, the relative risk reduction (RRR) and absolute risk reduction (ARR) must first be determined. The relative risk reduction (RRR) values are 25% (0.25) for placebo and 20% (0.20) for treatment A. The ARR can be obtained by subtracting the RRR of the treatment group from the placebo group: 0.25-0.20=0.05 (or 5% of ARR). This means that for every 100 patients that used Treatment A, 5 more patients showed improvement. The NNT is equal to 1/ARR. 1/0.05=20. This means that 20 patients need to be treated in order to prevent one multiple sclerosis (MS) relapse during the first six months of therapy. Educational Objective:

The NNT is the number of patients that need to be treated in order to prevent or cure one disease or medical condition. NNT is a useful measure to evaluate the efficacy of a given therapy. It is calculated by getting the inverse of the ARR (1/ARR). The ARR is calculated by subtracting the relative risk reduction (RRR) of the therapy group from that of the placebo group. 50% of people answered this question correctly.

The following vignette applies to the next 2 items A 6-year-old African American boy presents to the emergency department complaining of abdominal pain. He has a history of sickle cell disease and experienced an episode of sickle cell crisis six months ago. His medical history is otherwise unremarkable. Physical examination shows a thin boy in apparent distress. He is clutching his abdomen, whimpering, and lying uncomfortably on the examination table. His vital signs include a temperature of 38C (100.4F), blood pressure of 110/70 mm Hg, pulse of 92/min, and respirations of 24/min. His tympanic membranes are pearly gray and the light reflex is present. No exudates or lesions are visible in the oropharynx. Chest auscultation is unremarkable and heart sounds are normal. He does not allow you to palpate his abdomen. Item 1 of 2 What is the next best step in the management of this patient? A. Intravenous fluids B. Ibuprofen PO for pain relief C. Ketorolac IV for pain relief D. Urinalysis and 24-hour measurement of urinary output E. Peripheral blood smear to evaluate for sickled cells Explanation: The sickle cell crisis is an episode of acute pain that can range in severity from minimal to agonizing. It is the most common type of vasocclusive event in patients with sickle cell disease. Although most of these crises have no identifiable cause, events such as dehydration, cold, infection, stress, menses, alcohol consumption, and nocturnal hypoxemia are known triggers. Some studies have also shown hemoglobin >8.5 g/dL to be a major risk factor for the development of an acute painful episode. Various areas of the body can be affected, with the most common locations including the back, chest, extremities, and abdomen. Objective clinical signs such as fever or tachypnea are seen approximately half the time. The first step in management of patients (especially children) presenting with an acute painful episode is the administration of fluids orally or intravenously to ensure optimal hydration (Choice A). Ibuprofen (Choice B) is not typically administered to patients in sickle cell crisis, as it does not provide adequate pain relief. Intravenous morphine is the most commonly prescribed analgesic in sickle cell patients. Another option used (especially in patients stricken with bone pain) is the non-steroidal anti-inflammatory drug ketorolac (Choice C), which provides pain relief superior to meperidine. However, fluid resuscitation should be performed first. Urinalysis and the measurement of urinary output (Choice D) would be particularly helpful if urinary tract infection or dehydration were suspected in a patient with sickle cell crisis. However, ordering these tests would not be the very first step in management of sickle cell crisis. In a patient with sickle cell disease, the peripheral blood smear (Choice E) usually reveals irreversible sickling of 5-50% of red blood cells. However, the presence of sickled cells is not diagnostic of an acute pain episode. Unfortunately, there are no laboratory tests that can be used to differentiate this type of crisis from baseline condition. Educational Objective: The first step in the management of patients (especially children) presenting with sickle cell crisis is the intravenous administration of fluids. 64% of people answered this question correctly.

Item 2 of 2 The first step in the management of this acute pain crisis is undertaken. The child now allows for a brief physical examination of his abdomen, which is significant for splenomegaly. What is the next best step in management of this patient? A. Blood and urine cultures B. Chest x-ray C. Complete blood count D. Intravenous antibiotics E. Warn the boy?s parents that he should avoid contact sports until the splenomegaly resolves Explanation: Young children with sickle cell disease have a 30% risk of developing splenic sequestration crisis (SCC) and are more susceptible because their spleens have not yet become fibrotic. In SCC, a dramatic fall in hemoglobin concentration occurs secondary to vaso-occlusion within the spleen and splenic pooling of red blood cells. Typically the spleen rapidly enlarges, and the patient can go on to experience hypovolemic shock. Patients with SCC who do not receive blood transfusions in time have a mortality rate of 10-15%. Therefore, a complete blood count (Choice C) should be ordered next for this child with splenomegaly. Blood and urine cultures (Choice A) are appropriate inclusions in the workup of febrile sickle cell patients and should ideally be obtained before antibiotics are administered. However, splenic sequestration crisis is of greater concern in this patient. Chest x-ray (Choice B) is an appropriate inclusion in the workup of febrile sickle cell patients, especially those with signs or symptoms of pulmonary involvement. However, splenic sequestration crisis is of greater concern in this patient. Hospitalization and administration of intravenous antibiotics (Choice D) is recommended for those sickle cell disease patients in pain who are febrile (>40.0C), appear toxic, or are not receiving prophylactic penicillin. Patients with splenomegaly are at increased risk for splenic rupture, either from trauma or infiltrative disease that breaks the splenic capsule. Although the rupture itself may be painless, intraperitoneal bleeding, shock, and death can result. Therefore this child would be well advised to avoid contact sports until the splenomegaly resolves (Choice E). Since this warning is not immediately relevant, however, it can be delayed until discharge. Educational Objective: A complete blood count should be obtained immediately in sickle cell disease patients who present with abdominal pain and splenomegaly, as this patient group is at risk for developing splenic sequestration crisis. 48% of people answered this question correctly. A healthy 23-year-old Caucasian woman comes to the physician for a routine health maintenance examination. She has no complaints. She has no medical problems. She does not use tobacco, alcohol, or drugs. Her family history is not significant. She takes no medication. Her vital signs are within normal limits. Examination shows no abnormalities. Her Pap smear, performed three days ago, reveals "atypical squamous cells of unknown significance" (ASCUS). Which of the following is the most appropriate next step in the management of this patient? A. Refer for colposcopy B. Obtain HPV testing on the sample C. Treat with doxycycline and repeat Pap smear in 6 weeks D. Schedule repeat Pap smear in 3 months

E. Reassurance and repeat Pap smear in one year

Explanation: There are different approaches for the evaluation of ASCUS. Among the above choices, HPV DNA testing is the most important next step in management. In this method, samples are collected both for cytology and HPV DNA, and if cytology results are negative, the sample for HPV DNA is discarded. If cytology results are positive, HPV DNA testing is performed, and if this test is positive for high-risk HPV type, an immediate colposcopy is performed. If the test is negative for high-risk HPV type, the Pap smear is repeated after one year. (Choice A) An immediate colposcopy may be done to evaluate ASCUS, but this will result in a high percentage of false positive cases. (Choices C and D) Obtaining accelerated (every four to six months) serial Pap smears is another approach for the evaluation of ASCUS. In this method, serial Pap smear testing is done until two consecutive Pap smears are normal. If a second Pap smear is still abnormal, a colposcopy is performed. This approach is less favored because of the greater number of follow-up visits and delays in the diagnosis. (Choice E) Patients with ASCUS on cervical cytology possess a high risk of an underlying high-grade dysplasia; therefore, further diagnostic testing is recommended. Educational Objective: HPV DNA testing is the best way to evaluate atypical squamous cells of unknown significance. 22% of people answered this question correctly. A 78-year-old white male who lives in a nursing home was sent to the physician?s office because he is having recurrent falls. The nursing home staff noticed that he has a progressive decrease in vision. The patient denies double vision, dizziness, nausea, vomiting, headache, or vertigo. His other medical problems include Alzheimer?s dementia, peptic ulcer disease, diverticulosis, and degenerative joint disease. He has been taking donepezil, omeprazole, acetaminophen, and multivitamin tablets. His vital signs are stable. On examination, he has decreased vision in both eyes, and bilateral anterior cortical cataracts. Which of the following associated conditions is most likely contributing to his decreased vision? A. Open-angle glaucoma B. Closed-angle glaucoma C. Proliferative retinopathy D. Macular degeneration E. Retinal detachment Explanation: The common causes of decreased vision in elderly patients (especially above 75 years of age) are cortical cataracts and associated macular degeneration. The prevalence of cataracts in this age group is almost 40%, and that of associated macular degeneration is 20%. These patients should be thoroughly evaluated for the severity of macular degeneration, as they may not benefit from cataract surgery. (Choices A and B) Even though the prevalence rates of glaucoma increase with age, it is still less than 10%. (Choice C) Proliferative retinopathy is usually seen in diabetic patients. Without the history of diabetes, this diagnosis is very unlikely. (Choice E) Retinal detachment usually presents with a sudden onset of unilateral loss of vision.

Educational Objective: Cataracts and associated macular degeneration are extremely common in the elderly population and often result in visual impairment. Elderly patients should be thoroughly evaluated for the severity of macular degeneration, as they may not benefit from cataract surgery. A 27-year-old gravida 2, para 1 Caucasian woman at 38 weeks of gestation was admitted to the labor and delivery unit after the spontaneous rupture of membranes. After an unremarkable labor she vaginally delivered a son weighing 3720 grams (8 lbs, 3 oz). Shortly after birth, the infant was found to have hemolytic disease of the newborn secondary to Rh incompatibility. The infant?s serum total bilirubin concentrations then rose rapidly and now, at 24 hours after delivery, are measured at 22.8 mg/dL. Intensive phototherapy has not adequately reduced the bilirubin concentration in the infant?s blood. The parents of this child are Jehovah?s Witness adherents and refuse the proposed exchange transfusion. Without the transfusion, the infant will likely suffer severe brain damage or death. What is the next best step in managing this boy?s care? A. Administer RhoGAM to mother and infant B. Continue intensive phototherapy for additional 72 hours C. Proceed immediately with exchange transfusion to prevent kernicterus D. Consult with the hospital?s ethics committee about seeking court injunction to mandate exchange transfusion E. Transfer infant to another hospital Explanation: In difficult ethical situations such as this one, medical practitioners must seek to balance the autonomy of the family with the welfare of the child. Parental wishes should be honored within certain parameters. For instance, when surgeries are performed on Jehovah?s Witness adherents (be they adults or children), specific blood-sparing techniques or erythropoietin may allow for excellent outcomes. However, in circumstances in which an infant?s survival is at stake, it is appropriate for the medical team to seek approval from the courts to proceed with a blood transfusion (Choice D). Continuing the intensive phototherapy (Choice B) is inadvisable. Typically, phototherapy will result in a decrease in the bilirubin within 4-6 hours. Waiting another 72 hours puts the child at serious risk for developing kernicterus. In a R (D)-negative woman carrying a fetus that is or may be R (D)-positive, RhoGAM (anti-D immune globulin) must be given as a prophylactic measure at 28 weeks of gestation, after clinical procedures or trauma that can cause fetomaternal hemorrhage, and at the conclusion of any pregnancy. Once alloimmunization to the Rh (D) antigen has been established, however, RhoGAM administration (Choice A) is no longer effective. It therefore should not be administered to the mother in this case. Moreover, there is no current indication for the administration of RhoGAM to children, including those with hemolytic disease of the newborn. Proceeding with the exchange transfusion (Choice C) may save the infant?s life, but invites litigation if done without first obtaining the approval of the court. Transferring the infant to another hospital (Choice E) irresponsibly evades the ethical conflict by placing the child?s care in the hands of physicians who do not know his history. Transfers should only be initiated if the hospital currently caring for the boy is unable to properly address his medical issues, which was not implied in this case. Educational Objective: In providing medical care, clinicians must seek to balance the autonomy of the family with the welfare of the child. If the child?s life is endangered, it is appropriate for the clinician to seek approval from the courts to proceed with treatments deemed medically necessary. 23% of people answered this question correctly.

A 55-year-old Caucasian female presents to the emergency department with complaints of severe back pain and difficulty walking. She describes the back pain at the mid-thoracic level, which wraps around her upper abdomen in a band-like fashion bilaterally. She also complains of weakness, numbness, and tingling in both her lower extremities. Her symptoms started approximately two weeks ago and are progressively getting worse. She denies any difficulty with bowel or bladder function. She has a past history of non-Hodgkin?s lymphoma, which was appropriately treated eight months ago with chemotherapy and radiation therapy. Her neurological examination reveals a motor strength of 3/5 in both lower extremities. There is hyperreflexia of the knee and ankle reflexes, and a positive Babinski?s sign bilaterally. Which of the following is the most appropriate next step in the management of this patient? A. Give low-dose corticosteroids and consult radiation oncologist B. Give high-dose corticosteroids and order MRI of the spine C. Emergent surgical decompression with posterior laminectomy D. Get MRI of the spine and call the surgeon for emergent surgical decompression E. Call the radiation oncologist for therapy Explanation: Epidural spinal cord compression (ESCC) is a common complication of cancers, and it is sometimes the initial presentation of cancer. It frequently causes pain and some degree of neurological dysfunction. A metastatic tumor from any primary site can cause ESCC. Some of the common cancers with a tendency to metastasize to the spinal column are: prostrate, breast and lung cancer, nonHodgkin?s lymphoma, and renal cell carcinoma. This patient presents with a classic thoracic radicular pain (wraps around the abdomen) and neurological symptoms of ESCC. It is a medical emergency, and the treatment should be instituted as soon as possible. High-dose corticosteroids (especially dexamethasone) should be administered immediately, and MRI of the spine should be obtained to confirm the diagnosis. Corticosteroids exert their effect by decreasing the edema and swelling around the tumor tissue, thereby decreasing the compression. Radiation therapy is the definitive treatment of choice for most patients. The best response is seen in patients with radiosensitive tumors such as lymphoma, multiple myeloma, breast, and prostrate cancer. Once the MRI confirms the diagnosis, radiation therapy should be started. (Choices A and B) High-dose corticosteroids are considered a part of standard therapy for ESCC. These should always be used along with radiation therapy. (Choices C and D) Lymphoma is extremely radiosensitive and should be treated with radiation therapy. Moreover, in studies, posterior decompression with a laminectomy with or without radiotherapy has not been shown to be superior to radiotherapy alone. Radical surgical resection with tumor debulking has shown to improve outcomes if done early and should be considered in patients with ESCC, due to tumors other than lymphoma. Educational Objective: Symptomatic epidural spinal cord compression should be treated emergently with a combination of high-dose corticosteroids and radiation therapy. MRI is the investigative procedure of choice. 68% of people answered this question correctly. A 36-year-old Caucasian woman comes to examination. She is requesting a prescription boyfriend. Her other medical problems include asthma and gastroesophageal reflux disease. the physician for a routine health maintenance for oral contraceptive pills because she has a new hypertension, severe migraine headaches, bronchial She does not use tobacco, alcohol, or drugs. Her

mother has ovarian cancer. Her father has diabetes mellitus. Her maternal grandmother was diagnosed with ovarian cancer at the age of 56 years, and her paternal grandfather was diagnosed with prostate cancer at the age of 70 years. Her medications include hydrochlorothiazide, albuterol metered dose inhalers, and famotidine. Her blood pressure is 120/80mm Hg and pulse is 76/min. Examination shows no abnormalities. The presence of which of the following makes this patient an unsuitable candidate for oral contraceptive pills? A. Her age and nonsmoking history B. Her history of bronchial asthma C. Her well controlled hypertension D. Strong family history of ovarian cancer E. Her history of migraine headaches

Explanation: The absolute contraindications to the use of oral contraceptives are the following: 1. history of thromboembolic event or stroke 2. active liver disease 3. history of estrogen dependent tumor 4. pregnancy 5. abnormal uterine bleeding 6. heavy smokers who are older than 35 7. hypertriglyceridemia Relative contraindications are the following: 1. migraine headaches 2. poorly controlled hypertension 3. anticonvulsant drug therapy (Choice F) Diabetes mellitus, bronchial asthma, and a family history of malignancies are not contraindications to the use of oral contraceptives. Educational Objective: Migraine headache is a relative contraindication to the use of oral contraceptive pills. 61% of people answered this question correctly. The following vignette applies to the next 2 items A 26-year-old Caucasian female comes to your office because of fever, sweating, heat intolerance, and shortness of breath. She also has pain in front of her neck. Her temperature is 38.3 C (101 F), blood pressure is 156/60 mmHg, pulse is 110/min, and respirations are 22/min. The thyroid gland is slightly enlarged and tender. Extremities are warm and moist. Other systems are essentially unremarkable. Thyroid function tests are as follows: T3: T4: TSH: 256 ng/dL (normal 80-180 ng/dL) 15 mg/dL (normal 4-12 mg/dL) < 0.01 mU/mL (normal 0.35 ? 5.0 mU/mL)

Item 1 of 2 The erythrocyte sedimentation rate is 90 mm/hr. A radioactive iodine uptake scan is mostly likely to reveal: A. Decreased uptake B. Increased diffuse uptake C. Increased patchy uptake D. Hot nodule E. Normal uptake Explanation: This patient has classical clinical manifestations of subacute thyroiditis, which is characterized by very low radioiodine uptake (generally less than 5% at 24 hours). Thyrotoxicosis in subacute thyroiditis is due to the release of stored thyroid hormones by inflammatory damage. Viral infections have been implicated as a cause of subacute thyroiditis. Thyrotoxicosis with low radioactive iodine uptake is also seen in painless thyroiditis, postpartum thyroiditis, surreptitious thyroid hormone administration, and iodine-induced thyroiditis. (Choices B, C, and D) Increased diffuse uptake is seen in toxic multinodular goiter, and increased patchy uptake is seen in toxic adenoma. All these diseases have an overproduction of thyroid hormones. Radioactive iodine uptake is generally increased; however, this may normalize after treatment.

Educational Objective: Thyrotoxicosis with low radioactive iodine uptake is seen in subacute thyroiditis. Item 2 of 2 What is the best treatment option for this patient? A. Propylthiouracil B. Methimazole C. Radioactive Iodine D. Systemic antibiotics E. NSAID and beta-blocker Explanation: As explained before, thyrotoxicosis in patients with subacute thyroiditis is due to the release of preformed thyroid hormones; therefore, antithyroid drugs (Choices A and B), which work by decreasing the synthesis of thyroid hormones, are ineffective. Due to the same reason, radioactive iodine (Choice C) is incorrect. Standard therapy for subacute thyroiditis consists of a NSAID usually combined with beta-blockers (Choice E). Rarely, a systemic glucocorticoid (like prednisone) is required for short period in patients who are very sick and do not respond to standard therapy. The thyrotoxic phase lasts for a few weeks and is then usually followed by a phase of hypothyroidism that can last for a few months. Treatment with thyroxine may be temporarily required during the hypothyroid phase in severely symptomatic patients. The majority of patients with subacute thyroiditis become euthyroid. (Choice D) Occasionally, suppurative infection of the thyroid gland is difficult to differentiate from subacute thyroiditis. Patients with bacterial infections of the thyroid gland are usually not thyrotoxic because the involvemement in the thyroid gland is central. In suppurative thyroiditis, the left thyroid lobe is commonly involved (85%) and skin overlying the thyroid gland is erythematous. Thyroid ultrasound may reveal an abscess formation. Bacterial thyroid infections are treated with systemic antibiotics, and surgical drainage is required in more than 60% of the patients. Educational Objective: Standard therapy for subacute thyroiditis consists of a NSAID usually combined with beta-blockers. 62% of people answered this question correctly. A 32-year-old gravida 3, para 1 Caucasian woman at 38 weeks of gestation is admitted to the labor and delivery unit after complaining of uterine contractions spaced ten minutes apart. Her medical records are not currently available, but the patient states she had a normal pregnancy without any complications. During her initial pelvic examination, several flesh-colored, hyperkeratotic papules are discovered on her posterior introitus, labia major, and labia minora. The papules range from 1-2 centimeters in diameter, have a smooth surface, and are sessile. The patient denies any awareness of the lesions and says she has no history of sexually transmitted diseases. Her cervix is 70% effaced and dilated to 3 centimeters. What is the next best step in managing this situation? A. Allow labor to continue without any intervention B. Administer Pitocin to accelerate labor C. Administer magnesium sulfate for tocolysis D. Administer acyclovir intravenously E. Perform cesarean section Explanation: Condyloma acuminata is the dermatologic manifestation of an infection with the human papilloma virus, with over 90% of such condylomas arising from HPV subtypes 6 and 11 specifically. HPV is

primarily transmitted through sexual contact, and the areas affected include the penis, vulva, vagina, cervix, perineum, and perianal region. Less frequently, HPV may be found in the oropharynx, larynx, or trachea secondary to oral-genital contact or secondary to vertical transmission from mother to infant during childbirth. One relatively common benign laryngeal tumor in children, recurrent respiratory papillomatosis, is caused by the acquisition of HPV during passage through the vaginal canal. However, since HPV is thought to be contracted by the infant in less than 1% of all childbirths to women who have condylomas, no intervention is usually undertaken (Choice A). Pitocin (Choice B) is not indicated since the woman was just admitted and her labor is progressing satisfactorily at this time. Tocolysis with magnesium sulfate (Choice C) is not necessary as this woman is at 38 weeks of gestation and is expected to have a normal vaginal delivery. Acyclovir (Choice D) is a popular and effective medication for the treatment of herpes simplex virus (HSV), not human papilloma virus (HPV). Topical medications such as trichloroacetic acid, podophyllin, and fluorouracil are used in treating HPV. Cesarean section (Choice E) was often done in previous decades as a preventive measure to avoid HPV transmission. It is now only done for HPV infection if the condylomas are thought to be so large that they might bleed excessively during a vaginal childbirth. Educational Objective: Condyloma acuminata is a manifestation of infection with HPV and is not considered a contraindication to vaginal delivery in the pregnant woman. 44% of people answered this question correctly. A 41-year-old Hispanic woman comes to a family practice clinic to discuss her desire to stop smoking. Her husband was just diagnosed with lung cancer, and she is now extremely concerned about her own risk of developing lung cancer. She has smoked 1/2 pack of cigarettes daily for the past 20 years. On several occasions, she attempted to stop smoking by using the nicotine patch or nicotine gum, but always relapsed within the first week. She is now inquiring about other available options. Which of the following medications is commonly prescribed for smoking cessation? A. Fluoxetine B. Venlafaxine C. Trazodone D. Bupropion E. Citalopram Explanation: Because nicotine is so highly addictive, smoking cessation is a difficult undertaking for many patients. Some individuals find nicotine replacement therapy (e.g., nicotine gum, nicotine patch, nicotine inhaler) helpful in the quest to stop smoking. Another typically well-tolerated option is sustained release bupropion, an antidepressant that inhibits neuronal uptake of norepinephrine, serotonin, and dopamine. The combination of bupropion and nicotine replacement therapy is particularly effective. (Choices A and E) Fluoxetine and citalopram are antidepressants categorized as selective serotonin reuptake inhibitors. They are not typically prescribed for smoking cessation. (Choice B) Venlafaxine is an antidepressant that inhibits norepinephrine, serotonin, and dopamine reuptake. It is not typically prescribed for smoking cessation. (Choice C) Trazodone is an antidepressant that inhibits serotonin reuptake. prescribed for smoking cessation. It is not typically

Educational Objective: Bupropion is a drug prescribed both as an antidepressant and for smoking cessation. 95% of people answered this question correctly.

A 57-year-old Caucasian female presents to the primary care physician for a follow-up visit after undergoing a routine colonoscopy. The colonoscopy revealed adenocarcinoma of the ascending colon and two well-differentiated sessile adenomatous polyps of the sigmoid colon. Past medical history is significant for mitral valve prolapse and fibromyalgia. Past surgical history is significant for hysterectomy. Medications include hormone replacement therapy and ibuprofen. The patient eats a high-fiber vegetarian diet, consumes 3-4 alcoholic beverages per day, and has a remote 10-pack-year history of smoking tobacco. She rarely exercises. Family history is negative for cancer. The patient is very concerned about being diagnosed with colon cancer. Which of the following factors placed her at greatest risk for developing the disease? A. Alcohol intake B. NSAID intake C. Tobacco use D. Hormone replacement therapy E. High fiber diet Explanation: Several studies have suggested a strong correlation between alcohol intake (Choice A) and the development of colon cancer, especially in those individuals consuming 45 grams or more of alcohol per day (for sake of comparison, the amount of alcohol in a 12 oz beer is 13 grams). It is thought that the mechanism responsible may involve interference of folate absorption or decreased folate intake. Regular NSAID intake (Choice B), hormone replacement therapy in postmenopausal women (Choice D), and a diet high in fruits in vegetables (Choice E) are all thought to offer some protection against the development of colon cancer. Tobacco use (Choice C) has been linked to an increase in both incidence and mortality of colon cancer. However, this woman?s remote 10-pack-year history of smoking is likely not the most significant factor in the development of her colon cancer, especially given her heavy alcohol consumption. Educational Objective: Colon cancer has been associated with significant alcohol intake and cigarette smoking. Fibrous diets rich in fruit and vegetables, regular NSAID use, hormone replacement therapy, and regular exercise have been identified as protective factors. 22% of people answered this question correctly. The following vignette applies to the next 4 items A 22-year-old primigravida is hospitalized at 34 weeks gestation because of blurred vision, headache, and pain in the right upper quadrant of the abdomen. Her temperature is 36.7 C (98 F), blood pressure is 220/110 mmHg, pulse is 80/min, and respirations are 20/min. The fundoscopic exam is normal. On examination, there is swelling of her face and both her hands, bilateral exaggeration of deep tendon reflexes with clonus, and a positive Babinski. The pelvic exam shows 50% effacement, and 3 cm dilation of the cervix. While you are obtaining IV access, the patient starts to have generalized tonic-clonic seizures. An airway is secured, and breathing is present. Urinalysis reveals proteinuria of 3+. Item 1 of 4 Which of the following is the most effective strategy to decrease this patient's risk for developing further complications? A. Check vital signs every four hours B. Start magnesium sulfate infusion C. Speed vaginal delivery D. Start parenteral clonidine

E. Start phenytoin infusion

Explanation: This patient was admitted to the hospital because she has a severe preeclampsia, which was later complicated with eclampsia. Patients with severe preeclampsia are at greater risk of developing eclampsia. The first priority in patients with eclampsia or postictal coma is respiratory and cardiovascular resuscitation. Anticonvulsant medications can be administered after placing two largebore needles in the patient. Although the most effective agent for hemodynamic and seizure control is magnesium sulfate, the most effective treatment to prevent further complications is to accelerate delivery. Eclampsia can cause several other complications besides seizures, such as disseminated intravascular coagulopathy, acute renal failure, hepatocellular injury, liver rupture, intracerebral hemorrhage, etc. Magnesium sulfate prevents seizures, but it will not stop the pathologic process. (Choice B) Although magnesium sulfate is beneficial, it is not as effective as pregnancy termination. This would have been a correct choice if the question had asked about the best next step in the management of this patient, because hemodynamic stability and seizure control are important to attain before attempting delivery. (Choice D) Clonidine is not indicated in this setting. antihypertensive drugs of choice. Either hydralazine or labetalol are the

(Choice E) Phenytoin or diazepam is not as effective as magnesium sulfate in controlling seizures. Furthermore, as mentioned above, seizures are not the only complication of eclampsia. Speeding up the delivery is the most important. (Choice A) Frequent monitoring of vital signs is part of the management, but it will not prevent the patient from developing further complications. Educational Objective: Eclampsia is a serious complication of pregnancy. If the patient is in the third trimester, especially in the last six weeks, termination of pregnancy is advised in order to stop the pathologic process. There is no pharmacologic therapy more effective than this intervention. 40% of people answered this question correctly. Item 2 of 4 Which of the following is the most effective agent to treat this patient's hypertension? A. Methyldopa B. Labetalol C. Enalapril D. Amlodipine E. Atenolol Explanation: In pregnant patients with a hypertensive antihypertensive drugs of choice. crisis, either hydralazine or labetalol are the

(Choice C) ACE inhibitors are contraindicated in pregnancy. (Choice A) Methyldopa is the preferred agent for oral therapy in mild to moderate hypertension. (Choice D) Calcium channel blockers are added to methyldopa as second line agents. (Choice E) Atenolol is an oral agent (in USA) and is not indicated in the acute setting. Although betablockers are considered to be safe, there are some reports of impaired fetal growth, especially with atenolol if used in the early part of a pregnancy.

Educational Objective: Either hydralazine or labetalol are the drugs of choice for the acute management of hypertension during pregnancy. Methyldopa is preferred for oral therapy in mild to moderate hypertension during pregnancy. Item 3 of 4 During labor, the patient has another seizure. Which of the following is the most appropriate pharmacotherapy in order to avoid seizure recurrence in this patient? A. Phenytoin B. Magnesium sulfate C. Phenobarbital D. Diazepam E. Valproic acid Explanation: Anti-seizure prophylaxis in a patient with eclampsia has been a topic of prolonged debate. Recently, some studies have confirmed that magnesium sulfate is not only the best anticonvulsant medication for patients with eclampsia, but it is also the more effective agent to prevent further seizures. (Choice D) With diazepam, there is a greater risk of respiratory depression. Magnesium sulfate has been proven to be more effective and to have a low neonatal morbidity. Diazepam is more useful in the setting of status epilepticus, or if the patient has contraindications to use magnesium sulfate (e.g., myasthenia gravis). (Choice A) Phenytoin is also useful, but it is not as effective as magnesium sulfate. (Choice C) Phenobarbital is reserved only for those cases in which seizures persist, despite the use of magnesium sulfate, diazepam, or phenytoin. (Choice E) Valproic acid is not part of the therapy of eclampsia. Educational Objective: The best medication to prevent further seizures in a patient with eclampsia is magnesium sulfate. Diazepam or phenytoin can be added to the therapy if seizures persist, although the use of diazepam should be limited due to its depressant effects on the fetus. 73% of people answered this question correctly. Item 4 of 4 Which of the following is considered an extremely ominous sign/feature of this condition? A. Increased PGI 2 and Thromboxane A 2 ratio B. Retinal hemorrhages C. Glomerular capillary endotheliosis D. Microangiopathic hemolytic anemia E. Subcapsular hematoma of the liver Explanation: This patient has preeclampsia complicated by eclampsia. Retinal hemorrhage is considered to be an extremely ominous sign, because it reflects the vascular damage that has occurred in other organs. Retinal vasospasm can also be seen in preeclampsia, and can be visualized on ophthalmoscopic examination. (Choice A) Opposite to normal pregnancy, the PGI 2 to Thromboxane A 2 ratio decreases, and does not increase. This change results in an increase in peripheral resistance, thereby leading to the clinical symptoms and complications of preeclampsia and eclampsia. A deficiency in nitric oxide, as

well as an increase in Endothelin-I, have also been incriminated -- the former being a vasodilator and the latter a potent vasoconstrictor. (Choice C) Glomerular capillary endotheliosis is the typical glomerular lesion of preeclampsia/eclampsia. It involves a marked swelling of the glomerular capillary endothelium and deposits of fibrinoid material in and beneath the endothelial cells. On light microscopy, the glomerular diameter is increased, with endothelial and mesangial cell swelling. (Choice D) Microangiopathic hemolytic anemia can occur in preeclampsia and eclampsia, but it is not considered an ominous sign. It results from the injury of RBC by the damaged endothelium that is usually associated with the condition. (Choice E) Vasoconstriction of the hepatic vasculature can result in necrosis and hemorrhage of the periportal spaces and ultimately, subcapsular hematoma. Educational Objective: Retinal hemorrhage is an extremely ominous sign of preeclampsia/eclampsia. A 16-month-old African American boy is brought to the emergency department (ED) by his mother with several hours history of unlocalized pain and crying. His past medical history is insignificant, except for a runny nose that his mother noticed yesterday. He has been developing normally and has met all the developmental milestones appropriately. All his immunizations are up-to-date. His blood pressure is 100/70 mmHg, pulse is 130/min, temperature is 36.7C (98F) and respirations are 30/min. His physical examination is significant for peripheral cyanosis and symmetric swelling of the hands and feet. Which of the following would be most helpful in establishing the cause of this patient?s problem? A. Hemoglobin electrophoresis B. Chest x-ray C. Blood cultures D. Digital systolic pressure E. Aspirin trial Explanation: The classical clinical scenario of a vaso-occlusive painful crisis of sickle cell disease is described. This presents in infants as painful swelling of the hands and feet (dactylitis or hand-foot syndrome), and can be the first manifestation of sickle cell disease. Most bony vaso-occlusive events occur primarily within the bone marrow cavity. Most are multifocal and associated with mild tenderness and localized edema. In older children and adults, these episodes occur primarily in the joints, back and chest. Hemoglobin electrophoresis is employed to establish the diagnosis. (Choices B and C) Chest x-ray and blood cultures are frequently used as part of the work-up in patients with painful crises to reveal the precipitating factor. (Choice D) Digital systolic pressure in response to cooling is occasionally used to confirm the diagnosis of Raynaud phenomenon. Educational Objective: Vaso-occlusive painful crisis presents in infants as painful swelling of the hands and feet (dactylitis or hand-foot syndrome); this can be the first manifestation of sickle cell disease. A 52-year-old Caucasian woman calls you at 1:00 a.m. on a Saturday due to severe, low back pain. She is a patient of your physician friend who is away for the weekend. She is very upset and angry because she had to call multiple times and was put on hold before she finally got in touch with you. She tells you that she has had low back pain for many years, and her physician usually prescribes

oxycodone for the pain. Her pain is much worse, and she is unable to sleep at all. She wants you to prescribe something stronger, so that she can sleep through the night. You do not have any records available with you at this time, but she does not appear to be drug-seeking, based on the history. What is the most appropriate reaction to the patient's request? A. Ask her politely to calm down and call you back in the morning. B. Ask the patient to take an extra dose of her pain medication. C. Inquire about her pain in detail. D. Ask her to call her primary care physician on Monday. E. Give her long-acting morphine for pain relief.

Explanation: Chronic, low back pain is a common problem encountered in the primary care setting. It is not unusual to see a lot of patients being treated with long-term narcotic analgesics. The primary goal and challenge in the management of such patients is to recognize and treat the pain adequately and appropriately, and to avoid drug abuse and dependency at the same time. It is difficult to recognize the pattern of drug use or abuse in patients who have an underlying chronic disorder that causes pain. It becomes even difficult in the absence of a long-term relationship with the patient (as in the above vignette). It is important, however, to recognize any change in the pain character or severity, which should not be assumed to be due to underlying medical condition and should be thoroughly investigated. The patient in the above vignette should be inquired about her current symptoms in detail and should be managed accordingly. (Choices A and D) Asking the patient to call back while she is having acute pain is unethical and inappropriate. (Choices B and E) The patient should be inquired regarding her pain complaints and triage based on her response. Her worsening pain should not be automatically assumed to be from her chronic back pain. Increasing the dose of her current medications or prescribing a more potent narcotic analgesic is not appropriate without further investigation into the cause of her worsening symptoms. Educational Objective: Any change in the character or intensity of pain in a patient with chronic pain syndrome should be thoroughly investigated. 87% of people answered this question correctly. A 50-year-old African-American man comes to the emergency department (ED) because his left leg has been bothering him a lot. He has a vague and generalized pain in his left leg, and this is present both at rest and with activity. There is also marked swelling of the left leg as compared to the right lower extremity. He was recently discharged from the hospital one month ago ago after being diagnosed and treated for deep venous thrombosis of the left leg. Since his discharge from the hospital, he has been taking 2.5 mg of warfarin daily in the morning. His INR (International Normalized Ratio) in the ED is 1.4. A venogram reveals recurrent acute venous thrombosis extending into the internal iliac vein and inferior vena cava. He does not have any other medical illnesses and is not taking any medications. At this point, which of the following would you recommend for this patient? A. Increase the dose of warfarin to bring INR within therapeutic range B. Switch him to Coumadin C. Interrupt the inferior vena cava with a filter D. Ask him to take the warfarin dose at bedtime E. Discontinue warfarin and begin thrombolytic therapy Explanation: This patient has recurrent deep venous thrombosis with a more proximal extension into the inferior vena cava. This is most likely due to inadequate anticoagulation with warfarin. The anticoagulant effect of warfarin is mediated by the inhibition of vitamin K dependent gamma-carboxylation of factors II, VII, IX, and X. Although long-term therapy with warfarin is highly effective and causes a significant reduction in the frequency of recurrent venous thromboembolism, the dosage required to attain a therapeutic INR (International Normalized Ratio) is individualized for each patient. The dosage of warfarin is mainly determined by the difference in the genetically determined rate of drug metabolism and the patient?s vitamin K status. Warfarin is metabolized in the liver by the hepatic cytochrome P450 2C9 isoenzyme, which exhibits multiple genetic polymorphisms that are responsible for the individualized dose requirements of patients. Furthermore, patients on a vitamin

K-rich diet (green leafy vegetables) can have difficulty attaining a therapeutic INR; thus, they require higher doses of warfarin to achieve the desired INR. In this case, the patient's dosage of warfarin should be increased , and he should be followed closely with serial measurements of PT/INR. (Choice B) Warfarin is the generic form of Coumadin. Switching from warfarin to a different agent or brand name is not indicated at this point. (Choice C) Insertion of an IVC filter is usually indicated only in patients with acute venous thromboembolism who have a contraindication to anticoagulant therapy or who continue to have recurrent venous thromboembolism despite adequate anticoagulation. (Choice D) Changing the dose schedule does not affect the bioavailability of the drug and is therefore not required. (Choice E) The use of thrombolytic therapy in the treatment of acute deep venous thrombosis is controversial and is associated with a higher complication rate. It is only considered if there is significant swelling which may compromise the arterial circulation to the extremity. Educational Objective: The dosage required to attain a therapeutic INR (International Normalized Ratio) is individualized for each patient. Insertion of an IVC filter is usually indicated only in patients with acute venous thromboembolism who have a contraindication to anticoagulant therapy or who continue to have recurrent venous thromboembolism despite adequate anticoagulation. 43% of people answered this question correctly. The following vignette applies to the next 2 items A 17-year-old Caucasian male is brought to the emergency department by his friend due to acute shortness of breath. He is apparently in acute distress and speaks in broken sentences. His friend says that they were playing soccer and he went to the bushes to retrieve the ball. Soon after, he returned to the field and complained of shortness of breath and itching. The patient does not recall any bite or injury. The physical examination reveals widespread wheals all over the body. Item 1 of 2 What is the best next step in the management of this patient? A. Complete physical examination looking for a tick B. Epinephrine subcutaneously C. Epinephrine IV followed by IV fluids D. IV hydrocortisone and fluids E. IV diphenhydramine and furosemide Explanation: The clinical scenario described is consistent with an anaphylactic reaction. The two most typical causes of death in patients with anaphylactic reaction are respiratory failure and cardiovascular collapse. Respiratory failure usually results from airway obstruction due to bronchospasm and/or laryngeal edema; other complications that lead to respiratory failure are cardiogenic or noncardiogenic pulmonary edema, or acute respiratory distress syndrome. Cardiovascular collapse results from increased vascular permeability and hypovolemia, alterations in peripheral vascular resistance, and myocardial depression. Due to the potentially life-threatening nature of anaphylaxis, prompt treatment is required. Epinephrine is the drug of choice for anaphylaxis because it can reverse associated hypotension and bronchospasm. This patient seems to be in acute distress (broken speech indicates high degree of respiratory compromise) and should be given epinephrine IV.

In addition, fluid support should also be given to compensate for the relative hypovolemia due to the increased vascular permeability. (Choice A) Epinephrine should be given immediately because fatality rates are highest in patients whose treatment with epinephrine is delayed. (Choice B) Subcutaneous epinephrine can be used in patients without significant respiratory or cardiovascular symptoms. (Choices E and D) Antihistamine drugs and corticosteroids are usually given to patients with anaphylactic reactions, but these are less important in reversing acute life-threatening complications than epinephrine. Educational Objective: Epinephrine is the drug of choice for anaphylaxis because it can reverse the associated hypotension and bronchospasm. It should be given immediately because fatality rates are highest in patients whose treatment with epinephrine is delayed. 38% of people answered this question correctly. Item 2 of 2 The patient is appropriately treated. He states that he has never had such symptoms before, which scared him, but he wants to go home now. The most recent physical examination reveals minimal rash and clear lungs on auscultation. What is the most important step in the management of this patient? A. Immediate discharge B. Advise to wear a sign that indicates his allergy C. Prescribe a daily antihistamine drug D. Instruct how to use EpiPen injector E. Advice not to play on that playground Explanation: The most important intervention from the list is instructing the patient about the proper use of EpiPen. EpiPen is an epinephrine automatic injector which should always be immediately available in case of recurrence. Such a device may be lifesaving if patients are inadvertently re-exposed to the offending agent. The EpiPen for adults delivers 0.3 mL of 1:1000 epinephrine (0.3 mg). The injector is administered by taking off the cap and pushing the opposite end firmly into the upper lateral thigh. The needle is delivered into the thigh automatically. The patient should count to 10 before removing the pen to insure complete delivery of the medication. The patient should also be instructed to obtain immediate medical care after the injection of the drug. (Choice B) Another important measure would be wearing a Medic Alert bracelet or similar device at all times. This information can expedite diagnosis and appropriate treatment. (Choice A) Because of the potential for biphasic reactions, the patient should be observed for as long as 24 hours in severe episodes of anaphylaxis. (Choice E) The patients should be instructed to avoid the exposure if the allergen that caused the reaction is identified. (Choice C) Daily antihistamines are prescribed to patients with frequent anaphylactic reactions if the allergen can not be determined or avoided. Educational Objective: The most important intervention following an episode of anaphylaxis is instructing the patient about the proper use of EpiPen.

75% of people answered this question correctly. A healthy 24-year-old Caucasian woman comes to the physician for a routine health maintenance examination. She says her sister was recently diagnosed with systemic lupus erythematosus and has had several complications. She is requesting you to do a blood test to diagnose it early. She has no complaints. She has no other medical problems. She does not drink alcohol. She has smoked one pack of cigarettes daily for five years. Her mother has colon cancer. Her vital signs are within normal limits. Examination shows no abnormalities. You order a serum antinuclear antibody test, and it came back positive at 1:80 titers (Normal is less than 1:40). Other labs are unremarkable. Which of the following is the most appropriate course of action? A. Order anti-dsDNA antibody levels. B. Order anti-Smith and anti-dsDNA antibody levels. C. Order serum complement and ESR. D. Explain to her that she does not require further testing. E. Start her on low-dose hydroxychloroquine. Explanation: ANA tests are highly sensitive (>95%) for SLE and are positive in almost all patients with SLE; however positive ANA test results are commonly found in around 20-25% of the normal population. A study conducted on 125 normal individuals found an ANA titer above 1:40 in 32 percent, above 1:80 in 13 percent, and above 1:320 in three percent. In the absence of any SLE symptoms, elevated ANA in low titers has no significance. If the ANA test result is negative and the clinical picture is not highly suggestive of SLE, no further workup for SLE is indicated. The reason that many physicians still order this test, despite knowing the fact that it can yield false positive results, is that they can reassure the patient if the test is negative; however, if the test becomes positive, many will end up having further rheumatologic workup. In the absence of SLE symptoms, this test should therefore not be ordered. Educational Objective: ANA tests are highly sensitive (>95%) for SLE and are positive in almost all patients with SLE; however, positive ANA test results are commonly found in around 20-25% of the normal population. In the absence of any SLE symptoms, an elevated ANA titer has no significance. A mother brings in her 3-month-old son to the office because she is concerned about his "spitting up." The infant?s vomiting has increased in the past month, and occurs primarily at night. The vomit is typically composed of curdled formula, and is not bilious or projectile. The infant had a decrease in appetite, but his weight has been stable. His mother reports that the infant is mildly irritable after feedings, but does not appear to experience significant pain. Based on this presentation, functional gastroesophageal reflux is considered the most likely diagnosis. What should you recommend to the mother as the next best step? A. Prone positioning at night B. Thicken formula with cereal C. Prescription of ranitidine to protect esophageal lining D. Prescription of cisapride as prokinetic therapy E. Surgical repair with Nissen fundoplication Explanation: Gastroesophageal reflux (GER) is a clinical diagnosis. Reassurance should be offered to the mother that the "spitting up" is a normal occurrence in infants up to 24 months old. It typically requires no intervention if the child is otherwise healthy and developing appropriately (the "happy spitter"). In

children with mild GER symptoms, the mainstay of conservative treatment is thickening of formula with cereal, which usually results in decreased emesis, decreased cry, and better weight gain. (Choice A) Prone positioning is another conservative treatment that may alleviate symptoms; however, this treatment method is of some concern, because of the correlation between prone positioning and SIDS. Formula thickening should be attempted first. (Choice C) H2 receptor antagonists such as ranitidine are appropriate in those infants with a more severe GER presentation and who have failed conservative treatment. (Choice D) Cisapride enhances myenteric plexus acetylcholine release, and is highly effective in increasing gastrointestinal motility; however, because cisapride can cause cardiac arrhythmias in some individuals, its availability is heavily restricted in the United States. (Choice E) Surgery is reserved for cases of GER that do not respond to medical management. Educational Objective: Functional GER is extremely common in infants, and should be initially addressed with reassurance and formula thickening. Prescription medication and surgery are reserved for more severe cases of GER which have failed conservative treatment. A 70-year-old Caucasian male presents to the emergency department due to right-sided arm and leg weakness that occurred several hours ago. The episode lasted 15 minutes and resolved spontaneously. He had a similar episode of right arm weakness two days ago. He was diagnosed with hypercholesterolemia on routine check-up three months ago, and has been treated with pravastatin. He does not smoke or consume alcohol. His blood pressure is 140/90 mmHg and his heart rate is 70/min. Doppler studies shows 89% stenosis in his left carotid artery and 40% stenosis in his right carotid artery. He undergoes left carotid endarterectomy. The surgery goes well and he recovers from the anesthesia without any complications. In the recovery room, upon protrusion of the tongue, the tongue deviates to the left. Which of the following structures is most likely damaged? A. Left vagus nerve B. Left hypoglossal nerve C. Left recurrent laryngeal nerve D. Right facial nerve E. Ansa hypoglossus nerve Explanation: A number of nerve injuries can occur during carotid endarterectomy. Inadvertent retraction or transection of the hypoglossal nerve causes tongue deviation to the site of injury, as observed in this patient. (Choice C) Recurrent laryngeal nerve injury leads to unilateral vocal cord paralysis, with a resultant change in voice quality. This nerve lies distal to the area of carotid dissection . (Choice D) The facial nerve can be damaged after it exits from the stylomastoid foramen and courses along the inferior portion of the ear. Its marginal mandibular branch (which supplies the orbicularis oris muscle) is most commonly injured, with a resultant asymmetric smile. (Choice A) The vagus nerve lies posterolaterally in the carotid sheath and is also at risk for injury during the procedure. (Choice E) The ansa hypoglossus nerve innervates the strap muscles of the neck; unlike the other nerves, this nerve may be sacrificed with impunity. Educational Objective: A number of nerve injuries can occur during carotid endarterectomy. Inadvertent retraction or transection of the hypoglossal nerve causes tongue deviation to the site of injury. *Extremely important question for USMLE step-3

A 50-year-old female was found to have abnormal thyroid function on routine blood testing. Her TSH level is 9 mU/ml (0.35 to 5.0 mU/ml is normal) and free T4 is 1.3 ng/dl (normal 0.8 to 1.8 ng/dl). She is currently asymptomatic and is taking no medications. She denies use of tobacco, alcohol, and drugs. Family history is positive for a mother with hypothyroidism. She has four siblings, all are alive and well. Her menstrual cycles are regular. Examination is unremarkable. Her CBC, serum chemistries, and lipid profile are within normal limits. Which of the following will be most useful in guiding her therapy? A. T3 measurement B. Antithyroid peroxidase (anti-TPO) measurement C. Thyroid ultrasound D. Radioactive iodine uptake and scan E. Thyroglobulin measurement Explanation: The patient has subclinical hypothyroidism, which is defined as a mild elevation in TSH levels (5 to 10) accompanied by normal free T4 levels. Treatment is warranted in the presence of (1) antithyroid antibodies, (2) an abnormal lipid profile, (3) symptoms of hypothyroidism, and (4) ovulatory and menstrual dysfunction. When antithyroid antibodies are present with elevated TSH, there is a high chance for a patient to become overtly hypothyroid. A patient with a TSH level > 10 mU/ml is also generally treated with levothyroxine. However, the downside of the treatment in asymptomatic patients is the risk of overtreatment, leading to increased bone loss and atrial fibrillation. Routine screening of thyroid functions is controversial. The American Thyroid Association (ATA) recommends all individuals over the age of 40 to be screened for thyroid dysfunction. The American College of Physicians recommends screening women over 50 years with findings suggestive of thyroid disease. (Choices A, D, and E) T3 measurement, radioactive iodine uptake, and thyroglobulin measurement play no role in the management of subclinical hypothyroidism. (Choice C) Thyroid ultrasound could be useful if thyroid enlargement is present, in order to monitor any change in size and look for dominant nodules (> 1 cm in diameter). The patient's physical exam is normal; therefore, thyroid ultrasound in the above case will not help in making management decisions. Educational Objective: Asymptomatic subclinical hypothyroidism does not require treatment. Treatment is warranted in the presence of: - antithyroid antibodies - an abnormal lipid profile - symptoms of hypothyroidism - ovulatory and menstrual dysfunction 29% of people answered this question correctly. A 50-year-old Caucasian male presents to your office for routine check-up. He says, ?I gained some weight recently, and I know I should give up fast food.? He complains of frequent heartburns that is relieved with food intake and over-the-counter antacids. His past medical history is insignificant. He does not smoke currently, but he used to smoke 1-2 packs of cigarettes daily for 25 years. He consumes 1-2 bottles of beer on weekends. His blood pressure is 150/90 mmHg and heart rate is 85/min. His occult fecal blood test is positive. You recommend esophagogastroscopy, but he replies that the procedure would give him a lot of discomfort. He has heard of ?video pills? that he can swallow and ?these would show the stomach.? He requests for such a pill. What is the best response to this patient?s request? A. No such pills exist. You must be misinformed

B. Video pills have low resolution and are therefore inferior to endoscopy C. Video pills have limited view and endoscopy is clearly preferred D. Video pills may be associated with high rates of GI perforation E. Video pills are a good alternative to endoscopy in your case Explanation: The patient is most probably talking about wireless video endoscopy, a novel technique that is gaining popularity in gastroenterology. A video capsule is taken by the patient and video recording is traced by the outside sensors. Currently, it has been proven to be an effective tool to diagnose some small bowel disease because visualization of the majority of the small bowel mucosa is not possible with push endoscopy. It can be helpful to identify the source of small bowel hemorrhage, tumors, ulceration and inflammatory conditions. (Choice B) The images acquired are of excellent resolution and have an 8:1 magnification, which is higher than that of conventional endoscopes. (Choice E) Limited views of the esophagus, stomach and cecum are obtained by this technique; therefore, in no way can it currently replace endoscopy for the diagnosis of esophageal and stomach disease. (Choice D) Complications of wireless video endoscopy are rare. Retention of the capsule happens in less than 1% of patients. Educational Objective: Wireless video endoscopy is an effective tool to diagnose some small bowel disease because visualization of the majority of the small bowel mucosa is not possible with push endoscopy. Endoscopy is preferred for the diagnosis of esophageal and stomach disease. A 77-year-old Caucasian man is brought to the office by his daughter because of hematuria. He complains of gross, painless hematuria for the past week. He denies frequency, urgency, hesitancy, or dribbling. His other medical problems include hypertension, chronic low back pain, and chronic renal insufficiency. He has smoked one pack of cigarettes daily for 46 years. His medications include an acetaminophen and codeine combination, amlodipine, and hydrochlorothiazide. Rectal examination shows no abnormalities. The patient?s labs reveal the following: Urine: Specific gravity: 1.009 Blood: Gross Leukocyte esterase: Negative Nitrites: Negative WBC: 1-2/hpf RBC: many/hpf Serum Chemistry: Serum Na: 141 mEq/L Serum K: 4.3 mEq/L Chloride: 105 mEq/L Bicarbonate: 20 mEq/L BUN: 53 mg/dL Serum Creatinine: 2.5 mg/dL Ultrasonogram of the kidneys shows bilateral cortical atrophy but no other lesions. Which of the following is the most appropriate next step in the management of this patient? A. Cystoscopy B. Intravenous pyelography C. Prostate-specific antigen D. Urine cytology

E. Abdominal CT scan Explanation: This patient has a high risk of urinary tract malignancy, especially bladder or prostate cancer. This possibility has to be kept in mind in any elderly patient with painless hematuria. Exposure to aniline dyes and a significant smoking history increase the patient's risk. Cystoscopy is the gold standard in the detection of bladder malignancy, and it is also a very useful test to detect prostate cancer. USG is used if there is any suspected kidney mass. (Choice E) An abdominal CT scan would have been the first option if the patient was young, and if kidney stones were being sought. Since he has renal insufficiency, giving contrast for CT scan may worsen his condition. (Choice B) IVP is not advisable because the patient has renal failure. To look for ureteral tumors, a retrograde pyelogram at the time of cystoscopy is an alternative approach in these patients. (Choice C) An elevated PSA level will suggest the presence of prostate malignancy, but it will not detect bladder carcinoma. The likelihood of bladder carcinoma in this patient is higher than that of prostate cancer. (Choice D) Urine cytology has a lower diagnostic yield than cystoscopy; however, it can sometimes identify bladder malignancy when cystoscopy is negative. This is why it is often performed during cystoscopy. Urine cytology alone has a false negative rate of 65-80%. If the cytology is positive, and cystoscopy is negative, evaluation for upper tract and the prostatic urethra tumors should be sought . Educational Objective: The presence of hematuria should always be a matter of concern, especially if an elderly patient presents with painless hematuria. IVP is superior in the detection of malignancies of the upper urinary tract; however, its use is contraindicated in renal failure. Cystoscopy can reveal malignancies of the bladder and prostate. Kidney cancers are best evaluated with USG or CT scan. A 35-year-old man comes to the emergency department after being involved in a snowmobile accident. He complains of severe pain in his right groin and is unable to extend his right knee. He has no other past medical problems. He is an avid athlete and runs five miles every day. On physical examination, he is unable to extend his right knee against resistance; passive movements are within normal limits. There is a sensory loss over the anterior and medial thigh, medial shin, and the arch of the right foot. The knee jerk is markedly decreased on the right side. Which of the following nerve injuries can most likely explain the physical findings? A. Repeat Pap in 6 months B. Sciatic nerve injury C. Obturator nerve injury D. Peroneal nerve injury E. Prepare for extubation Explanation: This patient is most likely suffering from a traumatic femoral nerve injury, which is an uncommon lower extremity nerve injury due to the nerve's location within the pelvis and anterior thigh. On examination, patients with significant femoral neuropathies exhibit weakness involving the quadriceps muscle group, with sparing of leg adduction (which is a function of the obturator nerve). This muscle weakness is exemplified by an inability to extend the knee against resistance. In addition, sensory loss over the anterior aspect and most of the medial aspect of the thigh is typical. This sensory loss extends down the medial shin to the arch of the foot due to dysfunction of the saphenous nerve (branch of the femoral nerve). The knee jerk also decreases in amplitude or becomes absent.

(Choice B) A patient with a significant sciatic nerve injury will complain of weakness affecting most of the lower leg musculature, including the hamstrings. Hip flexion, extension, abduction, adduction, and knee extension are usually normal. There is sensory loss involving the lower leg. In contrast to a femoral nerve injury, the medial calf and arch of the foot may be spared secondary to the preserved innervation by the saphenous nerve (a branch of the femoral nerve). Furthemore, sensation is spared above the knee, both anteriorly and posteriorly. The knee jerk is normal, but the ankle jerk is unobtainable. The most common cause of compression or injury to the sciatic nerve in this region is trauma, which includes hip dislocation, fracture, or replacement. Other etiologies include wayward buttock injections, compression by external sources (e.g., prolonged bed rest), and any deep-seated mass in the pelvis (e.g., hematoma). (Choice C) A patient with an obturator nerve injury will present with pain, weakness in leg adduction, and sensory loss over a small area in the medial thigh. Obturator neuropathy is often secondary to pelvic trauma or surgery. (Choice D) A patient with a common peroneal nerve injury will usually present with an acute foot drop accompanied by weakness in foot dorsiflexion and eversion. In addition, the patient may also complain of paresthesias and/or sensory loss over the dorsum of the foot and lateral shin (superficial peroneal nerve territory). The injury is usually located at the knee, on the lateral aspect of the fibular head. Educational Objective: Femoral nerve injury is characterized by the following physical findings: inability to extend the knee, loss of knee jerk reflex, and sensory loss over the anterior and medial aspects of the thigh, medial aspect of shin, and arch of the foot. An asymptomatic, 21-year-old Caucasian woman comes to the physician for a routine health maintenance examination. She has no other medical problems. She does not use tobacco. She does not drink alcohol. Her family history is not significant. She has no medications. Her vital signs are within normal limits. Examination shows no abnormalities. Her annual pap smear reveals low-grade squamous intraepithelial lesions (LSIL). Satisfactory colposcopy examination confirms CIN 1. Which of the following is the most appropriate next step in the management of this patient? A. Repeat Pap in 6 months B. Cryosurgery C. Laser ablation D. Cold knife conization E. LEEP Explanation: Most of the low-grade squamous intraepithelial lesions (LSIL) or low-grade cervical intraepithelial neoplasia (CIN I) regress spontaneously; therefore, expectant management is preferred for biopsy proven CIN 1 with satisfactory colposcopic examination. A colposcopic examination is satisfactory when an entire lesion and a transformation zone are visualized. Expectant management includes repeat cytology at 6 and 12 months, or HPV DNA testing at 12 months. If there is progression during the follow-up, or lesions are persistent after one year, treatment is indicated. In the above patient, colposcopy examination is satisfactory; therefore, the next best step should be expectant management. (Choices B, C, D, and E) All other choices would have been appropriate if colposcopic examination was unsatisfactory. When the colposcopic exam is unsatisfactory, the next step is always excision as this allows for histological examination. Even if colposcopic examination is satisfactory, treatment may be indicated if the patient is anxious about her disease, or if she seems to be non-compliant with followup. If the patient opts for treatment, available modalities are either ablation or excision. Ablation can be done with cryosurgery or laser, and excision can be done with knife conization, laser conization, or Loop Electrosurgical Excision Procedure (LEEP).

Educational Objective: Expectant management is the preferred option for biopsy proven CIN 1 with satisfactory colposcopic examination. 43% of people answered this question correctly. A 43-year-old African female is recently diagnosed with adult polycystic kidney disease. While explaining the long-term management of the disease, her physician addresses the issue of different modalities of dialysis therapy. He asks the patient which modality (hemodialysis or peritoneal dialysis) she would choose if the need arises in the future. The patient shows a preference for peritoneal dialysis. Which of the following investigations is needed before the patient can be considered as a candidate for peritoneal dialysis? A. CT scan of the head B. Echocardiography C. Chest x-ray D. Intravenous pyelography E. Total colonoscopy Explanation: Patients with adult polycystic kidney disease are predisposed to the following extra-renal manifestations: 1. Hepatic, pancreatic, splenic, and pulmonary cysts 2. Cerebral aneurysms 3. Aortic aneurysm 4. Colonic diverticula 5. Mitral valve prolapse 6. Inguinal and abdominal hernias Before considering peritoneal dialysis as a treatment option in patients with adult polycystic kidney disease, it is important to rule out diverticulosis, which can potentially complicate peritoneal dialysis. This can be done by performing a total colonoscopy. (Choices A, B, C, and D) Other investigations may help detect other extra-renal manifestations of adult polycystic kidney disease, but none of these will affect the decision to forego peritoneal dialysis. Educational Objective: Before considering peritoneal dialysis as a treatment option in patients with adult polycystic kidney disease, it is important to rule out diverticulosis. 34% of people answered this question correctly. A large-scale clinical trial was conducted to assess the effect of a multi-vitamin supplement on the risk of future cardiovascular events. The outcomes of the study were cardiovascular mortality, non-fatal myocardial infarction, and coronary revascularization procedures. According to the study results, the overall relative risk of having the cardiovascular outcomes for the placebo group compared to the treatment group was 1.5, p = 0.30, although the relative risk for only the fifth year of follow-up was 2.05, p = 0.01. Survival curves for the two groups were parallel during the first three years of observation, but began to separate in the third year, favoring the treatment group. Which of the following statements is true concerning the study results given above? A. Multi-vitamin use seems to be ineffective in preventing cardiovascular events B. Inappropriate selection of the study subjects may be present C. Latent period can be demonstrated on the survival plot

D. The follow-up period is too long for such a study E. The sample size is not large enough, and the measure of outcome is unstable Explanation: The concept of latent period is demonstrated in this case. Latency assumption is a very important issue to consider in chronic disease epidemiology. The latent period from getting exposed to developing an outcome is relatively short in infectious diseases. On the other hand, chronic diseases (e.g., cancer or coronary artery disease) may have a very long latent period. In this case, at least three years of continuous exposure to multivitamins were required to reveal a protective effect of the exposure on the cardiovascular outcomes. On the survival plot, you can clearly see that the survival curves run parallel to each other (latent period) and then begin to separate at the third year of followup, favoring the treatment group. (Choice D) An extended period of continuous exposure may be necessary to affect the outcome of this study. (Choice A) Overall, the relative risk (RR) is not statistically significant, because it is ?diluted? by the earlier years of latency; however, the RR for only the fifth year of follow-up clearly demonstrates the beneficiary effect of the therapy. (Choice B) The latent period demonstrated in this study is a natural phenomenon, and does not imply selection bias. (Choice E) The sample size is large enough to demonstrate the efficacy of therapy. Educational Objective: The concept of latent period is an important issue in chronic disease epidemiology. The exposure must be continuous for a certain period of time (called latent period) in order to influence the outcome. 50% of people answered this question correctly. The following vignette applies to the next 2 items A 66-year-old male with chronic rheumatoid arthritis comes to the emergency department with fever and cough productive of yellow sputum for the past ten hours. He has had rheumatoid arthritis for the past twelve years. He has been taking methotrexate (5 mg/wk), prednisone (7.5 mg/d), and NSAIDs (as needed) for the last several years. Physical examination reveals a sick-appearing male with cushingoid features. His blood pressure is 90/70 mm Hg, heart rate is 110/min, temperature is 102 F(38.9C) and respirations are 24/min. His oxygen saturation by pulse oximetry is 92% on 2 liters of intranasal oxygen. His mucous membranes are moist. There is no pallor or icterus. Lung examination reveals crackles on the left base. The other systems are normal. Laboratory investigations reveal an increase in the total WBC count with a left shift. His basic serum chemistry profile is normal, except for a serum sodium level of 131 mEq/L and BUN level of 27 mg/dL. Chest xray reveals a left lower lobe infiltrate. EKG reveals sinus tachycardia with nonspecific ST-T wave changes.

Item 1 of 2 What is the most likely cause of hypotension in this patient? A. Sepsis B. Aldosterone deficiency C. Cortisol deficiency D. ACTH deficiency E. Cardiogenic shock Explanation: This patient's history of long-term exposure to a glucocorticoid drug makes him likely to have chronic suppression of the hypothalamic-pituitary-adrenal (HPA) axis, which may result in secondary or tertiary adrenal insufficiency due to the decrease in the plasma ACTH and cortisol levels, respectively. Since ACTH does not control the secretion of mineralocorticoids from the zona glomerulosa of the adrenal glands, patients have normal plasma aldosterone levels and are normotensive; however, these patients can develop vascular collapse under stressful situations such as an infection due to a decrease in cortisol, which is the hormone responsible for the maintenance of vascular tone. Furthermore, since cortisol promotes the synthesis of catecholamines in the adrenal medulla, cortisoldeficient patients are unable to mount a good pressor response during stress. The hypotension in this patient should respond to fluid repletion and a stress dose of corticosteroids. (Choice A) Although this patient may have sepsis, he is more likely to have hypotension secondary to a suppressed HPA axis. When his blood pressure does not respond to adequate hydration and a stress dose of glucocorticoids, sepsis should be considered. (Choice B) The secretion of aldosterone is not altered in patients with secondary or tertiary adrenal failure because aldosterone is not primarily regulated by ACTH. (Choice D) ACTH doesn?t have any direct effects on blood pressure or the vessel tone. ACTH mainly works through the release of cortisol from the adrenal gland. (Choice E) This clinical picture and insignificant EKG changes make the diagnosis of cardiogenic shock unlikely. Educational Objective: Patients with chronic suppression of hypothalamic-pituitary-adrenal axis can have hypotension during acute infections. Hypotension in these patients usually responds to administration of a stress dose of glucocorticoids and hydration. In secondary and tertiary adrenal failure, there are no significant abnormal mineralocorticoid levels. 27% of people answered this question correctly. Item 2 of 2 Intravenous fluids and antibiotics were administered after drawing blood cultures. What is the most appropriate next step in management? A. Place arterial line and start dopamine. B. Administer intravenous methylprednisone. C. Administer dexamethasone and perform cosyntropin stimulation test. D. Start fludrocortisone. E. Perform dexamethasone suppression test. Explanation: The primary goals of therapy in patients with acute adrenal insufficiency are to reverse the hypotension, correct the electrolyte abnormalities, and replace cortisol. Prompt administration of

intravenous steroids is indicated without waiting for the confirmation of the diagnosis. Intravenous dexamathasone is preferred because it is long-acting and does not interfere with the measurement of serum or urinary steroids during subsequent cosyntropin (ACTH) stimulation test. (Choice D) Mineralocorticoid treatment is not necessary since it takes a long time to show its sodiumretaining effects. The same effect can be achieved with normal saline very quickly. (Choice E) Dexamethasone suppression test is used for Cushing?s syndrome. Educational Objective: Prompt administration of dexamethasone followed by ACTH stimulation test are indicated in patients with suspected acute adrenal insufficiency. 20% of people answered this question correctly. A 38-year-old Caucasian primigravida presents to the physician's office at 20 weeks gestation for prenatal counseling. She is concerned about the risk of Down syndrome, and asks about the measures to diagnose it early. The physician explains that triple screening may detect up to 50% of the cases of chromosomal abnormalities, and that amniocentesis may detect approximately 90% of the cases. She decides not to undergo any test. Eighteen weeks later, she gives birth to a child with Down syndrome. When the physician compared amniocentesis to triple screening, which of the following did he emphasize? A. Increase in false negatives B. Increase in false positives C. Increase in positive predictive value D. Increase in negative predictive value E. Increase in sensitivity Explanation: Before expounding on the characteristics of a diagnostic test, it is important to correctly define the following terms: true positive, false positive, true negative, and false negative. True positive occurs when a patient who really has the disease obtains a positive test result (using the diagnostic test). False positive occurs when a patient who really does not have the disease obtains a positive test result. True negative occurs when a patient who really does not have the disease obtains a negative test result. False negative occurs when a patient who really has the disease obtains a negative test result. The most commonly discussed characteristic of a diagnostic test is its sensitivity. The sensitivity of a test determines the capacity of the test to correctly diagnose a patient with the disease. It is obtained by dividing the number of true positives by the number of people who have the disease (true positives + false negatives). In this case, the physician explained to the patient in layman?s terms that triple screening has a sensitivity of 50% in detecting chromosome abnormalities, while amniocentesis has a sensitivity of 90%. (Choice C) The predictive value of a test gives the probability of a disease after running a test and getting the results; for example, if triple screening turned out to be positive you should discuss the probability of having a given chromosome abnormality given positive test result, i.e., positive predictive value . (Choice D) The negative predictive value of the test determines the capacity of the test to correctly give a negative result. It is obtained by dividing the number of true negatives by the number of people who obtained a negative result (true negatives + false negatives). (Choices A and B) Based on the above values, amniocentesis gives less false negatives and more true positives.

Educational Objective: Sensitivity of a test is the probability of detecting the disease if it is present.

A 41-year-old female is seen for amenorrhea of eight months duration. Her pregnancy tests have been negative during this period. She denies headaches, visual change, galactorrhea, hot flashes, dyspareunia, weight change, or loss of secondary sexual characters. Her past medical history and family history are unremarkable. She is not on any medications, including over-the-counter medications. She denies the use of tobacco, alcohol or intravenous drugs. The physical examination is unrevealing. Her routine labs are within normal limits. Her hormone profile reveals a prolactin level of 50 ng/mL (normal 5-20 ng/ml). Her LH is undetectable, and FSH is low normal. The level of serum a-subunits is markedly increased. Her serum IGF1 levels are normal. MRI of the pituitary shows a 2 cm pituitary tumor with suprasellar extension. What is the most appropriate management of this patient? A. Transphenoidal pituitary surgery B. Estrogen-progesterone cyclically C. Bromocriptine orally D. Pituitary radiation E. Octreotide Explanation: The patient has hypogonadism with suppressed LH and FSH levels, and increased a-subunits. This is classically seen in a nonfunctioning pituitary adenoma, which usually arises from the gonadotropinsecreting cells (gonadotrophs) in the pituitary gland. Gonadotrophs secrete both LH and FSH, which are dimeric glycoprotein hormones. They are composed of a common a-subunit and hormone-specific b-subunit. TSH and Beta-HCG are also dimeric glycoprotein hormones similar to LH and FSH, which have common a and hormone-specific beta sub-units. Although these tumors arise from the gonadotrophs, they do not usually overproduce intact LH and FSH, but generally overproduce the a-subunit. Occasionally, intact FSH is overproduced by these tumors. LH overproduction is exceedingly rare. The increase in prolactin level in this patient is small, and is possibly due to compression of the pituitary stalk by the tumor. The first-line therapy for most nonsecretory pituitary adenomas is transphenoidal surgery. Patients might regain their normal gonadal functions after resection. (Choice B) Treatment with estrogen and progesterone will improve her amenorrhea, but will not have any effect on the large tumor. (Choice C) Bromocriptine, a dopaminergic receptor agonist, has minimal effects on pituitary tumors other than prolactin and growth hormone-secreting tumors. (Choice D) Pituitary radiation is almost never the first choice in the treatment of pituitary tumors due to its delayed effect and risk of hypopituitarism. (Choice E) Octreotide, a somatostatin analogue, acts on somatostatin receptors on the nonfunctioning pituitary adenomas. It is not the preferred primary therapy because the decrease in tumor size with octreotide is usually minimal. Educational Objective: Nonfunctioning pituitary adenomas generally arise from gonadotropin-secreting cells of the pituitary glands. Patients usually present with hypogonadism and low gonadotropin levels. The serum asubunit levels are characteristically increased. Large tumors can have local, compressive symptoms. The primary modality for the treatment is transphenoidal surgery. 34% of people answered this question correctly.

A 41-year-old female is seen for amenorrhea of eight months duration. Her pregnancy tests have been negative during this period. She denies headaches, visual change, galactorrhea, hot flashes, dyspareunia, weight change, or loss of secondary sexual characters. Her past medical history and family history are unremarkable. She is not on any medications, including over-the-counter medications. She denies the use of tobacco, alcohol or intravenous drugs. The physical examination is unrevealing. Her routine labs are within normal limits. Her hormone profile reveals a prolactin level of 50 ng/mL (normal 5-20 ng/ml). Her LH is undetectable, and FSH is low normal. The level of serum a-subunits is markedly increased. Her serum IGF1 levels are normal. MRI of the pituitary shows a 2 cm pituitary tumor with suprasellar extension. What is the most appropriate management of this patient? A. Transphenoidal pituitary surgery B. Estrogen-progesterone cyclically C. Bromocriptine orally D. Pituitary radiation E. Octreotide Explanation: The patient has hypogonadism with suppressed LH and FSH levels, and increased a-subunits. This is classically seen in a nonfunctioning pituitary adenoma, which usually arises from the gonadotropinsecreting cells (gonadotrophs) in the pituitary gland. Gonadotrophs secrete both LH and FSH, which are dimeric glycoprotein hormones. They are composed of a common a-subunit and hormone-specific b-subunit. TSH and Beta-HCG are also dimeric glycoprotein hormones similar to LH and FSH, which have common a and hormone-specific beta sub-units. Although these tumors arise from the gonadotrophs, they do not usually overproduce intact LH and FSH, but generally overproduce the a-subunit. Occasionally, intact FSH is overproduced by these tumors. LH overproduction is exceedingly rare. The increase in prolactin level in this patient is small, and is possibly due to compression of the pituitary stalk by the tumor. The first-line therapy for most nonsecretory pituitary adenomas is transphenoidal surgery. Patients might regain their normal gonadal functions after resection. (Choice B) Treatment with estrogen and progesterone will improve her amenorrhea, but will not have any effect on the large tumor. (Choice C) Bromocriptine, a dopaminergic receptor agonist, has minimal effects on pituitary tumors other than prolactin and growth hormone-secreting tumors. (Choice D) Pituitary radiation is almost never the first choice in the treatment of pituitary tumors due to its delayed effect and risk of hypopituitarism. (Choice E) Octreotide, a somatostatin analogue, acts on somatostatin receptors on the nonfunctioning pituitary adenomas. It is not the preferred primary therapy because the decrease in tumor size with octreotide is usually minimal. Educational Objective: Nonfunctioning pituitary adenomas generally arise from gonadotropin-secreting cells of the pituitary glands. Patients usually present with hypogonadism and low gonadotropin levels. The serum asubunit levels are characteristically increased. Large tumors can have local, compressive symptoms. The primary modality for the treatment is transphenoidal surgery. 34% of people answered this question correctly.

A 41-year-old female is seen for amenorrhea of eight months duration. Her pregnancy tests have been negative during this period. She denies headaches, visual change, galactorrhea, hot flashes, dyspareunia, weight change, or loss of secondary sexual characters. Her past medical history and family history are unremarkable. She is not on any medications, including over-the-counter medications. She denies the use of tobacco, alcohol or intravenous drugs. The physical examination is unrevealing. Her routine labs are within normal limits. Her hormone profile reveals a prolactin level of 50 ng/mL (normal 5-20 ng/ml). Her LH is undetectable, and FSH is low normal. The level of serum a-subunits is markedly increased. Her serum IGF1 levels are normal. MRI of the pituitary shows a 2 cm pituitary tumor with suprasellar extension. What is the most appropriate management of this patient? A. Transphenoidal pituitary surgery B. Estrogen-progesterone cyclically C. Bromocriptine orally D. Pituitary radiation E. Octreotide Explanation: The patient has hypogonadism with suppressed LH and FSH levels, and increased a-subunits. This is classically seen in a nonfunctioning pituitary adenoma, which usually arises from the gonadotropinsecreting cells (gonadotrophs) in the pituitary gland. Gonadotrophs secrete both LH and FSH, which are dimeric glycoprotein hormones. They are composed of a common a-subunit and hormone-specific b-subunit. TSH and Beta-HCG are also dimeric glycoprotein hormones similar to LH and FSH, which have common a and hormone-specific beta sub-units. Although these tumors arise from the gonadotrophs, they do not usually overproduce intact LH and FSH, but generally overproduce the a-subunit. Occasionally, intact FSH is overproduced by these tumors. LH overproduction is exceedingly rare. The increase in prolactin level in this patient is small, and is possibly due to compression of the pituitary stalk by the tumor. The first-line therapy for most nonsecretory pituitary adenomas is transphenoidal surgery. Patients might regain their normal gonadal functions after resection. (Choice B) Treatment with estrogen and progesterone will improve her amenorrhea, but will not have any effect on the large tumor. (Choice C) Bromocriptine, a dopaminergic receptor agonist, has minimal effects on pituitary tumors other than prolactin and growth hormone-secreting tumors. (Choice D) Pituitary radiation is almost never the first choice in the treatment of pituitary tumors due to its delayed effect and risk of hypopituitarism. (Choice E) Octreotide, a somatostatin analogue, acts on somatostatin receptors on the nonfunctioning pituitary adenomas. It is not the preferred primary therapy because the decrease in tumor size with octreotide is usually minimal. Educational Objective: Nonfunctioning pituitary adenomas generally arise from gonadotropin-secreting cells of the pituitary glands. Patients usually present with hypogonadism and low gonadotropin levels. The serum asubunit levels are characteristically increased. Large tumors can have local, compressive symptoms. The primary modality for the treatment is transphenoidal surgery. 34% of people answered this question correctly.

The following vignette applies to the next 2 items A 33-year-old Caucasian male presents to the emergency department with two episodes of coffeeground colored vomiting. He denies ever having such symptoms in the past. He also denies any abdominal pain, dizziness, syncope and black stool. His past medical history is insignificant. He does not smoke or consume alcohol. His blood pressure is 120/72 mmHg while supine and 122/70 mmHg while standing. His heart rate is 90/min. The physical examination is within normal limits. Endoscopy reveals a single small longitudinal tear at the gastro-esophageal junction that is not actively bleeding. Item 1 of 2 Which of the following is the best management for this patient? A. Observation and supportive care B. Thermal coagulation C. Sclerotherapy D. Band ligation E. IV infusion of vasopressin Explanation: This patient presents with upper GI hemorrhage caused by a Mallory-Weiss tear. The "classic" presentation of hematemesis preceded by a bout of retching/vomiting only occurs in 30% of patients. Endoscopy is the gold standard in establishing the diagnosis. This procedure typically reveals a single longitudinal tear at the gastro-esophageal junction. In patients with Mallory-Weiss tear who are not actively bleeding (such as the patient in this case), observation and supportive care are typically necessary. (Choices B, C, and D) Endoscopy also allows for therapeutic interventions (i.e., thermal coagulation, sclerotherapy and band ligation) to stop active bleeding. (Choice E) IV infusion of vasopressin, esophageal balloon tamponade, and angiographic arterial embolization have been used occasionally to control severe or refractory hemorrhage. Educational Objective: In patients with Mallory-Weiss tear who are not actively bleeding, observation and supportive care are typically necessary. Item 2 of 2 Which of the following is the most likely associated finding in this type of patient? A. Increased lower esophageal sphincter (LES) tone B. Hiatal hernia C. Gastric atrophy D. Peptic ulcer disease E. Esophageal varicosities Explanation: Hiatal hernia is the most well known anatomical predisposing factor for Mallory-Weiss syndrome. According to different sources, it is present in 40-100% of patients with this syndrome. During retching or vomiting, the transmural pressure gradient is greater within the hernia than the rest of the stomach, thereby making this location the most likely to sustain a tear. Other precipitating factors include retching, vomiting, straining, hiccuping, coughing, primal scream therapy, blunt abdominal trauma, cardiopulmonary resuscitation, and diagnostic or therapeutic manipulation (e.g., endoscopy). (Choice E) Esophageal varicosities may be revealed in chronic alcoholics who present with this syndrome, but these are not predisposing factors to tears per se. (Choices C, D, and A) No significant association is present with gastric atrophy, peptic ulcer disease and achalasia.

Educational Objective: Hiatal hernia is present in 40-100% of patients with Mallory-Weiss syndrome. A 54-year-old Hispanic male comes for a routine follow-up visit and states that he might be losing weight, particularly from his extremities. He has HIV infection, for which he has been receiving highly active antiretroviral therapy (HAART) for one year. Following his HAART treatment, he showed a significant improvement in his CD4 lymphocyte count and a decrease in his viral titers. His past medical history, aside from his HIV infection, is unremarkable. He does not have hepatitis C or hepatitis B. His father had coronary artery disease and died of a heart attack at the age of 55 years. He does not drink alcohol. He has smoked one pack of cigarettes daily for the last 28 years. He has a history of intravenous drug abuse in the past, but he is currently not using any recreational drugs. On examination, he weighs 168 pounds (his weight one year ago was 160 pounds). His height is 5?9?? (175cm). He has fat tissue depositions on the back of his neck and on his abdomen. His extremities and face appear to be thinned out. The rest of the physical examination is unremarkable, except for minimally palpable hepatomegaly. Lab investigations reveal a normal complete blood count and routine basic chemistry. His lipid profile revealed a total cholesterol level of 280 mg/dL, triglyceride level of 530 mg/dL, and high-density lipoprotein level of 29 mg/dL. His fasting blood sugar is 140 mg/dL and LFTs shows mild elevations of AST and ALT levels. What is the next best step in the management of this patient? A. Start nicotinic acid therapy. B. Start gemfibrozil therapy. C. Start atorvastatin therapy. D. Suggest NCEP ATP III dietary restrictions with other lifestyle modifications. E. Measure 24-hour urinary cortisol levels. Explanation: This is a classic presentation of HIV lipodystrophy occurring after HAART. Although the mechanism of HIV lipodystrophy is unclear, protease inhibitors are most likely responsible. Elevated liver enzymes are most likely due to steatosis. In all patients with hyperlipidemia, including those with HIV lipodystrophy, dietary restrictions and lifestyle modifications are necessary. In this patient, drug therapy is also indicated because of severe hyperlipidemia (triglycerides 530 mg/dL) and the presence of risk factors for a coronary event (male sex, positive family history, smoker, impaired glucose tolerance). His main lipid abnormality appears to be hypertriglyceridemia. His increased total cholesterol is most likely due to an increase in VLDL (rather than LDL) cholesterol. Gemfibrozil is a fibric acid derivative which is mainly used for hypertriglyceridemia but can also cause a modest reduction of LDL cholesterol levels. It is preferred over other fibric acid derivatives because it has been extensively studied in patients with HIV infection. The drug interactions with gemfibrozil are less than those with other fibric acid derivatives. (Choice A) Nicotinic acid is also likely to improve his lipid profile; however, it will not help to treat the glucose intolerance, and is therefore not preferred over gemfibrozil for this patient. (Choice C) Atorvastatin is not the first line therapy for patients with hypertriglyceridemia. It primarily reduces serum LDL cholesterol levels by inhibiting HMG-CoA reductase. Although it decreases triglycerides levels and increases HDL levels, the decrease in triglyceride levels is mainly seen in patients who have elevated LDL levels. Since the increase in total cholesterol in this patient is most likely due to the elevation of VLDL cholesterol, treatment with atorvastatin will not be beneficial. (Choice D) Dietary modifications alone usually lead to an improvement in the lipid profile; however, this patient has severe triglyceride elevation. Drug therapy is necessary in addition to dietary and lifestyle modifications. (Choice E) Although central deposition of fat, a buffalo hump , and impaired glucose tolerance could suggest a diagnosis of Cushing?s syndrome in this patient, this presentation is classic and therefore more suggestive of HIV lipodystrophy. One of the important differences between Cushing?s syndrome

and HIV lipodystrophy is the absence or loss of fat from the face. Cushing?s syndrome presents with rounding of the face, accompanied by facial flushing called "moon facies". In patients with HIV lipodystrophy, the face is thinned out. Measurement of 24-hour urinary cortisol will be normal in patients with HIV lipodystrophy and will be elevated in patients with Cushing?s syndrome. Educational Objective: HAART can lead to lipodystrophic syndrome (redistribution of fat and insulin resistance) and significant hyperlipidemia. Gemfibrozil is the treatment of choice for hypertriglyceridemia seen in patients with HIV lipodystrophy. The following Vignette applies to the next 2 items A 43-year-old HIV-positive Caucasian male with a history of intravenous drug abuse was admitted to the hospital for nausea and vomiting and severe pain in the mouth, throat, and epigastric region. He had been diagnosed with AIDS four years ago after presenting with pneumocystis carinii pneumonia, which resolved with treatment. Since his diagnosis he has been on prophylactic antibiotics and highly active antiretroviral therapy, which has been effective in slowing the deterioration of his immune function. After admission, it proved necessary to begin total parenteral nutrition via a central venous catheter because of his inability to take in food or liquid by mouth. When he then became increasingly febrile, hypotensive, tachycardic, and tachypneic, he was transferred to the intensive care unit. While there he began to complain of significant pain and sensitivity to light in his right eye. On funduscopic examination, several large, glistening, off-white lesions with indistinct borders are seen. The lesions are three-dimensional and appear to extend from the chorioretinal surface into the vitreous. A vitreous haze is present. Visual acuity is decreased. Item 1 of 2 What is the most likely cause of his ocular pain? A. Mycobacterium avium complex B. Infective endocarditis C. Cryptococcosis D. Candidiasis E. Aspergillosis Explanation: Oropharyngeal and esophageal candidiasis arises commonly in HIV-positive patients and can disseminate widely to multiple organs, including the eye. Endogenous candida endophthalmitis (Choice D) is caused by hematogenous seeding of the eye with Candida species (especially C. albicans), and is found in 10-28% of patients with candidiasis. The presence of candida endophthalmitis is a marker for widespread disseminated candidiasis and should not be taken lightly. The condition is frequently characterized by ocular pain, photophobia, scotomas, and fever. Risk factors that place patients at greatest risk for developing candida endophthalmitis include central venous catheters, total parenteral nutrition, broad-spectrum antibiotic therapy, prior abdominal surgery, neutropenia, corticosteroid therapy, and injection drug abuse. Mycobacterium avium complex (Choice A) is characterized by fever, night sweats, fatigue, shortness of breath, abdominal pain, diarrhea, weight loss, and lymphadenopathy. Ocular manifestations are not expected. Infective endocarditis (Choice B) is often subtle and nonspecific in presentation. It may be characterized by fever, fatigue, anorexia, back pain, and weight loss. One rarely seen ocular manifestation of infective endocarditis is the Roth spot, which is a retinal hemorrhage with a pale center. Pulmonary cryptococcosis (Choice C) is characterized by fever, malaise, cough, dyspnea, and pleuritic pain. CNS cryptococcosis is characterized by headache, altered mental status, nausea, and vomiting. Ocular manifestations of CNS cryptococcosis are uncommon, but can include papilledema, optic neuritis, and chorioretinitis.

Invasive aspergillosis (Choice E) can occur in immunosuppressed patients and is characterized by fever, cough, dyspnea, pleuritic chest pain, and occasionally hemoptysis. Aspergillus endophthalmitis results from hematogenous spread and can cause eye pain and visual changes. Educational Objective: Risk factors that place patients at greatest risk for developing candida endophthalmitis include central venous catheters, total parenteral nutrition, broad-spectrum antibiotic therapy, prior abdominal surgery, neutropenia, corticosteroid therapy, and injection drug abuse. 41% of people answered this question correctly.

Item 2 of 2 Given the patient?s condition, which of the following treatment options is likely to be of greatest benefit? A. Vitrectomy and systemic ketoconazole B. Vitrectomy and systemic amphotericin B C. Intravenous vancomycin and gentamycin D. Systemic amphotericin B only E. Clarithromycin and rifabutin Explanation: Patients with candida endophthalmitis who have chorioretinitis with vitreal involvement should be treated with vitrectomy and systemic antifungal therapy with amphotericin B (Choice B) and/or fluconazole. An early vitrectomy improves the likelihood of a positive outcome, and intravitreal injection of amphotericin B may be of help. Prompt diagnosis and treatment of candida endophthalmitis is essential, as the condition can worsen quickly. Although ketoconazole (Choices A) can treat candidiasis, it does not attain sufficient concentration within the eye and is therefore not an ideal choice in this case. Systemic amphotericin B (Choice D) attains sufficient concentration within the choroid and retina, but not within the vitreous body. Patients who have candida endophthalmitis with vitreal involvement will therefore require vitrectomy in conjunction with systemic antifungal treatment. (Choice C) Combination of vancomycin and gentamycin is used in patients with infective endocarditis (in IV drug users) as empirical therapy. Educational Objective: Candida endophthalmitis with vitreal involvement should be treated with vitrectomy and systemic antifungal therapy (amphotericin B and/or fluconazole). 45% of people answered this question correctly. A 56-year-old male accountant is brought to the emergency department by paramedics because of a sudden onset of severe crushing chest pain and dizziness. The pain started while he was in a meeting with some of his clients. He has a history of longstanding hypertension, gastroesophageal reflux disease, hyperlipidemia, and chronic stable angina. He occasionally gets substernal chest pressure with brisk walking and running, and this is completely resolved by sublingual nitrates. He is a lifelong smoker and currently smokes two packs of cigarettes a day. His other medications include aspirin, hydrochlorothiazide, metoprolol and atorvastatin. During this episode, he took four sublingual nitroglycerine tablets in the first 10 minutes, which did not change the character and severity of the

pain. His temperature is 36.7C(98F), blood pressure is 160/86 mm Hg, pulse is 80/min, and respirations are 16/min. Oxygen saturation is 94% on room air. An initial EKG and chest x-ray done in the emergency department are normal. Which of the following is the most appropriate statement to tell the patient? A. You may be having an acute myocardial infarction. B. You may have a perforated peptic ulcer. C. You may have an aortic dissection. D. You may have a pulmonary embolism. E. You may have a tension pneumothorax. Explanation: Aortic dissection is an uncommon but potentially life threatening cause of severe and sudden chest pain. It has the same predisposing risk factors as an acute coronary syndrome (unstable angina and myocardial infarction), including age greater than 55 years, hypertension, hyperlipidemia, and chronic smoking. The diagnosis of aortic dissection is usually based on the history and the physical examination findings. Patients with acute aortic dissection typically present with a sudden onset of severe or "tearing" chest pain radiating to the back. The pain is usually not relieved by sublingual nitroglycerine. Other clinical clues favoring the diagnosis of an aortic dissection include the blood pressure difference in the two arms, an early diastolic murmur (reflecting aortic insufficiency), and the presence of mediastinal widening on chest x-ray. (Choice A) The patient certainly has risk factors and a history of coronary artery disease based on his symptoms of exertional angina. A sudden onset of severe chest pain, unrelieved by nitrates, can be due to an acute myocardial infarction; however, it would be unusual to have a normal EKG in a patient with acute MI while he is having active chest pain. (Choice B) Peptic ulcer perforation is an unlikely diagnosis in the absence of any abnormal abdominal exam findings. Furthermore, a chest-x ray/erect abdominal-x ray usually shows air under the diaphragm. (Choice D) Pulmonary embolism usually presents with pleuritic chest pain that is less severe in intensity, and is associated with shortness of breath and hypoxia. (Choice E) Tension pneumothorax presents as a sudden onset of shortness of breath, hypoxia, absent airflow, and hyperresonance to percussion on the affected side. The patients chest x-ray and examination findings are not consistent with tension pneumothorax. Educational Objective: Aortic dissection is a catastrophic condition, and should be suspected from the initial history and physical examination findings. A 35-year-old woman comes to her physician after she palpated a lump in her right breast. She doesn?t have any other complaints. She hasn?t seen a doctor for ten years. She states that she regularly performs a self-breast exam after menses. Her past medical history is unremarkable. Her mother died of breast cancer at the age of 40. Inspection of her breasts is normal. A 1 x 1cm rubbery, firm, freely mobile round mass is palpated in the upper, outer quadrant. No axillary lymph nodes are palpated. What is the best next action? A. Observation B. Ultrasonography C. Excisional biopsy D. Fine needle aspiration E. Mammography Explanation:

A baseline mammogram at age 35 is recommended for any woman who has an increased risk for breast cancer. All patients with breast lumps after the age of 35 should be evaluated using mammography, especially if they have a family history of breast cancer. The findings on the physical exam are consistent with a fibroadenoma. Apart from the fibroadenoma, this patient might have other anomalies imperceptible on physical exam that might be seen on the mammogram, which is more superior for this purpose than an ultrasound (Choice B). It is best to make further decisions for this patient?s management after the mammogram is obtained. (Choices A, C, and D) Although the remaining choices are also appropriate for the management of breast masses, mammography is the best initial step for this patient. Educational Objective: All patients with breast lumps after the age of 35 should be evaluated with mammography, especially if they have a family history of breast cancer. 32% of people answered this question correctly.

A 46-year-old man is admitted to the hospital with complaints of constant mid-abdominal pain for the past five days. He has a history of intravenous drug abuse, alcoholism, and chronic pancreatitis. He continues to drink heavily. He has had three recent admissions for pancreatitis in the past four months. You consult a gastroenterologist, a close friend of yours, for a possible endoscopic retrograde cholangiopancreatography. He has also known the patient from his previous admissions. After the procedure, he calls and tells you, "I do not want to get involved in this patient?s care. He is extremely non-compliant and drug-seeking, and has slapped one of the nurses in the endoscopy lab for not giving him enough pain medications." You have been the patient?s primary care provider for the past 12 years and know that the patient can be difficult at times. Which of the following is the most appropriate next step in the management of this patient? A. Ask him to withdraw from the patient?s case if he wishes. B. Tell him that he can withdraw from the case but he will not be consulted again for any of your patients in the future. C. Ask him to document the patient?s behavior in the chart and then withdraw from patient?s case. D. Tell him that he cannot withdraw from the case once he is involved in patient care. E. Tell him that he can withdraw from the case if some other gastroenterologist is willing to take care of this patient. Explanation: All physicians have a moral obligation to provide continuity of care for their patients. It is not appropriate to withdraw or discontinue patient care in the middle of a hospitalization or during an acute ongoing medical treatment. All physicians have the option to choose who they want to serve; however, once they are involved in a case, they should not neglect the patient. If a physician wishes to withdraw from a case, he can do so if he provides the patient or the caregivers a notice long in advance so as to sufficiently permit the transfer of medical care to another physician. In the above vignette, the gastroenterologist is already involved with the patient's care since he has already performed the procedure. He can opt to withdraw from the patient?s case only if there is another gastroenterologist willing to assume patient care responsibility. (Choices A and B) Asking the physician to withdraw without securing another gastroenterologist for the patient is not appropriate. (Choice C) The patient?s behavior should certainly be documented in the chart; however, this does not provide sufficient grounds for withdrawal in the middle of the treatment. (Choice D) As described above, the physician can withdraw from the case if another gastroenterologist is willing to assume the patient care responsibility. Educational Objective: Physicians can terminate the physician-patient relationship by providing a notice long in advance to sufficiently permit the transfer of care to another healthcare provider. 29% of people answered this question correctly. A 39-year-old Caucasian man is brought to the emergency department because of epigastric pain and melena. He was diagnosed of peptic ulcer disease five years ago. He underwent highly selective vagotomy with antral ulcer resection due to persistent gastrointestinal bleeding one year later. He has no other medical problems. He is a real estate agent. He does not use tobacco or illicit drugs, but drinks alcohol occasionally. His medications include omeprazole and antacids. His blood pressure is 130/80 mm Hg, pulse is 120/min, and respirations are 18/min. He looks pale and frightened. Examination shows a midline scar in the epigastrium. There is moderate tenderness on palpation of the upper half of the abdomen. Some muscular guarding can be appreciated. There is no rebound tenderness, and bowel sounds are present. A rectal exam reveals dark blood. During the examination, the patient feels nauseous and has one episode of hematemesis. He receives

intravenous hydration and parenteral pantoprazole. Some laboratory tests are done, including crosstyping for a possible transfusion. The results show the following:

CBC Hb: 7.6 g/dL Ht: 23% Platelet count: 450,000/cmm Leukocyte count: 8,000/cmm Segmented neutrophils: 72% Bands: 3% Lymphocytes: 25% Serum chemistry Serum Na: 145 mEq/L Serum K: 3.9 mEq/L Chloride: 110 mEq/L Bicarbonate: 25 mEq/L BUN: 28 mg/dL Serum creatinine: 1.2 mg/dL Calcium: 9.9 mg/dL Glucose: 77 mg/dL Which of the following is the most appropriate test to confirm the diagnosis? A. Abdominal ultrasound B. Upper gastrointestinal (GI) radiology series C. Abdominal computerized tomography (CT) with contrast D. Upper gastrointestinal (GI) endoscopy E. Radionuclide imaging Explanation: There is a significant rate of rebleeding after gastric surgery for peptic ulcer treatment. For instance, truncal vagotomy with partial antrectomy has a recurrence rate of 5 % to 10% after a mean follow-up of 3.5 years. The preferred method to confirm the source of bleeding is upper gastrointestinal (GI) endoscopy, because it also has therapeutic applications such as photocoagulation or local injection of vasoconstrictor agents. In addition, early endoscopy has been associated with a significant decrease in mortality and hospitalization time. (Choice A) Abdominal ultrasound is not adequate for the evaluation of gastrointestinal bleeding because it cannot identify intraluminal lesions. (Choice C) Abdominal CT scan is not regularly used to evaluate GI bleeding because endoscopy is superior to it. Abdominal CT scan with contrast can sometimes localize the source of bleeding of lower gastrointestinal hemorrhages. (Choice B) Although a contrast study of the esophagus and the stomach can certainly show the presence of ulcerated lesions, it does not offer therapeutic advantages. This is why it is not recommended for a patient who needs immediate evaluation, such as those who are actively bleeding. (Choice E) Radionuclide scanning is less specific than upper GI endoscopy. Its accuracy depends on the experience of the operator and the type of clinical center; values may be as low as 24% or as high as 91%. Educational Objective:

Upper GI endoscopy is the preferred method to evaluate upper gastrointestinal bleeding because it also has therapeutic applications. The most common reason for bleeding in a patient with a history of previous gastric lesion is the development of a new ulcerated lesion or the recurrence of the previous one. These lesions are readily diagnosed and managed through endoscopy. 84% of people answered this question correctly.

Two groups of investigators are interested in the problem of esophageal cancer in a population that has a high prevalence of hot beverage consumption. One group conducted study A, which showed that hot beverage consumption is associated with esophageal cancer, with an odds ratio (OR) of 1.51 and 95% confidence interval of 1.19 to 1.81. The other group conducted study B, which demonstrated an OR of 1.46 and 95% confidence interval of 0.95 to 1.97. Which of the following is the best statement concerning the results of these two studies? A. The result of study A is not statistically significant B. The sample in study B is poorly selected C. The result in study A is not valid D. The sample size in study B is small E. P value in study B is less than 0.05 Explanation: Although both studies have almost the same point estimate of association (OR of about 1.5), study B has a wider confidence interval than study A. Furthermore, the confidence interval of study B includes 1.0; therefore, it is not statistically significant. The lack of statistical significance of the results in study B is most probably due to a smaller sample size, which resulted in insufficient power to detect the difference between the exposed and unexposed subjects. Increasing the sample size of study B will make the confidence interval tighter. Both studies reached the same conclusion, but because of a smaller sample size of study B, its estimated odds ratio does not reach statistical significance. (Choice A) The 95% confidence interval in study A does not include the ?null? value for the odds ratio (that is 1.0); therefore, the odds ratio achieves statistical significance. (Choices B and C) The accuracy or validity of the studies cannot be evaluated without any information on how the studies were designed and conducted. (Choice E) The P value of statistically insignificant results should be greater than 0.05. Educational Objective: The power of a study is the ability to detect the difference between two groups (treated vs nontreated, exposed vs non-exposed). Increasing the sample size increases the power of a study. As a result, the confidence interval of the point estimate (e.g., odds ratio) becomes tighter. 25% of people answered this question correctly. A 58-year-old female with a history of diabetes mellitus presents to the family physician for diarrhea, weight loss, bloating, and excess flatulence. She was diagnosed with diabetic enteropathy last year. Laboratory examinations show anemia with MCV of 101 fl. Which of the following is the best test to diagnose bacterial overgrowth in this patient? A. Quantitative jejunal cultures B. [14C]-d-xylose breath test C. Bentiromide test D. Breath hydrogen analysis E. Quantitative fecal fat Explanation: The demonstration of excessive bacterial concentrations in a jejunal aspirate is the gold standard for the diagnosis of bacterial overgrowth. It can be performed during endoscopy or by fluoroscopy with jejunal intubation. The patient's bacterial overgrowth is most likely due to her diabetic enteropathy. (Choice B) The Clinical Efficacy Committee of the American College of Physicians also advocates alternate use of one-gram [14C]-d-xylose in routine clinical practice. Xylose is a pentose sugar that is

catabolized by gram-negative aerobes present in the intestine, releasing the radioactive isotope 14CO2, which is detectable in breath samples. (Choice D) Breath hydrogen testing is performed by the administration of a test dose of carbohydrate, which would be associated with a rise in breath hydrogen levels in patients with bacterial overgrowth. (Choice C) A bentiromide test is a test for pancreatic insufficiency and not bacterial overgrowth. (Choice E) Quantitative fecal fat is a test for fat malabsorption, but is not specific for bacterial overgrowth. Educational Objective: The gold standard for the diagnosis of bacterial overgrowth is the demonstration of excessive bacterial concentrations in a jejunal aspirate. 25% of people answered this question correctly. You are managing the care of a 32-year-old football player in the intensive care unit. He was involved in a motor vehicle accident two days ago, and had sustained severe head injuries. The paramedics intubated him at the scene of the accident. On the third day of hospitalization, he remains unresponsive to all stimuli and has no spontaneous respiratory drive. He is on full ventilator support. His temperature is 36.7 C (98 F), blood pressure is 120/70 mmHg, and pulse is 76/min. He is declared braindead by the ICU team. His family is informed of his status, and they wish to proceed with the withdrawal of life support. His fianc?nforms you that he had always wanted to donate his organs in case of an unexpected death. Which of the following is important to maximize the viability of donor organs? A. Anti-hypertensive medications to keep systolic blood pressure less than 100 mmHg B. Low-dose beta blockers to keep heart rate less than 60/minute C. Lower the ventilator rate to induce hypercapnia D. Maintain body temperature to less than 35 C (95 F) E. Maintain normothermia with blankets Explanation: Organ transplantation from cadaveric donors has become increasingly common in the last few years. Most of the organs are obtained from brain dead donors. The successful recovery of viable organs for transplantation depends on the appropriate medical care of brain dead patients; therefore, it is important to learn the basic principles of management of these patients. The primary aim is to achieve hemodynamic stability and to maintain physiologic homeostasis to maximize the viability of organs. A common misconception is that the body temperature should be kept low to improve the viability of organs. In contrast, normothermia should be maintained passively in all brain dead patients using blankets. If the body temperature is less than 35 C (95 F), active rewarming should be attempted with warm air blankets and warm intravenous fluids. (Choices A and B) It is critical to achieve and maintain a normotensive, euvolemic state. Hypotension is very common in brain dead patients, and may be due to the loss of sympathetic tone, systemic infections, volume depletion secondary to losses, and diabetes insipidus. Adequate volume resuscitation is the first step in correcting hypotension and ensuring donor viability. If systemic blood pressure still remains low, pressors and inotropic agents may be used to improve the blood pressure. (Choice C) Hypercapnia has not been shown to improve organ viability, and can be even damaging to the tissues and organs. The goal of management is to provide optimal ventilator support to prevent hypoxia and hypercapnia. Educational Objective:

The primary goal of medical management of a brain dead organ donor is to maintain a euvolemic, normotensive, and normothermic state. 45% of people answered this question correctly. A 30-year-old male is brought to the emergency department (ED) by his relatives following a generalized tonic-clonic seizure. Physical examination shows tetany and cramping of muscles. Chvostek and Trousseau signs are positive. The EKG tracing shows prolongation of QT intervals. His blood pressure is 100/70 mmHg, pulse is 67/min, temperature is 36.7C (98F), and respirations are 12/min. Hypocalcemia is suspected. Apart from measuring the patient's serum calcium levels, which of the following is the most important laboratory investigation to conduct? A. Serum parathyroid levels B. Serum phosphorus levels C. Serum albumin levels D. 25-hydroxy vitamin D levels E. Serum glucose levels Explanation: This patient has clinical features suggestive of hypocalcemia. Aside from measurement of the serum calcium level, measurement of the serum glucose level is also very important. Hypoglycemia is the greatest mimmicker of central nervous system diseases. It can present with protean signs and symptoms. One remote possibility of the association of hypocalcemia and hypoglycemia is autoimmune polyglandular failure, where hypoparathyroidism (decreased calcium levels) is associated with primary adrenocortical insufficiency (decreased glucose levels). (Choices A, B, C & D) Measurement of the serum parathyroid hormone, phosphorous, albumin and vitamin D levels are also required in evaluation of patients with hypocalcemia; however, measurement of the serum glucose levels should be done right away by doing a fingerstick. The PTH level is low in patients with hypoparathyroidism, and increased in patients with pseudohypoparathyroidism and vitamin D deficiency. The serum phosphorus level is increased in patients with hypoparathyroidism and low in vitamin D deficiency. Approximately 50% of calcium is bound to albumin; therefore a change in the serum albumin levels can result in a change of the total calcium levels without a change in the ionized fraction. Total serum calcium levels should always be corrected for corresponding serum albumin level. Hypovitaminosis D is a very important cause of hypocalcemia and can be assessed by measuring the 25-hydroxy vitamin D levels. Educational Objective: Hypoglycemia can lead to protean central nervous system manifestations which include focal neurological deficits, alerted mental status, seizures, and coma. Serum glucose levels should therefore be measured in all patients presenting with central nervous system dysfunction. A healthy 14-year-old Caucasian girl comes to your office and asks for a prescription for oral contraceptive pills. She recently started having sex with her boyfriend, who is 16 years old. She started menstruating at the age of 13, and she has always had regular cycles. She is a non-smoker and non-alcoholic. She does not have any medical problems. She uses condoms infrequently. Which of following is the most appropriate next step in the management of this patient? A. Prescribe her oral contraceptives and inform the parents later. B. Call the parents from the office and prescribe if they are okay. C. Advise her to use barrier methods instead of oral contraceptives. D. Advise her to use barrier methods in addition to oral contraceptives. E. Educate about teenage sex and do not give contraception. Explanation:

This patient does not have any absolute or relative contraindication to the use of oral contraceptive pills (OCPs) and has a personal preference for this mode of contraception; therefore, she should be prescribed OCPs. In addition, barrier methods should be offered to reduce her risk for STDs. (Choices A and B) Adolescents do not need consent from their parents for the use of oral contraceptives. The biggest reason for failure of adolescents to obtain contraceptive services is the concern about confidentiality; therefore, confidentiality should always be maintained. Educational Objective: Adolescents do not need consent from their parents for contraceptive services. Barrier methods should be used by all sexually active adolescents even if an additional method of contraception is being used. A 16-year-old Caucasian girl is being worked up for primary amenorrhea. She has short stature, webbed neck, high palate, and a short fourth metacarpal. Her karyotype analysis showed 45, XO. Her mother is concerned about the patient's chances to conceive. Which of the following is the best response? A. IV labetalol B. She has the same chances of pregnancy as the general population. C. She can become pregnant, but pregnancy is contraindicated in her condition. D. Pregnancy is absolutely impossible. E. Educate about teenage sex and do not give contraception. Explanation: Ovarian function and morphology is highly variable in patients with Turner syndrome. Most patients do not undergo puberty, and present with primary amenorrhea. Some patients may develop normally and undergo spontaneous menarche, but secondary amenorrhea may occur after some time. Such patients may become pregnant without medical assistance before developing secondary amenorrhea; however, most women are infertile. In one retrospective study of 522 patients, three women became pregnant spontaneously. (Choice C) Turner syndrome is not a contraindication for pregnancy; however, patients should undergo a complete medical evaluation. Educational Objective: Pregnancy is almost impossible, but there still remains a very small chance, in patients with Turner syndrome. 42% of people answered this question correctly. The following vignette applies to the next 2 items A 65-year-old Caucasian male presents to the emergency department six hours after the onset of leftsided weakness in his arm and leg. He denies headache, nausea, vomiting and swallowing difficulty. He had two episodes of extremity weakness and numbness within the last three months that lasted 15-20 minutes and resolved spontaneously. His past medical history is significant for hypertension and diabetes. His current medications are hydrochlorothiazide, lisinopril and metformin. He also takes ibuprofen for muscle cramps and antacids for heartburns occasionally. He does not smoke or consume alcohol. His family history is insignificant. His blood pressure is 170/100 mmHg and heart rate is 80/min, regular. The CT scan of his head is negative for intracranial hemorrhage. Item 1 of 2 What is the best next step in the management of this patient?

A. IV labetalol B. IV isotonic D5W and oral metoprolol C. IV isotonic saline and aspirin D. IV isotonic saline and nimodipine E. Lumbar puncture Explanation: The acute management of ischemic stroke emphasizes the concept of ?first, do no harm.? Despite very intensive clinical research in this field, very few interventions have been proved to be beneficial. Cautious IV hydration with isotonic saline is generally indicated. Aspirin has been shown to be effective in improving prognosis in two large-scale clinical trials, and is recommended in all patients with ischemic stroke who do not receive thrombolytics or anticoagulants. (Choice A) Some interventions that sometimes seem intuitively appropriate can be harmful. For example, the reduction of blood pressure in patients with acute ischemic stroke can deteriorate their condition, and should be used only if the blood pressure is extremely high (>220/120). If blood pressure reduction is required, labetalol is the drug of choice. (Choice B) Overzealous hydration and D5W solution are usually avoided. (Choice D) Despite initial encouraging evidence, large-scale studies demonstrated no improvement in the outcome of patients with thrombotic stroke receiving nimodipine. (Choice E) Lumbar puncture is indicated when a patient has a negative CT scan and the clinical suspicion of subarachnoid hemorrhage remains high. Educational Objective: The reduction of blood pressure in patients with acute ischemic stroke can deteriorate their condition, and should be used only if the blood pressure is extremely high (>220/120). Despite initial encouraging evidence, large-scale studies demonstrated no improvement in the outcome of patients with thrombotic stroke receiving nimodipine. 48% of people answered this question correctly. Item 2 of 2 The patient receives the appropriate therapy. Two days after the event, he does not show any progression of neurological deficit. The EKG shows no rhythm abnormalities, and carotid duplex scanning is normal. What is the best next step in the management of this patient? A. Discharge the patient B. Order echocardiography C. Repeat CT scan D. Schedule carotid angiography E. Obtain EEG Explanation: Determining the stroke subtype is important in patients with ischemic stroke in terms of short-term as well as long-term treatment decisions. Thrombotic, embolic and lacunar strokes may require different management approaches that underscore the importance of the underlying pathophysiology. Cardiac evaluation should be performed in almost all patients with brain ischemia. Echocardiography is indicated in patients with suspected embolic stroke and in patients in whom the mechanism of stroke is not clear, especially if carotid studies are negative. Left ventricular systolic dysfunction appears to be more common in patients who present with stroke than in the general population, and many patients with stroke have asymptomatic left ventricular dysfunction that can be detected on echocardiography.

(Choice C) In stable patients with no progression of neurological deficits, repeat CT scan has little diagnostic significance. (Choice D) Conventional angiography is rarely performed nowadays. It may be helpful in patients with fibromuscular dysplasia and carotid dissection, as well as patients undergoing preoperative evaluation of carotid artery disease. Educational Objective: Echocardiography is indicated in patients with suspected embolic stroke and in patients in whom the mechanism of stroke is not clear, especially if carotid studies are negative. 42% of people answered this question correctly.

A 55-year-old Caucasian female is brought to the emergency department by her son because of slurred speech and confusion. These symptoms started several days ago and gradually progressed. Her past medical history is significant for hypertension, bipolar disorder, and deep vein thrombosis. Her son says that the ?mood disorder? runs in the family because his grandmother had the same problem. She takes lithium, which seems to control her symptoms well. One month ago, she was diagnosed with deep vein thrombosis and hospitalized. Warfarin treatment was started at the hospital, and she continues to take this drug with periodic ?blood tests?. The physical examination reveals a mild tremor of her hands and an unstable gait. She is oriented to person, but not to time and space. If medication toxicity is responsible for this patient?s current symptoms, which of the following is the most likely additional finding? A. Hormone replacement therapy (HRT) for hot flushes B. Hydrochlorothiazide for hypertension treatment C. Low INR value on routine coagulation studies D. Right-sided hemiparesis with slight motor aphasia E. Bilateral hemianopsia on visual field testing Explanation: Lithium is widely used as an effective agent to treat bipolar disorder. It has a low therapeutic index. An overdose may result due to intentional poisoning or other factors. The mortality is 25% in patients with acute lithium poisoning and 8% in patients on maintenance dose. Manifestations of lithium toxicity are diverse: gastrointestinal, renal, endocrine, dermal, cardiovascular, hematological, as well as neurological abnormalities have been described. A significant number of patients suffers irreversible neurological damage. Typical neurological symptoms include lethargy, confusion, tremors, fasciculations and ataxia. This scenario demonstrates how various factors can affect serum lithium levels and precipitate lithium toxicity. Any factor that decreases renal excretion, such as renal insufficiency or effective volume depletion (e.g., diuretics or congestive heart failure) may be responsible. (Choice A) HRT may be responsible for deep vein thrombosis in this patient, but the question asked about the current neurological symptoms. (Choice C) A low INR value on routine coagulation studies indicates inadequate anti-coagulation. (Choices D and E) Right-sided hemiparesis would suggest a stroke, and bilateral hemianopsia on visual field testing would most probably indicate the presence of an intracranial mass lesion in this patient. Educational Objective: Any factor that decreases renal excretion may be responsible for the increase of serum lithium levels. 25% of people answered this question correctly. The following vignette applies to the next 2 items A 61-year-old male with hypercholesterolemia, diabetes mellitus type 2 and hypertension comes to the office because of constipation. He has hyperlipidemia, hypertension and diabetes mellitus controlled with atorvastatin, hydrochlorothiazide and rosiglitazone, respectively. He denies alcohol intake, but admits smoking two packs of cigarettes daily for the last several years. Colonoscopy performed one year ago was normal. Lab investigations reveal a serum calcium level of 14.2 mg/dL and an albumin level of 4.0 gm/dL. Item 1 of 2 What is the most likely cause of this patient?s hypercalcemia? A. Primary hyperparathyroidism

B. Secondary hyperparathyroidism C. Drug-induced D. Malignancy E. Diabetic nephropathy Explanation: This patient has a longstanding history of smoking; therefore, he has higher chances of developing lung cancer. Patients with squamous cell carcinoma of the lung can produce parathyroid hormone related protein (PTHrP), which can cause hypercalcemia. It is sometimes difficult to differentiate primary hyperparathyroidism from hypercalcemia seen during malignancy. Typically, the serum calcium level in patients with malignant lung tumors is much higher than those with primary hyperparathyroidism. This patient?s serum calcium level is 14.2 mg/dL, and is most likely due to a malignancy. (Choice A) Primary hyperparathyroidism is one of the most common causes of hypercalcemia in an outpatient setting. As stated above, the level of serum calcium is lower in patients with primary hyperparathyroidism compared to patients with malignancy-induced hypercalcemia. Calcium levels above 12mg/dL are rare. (Choice B) Secondary hyperparathyroidism due to renal failure or vitamin D deficiency is not typically associated with hypercalcemia. Patients with secondary hyperparathyroidism can sometimes develop autonomously functioning nodules of the parathyroid gland, which is then called tertiary hyperparathyroidism. Hypercalcemia can occur in the presence of very high serum PTH levels in tertiary hyperparathyroidism. (Choice C) HCTZ can cause hypercalcemia, but the elevations in serum calcium levels are usually minimal. (Choice E) Diabetic nephropathy per se is not associated with hypercalcemia. Educational objectives: The three most common causes of hypercalcemia include primary hyperparathyroidism, malignancyinduced hypercalcemia and vitamin D-induced hypercalcemia. In hypercalcemia secondary to malignancy, the serum calcium levels are generally very high. 34% of people answered this question correctly. Item 2 of 2 The physical examination of the patient is essentially unremarkable. What is the most appropriate next step in management? A. Serum PTH measurement B. Bone scan C. CT scan of the chest D. Chest x-ray E. Renal biopsy Explanation: Hypercalcemia can be divided in to two broad categories: (1) parathyroid dependent and (2) parathyroid independent. Examples of parathyroid dependent hypercalcemia include primary or tertiary hyperparathyroidism, lithium-induced hypercalcemia, and familial hypocalciuric hypercalcemia. Parathyroid independent causes include malignancies, vitamin D toxicity, granulomatous disease, and milk-alkali syndrome. Malignancy-induced hypercalcemia is generally due to the secretion of PTHrP, but other mechanisms have also been described, particularly in hematological malignancies.

These categories can be distinguished by measuring the serum parathyroid hormone levels; therefore, the most appropriate next step in this case is to do a parathyroid estimation to help classify this patient's hypercalcemia as parathyroid dependent or independent. Specific PTHrP assays are commercially available and are usually ordered with PTH if malignancy is highly being considered in the differential diagnosis. (Choices B, C, D and E) Bone scan, CT scan, chest x-ray, and renal biopsy are not indicated at this point. In majority of endocrinological disorders, biochemical confirmation is required before any imaging is performed. Educational Objective: Hypercalcemia can be divided in to two broad categories: (1) parathyroid dependent and (2) parathyroid independent. The most appropriate next step in management is to do a parathyroid estimation. A 36-year-old African-American man was brought to the hospital because of fever, chills, productive cough, and shortness of breath. He also has night sweats, loss of appetite, and weight loss. He is unemployed and homeless; he was found in the street and brought to the hospital by some concerned policemen. He has no other medical problems. He drinks 6-8 bottles of beer daily. His family history is not significant. He has no medications. He has no known drug allergies. His temperature is 38.9 C (102 F), pulse is 104/min, and respirations are 22/min. His pulse oximetry is 92% at room air. His chest x-rays show a left upper lobe alveolar infiltrate. He is admitted and started on clindamycin and gatifloxacin. On his fourth hospital day, he is breathing better and has more appetite, but he continues to be febrile. His laboratory results reveal that one of his sputum samples is positive for acid-fast bacilli (AFB), and he is started on anti-tuberculosis (TB) therapy. The nurses that were taking care of him in the unit approach you to ask about their risk of acquiring tuberculosis (TB). They want you to tell them what they can do about it. Which of the following is the most appropriate course of action? A. Place a PPD in three weeks and evaluate B. Place a PPD now and repeat it in three weeks C. Take a chest x-ray in three weeks D. Start prophylactic therapy with isoniazid (INH) for those who attended him for more than two consecutive days without using a mask E. Take a chest x-ray and place a PPD in three weeks Explanation: The Center for Disease Control (CDC) recommends immediately giving a PPD test to all health care workers (HCW) exposed to a contagious patient with tuberculosis. If the result is negative, the PPD test will be repeated after three weeks to check for any changes that could have been induced by the recent exposure to the bacteria. (Choice D) Prophylactic therapy with INH is not needed at this point, as it is not known if the HCWs were infected or not. (Choices C and E) PPD test must be given before ordering a chest x-ray, unless the HCW has specific respiratory symptoms. (Choice A) The initial PPD must always be given immediately because the HCW can be PPD positive from before, and not due to the most recent exposure. Educational Objective: When a HCW is exposed to a patient with a contagious form of tuberculosis (i.e., laryngeal, bronchial, or pulmonary), the CDC recommends immediate PPD testing to determine the baseline immunologic status; three weeks after, PPD retesting will be given to check for any changes due to the recent TB exposure.

The following Vignette applies to the next 3 items A 25-year-old Caucasian man comes to the emergency department because of persistent pain and limited range in motion of the right wrist. Five hours ago, while playing football, he fell to the ground and landed on his outstretched right hand. He went home after the incident because his hand did not look too swollen and the pain was tolerable. A few hours later, he noticed that he was unable to move his hand, and decided to come to the hospital. He has no other medical problems. He denies the use of tobacco, alcohol or drugs. Examination shows mild swelling in the dorsum of the right hand. The hand cannot be fully flexed, and can be extended only by passive motion due to pain. There is severe tenderness on palpation of the anatomical snuffbox. Item 1 of 3 Which of the following is the most likely diagnosis? A. Colles? fracture B. Chondral fracture C. Wrist sprain D. Scaphoid fracture E. Triangular cartilage injury Explanation: A history of falling on an outstretched hand is characteristic of a scaphoid fracture. It is most commonly seen in young adults between the ages of 15 and 30 years. It results from a fall on the outstretched hand, resulting in severe hyperextension and slight radial deviation of the wrist. Accompanying symptoms are complete (or greater than 50%) loss in range of motion of the wrist joint, severe pain, and stiffness. Physical examination reveals tenderness on palpation of the scaphoid within the anatomic snuffbox. (Choice C) Wrist sprain is characterized by mild pain, stiffness, normal range of motion of the wrist joint, minimal tenderness on palpation of the lunate and scaphoid (navicular) bones, and mild swelling of the dorsum. (Choice B) Chondral fracture is characterized by moderate pain, stiffness, and approximately 20% loss in range of motion of the wrist joint. (Choice E) Triangular cartilage injury is characterized by tenderness of the region distal to the ulnar styloid, which increases in severity with forced ulnar deviation. (Choice A) Patients with Colles? fracture have tenderness in the region located 2 cm below the radioulnar joint. In almost half of the cases, the ulnar styloid separates from the rest of the bone, and lateral view of the wrist reveals a dinner fork deformity. Educational Objective: A history of falling on an outstretched hand, complete (or greater than 50%) loss in range of motion of the wrist joint, severe pain, and tenderness on palpation of the scaphoid within the anatomic snuffbox is diagnostic of scaphoid fracture until proven otherwise. Item 2 of 3 An x-ray of the wrist reveals no evidence of dislocation or fracture. After receiving analgesic therapy, the patient reports that the pain in his wrist has improved. Which of the following is the most appropriate next step in the management of this patient? A. Prescribe oral analgesics, rest for two weeks, and discharge the patient home B. Order a computerized tomography (CT) scan of the right wrist C. Order a magnetic resonance imaging (MRI) study of the right wrist D. Place a cast brace with the hand on dorsal hyperextension

E. Place a cast brace with the hand on dorsal flexion Explanation: Patients with a non-displaced scaphoid fracture can have normal radiographs for up to two weeks after a traumatic incident. Overlooking this lesion may lead to complications such as traumatic arthritis (which results from nonunion of the fracture) and avascular necrosis of the scaphoid bone. A high clinical suspicion for scaphoid fracture warrants the use of further diagnostic studies (i.e., CT scan of the hand, bone scan), even if the initial x-ray results are negative. (Choice A) Analgesic therapy and resting the affected hand are indicated in the management of wrist sprain. (Choice C) MRI studies are used in the diagnosis of triangular cartilage injury. Although it may also be used in this case, its higher cost makes it less appealing than a CT scan, which is enough to make the diagnosis. (Choice E) Cast placement in the management of a patient with uncomplicated Colles? fracture involves immobilization of the hand in the neutral position, or ideally, in the normal volar tilt position. (Choice D) Cast placement with the hand in a hyperextended position is not beneficial for both Colles? and scaphoid fractures. The uncomplicated, undisplaced scaphoid fractures are treated with immobilization in a long or short arm thumb spica cast with the wrist in slight radial deviation and in neutral flexion.

Educational Objective: Although displaced fractures of the scaphoid can be identified in x-rays immediately after trauma, some scaphoid fractures can take one or two weeks before it becomes apparent in x-rays. A high clinical suspicion for scaphoid fracture therefore warrants the use of further diagnostic studies (i.e., CT scan of the hand, bone scan), even if the initial x-ray results are negative. The uncomplicated, undisplaced scaphoid fractures are treated with immobilization in a long or short arm thumb spica cast with the wrist in slight radial deviation and in neutral flexion. 9% of people answered this question correctly. Item 3 of 3 The patient is concerned about the complications that he might develop. Which of the following is the most common complication of his injury? A. Avascular necrosis B. Nonunion C. Malunion D. Infection E. No complications Explanation: The most common complication of scaphoid fractures is nonunion. The other important complication is avascular necrosis. These two complications often result because the blood flows from the distal to proximal portion of the scaphoid bone, and this proximal portion is completely dependent on the distal blood supply. Proximal fractures of the scaphoid therefore require longer immobilization (up to 12 weeks) to ensure adequate healing. Educational Objective: The most common complication of scaphoid fracture is nonunion.

9% of people answered this question correctly. A follow-up study was conducted to assess the effect of circumcision on the incidence of penile carcinoma. A large cohort was investigated. The rate in uncircumcised males was 8/1000, and the rate in circumcised males was 6/1000. The investigators concluded that circumcision may be markedly effective in reducing the incidence of the disease in the population. According to the study results, how many patients should be circumcised to prevent one case of penile carcinoma? A. 100 B. 250 C. 500 D. 1000 E. 5000 Explanation: The "number needed to treat" (NNT) is the number of patients who need to be treated in order to prevent one additional bad outcome. It is an important way to present the results of a study or assess the usefulness of treatment or prophylaxis. Sometimes, it is more convenient for practitioners to use NNT than measures of association (which represents the strength of association, not the practical aspects of treatment efficacy). The calculation of NNT is easy: it is actually the inverse of absolute risk reduction (ARR). ARR = Control group event rate ? Experimental group event rate = 8/1000 ? 6/1000 = 2/1000 NNT = 1/ARR = 1/0.002 = 500 Therefore, according to the study results, 500 males should be circumcised in order to prevent one case of penile carcinoma. Educational Objective: NNT is the number of patients who need to be treated in order to prevent one additional bad outcome; it is actually the inverse of absolute risk reduction. The following vignette applies to the next 2 items A 21-year-old Caucasian male presents to your office with a non-pruritic rash. He says that he noticed it in the spring season, and it got worse during the summer. His past medical history is insignificant. He does not smoke or consume alcohol, and denies any recreational drug use. He has been sexually active with two partners during the last year, and he uses condoms occasionally. The physical examination reveals multiple circular hypopigmented macules on the chest, back and upper arms. Item 1 of 2 Which of the following is the most likely diagnosis in this patient? A. Pityriasis rosea B. Tinea versicolor C. Secondary syphilis D. Eczema E. Rocky mountain spotted fever Explanation: Tinea versicolor is a fungal infection of the skin that is caused by the dimorphic yeast Pityrosporum orbiculare (also known as Malassezia Furfur). The clinical picture is usually very characteristic: multiple small circular maculae are observed that may vary in color (white, pink or brown). The rash is typically more prominent in the summer time because the yeast inhibits pigment transfer to keratinocytes and makes the affected skin more demarcated from unaffected tanned skin. The lesions are usually asymptomatic, although mild pruritus may be present. The typical location of the lesions is the upper trunk, but the rash may also involve the upper arms, neck and abdomen.

(Choice A) Pityriasis rosea is characterized by the presence of a herald patch, confinement of the lesions to the central trunk, orientation of the lesion along the lines of cleavage of the skin, and a typical pink color of the lesions. (Choice C) Secondary syphilis commonly involves the hands and feet. (Choice D) Eczema typically affects the extremities and produces scaling and severe pruritus. (Choice E) RMSF is an infectious disease that is characterized by an acute onset and severe systemic symptoms. Educational Objective: In patients with tinea versicolor, multiple small circular maculae are observed which may vary in color (white, pink or brown). 75% of people answered this question correctly. Item 2 of 2 Which of the following is the best treatment for this patient? A. Oral erythromycin B. Topical terbinafine C. Penicillin IM D. Topical corticosteroids E. Oral doxycycline Explanation: The treatment of choice for patients with tinea versicolor is topical anti-fungal therapy. Any anti-yeast topical agent can be used, including terbinafine, clotrimazole and ketoconazole. The success rate with topical anti-fungal agents exceeds 80%. With extensive disease or recalcitrant infection, oral antifungals are preferred (ketoconazole, itraconazole or fluconazole). (Choice A) Oral erythromycin is effective in patients with pityriasis rosea. (Choices C) Penicillin is used for the treatment of syphilis. (Choice E) Doxycycline is used in the treatment of patients with RMSF. (Choice D) Topical corticosteroids can be used in patients with eczema. Educational Objective: The treatment of choice for patients with tinea versicolor is topical anti-fungal therapy. 70% of people answered this question correctly. A 65-year-old Caucasian female presents to the emergency department with progressive shortness of breath on minimal exertion and fatigue. She was diagnosed with myelodysplasia two years ago, and has been receiving supportive therapy with frequent RBC transfusions. Her last transfusion was 3 months ago. She experienced an episode of severe pneumonia one year ago that required hospitalization and IV antibiotic therapy. She is taking no medications currently and has no known allergies. Her blood pressure is 120/70 mmHg and heart rate is 95/min. Physical examination reveals pallor. Systolic murmur with intensity of II/VI is heard over the cardiac apex. Laboratory findings are significant for hematocrit of 24% and hemoglobin level of 5.7 mg/dL. You consider RBC transfusion in this patient. Blood grouping and cross-matching are done, but the blood bank is unable to find suitable blood. This is the first time such an incompatibility has occurred. What is the most likely reason for this incompatibility?

A. Autoantibodies B. Alloantibodies C. Anti-Rh (D) antibodies D. Anti-HLA antibodies E. ABO incompatibility Explanation: After blood is ordered for transfusion, the following compatibility testing is usually performed. First, the patient?s ABO and Rh types are determined. After this, the patient's serum is screened for unexpected antibodies, a procedure called pretransfusion antibody screening. Pretransfusion antibody screening is intended to detect any of all clinically significant RBC antibodies. If negative, the patient can be safely transfused. If positive, further investigation is usually warranted to evaluate the identity of the antibody. The major problem that leads to difficulties finding cross-matched blood in patients with a history of multiple transfusions is alloantibodies (e.g., in patients with sickle cell anemia or myelodysplasia). The most commonly implicated RBC antigens in that case are E, L and K. Moreover, these patients tend to develop multiple alloantibodies that make finding compatible blood even more difficult. (Choices C and E) Finding ABO and Rh-compatible blood is usually not a big challenge. (Choice A) Autoantibodies are less likely to cause difficulties in cross-matching in this patient; they are commonly implicated as a cause of the incompatible cross-match in patients with autoimmune anemia and taking certain drugs (e.g., methyldopa and procainamide). (Choice D) HLA allosensitization increases risk of graft rejection in patients awaiting organ or bone marrow transplantation and platelet refractoriness in those requiring subsequent platelet transfusion support. Remember that RBCs do not express HLA antigens. Educational Objective: The major problem that leads to difficulties finding cross-matched blood in patients with a history of multiple transfusions is alloantibodies. 52% of people answered this question correctly.

A 31-year-old multigravid woman presents to your office for counseling on the various methods of contraception. She has four children, and had three elective abortions in the past. She wants the most effective method of contraception, but does not want it to be "permanent". Her past medical history is insignificant. Which of the following is the most appropriate method of contraception for this patient? A. Oral contraceptives B. Implantable or injectable contraceptives C. Diaphragms/cervical caps D. Male condoms E. Spermicides Explanation: Implantable and injectable contraceptives, including implantable levonorgestrel and depot medroxyprogesterone acetate, have the lowest rate of pregnancy (does not exceed 2-3%). The actual pregnancy rate may be as low as 0.3%. (Choice A) Oral contraceptives are also very effective, but the actual pregnancy rate is higher than with implantable and injectable contraceptives due to inconsistent or incorrect use. (Choices C and D) Barrier methods of contraception such as diaphragms, cervical caps, and male condoms are moderately effective due to inconsistent or incorrect use. The actual pregnancy rate is 12-14%. (Choice E) Spermicides, if used alone, have a high failure rate. Educational Objective: Implantable and injectable contraceptives, including implantable levonorgestrel and depot medroxyprogesterone acetate, have the lowest rate of pregnancies among nonpermanent methods of contraception. 62% of people answered this question correctly. A 26-year-old white woman who is a long distance runner comes to your office. She trains daily and participates frequently in marathons and other long-distance competitive events all around the country. She denies smoking or alcohol drinking, and says that she follows a healthy, low-fat diet. Her last menstrual period was three years ago, but this is not a problem for her as she does not want to get pregnant "at least in the next five years." After an extensive work-up, her amenorrhea was attributed to exercise. She is 168 cm (5?6??) tall and weighs 50 kg (110 lbs). Her blood pressure is 100/60 mmHg, pulse is 58/min, and respirations are 14/min. Physical examination shows no abnormality. You alert the patient that her amenorrhea poses her in risk of another condition. Which of the following conditions are you referring to? A. Thyroid disease B. Hyperprolactinemia C. Early menopause D. Osteoporosis E. Hypocholesterolemia Explanation: Females who maintain a lower weight or body mass index (BMI) due to the sport or activity that they regularly engage in (e.g. ballet dancers, gymnasts, and runners) may become hypoestrogenic and present with exercise-induced amenorrhea. They are at special risk of developing osteopenia, and even osteoporosis. Spontaneous fractures have been reported in these types of patients, with osteoporosis or osteopenia confirmed through bone densitometry. Treatment consists of improving

caloric intake; if this is not possible, patients are started on hormonal replacement with oral contraceptives and supplementation with calcium and vitamin D. (Choice E) Paradoxically, there is a tendency toward hypercholesterolemia, not the opposite. (Choice C) There is no risk of early menopause, but exercise-induced amenorrhea can mimic early menopause because of estrogen deficiency. (Choices A and B) Exercise-induced amenorrhea is a diagnosis of exclusion. Thyroid hormone and prolactin level determinations are necessary in the evaluation of this patient because one/both of these may be the underlying cause (but not a complication or aftermath) of the amenorrhea. Educational Objective: Exercise-induced amenorrhea is due to a decrease in the pulsatile secretion of LH, which leads to a decline in estrogen production. It can lead to osteopenia, osteoporosis, breast and vaginal atrophy, mild hypercholesterolemia, and infertility. 72% of people answered this question correctly. The following vignette applies to the next 2 items A 62-year-old Caucasian man comes to the physician because of progressive weight loss and right lower extremity weakness for the last two months. On further questioning, he tells you that he has been having frequent leg cramps in the right leg. These cramps are most prominent in the morning, after he wakes up. He also has difficulty with swallowing and chewing. He denies any problems with bowel or bladder function. His other medical problems include hypertension and hypercholesterolemia. He does not use tobacco. His family history is not significant. His vital signs are within normal limits. The physical examination reveals weakness in his right lower leg, atrophy and fasciculation of the thigh and calf muscles, a hyperactive knee jerk, and tongue fasciculations. The rest of his neurological examination, including the sensory examination, is normal. Item 1 of 2 Which of the following is the most likely diagnosis? A. Binswanger?s disease B. Vascular dementia C. Multiple sclerosis D. Amyotrophic lateral sclerosis E. Brain stem glioma Explanation: Amyotrophic lateral sclerosis (ALS) is the most common form of progressive motor neuron disease. It is a relentlessly progressive disorder that involves both the lower motor neurons (consisting of anterior horn cells in the spinal cord and brainstem neurons innervating the bulbar muscles) and upper or corticospinal motor neurons. At its onset, ALS may involve selective loss of function of only upper or lower motor neurons, but it ultimately causes progressive loss of both upper and lower motor neurons. The initial sign of the disease with lower motor neuron involvement is an insidiously developing asymmetric weakness, usually first evident distally in one of the limbs. Patients may disclose a history of recent development of cramping with volitional movements that typically occur in the early morning hours (e.g. while stretching in bed). Weakness is associated with progressive wasting, atrophy of muscles, and spontaneous twitching or fasciculations of motor units. Involvement of the bulbar muscles leads to difficulty with chewing and swallowing, as well as fasciculations of the face and tongue.

ALS with prominent corticospinal involvement is characterized by hyperactivity of muscle-stretch reflexes (tendon jerks) and frequent spastic resistance to passive movements of the affected limbs. Ocular motility, sensory, bowel, bladder, and cognitive functions are preserved, even with advanced disease. (Choice B) Vascular dementia is characterized by the presence of behavioral disturbances and cognitive deficits associated with clinical or radiographic evidence of a stroke. The decline in the level of cognition is relatively abrupt and progresses in a stepwise fashion. (Choice A) Binswanger?s disease is a type of vascular dementia that involves white matter infarcts. Patients with this disease usually present with apathy, agitation, and bilateral corticospinal or bulbar signs. (Choice C) Multiple sclerosis is usually seen in younger females with two or more clinically distinct episodes of CNS dysfunction. Patients typically have sensory, visual, or bladder and bowel dysfunction. (Choice E) Brain stem tumors may compress the cervical cord and produce weakness, fasciculations in the upper limbs, and spasticity in the legs. The presentation may closely resemble ALS; however, absence of pain or sensory changes and normal bowel and bladder function in the patient favor ALS. Educational Objective: ALS should be suspected in patients who present with progressive weakness accompanied by both upper and lower motor neuron deficits. Ocular motility, sensory, bowel, bladder, and cognitive functions are preserved, even with advanced disease. 79% of people answered this question correctly. Item 2 of 2 Which of the following treatment has been shown to be beneficial in these patients? A. Riluzole B. Corticosteroids C. IV immunoglobulins D. Plasmapheresis E. Surgery Explanation: Riluzole is a glutamate inhibitor that is currently approved for the management of amyotrophic lateral sclerosis. Although it cannot arrest the underlying pathological process, it may prolong survival and delay the need for a tracheostomy. (Choices B, C, and D) Amyotrophic lateral sclerosis is a neurodegenerative disease. Agents such as corticosteroids, intravenous immunoglobulins, and cyclophosphamide, have no role in its management. These agents may be useful in immunologically-mediated neurological diseases such as multiple sclerosis and Guillain-Barr yndrome. (Corticosteroids are used to treat acute exacerbations of multiple sclerosis, while plasmapheresis and intravenous immunoglobulins are the main treatment modalities of Guillain-Barr yndrome.) Educational Objective: Riluzole is a glutamate inhibitor that is currently approved for the management of amyotrophic lateral sclerosis. 42% of people answered this question correctly. A 65-year-old female is admitted to the intensive care unit because of unstable angina. She has a history of hypertension, diabetes, hyperlipidemia, and osteoporosis. Cardiac catheterization is

performed and shows severe, triple-vessel, coronary artery disease. She has had recurrent angina while in the hospital. A coronary artery bypass graft procedure is planned. Your intern reported to you that her thyroid-stimulating hormone level in a blood sample drawn in the emergency department is 36 mU/mL (0.35 ?5.0 mU/mL is normal). Which of the following is the most appropriate next step in the management of this patient? A. Call the cardiac surgeon to cancel the surgery B. Proceed with cardiac surgery C. Start her on liothyronine (T3) orally and postpone surgery D. Start liothyronine (T3) intravenously and proceed with cardiac surgery E. Start her on a usual dose of levothyroxine (T4) and perform surgery when euthyroid

Explanation: The patient needs to undergo cardiac surgery on an emergent basis. Hypothyroidism, even if severe, is not a contraindication to emergency surgical procedures. Postoperative mortality rate is not increased in patients with hypothyroidism undergoing major surgeries. These hypothyroid patients, however, are at a slightly higher risk for developing postoperative ileus and hyponatremia, and are more prone to oversedation with narcotic medications. (Choice A) There is no need to cancel the scheduled surgery. (Choices C and D) Liothyronine is not indicated in the treatment of hypothyroidism . (Choice E) Levothyroxine (T4) is the treatment of choice in patients with hypothyroidism; however, due to this patient's age and significant coronary artery disease, levothyroxine should be started on a lower dose. Higher doses of levothyroxine in elderly patients or in patients with significant coronary artery disease will increase myocardial oxygen demand and can precipitate myocardial infarction. In such cases, lower doses of levothyroxine should be started postoperatively when the coronary blood flow improves. The dose is then gradually increased to achieve euthyroidism. Educational Objective: Hypothyroidism, even if it is severe, is not a contraindication to an emergency surgical procedure. 40% of people answered this question correctly. A 27-year-old Caucasian man comes to the physician for a routine health maintenance examination. He has no other medical problems. His father had colon cancer. His vital signs are within normal limits. Examination shows no abnormalities. Which of the following information is the most important to determine the patient's risk for colon cancer? A. Age of onset of colon cancer in his father B. History of cigarette smoking C. Type of diet D. Associated symptoms and signs E. History of alcohol use Explanation: The history of colon cancer in a first-generation relative is the most important risk factor. This risk is further increased if more than one first-generation relatives are affected, or if colon cancer develops at an age younger than 55. Colon cancer occurs at an earlier age in patients with familial adenomatous polyposis and familial nonpolyposis colorectal cancer, which are very important risk factors for colon cancer. (Choice C) A diet rich in fiber, folate, and calcium has a protective effect against colon cancer. (Choice B and E) Both alcohol use and cigarette smoking have been associated with an increased risk of colon cancer; however, early development of colon cancer in a first-generation relative is a more important risk factor. (Choice D) Associated signs and symptoms give little information regarding the risk of colon cancer in the above patient. Educational Objective: Early development of colon cancer in a first-generation relative is a very important risk factor for evaluating a patient for colon cancer prevention. 73% of people answered this question correctly.

A 42-year-old Caucasian man comes to see you in the office with symptoms of dyspepsia and heartburn for the last two years. His symptoms are progressively getting worse. He has tried lifestyle changes, over-the-counter antacids, and ranitidine earlier, but these have not significantly relieved his symptoms. He denies any history of dysphagia, odynophagia, weight loss, or gastrointestinal bleeding. He has no other medical problems. He works as an assistant to a lawyer. He smokes a pack of cigarettes daily and drinks 6 to 8 cups of coffee daily. His physical examination is unremarkable. Which of the following is the most appropriate next step in the management of this patient? A. Start him on a trial of a proton pump inhibitor B. Obtain esophageal manometry C. Schedule him for an ambulatory pH monitoring D. Refer to a gastroenterologist E. Refer to a surgeon for antireflux surgery Explanation: The patient has symptoms consistent with gastroesophageal reflux disease. This results from a combination of excessive gastric acid reflux and impaired clearance of the acid by the esophagus. Mild symptoms can usually be managed by simple lifestyle and dietary modifications, antacids, and non-prescription H2 blockers. Patients with more severe and prolonged symptoms, and those who fail initial management usually require more aggressive therapy, which involves a trial of proton pump inhibitors for at least eight weeks. All the available agents in this class have similar efficacy when used in equivalent doses. (Choices B and C) Esophageal manometry and pH monitoring have very limited and specific roles. Manometry is used to facilitate the placement of ambulatory pH probes and to guide antireflux surgery. Ambulatory pH monitoring helps to confirm the diagnosis in patients with persistent symptoms without the endoscopic evidence of mucosal damage. It also helps to evaluate refluxassociated pulmonary and upper respiratory symptoms. (Choice D) The patient should be referred to a gastroenterologist for an upper endoscopy if he fails a trial of proton pump inhibitors. Endoscopy should be considered early in the course if patients present with symptoms of dysphagia, odynophagia, weight loss, or gastrointestinal bleeding. (Choice E) Antireflux surgery is too invasive, and should not be recommended unless all other modalities have failed. Educational Objective: A trial of proton pump inhibitors should be used in patients who have failed other conservative therapies. Endoscopy should be done early if the patient complains of dysphagia, odynophagia, significant weight loss, and GI bleeding. 61% of people answered this question correctly. The following vignette applies to the next 2 items An 82-year-old Caucasian female comes to the office for the evaluation of a low-grade fever and rash on her left flank area for the past two days. She has a past medical history of hypothyroidism, diabetes mellitus, rheumatoid arthritis, osteoporosis, and a recent hospital admission for urosepsis. Her medications include aspirin, ciprofloxacin, alendronate, levothyroxine, glipizide, low-dose prednisone, calcium, and vitamin D supplementation. She has no known drug allergies. She does not smoke or drink alcohol. On physical examination, her temperature is 37.8C(100F), heart rate is 92/min, blood pressure is 124/72 mmHg, and respiratory rate is 14/min. Examination of the rash reveals the presence of multiple small vesicles over an erythematous base in the thoracic 9-10 nerve root distribution. Item 1 of 2

Which of the following is the most appropriate step in the management of this patient? A. Perform a Tzanck smear. B. Obtain a complete blood count with differential count. C. Start the patient on acyclovir. D. Obtain a skin biopsy to confirm the diagnosis. E. Obtain blood for varicella-zoster virus serology. Explanation: The presentation and physical findings of the patient are classic for herpes zoster, which occurs as a result of reactivation of latent varicella-zoster virus (VZV) infection in the sensory or dorsal nerve root ganglion. The population at greatest risk of developing herpes zoster includes the elderly and immunosuppressed patients (i.e., patients on cancer chemotherapy or chronic steroid therapy, transplant recipients, and HIV-infected patients). The disease is characterized by the development of a painful vesicular rash in the distribution of specific nerve roots. Most patients have a prodromal phase consisting of fever, malaise, headache, and localized pain, followed by the development of a vesicular eruption several days later. The vesicles typically appear along the thoracic or lumbar dermatomes, and these evolve into pustules before crusting in about 7-10 days. The pain is the most consistent symptom, and can persist for days to months after the rash resolves. The treatment of choice is oral acyclovir. The goal of antiviral treatment is to promote early healing, and to prevent or reduce the duration and severity of complications. If started within 48 hours of the onset of rash, acyclovir has been shown to improve acute pain and prevent the development of postherpetic neuralgia in a significant number of patients. (Choices A, B, D and E) Herpes zoster is a clinical diagnosis. Treatment should be started based on the typical signs and symptoms. Diagnostic workup such as Tzanck smear, skin biopsy or serologies is not necessary. Educational Objective: Antiviral therapy with acyclovir is the mainstay of treatment for herpes zoster. Treatment should be initiated based on clinical suspicion within 48 hours of the onset of rash for maximum efficacy. 70% of people answered this question correctly. Item 2 of 2 The appropriate step was taken for the patient. Over the next 24 hours, she developed a similar rash in other parts of her body. Due to severe pain, she was admitted to the hospital. Which of the following is the most appropriate next step in the management of this patient? A. Isolate the patient until the lesions are crusted. B. Isolate the patient until the pain resolves. C. There is no need to isolate the patient. D. Isolate the patient only if varicella-zoster virus serology is positive. E. Isolate the patient only if she has extensive or disseminated herpes zoster. Explanation: All major organizations, including the Center for Disease Control and Prevention (CDC) have recommended guidelines regarding the isolation of patients with varicella-zoster virus (chicken pox) infections. Varicella-zoster virus is a highly contagious virus that is transmitted from person to person by droplets and close contact. The risk of transmission and development of varicella is extremely high in susceptible patients. Most people, including healthcare workers, have a history of varicella and have immunity against the infection; however, nosocomial transmission of the infection can occur in all susceptible adults, healthcare workers, and immunocompromised patients. This can result in severe disease and complications such as bacterial superinfection of the lesions, varicella pneumonia, cerebellar ataxia, meningoencephalitis, and death. Infection in pregnant woman may lead to the development of congenital varicella syndrome in newborn infants.

Herpes zoster is also considered infectious, although the risk of transmission is not as high as that of varicella. According to the current recommendations, all patients with disseminated herpes zoster should be maintained on contact and airborne isolation until all skin lesions are dry and crusted. All healthcare workers without a history of varicella-zoster virus infection are strongly encouraged to avoid contact with patients with varicella or herpes zoster infection. (Choice B) The pain of herpes zoster may persist for weeks to months after all the lesions have resolved. It is not necessary to isolate the patient until that point. (Choice C) All patients with herpes zoster should be isolated to prevent nosocomial spread to susceptible healthcare workers. (Choice D) There is no need to check VZV serology in patients with herpes zoster. (Choice E) All hospitalized patients with localized dermatomal herpes zoster should be isolated to prevent nosocomial spread. Educational Objective: Airborne and contact isolation should be maintained for all patients with disseminated herpes zoster infection until all their lesions are dry and crusted. Contact precautions are indicated in hospitalized patients with localized herpes zoster, but these are not currently recommended in a community setting. 53% of people answered this question correctly. A 35-year-old white male comes to the emergency department. He is complaining of a severe, rightsided abdominal pain that started two hours ago, and has been gradually worsening. The pain now radiates to his groin, and is described as 10/10 in intensity and sharp in nature. The patient feels nauseated, but has not actually vomited. He denies any history of trauma to his flank. His past medical history is not significant. His family history is significant for three first-degree relatives with kidney stones. On examination, he has very mild tenderness on the right lumbar region. Murphy?s sign is negative. Urinalysis results are as follows: Blood: moderate RBC: 25-30/HPF WBC: 0-2/HPF Nitrite: negative Leukocyte esterase: negative After relieving the patient?s pain with intravenous hydromorphone (Dilaudid), important next step in the management of this patient? A. Order ultrasonogram B. Order x-ray of kidney, ureter, and bladder (KUB) C. Order an intravenous pyelogram (IVP) D. Order a CT scan of the abdomen and pelvis E. Order a urine culture and sensitivity Explanation: Nephrolithiasis should strongly be suspected in any patient who presents with atraumatic flank pain, particularly if abdominal tenderness is absent, and urinalysis shows hematuria. The history of the patient is very suggestive of nephrolithiasis. Studies have shown the superior sensitivity and specificity of non-contrast helical CT scan over other diagnostic modalities in the diagnosis of nephrolithiasis. The standard procedure is to obtain 8 mm slices using the CT scan, however decreasing the size to 3-5 mm slices will further increase sensitivity and specificity. An added advantage is that this modality is fast, widely available, and enables doctors to search for other what is the most

possible etiologies of the pain without unnecessary delay, if the CT scan results turn out to be negative. (Choice A) USG of the kidneys, ureters and bladder (KUB) is the procedure of choice for cases wherein IVP or CT scan cannot be obtained or is contraindicated (i.e. pregnant patients); however, it may miss small stones, and sometimes miss even ureteral stones. (Choice B) X-Ray of KUB without IVP detects calcium-containing stones only. Since 20% of kidney stones do not contain calcium, failure to visualize these types of stones makes this procedure unfavorable. (Choice C) IVP is a very sensitive and specific procedure. However, it has a few disadvantages compared to CT scan. IVP is relatively more time-consuming, and is only slightly less expensive than CT scan. These reasons, along with the possible risk for allergic reactions occurring secondary to the dye, have resulted to IVP being used as a second line investigation after CT scan. (Choice E) The rapid onset of symptoms and urinalysis results (negative for WBC, leukocyte esterase and nitrites) has made this case unlikely to be a urinary tract infection. Ordering a urine culture and sensitivity at this point is not needed, and may add an unnecessary delay to the treatment of this patient. Educational Objective: Non-contrast helical CT scan is the gold standard for the diagnosis of nephrolithiasis, followed by an IVP. In pregnant patients, or in individuals where you want to avoid the radiation exposure, choose the USG as the procedure of choice. 30% of people answered this question correctly. A 65-year-old Caucasian male comes to the clinic in September for a follow-up visit regarding an episode of acute sinusitis he developed after a lingering cold. He reports some improvement in his symptoms of mild headache, cough, nasal congestion, and increased sinus drainage. He attributes this improvement in symptoms to his usage of over-the-counter decongestants and the amoxicillin prescribed a few days ago. He received his last dose of tetanus toxoid 5 years ago. Given his recent susceptibility to illness, one of his close friends suggested he seek out any recommended vaccinations at this visit. To address this issue, which of the following is the most appropriate next step? A. Provide reassurance but do not vaccinate today B. Administer tetanus toxoid C. Administer tetanus toxoid and influenza vaccine D. Administer influenza vaccine and pneumococcal vaccine today E. Administer influenza vaccine today and pneumococcal vaccine next week Explanation: The three most commonly administered vaccinations for adults include the influenza vaccine, the pneumococcal vaccine, and tetanus toxoid. Influenza season includes the winter months (with a peak incidence in January), and the vaccine is only effective for the year in which it is given. The indications for influenza vaccination in adults include the following: 1. 50 years of age and older 2. chronic respiratory, cardiac, or metabolic conditions 3. immunosuppressed state 4. residents of long-term care facilities 5. pregnant women 6. healthcare workers and people in prolonged contact with adults in these high-risk categories

On the other hand, the pneumococcal vaccine provides protection against Streptococcus pneumoniae, a pathogen responsible for a substantial proportion of community-acquired pneumonias and subsequent hospitalizations. Moreover, pneumococcal pneumonia is a known common complication of influenza infection. For these reasons, pneumococcal vaccination is recommended for all adults 65 years of age and older. Adults who were vaccinated more than five years ago (while younger than 65 years of age) are eligible for one additional re-vaccination. Concurrent administration of the pneumococcal and influenza vaccines is safe and effective; therefore, this patient should receive both vaccines. (Choices B and C) Tetanus toxoid should be administered to the following individuals: 1. those who have serious or dirty wounds (e.g., puncture wounds or wounds contaminated with soil, feces, or saliva) and who received their last dose more than five years ago 2. those who have clean minor wounds and who received their last dose more than ten years ago Since this patient has not incurred a wound recently, he does not need to receive tetanus toxoid. (Choice E) Resolving sinusitis and antibiotic usage are not contraindications to vaccination; therefore, there is no need to postpone administration of the pneumococcal vaccine until next week. (Choice A) Providing only reassurance is not advisable, since this patient is clearly prone to upper respiratory tract infections, and is in the recommended age group for both influenza and pneumococcal vaccinations. Educational Objective: Pneumococcal vaccine should be administered to adults 65 years of age and older. Influenza vaccine should be administered to adults 50 years of age and older, as well as those adults in various high-risk categories (e.g., pregnant women, healthcare workers, the immunosuppressed or chronically ill). 72% of people answered this question correctly. A 7-year-old Caucasian boy is brought to the emergency department due to a two-day history of fever, chills, cough and progressive shortness of breath. The cough is productive of greenish-yellow sputum. The patient was diagnosed with cystic fibrosis after three episodes of severe pneumonia two years ago. He is currently not taking any medications. He has no known allergies, and all his immunizations are up-to-date. His blood pressure is 110/60 mmHg, pulse is 130/min, temperature is 38.9C(102F) and respirations are 32/min. Physical examination reveals decreased breath sounds over the left lower lobe and scattered wheezing bilaterally. Chest x-ray shows an infiltrate in the lower left lobe. Which of the following is the best treatment option for this patient? A. Vancomycin B. Azithromycin C. Piperacillin plus tobramycin D. Ampicillin plus clindamycin E. Dicloxacillin Explanation: Severe exacerbations of pulmonary disease in patients with cystic fibrosis usually require IV antibiotic therapy. Ideally, the selection of antibiotics should be done according to the results of the sputum culture. Empirical therapy should cover Pseudomonas aeruginosa as a common pathogen, and it usually includes at least two drugs. The most popular combination is an aminoglycoside (e.g., tobramycin) with an anti-pseudomonal semi-synthetic penicillin (e.g., piperacillin or ticarcillin). Sometimes, a third generation cephalosporin or a carbapenem is used. (Choice A) If methicillin-resistant S. aureus infection is diagnosed, vancomycin is employed.

(Choices B and E) Azithromycin or dicloxacillin can be used as oral agents in less severe cases in patients with susceptible flora. Azithromycin may be a useful long-term agent to prevent acute exacerbations. (Choice D) Ampicillin plus clindamycin is not a good option for empirical therapy in this patient. Educational Objective: The most commonly used empiric antibiotic therapy in cystic fibrosis patients with severe exacerbations of pulmonary disease is an aminoglycoside with anti-pseudomonal semi-synthetic penicillin. 78% of people answered this question correctly. A 7-year-old Caucasian boy is brought to the emergency department due to a two-day history of fever, chills, cough and progressive shortness of breath. The cough is productive of greenish-yellow sputum. The patient was diagnosed with cystic fibrosis after three episodes of severe pneumonia two years ago. He is currently not taking any medications. He has no known allergies, and all his immunizations are up-to-date. His blood pressure is 110/60 mmHg, pulse is 130/min, temperature is 38.9C(102F) and respirations are 32/min. Physical examination reveals decreased breath sounds over the left lower lobe and scattered wheezing bilaterally. Chest x-ray shows an infiltrate in the lower left lobe. Which of the following is the best treatment option for this patient? A. Vancomycin B. Azithromycin C. Piperacillin plus tobramycin D. Ampicillin plus clindamycin E. Dicloxacillin Explanation: Severe exacerbations of pulmonary disease in patients with cystic fibrosis usually require IV antibiotic therapy. Ideally, the selection of antibiotics should be done according to the results of the sputum culture. Empirical therapy should cover Pseudomonas aeruginosa as a common pathogen, and it usually includes at least two drugs. The most popular combination is an aminoglycoside (e.g., tobramycin) with an anti-pseudomonal semi-synthetic penicillin (e.g., piperacillin or ticarcillin). Sometimes, a third generation cephalosporin or a carbapenem is used. (Choice A) If methicillin-resistant S. aureus infection is diagnosed, vancomycin is employed. (Choices B and E) Azithromycin or dicloxacillin can be used as oral agents in less severe cases in patients with susceptible flora. Azithromycin may be a useful long-term agent to prevent acute exacerbations. (Choice D) Ampicillin plus clindamycin is not a good option for empirical therapy in this patient. Educational Objective: The most commonly used empiric antibiotic therapy in cystic fibrosis patients with severe exacerbations of pulmonary disease is an aminoglycoside with anti-pseudomonal semi-synthetic penicillin. 78% of people answered this question correctly. A healthy, 44-year-old, Middle-eastern, male taxi driver had a motor vehicle accident. He was a restrained driver. You come to help him, along with the paramedics, and find him unresponsive with spontaneous respirations. His blood pressure is 100/60 mmHg, pulse is 112/min, and respirations are 12/min. Examination shows clear lung fields and normal first and second heart sounds. He could not

be extricated from the car. Suddenly, he starts to cough and spit some blood. Which of the following is the most appropriate course of action? A. Intubate immediately B. Suction the mouth C. Do cricothyrotomy D. Do the jaw lift maneuver E. Stabilize the spine first Explanation: There is a considerable risk of neck trauma during most motor vehicle accidents. The first thing to always do in such trauma cases is to secure the airway. Whenever the airway needs to be protected, it is advisable to use the jaw lift maneuver to avoid further complications if there is any neck injury. (Choice B) After securing the airway, the paramedics can proceed to suction the mouth to prevent any possible aspiration. (Choices A and C) The patient has spontaneous respirations. There is no indication for immediate intubation or cricothyrotomy at this point. (Choice E) Stabilization of the spine is also done after securing the airway. Educational Objective: Whenever there is an accident with possible neck trauma, the airway must be secured through the jaw lift maneuver to avoid further strain in the neck area. Cricothyrotomy may be necessary if there is complete obstruction of the airway. If the patient is unconscious, completely unresponsive, unable to protect the airway, not breathing or coughing, he or she will need to be intubated. 20% of people answered this question correctly.

A group of investigators is exploring trends in the incidence of coronary artery disease (CAD) in a large population. They studied several factors in relation to the incidence of the disease. The relationships between the cigarette sales in the population (average number of cigarette packs sold per person per day) and the incidence of CAD (per 1000 people) over time are given on the figure below. Which of the following is the best statement concerning the results of the study? A. No stable trend is present in the incidence of CHD over time. B. Quitting smoking decreases the risk of CHD for a person living in this population. C. Cigarette sales were stable over time in the population. D. No individual-level conclusions can be ascertained from the given plot . E. Discordant changes in cigarette sales and incidence of CHD are present. Explanation: No individual-level conclusions can be ascertained from the given plot. The study described is a typical ecologic (correlational) study. It is important to remember that ecologic studies give population-level information, not individual-level information! Applying population-level information to an individual level may lead to a bias called ecologic fallacy. (Choices A and C) The given plot illustrates the temporal trends of cigarette sales and incidence of CHD; it is clear that the incidence of CHD, as well as cigarette sales, steadily declined over time in the population. (Choice E) Because both the incidence of CHD and cigarette sales declined progressively, the changes over time are concordant, not discordant. (Choice B) It cannot be concluded directly from this plot that quitting smoking decreases the risk of CHD for a person living in this population. A similar trend could be possibly observed by plotting personal computer usage in this population (increasing over time); however, it also cannot be concluded that personal computer use decreases the risk of CAD. Educational Objective: It is important to remember that ecologic studies give population-level information, not individuallevel information! 35% of people answered this question correctly. A 2-year-old boy is brought to the physician because of a delay in language development. According to his mother, the only words that he says are "mama" and "dada." He runs well and walks up and down stairs without assistance. He builds a tower of seven blocks and turns pages one at a time. He was delivered vaginally at 39 weeks? gestation; his mother's pregnancy was uncomplicated, but prenatal visits were not regularly followed. His past medical history is unremarkable. His mother is worried and asks if such development is normal. Which of the following is the most appropriate next step in the boy's management? A. Reassurance B. Watchful waiting C. Hearing test D. Screening for TORCH organisms E. CT scan of the head Explanation: This child has normal developmental milestones, except for language which is delayed. A 2-year-old child is supposed to know a 50+ word vocabulary, use 2- to 3-word phrases, and follow two step commands. Delayed speech development may indicate an underlying hearing impairment; therefore, such children should be referred for audiologic assessment. Other candidates for hearing evaluation

include children with a family history of hereditary childhood hearing loss, history of meningitis, history of recurrent or persistent otitis media with effusion for more than three months, documented intrauterine infections, craniofacial anomalies, and use of ototoxic medications such as aminoglycoside. Hearing loss early in life, even mild or unilateral, may affect speech and language development, social and emotional development, and academic achievements. Early identification is thus mandatory for a better prognosis; hence, screening programs have been widely and strongly advocated. Although the American Academy of Pediatrics endorses universal screening, at present, systematic screening is implemented in 32 states only. The etiology of hearing impairment depends on whether the hearing loss is conductive or sensorineural. The most common cause of conductive hearing loss in children is presence of fluid in the middle ear. Other causes include tympanic membrane perforation (trauma or infection), ossicular discontinuity (infection, cholesteatoma, and trauma), tympanosclerosis, and congenital anomalies of the external ear canal or middle ear components. As for sensorineural hearing loss (SNHL), it may be congenital or acquired. The most common infectious cause of congenital SNHL is CMV. Other less common congenital infectious causes include toxoplasmosis, syphilis, and rubella. Postnatal infectious causes include group B streptococcus and bacterial meningitis. Hearing loss can also be genetically determined; it may be autosomal dominant or recessive, isolated or in a syndromic association with other anomalies. (Choices A and B) Reassurance and watchful waiting are inappropriate because hearing impairment should be identified promptly. (Choices D and E) The other diagnostic methods may be more or less useful for evaluation of this child; however, hearing impairment should first be confirmed, and its type identified by a hearing test. Educational Objective: Delayed speech development may indicate an underlying hearing impairment and should prompt referral of the patient for audiologic assessment. Other candidates for hearing evaluation include children with a family history of hereditary childhood hearing loss, history of meningitis, history of recurrent or persistent otitis media with effusion for more than three months, documented intrauterine infections, craniofacial anomalies, and use of ototoxic medications such as aminoglycoside. 61% of people answered this question correctly. A 78-year-old Caucasian man with end stage esophageal cancer is admitted to the hospital with severe malnutrition and failure to thrive. The patient's caretaker tells you that he has not been able to eat or drink for the last six weeks. His weight dropped from 160 pounds to 120 pounds during that time. The cancer has spread to his lungs and liver. The patient expresses his wishes to not receive any further treatment for the cancer, and specifies that he does not want any heroic measures or interventions done to keep him alive. Keeping his current clinical condition in mind, you think about offering hospice care to the patient. When is the most appropriate time to refer this patient to hospice care? A. Tell the patient that you would not be able to treat her if she refuses to sign the consent. B. He should be referred to hospice care two weeks prior to his anticipated death. C. He should be referred to hospice care two months before his anticipated death. D. He should be referred to hospice care after he has cleared his hospital bills. E. CT scan of the head Explanation: Hospice care is usually provided to terminally ill patients who have a predicted life expectancy of six months or less. The largest population of patients receiving hospice care consists of cancer patients.

Other patients receiving hospice care have terminal medical conditions such as endstage cardiomyopathy, endstage chronic obstructive pulmonary disease, or pulmonary fibrosis. Hospice care is based on the principle of providing compassionate and comprehensive support and care, which includes, but is not limited to psychological, social, nursing, and palliative medical care to a dying patient. It also provides support and respite care to the family members or caregivers of the terminally ill patient. Hospice care is provided by a multidisciplinary team, which includes a registered nurse, nurse?s aide, social workers, chaplains, and a hospice physician who closely coordinates with the patient?s attending physician. It is usually provided at the patient?s own home (home hospice care), but it can also be given as an inpatient hospice care for patients who are not functionally independent. (Choices B and C) There is a general tendency to delay hospice care to eligible patients by their caregivers or attending physicians. This leads to a shorter length of stay in hospice programs, and deprives the patients of the full benefits available to them. The patient in the above scenario has advanced esophageal cancer with an expected survival of less than six months. He should be referred to hospice care program now, or as soon as possible. Educational Objective: Patients with advanced metastatic cancers or other terminal illnesses with an expected life expectancy of less than six months should be evaluated for hospice care. 95% of people answered this question correctly. A 32-year-old pregnant woman who recently migrated from Eastern Asia is brought to the hospital by her husband. She is on her 37th week of gestation. She has been having regular uterine contractions for the last four hours. Examination reveals that she is in active labor. She agrees to be admitted for delivery. You then explain to the patient and her husband the various possible methods of delivery and the complications that can arise from them. The couple appears to understand everything that you explain. When you ask the patient to sign the consent sheet for treatment, she becomes reluctant, and tells you that women in their culture are not allowed to sign any papers if their husbands are alive. She asks you to have her husband sign the consent sheet for her treatment. Which of the following is the most appropriate reaction to the patient's request? A. If the child does not cry after she finishes feeding him. B. Ask the hospital not to admit the patient if she refuses to sign the consent. C. Tell her that you would not be responsible for any malpractice claims if her husband signs the paper. D. Obtain a witness to the above conversation, and treat the patient. E. CT scan of the head Explanation: Informed consent involves a process whereby a patient or his/her surrogate decision-maker makes a healthcare decision based on the physician?s recommendation. Before arriving at a decision, the patient and the healthcare professional work must together as a team. It is the physician?s obligation to provide all the medical facts accurately to the patient or the surrogate decision maker in accordance with good medical practice. It is important to ensure that the information provided to the patient is adequate, and is easily and completely comprehended. The patient or surrogate then exercises the right of autonomy and makes the healthcare decision. In the above vignette, the patient was provided with adequate information. She and her husband appear to understand the information completely. She has a right to choose a surrogate decision maker, such as her husband, to make healthcare decisions on her behalf. Both the patient and her husband want to proceed with the planned treatment as well. The patient should be treated appropriately even if she refuses to sign the consent for treatment herself. (Choices A and B) Refusal of admission or treatment of the patient is unethical and inappropriate.

(Choice C) This statement is not true. The physician has to assume responsibility for all actions which directly involves the management of his patient. Educational Objective: Informed consent is considered effective when sufficient information is provided by a physician, and is easily and adequately comprehended by the patient or their surrogate decision maker. 80% of people answered this question correctly.

A healthy 28-year-old Middle-Eastern woman comes to the physician for a routine health maintenance examination. She has just delivered a 3.5 Kg (7.7 lb) baby boy three days ago. This was her first pregnancy. She has no complaints. She is a housekeeper. She does not use tobacco, alcohol, or drugs. Examination shows no abnormalities. She is breastfeeding, and she asks you what the best way is to know if the baby is being fed properly. Which of the following is the most accurate answer? A. If the child does not cry after she finishes feeding him. B. Make sure you feed him using both breasts. C. Make sure you feed him every 2 hours. D. Make sure you feed him every 4 hours. E. Make sure you feed him anytime he cries. Explanation: Most breastfeeding authorities, as well as the CDC (Center for Disease Control), recommend feeding infants at least every four hours, as well as to look for early signs of hunger (like hand or finger suckling or arms movement toward the mouth) to determine proper breastfeeding. (Choice A) Breastfeeding has a soporific effect in the infant. If the baby does not cry, this does not mean that he was properly fed. (Choice C) Some infants may need feeding every two hours; however, there is no scientific evidence to support this method. (Choice E) Newborns cry for many different reasons, and not always because they need to be fed. Furthermore, it is not advisable to wait until the newborn cries to breastfeed him. (Choice B) It is a good practice to feed the newborn using both breasts, as complete emptying of the breast will increase the milk?s nutritional qualities. Unfortunately, sometimes there may be a decreased production of milk, and the baby has to be fed using artificial formula. Feeding the baby using both breasts will therefore not guarantee the infant?s satiety. Educational Objectives: Every primary care physician should encourage breastfeeding. It is recommended that the infant be fed at least every four hours. The mother should be able to identify early signs of hunger, like suckling of the hand or fingers or arm movements towards the mouth, as preterm or debilitated infants may not be able to cry vigorously or show agitation. 26% of people answered this question correctly. A 14-year-old Caucasian male presents to the pediatrician for a follow-up visit regarding his acne. On his face are numerous comedones and many inflamed papules and pustules, and there is some evidence of mild scarring. At his first visit three months ago for this problem, he was prescribed topical retinoid and benzoyl peroxide, which he has been using regularly. He feels that the acne has improved slightly with this treatment regimen. What is the most appropriate means of managing his condition? A. Add topical erythromycin to the regimen B. Add oral corticosteroids to the regimen C. Begin treatment with oral tetracycline only D. Begin treatment with oral isotretinoin only E. No change in treatment; re-evaluate in three months

Explanation: Although there are no definitive evidence-based guidelines on the treatment of acne, some general principles are observed by most clinicians. Treatment regimens usually progress through several stages, based upon how severe the acne is at presentation and how effective the treatment options are in resolving the symptoms. Mild acne is initially treated with topical retinoid. Moderate acne (or refractory mild acne) is treated with topical retinoid and either benzoyl peroxide or a topical antibiotic. More severe acne (or refractory moderate acne) is treated with: 1) topical retinoid, benzoyl peroxide, and topical antibiotic (Choice A); or 2) systemic antibiotic plus topical retinoid or benzoyl peroxide. Note that systemic antibiotics (Choice C) are not typically used exclusively. The most severe cases of acne should be treated with a systemic antibiotic and topical retinoid or benzoyl peroxide; if there is no response after 3-6 months, then oral isotretinoin (Choice D) is indicated. Corticosteroids (Choice B) are used in patients with excessive adrenal androgen production (e.g., cases of congenital adrenal hyperplasia). Microcomedones take eight weeks to mature, so any acne therapy regimen must be continued for at least this length of time before determining its efficacy. Since this boy has already waited three months, it would be unreasonable to ask him to wait another three months (Choice E) before reevaluating the treatment regimen. Educational Objective: Topical products (e.g., retinoid, benzoyl peroxide, antibiotics) are first-line treatments for mild to moderate acne. Systemic products such as oral antibiotics are reserved for more significant acne, and oral isotretinoin is used for the most severe, refractory forms of acne. 36% of people answered this question correctly. The following vignette applies to the next 2 items A 16-year-old Caucasian boy is brought to the ED via ambulance by his school nurse. The nurse tells you that the boy has not been feeling well for the past two days. She first saw him two days ago when he was complaining of headache and generalized body aches. At that time, she evaluated him, concluded that he was suffering from a viral prodrome, and prescribed him acetaminophen for his symptoms. Earlier this morning, he had two episodes of vomiting while sitting in his classroom, and he was extremely drowsy. In the ED, he is minimally responsive. He experiences another episode of emesis and is emergently intubated by the emergency room resident for airway protection. Subsequent evaluation, including a lumbar puncture for CSF analysis, reveals that he is suffering from acute meningococcal meningitis. Item 1 of 2 Which of the following contacts should be offered antibiotic prophylaxis to prevent the development of meningococcal meningitis? A. A single intramuscular injection of penicillin G B. His teachers at school C. Emergency room physicians and other staff members D. Paramedics who brought the patient to the emergency room E. No change in treatment; re-evaluate in three months Explanation: Neisseria meningitidis is the most common cause of acute bacterial meningitis in children and adolescents from ages 2-18 years. The portal of entry of this bacteria is via the nasopharynx.

Although colonization of Neisseria meningitidis in the nasopharynx is relatively common and affected patients are usually asymptomatic, it can cause mucosal invasion with systemic bacteremia, seeding of the meninges, and subsequent development of potentially fatal meningitis in predisposed individuals. Meningococcal meningitis is a devastating illness and is associated with significant morbidity and mortality. The mortality rates are even higher in adult patients with meningitis. It is therefore recommended that antibiotic chemoprophylaxis be used to prevent the spread of meningococcal meningitis. All close contacts must receive antibiotic prophylaxis. These include people who live in the same household and persons who have had prolonged, close contact with the patient. In addition, all healthcare workers (including physicians) who have had direct exposure to the respiratory secretions of the patient (e.g., while suctioning or intubating the patient) should receive chemoprophylaxis. (Choices B, C, & D) Those persons with brief exposure to the patient (e.g., either at school or work, or during transportation) do not require antibiotic prophylaxis. Antibiotic prophylaxis is not indicated if the exposure to the patient is brief or short, and if there is no direct exposure to the respiratory or nasopharyngeal secretions. It is only required for persons who have had prolonged contact in a closed environment with the patient. Educational Objective: All persons with prolonged, close contact or direct exposure to the respiratory secretions of a patient with meningococcal meningitis should receive antibiotic prophylaxis to prevent the spread and development of meningitis. 37% of people answered this question correctly. Item 2 of 2 You identify and contact the persons in whom the antibiotic prophylaxis is indicated. Which of the following is the most appropriate therapy for chemoprophylaxis against meningococcal meningitis? A. A single intramuscular injection of penicillin G B. Inhaled tobramycin for two days C. A single dose of intravenous vancomycin D. Oral rifampin E. Two days of cefazolin Explanation: As described above, antibiotic prophylaxis is indicated in all the close contacts of patients with meningococcal meningitis. Neisseria meningitidis tends to colonize the nasopharynx and invade the mucosa, thereby causing systemic bacteremia and subsequent meningitis in predisposed patients. Antibiotic prophylaxis is used to eliminate the nasopharyngeal colonization of meningococci, and is effective in preventing the development of meningitis in close contacts. Oral rifampin (600 mg every 12 hours for a total of four doses) is the recommended regimen for chemoprophylaxis. Other antimicrobials that can be used for chemophrophylaxis include fluoroquinolone (a single 500 mg dose of ciprofloxacin), sulfonamides, ceftriaxone, and minocycline. (Choices A, B, & C) Penicillin G, inhaled tobramycin, cefazolin, or intravenous vancomycin is not recommended for chemoprophylaxis to prevent the spread of meningococci. Educational Objective: Oral rifampin is the recommended antimicrobial for chemoprophylaxis against the spread of meningococcal meningitis. The other commonly used regimens are single oral dose of ciprofloxacin or single intramuscular injection of ceftriaxone. 87% of people answered this question correctly.

A 26-year-old Caucasian woman who is suffering from cystic fibrosis comes to the physician. She has recently changed her boyfriend and is trying to conceive these days. Her boyfriend does not have any medical problem. She is a non-smoker and non-alcoholic. Which of the following statements is true regarding fertility in a patient with cystic fibrosis? A. Prazosin B. There is a 20% chance that you may not become pregnant. C. There is a 50% chance that you may not become pregnant. D. Only males are infertile; you will not have any problem. E. Air pollution

Explanation: Chances of infertility in a female with cystic fibrosis are 20% percent. This increased risk is due to secondary amenorrhea caused by malnutrition, and is also due to thick, tenacious cervical mucus. Chances of infertility in a male with cystic fibrosis are 95%. Spermatogenesis is normal in a patient with cystic fibrosis, but sperm transport is impaired because of impaired development of the Wolffian duct. Educational Objective: Chances of infertility in a female with cystic fibrosis are 20% percent. Chances of infertility in a male with cystic fibrosis are 95%. 27% of people answered this question correctly. The following Vignette applies to the next 2 items A 33-year-old Caucasian female presents to your office with a six-month history of cold intolerance. When the weather is cold, her third and forth fingers on both hands get numb and turn blue. She says, ?I wear gloves to keep my hands warm, but it does not always work?. Her past medical history is significant for non-ulcer dyspepsia treated with omeprazole. She does not smoke or consume alcohol, and denies any recreational drug use. She has no known drug allergies. She works as a nurse at a local hospital and considers her job moderately stressful. She is sexually active with her husband and uses condoms for contraception. Physical examination findings are within normal limits. Item 1of 2 Which of the following medications is the best initial treatment for this patient? A. Prazosin B. Nifedipine C. Verapamil D. Aspirin E. Topical nitroglycerin Explanation: This patient presents with symptoms suggestive of Raynaud phenomenon, an exaggerated vascular response to cold temperature or emotional stress. Calcium antagonists are considered the first-line pharmacological therapy for these patients, but it should be mentioned that not all calcium antagonists are equally effective. Most dihydropyridine calcium channel blockers (e.g., nifedipine and amlodipine), as well as diltiazem, are proven to work well in these patients. Many other vasodilator agents have been tried to treat this condition. (Choice C) Verapamil does not seem to be effective or at least equally effective as calcium antagonists.

(Choice A) Alpha-adrenergic blockers such as prazosin are effective; however, patients eventually become refractory to these agents after prolonged usage. (Choice E) Nitroglycerin is usually used as an adjunct agent to calcium antagonist therapy in patients with severe Raynaud phenomenon. It is not recommended as the first-line agent. (Choice D) Some authors recommend adding aspirin to the treatment regimen in patients with severe ischemia and necrosis to prevent platelet aggregation; however, aspirin should be used with caution because it can actually worsen vasospasm by blocking the synthesis of vasodilatory prostaglandins. Educational Objective: Calcium antagonists, typically nifedipine or amlodipine, are considered the first-line pharmacological therapy for patients with Raynaud phenomenon. 59% of people answered this question correctly.

Item 2 of 2 The patient returns in one month and complains that the treatment ?does not help much.? In addition, she has started to experience periodic joint and muscle pain. She says that she has been adherent to the medication and has tried to avoid going out in cold weather as much as possible. Her husband even gave up smoking because you had warned her that passive smoking may worsen her condition. Which of the following is the best next step in the management of this patient? A. Increase the dose of the medication B. Add a second agent C. Order arterial Doppler ultrasonography D. Obtain ANA and RF E. Reassure and observe Explanation: Raynaud phenomenon may be idiopathic (also called Raynaud disease) or secondary to other conditions such as connective tissue diseases, vascular lesions, medications, etc. When managing patients with Raynaud phenomenon, potential precipitating factors such as medications and environmental factors (e.g., frostbite, vibration) should first be excluded. The presence of symptoms suggestive of systemic disease (arthralgias and myalgias) and resistance to treatment (such as in this patient) warrant further evaluation; this may include ANA, RF, CBC, blood chemistry, urinalysis and measurement of complement levels. (Choice E) Patients are diagnosed with primary Raynaud phenomenon when there are no historic clues to a secondary condition, normal physical findings and no ischemic digital lesions. No specialized studies are indicated in these patients. (Choice C) In patients who have symptoms suggestive of vascular lesions (e.g., asymmetric involvement and deficient pulses), further evaluation with digital plethysmography, arterial Doppler ultrasonography or angiography should be considered. (Choices A and B) It is not reasonable to change the treatment without first excluding a systemic disease that may be responsible for the condition. Educational Objective: The presence of symptoms suggestive of systemic disease (arthralgias, myalgias, fever, and weight loss) in patients with Raynaud phenomenon warrants further evaluation; this may include ANA, RF, CBC, blood chemistry, urinalysis and measurement of complement levels. 67% of people answered this question correctly. A 26-year-old male who was initially admitted in the intensive care unit three days ago for diabetic ketoacidosis (DKA) is transferred to the medical floor after getting the appropriate initial treatment. He has had type-1 diabetes for the past two years, and is currently on glargine (14 units at bedtime) and lispro (8 units three times a day before meals). His blood glucose levels during the last two days have ranged from 120?200 mg/dl. He started tolerating oral feeding yesterday. Twenty-four hours after being transferred to the medical floor, he started to have a spiking fever up to 102?F (39? C), and developed foul-smelling nasal discharge and pain in his right paranasal area. On examination, the nasal mucosa was inflamed, and there was a black discoloration in the antero-inferior aspect. Which of the following is the most effective agent for the treatment of this infection? A. Amphotericin B B. Terbinafine C. Itraconazole D. Voriconazole E. Flucytosine

Explanation: Patients with DKA are predisposed to get mucormycosis, an infection caused by fungi from the class Zygomycetes. Although this class of fungi does not cause infections in a normal host, it may do so in patients with DKA. Because of the acidic environment and high free iron, these pathogens can infect the nasal mucosa, and invade the surrounding structures rapidly. The common clinical features are fever, facial swelling, maxillary pain and tenderness, nasal discharge, ophthalmoplegia and headache. The diagnosis is suggested by the characteristic clinical picture, and confirmed by performing a biopsy of the infected tissues. Treatment consists of debridement of the necrotic tissues and amphotericin B. Despite aggressive treatment, the mortality remains high. (Choices C and D) Voriconazole and itraconazole are not effective against mucormycosis. (Choice B) Terbinafine is approved for the treatment of onychomycosis of the fingernails. It is not used for systemic fungal infections. (Choice E) Flucytosine is used in cryptococcal meningitis in combination with amphotericin B. It is a weak anti-fungal agent on its own. Educational Objective: Rhinocerebral mucormycosis is best treated with surgical debridement and intravenous amphotericin. 78% of people answered this question correctly. A 53-year-old African-American female is seen for her annual physical examination. She denies any symptoms, except for minimal hot flashes. She had menopause one year ago. Since then, she started having some menopausal symptoms, which have been gradually improving during the last two months. She has not been on any hormone replacement therapy. Her past medical history is unremarkable. She is currently on no medication. Her father has premature coronary artery disease. There is no history of cancer or osteoporosis in her family. She exercises regularly, and does not smoke or drink alcohol. Her mammogram and pelvic exam were performed eight months ago, and were reported to be normal. Her physical examination is unremarkable. What is the next best step in the management of this patient? A. Start her on hormone replacement therapy B. Start calcium and vitamin D supplementation C. Start low-dose alendronate D. Tell her to discontinue the exercise program E. Start raloxifene Explanation: Most females rapidly lose considerable bone mass following menopause. Lifestyle modification, including weightbearing exercises and optimum calcium and vitamin D supplementation, prevents postmenopausal bone mass loss and possibly reduces the risk of fragility fractures. The National Academy of Science recommends daily supplementation of elemental calcium (1200 mg) and vitamin D (400 to 800 international units) in women after 50 years of age. This can be easily achieved by taking one multivitamin tablet (which contains 400 international units of vitamin D) with two tablets of calcium/vitamin D (containing 600 mg of elemental calcium and 200 international units of vitamin D) everyday. (Choice D) Exercise with weightbearing leads to improvement in bone mass, and should be continued in this patient; however, exercise should be individualized in patients with significant osteoporosis, since they have a higher risk of fracture. In such patients, stressful exercises should be avoided as much as possible. (Choices C and E) Short-term use of low-dose estrogen for menopausal symptoms does not appear to be harmful; however, hormone replacement therapy fell out of favor following a recently published study of the women?s health initiative because of increased, undesirable cardiovascular and other side effects. This patient has minimal hypoestrogenic symptoms in the form of hot flashes, which are likely to improve with hormone replacement therapy; however, because her symptoms appear to improve with time, hormone replacement therapy could be possibly avoided. This patient does not

have any risk factors for osteoporosis, except for postmenopausal stature. There is no documentation of low bone density (osteopenia or osteoporosis). There is also no history of fragility fractures. Treatment with bone-specific antiresorptive therapy (e.g., alendronate, raloxifene) is not warranted at this time. Educational Objective: Calcium and vitamin D supplementation should be recommended in all postmenopausal females to protect bones. A 56-year-old female presents with lethargy, weight loss, nausea, vomiting, and constipation for the past two months. She is three years postmenopausal, and has no long-standing medical problems. She currently takes no prescription medications. She denies allergies to drugs. She does not use tobacco or alcohol. Her family history is positive for osteoporosis and hypertension. Her heart rate is 64/min and blood pressure is 124/66 mmHg. Her height is 5?3" (160cm) and she weighs 138 lbs (63kgs). She has mild pallor. The thyroid gland is normal. The chest is clear on auscultation. Heart sounds are normal. Neurological examination reveals hung-up ankle jerks. Abdominal examination is unremarkable. Laboratory tests reveal a hemoglobin of 11.2 g/dL, and a hematocrit of 34%. RBCs are normochromic and normocytic. Serum sodium is 129 mEq/dL , while the rest of the basic chemistries are normal. TSH is 0.35 mU/ml (normal 0.35-5.0 mU/ml) and free T4 is 0.5 mg/dl (normal 0.8 to 1.8 mg/dl). What is the next best step in this patient?s care? A. Measurement of antithyroid antibodies B. TRH stimulation test C. Serum T3 levels D. Measurement of cortisol with cosyntropin stimulation E. Thyroglobulin levels Explanation: The patient most likely has central hypothyroidism, as suggested by borderline TSH and low free T4 levels. Central hypothyroidism can also have slightly elevated TSH levels; however, this elevated TSH is not biologically active. Imaging studies (preferably a MRI) are done to look for a mass lesion in the sellar area. When central hypothyroidism is suspected, it is pragmatic to rule out concomitant central adrenal insufficiency before beginning therapy. Patients with primary adrenal insufficiency (Addison?s disease) can have increased TSH levels without hypothyroidism due to the loss of an inhibitory effect of glucocorticoids on TSH secretion. Conversely, a patient on high-dose glucocorticoid treatment will have a transient decrease in TSH levels. Treating a patient with central hypothyroidism and concomitant adrenal insufficiency could precipitate adrenal crisis. A cosyntropin stimulation test is the most preferred initial test to screen for adrenal insufficiency. (Choice A) Antithyroid antibodies are useful in the evaluation of goiter and primary hypothyroidism, but not suspected central hypothyroidism. (Choice B) After the advent of sensitive TSH assays, TRH stimulation test is now rarely performed. It still haa a role in differentiating isolated pituitary resistance to thyroid hormones (normal response) from TSH secreting pituitary tumors (diminished response). (Choice C) T3 level determination has no role in the evaluation of hypothyroid patients. (Choice E) Thyroglobulin level determination is useful in the follow-up management of patients with differentiated thyroid cancers, and in the evaluation of suspected factitious thyrotoxicosis. Educational Objective: Evaluation of the adrenal status is warranted in patients with suspected central hypothyroidism. 31% of people answered this question correctly.

A 65-year-old white female presents to the emergency department with a sudden onset of severe periumbilical pain, nausea and vomiting. She denies any fever, diarrhea, or constipation. She denies any urinary complaints. Her last menstrual period was fourteen years ago. Her past medical history is significant for coronary artery disease, a recent acute anterior myocardial infarction, hypertension, and hyperlipidemia. She had an appendectomy at the age of eighteen. She has a history of 20 pack years of smoking. She is an occasional alcohol drinker. Her temperature is 36.7C(98F), blood pressure is 120/76 mm Hg, pulse is 120/min, irregular, and respirations are 16/min. The abdomen is soft, non-distended and non-tender, with normal bowel sounds. There is no rebound tenderness or rigidity. Pelvic examination is within normal limits. EKG shows atrial fibrillation. CBC shows a WBC count of 14,000/microL. Serum sodium is 140 mEq/L, serum K is 4.0 mEq/L, blood glucose is 110mg/dL, BUN is 20 mg/dL, and serum creatinine is 0.7 mg/dL. The serum lipase is normal, while the serum amylase is slightly elevated. Urinalysis is normal. Liver function panel is also normal. Fecal occult blood test is negative. Plain abdominal x-ray fails to show any abnormalities. Which of the following is the most likely diagnosis of this patient? A. Intestinal obstruction B. Acute mesenteric ischemia C. Acute colonic ischemia D. Acute intermittent porphyria E. Acute diverticulitis Explanation: This patient is most likely suffering from acute mesenteric ischemia secondary to embolic phenomenon caused by atrial fibrillation. This patient also had a recent anterior myocardial infarction. Embolization usually throws clots into the superior mesenteric artery, resulting in mesenteric ischemia. Patients usually present with a sudden onset of periumbilical abdominal pain. The abdominal examination is initially normal, and the pain felt by the patient is usually out of proportion to the initial examination findings. (Choice A) Patients with intestinal obstruction usually have a history of absolute constipation. Plain abdominal radiographs will show air fluid levels. (Choice C) Patients with acute colonic ischemia usually have a more lateralized abdominal pain with tenderness, along with a history of bloody diarrhea or hematochezia. This usually results from hypovolemic states or transient ischemia to the bowel. This is not because of the obstruction of flow to the colon. (Choice D) Acute intermittent porphyria is a rare disease. Patients will usually have recurrent episodes of abdominal pain that is precipitated by several factors. The symptoms are usually manifested much earlier (usually after puberty), and occur in the presence of other manifestations such as neuropathy, hyponatremia, etc. (Choice E) Acute diverticulitis usually causes a more localized pain (LLQ). The physical exam would show tenderness in the LLQ. Patients may also present with diarrhea. Educational Objective: A sudden onset of severe periumbilical pain that is out of proportion to the physical examination findings, and occurs in a setting with risk factors such as atrial fibrillation, congestive heart failure, and peripheral vascular disease, should always raise the suspicion for acute mesenteric ischemia. 90% of people answered this question correctly. A 53-year-old asymptomatic, Caucasian woman comes to the physician for a routine health maintenance examination. She has no complaints; however, she is concerned about her risk of developing osteoporosis. She is an executive secretary with a sedentary lifestyle. Her last menstrual period was two years ago. She drinks coffee for at least five times daily, and soda at least twice daily.

She is a vegetarian. She does not use drugs. She has smoked one pack of cigarettes daily for 35 years. She drinks 1-2 ounces of alcohol daily. Her father has hypertension. Her mother had a stroke. She takes no medication. She has no known drug allergies. Examination shows no abnormalities. Which of the following interventions will be the most beneficial to decrease her risk for osteoporosis? A. Stop drinking alcohol B. Stop drinking coffee and soda C. Stop smoking D. Start hormonal replacement therapy E. Start an exercise program Explanation: Although the most important risk factors for osteoporosis are the non-modifiable ones (e.g., age, race, family history, and history of previous fractures), it is also necessary to know the modifiable factors in order to provide interventions which can lower a patient's risk for the disease. For instance, smoking half a pack of cigarettes a day has a mild negative effect on osteoporosis, but a smoker who consumes one or more packs of cigarettes a day will benefit more from smoking cessation than from any other intervention. (Choice D) Although hormonal replacement therapy (HRT) can effectively decrease a patient?s risk of osteoporosis, HRT can unfortunately increase the risk of breast and endometrial cancer. Some recent studies even show that it may also increase the risk of cardiovascular disease. (Choices A, B and E) Lack of exercise, alcohol, coffee, and soda ingestion mildly increase the risk of osteoporosis compared to menopause or smoking. Educational Objective: There are multiple risk factors for osteoporosis, the majority of which are non-modifiable. The interventions that have proven to offer more benefit are: smoking cessation (if the patient smokes one pack or more of cigarettes a day), raloxifene, calcium and vitamin D supplementation, and discontinuation of medications such as heparin or glucocorticoids. HRT is extremely effective in osteoporosis prevention, but widespread use is not advisable because of the risk of malignancy or cardiovascular disease. 39% of people answered this question correctly. The following vignette applies to the next 3 items A healthy 8-year-old Caucasian boy is brought to the office by his mother because of redness and copious, purulent discharge from his right eye. The school nurse informed the mother that the boy's disease is very contagious. The boy denies fever or blurred vision. His vital signs are within normal limits. Examination shows conjunctival erythema and yellow exudates in his right eye. The conjunctiva of the left eye is also erythematous, but there are no secretions. The mother is especially concerned about the possible complications of the disease. Item 1 of 3 Which of the following complications is most likely to occur with his condition? A. Endophthalmitis B. Nothing C. Recurrence D. Cataracts E. Keratitis Explanation: This is a typical presentation of bacterial conjunctivitis. increased risk of a corneal lesion in the form of keratitis. When the infection is severe, there is an

(Choice B) Although this is a self-limiting disease in the majority of situations, this is not always true. (Choice A) Endophthalmitis is acquired through an open wound to the eye (post-cataract surgery, post-traumatic) or a systemic route, and not usually through conjunctivitis. (Choice D) Cataracts have not been described as complications of this disease. (Choice C) Recurrence is only possible if the patient has bad hygienic habits or if he acquires viral conjunctivitis. Educational Objective: Bacterial conjunctivitis is usually a self-limiting disease; however, it must be treated because there is a small but real risk of keratitis, which can lead to visual impairment. 34% of people answered this question correctly. Item 2 of 3 Which of the following is the most appropriate pharmacotherapy? A. Ciprofloxacin eye drops B. Ofloxacin eye drops C. Antihistamine eye drops D. Corticosteroid eye drops E. Erythromycin ointment Explanation: Erythromycin ointment or sulfa drops are the first line agents for the treatment of uncomplicated bacterial conjunctivitis, as these cover most organisms. (Choices A and B) Fluoroquinolones are preferred for contact lens wearers and corneal ulcers because of their activity against Pseudomonas. These should not be used as first line agents in uncomplicated bacterial conjunctivitis because of the risk of the emergence of resistance. (Choice C) Antihistamines are used for allergic conjunctivitis, which is characterized by watery discharge and usually involves both eyes. (Choice D) Primary care physicians should not prescribe corticosteroids, as these can cause sightthreatening complications in patients with bacterial conjunctivitis and herpes keratitis. Educational Objective: Erythromycin ointment or sulfa drops are the first line antibiotic agents for the empirical treatment of patients with bacterial conjunctivitis. 53% of people answered this question correctly. Item 3 of 3 The appropriate evaluation was performed. The mother wants to know when her son can go back to school. What is the best response? A. One week from now. B. This is not contagious; he can go to school tomorrow. C. He should stay at home until the discharge has cleared. D. With the treatment, he can go to school tomorrow . E. He has to wait at least 2 weeks from now and will have to be reevaluated by me. Explanation:

Bacterial conjunctivitis is very contagious. In such cases, the best thing to do is to keep the child at home until the discharge has cleared. However, this may not be feasible for many patients, especially for those with work/demanding occupations. In those patients, at least 24-hours of topical antibiotic therapy should be applied before returning to work. Educational Objective: Bacterial conjunctivitis is very contagious. Patients should be advised to stay home until the discharge has cleared. If this is not possible, patients should have had at least 24 hours of topical antibiotic therapy before returning to work/school. A 55-year-old African-American man is brought to the emergency department because of an episode of hematemesis. He has a history of alcoholic cirrhosis with ascites for the last two years. In the emergency room, he is appropriately treated, and has no new episodes of bleeding. He is admitted to the hospital for observation and further work-up. His upper GI endoscopy reveals the presence of esophageal varices with stigmata of recent bleeding. Which of the following is the most likely complication that he is at risk of developing during his hospitalization? A. Spontaneous bacterial peritonitis B. Renal failure C. Congestive heart failure D. Disseminated intravascular coagulopathy E. Hemolytic uremic syndrome

Explanation: Patients who are admitted to the hospital because of recent variceal bleeding are at an increased risk of developing complications during their hospitalization. The principal complications in these patients that lead to increased mortality are: infections, hepatic encephalopathy, and renal failure. The most common complication is the development of an infection, which usually occurs as a urinary tract infection, spontaneous bacterial peritonitis, respiratory infection, aspiration pneumonia or primary bacteremia. A large number of trials have evaluated the efficacy of prophylactic antibiotics in cirrhotic patients that were hospitalized for variceal bleeding. All these trials have suggested a decreased incidence of infectious complications with the use of prophylactic antibiotics. The optimal choice of antibiotics and the duration of therapy remain unclear. The currently preferred regimen is the use of a fluoroquinolone (ofloxacin, norfloxacin, or ciprofloxacin) agent for 7-10 days. (Choice B) The development of renal failure in these patients can have multiple etiologies, such as acute tubular necrosis (ischemic or toxic) or precipitation of hepatorenal syndrome; however, this complication is not as common as the development of an infection. (Choice C) Congestive heart failure is uncommonly seen in these patients. (Choice D) Disseminated intravascular coagulopathy occurs rarely in patients with acute variceal hemorrhage, and is usually seen in the presence of underlying infections. (Choice E) Hepatorenal syndrome is a complication of cirrhosis and a variceal bleed. Hemolytic uremic syndrome (HUS) is associated with Shiga toxin-producing Escherichia coli (E. coli O157:H7), and is not associated with cirrhosis. Educational Objective: Bacterial infections can develop in up to 50% of patients who are hospitalized for acute variceal bleeding; therefore, these patients should be treated prophylactically with antibiotics. The preferred regimen involves the use of a fluoroquinolone (ofloxacin, norfloxacin, or ciprofloxacin) agent for 7-10 days. 70% of people answered this question correctly. A 17-year-old, Caucasian, female, college student presents to the student health center on a Monday morning after having unprotected intercourse during a party the last Saturday evening. She is concerned about getting pregnant and requests contraception. She has been sexually active with two partners over the last several months, but her partners used condoms. Her past medical history is insignificant. She is currently not taking any medications. She does not smoke, but consumes alcohol occasionally. She confessed that she has tried several recreational drugs, but does not remember their names. The physical examination is insignificant. Which of the following is the best contraception modality for this patient? A. High dose estrogen B. Ethinyl estradiol plus levonorgestrel C. Levonorgestrel D. Copper intrauterine device E. Reassurance and no intervention Explanation: Levonorgestrel is the recommended method of emergency contraception if used soon enough after an unprotected sexual intercourse. It has maximal efficacy when used within the first 12 hours after intercourse, good efficacy within 48 hours, and appears to work up to 120 hours after intercourse. (Choices B and A) Levonorgestrel seems to be more effective than ethinyl estradiol /levonorgestrel combination or high dose estrogen. It also has fewer side effects. One controlled trial showed that

levonorgestrel prevents 85% of expected estrogen/progesterone combination).

pregnancies

(compared

to

57%

prevented

by

(Choice D) The copper intrauterine device is an effective emergency contraception tool that can be used if a patient presents more than 120 hours after unprotected intercourse. Educational Objective: Levonorgestrel is the recommended method of emergency contraception if used soon enough after an unprotected sexual intercourse. It has maximal efficacy when used within the first 12 hours after intercourse, good efficacy within 48 hours, and appears to work up to 120 hours after intercourse. The copper intrauterine device is an effective emergency contraception tool that can be used if a patient presents more than 120 hours after unprotected intercourse. 30% of people answered this question correctly. A 9-year-old Caucasian boy is brought to your office with a two-month history of periodic clear rhinorrhea, nasal and orbital itching, and dry cough. His mother notes that the symptoms usually appear after the child plays with a neighbor?s dog. His past medical history is significant for an episode of severe pneumonia experienced two years ago. Physical examination reveals excoriations of the external nares. The lungs are clear on auscultation. Which of the following is the next best step in the management of this patient? A. Avoidance of the dog B. Nasal decongestant sprays C. Nasal corticosteroids D. Second-generation antihistamines E. Cromolyn sodium Explanation: This patient presents with signs and symptoms suggestive of allergic rhinitis. The neighbor?s dog is the most likely allergen trigger for this patient?s condition. Dog or cat dander is one of the most common identifiable allergen in patients with allergic rhinitis. Allergen avoidance is traditionally considered the first step in the management. (Choice C) If the allergen is not identified, or if the symptoms persist after avoidance measures, nasal corticosteroids should be the first-line therapy. (Choices D and E) Second-generation antihistamines and cromolyn are less effective than topical steroids. (Choice B) Nasal decongestive sprays are not recommended because tachyphylaxis usually develops, and rebound phenomena may result. Educational Objective: When possible, avoidance of the antigen triggers is typically the first step in the management of allergic disorders. 75% of people answered this question correctly. A 23-year-old Caucasian female has just delivered a 9 lb healthy baby at 37 weeks gestation thirty minutes ago. Although the labor was prolonged, the baby and placenta were delivered spontaneously, without application of forceps. Even after placental delivery, the patient is bleeding continuously. She has lost approximately 1.5 liters of blood. IV ringers lactate is administered. Her temperature is 36.7 C (98 F), blood pressure is 100/60 mmHg, pulse is 102/min, and respirations are 18/min. Which of the following is the most appropriate next step in the management of this patient?

A. Intravenous oxytocin B. Pelvic examination C. Observation D. Bimanual uterine massage E. Uterine artery ligation

Explanation: The most common cause of post partum hemorrhage (PPH) is uterine atony, and the first step in the management of all patients with PPH is to do a pelvic examination to identify any retained placental fragments. (Choice D) If the retained placental products are not identified, manual uterine massage should be started. Uterine massage stimulates the uterus, and the resulting contractions stop the bleeding. (Choice A) If bimanual uterine massage fails to control bleeding, uterotonic drugs (e.g., IV oxytocin) are used to control the bleeding. (Choice E) If bleeding does not stop with medical measures, surgical measures are taken. Uterine artery ligation is one of the surgical measures for the treatment of PPH. (Choice C) PPH is a serious obstetrical complication. Measures should be taken immediately to stop the bleeding. Observation alone may be fatal. Educational Objective: Pelvic examination to look for any retained placental products is the first step in the management of PPH. 20% of people answered this question correctly. An 83-year-old Caucasian man comes to the emergency department because of a sudden onset, transient visual loss in his right eye. He is currently symptom-free, and denies any other new symptoms. His other medical problems include hypertension and peripheral vascular disease. His medications include aspirin, hydrochlorothiazide, and enalapril. His temperature is 36.7 C (98 F), blood pressure is 160/90 mmHg, pulse is 80/min., and respirations are 12/min. Which of the following signs is most likely to be found during the physical examination? A. Systolic heart murmur B. Diastolic heart murmur C. Papilledema D. Hemorrhagia exudates in the retina E. Carotid bruit Explanation: This patient has developed amaurosis fugax, an acute ischemic event involving the retinal artery. This condition is sudden and usually reversible. It is a marker of carotid artery atherosclerotic disease. A carotid bruit on auscultation of the neck is a very frequent finding; this indicates a significant carotid obstruction. (Choice B) A diastolic heart murmur can be a marker of aortic insufficiency or mitral stenosis. Both conditions are related to ischemic cerebrovascular accidents (CVA). Although these could occur, this elderly patient's transient symptom is more likely to be concurrent with findings suggestive of carotid artery disease. (Choice A) Conditions producing systolic heart murmurs such as mitral regurgitation, aortic stenosis, or tricuspid regurgitation, are more related to heart failure symptoms. Even though aortic stenosis can be related to syncope, it does not cause amaurosis fugax . (Choice D) Acute retinal vein thrombosis can be diagnosed by fundoscopy, when extensive hemorrhage is seen in the retina. It will not present as amaurosis fugax.

(Choice C) Papilledema is seen as a result of increased intracranial pressure. Pseudopapilledema, on the other hand, may be seen in an established nerve damage or in optic neuritis, but not in amaurosis fugax. Ischemia will manifest as disc paleness, not papilledema, especially in a transient event.

Educational Objective: Amaurosis fugax is a sudden and transient monocular blindness. Even though it is reversible, it is a marker of carotid artery atherosclerotic disease that is usually advanced. Physical examination will often reveal a carotid bruit. Carotid Doppler evaluation is necessary to evaluate the extent of the disease and to assess the need for a carotid endarterectomy. 86% of people answered this question correctly. The mother of a two-year-old girl comes to your office. She is extremely anxious and wants you to see her daughter right away. She tells you that her daughter came back two months ago after spending six weeks with her grandmother in a ranch in Texas. The grandmother was recently hospitalized, diagnosed with pulmonary tuberculosis, and prescribed six months of antitubercular therapy. The mother is concerned about her daughter?s risk of contracting the illness, and asks for your advice. The girl does not have any new symptoms suggestive of active tuberculosis. Which of the following should be your next step in management? A. Reassure the mother and do nothing at this point. B. Perform a tuberculin skin test on the daughter. C. Obtain a plain chest x-ray. D. Schedule a serial followup examination of the patient every month. E. Prescribe a weight-based isoniazid therapy for nine months. Explanation: Tuberculosis is spread from person to person through airborne droplets. The risk of spread of latent or active tuberculosis is greatest when there is a history of contact with patients who have active (sputum-positive) pulmonary tuberculosis. Once infected, a person?s body mounts a cell-mediated immune response which is detected by a tuberculin skin test. This is why the first step in evaluating an asymptomatic person with recent significant exposure to tuberculosis patients is to obtain a tuberculin skin test. If the initial test result is negative, a second test should be performed 10 weeks after the last known exposure. Tuberculin skin testing detects the presence of latent tubercular infection. The treatment of latent tubercular infection should be strongly considered in all patients, especially in children less than 15 years of age and in HIV-positive patients. (Choice A) A person exposed to a known tuberculosis patient has the greatest risk of developing tuberculosis within the next two years. This person should receive a tuberculin skin test to document the presence of latent tuberculosis. Reassurance is not appropriate at this point. (Choices C and D) The majority of primary pulmonary tubercular infections are radiographically and clinically silent. In most cases, a positive tuberculin skin test is the only indication that infection with Mycobacterium tuberculosis has occurred. (Choice E) Isoniazid therapy for nine months should be recommended for all patients with a positive PPD or recent tuberculin skin test conversion (at least 10 mm increase in the skin reaction within a two-year period). It is important to document latent tubercular infection with a positive tuberculin skin test before prescribing isoniazid therapy. It is also important to document that the patient does not have active tuberculosis, in which case she should be treated with a full course of antitubercular therapy. Educational Objective: Tuberculin skin testing should be initially performed in all persons exposed to patients with tuberculosis. 85% of people answered this question correctly.

A 76-year-old gravida 4, para 4 woman presents to the clinic complaining of pelvic pressure and heaviness, lower back pain, and constipation. These symptoms were gradual in onset and have been present for a period of months. Her past medical history is significant for mild congestive heart failure, emphysema, hypertension, and peptic ulcer disease. She has a 52 pack-year smoking history and occasionally drinks bourbon with her friends. Her vital signs are normal. The physical examination reveals a moderately overweight woman in no apparent distress. The gynecologic examination demonstrates protrusion of the posterior vaginal wall that is most prominent with bearing down while in the lithotomy position. The patient is diagnosed with rectocele, and surgical repair is recommended. The woman declines to have surgery and remains steadfast on this point. What is the most appropriate response in this situation? A. Recommend oral hormone replacement therapy B. Recommend pelvic exercises C. Recommend pessary use D. Convince her to undergo surgical repair E. Inform the patient she needs to find a new physician Explanation: Rectocele is a relatively common condition in older women and is characterized by the displacement of the rectum through posterior vaginal wall defect(s). The condition is typically caused by damage to the rectovaginal septum incurred during vaginal childbirth and is exacerbated by periodic increases in intraabdominal pressure (e.g., when laughing or coughing) and the effects of gravity. Women with symptomatic rectoceles who are poor surgical candidates may be treated with pessaries, which are structures designed to support the vaginal wall. Pessaries should only be used in conjunction with vaginal estrogen; without it, these can cause chronic discharge and bleeding secondary to injury of the vaginal tissues. (Choice A) Although estrogen creams are useful in conjunction with pessary usage, there is no indication for oral hormone replacement therapy in the treatment of rectoceles. (Choice B) Pelvic exercises are appropriate to recommend in women with asymptomatic rectocele. Additional recommendations for this patient group include avoidance of activities related to increased intraabdominal pressure and regular usage of intravaginal estrogen to prevent tissue atrophy. (Choice D) Surgical repair, most commonly via a posterior colporrhaphy, is an appropriate recommendation for women with symptomatic rectoceles. However, it is important to advise these patients that correction of the condition does not always provide symptomatic relief. Patients who are not good surgical candidates or who prefer not to undergo surgery should not be pressured to proceed. Rather, they should be advised about alternative treatment options (e.g., pessaries). (Choice E) There is no need to deny this patient further care simply because she does not want to undergo surgery. Since there is a reasonable alternative available, and since she is an elderly woman with numerous serious medical conditions, it is preferable to avoid elective surgical procedures. Educational Objective: Women with symptomatic rectoceles who are poor surgical candidates may be treated with pessaries in conjunction with estrogen cream. 70% of people answered this question correctly. The following Vignette applies to the next 5 items A 26-year-old Italian-American man comes to your office for the evaluation of low back pain. He has had this pain for the last eight months, and it is progressively getting worse. The pain is worse in the morning and gets better as the day progresses. He denies any history of trauma in the past. He works as a chef at a local restaurant. His father and uncles also have "back problems." His physical

examination reveals a limited range of motion of his lumbosacral spine and markedly reduced chest expansion. The rest of his physical and musculoskeletal examination is unremarkable.

Item 1 of 5 Which of the following is the most appropriate next step in the management of this patient? A. HLA-B 27 levels B. ANA and rheumatoid factor C. Bone scan D. A MRI of the spine E. X-ray of the sacroiliac joint Explanation: The patient has typical features of ankylosing spondylitis. These include back pain with morning stiffness which improves with exercise, insidious onset, presence of symptoms for more than three months, reduced range of forward flexion of the lumber spine on Schober testing, and reduced chest expansion. A diagnosis of ankylosing spondylitis cannot be made unless there is evidence of sacroiliitis, which is the earliest change that is seen radiographically. Therefore, when a patient has typical features of ankylosing spondylitis, a plain x-ray of the sacroiliac joint is the next step to establish the diagnosis. Other radiographic abnormalities suggesting the diagnosis of ankylosing spondylitis include erosions of the ischial tuberosity and iliac crest. Early in the course of disease, squaring of the vertebral disease on plain x-ray may also suggest the diagnosis. (Choice D) When ankylosing spondylitis is strongly suspected clinically and radiographic findings of plain x-ray are negative or equivocal, a CT scan is indicated to establish the diagnosis. MRI may also be used for this purpose, but a CT scan is usually recommended in such setting. (Choice A) HLA-B 27 is frequently (>90% of the patients) present in patients with ankylosing spondylitis and other spondyloarthropathies. Its presence is not specific for the diagnosis of ankylosing spondylitis; however, its absence (negative for HLA B 27) makes the diagnosis of ankylosing spondylitis unlikely. (Choice B) Testing for ANA and rheumatoid factor is not the next appropriate step when a patient has a history typical of ankylosing spondylitis. ANA and rheumatoid factor are absent in ankylosing spondylitis and other seronegative spondyloarthropathies. (Choice C) A bone scan has no role in the diagnosis of ankylosing spondylitis. Educational Objective: The three important clinical criteria for the diagnosis of ankylosing spondylitis are: Presence of low back pain and stiffness for more than a three month duration that improves with exercise or activity. Limitation of the range of motion of the lumbar spine. Limitation of chest expansion relative to the normal values. Plain x-ray of the sacroiliac joint is the next best step in the management of a patient who is suspected with ankylosing spondylitis. 63% of people answered this question correctly. Item 2 of 5 If the initial evaluation shows mild disease, what would you do to monitor the disease activity? A. Repeat x-rays after 3 months B. Repeat MRI in 3 months C. Repeat bone scan in 3 months D. Repeat HLA-B 27 levels in 3 months E. Repeat ANA and rheumatoid factor levels in 3 months Explanation:

The following radiographs are used in monitoring the disease progression of patients with ankylosing spondylitis: 1. Anteroposterior and lateral views of the lumbar spine 2. Lateral view of the cervical spine 3. Pelvic radiograph, including the sacroiliac joints and hip Other acute phase reactants such as ESR can also be used. (Choice B) CT scans are almost never used to monitor the disease activity. MRI is also not used to monitor the disease activity, unless the patient develops complications such as cauda equina syndrome. Educational Objective: Radiographs and acute phase reactants (i.e., ESR) are used to monitor the disease progression of patients with ankylosing spondylitis. 59% of people answered this question correctly. Item 3 of 5 Which of the following conditions is associated with the patient?s diagnosis? A. Restrictive lung disease B. Acute narrow angle glaucoma C. Coarctation of aorta D. Early development of testicular cancer E. Sclerosing cholangitis Explanation: Once a diagnosis of ankylosing spondylitis is being considered, it is important to search for concurrent extraarticular manifestations. Although they are usually asymptomatic in the initial stages, many patients have restrictive lung disease due to the limited costovertebral joint motion and development of apical pulmonary fibrosis. It is important to counsel such patients regarding smoking cessation to prevent early deterioration of lung function. (Choice B) Ocular manifestations with AS include acute anterior uveitis, cataracts, and cystoid macular edema. Uveitis presents with acute onset unilateral eye pain, photophobia, and blurring of vision. It is sometimes the initial presentation of ankylosing spondylitis. (Choice C) The most common valvular abnormalities associated with ankylosing spondylitis include aortic regurgitation and mitral valve prolapse. The incidence of coarctation of the aorta and septal defects is not increased in these patients. (Choice D) There is an increased prevalence of varicocele, not testicular cancer, in these patients. (Choice E) The gastrointestinal manifestations are nonspecific ileal and colonic mucosal ulcerations. Some other extraarticular features include atlanto-axial subluxation causing spinal cord compression, cauda equina syndrome, IgA nephropathy, and secondary amyloidosis presenting as nephrotic syndrome. Educational Objective: The most common and important extraarticular manifestations of ankylosing spondylitis are acute anterior uveitis, aortic regurgitation, apical pulmonary fibrosis, IgA nephropathy, and restrictive lung disease. 58% of people answered this question correctly. Item 4 of 5

The patient is given a prescription for non-steroidal anti-inflammatory agents. He returns for a followup visit after two weeks, and says that he feels a lot better. His pain and stiffness have almost resolved with daily intake of medications. He used to swim and exercise regularly before his symptoms developed a few months ago, and he wants to resume these activities, but he is concerned that his disease may get worse if he does so. Which of the following is the most appropriate response? A. Aerobic exercises will accelerate the joint destruction B. Extension exercises of the spine can be harmful C. He should not resume exercise for at least 1 year D. Swimming should be avoided as it increases the strain on back muscles E. Aerobic exercises improve overall functional status without increasing the disease activity

Explanation: Regular aerobic exercises such as swimming, walking, and bicycling improve joint stability, muscle strength, and overall functional status without an increase in the disease activity. These also help to prevent bone loss due to immobility and use of medications. The exercise regimen should be tailored to the individual, depending on the exercise tolerance, extent and severity of joint disease, and other coexisting medical conditions. An initial evaluation by a physical therapist should be a part of therapy for ankylosing spondylitis. (Choice A) Supervised physical therapy and exercises can lead to significant improvement of the range of motion of the spine. It does not lead to acceleration of the joint destruction. (Choice B) In ankylosing spondylitis, postural training and extension exercises help to prevent spine fusion in a flexed position. (Choice C) There is no reason to avoid exercise for a year. The patient should be evaluated by a physical therapist and initiate an exercise program as early as possible. (Choice D) Swimming (hydrotherapy) is a type of aerobic exercise with special benefits for patients with rheumatologic disorders. It is associated with less joint stress and weight bearing and is useful for patients with advanced disease. Educational Objective: Aerobic exercises improve overall functional status of patients with rheumatologic disorders and should be encouraged. 81% of people answered this question correctly. Item 5 of 5 Which of the following is the most appropriate statement regarding the prognosis of this condition? A. Overall mortality is increased in this patient population B. Most patients will have functional and employment disability C. Smoking is surprisingly protective in this population D. Life expectancy is not reduced E. Prolonged standing at the work place is protective Explanation: There are no studies that have demonstrated an increased overall mortality or reduced life expectancy in patients with ankylosing spondylitis. Most patients do well and have no functional or employment disabilities. (Choice C) Smoking cessation should be strongly advised, as this is associated with a very bad prognosis. (Choice E) Prolonged standing at work or exposure to cold conditions are risk factors for disability. Educational Objective: Most patients with ankylosing spondylitis do well and have no functional or employment disabilities. There is no increased overall mortality or reduced life expectancy. 46% of people answered this question correctly.

The following vignette applies to the next 3 items A 45-year-old Caucasian woman comes to the emergency department (ED) with complaints of severe abdominal pain. She started having pain in her upper abdomen approximately twelve hours ago. The pain is sharp and radiates to her back. She had two episodes of vomiting before coming to the ED. She has been hospitalized four times in the past with similar complaints. On review of her previous hospitalizations, you notice that there was never a specific cause found for her symptoms. She has a history of diabetes and hypertension, and both are well controlled by diet and exercise. Her blood pressure is 130/70 mmHg, pulse is 92/min, temperature is 36.7C (98F), and respirations are 16/min. She is moderately obese, and is in distress. Her lungs and cardiovascular examination are within normal limits. There is diffuse tenderness to palpation over the whole abdomen, with maximum tenderness present at the epigastric area. Item 1 of 3 Which of the following is the next step in the diagnosis of the patient?s condition? A. Serial abdominal examination B. CT scan of the abdomen C. LFTs with lipase levels D. No further tests are needed at this point as prior workup was normal E. Ultrasound of the right upper quadrant Explanation: The clinical presentation of the patient is consistent with acute pancreatitis, which is characterized by an acute onset of steady, upper abdominal pain radiating to the back, with associated nausea and vomiting. However, since these symptoms are nonspecific and can be seen in a number of acute abdominal illnesses, confirmation of the diagnosis is necessary with clinical and biochemical markers, as well as radiographic imaging. A variety of biochemical tests have been devised to diagnose acute pancreatitis. Of these, measurement of the serum amylase and lipase levels are the most frequently used. Serum lipase is more sensitive and specific than serum amylase; therefore, measurement of the former is the diagnostic test of choice. (An elevated serum amylase level is nonspecific as it is also elevated in a number of other conditions such as acute parotitis, intestinal disorders, renal failure, cholecystitis, and fallopian tube diseases.) In addition, a liver function test (LFT) is also obtained since an elevated alkaline phosphatase level may point towards gallstone/common bile duct stones as the etiology of the acute pancreatitis. (Choice A) Serial abdominal examination can delay the diagnosis of acute pancreatitis, and may even prove to be potentially fatal in these patients. It is only useful to follow the clinical progress of the patient once the diagnosis has been made. (Choice B) CT scan may be eventually needed to document the severity of pancreatitis and to detect the presence of other intra-abdominal complications. These should only be done if all other biochemical markers fail to provide a diagnosis, or if the patient fails to improve with initial conservative treatment. (Choice D) Her current symptoms should not be assumed to be benign based on her past history of hospitalizations. (Choice E) An ultrasound of the right upper quadrant of the abdomen may provide a clue to the cause of her symptoms; however, up to one-third of patients have bowel or intestinal gas that may obscure the pancreas, and thus, the diagnosis. Educational Objective: The diagnosis of acute pancreatitis is confirmed initially with an elevated level of serum biochemical markers (amylase or lipase).

*Extremely important question for USMLE step-3 56% of people answered this question correctly.

Item 2 of 3 The initial lab results of the patient reveal the following: Total bilirubin: 1.2 mg/dL Direct bilirubin: 1.0 mg/dL Alkaline phosphatase: 382 units/L AST: 40 units/L ALT: 38 units/L Albumin: 4 g/dL Serum calcium: 8.9 mg/dL Amylase: 1026 mg/dL Lipase: 662 mg/dL A right upper quadrant ultrasound reveals an enlarged hypoechoic area in the head of the pancreas with mild dilatation of the common bile duct. A contrast-enhanced CT scan of the abdomen reveals enlargement and inflammation of the pancreatic head without any areas of necrosis. Which of the following is the best next step in the management of this patient? A. Intravenous antibiotics B. CT-guided needle aspiration of the pancreatic tissue C. Daily abdominal CT scans D. IV fluids and nasogastric tube aspiration E. Surgical consult Explanation: Most episodes of acute pancreatitis are mild, and patients usually recover in five to seven days. Treatment is usually conservative (i.e., supportive therapy) and aimed at correcting the underlying predisposing condition and preventing any further damage to the pancreatic tissue. Patients can have a significant amount of third space loss of fluids, thereby causing hypotension, acute renal failure and even worsening of the pancreatitis; therefore, all patients should have early and aggressive fluid resuscitation with close monitoring of the urine output to ensure adequate tissue perfusion. Pain control is also important, and is generally achieved with the use of intravenous narcotics. Lastly, patients are kept off an oral diet (NPO), and their gastric contents are aspirated via a nasogastric tube (NGT) to prevent further pancreatic stimulation. Based on the radiographic imaging, the patient is suffering from a mild, edematous, acute pancreatitis. The next best step in management is nasogastric tube (NGT) aspiration and administration of IV fluids. (Choice A) Intravenous antibiotics are indicated only in patients with severe necrotizing pancreatitis or in patients with clinical or tissue evidence of infection of the necrotic pancreatic tissue. These are not given to patients with mild attacks of acute pancreatitis. (Choice B, C) There is no indication for CT-guided aspiration of the pancreatic tissue or daily CT scans. (Choice E) Surgical d?idement is indicated only in patients with extensive tissue necrosis or localized abscess formation. Educational Objective: Most of the patients with mild acute pancreatitis can be managed conservatively by adequate pain control and intravenous fluid resuscitation. *Extremely important question for USMLE step-3 77% of people answered this question correctly.

Item 3 of 3 While you are making your rounds the next morning, the nurse informs you that the patient has developed a temperature of 39.4C (103?F). The rest of her vital signs are within normal limits. Which of the following is the next best step in the management in this patient? A. Obtain blood cultures B. Obtain blood cultures and start the patient on imipenem C. Obtain urine cultures D. Repeat a stat CT scan E. Start the patient on intravenous ampicillin Explanation: The occurrence of fever in a previously afebrile patient with acute pancreatitis is one of the earliest signs of pancreatic infection. Infection of the pancreatic tissue is the major cause of morbidity and mortality in patients with acute pancreatitis. Most infections are seen late in the clinical course of the disease. There is considerable evidence from several studies that the early use of antibiotics in patients with severe necrotizing pancreatitis or in patients with evidence of pancreatic infection improves the outcome and reduces mortality. It is important to use a broad-spectrum antibiotic that can achieve good penetration in the pancreatic tissue. Examples of such antibiotics are: imipenem, third generation cephalosporins, piperacillin, fluoroquinolones, and metronidazole. If the patient?s condition fails to improve after one week of antibiotic therapy, a CT-guided aspiration of the tissue should be performed to obtain tissue samples for culture and sensitivity. (Choices A and C) It is important to start the patient on antibiotics as soon as the cultures are drawn. Obtaining blood or urine cultures alone is not sufficient. (Choice D) Another CT scan is unlikely to change the treatment at this point. The patient should be started on broad-spectrum antibiotics (imipenem) and followed closely for signs of clinical improvement or deterioration. (Choice E) Ampicillin has a poor tissue penetration into the pancreas, and there is considerable drug resistance against its action by most of the gut bacteria which are associated with pancreatic infection. Educational Objective: All patients with severe necrotizing pancreatitis or suspected pancreatic infection should be started on imipenem to decrease the morbidity and mortality associated with the disease. People rarely use a combination of ampicillin, gentamycin, and metronidazole. *Extremely important question for USMLE step-3 68% of people answered this question correctly. A 26-year-old Caucasian male presents to the emergency department with a three day history of dull back pain, progressive difficulty in walking, and urinary retention. He had a mild upper respiratory infection one week ago, but he is otherwise healthy. He is not taking any medications and denies drug abuse. His temperature is 36.7 C (98 F), blood pressure is 120/76 mmHg, pulse is 80/min, and respirations are 16/min. Neurologic examination reveals a decreased muscle strength (2/5) bilaterally in the lower extremities, hyporeactive knee and ankle reflexes, and decreased pain sensation up to the umbilical level. Which of the following is the most likely diagnosis of this patient? A. Herniated intervertebral disk B. Epidural abscess C. Acute subdural hemorrhage

D. Malignancy E. Transverse myelitis

Explanation: Rapidly progressive weakness of the lower extremities following an upper respiratory infection, accompanied by sensory loss and urinary retention, is characteristic for transverse myelitis. Dull back pain may be present. Neurologic examination initially reveals muscle flaccidity and hyporeflexia, but spasticity and hyperreflexia develop subsequently. In patients presenting with acute transverse myelopathy, other causes (especially compressive lesions) should be ruled out by obtaining a careful history and performing the appropriate imaging procedures. (Choice A) A herniated intervertebral disk is characterized by an abrupt onset of neurologic deficit and may be related to strenuous activity. A straight-leg raising test will be positive. (Choices B and C) Although an epidural abscess or an acute subdural hemorrhage may develop in otherwise healthy individuals, an underlying cause is usually present (e.g., IV drug abuse or treatment with anticoagulants). (Choice D) Spinal cord compression from a metastatic tumor may present acutely, but the history of systemic malignancy is typical. Educational Objective: Rapidly progressive weakness of the lower extremities following an upper respiratory infection, accompanied by sensory loss and urinary retention, is characteristic for transverse myelitis. 78% of people answered this question correctly. A healthy 29-year-old Caucasian woman comes to the physician for contraception counseling. She has had two unplanned pregnancies, and doesn?t want to have more. She has no medical problems. She does not use tobacco, alcohol, or drugs. She takes no medications. Her friend recommended a contraceptive method named "Norplant." The patient is concerned about the risks that this method could pose to her health. Which of the following complications is most likely to occur in a patient who uses this method? A. Vaginal spotting B. Thromboembolism C. Rash D. Menorrhagia E. Breast cancer Explanation: "Norplant" consists of six capsules of levonorgestrel which are placed subdermally, generally in the upper arm. It offers contraceptive protection for about five years. The most common complication is menorrhagia (prolonged vaginal bleeding during each period), which occurs in about 28% of the cases. (Choice A) Vaginal spotting is the second most frequent complication, affecting around 17% of the patients. (Choices B, C, and E) "Norplant" increases the risks of thromboembolism and breast cancer. It can also produce a rash. However, all these three complications are not frequent, and occur in less than 10% of the cases. Educational Objective: The use of "Norplant" is associated with menorrhagia and vaginal spotting. Other possible but less common complications are: venous thromboembolism, myocardial infarction, pulmonary embolism, thrombotic thrombocytopenic purpura (TTP), stroke, and breast cancer. 18% of people answered this question correctly.

A 24-year-old male presents to your office with a one-week history of night sweats and progressive fatigue. He also complains of palpitations and shortness of breath on moderate exertion. His past medical history is insignificant. His blood pressure is 120/80 mmHg, pulse is 98/min, temperature is 38.9C (102F) and respirations are 24/min. Lung auscultation reveals scattered bilateral rales. A grade II/VI blowing systolic murmur that increases with respiration is heard over the lower part of the left sternal border. The chest x-ray demonstrates several circumscribed round pulmonary infiltrates on both sides. Transesophageal echocardiography reveals small vegetations confined to the tricuspid valve. What is the most likely cause of this patient?s problem? A. Rheumatic fever B. Rheumatoid arthritis C. Illicit drug use D. SLE E. Congenital valve defect Explanation: The clinical scenario is highly suggestive of infective endocarditis involving the right cardiac chambers. Two very important clues to the diagnosis are: evidence of right-sided cardiac involvement (systolic murmur with inspiratory accentuation and tricuspid vegetation on echocardiography) and evidence of septic pulmonary emboli (present in 75% of patients with right-sided endocarditis). Peripheral manifestations and heart failure are uncommon in these patients, and a vast majority are IV drug users. The endocarditis seen in drug users is different from non-drug users in that the tricuspid valve is involved in 30 to 70% of endocarditis cases in injection drug users. Overall, rightsided endocarditis (predominantly involving the tricuspid valve) is far less common than left-sided disease. (Choices A and D) Rheumatic fever and SLE predominantly involve left cardiac structures. Isolated lesions of right cardiac valves are quite uncommon with these diseases. Educational Objective: A vast majority of patients with right-sided endocarditis are IV drug users. 94% of people answered this question correctly. After drawing blood from an HIV-positive patient, a 28-year-old Caucasian female nurse accidentally pricks her finger with the needle. Recent laboratory testing of the patient demonstrated a CD4 count of 410/?L and HIV load 12,000 copies/mL. The nurse has no significant medical history and, for baseline purposes, is found to be HIV-negative. She takes no medications and is not pregnant or breastfeeding. Her immunizations are current. What is the best approach to handling this situation? A. Therapy with one nucleoside reverse transcriptase inhibitor and one protease inhibitor to be initiated immediately and continue for four weeks B. Therapy with one nucleoside reverse transcriptase inhibitor and one protease inhibitor to be initiated immediately and continue for six months C. Therapy with two nucleoside reverse transcriptase inhibitors to be initiated immediately and continue for four weeks D. Therapy with two nucleoside reverse transcriptase inhibitors to be initiated immediately and continue for six months E. Unless seroconversion is documented, no antiviral therapy is necessary Explanation: A prophylactic antiretroviral regimen is always indicated when a healthcare worker is exposed to the bodily fluids of an HIV-positive patient. Immediate consultation with the hospital?s infectious disease expert is recommended. The CDC recommends that two nucleoside reverse transcriptase inhibitors (e.g., zidovudine and lamivudine) be started immediately after exposure and continued for the next

four weeks (Choice C). Some experts have suggested modifying the regimen to also include a protease inhibitor (e.g., indinavir) if viral resistance to the postexposure prophylaxis is known or suspected. Therapy with one nucleoside reverse transcriptase inhibitor and one protease inhibitor (Choices A and B) is not recommended. Therapy with two nucleoside reverse transcriptase inhibitors (Choice D) is indeed the recommended regimen, but the therapy should be continued for four weeks, not six months. Foregoing antiviral therapy (Choice E) is not advisable. Systemic infection does not occur immediately after inoculation with HIV; early on, there is a brief period in which antiretroviral therapy is thought to limit viral replication. One study showed that healthcare workers using zidovudine alone as postexposure prophylaxis were 81% less likely to seroconvert as compared to those who used no treatment regimen. Therefore, any healthcare worker exposed to the bodily fluids of an HIV-positive individual should be strongly urged to use a prophylactic regimen to minimize the likelihood of becoming HIV-positive. Educational Objective: Exposure to the body fluid of an HIV-positive individual requires antiretroviral prophylaxis. Accepted regimens include zidovudine and lamivudine or zidovudine, lamivudine, and indinavir. Treatment should begin within hours of the exposure and be continued for four weeks. 18% of people answered this question correctly. A 44-year-old Caucasian male presents to the emergency department with a 12 hour history of severe epigastric pain and vomiting. He had a similar episode one year ago that required hospitalization. He admits to excessive drinking for the past several weeks. He is not taking any medications, and denies any drug abuse. His blood pressure is 100/70 mmHg, and heart rate is 90/min. His serum lipase level is elevated. Which of the following admission criteria is indicative of a severe attack in this patient? A. Age B. Hypercalcemia C. High blood glucose level D. Hypokalemia E. Elevated ESR Explanation: Ranson?s prognostic criteria gained widespread use in predicting the severity of acute pancreatitis. Since the criteria were initially designed for pancreatitis not induced by gallstones, the scoring system was modified so it could also be used in the evaluation of patients with gallstone-induced pancreatitis. The prognostic factors include admission criteria, and criteria observed during the initial 48 hours. Admission criteria are age > 55 years, high WBC count, high serum AST, high serum LDH, and high blood glucose level. Criteria observed during the initial 48 hours include a hematocrit fall (>10%), BUN elevation, hypocalcemia, hypoxemia (PO2 < 60 mmHg), base deficit > 4 mEq/L, and an estimated fluid sequestration > 6L. (Choice A) The admission criteria includes an age > 55 years; however, this patient is only 44 years old. (Choice B) Hypocalcemia, not hypercalcemia, is a part of the Ranson's criteria during the initial 48 hours. (Choices D and E) Hypokalemia and elevated ESR are not considered significant prognostic factors. Educational Objective:

Ranson?s criteria gained widespread use in predicting the severity of acute pancreatitis. 59% of people answered this question correctly.

A 44-year-old Caucasian male presents to his primary care physician for an annual physical exam. He has no significant past medical history and takes no medications. He admits to drinking socially. He has a 25 pack-year history of tobacco use, and denies recreational drug use. He states that his mother and brother are both recovering alcoholics. What is the most effective method of detecting alcohol abuse or dependence in this patient? A. Inquire about psychiatric history B. Inquire about the type, frequency, and quantity of alcohol use C. Administer a screening questionnaire such as CAGE D. Administer a breath test to rule out alcohol intoxication E. Order laboratory testing to detect hepatic dysfunction Explanation: The approach to a patient with alcoholism is a stepwise process. Step 1: Inquire about current and past alcohol use, and about a family history of alcohol problems. Step 2: Obtain information regarding the type, quantity and frequency of alcohol use. This is useful to distinguish between moderate and heavy drinkers. Step 3: Use a standard screening questionnaire such as CAGE. Step 4: Ask more specific questions based upon steps 1 to 3 in patients with suspected alcohol problems. Studies have shown screening questionnaires (Choice C) to be superior to both laboratory testing (Choice E) and inquiries about quantity or frequency of drinking (Choice B) in evaluating individuals for alcohol abuse or dependence. Appropriate screening questionnaires include CAGE, AUDIT, and TWEAK. (Choice A) Although there is a high rate of comorbid psychiatric disorders in individuals with alcohol abuse and dependence, the likelihood of alcohol abuse should be more fully explored before discussing the psychiatric history. (Choice D) Breath tests are administered by law enforcement personnel when evaluating an individual for acute alcohol intoxication. They are not typically used in the setting of a physician?s office, and are more likely to inspire annoyance than compliance. Educational Objective: Studies have shown screening questionnaires to be superior to both laboratory testing and inquiries about quantity or frequency of drinking in evaluating individuals for alcohol abuse or dependence. * If this question had asked about ?next step? the answer would be choice B. 84% of people answered this question correctly. A 6-year-old Caucasian girl is brought to the office by her mother because of loss of appetite, malaise, and pain in the right arm. She has had these symptoms for the past five days. She has been playing with a small cat for the past month. She also has a small pet dog, which she feeds personally everyday. Examination shows tender cervical and right axillary lymphadenopathy. Over the next week, which of the following complications is most likely? A. Myocarditis B. Seizures C. Renal failure D. Hepatosplenomegaly E. Suppuration of the lymph nodes Explanation:

Cat-scratch disease is an infection that usually affects the young immunocompetent population. It is produced by Bartonella henselae. The most common complication is suppuration of the lymph nodes. (Choice D) Hepatomegaly and splenomegaly can sometimes be seen. (Choice C) Renal compromise has not been reported. (Choices A and B) Myocarditis and seizures are not part of the expected complications. Educational Objective: Approximately 10% of patients with cat-scratch disease can develop suppuration of the lymph nodes. Other complications are: visual loss due to neuroretinitis, encephalopathy, fever of unknown origin, and hepatosplenomegaly. 76% of people answered this question correctly. The following vignette applies to the next 2 items A previously healthy, 30-year-old Caucasian female is admitted to the hospital. She has a rash over her lower legs and ankles. She first noticed the rash over her ankles 2 weeks ago, and it slowly spread up to her knees. She has no other past medical history. She denies any regular medication intake. Her vital signs are within normal limits. Physical examination reveals the presence of fine petechiae and purpura around her ankles and lower legs. The rest of her physical examination is normal. The labs reveal the following: Hb: 14.4 g/dL MCV: 90 fl Platelet count: 16,000/cmm Leukocyte count: 8,000/cmm Segmented neutrophils: 60% Bands 3% Eosinophils 6% Lymphocytes 24% Monocytes 6% Prothrombin time: 14 sec Partial thromboplastin time: 30 sec Plasma fibrinogen: 300 mg/dL D-dimer: 0.22 ?g/mL Peripheral blood smear is normal. The admitting resident gets concerned with the lab results and orders 6 units of random donor platelet transfusion. The following day, the platelet count drops even further to 10,000/microliter. Item 1 of 2 Which of the following is the most likely cause of the drop in the platelet count? A. Disseminated intravascular coagulation B. Thrombotic thrombocytopenic purpura C. Antiplatelet antibodies D. Septicemia E. Drug induced thrombocytopenia Explanation: A presumptive diagnosis of idiopathic thrombocytopenic purpura (ITP) can be made when the history, physical examination, complete blood count, differential count, and peripheral blood smear do not suggest any causes for isolated thrombocytopenia. ITP occurs as a result of platelet destruction by specific autoantibodies. Platelet transfusions are rarely necessary to maintain the platelet count. The circulating platelets are rapidly removed by the autoantibodies anyway, making

platelet transfusions futile. Patients with mild and asymptomatic thrombocytopenia (platelet count between 30,000 to 50,000/microliter) should not be treated. Transfusions are only necessary in severe or symptomatic thrombocytopenia (i.e. life threatening emergencies such as intracerebral or massive gastrointestinal hemorrhage). (Choice A, B, and C) There is no clinical or laboratory evidence of DIC, TTP or septicemia in this patient. (Choice E) The patient has not been exposed to any drugs which could cause or precipitate thrombocytopenia. Educational Objective: ITP is due to autoimmune phenomena involving the formation of antiplatelet antibodies. Treatment is only necessary in severe or symptomatic thrombocytopenia. Item 2 of 2 Which of the following is the most appropriate next step in the management of this patient? A. Corticosteroids B. Plasmapheresis C. Low-dose heparin D. Intravenous immunoglobulins E. Give more platelets Explanation: ITP is acute and self-limiting in children, but it usually becomes a chronic disorder in adults. In such cases, corticosteroid therapy is the treatment of choice. Most adults respond to steroids within two weeks, with most patients responding within the first week. IV IG can be used if steroids fail. A splenectomy is rarely performed in refractory cases. (Choice B) Plasmapheresis is the treatment of choice for hemolytic uremic syndrome (HUS) and thrombotic thrombocytopenic purpura (TTP). (Choice C) Heparin is used very cautiously in patients with early disseminated intravascular coagulation (DIC). Once the patient starts to bleed, heparin should not be used. Educational Objective: Corticosteroid therapy is the treatment of choice for ITP in adult patients. 43% of people answered this question correctly. A 56-year-old Caucasian man comes to the emergency department with complaints of fever, pain, and swelling around the right lower leg for the last five days. His symptoms are getting worse, and he has been having difficulty weight bearing and ambulating for the last two days. He has a past medical history of coronary artery disease, ischemic cardiomyopathy with an ejection fraction of 25%, pacemaker implantation for sick sinus syndrome, and insulin-dependent diabetes mellitus with diabetic neuropathy. He quit smoking 12 years ago. He is a manager at a local restaurant. His temperature is 38.9 C (101 F), heart rate is 86/minute, blood pressure is 132/76 mmHg, and respiratory rate is 16/minute. Physical examination reveals significant swelling and erythema around the right lower leg. The lower extremity ultrasound is negative for deep vein thrombosis. A plain radiograph of the lower extremity is also negative for any acute pathology. The patient is started on the appropriate antibiotics; the blood culture results are still pending. What is the most appropriate next step in the management of this patient? A. Obtain a MRI scan of the lower extremity. B. Order a three phase technetium 99m bone scan. C. Repeat the x-ray in 2 weeks to confirm the diagnosis. D. Treat the patient for 4-6 weeks for tibial osteomyelitis. E. Obtain open bone biopsy to confirm the diagnosis.

Explanation: Osteomyelitis is an infection of the bone which causes bone destruction and necrosis. An acute infection usually presents with fever, chills, erythema, and swelling over the involved bones, although it is difficult to diagnose early in its course because the presenting symptoms and signs can be nonspecific. Radiologic imaging plays an important role in the evaluation of patients with suspected osteomyelitis. A three-phase technetium bone scan is the diagnostic test of choice for suspected uncomplicated osteomyelitis if the plain films are negative. It uses technetium 99-m bound to phosphorus as a tracer, which accumulates in the area of increased osteoblast activity. It is usually positive 2-3 days after the onset of infection. Some other nuclear scans that can be used include indium-labeled white cell scan and gallium citrate scans. (Choice A) CT scan and MRI are very accurate in detecting changes of osteomyelitis in the early stages. MRI is especially useful in patients with suspected vertebral osteomyelitis and in infected diabetic foot lesions. It is also useful in detecting abscesses around the site of infection. However, MRI cannot be used in patients with certain metal hardware and implants. This patient has a pacemaker implant, which precludes the use of MRI to evaluate suspected osteomyelitis. Furthermore, a technetium bone scan can easily make the diagnosis. For all these reasons, MRI is not needed in this patient. (Choice C) Plain films are insensitive for the diagnosis of acute osteomyelitis. This is due to the 2-3 week lag time for the changes of acute osteomyelitis to appear on the plain films. Repeating the films is also an insensitive procedure, and will only serve to delay the diagnosis. (Choice D) Empiric treatment for 4?6 weeks is inappropriate in the absence of a specific diagnosis. Furthermore, imaging is necessary to rule out the presence of potential complications (e.g., abscess or gas formation in the soft tissue). (Choice E) Although a bone biopsy is the gold standard for the diagnosis of osteomyelitis, it should only be performed if the non-invasive tests are inconclusive and the index of suspicion for osteomyelitis is high. Educational Objective: The three-phase technetium 99-m bone scan is the diagnostic test of choice for evaluation of acute uncomplicated osteomyelitis. MRI is better for evaluating spine and complicated foot osteomyelitis, including diabetic foot ulcers. A 33-year-old African-American female comes to your clinic. She has no significant past medical history. She works as an executive secretary. She drinks alcohol occasionally (twice a month), and denies any smoking or use of illegal drugs. She is married, and has a 5-year-old daughter. She was pregnant only once, and denies a prior history of miscarriages. She had preeclampsia during her first pregnancy at the 34th week of gestation. She is using oral contraception. She wants to get pregnant again, but she is afraid of having preeclampsia again. What is the most proper statement to tell her? A. If you had preeclampsia once, it does not mean that it will happen again. B. You will have preeclampsia for sure again, so you have to be very careful. C. Nobody can predict if it will happen again, so you must be careful. D. You don?t have to worry about that, preeclampsia usually occurs during the first pregnancy, it is rare during the second. E. You have a higher risk of developing preeclampsia than other women, but that does not mean it will happen again. Explanation:

A history of preeclampsia in the first pregnancy increases the possibility of a second episode in the following pregnancy. The risk is at least seven times higher, but it can be as high as 15 if the previous preeclampsia presented before 33 weeks of pregnancy. The patient had preeclampsia between 33 and 36 weeks of pregnancy, and is therefore ten times more likely to develop preeclampsia than a woman without that history. (Choices A and D) It is inappropriate to give the patient false reassurance. She has to be informed that the risk for preeclampsia exists. (Choices B and C) Although the patient is at risk, her chances for getting preeclampsia again are uncertain; therefore, one cannot make false assumptions, nor ignore that there are studies which address the topic and supply the patient with an ambiguous answer. Educational Objective: A history of preeclampsia in the past increases the risk of developing this complication during a subsequent pregnancy. The risk is higher if the preeclampsia presented earlier (age of delivery was less than 32-33 weeks), or if the patient has renal disease or chronic hypertension.

The following Vignette applies to the next 3 items A 34-year-old Caucasian male presents to your office with abdominal cramps, watery stools and mild fever. His symptoms started two days ago, and he has tried several over-the-counter anti-diarrheal agents without any success. He denies any gross blood or black discoloration of the stool, nausea and vomiting, although he admits that his appetite has decreased a little. His past medical history is significant for a recent episode of acute sinusitis treated with amoxicillin. He does not smoke or consume alcohol. He denies any recent travel or contact with a patient with similar symptoms. He is sexually active with his wife and uses condoms for contraception. He works as a programmer at a private firm and does not consider his job stressful. Physical examination reveals mild tenderness in the left lower quadrant of his abdomen. His stool is positive for occult blood, but is negative for C. difficile toxin by rapid immunoassay. Item 1 of 3 Which of the following is the best next step in the management of this patient? A. Repeat immunoassay for C. difficile toxin B. Do colonoscopy C. Order abdominal CT D. Order stool culture for C. difficile E. Obtain blood cultures and liver function tests Explanation: The given symptoms and findings in this patient, as well as the history of recent antibiotic treatment, are highly suggestive of C. difficile colitis. Rapid immunoassays to detect C. difficile toxins are gaining popularity among clinicians because these are less time-consuming and less expensive than the older stool cytotoxin test. These rapid tests have very high specificity (close to 100%); however, their sensitivity is about 70-87%, which is lower than that of the stool cytotoxin test (94-100%). Because of this lower sensitivity, repeating the test may be necessary in patients in whom the pretest probability of C. difficile infection is high and an initial test result is negative. (Choices B and C) Repeating the rapid immunoassay test is more reasonable and cost-effective in this case than proceeding with colonoscopy or abdominal CT. Furthermore, endoscopy is not generally recommended in patients with a classical scenario of C. difficile colitis, although this may provide valuable information in specific situations when a quick diagnosis is imperative, or when the diagnosis is in doubt.

(Choice D) Stool cultures of C. difficile are very labor-intensive and are not helpful because nontoxigenic strains of C. difficile exist. Educational Objective: Rapid stool tests to detect C. difficile toxins have very high specificity (close to 100%); however, their sensitivity is about 70-87%. Due to this lower sensitivity, repeating the test may be necessary in patients in whom the pretest probability of C difficile infection is high and an initial test result is negative. Item 2 of 3 After running the appropriate tests, you proceed with treatment. Which of the following agents is the best initial choice for this patient? A. Vancomycin B. Cholestyramine C. Bacitracin D. Metronidazole E. Clindamycin Explanation: There are several agents available for the treatment of C. difficile colitis. Vancomycin and metronidazole are the most popular agents because their efficacy and safety have been tested in many clinical trials. Metronidazole can be given in either oral or intravenous form because of its biliary secretion, whereas vancomycin should be given only in oral form as it will not be absorbed from the intestine. Most authors currently recommend metronidazole as the first-line agent over vancomycin because of the following reasons: 1) both agents have been shown to be equally effective; 2) metronidazole treatment is much more cheaper than vancomycin treatment; 3) vancomycin may lead to the selection of vancomycin-resistant enterococci, and this may create a future problem in public health. (Choice C) Oral bacitracin has been tried with success for C. difficile-induced diarrhea, but this is not commonly used, mainly due to its limited availability. (Choice D) Cholestyramine has also been proven to be effective for this condition, but its efficacy is much lower than that of vancomycin and metronidazole. (Choice E) Clindamycin is an agent that is often implicated as a cause of C. difficile-induced diarrhea. It is not the agent used for its treatment. Educational Objective: Most authors currently recommend oral or intravenous metronidazole as the first-line agent over oral vancomycin for the treatment of C. difficile-induced colitis. Item 3 of 3 Soon after the initiation of therapy, the patient seems to recover completely. One week after the completion of therapy, he returns to your office and complains of abdominal cramps and watery diarrhea again. His stool is positive for occult blood and the rapid immunoassay test is positive for C. difficile toxin in the stool. Which of the following is the best next step in the treatment of this patient? A. Vancomycin B. Cholestyramine C. Bacitracin D. Metronidazole E. Clindamycin

Explanation: This patient seems to be suffering from a relapse of C. difficile colitis. The most likely reason for the recurrence is the regeneration of the replicating form, with toxin production from the spore form. In patients with recurring symptoms after successful treatment, the cause of diarrhea should be ascertained first (the scenario says that rapid immunoassay is positive for C. difficile toxin in the stool). The management of a first relapse following therapy for C. difficile diarrhea and colitis does not differ substantially from treatment of the initial episode: metronidazole is preferred to vancomycin due to cost and bacterial resistance considerations. (Choice A) There is no reason to suspect C. difficile resistance to metronidazole because it is believed to be very rare, if it even exists; however, most authors recommend switching to vancomycin after more than one relapse because it seems to be more effective in such cases than metronidazole treatment. (Choices C and B) Bacitracin and cholestyramine have very limited roles in the treatment of patients with relapsing C. difficile diarrhea. (Choice E) Clindamycin is an agent that is often implicated as a cause of C. difficile-induced diarrhea; it is not the agent for its treatment. Educational Objective: The most common cause of recurrence of C. difficile is not due to resistance to the treatment. Rather, it is due to the development of the replicating form with toxin production from the spore form. The management of a first relapse following therapy for C. difficile diarrhea and colitis does not differ substantially from treatment of the initial episode.

A male infant is born at term to a 32-year-old African-American multigravida. The mother was diagnosed with gestational diabetes mellitus. The newborn?s length is 55 cm and weight is 4,500 kg. No respiratory symptoms or breathing difficulty is present. Physical examination reveals an adducted and internally rotated right arm with forearm pronation and flexed wrist. Moro reflex is asymmetric. Palmar grasp reflex is present on both sides. Which of the following is the best statement concerning the prognosis in this patient? A. By the age of one year the infant would most probably have no or mild paresis B. The condition most probably would not improve or may improve mildly C. The condition is progressive and would eventually involve the hand muscles D. Symmetric palmar grasp reflex has no prognostic significance E. Horner?s syndrome is typically associated with this type of injury Explanation: The classical scenario of Erb?s palsy is described. It is the most common form of obstetrical brachial plexus injury and involves the upper roots (C5, C6, and sometimes C7) of the plexus, resulting in an adducted and internally rotated right arm with forearm pronation and flexed wrist. A serious complication is diaphragmatic paralysis due to phrenic nerve involvement. The prognosis of Erb?s palsy is typically good, with an 80% chance of full or near-full recovery. (Choice D) Symmetric palmar grasp reflex is a good prognostic sign because it indicates that the lower roots of the brachial plexus are intact. (Choice E) Horner?s syndrome may be associated with injury of the lower roots of the brachial plexus.

Educational Objective: The prognosis of obstetrical Erb?s palsy is typically good, with an 80% chance of full or near-full recovery. A 22-year-old Asian-American woman returns to your clinic for a follow-up visit. You recently evaluated her for weight loss, amenorrhea, chronic diarrhea, and anxiety. Her laboratory tests show a suppressed TSH and elevated levels of FT3 and FT4. You then make a diagnosis of hyperthyroidism. The patient is currently complaining of palpitations. The physical examination shows that her heart rate is 118/min, with a blood pressure of 100/60 mmHg, and a respiratory rate of 18/min. Her heartbeat and pulse are rhythmic, but fast. No murmurs are detected. What is the most immediate step in the management of this patient?s condition? A. Propylthiouracil (PTU) B. Propranolol C. Methimazole D. Thyroidectomy E. Radioactive Iodine ablation therapy (RAI) Explanation: Symptomatic tachycardia in a hyperthyroid patient can be rapidly managed with beta-blockers. (Choices A and C) Methimazole or PTU is needed to control the thyrotoxic state, but is not an urgent indication, unless the patient is going through a thyroid storm (fever, altered mental status, CHF, liver disease). These drugs improve the cardiovascular complications of hyperthyroidism, but will not relieve palpitations or control tachycardia. (Choices D and E) Surgery and RAI are definitive therapies to be attempted after oral medication has decreased or controlled the thyrotoxic state, or if drug therapy fails. Educational Objective: Symptomatic tachycardia in patients with hyperthyroidism can be quickly controlled with betablockers (usually propranolol or atenolol). Treatment with PTU or methimazole will certainly improve the cardiac condition of the thyrotoxic patient, but only after several weeks. A 57-year-old female is started on subcutaneous erythropoietin injections for the treatment of anemia associated with endstage renal disease. Her hemoglobin and hematocrit levels at the start of erythropoietin therapy are 7 g/dL and 20%, respectively. She returns for a follow-up visit after six weeks of erythropoietin treatment, and her current hemoglobin and hematocrit levels are 7.1 g/dL and 22%, respectively. Her MCV is 82 cubic microns. She has been taking erythropoietin regularly. What is the next best step in the management of this patient? A. Increase erythropoietin dose. B. Start to administer erythropoietin intravenously in a higher dose. C. No change in treatment and follow up in 4 to 6 weeks. D. Measure percent transferrin saturation and ferritin levels. E. Start weekly blood transfusions. Explanation: Anemia of endstage renal disease is generally due to the decreased production of erythropoietin by the kidneys; however, other causes may contribute to the patient's anemia. These causes include iron deficiency, severe hyperparathyroidism (causes erythropoietin resistance), folate deficiency, systemic inflammation, and aluminum toxicity.

The goal of erythropoietin therapy is to increase the hematocrit level by 4 to 6% in 4-6 weeks time, and the final hematocrit level should lie between 33% and 36%. If the hematocrit level fails to increase adequately within 4 to 6 weeks following erythropoietin treatment, iron saturation and ferritin levels should be measured. Iron deficiency is treated by weekly intravenous administration of iron. (Choice A) There is no indication to increase the erythropoietin dose at this point. (Choice B) Intravenous administration is generally less successful than subcutaneous administration; the target hemoglobin level can be achieved at a lower dose with subcutaneous administration. (Choice C) The patient needs investigations for the cause of her suboptimal increase in hematocrit and hemoglobin levels. A follow-up visit in 4 to 6 weeks is not justified. (Choice E) Blood transfusion is usually avoided in patients with ESRD because they can sensitize a patient for allograft rejection following kidney transplantation. Educational Objectives: The anemia of patients with ESRD is generally due to erythropoietin deficiency. Other causes of anemia should be ruled out before starting erythropoietin therapy. Iron-deficiency anemia is a common cause of suboptimal increase in hematocrit and hemoglobin levels following erythropoietin therapy. A 55-year-old Caucasian male presents to the ER with fever, cough and chest pain. He also complains of right-sided headaches and recurrent nasal bleeding over the last two days. He underwent allogeneic bone marrow transplantation because of aplastic anemia. He had a mild episode of acute graft-versus-host disease soon after the engraftment, and this was successfully treated by high-dose corticosteroids. He received standard antimicrobial prophylaxis with ganciclovir, fluconazole and ciprofloxacin. Chest x-ray demonstrates bilateral patchy infiltrates. Which of the following is the most likely cause of this patient?s condition? A. Pneumocystis carinii B. Cytomegalovirus (CMV) C. Aspergillus D. Pseudomonas aeruginosa E. Legionella Explanation: Bone marrow transplant recipients are susceptible to a variety of infections, including bacterial, fungal, viral and protozoal infections. Carefully planned antimicrobial prophylaxis, as well as early recognition and treatment of established infections are important in decreasing morbidity and mortality in these patients. The clinical scenario describes three main categories of complaints: systemic infection (fever), pulmonary symptoms (cough and chest pain), and probable sinus symptoms (localized headache, nasal bleedings). Such constellation of symptoms is suggestive of an invasive fungal infection, which is most probably invasive aspergillosis. Risk factors for invasive aspergillosis include allogeneic transplantation (vs autologous transplantation), older age, acute graft-versus-host disease and corticosteroid therapy. Disseminated Candida infection is less likely because fluconazole prophylaxis was used. (Choices D, E and A) Gram-negative bacterial infection (P. aeruginosa and Legionella) and Pneumocystis carinii are important pathogens in patients who received bone marrow transplantation. These can cause pulmonary or disseminated infections; however, the combination of lung involvement and nasal/sinus infection is less typical in these infections.

(Choice B) CMV most commonly presents as fever of unknown origin, interstitial pneumonitis, or enteritis. Less often, these patients can develop retinitis, encephalitis, hepatitis, or bone marrow suppression. Ganciclovir prophylaxis significantly reduces the risk of CMV infection. Educational Objective: Invasive aspergillosis is common in bone marrow transplant recipients. respiratory tract, including the lungs and the sinuses. The following vignette applies to the next 3 items A 32-year-old woman comes to the emergency department (ED) for the evaluation of a low-grade fever and cough for the past five days. The cough is nonproductive and is associated with progressive difficulty in breathing. The patient easily gets short of breath with minimal walking and going upstairs. She was diagnosed with HIV infection approximately one year ago. She was started on antiretroviral treatment at that time, but it was stopped after two months due to intolerable side effects. Her last CD4 count done two months ago was 215 cells/cmm. On physical examination in the ED, her vital signs reveal a temperature of 37.9?C (100F), blood pressure of 110/64 mmHg, heart rate of 106/min and respiratory rate of 26/min. Her oxygen saturation is 92% on room air, and decreases to 86% with minimal walking. Her lung exam reveals scattered bilateral crackles and rhonchi. You suspect that she is suffering from Pneumocystis carinii pneumonia. Item 1 of 3 What should be the most appropriate next step for establishing a definite diagnosis of Pneumocystis carinii pneumonia? A. Start her on anti-retroviral therapy if the CD4 count is less than 200 cells per cubic millimeter. B. Obtain a high-resolution CT scan for confirmation of the diagnosis. C. Measure the diffusion capacity for carbon monoxide (DLCO). D. Perform a fiberoptic bronchoscopy with bronchoalveolar lavage for diagnosis. E. Legionella Explanation: Pneumocystis carinii pneumonia is seen in patients with defects in cell-mediated immune response, especially in those with HIV infections. It is usually seen in HIV-positive patients when their CD4 cell count decreases below 200 cells/cubic mm. It has an insidious onset and presents with fever, nonproductive cough, and progressive dyspnea and tachypnea. Chest x-ray of patients with Pneumocystis carinii pneumonia reveals diffuse and bilateral ground glass or alveolar infiltrates. It is generally recommended to establish a definite diagnosis of Pneumocystis carinii pneumonia before instituting specific therapy (some advocate initiating an empiric therapy in HIV-positive patients presenting with typical signs and symptoms). Fiberoptic bronchoscopy with bronchoalveolar lavage is the recommended procedure for the direct identification of organisms in the bronchoalveolar tree or respiratory secretions. It is a highly sensitive procedure and can be combined with a transbronchial biopsy to further improve its diagnostic yield. (Choice A) It is recommended to obtain further diagnostic workup for a definite diagnosis of Pneumocystis carinii pneumonia. (Choice B) Although a high-resolution CT scan has a high sensitivity for the diagnosis of Pneumocystis carinii pneumonia in a HIV-positive patient, it does not provide a specific diagnosis or documentation of the presence of Pneumocystis carinii in the respiratory secretions. (Choice C) A normal diffusion capacity for carbon monoxide essentially rules out the presence of Pneumocystis pneumonia in HIV-positive patients; however, an abnormal DLC does not provide a specific diagnosis of Pneumocystis carinii pneumonia. Educational Objective: It typically involves the

Fiberoptic bronchoscopy with bronchoalveolar lavage should be performed for direct identification of organisms in HIV-positive patients suspected of having Pneumocystis carinii pneumonia. Item 2 of 3 Which of the following is the most appropriate next step in the management of this patient? A. Patient aged less than 60 years B. Obtain a chest x-ray and treat with oral trimethoprim-sulfamethoxazole. C. Supply her with a two-month supply of aerosolized pentamidine. D. Admit the patient to the hospital and treat her with intravenous trimethoprim-sulfamethoxazole. E. Legionella Explanation: The patient in the above vignette has typical signs and symptoms of bilateral pneumonia, most likely Pneumocystis carinii pneumonia (PCP), in the setting of an HIV infection. She also has significant respiratory distress and marked desaturation with minimum activity. She has a very high risk of developing hypoxic respiratory failure and should be admitted to the hospital for close observation, oxygen therapy, and intravenous antibiotics. Trimethoprim-sulfamethoxazole (TMP-SMX) is the initial treatment of choice for all patients with suspected or documented PCP. Intravenous therapy should be given in the early stages of hospitalization and should be converted to oral therapy when the patient has signs of clinical recovery. (Choice A) Anti-retroviral treatment is an important part of the overall management of this patient, regardless of the CD4 cell count; however, specific treatment of Pneumocystis carinii pneumonia using TMP-SMX should first be started at this point. Anti-retroviral treatment alone may cause worsening of her respiratory failure due to an immune reconstitution phenomenon. (Choice B) Oral trimethoprim-sulfamethoxazole should be used only in patients with a mild pneumonia without any signs of impending respiratory failure. (Choice C) Intravenous pentamidine is the drug of choice for moderate or severe Pneumocystis carinii pneumonia in patients who are intolerant of trimethoprim-sulfamethoxazole. Aerosolized pentamidine is associated with a higher relapse rate, and is not recommended for treatment. Educational Objective: Intravenous trimethoprim-sulfamethoxazole should be used initially for all patients with moderate to severe Pneumocystis carinii pneumonia. Item 3 of 3 Which of the following is the most appropriate indication for the use of corticosteroids in this patient? A. Patient aged less than 60 years B. CD4 cell count of less than 50 per cubic millimeter C. Alveolar-to-arterial oxygen gradient of more than 35 mmHg on room air D. Suspected co-infection with Mycobacterium avium complex E. An arterial oxygen tension (PaO2) of less than 85 mmHg on room air

Explanation: The early use of corticosteroids has been shown to decrease the mortality rate and the rate of respiratory failure in patients with moderate to severe Pneumocystis carinii pneumonia (PCP). These observations are thought to be secondary to the anti-inflammatory effects, thus preventing further alveolar damage. Corticosteroid use is recommended in moderate to severe PCP infections with an

alveolar-arterial oxygen gradient of 35 mmHg or more, and/or an arterial oxygen tension (PaO2) of 70 mmHg or less on room air. (Choices A and B) The patient?s age and CD4 cell count do not influence the decision to use corticosteroids in patients with moderate to severe cases of PCP. (Choice D) Patients with suspected co-infections with Mycobacterium tuberculosis or Mycobacterium avium complex should not be treated with adjunctive corticosteroids. (Choice E) An arterial oxygen tension of less than 70 mmHg or less (not 85 mmHg) on room air is an indication for treatment with corticosteroids. Educational Objective: Adjunctive treatment with corticosteroids is recommended in patients with an alveolar-to-arterial gradient of 35 mmHg or more and/or an arterial oxygen tension of 70 mm or less in patients on room air. A new screening test is being evaluated for the early detection of stomach cancer. It is based on the measurement of a new serologic marker of gastric adenocarcinoma. The test is compared to traditional strategy of endoscopic evaluation of high-risk patients; this showed an increase in the survival of patients by several weeks. This increase in survival is statistically significant, although no difference is detected in the rate of radical gastrectomy between the two groups. Which of the following is the best explanation for the study results presented above? A. Low sensitivity B. Selection bias C. Length-time bias D. Lead-time bias E. Recall bias Explanation: A useful screening test is one that can detect the disease early enough to allow for an intervention which will improve the patient?s prognosis. The two components of a screening test which should then be evaluated are (1) early detection, and (2) associated increased survival; however, it is also important to detect any errors/bias which can possibly lead to an incorrect conclusion about the test. For instance, the phenomenon involving the screening test in this case can be explained by lead-time bias. Lead-time bias occurs when a screening test detects the disease at an earlier point in time (making it look like the survival rate increased), but the associated prognosis of the disease does not actually change. Since this test only diagnoses patients earlier than the standard test (i.e., endoscopy) but its associated prognosis is the same as the standard, its use offers no advantage to the standard screening test. (If you use this new test, you?re only going to let the people know that they have the stomach cancer at an earlier point in time, but you won?t be able to change their prognosis). (Choices A and B) Selection bias and low sensitivity cannot adequately explain the results of this study. (Choice E) Recall bias can distort the results of a case-control study because the investigators have to rely on the accounts of exposure as recalled by the participants of the study. Educational Objective: Lead-time bias occurs when a screening test detects the disease at an earlier point in time (making it look like the survival rate increased), but the associated prognosis of the disease does not change. (If you use this new test, you?re only going to let the people know that they have the disease at an earlier point in time, but you won?t be able to change their prognosis).

A 66-year-old African-American man is admitted to the hospital with complaints of constant low back pain for the last six weeks. The pain is insidious in onset, and is progressively getting worse. It does not radiate down his legs, and is not associated with any neurological symptoms. He has a history of hypertension, diabetes mellitus, and prostrate cancer treated with radical prostatectomy six months ago. His medications include aspirin, amlodipine, and metformin. You suspect vertebral metastasis from the prostrate cancer, and a radionuclide scan confirms your suspicion of the osteoblastic bony metastatic disease. Which of the following is the most appropriate initial treatment for this patient? A. Leuprolide alone B. Flutamide alone C. Diethylstilbestrol alone D. Chemotherapy with mitoxantrone E. Leuprolide and flutamide Explanation: This patient has presented with a metastatic prostrate cancer with spread to the vertebral column. Metastatic prostrate cancer remains a common clinical problem, despite early detection and treatment of the cancer. The most common site of spread is the bones, especially the axial skeleton. Skeletal metastases are usually osteoblastic, and should be suspected in a patient with prostrate cancer presenting with a new onset or worsening of chronic back pain. A radionuclide bone scan is the single most useful imaging modality for assessing any new bony metastasis. Androgen depletion is the primary treatment for men with a metastatic prostrate cancer. It is usually palliative rather than curative, and most patients lose the responsiveness to hormonal manipulations within two years. In most cases, luteinizing hormone releasing hormone (LHRH) agonists (leuprolide) are used as the first line therapy for advanced metastatic prostrate cancer. These bind to the LHRH receptors in the anterior pituitary gland and cause an initial release of luteinizing hormone (LH) and follicle stimulating hormone (FSH), which causes a transient rise of testosterone levels. After approximately one week of continuous therapy, there is down regulation of LHRH receptors, causing a decrease in the levels of LH. This leads to a significant decrease in the levels of testosterone. In the first week of therapy, the patient's symptoms tend to get worse during the initial testosterone surge. LHRH agonist monotherapy is contraindicated in patients with painful vertebral metastases and severe ureteral obstruction. These symptoms get worse during the initial testosterone surge seen in the first week of therapy. In such cases, an antiandrogen (e.g., flutamide) is given for one week before starting LHRH agonist monotherapy to block the effects of the initial testosterone surge. (Choice A) As mentioned above, LHRH agonist monotherapy is contraindicated in this patient. (Choice B) Antiandrogens used in patients with metastatic prostate cancer are flutamide, bicalutamide, and nilutamide. These block androgen receptors and prevent the growth-promoting effects of endogenous testosterone and dihydrotestosterone. These are associated with lower survival rates when used as a single agent. (Choice C) Diethylstilbestrol is a nonsteroidal estrogen, which exerts its effect by negative feedback action on the hypothalamic pituitary axis. It reduces the release of LHRH from the hypothalamus, thus decreasing the release of LH from the anterior pituitary. It is not available in the United States for commercial use, and its use is discouraged, due to the frequent side effects (e.g., myocardial infarction, stroke, and pulmonary embolism) associated with estrogen use. (Choice D) Patients with advanced, symptomatic, hormone-resistant prostrate cancer can be treated palliatively with chemotherapy. The optimal regimen for the chemotherapy is uncertain. Educational Objective:

LHRH agonists are the initial treatment of choice in patients with metastatic prostrate cancer. The concurrent administration of an antiandrogen for a week prior to LHRH therapy reduces the initial symptom flare.

An asymptomatic 42-year-old Caucasian woman comes to the physician because of "hair loss in groups" for the past six weeks. Her other medical problems include diabetes mellitus-type 1, gastroesophageal reflux disease, and bipolar disorder. She has smoked one pack of cigarettes daily for 15 years. She drinks 1-2 ounces of alcohol daily, and is a vegetarian. Her mother has bipolar disorder and breast cancer. Her medications include lansoprazole, glyburide, and lithium. Her vital signs are within normal limits. On close examination, the hairs have split-ends, and oil is visible on the strands. Which of the following is the most likely cause of her condition? A. Lithium B. Chemical reaction C. Oil toxicity D. Diabetes mellitus E. Allergic reaction Explanation: Because the hairs have split-ends, there should be a toxic or chemical reaction causing the hair loss. This can be the application of certain substances over the scalp or hair. (Choice A) Lithium is a recognized cause of alopecia. It produces thinning of the hair and widespread hair loss without damage to the hair itself. (Choice E) Allergic reactions will affect the scalp, and not the hair shaft. (Choice C) Oil is commonly used by women for hair grooming. It is only harmful if applied through hot compresses, in which case, it will also damage the scalp. (Choice D) Diabetes Mellitus has not been linked to alopecia. Educational Objective: Numerous drugs can produce hair loss. Lithium, thallium, and chemotherapeutic agents are between the most common ones; however, they characteristically do not affect the hair shaft, produce splitends, or other lesions. Traumatic alopecia, caused by traction, hair eating (trichotillomania), or chemical reactions, usually produces lesions on the hair strand or shaft itself, and is manifested as split-ends, trichoclasis, or fractured hair. The following vignette applies to the next 3 items A 45-year-old Caucasian female presents to the office because of progressive swelling, pain, and stiffness of the joints of her hand and wrist for the past several months. She says it takes hours to get some relief because her hands are so stiff in the early morning. Her past medical history is not insignificant. Her mother has a history of systemic lupus erythematosus. Examination of her joints shows warmth, swelling, and tenderness of the proximal interphalangeal joints, metacarpophalangeal joints, and wrists. Item 1 of 3 Which of the following is the most appropriate next step in the management of this patient? A. Order rheumatoid factor and prescribe ibuprofen B. Order antinuclear antibodies and start her on prednisone C. Check parvovirus titers and prescribe ibuprofen

D. Order anti dsDNA level and start naproxen E. Check serum uric acid level and start on colchicine Explanation: The above patient is most likely suffering from rheumatoid arthritis (RA). His clinical features of symmetric polyarthritis (involving MCP and PIP joints) associated with morning stiffness lasting for more than 30 minutes for several (more than six) weeks are compatible with the diagnosis of RA. This diagnosis is made clinically. The best next step in this case is to order rheumatoid factor and antinuclear antibodies because SLE patients can have a similar presentation, even though this is not classic. Hand x-rays should be ordered, and steroids should be avoided, if possible. For symptomatic relief, the patient should be started on NSAIDs, which are the first line drugs in the management of RA. (Choice C) Joint involvement in parvovirus infections is also symmetrical. The hands, wrists, knees, and feet are the most frequently involved joints. The patient may have arthralgias or arthritis, and a rash may or may not be present. Joint involvement most frequently occurs in adult females; however, the parvovirus infection will not last that long. (Choice E) Although the patient could be having a very atypical presentation of gout, NSAIDs should still be given to relieve her pain. Serum uric acid has no value in diagnosing the gouty arthritis. Educational Objective: Clinical features of symmetric polyarthritis (involving MCP and PIP joints) associated with morning stiffness lasting for more than 30 minutes for several (more than six) weeks are compatible with the diagnosis of rheumatoid arthritis. Item 2 of 3 X-ray of the hands shows periarticular osteopenia and erosions of the proximal interphalangeal and metacarpophalangeal joints. Her lab results are as follows: ANA negative anti dsDNA negative rheumatoid factor negative Parvovirus titers positive IgG One week later, she comes back and says that her symptoms are relieved by her new medications. Which of the following is the most appropriate next step in the management of this patient? A. Start her on low dose maintenance glucocorticoids B. Start her on methotrexate C. Start her on allopurinol D. Obtain PPD and start her on infliximab E. Tell her that she does not need additional treatment Explanation: The above patient has erosive joint disease. She should be started on disease-modifying antirheumatic drugs (DMARDs), because these agents slow down the progression of bony erosions and cartilage loss. According to current recommendations, DMARDs should be be used earlier in the course of disease. Other indications for the use of these agents are: disease refractory to conservative treatment, and dependence on steroids. Some examples of DMARDs are methotrexate, hydroxychloroquine, sulfasalazine, leflunomide, etanercept, infliximab, and azathioprine. Methotrexate is the initial drug of choice, but if the response is not adequate, other DMARDs may be used.

(Choice E) Hydroxychloroquine is inferior to methotrexate, even though it is very safer than methotrexate. Leflunomide has a role in patients who have contraindications to methotrexate, such as significant pulmonary fibrosis. (Choice D) Etanercept and infliximab are the new generation TNF inhibitors, and are highly effective in patients who have refractory disease with methotrexate. They are very expensive and are not indicated as the first line drugs of treatment. PPD is indicated before the treatment with TNF inhibitors is started. (Choice A) Glucocorticoids do not alter the natural course of the disease, and are indicated when the disease is refractory to NSAIDs and DMARDs, or when there are severe constitutional symptoms or extra-articular manifestations. (Choice E) Positive IgM antibodies make the diagnosis of parvovirus infection. IgG can be present in the general population. Educational Objective: Erosive joint disease in rheumatoid arthritis is a clear-cut indication for the use of disease-modifying anti-rheumatic drugs (DMARDs), and methotrexate is the initial drug of choice for this purpose. Item 3 of 3 What is the most likely diagnosis of this patient? A. Rheumatoid arthritis B. Systemic lupus erythematous C. Polymyalgia rheumatica D. Parvovirus infection E. Polyarticular gout Explanation: Rheumatoid arthritis (RA) is a clinical diagnosis. In patients with negative rheumatoid factor (RF) levels but with the classic clinical and radiologic presentation, the most likely diagnosis is still RA. Elevated RF levels are detected in approximately 75 to 80% of patients at some time during the course of their disease. A new test, called anti-CCP antibodies (anti-citrulline containing peptide antibodies) has some value in suspected patients with negative RF levels. Educational Objective: RA is a clinical diagnosis. RF levels may be negative in approximately 20% of the patients. An 82-year-old Caucasian female is brought to the emergency department (ED) with high-grade fever. She resides at a local nursing home. The nursing home staff reports that she has been acting strangely for the past two days. She has a past medical history of COPD, diabetes mellitus, hypothyroidism, atrial fibrillation, and severe, deforming rheumatoid arthritis. Her medications include aspirin, insulin, albuterol inhaler, levothyroxine, warfarin, and low-dose prednisone. In the ED, her temperature is 38.9?C (102F), blood pressure is 126/84 mmHg, respiratory rate is 20/min, and heart rate is 92/min. Her oxygen saturation is 96% on 3 liters oxygen. She is awake and alert, but does not appear oriented to her surroundings. Her mucous membranes are extremely dry. The rest of her physical examination is unremarkable. Initial diagnostic work-up reveals the following: Serum chemistry Serum Na: 158 mEq/L Serum K: 4.2 mEq/L Chloride: 108 mEq/L Bicarbonates: 22 mEq/L BUN: 68 mg/dL Serum Creatinine: 2.4 mg/dL Calcium: 10.2 mg/dL Blood Glucose: 140 mg/dL

Urine Specific gravity: 1.020 Blood: trace Glucose: negative Ketones: negative Leukocyte esterase: positive Nitrites: positive WBC: 50+/hpf RBC: 10-20/hpf Casts: none The patient is admitted to the hospital and started on intravenous fluids and antibiotic therapy. On the first night of admission, she becomes very confused and agitated. She pulls her IV line out, and this causes significant bleeding. She then hits the head nurse on the face. Which of the following is the best initial treatment option for this patient? A. Lorazepam B. Haloperidol C. Venlafaxine D. Buspirone E. Zolpidem Explanation: The patient in the above vignette is suffering from an acute confusional state, also known as delirium. This is a common occurrence in elderly, hospitalized patients, and is manifested as a change in the level of awareness, cognitive and perceptual disturbances, disorientation and agitation. Delirium is a multifactorial disorder. Some common conditions that can precipitate acute delirium include the use of multiple medications (polypharmacy), fluid and electrolyte disturbances (dehydration, hypo or hypernatremia), infections (urinary tract infections and pneumonia), malnutrition, immobility (hospitalization), and the use of bladder catheters. Supportive measures such as reassurance, frequent orientation, and constant supervision should be used initially to manage confusion and disorientation in such patients. If the patient demonstrates combative behavior, low-dose antipsychotics (haloperidol or risperidone) should be used to acutely control the symptoms, in order to prevent any harm and to allow for the safe continuation of treatment. (Choice A) Benzodiazepines have a rapid onset of action and can rapidly control agitation and combative behavior; however, these drugs can cause disinhibition and potentially worsen the acute confusional state. These can also suppress the respiratory drive, and should therefore be used with caution in patients with chronic respiratory disorders. (Choices C and D) Venlafaxine is a serotonin/norepinephrine re-uptake inhibitor which is used in the treatment of depression and generalized anxiety disorder. Buspirone is an anti-anxiety drug used for the treatment of generalized anxiety disorder. Both drugs have a slow onset of action and should not be used for acute management of agitated or aggressive behavior. (Choice E) Zolpidem is used in the treatment of insomnia. Educational Objective: High potency neuroleptics (haloperidol) should be used for the acute management of agitated and combative hospitalized patients. The following vignette applies to the next 2 items

A 60-year-old woman is brought to the emergency department following a motor vehicle accident. She hit a car from her side and claims that she did not see another car coming towards her. This accident took place at a small intersection with little traffic. Her past medical history is significant for Cushing?s disease and an intra-abdominal operation 15 years ago for the disease. Physical examination showed a tanned female with normal vital signs. She has a few abrasions on her face and chest. Ophthalmologic examination reveals bitemporal hemianopsia. Her injuries are managed appropriately. Item 1 of 2 What is the next best step in the management of this patient? A. X-ray of the skull B. MRI of the brain C. CT angiogram D. Waters? view E. Slit lamp examination Explanation: The clinical features in this patient are suggestive of a mass lesion in the sellar and suprasellar region. Furthermore, the history of bitemporal hemianopsia and hyperpigmentation following an abdominal operation (bilateral adrenalectomy) is very suggestive of Nelson?s syndrome. The increased pigmentation is due to melanocytes, stimulating the activity of ACTH. MRI and plasma ACTH levels are required for making the diagnosis. A pituitary microadenoma with suprasellar extension on MRI and extremely high plasma ACTH levels are diagnostic. (Choice A) X-ray of the skull may show an enlarged cella, but a plain x-ray is much less sensitive than MRI in the diagnosis and evaluation of pituitary masses. (Choice C) Bitemporal hemianopsia cannot be localized to any specific cranial arterial territory; therefore, CT angiography is not warranted. (Choice D) Waters? view is done to evaluate sinusitis. It is not used in the evaluation of pituitary adenomas. (Choice E) The clinical features are not suggestive of intraocular defect; thus, a slit lamp exam is unlikely to be helpful. Educational Objective: Bitemporal hemianopsia and hyperpigmentation following bilateral adrenalectomy for Cushing?s disease is suggestive of Nelson?s syndrome. A pituitary microadenoma with suprasellar extension on MRI and extremely high plasma ACTH levels are diagnostic. Item 2 of 2 What is the most likely diagnosis? A. Prolactinoma B. Nelson?s syndrome C. Craniopharyngioma D. Brain tumor E. Empty cella syndrome Explanation: Pituitary enlargement and hyperpigmentation following bilateral adrenalectomy for Cushing?s disease is termed as Nelson?s syndrome. The cause of pituitary enlargement is the loss of feedback by the adrenal glucocorticoids following bilateral adrenalectomy. The tumor in Nelson?s syndrome is aggressive and is treated by surgery and/or pituitary radiation. Following bilateral adrenalectomy, prophylactic pituitary radiation sometimes prevents the development of Nelson?s syndrome; however,

this leads to an increased risk for hypopituitarism. Previously, bilateral adrenalectomy was the preferred treatment for Cushing?s disease; however, with the advent of better localization and improved techniques of transsphenoidal surgery, primary pituitary surgery is now the preferred treatment for Cushing?s disease. (Choice A) Prolactinoma can cause bitemporal hemianopsia but will not lead to increased pigmentation. Furthermore, the past history of bilateral adrenalectomy favors the diagnosis of Nelson?s syndrome. (Choice C) Although craniopharyngioma can occur in adults, it is uncommon at this age. The clinical features of craniopharyngioma are visual field defects and hypopituitarism. Diabetes insipidus is more common compared to pituitary macroadenoma. (Choice D) Except for parasellar or intrasellar meningiomas, brain tumors usually do not present with bitemporal hemianopsia. Hyperpigmentation does not occur in brain tumors. (Choice E) Visual field defects are uncommon in patients with empty sella syndrome. Educational Objective: Nelson?s syndrome is characterized by pituitary enlargement, hyperpigmentation, and visual field defect following bilateral adrenalectomy. Usually, these tumors are rapidly growing and can be treated with surgery and/or local radiation.

A group of researchers wants to investigate an outbreak of acute diarrhea that occurred in a small coastal town. Approxiately 50 people had a severe hemorrhagic diarrhea, and one fatal case was reported. The researchers believe that the outbreak is related to the seafood prepared at one of the coastal restaurants. Which of the following study designs is most appropriate to test the hypothesis? A. Cohort study B. Cross-sectional study C. Case-control study D. Correlational study E. Clinical trial Explanation: A case-control study is the most appropriate study design to investigate an outbreak of an acute infectious disease. The subjects who developed the disease should be questioned about their recent exposures (eating a particular dish at a particular restaurant in this case), and unaffected subjects should be chosen as controls to reflect the exposure experience of the general population. If cases are more likely to be exposed than the controls (i.e., the odds ratio is > 1 and statistically significant), then an association between the exposure and the disease can be established. (Choice A) A cohort study is not appropriate because the exposure status of already diseased subjects is of primary interest. (Choice E) A clinical trial is clearly not ethical. (Choices B and D) Cross-sectional and correlational studies are not informative in such cases.

Educational Objective: A case-control study is the most appropriate study design to investigate an outbreak of an acute infectious disease. It allows for the quick localization of the source of the outbreak. The following vignette applies to the next 2 items A 19-year-old female was found to have a 2.1 cm nodule in her right thyroid lobe. She is otherwise healthy and takes no medications. She has no family history of thyroid diseases or a history of exposure to radiation in the past. She denies any pain, difficulty in swallowing, or voice change. Physical examination is otherwise normal. Labs reveal normal T3, T4, and TSH levels. Ultrasound of the thyroid gland shows a solid solitary nodule measuring 2.6 x 1.7 x 1.3 cm in the right lobe. The left lobe appears to be normal in size and echotexture. Item 1 of 2 What is the next best step in the management of this patient? A. Perform a fine needle aspiration biopsy of the right lobe nodule B. Start the patient on suppressive doses of levothyroxine to decrease the size of the nodule C. Refer to a surgeon for thyroidectomy D. Repeat thyroid ultrasound next year to monitor the size of the nodule E. Perform a radioactive iodine uptake and scan Explanation: Nodules in the thyroid gland are very common, particularly in older subjects. Thyroid nodules measuring 1 cm or more should be subjected to a fine needle aspiration biopsy. Nodules measuring less than 1 cm in size are generally followed with thyroid ultrasound on a yearly basis. (Choice D) This nodule is more than 1 cm, therefore repeating the ultrasound after one year is incorrect . (Choice E) A radioactive scan is seldom useful in the management of thyroid nodules. Although the incidence of thyroid cancer is higher in cold nodules (a focal decrease in radioactive iodine uptake), the majority of cold nodules are benign. Radioiodine uptake and scan are useful for diagnosing toxic adenoma in thyrotoxic patients. (Choice C) Most of these nodules are benign. Surgery is not warranted in benign thyroid nodules unless the nodules are very large or the patients have compressive symptoms. (Choice B) Benign nodules grow very slowly over time. A suppressive dose of levothyroxine has not been found to be consistently useful in reducing their size, therefore its use is not recommended. Furthermore, suppressive doses of levothyroxine can cause bone loss and atrial fibrillation. Educational Objective: Thyroid nodules measuring 1 cm or more should be subjected to a fine needle aspiration biopsy. Nodules measuring less than 1 cm in size are generally followed with thyroid ultrasound on a yearly basis. A radioactive scan is seldom useful in the management of thyroid nodules. Radioiodine uptake and scan are useful for diagnosing toxic adenoma in thyrotoxic patients. Item 2 of 2 Further workup was very suspicious for medullary thyroid cancer. Serum calcitonin was checked and found to be elevated at 300 pg/mL (0-10 pg/mL is normal). Genetic studies reveal a mutation in the RET proto-oncogene. Which of the following is the next best step in the management of this patient? A. Refer for thyroidectomy B. Measure plasma free metanephrine C. Refer to an oncologist for cancer chemotherapy D. Treat with radioactive iodine

E. Treat with external beam radiation Explanation: This patient is likely to have MEN 2 syndrome caused by RET proto-oncogene mutation. Medullary cancer arises from parafollicular cells rather than thyroid follicular cells. Medullary cancer could be familial (familial medullary cancer) or associated with other endocrinopathies, like pheochromocytoma and hyperparathyroidism in MEN 2 syndromes. Pheochromocytoma may be present in 40% of MEN 2 patients, although some of them are not clinically apparent. (Choice A) Screening for pheochromocytoma is mandatory in these cases before subjecting them to thyroid surgery. Undiagnosed pheochromocytoma can cause life-threatening hemodynamic complications during and after surgery. (Choice D) Medullary thyroid cancer cells do not take up iodine; therefore, radioactive iodine is not used in these patients. (Choices C and E) Anti-cancer drugs and external beam radiation play no role in the initial management of medullary thyroid cancer . Educational Objective: Recognize the importance of screening for pheochromocytoma in patients with medullary thyroid cancer. A healthy 15-year-old girl comes to the clinic with her boyfriend because she has just discovered she is pregnant. Her last menstrual period was approximately seven weeks ago. She states that her parents would be quite upset if they discovered she was sexually active. She is certain that she wants to have an elective abortion and requests a referral to a local provider. Given the circumstances, who should provide legal consent for the abortion? A. Patient B. Parent C. Sibling D. Boyfriend E. Court Explanation: As a physician, it can be difficult to balance the adolescent?s right to confidentiality against the parental right to oversee the child?s health. To address this conflict, most states have implemented laws that allow for a physician to provide care to adolescents without parental consent. These laws are typically limited to the management of certain issues, including pregnancy, contraception, sexually transmitted diseases, substance use, and emotional illness. Therefore, it is the teenaged patient who will ultimately provide consent for the abortion. (Choices B, C, and D) Abortion providers do not normally consult a patient?s parents, siblings, or boyfriend for approval before carrying out the procedure. Some states, however, do have parental notification laws regarding abortion, which the patient should be instructed about. (Choice E) Typically, teenaged girls are able to obtain abortions without the approval of the court. However, the court is sometimes petitioned to allow for an exception to parental notification laws, especially in cases of abuse or incest. Educational Objective: Most states have implemented laws that allow for a physician to provide certain types of medical care to adolescents without parental consent. Elective abortions are typically protected under these laws. Therefore, a teenaged girl would have the authority to provide legal consent for an abortion.

A 17-year-old female comes to your office for an annual physical examination. Her best friend's mother recently died from metastatic breast cancer. She seems very concerned, and requests a clinical breast examination and a referral for a screening mammography for herself. Which of the following is the most appropriate response to her request? A. Clinical Breast Examination (CBE) alone has never been shown to reduce breast cancer mortality in the clinical studies B. Monthly Breast Self Examination (BSE) is the most effective screening tool to detect early breast cancer and reduce mortality C. She should start an annual screening mammography to detect early breast cancer from age 30 years D. Annual or biennial screening mammography has shown to reduce breast cancer mortality across all age groups E. We should start annual clinical breast examination and screening mammography at 20 years of age Explanation: Breast cancer is the second leading cause of cancer deaths in women after lung cancer. The concerns and apprehension in women associated with breast cancer are understandable, given the physical and emotional impact of the disease. According to the USPSTF guidelines, there is insufficient evidence to recommend for or against routine clinical breast examination (CBE) alone as a screening tool for breast cancer. This is based on the fact that there are no clinical studies that have examined the benefits of CBE alone. (Choice B) The USPSTF found poor evidence to support the hypothesis that breast self examination (BSE) reduces breast cancer mortality. On the other hand, it found fair evidence that BSE increases the chances of false positive results and biopsies. (Choices C, D, and E) The USPSTF recommends screening mammography, with or without clinical breast examination, every one to two years for women aged 40 or older. This is based on the evidence from clinical studies that screening mammography every 12 to 33 months reduces mortality from breast cancer. This evidence is strongest for women 50-69 years of age, and weaker for women 40-49 years of age. Older women (> 70 years) have a higher probability of developing breast cancer, but also have a higher chance of dying from an unrelated cause. Therefore, screening in patients who are 40-49 years or older than 70 years of age should be done after appropriate clinical considerations and patient counseling. Educational Objective: The current USPSTF guidelines recommend screening mammography, with or without clinical breast examination, every one to two years for women aged 40 or older.

A 54-year-old Caucasian woman comes to the office for abdominal discomfort. She has been feeling weak for the past seven days. She is not eating well because of nausea, and has noticed that her urine has become darker. She has hypertension, generalized anxiety disorder, and obesity. Her medications include lisinopril, hydrochlorothiazide and aspirin. She has been taking some herbal medications for the past four months. She was using gingko biloba as an energy booster and kava to control her anxiety. Two months ago, she started to use ginseng in an attempt to improve her memory problems. She states that she had been feeling better, less anxious, and more alert, until last week. She does not use tobacco, alcohol or drugs. Her family history is not significant. She has no known allergies. Examination shows mild jaundice. Heart sounds are normal and lung sounds are clear. The liver is palpable 2 cm below the costal margin and tender. Neurologic examination reveals no abnormalities. Her laboratory tests reveal the following: CBC

Ht: 44% Platelet count: 220,000/cmm Leukocyte count: 9,000/cmm Segmented neutrophils: 55% Lymphocytes: 42% Monocytes: 3% Serum Chemistry Serum Na: 143 mEq/L Serum K: 4.2 mEq/L Chloride: 106 mEq/L Bicarbonate: 26 mEq/L BUN: 21 mg/dL Serum Creatinine: 1.0 mg/dL Calcium: 9.7 mg/dL Blood Glucose: 98mg/dL LFT Total bilirubin: 3.2 mg/dL Direct bilirubin: 2.8 mg/dL Alkaline phosphatase: 200 U/L Aspartate aminotransferase: 350 U/L Alanine aminotransferase: 420 U/L Which of the following is the most probable cause of the patient?s disease? A. Viral hepatitis B. Enalapril C. Kava D. Gingko biloba E. Ginseng Explanation: The patient?s laboratory test results show that there is moderate hepatic injury. Kava (Piper methysticum) has been identified as a cause of hepatitis, cirrhosis and liver failure. Several patients have died in Europe, Canada and the United States after its ingestion. The mechanism is still unclear, but the United States Food and Drug Administration (FDA) has already issued a warning to the public concerning its use (year 2002). (Choice A) Viral hepatitis is usually associated with AST and ALT levels over 500 units. The classic presentation is a period of fever, malaise, asthenia, nausea and vomiting, followed by the development of jaundice. This patient?s clinical features are different. (Choice B) Enalapril is metabolized in the kidney. hepatotoxic drug. There are no cases reporting its role as a

(Choice D) Gingko has been related to bleeding (due to its antiplatelet activity) and seizures. These adverse effects are generally reversible, temporary and mild. There are no reports of liver toxicity with this herb. (Choice E) Ginseng has been associated with some serious adverse effects (i.e., schizophrenia, severe headache, and Stevens-Johnson syndrome), as well as other less dangerous effects (i.e., vaginal bleeding, insomnia, diarrhea or mastalgia). Its use has not been related to liver damage. Educational Objective: Some popular herbal remedies have significant side effects and can be toxic to a specific organ. Aconite is cardiotoxic, and may cause arrhythmias and hypotension. Kava can cause liver injury.

Gingko biloba has been related to bleeding, while ginseng has been associated with Stevens-Johnson syndrome and psychosis. An 82-year-old Caucasian man presents to the emergency department for the evaluation of increased urinary frequency and burning, and mental confusion. He is admitted to the hospital, and is started on intravenous fluids and antibiotics. The next day, during the rounds, he is found to be in mild respiratory distress. His physical examination reveals the presence of jugular venous distention (9 cm) and bilateral basilar crackles. His chest x-ray is consistent with pulmonary edema. A 2D echocardiogram done two months ago revealed an ejection fraction of approximately 35%. You give the patient 40 mg of IV furosemide (Lasix), after which his respiratory status improves. His mental status remains unchanged, and he occasionally has episodes of agitation and visual hallucinations. The attending physician tells you to start him on medications for heart failure that would not affect his mental status. Which of the following is the most appropriate drug to start at this point? A. Perform lumbar puncture and start antibiotics B. Digoxin C. Spironolactone D. Verapamil E. Ginseng Explanation: The patient in the above vignette is admitted to the hospital with urinary tract infection and possible urosepsis. Acute infections are a common cause of delirium, especially in elderly patients being admitted to the hospital. An important goal of treatment in such patients is to avoid the use of medications that can potentially worsen the acute delirious state. Diuretics, ACE inhibitors, spironolactone, beta-blockers, and digoxin are all a part of standard therapy for patients with heart failure or cardiomyopathy due to any cause. ACE inhibitors (lisinopril) have consistently been shown in multiple, large, randomized trials to have a significant beneficial effect on patients with heart failure. They have a very low incidence of central nervous system (CNS) side effects, and do not cause worsening of confusional states or delirium. Lisinopril can be safely started in this patient without the risk of exacerbating his confusion and agitation. (Choice B) Digoxin is a cardiac glycoside frequently used in patients with congestive heart failure due to systolic dysfunction and atrial dysrhythmias; however, its use has been associated with significant CNS side effects. These include blurred vision, dizziness, confusion, mental disturbances, anxiety, delirium, and hallucinations. Its use should be avoided in patients with ongoing acute mental confusion or delirium. (Choice C) Spironolactone is a useful adjunct in the management of patients with congestive heart failure, especially in patients with a low ejection fraction; however, its use can also cause CNS disturbances such as drowsiness, lethargy, and mental confusion. (Choice D) Verapamil is a calcium channel blocker used in the treatment of hypertension, angina, and supraventricular tachyarrhythmias. It has a lower incidence of CNS side effects, but can precipitate or exacerbate heart failure symptoms. It should be avoided in patients with congestive heart failure. Educational Objective: Digoxin use can exacerbate confusion and mental disturbances and should be avoided in patients with delirium.

A 66-year-old Asian-American woman presents to the emergency department and complains of lower back pain, difficulty walking, urinary incontinence, fever of 39.3C (102.8F), and decreased appetite.

She was recently diagnosed with a herniation of disc L4-5 and was given an epidural injection for radicular pain two days ago. Her past medical history is significant for diabetes mellitus, chronic renal failure, and hypertension. She is alert and oriented. Her temperature is 38.3C (100.9F), blood pressure is 136/88 mm Hg, pulse is 86/min, and respirations are 14/min. Physical findings include poor dentition, diffusely tender abdomen, decreased rectal sphincter tone, mild tenderness upon palpation of the lumbar region, and absent deep tendon reflexes in both lower extremities. Initial laboratory examination includes a WBC of 22,000/mm3 with 69% neutrophils and 15% bands, and an erythrocyte sedimentation rate of 104 mm/hr. The result of her urinalysis is normal. What is the most appropriate course of action in this woman?s care? A. Perform lumbar puncture and start antibiotics B. Obtain MRI of spine C. Immediate neurosurgery consult D. Administer antibiotics E. Obtain bone scan Explanation: An epidural abscess should be suspected in a susceptible patient who presents with fever and back pain. Patients who are susceptible are the elderly, immunocompromised individuals (i.e., patients with AIDS, cancer, diabetes mellitus, chronic renal failure, or alcoholism), and patients who were recently involved in trauma, spinal surgery, or administration of epidural anesthesia. The diagnosis must be made immediately because delays in treatment increase the risk of permanent neurologic damage or death. A gadolinium-enhanced MRI is the preferred imaging modality to confirm the diagnosis because it adequately demonstrates inflammatory changes in the soft tissue. When MRI is not available, CT with myelography is an alternative. Once the diagnosis is made, antibiotic therapy should be guided by obtaining a culture specimen via CT-guided aspiration or open biopsy. The most common etiologic agent is Staphylococcus aureus. Spinal epidural abscesses is a surgical emergency; early surgical decompression and drainage, preferably within the first 24 hours, is the most important part of management to improve the ultimate prognosis. (Choice A) Lumbar puncture is not required to make the diagnosis and is not recommended. However, blood cultures should be drawn and antibiotics should be started. (Choice C) A neurosurgery consult should be obtained once more information is collected that suggests a diagnosis. (Choice D) Even though antibiotics are important, making the diagnosis and urgent surgical drainage is more important. Educational Objective: Epidural abscesses are best diagnosed with gadolinium-enhanced MRI of the spine. Spinal epidural abscess is a surgical emergency. Early surgical decompression and drainage, preferably within the first 24 hours is the most important part of management to improve the ultimate prognosis. A healthy 48-year-old African-American man comes to the physician for a routine health maintenance examination. He has no complaints. He has no medical problems. He does not drink alcohol. He has smoked one pack of cigarettes daily for 22 years. He takes no medication. His father was diagnosed with type-2 diabetes at the age of 48. His vital signs are within normal limits. Examination shows no abnormalities. Which of the following is the most appropriate next step in the management of this patient? A. Check random blood sugar B. Obtain fasting blood sugar C. Obtain glycosylated hemoglobin level D. No screening required at this time E. Perform glucose tolerance test

Explanation: Screening for diabetes mellitus (DM) is indicated in everyone aged 45 years or older, and should be repeated every three years in the absence of other risk factors. Screening should be performed earlier and more frequently if risk factors are present. The recommended screening test is measurement of the fasting blood glucose level. Risk factors for DM include: 1. age 45 years or greater 2. family history of diabetes in a first-degree relative 3. overweight 4. dyslipidemia 5. hypertension 6. impaired glucose tolerance or impaired fasting glucose 7. history of vascular disease 8. habitual physical inactivity 9. history of gestational DM or history of delivering a baby weighing greater than 9 lbs 10. polycystic ovarian disease 11. high-risk race (like African-American or Hispanic) The above patient has two risk factors for DM (e.g., history of diabetes mellitus in his father, and above 45 years of age); therefore, he needs to be screened by obtaining his fasting blood glucose levels. (Choice C) Glycosylated hemoglobin levels are not currently recommended for diabetes mellitus screening. It is used for monitoring the disease activity. (Choice A) Measurement of random blood sugar levels may be used for DM screening when a person comes without overnight fasting; however, obtaining fasting blood sugar levels is the more preferred screening tool. (Choice E) A glucose tolerance test is usually not used for DM screening in non-pregnant patients. Educational Objective: Measurement of fasting blood sugar levels to screen for DM is recommended in (1) all persons aged 45 or above, and (2) all persons who are at high risk for DM. The following vignette applies to the next 2 items Item 1 of 2 A 26-year-old African American female comes to the emergency department with fever and a dry cough. She is also complaining of weakness, loss of appetite, a 10-lb weight loss, nausea, and mild constipation. She denies any medical problems in the past and was fine until four weeks ago, when all these symptoms started. Her symptoms are gradually worsening. She denies any headaches, visual problems, abdominal pain, skin rashes, dysuria or polyuria. She is currently taking over-the-counter acetaminophen. Her family history is unremarkable. She is a single mother, and lives with her three children. She has a 5-pack year history of smoking, and still smokes one pack daily. She occasionally drinks alcohol. The physical examination reveals a sick-appearing woman in no acute distress. Her blood pressure is 100/70 mm Hg, heart rate is 96/min regular, and temperature is 99? F (37.2C). Lymph nodes are palpated in the cervical, axillary and inguinal areas bilaterally. Her skin is normal; and there is no pallor or icterus. Her extremities have no clubbing, cyanosis or edema. The other systems are normal. Laboratory investigations: Serum Na: 142 mEq/L Serum K: 3.9 mEq/L Chloride: 100 mEq/L Bicarbonate: 32 mEq/L BUN: 32 mg/dL Serum Creatinine: 1.3 mg/dL

Calcium: 11.4 mg/dL Blood Glucose: 65 mg/dL Aspartate aminotransferase: 45 U/L Alanine aminotransferase: 56U/L Albumin: 3.2 g/l Serum TSH: 0.4 mU/ml Chest-x ray showed multiple parenchymal nodules. What is the most likely cause of her hypercalcemia? A. Hyperparathyroidism B. Hyperthyroidism C. Overproduction of 1, 25-dihydroxyvitamin D D. Renal failure E. Humoral hypercalcemia of malignancy Explanation: The patient has several clinical features suggestive of sarcoidosis. Sarcoid granulomas produce the enzyme 1-alpha-hydroxylase, which converts 25-hydroxyvitamin D to 1,25-dihydroxyvitamin D, leading to an increase in the gastrointestinal absorption of calcium. In hypercalcemia due to sarcoidosis, PTH secretion is suppressed, and urinary calcium excretion is increased. Hypercalcemia caused by overproduction of 1,25-dihydroxyvitamin D can also occur in other granulomatous diseases, such as fungal infections, berylliosis and tuberculosis. (Choices A and E) Hypercalcemia in this patient is unlikely to be induced by an increase in PTH or PTHrP. (Choice B) Hypercalcemia in thyrotoxicosis is very mild. Although some of the patient's clinical manifestations suggest thyrotoxicosis, her TSH level is borderline-low and not suppressed. Furthermore, the lymphadenopathy cannot be explained by thyrotoxicosis. (Choice D) The patient has an increased BUN level due to dehydration. This may be corrected with hydration, and is an unlikely cause of her moderately severe hypercalcemia. Educational Objective: Sarcoid granulomas produce the enzyme 1-alpha-hydroxylase, which converts 25-hydroxyvitamin D to 1,25-dihydroxyvitamin D, leading to an increase in the gastrointestinal absorption of calcium. The resulting hypercalcemia leads to a suppressed PTH secretion and an increased urinary calcium excretion. Item 2 of 2 Which of the following is the most appropriate next step in the management of this patient? A. Alendronate B. Glucocorticoids C. Hydration and glucocorticoids D. Hydration and Pamidronate E. Calcitonin Explanation: Since the hypercalcemia in sarcoidosis is caused by an increased calcium uptake, glucocorticoids and hydration are sufficient to bring the calcium levels to normal. Hypercalcemia secondary to sarcoidosis responds quickly to glucocorticoids. Serum calcium levels return to normal within days after starting 40-60 mg/d of prednisone. If within 10 days of glucocorticoid use, the serum calcium levels remain elevated, alternative diagnoses for hypercalcemia should be sought.

(Choice B) Since this patient is also dehydrated, giving intravenous fluids with a glucocorticoid is a better choice for her treatment than treatment with a glucocorticoid alone. (Choices A and D) Alendronate is a bisphosphonate given orally. It is approved for the treatment of osteoporosis and Paget?s disease. Its oral absorption is very poor (<1 %), and it has a modest effect in lowering calcium levels. Pamidronate is a potent bisphosphonate given as a slow intravenous infusion for the treatment of hypercalcemia. The calcium lowering effect of bisphosphonates is mainly due to the decrease in bone reabsorption. Furthermore, the use of bisphosphonates has not been studied systematically in patients with sarcoidosis. (Choice E) Similarly, calcitonin acts mainly by decreasing bone resorption, and is not warranted in this patient. Educational Objective: Hypercalcemia secondary to sarcoidosis responds quickly to glucocorticoids. A 27-year-old male comes to the emergency department with bilateral flank pain and red-colored urine. Approximately three days ago, he experienced flu-like symptoms with nasal discharge and throat pain. His family history is unremarkable. He has been healthy all his life. He does not smoke cigarettes or drink alcohol. Review of systems is otherwise unremarkable. His blood pressure is 126/76 mmHg, pulse rate is 88/min, temperature is 99?F (37.2C), and respiratory rate is 18/min. He has mild bilateral costovertebral tenderness. The rest of the physical examination is normal. Lab investigations reveal normal complete blood count and basic chemistry profile. The urine is positive for blood and protein, with a specific gravity of 1.018 and pH of 5.3. Microscopic examination of urine reveals numerous dysmorphic red blood cells with occasional red blood cell casts. Spiral CT scan of his abdomen performed in the emergency department is normal. Levels of serum complements are also normal. What is the most likely diagnosis? A. Acute poststreptococcal glomerulonephritis B. Thin membrane disease C. Hereditary nephritis D. Renal stones E. IgA nephropathy Explanation: IgA nephropathy is a disease characterized by the deposition of IgA in the renal glomerulus. It has an unclear etiology. The common presentation is gross hematuria following an acute upper respiratory infection. Flank pain can also occur, and this is possibly caused by stretching of the renal capsule. Dysmorphic red blood cells in the urine are very suggestive of a glomerular process. Progression of renal disease in IgA nephropathy is highly variable. Majority of the patients have a benign cause with very slow progression of the renal disease. Male patients with hypertension and 24-hour urinary proteins more than 1 g are likely to progress rapidly. There is no definitive therapy for IgA nephropathy. ACE inhibitors and fish oil treatment have been tried. (Choice A) Acute poststreptococcal glomerulonephritis follows a streptococcal throat infection with a gap of more than 10 days between throat infection and onset of renal disease. Gross hematuria is uncommon. Patients are typically hypertensive. Urine sediment reveals RBCs and RBC casts. Complement levels are decreased. (Choice B) IgA nephropathy and thin membrane disease have certain similarities which may sometimes make it extremely difficult to differentiate. For instance, these two diseases have a benign cause with virtually no progression of the renal disease. Thin membrane disease also presents with episodic hematuria, although most episodes of hematuria in thin membrane disease are unrelated to an upper respiratory tract infection. The best way to differentiate these two diseases is by performing a renal biopsy. (Choice C) Hereditary nephritis also causes micro- or gross hematuria; however, patients have a positive family history of hematuria and renal failure, and there is usually an associated deafness.

(Choice D) Patients with renal stones present with flank pain and hematuria; however, urine examination will usually show normal appearing RBCs, which suggests a non-glomerular origin of urinary bleeding. RBC casts are distinctly uncommon. Spiral CT scan is a very sensitive test for the screening of renal stones. Due to all these reasons, the diagnosis of renal stones is unlikely in this patient. Educational Objective: Gross hematuria following acute upper respiratory infection is a common presentation of IgA nephropathy. Acute poststreptococcal glomerulonephritis follows a streptococcal throat infection with a gap of more than 10 days between throat infection and onset of renal disease. Many patients with IgA nephropathy have a benign course. There is no definitive treatment for IgA nephropathy.

A 5-year-old toxic-appearing boy is brought to the emergency department by his babysitter, who reports that over the past 12 hours he has become increasingly irritable and lethargic. The boy keeps saying that his "head hurts bad." The babysitter says that the boy is refusing food and has vomited twice. She has been watching him while his parents attended a local retreat hosted by their church, and has already called to alert them of the situation. Physical examination reveals evidence of fever and nuchal rigidity, and meningitis is suspected. Just before treatment is begun, the parents arrive and insist that they be allowed to take the boy home immediately. Their religious beliefs preclude them from "interfering with God?s will," and they refuse all medical treatment. What is the best means of handling this situation? A. Proceed with evaluation and treatment of child B. Obtain consent from babysitter and proceed with evaluation and treatment of child C. Obtain consent from grandparents and proceed with evaluation and treatment of child D. Consult with the hospital?s ethics committee about seeking court injunction to mandate evaluation and treatment E. Contact court directly to obtain injunction to mandate evaluation and treatment Explanation: It can be challenging to balance the autonomy of the family with the welfare of the child. Parental wishes should be honored within certain parameters, but if their requests will result in severe injury or death to the child, it is accepted practice among physicians to challenge parental authority. In emergency situations requiring imminent treatment, most medical teams choose to proceed with treatment despite the parental refusal of care (Choice A). Very rarely do courts uphold charges against physicians in such cases, and some states explicitly allow for such treatment (though they may require at least two doctors to agree that medical treatment is necessary to prevent immediate harm to the child). Consent from the babysitter (Choice B) or grandparents (Choice C) would be of no use in this situation. The parents have made their wishes clear, and they are the legal caretakers of the child. The necessary treatment will have to be performed without consent. If there is sufficient time, the hospital ethics committee is normally contacted about seeking a court injunction to legalize the process (Choice D). However, bacterial meningitis is a medical emergency and requires immediate treatment; in such circumstances, there is not even enough time to contact the courts directly for permission to proceed (Choice E). Educational Objective: When providing medical care, clinicians must seek to balance the autonomy of the family with the welfare of the child. In true emergency situations requiring imminent treatment of a child, most medical teams choose to proceed with treatment despite the parental refusal of care. Very rarely do courts uphold charges against physicians in such cases. A 22-year-old Caucasian male graduate student presents to the clinic at his mother?s urging because he has been feeling very sad and apathetic. He has been experiencing recurrent nightmares and auditory hallucinations about a motor vehicle accident that he was in three weeks ago. He was the driver of the vehicle, and his father and sister were with him at that time. He remains unable to recall the moment of the actual accident, although witnesses reported that the other driver lost control of his vehicle before colliding with the patient?s car. The patient was unable to extricate either of his family members from the car, and they both died before emergency personnel arrived at the scene. He felt tremendous horror and fear upon realizing that he would not be able to help them in time, and continues to feel guilty about this failure. He cannot recall much more about the accident, and says that he was "in a daze," feeling numb and unable to grieve for many days afterward. He continues to avoid the freeway where the accident took place, even if it means taking a lengthy detour. Since the accident, he has had some difficulty sleeping because he dreads his recurrent nightmares, and he finds that he is very irritable and restless throughout the day. He is unable to concentrate on his studies at school and stopped attending classes. Because of this, a graduate advisor filed leave of

absence paperwork on his behalf. The patient?s medical history is significant for patellofemoral syndrome and acne vulgaris. He takes no medications and has no allergies. He lives at home with his mother and does not drink alcohol, smoke cigarettes, or use recreational drugs. The physical examination is unremarkable. What is the most likely diagnosis? A. Major depressive disorder B. Dysthymic disorder C. Post-traumatic stress disorder D. Acute stress disorder E. Normal bereavement Explanation: The DSM-IV criteria for acute stress disorder include the following: 1) the individual must be exposed to a traumatic event in which he witnessed an event that involved actual or threatened death or injury to himself or others, and his response included intense fear, helplessness, or horror; 2) during or after the experience, the individual had three or more of the following symptoms ? sense of numbing or emotional detachment, reduction of awareness of surroundings, derealization, depersonalization, or dissociative amnesia; 3) the individual continues to relive the event through thoughts, dreams, illusions, flashbacks; 4) the individual displays marked avoidance of thoughts, people, places, etc. that are reminders of the traumatic event; 5) the individual experiences symptoms of anxiety or heightened arousal, such as irritability or difficulty sleeping; 6) the individual experiences distress that impairs social, occupational, or other important areas of functioning; 7) the disturbance lasts at least 2 days and a maximum of 4 weeks, and occurs within 4 weeks of the traumatic event; 8) the condition is not caused by medical or psychiatric conditions or substance abuse. (Choice A) Major depressive disorder (MDD) is diagnosed when a depressed mood or anhedonia and four of the following symptoms are present for two weeks or longer: Sleep disorder; Interest deficit (anhedonia); Guilt (worthlessness, hopelessness, regret); Energy deficit; Concentration deficit; Appetite disorder; Psychomotor retardation or agitation; Suicidality (the "SIGECAPS" mnemonic). MDD may be triggered by a stressful event or may have no known cause. (Choice B) Dysthymic disorder is a chronic depressive condition in which the individual experiences persistent sadness, loss of interest, and withdrawal from activities for two or more years. The symptoms are typically milder than those seen in major depressive disorder. (Choice C) Post-traumatic stress disorder (PTSD) is similar to acute stress disorder but is typically more severe and prolonged. PTSD is associated with recurrent intrusive thoughts, nightmares and flashbacks about a traumatic event; hypervigilance; physiologic hyperarousal; and avoidance of people, places, or events associated with the trauma. Known triggers of PTSD include military combat, natural disasters, serious motor vehicle accidents, violent assault, kidnapping, rape, incest, and childhood sexual abuse. Symptoms last for a minimum of one month and may persist for many years. (Choice E) Individuals experiencing normal bereavement tend to have intermittent periods of depression with a baseline of normal functioning, and approximately 95% have returned to normal functionality by one year after the loss. Educational Objective: Acute stress disorder is an anxiety reaction that arises in response to a traumatic event and lasts for four weeks or less. Post-traumatic stress disorder lasts longer than four weeks and may persist for years. A 60-year-old Caucasian male presents to your office with shaking of his right hand. He first noticed this shaking several months ago, and it progressed over time. He says that it is worse when he is watching TV, but he frequently forgets about it while working or doing some daily activities. He is an accountant and works at a bank. His past medical history is significant for left hip osteoarthritis. He occasionally takes naproxene to relieve the hip pain and over-the-counter antacids to relieve periodic

heartburns. His mother had diabetes mellitus. His father died of a heart attack at the age of 60. His blood pressure is 130/80 mmHg on both arms and heart rate is 70/min. Physical examination reveals tremors of the right hand that diminishes with movement, and involuntary resistance to flexion of the right wrist and right arm. No sensory abnormalities are present. Muscular strength is preserved on both sides. Which of the following is the best diagnostic strategy in this case? A. Order MRI to look for intracranial abnormalities. B. Check serum copper and ceruloplasmin levels. C. Do toxicological screening. D. Do lumbar puncture and observe after removal of 20 ml of CSF. E. Rely on clinical findings and prescribe treatment. Explanation: This patient presents with symptoms and signs suggestive of Parkinson?s disease: resting tremors and rigidity are characteristic. Interestingly, Parkinson?s disease typically begins on one side and eventually involves both sides. Diagnosis of the disease is based almost entirely on history and physical findings. Laboratory and imaging tests are rarely necessary, except when a patient presents with atypical features. (Choice B) Serum copper and ceruloplasmin levels can be measured in young patients with extrapyramidal symptoms to exclude Wilson?s disease, but it almost always manifests before the age of 30. (Choice A) Imaging studies are helpful in confirming or excluding normal pressure hydrocephalus, subdural hematoma, multiple infarcts, tumors, etc. (Choice C) Rarely, Parkinson-like symptoms can be caused by some toxic agents such as pesticides, methanol, and organic solvents. Toxicological screening may be indicated if the history suggests a possible exposure. (Choice D) Improvement of the clinical picture after lumbar puncture is typical for normal pressure hydrocephalus. Educational Objective: Diagnosis of Parkinson?s disease is based almost entirely on the history and physical findings. A 76-year-old female is admitted to the hospital because of an altered mental status. She has a history of diabetes, hypertension, osteoporosis, and mild depression. At home, she was on metoprolol, rosiglitazone, hydrochlorothiazide, and sertraline. She also takes alendronate on a weekly basis. She has been on these medications for a number of years. Before admission, she was living independently in her apartment and was able to perform nearly all day-to-day activities. It was her daughter who found her in her apartment, very confused in her bed. On examination, you see a drowsy, frail, elderly woman in no acute distress. Her temperature is 36.7 C (98 F), blood pressure is 110/50 mmHg, pulse is 88/min, and respirations are 16/min. Her oxygen saturation is 97% on room air. The neck is supple without any jugular venous distention or thyroid enlargement. The oropharynx is dry. Pupillary reflexes are intact. There is no obvious pallor or icterus. Lungs are clear to auscultation. Cardiovascular and gastrointestinal systems are unremarkable. On neurological examination, the patient is not very cooperative. She knows her name but is disoriented to time and place. She can move all her extremities. Touch sensations are grossly intact and deep tendon reflexes are symmetric. On laboratory investigation, total and differential white count is normal. Her hemoglobin and hematocrit are 12.3 g/dL and 36.5, respectively. The basic metabolic panel is normal, including calcium levels. Liver function tests are normal. Her urine examination is unremarkable. Chest x-ray is normal. A CT scan of the head performed in the emergency department shows generalized cerebral atrophy. What is the most appropriate next step in the management of this patient?

A. Discontinue rosiglitazone B. Discontinue hydrochlorothiazide C. Discontinue metoprolol D. Order thyroid function tests E. Perform lumbar puncture Explanation: The most common causes of mental status change in elderly patients are medications, infections (UTI and pneumonia), and metabolic abnormalities. A high index of suspicion is required for making a diagnosis of apathetic thyrotoxicosis. Elderly patients may not have classical manifestations of thyrotoxicosis and can present with apathy, depression, loss of appetite, and severe weight loss. This patient may have apathetic thyrotoxicosis; therefore, this needs to be ruled out first. (Choice A) Discontinuing rosiglitazone will not help in the management of this patient. Rosiglitazone improves blood glucose in diabetes by improving insulin sensitivity. It does not usually cause hypoglycemia. The effect of rosiglitazone can last for few days after discontinuation. (Choice B) Hydrochlorothiazide can cause mental status changes by lowering sodium levels and, in some cases, by increasing serum calcium levels. The patient?s basic metabolic panel and calcium levels are normal; therefore, hydrochlorothiazide is not responsible for causing the change in her mental status. (Choice C) The patient has been on metoprolol for a number of years. It is unlikely that metoprolol is responsible for her current symptoms. (Choice E) There is no indication that this patient has meningitis (no fever, no headache, and no neck stiffness); therefore a lumbar puncture should not be performed as a next step. Educational Objective: The most common causes of mental status change in elderly patients are medications, infections (UTI and pneumonia), and metabolic abnormalities. A high index of suspicion is required for making a diagnosis of apathetic thyrotoxicosis. Eelderly patients with thyrotoxicosis may not have classical symptoms. A 24-year-old Caucasian male is brought to the emergency department by his girlfriend with lethargy and shortness of breath. The girl friend said that he felt okay two hours ago when she was leaving for a while, ?just a little bit exhausted? because they cleaned their house today. She found him confused and sleepy when she returned home. His past medical history is insignificant. He does not smoke or consume alcohol and is not known to take recreational drugs. His blood pressure is 110/70 mmHg and heart rate is 50/min. Petroleum odor is present on his clothes. Physical examination reveals skin flushing and bilateral myosis. Wheezing is heard on lung auscultation. Which of the following is most likely to confirm the diagnosis in this patient? A. Urine toxicology B. Plasma cholinesterase C. Chest x-ray D. Arterial blood gases E. Carboxyhemoglobin level Explanation: The symptoms and signs described are suggestive of organophosphate poisoning: confusion, lethargy, bradycardia, skin flushing, myosis and wheezing are typical. Sudden onset of symptoms and petroleum odor from clothing (some organophosphates have petroleum odor) are also characteristic. Plasma cholinesterase activity test is easily performed and can be used to confirm the diagnosis, but it does not seem to correlate with the severity of intoxication. Direct measurement of RBC

acetylcholinesterase activity provides a measure of the degree of toxicity, but this is more difficult to perform and most hospital laboratories are unable to perform this test. (Choice A) Urine toxicology is helpful in diagnosing intoxications with illicit drugs and some medications. (Choice C) Certain radiopaque toxins may be visualized by plain film radiographs, e.g., chlorinated hydrocarbons or iron tablets. (Choice D) Arterial blood gases may help to diagnose salicylates, ethylene glycol, and methanol intoxications. (Choice E) Carboxyhemoglobin level measurement may be done in patients with CO intoxication. Educational Objective: Plasma cholinesterase activity test is easily performed and can be used to confirm the diagnosis of organophosphate poisoning. You are visiting a 76-year-old Caucasian male in the nursing home. He has been complaining of pain in his anal region for the last three days, and he had an episode of a small amount of rectal bleeding. His other medical problems include constipation, benign prostatic hypertrophy, and chronic obstructive pulmonary disease. His temperature is 37.2 C (99 F), blood pressure is 110/60 mmHg, pulse is 100/min., and respirations are 20/min. Examination of the anal area reveals a thick protruding rectal mucosal mass with bluish discoloration and friability. He appears to be in moderate distress. Which of the following is the most appropriate next step in his management? A. Observation alone B. Observation with narcotic analgesics C. Send the patient to the emergency department D. Give antibiotics and schedule an outpatient follow up visit in 1 week E. Obtain surgical consultation in the nursing home Explanation: The patient in the above scenario has developed a complete rectal prolapse with strangulation and gangrene. He should be transferred to an emergency department as soon as possible to prevent further complications. Rectal prolapse occurs when a mucosal or full-thickness layer of rectal tissue slides through the anal orifice. It is usually associated with conditions causing increased intraabdominal pressure such as chronic straining with constipation, benign prostatic hypertrophy, and COPD. It usually presents with difficulty with defecation, diarrhea/fecal incontinence, pain in the anal area, rectal bleeding, and a protruding rectal mass. Untreated complete rectal prolapse can lead to strangulation and gangrene of the rectal mucosa. Uncomplicated prolapsed rectal mucosa can be reduced by gentle digital pressure. Adequate local anesthesia and sedation help reduce the prolapsed mucosa. Topical application of granulated sucrose helps reduce the mucosal edema and makes manual reduction easier. Irreducible prolapsed mucosa and gangrene of the mucosa warrant a prompt surgical consultation. The prognosis is generally good with prompt and appropriate care. (Choices A and B) Observation alone or with pain medications is not appropriate in this setting. The patient has developed gangrene of the mucosa that can be fatal unless treated emergently. (Choices D and E) The patient needs prompt treatment and possible emergent surgical intervention in the form of a rectosigmoidectomy. Outpatient medical management and surgical consultation at home are not recommended at this point. Educational Objective:

Complete rectal prolapse should be promptly evaluated for signs of incarceration and gangrene. Surgical intervention is required for irreducible prolapse and gangrene of rectal mucosa. A 44-year-old Caucasian female presents to your office complaining of severe, episodic dizziness for the last year or two. During the episodes, she feels that the room is spinning and she can?t walk. The episodes last for one or two hours and resolve spontaneously. She also noticed periodic decreased hearing and ringing sensation in her left ear. She has right knee osteoarthritis, and takes Tylenol and aspirin 2-3 times a day to cope with the pain. She does not smoke or consume alcohol. Her blood pressure is 130/83 mmHg and heart rate is 74/min while standing, and 128/85 mmHg and 76/min, respectively, while supine. What is the most likely diagnosis in this patient? A. Acoustic neuroma B. Benign positional vertigo C. Aspirin toxicity D. Meniere?s disease E. Multiple sclerosis Explanation: This patient presents with a classic triad of periodic vertigo, unilateral hearing loss and tinnitus that is characteristic for Meniere?s disease. Although many patients initially experience only some symptoms, most eventually go on to experience the full complement of the disease. Meniere?s disease should be differentiated from a range of other conditions having similar presentations. (Choice B) Benign paroxysmal positional vertigo (BPPV) is characterized by typical acute vertigo attacks that are provoked by a change in position; however, unlike Meniere?s disease, BPPV is not accompanied by hearing loss. (Choice A) Acoustic neuroma is an important consideration in this patient, but acute vertigo episodes are uncommon in patients with acoustic neuroma due to its gradual tumor growth. (Choice C) Aspirin toxicity may manifest as tinnitus and vestibular symptoms, but unilateral hearing loss does not occur with this condition. (Choice E) Multiple sclerosis can sometimes mimic Meniere?s disease but usually presents with more widespread non-localized symptoms. Educational Objective: A classic triad of periodic vertigo, unilateral hearing loss and tinnitus is characteristic for Meniere?s disease. The following vignette applies to the next 3 items A 34-year-old male presents to the emergency department (ED) with severe headache, lethargy, vomiting, fever and blurring of vision. His symptoms started one week ago and increased progressively over time, especially during the last two days. He is a known IV drug abuser. He was admitted to the hospital three years ago for infective endocarditis, and at that time his HIV test was negative. His family history is not significant. He is allergic to penicillin. Examination shows neck stiffness and bilateral papilledema. Examination of the cerebrospinal fluid shows the following: Opening pressure: 340 mm H2O Protein: 100 mg/dL Glucose: 30 mg/dL WBC: 40/mm3 Lymphocytes: 96% RBC: 1/mm3

Item 1 of 3 Which of the following is the best next step in the management of this patient? A. Start ceftriaxone and vancomycin B. Start amphotericin B and flucytosine C. Start ampicillin and gentamicin D. Start intravenous acyclovir E. Check the HIV status again Explanation: Although this patient's past HIV test result is negative, his high-risk behavior still places him at risk of having HIV and its associated diseases. His presentation and CSF findings are classic for cryptococcal meningitidis, which is managed with amphotericin B and flucytosine. The CSF findings in patients with cryptococcal meningitis are: Markedly elevated opening pressure, often >200 mm on the initial spinal tap Characteristically low (<50/?L) WBC count with mononuclear predominance (lymphocytes) Elevated protein and low glucose Positive India Ink preparation (Choice A) The classic CSF findings seen in bacterial meningitis are: (1) polymorphonuclear leukocytosis (>100 cells/L) - not lymphocytosis, (2) decreased glucose concentration (<40 mg/dL), (3) increased protein concentration (>45 mg/dL), (4) increased opening pressure (>180 mmH2O). (Choice D) The typical CSF findings seen in viral meningitis are: (1) lymphocytic pleocytosis (25 to 500 cells/L) (2) a normal or slightly elevated protein concentration (20 to 80 mg/dL), (3) a normal glucose concentration, (4) a normal or mildly elevated opening pressure (100 to 350 mmH2O). (Choice E) His HIV status needs to be checked as soon as possible, but this should not delay the treatment. Item 2 of 3 The patient?s symptoms improved rapidly during the following three hours. The next day, he began to complain of severe headaches and vomiting again. Which of the following is the best next step in the management of this patient? A. Start high dose amphotericin B and flucytosine B. Start fluconazole C. Do repeat lumbar puncture D. Start ceftriaxone and vancomycin E. Add corticosteroids Explanation: Some patients with cryptococcal meningitis may present with increased intracranial pressure that manifests as severe headache, altered sensorium and blurred vision. The opening pressure during lumbar puncture may be markedly elevated, and patients obtain significant relief after the procedure. Almost all of these patients will rapidly respond to the removal of large volumes of CSF, which can be accomplished with serial lumbar punctures, although some patients may require lumbar drains or ventriculostomies. (Choice E) Corticosteroids are not indicated to decrease raised intracranial pressures in these patients. (Choice B) If there is clinical improvement during induction therapy, fluconazole can be initiated and the amphotericin B and flucytosine discontinued. (Choice D) There is no indication to switch to ceftriaxone and vancomycin.

Educational Objective: Some patients with cryptococcal meningitis may present with increased intracranial pressure that manifests as severe headache, altered sensorium and blurred vision. Patients may require repeated lumbar punctures to relieve the pressure. Item 3 of 3 The patient becomes asymptomatic two weeks after the initiation of the therapy. following is the best next step in the management of this patient? A. Decrease the dose of initial therapy B. Discontinue the initial therapy and start fluconazole C. Do lumbar puncture D. Discontinue the initial therapy and discharge to home E. Discontinue the initial therapy and start amoxicillin Explanation: In HIV-positive patients with cryptococcal meningitis, typical induction therapy consists of amphotericin B and flucytosine and usually lasts 10 to 14 days. If there is clinical improvement during induction therapy, fluconazole can be initiated and the amphotericin B and flucytosine discontinued. Fluconazole is usually continued as maintenance therapy to prevent recurrence of the disease. (Choices A, D and E) Since the patient became asymptomatic after two weeks, he can be given fluconazole, and his other medications should be discontinued. Educational Objective: Low-dose fluconazole is used for maintenance therapy after an episode of cryptococcal meningitis in HIV-positive patients. Which of the

A 36-year-old HIV-positive man comes to the emergency room with complaints of burning and pain during urination for the past two days. He has been on highly active antiretroviral treatment (HAART) for the last three months, and his last CD4 count a month ago was 250 cells/cu mm. You order a urine analysis to look for evidence of an infection. The urine sample appears clear on gross inspection, and is sent to the laboratory. While the nurse is transporting the urine sample to the lab, she suddenly trips, and spills some of the urine over an open laceration on her hand. She gets very concerned, and asks you for further advice. Apart from cleaning the wound with generous amounts of soap and water, what is the most appropriate next step in HIV prophylaxis of this nurse? A. Do nothing further at this point. B. Consult an infectious disease specialist. C. Start her on AZT. D. Draw the patient?s HIV viral load. E. Start her on triple therapy with AZT, DDI and a protease inhibitor. Explanation: Occupational exposure and the subsequent transmission of HIV infection has been documented in different settings. Almost all the cases of occupational transmission of HIV have been due to transmission via exposure to blood and certain body fluids. The body fluids wherein standard precautions have been recommended include semen, vaginal secretions, and any other body fluid containing visible blood. Other standard precautions, according to the Center for Disease Control and Prevention (CDC), also apply to cerebrospinal, peritoneal, pleural, pericardial, synovial fluid, or any other tissue, even though the epidemiologic data regarding the risk of HIV transmission from these fluids is insufficient. Standard precautions do not apply to urine, sweat, tears, sputum, vomitus, and nasal secretions or feces, as long as there is no gross visible blood. The occupational transmission of HIV has never been documented from the above sources. The patient in the above vignette has exposure to a clear urine sample with no evidence of blood in it; therefore, no further intervention, other than thorough washing with soap and water, is recommended. (Choice C, E) Post-exposure prophylaxis with drugs is recommended in health care workers with exposure to blood or blood-containing body fluids. (Choice D) HIV viral load may be drawn to follow the patient?s response to HAART, but is not required in this particular setting. Educational Objective: Health care workers with exposure to blood or blood-containing fluid from an HIV-positive patient should be evaluated further for post-exposure prophylaxis. The exposure to urine without visible blood is not considered a "significant exposure," and does not need further intervention. An asymptomatic, 3-month-old Asian baby boy is brought to the office by his mother for a routine follow-up visit. The mother has no complaints about her baby. She had hepatitis B infection during the last three months of pregnancy. The baby?s father has chronic, active hepatitis B. The baby takes no medication. The baby received hepatitis B immunoglobulin and vaccination at birth, and a second dose of hepatitis B vaccine when he was two months old. His temperature is 37.2 C (99 F) and respirations are 16/min. Examination shows no abnormalities. The patient?s liver function tests (LFT) reveal: Direct bilirubin: 1.0 mg/dL Alkaline phosphatase: 100 U/L Aspartate aminotransferase: 48 U/L Alanine aminotransferase: 55 U/L When is the most appropriate time for measurement of hepatitis B serology in this patient?

A. At birth B. At three months of age C. At six months of age D. At nine months of age E. At four months of age Explanation: With prophylaxis, the risk of developing chronic hepatitis B in infants born to mothers with hepatitis B decreases from 30% to 2%. Such infants should therefore receive hepatitis B immunoglobulin and hepatitis B vaccine 12 hours after birth. The second vaccination must be given between one to two months, and the third dose at six months. Hepatitis B surface antigen (HbsAg) and anti-hepatitis B surface antigen antibody (anti-HbsAg) needs to be monitored in order to identify those who will need further treatment, if diagnosed of chronic hepatitis B infection. It will also help to determine who needs to be revaccinated. Studies have determined that the best time to check the titers is between nine to fifteen months of age. Serology controls must be done three to four months after the third dose. Educational Objective: Infants born to a woman with hepatitis B infection have a 30% risk of acquiring chronic hepatitis B and a 25% risk of developing liver cirrhosis or hepatocarcinoma in adulthood. Prophylaxis with hepatitis B immunoglobulin and hepatitis B vaccine is mandatory. Serology controls must be done 3-4 months after the third dose, or when the infant is between 9 and 15 months old. The following vignette applies to the next 2 items A 27-year-old Caucasian female presents to your office with a one-year history of periodic irritability, anger, tension, and sleep disturbance. The symptoms begin one week before her periods and resolve with menstrual bleeding. She also noticed increased appetite, headaches, breast tenderness and ankle swelling before her periods. Her past medical history is insignificant. Her cycles are regular, lasting 29 days. She has no intermenstrual bleeding or vaginal discharge. She has never been pregnant. Her Pap smear taken 6 months ago was normal. She denies smoking, alcohol consumption, and any recreational drug use. She has been sexually active with one partner over the last three years and uses condoms for contraception. She has no known allergies. Her blood pressure is 110/70 mmHg and heart rate is 90/min. You prescribe standard dose (20 mg/day) fluoxetine. The patient returns after several months and says that the symptoms failed to improve significantly. Item 1 of 2 What is the best next step in the management of this patient? A. High-dose fluoxetine B. Alprazolam C. Leuprolide D. Danazol E. Spironolactone Explanation: Premenstrual syndrome (PMS) is characterized by both behavioral and physical symptoms that occur in the second half of the menstrual cycle. The presentation of PMS varies significantly; over 150 symptoms have been described. Premenstrual dysphoric disorder (PMDD) is a severe form of PMS characterized by the prominence of anger and irritability. Serotonin reuptake inhibitors (SSRI) are considered the first line therapy for PMDD. Fluoxetine has been studied best and has been demonstrated to relieve both behavioral and physical symptoms in patients with PMS/PMDD. Approximately 15% of patients are resistant to fluoxetine. In such instances, alprazolam is a good alternative and is given in the luteal phase of the menstrual cycle. (Choice A) Increasing the dose of fluoxetine increases the incidence of side effects, but does not seem to improve symptoms. The effective dose is 20 mg/day.

(Choices C and D) Patients who do not respond to SSRIs or alprazolam are candidates for ovulation suppression agents such as GnRH and danazol. GnRH agents are usually preferred to danazol because of its more favorable side effect profile. (Choice E) Spironolactone should be considered in selected cases because it has not been consistently shown to be effective in patients with PMS. Educational Objective: Selective serotonin reuptake inhibitors (SSRI) are considered the first line therapy for PMS/PMDD; alprazolam is a good alternative and is considered a second-line agent. *Extremely important question for USMLE step-3 Item 2 of 2 The patient returns in 6 months and says that she feels much better. Her symptoms are less disabling, and she is able to work without significant discomfort. She is concerned about the future risk of any disease that may be associated with her condition. Which of the following is most likely to be present later on in this patient? A. Migraine B. Chronic fatigue syndrome C. Irritable bowel syndrome D. Hyperthyroidism E. Depression Explanation: PMS has a significant association with psychiatric disorders. It has been estimated that the lifetime risk of a psychiatric disorder in patients with PMS approaches 80%! These mainly include mood and anxiety disorders. Studies have demonstrated that women who present with PMS have a much higher incidence of major depression in the past; they also seem to have a greater incidence of depressive episodes in the future. At the same time, a mood or anxiety disorder that usually exacerbates before menses is often misdiagnosed as PMS. (Choice A) Some medical conditions may be misdiagnosed as PMS because of the periodic pattern (e.g., migraine headaches with associated affective symptoms may show premenstrual exacerbation). (Choices B and C) Other diseases that can demonstrate a periodic pattern include chronic fatigue syndrome and irritable bowel syndrome. (Choice D) Thyroid tests should be done before diagnosing PMS because hyperthyroidism can mimic the disorder. Educational Objective: The lifetime risk of a psychiatric disorder in patients with PMS approaches 80%. These mainly include mood and anxiety disorders. An 84-year-old Asian man on total parenteral nutrition (TPN) is being evaluated in the internal medicine ward. He was transferred to this hospital from another facility two days ago. Ten days ago, he was admitted there and started on TPN because he was unable to eat. He has very advanced Alzheimer?s disease. He has been bedridden for the past four years, and has been unable to eat by himself for the past two years. He is no longer able to swallow properly. He had two episodes of near aspiration during the past four weeks. His other medical problems include hypertension, osteoporosis and Parkinson?s disease. He had a stroke several years ago. His medications include aspirin, levodopa/carbidopa, galantamine and lisinopril. He has a visiting nurse and a home attendant who feeds, bathes and grooms him. His older son lives with him, and also helps with his care. The results of the evaluations done at the hospital reveal moderate microcytic anemia and mild hypoalbuminemia. Any possibility of infection, malignancy or drug overdose has already been ruled out. The patient weighs 132 pounds (60 kg), and is receiving 3,000 calories and 90 grams of protein daily, through a right subclavian catheter exclusively used for TPN. His lab values are as follows:

CBC Hb: 9.4 g/dL Ht: 28% MCV: 80fl Platelet count: 250,000/cmm Leukocyte count: 5,000/cmm Segmented neutrophils: 74% Lymphocytes: 20% Monocytes: 6% Serum Chemistry Serum Na: 138 mEq/L Serum K: 4 mEq/L Chloride: 108 mEq/L Bicarbonate: 22 mEq/L BUN: 18 mg/dL Serum Creatinine: 0.8 mg/dL Calcium: 8.6 mg/dL Blood Glucose: 180 mg/dL Which of the following is the most appropriate next step in the management of this patient? A. Continue TPN, but decrease diet to 2600 calories B. Insert a gastrostomy tube, but decrease diet to 2200 calories C. Continue TPN with the same amount of calories D. Insert a gastrostomy tube, with the same amount of calories E. Continue TPN but decrease diet to 2200 calories Explanation: According to the American Gastroenterological Association Guidelines for Enteral Support, tube feeding should be considered for patients who cannot or will not eat, for patients who have a functional gut, and for whom a safe method of access is possible. Nutrition support should be initiated in most patients after 1-2 weeks without nutrient intake. Enteral feeding is preferable to parenteral therapy provided there is no mechanical obstruction, access can be attained safely, and oral intake is not possible. The usual caloric requirement for the average man is around 30 cal/Kg per day; the average protein intake is 1gram/kg/day. The caloric requirement of malnourished or critically ill patients is 35-40 cal/Kg (2100-2400 cal/day), and their protein requirement is 1.5 grams/Kg/day. The required amount of protein is generally increased in patients with high metabolic demands (e.g., trauma or sepsis) or protein-calorie malnutrition. In the above vignette, the patient?s caloric intake is excessive. He is currently receiving 3,000 cal/day, which is approximately 50 cal/kg/day. Overfeeding leads to hyperinsulinemia, hyperglycemia, increased TNF (tumor necrosis factor) receptor activity, and in consequence an increased incidence of sepsis. Therefore, his caloric intake should be reduced. (Choices A, C and E) Total parenteral nutrition (TPN) increases the risk of intravascular infections because it needs to be administered through a central vein. Educational Objective: Tube feeding should be considered for patients who cannot or will not eat, for patients who have a functional gut, and for whom a safe method of access is possible. Nutrition support should be initiated in most patients after 1-2 weeks without nutrient intake. The caloric requirement of malnourished or critically ill patients is 35-40 cal/Kg (2100-2400 cal/day), and their protein requirement is 1.5 grams/Kg/day.

A 29-year-old African-American female who is in her first trimester of pregnancy develops severe pain in her right flank that radiates to her right labia. The pain is 10/10 in severity and paroxysmal in character. She denies any fever, chills, nausea, vomiting, increased urinary frequency, hematuria, or burning micturition. Her pregnancy is otherwise uncomplicated. She has a past history of uric acid stones. She is given ketorolac, which relieves her pain. Which of the following is the most appropriate next step in the management of this patient? A. Helical CT scan B. Abdominal ultrasound C. Intravenous pyelogram D. Plain abdominal x-ray E. MRI of the abdomen Explanation: The patient most likely has nephrolithiasis. The ultrasound is the test of choice in evaluating nephrolithiasis in those patients who should avoid radiation exposure (e.g., pregnant patients). (Choice A) Helical CT scan is the test of choice for suspected nephrolithiasis; however, a CT scan is associated with radiation exposure, and is not the best test in pregnant patients. (Choice C) Intravenous pyelogram is no longer the test of choice because of the serious allergic reactions associated with contrast injection. (Choice D) Plain abdominal films do not detect radiolucent stones, and these should not be obtained in a pregnant patient because of the associated risk of teratogenic effects. Educational Objective: Ultrasound is the test of choice for suspected nephrolithiasis in a pregnant patient. A 54-year-old male presents to your office with fatigue, abdominal bloating, diarrhea and weight loss. He states that he occasionally has right upper quadrant pain after meals. He denies nausea, vomiting, cough, fever, dark urine or stool, bright red blood per rectum, and dysuria. His past medical history is unremarkable. He is not on any medications. His family history is negative for diabetes, hypertension, coronary artery disease and cancer. He does not smoke, but drinks alcohol on the weekends. He is married and lives with his wife. On physical examination, vague fullness is noted in the right upper quadrant. The rest of the physical examination is normal. Laboratory results reveal normal CBC, normal basic metabolic panel except for a blood glucose level of 212 mg/dl, and normal liver function tests. Ultrasonography of the right upper quadrant reveals multiple gallstones without any evidence of obstruction or gall bladder wall thickening. What is the next best step in the management of this patient? A. Start proton pump inhibitors B. Refer to a surgeon for cholecystectomy C. Order fasting somatostatin level D. Order CT scan of the abdomen E. Order aortic angiography Explanation: This patient presents with the classical triad of somatostatinomas, which include gallstones, malabsorption and diabetes. Somatostatinomas are tumors arising from delta cells in the pancreas, although sometimes, these may arise from somatostatin-secreting cells of the duodenum. These tumors lead to diabetes by inhibiting insulin secretion. Gallstones are formed due to the inhibitory effect of somatostatin on gall bladder motility. Similarly, inhibition of pancreatic secretion leads to malabsorption. Majority of somatostatinomas are cancerous. Fasting somatostatin levels are obtained to diagnose somatostatinomas; levels over 160 pg/ml are very suggestive.

(Choice D) Localization is performed by CT scan of the abdomen, which should be performed after a biochemical diagnosis is made. (Choice B) Although this patient may later require a cholecystectomy, referring to a surgeon is not the next best step. (Choice A) Proton pump inhibitors would not be helpful in this case. (Choice E) Postprandial abdominal pain, diarrhea and weight loss suggest mesenteric ischemia. Gallstones and diabetes are not seen with mesenteric ischemia. Even if gallstones and diabetes could coincide with mesenteric ischemia, this is highly unlikely in the patient because he does not have any risk factors for diabetes, gallstones or atherosclerosis. Angiography is not the next best test to be performed. Educational Objective: The classical triad of somatostatinomas include gallstones, malabsorption and diabetes. Somatostatinomas are tumors arising from delta cells in the pancreas. Fasting somatostatin levels over 160 pg/ml are very suggestive. A one-week-old male infant underwent postnatal screening for hypothyroidism and was found to have a T4 level of 6 mcg/dl (normal 10-22 mcg/dl for the first week after birth). His TSH level is 26 U/l. The blood sample was drawn from the heel pad. The mother has no obvious medical problems and had an uneventful pregnancy, except for slight post-maturity. At this point, what is the next best step in the management of this patient? A. Start levothyroxine (LT4) immediately. B. Start liothyronine (T3). C. Start levothyroxine only if clinical features of hypothyroidism are present. D. Repeat Free T4 and TSH levels on a regular blood draw. E. Radioactive iodine uptake and scan. Explanation: Neonatal screening for hypothyroidism is performed by obtaining a small sample (few drops) of blood from the heel pad and using a piece of filter paper to absorb the blood sample. This test may be done within two to five days following delivery. It is performed after the first 24 hours of life because there is a normal physiologic TSH surge following delivery. After 24 hours, the TSH levels gradually drop to normal levels or may remain slightly elevated for the next few days. It is very important to avoid any delays in the diagnosis and treatment to avoid permanent neurological deficits. Most centers in the United States use total T4 measurement as a primary tool for screening for neonatal hypothyroidism. If the total T4 levels are low, TSH levels of the same sample is measured, and if the TSH levels are over 20 U/L, repeat testing (measurement of both Free T4 and TSH levels) is performed from a regular blood draw to confirm the diagnosis. Some programs use TSH as a primary screening tool. (Choice A) Levothyroxine is the treatment of choice for hypothyroidism, and the dose in the neonate is higher than the adult dose; however, levothyroxine therapy should be started once the diagnosis is confirmed with a regular blood draw. (Choice B) T3 is not indicated for the treatment of most patients with hypothyroidism. (Choice C) The clinical features of hypothyroidism in neonates can be subtle and unreliable. Levothyroxine therapy in neonates is therefore started once the biochemical confirmation is obtained, even if the clinical features of hypothyroidism are not present. (Choice E) Once the diagnosis of neonatal hypothyroidism is confirmed by measuring the Free T4 and TSH levels from a regular blood draw, other ancillary procedures such as thyroid ultrasonography, radioactive iodine scanning and urinary iodide levels are performed to investigate the etiology of the hypothyroid state. Radioactive iodine uptake and scanning are useful to determine the location, size and function of the thyroid gland.

Educational Objective: Neonatal screening for hypothyroidism is performed within two to five days following birth. Low T4 and elevated TSH levels of the blood sample from the heel pad are very suggestive of primary hypothyroidism; however, confirmation of the diagnosis by measuring the free T4 and TSH levels from a regular blood draw should be performed before starting levothyroxine therapy. A 35-year-old white male comes to the emergency department because he cut his finger with a rusty knife while doing yard work. His cut was quite deep, and his wife immediately applied a pressure dressing to stop the bleeding. His immunization status is unknown. He last saw his doctor approximately ten years ago. On examination, a deep laceration is noted on the palmar aspect of his right middle finger. Which of the following is the most appropriate next step in the management of this patient? A. Tuberculous infiltration of the liver B. Tetanus antitoxin C. Tetanus immune globulin and Tetanus antitoxin D. No immunization E. Radioactive iodine uptake and scan. Explanation: The above patient sustained a penetrating injury to his hand with a rusty knife. He is at risk for the development of tetanus; therefore, he needs tetanus prophylaxis. As his immunization status is unknown and his wound is not clean, he should be given both tetanus immune globulins and tetanus antitoxin. (Choice A) Tetanus immune globulin is indicated only for contaminated wounds when the immunization status is unknown, or when the patient has received less than three doses of tetanus antitoxin.

(Choice B) The situations in which only tetanus antitoxin is given are the following: 1. when the patient has a clean wound but unknown immunization status, or if he has received less than three doses of tetanus antitoxin 2. when the patient has a clean wound and has received three or more doses of tetanus antitoxin with the last dose received more than ten years ago 3. when the patient has a contaminated wound and has received three or more doses of tetanus antitoxin with the last dose received more then five years ago Educational Objective: For those patients with a contaminated wound and unknown immunization status, both tetanus antitoxin and tetanus immune globulins are indicated. A 53-year-old Asian female comes with complaints of fatigue and weight loss of 4.5kg (10 lbs) within the past four months. She has been an alcoholic for the last 10 years, and she still consumes approximately two bottles of beer daily. She was diagnosed with pulmonary tuberculosis two years ago, for which she took isoniazid and rifampin for two months. She has normal vital signs. The abdominal examination is unremarkable. Her laboratory test results reveal the following: Total bilirubin: AST: ALT: alkaline phosphatase: serum albumin: 1.0 mg/dl 35 IU/L 40 IU/L 400 IU/L 3.5 mg/dl

PT: 11 seconds Which of the following is the most likely cause of these findings? A. Tuberculous infiltration of the liver B. Alcoholic cirrhosis C. Viral hepatitis D. Common bile duct stone E. Carcinoma of the head of the pancreas Explanation: This patient has elevated alkaline phosphatase levels and normal aminotransferases, bilirubin, serum albumin, and PT. Isolated elevation of alkaline phosphatase levels due to hepatic causes is characteristically seen in infiltrative diseases of the liver (i.e., tuberculosis and lymphoma). Her past history of incompletely treated tuberculosis and her complaints of weight loss and fatigue make the diagnosis of tuberculous infiltration of the liver more likely. (Choice B) Although this patient is at risk of alcoholic liver disease, her normal aminotransferase levels and the absence of cirrhotic stigmata makes this diagnosis unlikely. (Choice C) Viral hepatitis would not cause an isolated elevation of alkaline phosphatase levels. Rather, it would cause an elevation of aminotransferases with or without jaundice. (Choice D) Common bile duct stones can cause a predominant elevation of alkaline phosphatase levels, but these would invariably cause some degree of hyperbilirubinemia with or without abdominal pain. (Choice E) Pancreatic cancer can cause obstructive jaundice with fatigue, weight loss, and elevation of alkaline phosphatase, but it can never cause an isolated elevation of alkaline phosphatase levels. Educational Objective: Infiltrative diseases of the liver (i.e., lymphoma and tuberculosis) can be characterized by isolated elevations of alkaline phosphatase levels.

A 16-month-old girl is brought to your office for routine check-up. She is the first child of a 24-year-old Caucasian female and a 30-year-old African American male. Her past medical history is insignificant. The prenatal course was uncomplicated and she was born at term by vaginal delivery. She can walk alone, crawl up- and downstairs, sit with her back straight, make a four cube tower, and turn the pages of a book. Her diet mainly consists of whole cow?s milk, and she received daily vitamin D supplements six months ago. Physical examination reveals pallor of the conjunctivae, but is otherwise insignificant. Her laboratory findings are: Hemoglobin: 9.2 g/L MCH: 22.0 pg/cell MCHC: 28% MCV: 70 fL Leukocyte count: 7,000/mm3 ESR: 12 mm/hr Further history taking reveals that her father has a sickle cell trait and her first cousin has sickle cell anemia. Her mother is of Greek origin, and denies any familial blood disorders. The hemoglobin electrophoresis reveals 40% HbS and 60% HbA. Which of the following is the most likely cause of this patient?s anemia? A. Sideroblastic anemia

B. Sickle cell-beta thalassemia C. Hemoglobin SC disease D. Iron-deficiency anemia E. Anemia of chronic disease Explanation: Iron-deficiency is the most likely cause of this patient?s anemia. Children with hypochromic microcytic anemia are usually treated with iron because iron-deficiency anemia is the most common cause of this type of anemia. More sophisticated studies are indicated only if the response to iron is inadequate or if the medical history is complicated. Dietary factors play an important role in the development of iron-deficiency anemia in children. Three of these factors were found to be associated with anemia in children: low intake of meat, grains, vegetables, and fruit; high intake of milk; and daily intake of fatty snacks, sweets, or soft drink. (Choice B) This patient also has a sickle cell trait according to the findings on hemoglobin electrophoresis; however, sickle cell trait alone is not associated with anemia and microcytosis. At the same time, differentiation from sickle cell-beta thalassemia may be difficult, especially sickle cell-beta (+) thalassemia. Unlike sickle cell-beta (0) thalassemia where no HbA is observed, sickle cell-beta (+) thalassemia and sickle cell trait both have substantial amounts of HbA and HbS. The usual partition of Hb A and Hb S in sickle cell trait is 60:40. Sickle cell-beta (+) thalassemia is subclassified according to the percentage of Hb A present: type I has 3?5%; type II has 8?14%; and type III has 18?25%. Therefore, the maximum percentage of Hb A level in sickle cell-beta (+) thalassemia is 25%. (Choice C) The diagnosis of HbSC disease is straightforward, as nearly equal amounts of HbS and HbC are detected on hemoglobin electrophoresis. (Choices A and E) Sideroblastic anemia and anemia of chronic disease are much less likely causes of hypochromic and microcytic anemia in children than iron deficiency. These are usually suspected if the response to iron therapy is inadequate, or if additional historical, physical or laboratory findings are suggestive (e.g., hepatosplenomegaly is often present in patients with sideroblastic anemia, and ESR is elevated in patients with anemia of chronic disease). Educational Objective: Children with hypochromic microcytic anemia are generally treated with iron because iron-deficiency anemia is the most common cause of this type of anemia. The usual partition of Hb A and Hb S in sickle cell trait is 60:40. In sickle cell-beta (0) thalassemia no HbA is observed. The maximum percentage of Hb A level in sickle cell-beta (+) thalassemia is 25%. *Very high yeild question*

A 32-year-old male was admitted to the orthopedic department after being involved in a motor vehicle accident. His initial injuries included a transverse fracture of the right femur, tibia, fibula, and a comminuted fracture of the left femur shaft. The secondary survey revealed no other fracture or injuries. Thirty-six hours after admission, the patient complains of shortness of breath. His blood pressure is 110/80 mmHg, heart rate is 80/min, respirations are 34/min, and temperature is 37.9 C (100.1 F). His oxygen saturation on 3L per nasal cannula is 88%. Physical examination reveals a petechial rash on his head, neck, anterior thorax, and axilla. He appears confused. The portable chest x-ray result reveals patchy consolidation. Emergency V/Q scan reveals a mottled pattern of subsegmental perfusion defects with normal ventilation. Which of the following would have prevented this complication in this patient? A. Early immobilization and operative correction of the fracture B. Low molecular weight heparin C. Breathing exercises and chest physiotherapy

D. Sequential compressing devices for lower limbs E. Prophylactic antibiotics Explanation: Fat embolism is a clinical diagnosis. It typically manifests 24-72 hours after severe trauma, and is usually characterized by a triad of respiratory insufficiency, neurological impairment, and a petechial rash. Other common findings include fever, tachycardia, and altered mental status. The classic petechial rash is most commonly found on the trunk; this is typically a late manifestation, and is described in fewer than 50% of cases. Petechiae result from the occlusion of dermal capillaries by fat globules, thereby leading to extravasation of erythrocytes. No abnormalities of platelet function have been documented. Early immobilization and operative fixation of fractures reduces the chances of fat embolism. Supportive care is the mainstay of therapy for clinically apparent fat embolism. (Choice B) There is no evidence that prophylactic heparin reduces the risk of fat embolism. The use of corticosteroid prophylaxis is controversial, largely because it is difficult to definitively prove its efficacy in a condition with a low incidence, unclear risk factors, low mortality, and a good outcome with conservative management. Educational Objective: Always consider the risk of fat embolism in patients with multiple complicated fractures. Fat embolism is a clinical diagnosis that is usually characterized by a triad of respiratory insufficiency, neurological impairment, and a petechial rash. Early immobilization and operative fixation of fractures reduce the chances of fat embolism. A 23-year-old African-American woman, gravida 2 para 1, presents to the emergency department at 32 weeks gestation with regular menstrual-like cramping and low back pain. Her past medical history is significant for preterm labor at 28 weeks gestation, which resulted in infant death in the early postnatal period. Physical examination shows regular uterine contractions and cervical dilation (3 cm). No evidence of membrane rupture is present. The contraction stress test is negative. You decide to proceed with adequate hydration and tocolysis. Which of the following routes is preferred for administration of antenatal corticosteroids? A. Intravenous B. Intramuscular C. Oral D. Inhalational E. Subcutaneous Explanation: Antenatal corticosteroid therapy has been proven to be effective in reducing perinatal morbidity and mortality associated with preterm labor. It reduces the risk of infant respiratory distress syndrome and intraventricular hemorrhage. The two commonly employed regimens are betamethasone or dexamethasone administered intramuscularly. Intramuscular administration of steroids provides stable and predictable concentration of the drug in the blood that is required to achieve the desired fetal effects. (Choice A) Intravenous administration results in peaks and troughs in the blood concentration of the drug, and is therefore not recommended. (Choice C) One study that compared an oral regimen to intramuscular administration of dexamethasone found an increased risk of intraventricular hemorrhage and sepsis in the oral dexamethasone group.

(Choices D and E) Inhalational and subcutaneous routes are not used for antenatal corticosteroid therapy. Educational Objective: Antenatal corticosteroid therapy has been proven to be effective in reducing perinatal morbidity and mortality associated with preterm labor. Intramuscular administration of steroids provides stable and predictable concentration of the drug in the blood that is required to achieve the desired fetal effects. A healthy 41-year-old Asian woman comes to the physician because of pain and persistent morning stiffness in both hands. She has had these symptoms for the past six months. She has used ibuprofen and tapering steroids; however, as soon as she stops using the steroids, the stiffness and pain recur. She does not use tobacco, alcohol, or drugs. Examination reveals synovitis in both metacarpophalangeal and proximal interphalangeal joints, bilaterally. She is started on chronic steroid therapy with prednisone 7.5 mg/day. Which of the following is the most appropriate initial strategy to decrease this patient's risk for developing osteoporosis? A. Bone densitometry every six months B. N-telopeptide in urine, every six months C. Calcitriol and calcium D. Calcitonin E. Alendronate Explanation: Glucocorticoids decrease the intestinal absorption of calcium and increase the calcium excretion in the urine. They also accelerate bone resorption. Treatment should include the administration of calcium (three times a day) and calcitriol (vitamin D) because this therapy has been shown to decrease the rate of bone loss. (Choice A) Patients who are going to be treated with corticosteroids for more than three months (or six months if low doses, i.e., <10 mg/day), should be evaluated with baseline densitometry, and the test should be repeated every year as long as the therapy continues. (Choice E) Alendronate can be added to calcium and vitamin D if the risk of osteoporosis is very high. Alendronate (bisphosphonates) alone, without calcium and vitamin D supplementation, is not efficacious. (Choice D) Calcitonin is used if the bisphosphonates are contraindicated or not tolerated. Educational Objective: Chronic glucocorticoid therapy increases the risk of osteoporosis. For this reason, it is important to provide the lowest dose of steroids for the shortest duration, and to add vitamin D as well as calcium supplementation to the patient?s therapy. Based on the duration of therapy, bone densitometry is indicated for every year. The following vignette applies to the next 2 items A 9-year-old Caucasian boy is brought to the office by his parents. He complains of left knee pain and swelling for two weeks, and these symptoms have worsened over the past five days. He now has difficulties in walking. His mother recalls that he was bitten by a tick six months ago. Three weeks later, he developed an annular erythematous lesion in his left groin. The lesion improved with a twoweek course of topical clotrimazole. At that time, he had malaise and low-grade fever. His serology was suspicious for Lyme disease, but since his symptoms disappeared quickly, no treatment was given. Physical examination reveals a swollen, erythematous, warm and mildly tender left knee. Minimal pain is elicited with full extension of the knee. The range of motion is decreased due to stiffness, and there are no lesions noted on the hands or feet. There are no neurological abnormalities.

Item 1 of 2 Which of the following is the most appropriate test to confirm the diagnosis? A. Serum ELISA antibodies study B. Western blot serology C. Rheumatoid factor and erythrocyte sedimentation rate (ESR) D. Arthroscopy with synovial biopsy E. ELISA antibodies in synovial fluid Explanation: The patient?s presentation and past history (previous tick bite and a rash that seems like erythema migrans) highly suggests Lyme arthritis. Lyme monoarthritis usually develops after 4 months of exposure to the vector. It is characterized by accumulation of a great amount of synovial fluid, marked joint stiffness, and mild pain. The most frequent affected joint in children is the knee. Polyarticular and migratory forms are usually seen early in the disease, but these are really polyarthralgias without much local joint inflammation. The evaluation of antibodies (anti-Borrelia burgdorferi) through ELISA and western blot of the synovial fluid is an excellent way to confirm Lyme arthritis. (Choices A and B) The general approach to testing for Lyme disease has two steps, in which a Western blot is performed to confirm a previous positive ELISA test. However, the diagnosis of Lyme arthritis cannot be confirmed with serology tests. Serology studies are not enough to prove that the joint?s compromise is due to Lyme disease. Serology identifies not only those with the disease, but also those with a previous exposure; therefore, a positive result is non-discriminatory. Moreover, serology can be negative in patients who were partially treated or in those who received antibiotics for another reason. (Choice C) Rheumatoid factor and ESR are helpful in the evaluation of rheumatoid arthritis. (Choice D) Although Lyme arthritis is a chronic joint infection, synovial biopsy is not needed. Educational Objective: The diagnosis of Lyme arthritis cannot be confirmed with serology tests. Serology identifies not only those with the disease, but also those with a previous exposure; therefore, a positive result is nondiscriminatory. Moreover, serology can be negative in patients who were partially treated or in those who received antibiotics for another reason. Synovial fluid analysis with measurement of antibody titers through ELISA or Western blotting is needed to confirm the diagnosis. Item 2 of 2 A left knee arthrocentesis is performed, and this greatly relieves the patient?s joint stiffness. He is now able to walk almost normally. Some blood tests are also performed. After one week, he comes to the office with his parents for a follow-up visit. His laboratory test results are as follows: CBC Hb: 13.8 g/dL Platelet count: 400,000/cmm Leukocyte count: 7,500/cmm Segmented neutrophils: 68% Bands: 3% Lymphocytes: 29% Serum chemistry Serum Na: 140 mEq/L Serum K: 3.8 mEq/L Chloride: 105 mEq/L Bicarbonate: 24 mEq/L

BUN: 17 mg/dL Serum creatinine: 0.8 mg/dL Calcium: 10.0 mg/dL Glucose: 84 mg/dL Rheumatoid factor: Negative Erythrocyte sedimentation rate (ESR) 40 mm/h ELISA test for Lyme disease: Positive for IgG, equivocal for IgM Western blot for Lyme disease: Positive for 5 IgG bands Synovial fluid cell count: 12,000 cells /uL; Neutrophils 75% Lymphocytes 25% Crystals: Absent Synovial fluid ELISA for Lyme disease: Pending Synovial fluid Western blot for Lyme disease: Pending Therapy is started while waiting for the confirmatory results. The parents are especially concerned about their son?s prognosis. Which of the following is the most accurate statement regarding this matter? A. The prognosis is excellent. Almost all patients are cured after six months of therapy. B. The prognosis is fair. Patients usually have recurrent episodes of arthritis despite therapy, but most of them are disease-free after eight years. C. The prognosis is guarded. Improvement occurs early, but it is transient. The disease comes and goes for many years. D. The prognosis is good. More than 90% of cases are cured by antibiotic therapy and are free of the disease after one year. E. The prognosis is fair. Recurrent episodes are around five per year during the first year, but these later decrease and cease after eight years. Explanation: This patient most likely has Lyme disease. The presence of 2 IgM bands (in the first few weeks) and 5 IgG bands (after the first few weeks) is the criteria for the positive western blot. The prognosis of Lyme arthritis is good. Most patients (approximately 90%) are disease-free one year after the start of therapy. If the clinical suspicion of Lyme arthritis is high, antibiotic therapy may be started even when results are pending because synovial fluid serology usually takes time. Majority of patients will respond immediately. Recurrence of the disease is possible, but this is not the norm. (Choice B) Patients who are left untreated can have recurrent episodes of arthritis until eight years after the diagnosis; however, majority of the cases are self-limited. (Choice A) Although a great number of patients respond immediately, others may continue to be symptomatic or they may experience recurrences. Due to this possibility of recurrence, the clinician must wait until one year has passed to consider the patient cured. (Choice E) Those who are not treated have approximately five recurrences per year, and the cure rate is approximately 20 to 10% per year, until they are disease-free after approximately eight years. (Choice C) Lyme disease is curable. Antibiotic therapy does not offer only transient improvement. Educational Objective: Lyme arthritis in children can be treated with a 30-day course of amoxicillin or parenteral ceftriaxone. Doxycycline and ceftriaxone are the drugs of choice for adults. Those allergic to penicillin can be managed with erythromycin. Oral therapy is preferred, unless there is a suspicion of neurologic compromise. The prognosis is good. More than 90% of children are disease-free one year after therapy. If arthritis recurs despite antibiotics, parenteral therapy is indicated. Those who are not treated can have recurrent episodes of arthritis even eight years after the onset of the disease. An 82-year-old Caucasian man is admitted to the hospital after an out-of-hospital cardiac arrest. He was successfully resuscitated and intubated in the field by paramedics, and transferred to your

intensive care unit. On day three of hospitalization, he remains unresponsive to all stimuli, and is deemed braindead by the criteria. His wife brings in the advance directives that day, which clearly states that the patient does not wish to be on any kind of life sustaining treatment, including mechanical ventilation. The wife agrees with the advance directive, and wants to proceed with withdrawal of life support. The rest of the family, including three sons and two daughters, gets very upset with the decision, and strongly oppose the withdrawal of life support. They want to have "at least two weeks" before the final decision regarding withdrawing mechanical ventilation can be made. Which of the following is the most appropriate next step in the care of this patient? A. Terminate all life sustaining support now. B. Terminate all life sustaining support after 2 weeks if there is no improvement in the patient?s condition. C. Talk with the family and discuss any secondary gains that may be affecting their decision to withhold life support. D. Bring up the issue with the ethics committee of the hospital. E. Move a petition in the court against the family?s decision. Explanation: Physicians have an obligation to relieve pain and suffering, and to promote the dignity and autonomy of dying patients in their care. If the performance of these duties conflict with the interests of other people involved in the patient care, the preferences of the patient should prevail. The principle of patient autonomy requires that the physicians should respect the decision to withhold or withdraw life-sustaining treatment of a patient who possesses decision-making capacity. Life-sustaining treatment is any treatment of a patient that serves to prolong his life without reversing the underlying medical condition. A competent adult patient may, via an advance directive, formulate and provide valid consent to withhold or withdraw life-supporting treatment in the event that any injury or illness renders that individual incompetent to make such a decision. If the patient was previously clear in expressing his wishes, and is subsequently rendered incapable of making a decision, the physician has the obligation to respect these wishes, and inform the family/surrogate about the patient's prior statements. (Choice B) The decision to withdraw life support should be made now, and not after two weeks. (Choice C) There is no evidence of any secondary gains that could affect the decision to withhold life support. Furthermore, the patient has previously stated his wishes in an advance directive. Bringing up this issue would be inappropriate. (Choices D and E) The use of courts or an ethics committee to resolve a dispute and aid in sound decision making is necessary only in incompetent patients who do not have an advance directive or a surrogate decision maker. Educational Objective: Patient preference should always prevail in case of any conflict regarding the withdrawal of life support. A 46-year-old Caucasian man is brought to the emergency department. His wife states that he has been having "a bizarre behavior" for two days. His past medical history is significant for cirrhosis secondary to chronic Hepatitis C infection. His daily medications are spironolactone, lactulose and furosemide. He is an ex-heroine user. Examination shows a drowsy, but restless patient with an altered mental status. The abdomen is distended, soft, and non-tender to palpation. Shifting dullness is present. There is moderate asterixis, scattered facial telangiectasis, and jaundice. Intensive therapy was started: diuretics were stopped, and a paracentesis was done. Three days later, the patient appears more awake, alert and responsive. Hand tremors are present, but minimal. The nurse calls you and reports that the patient?s urinary output has been low since admission, and has fallen further, from 450 ml to 200 ml per day in the last 24 hours. Blood pressure is 100/60 mmHg. Examination shows no other changes. The patient has a Foley catheter in place. Laboratory tests are ordered, which show the following:

Serum chemistry Serum Na: 132 mEq/L Serum K: 3.8 mEq/L Chloride: 106 mEq/L Bicarbonate: 20 mEq/L BUN: 60 mg/dL Serum Creatinine: 2.8 mg/dL Calcium: 8.2 mg/dL Blood Glucose: 68 mg/dL LFT Total bilirubin: 6.2 mg/dL Direct bilirubin: 3.7 mg/dL Alkaline phosphatase: 200 U/L Aspartate aminotransferase: 90 U/L Alanine aminotransferase: 108 U/L Total Protein: 6 g/dL Albumin: 2.4 g/dL Which of the following is the more appropriate diagnostic test? A. Urinary sodium B. Urinalysis C. Fractional excretion of sodium D. Renal ultrasound E. Radionuclide scan of the kidney Explanation: Patients with cirrhosis and ascites have up to 39% risk of developing hepatorenal syndrome (HRS) at five years. When a cirrhotic patient with portal hypertension presents with signs of renal failure (altered mental status, low urine output, increased creatinine level), has no other possible cause for the renal dysfunction, and does not improve after volume expansion or diuretic withdrawal, HRS must be suspected. To make a diagnosis of HRS, urinary sodium levels (which should be below 10 mEq/L) and urine osmolality (which should be higher than plasma osmolality) should be determined. Other available modalities may be used to rule out other etiologies of renal failure; however, they are not specific for HRS. (Choice B) Urinalysis can show granular and epithelial cell casts in both acute tubular necrosis (ATN) and marked isolated hyperbilirubinemia. (Choice C) A fractional excretion of sodium under 1% will be found in both HRS and cirrhosis. (Choices D and E) Renal imaging studies are useful in order to check for other causes of renal failure; they are not specific for HRS. Educational Objective: HRS is a serious complication of cirrhosis, and is characterized by oliguria, a creatinine higher than 1.5 mg/dL, and a decreased sodium excretion. It can be seen in patients affected by spontaneous bacterial peritonitis, infection or gastrointestinal bleeding. The best diagnostic tests are determination of urinary sodium levels (which should be lower than 10 mEq/L) and urine osmolarity (which should be greater than plasma osmolality). A 52-year-old Mexican man comes to your clinic. As his primary care physician, you are following him for his chronic conditions, which include diabetes mellitus type-2, obesity, and mild hypertension. He has been on glyburide 5 mg twice a day and enalapril 10 mg daily for the past eight months. The fingerstick done by the nurse shows that his glucose level is 275 mg/dL. During the funduscopic

evaluation, background diabetic retinopathy is evident. How will you proceed to avoid further retinal damage? A. Refer the patient to an ophthalmologist for evaluation. B. Add insulin to the regimen to tighten glucose control. C. Add aspirin to prevent proliferative retinopathy. D. Refer the patient to an ophthalmologist for photocoagulation. E. Increase the dose of glyburide to tighten glucose control. Explanation: A tight glucose control is paramount to prevent or stop the progression of diabetic retinopathy. Intensive insulin therapy can prevent more than 50% of the cases and retard the progression of the disease. A good control of hypertension has also proved to be useful, especially through the use of ACE inhibitors. (Choice E) Unfortunately, conventional therapy with glyburide has shown only meager results in the prevention of diabetic retinopathy progression. (Choice C) Aspirin has no role in diabetic retinopathy management, as shown in the Early Treatment of Diabetic Retinopathy Study. (Choice D) Photocoagulation is only needed in advanced, proliferative disease, not in the initial stages. (Choice A) An evaluation by an ophthalmologist is always advisable since primary care physicians miss the diagnosis of background retinopathy almost two-thirds of the time; however, in this case, the disease has already been identified, and there is no need for photocoagulation. A referral to the ophthalmologist is therefore not necessary. Educational Objective: Tight glucose control through insulin therapy and adequate blood pressure control is currently the best means to slow the progression of background diabetic retinopathy. Conventional oral therapy for diabetes has not been found to be so beneficial. A 76-year-old Caucasian male comes to see you in the office. He complains that he has visible and palpable tortuous swelling on both of his lower legs. It is associated with mild leg heaviness and cramping, especially in the evening. His symptoms are worse with prolonged standing, and improve with leg elevation. He has a past medical history of hypertension, hyperlipidemia, benign prostatic hyperplasia, and degenerative joint disease of both knees. On examination of his lower legs, you notice a few scattered palpable veins consistent with a diagnosis of varicose veins. There is no edema, chronic skin hyperpigmentation, or leg ulcers. Peripheral pulses are full and equal on both sides. Which of the following is the most appropriate next step in the management of this patient? A. Leg elevation and compression stockings B. Injection sclerotherapy with saline C. Injection sclerotherapy with saline and local anesthetic D. Surgical ligation and stripping E. External laser therapy Explanation: Varicose veins refer to visible, palpable, and tortuous superficial veins of the legs, usually on the calf or medial thigh. Most of the patients with varicose veins are asymptomatic. Some patients may complain of leg cramping, heaviness, fatigue, and swelling. The symptoms are generally worse in the evening with prolonged standing, and improve with leg elevation.

Treatment of varicose veins is aimed at alleviating symptoms, although some patients may seek a physician's attention due to cosmetic reasons. Various treatment options for varicose veins include leg elevation, compression stockings, sclerotherapy, and surgical ligation. Most patients with symptomatic varicose veins should initially be treated with conservative measures such as leg elevation and compression stockings. Compression stockings, along with leg elevation and weight reduction, decreases the venous pressures in the lower extremities by direct compression, and improves the patient?s symptoms. Compression stockings should not be used in patients with an underlying arterial insufficiency. (Choices B and C) Injection sclerotherapy with or without anesthetics is used in patients with symptomatic, small, varicose veins who have failed at least three to six months of conservative treatment. It involves the injection of a sclerosing agent into the affected vein. It causes endothelial damage and sclerosis of the involved vein, thus preventing further vein filling. (Choice D) Surgical ligation and stripping is used in patients with large symptomatic varicose veins with ulcers, bleeding, or recurrent thrombophlebitis of the veins. (Choice E) External laser treatment is used to treat particular veins and/or telangiectasias. It is not used for the treatment of varicose veins. Educational Objective: Most patients with symptomatic lower extremity varicose veins should be treated initially with conservative measures such as leg elevation, weight reduction, and compression stockings.

The following Vignette applies to the next 2 items A 26-year-old HIV-positive woman presents to the emergency department complaining of "a nosebleed that just won?t stop." Earlier today, she was playing a game of softball with some friends when she was hit in the face by a ball. The impact caused her nose to start bleeding profusely from the right nostril. She then pinched her nose shut and leaned forward for 15 minutes but the bleeding persisted, which prompted her to come to the emergency department. She does not recall any similar episodes occurring before, though her menstrual flow has become heavier lately. She was diagnosed with HIV two years ago after attempting to donate blood. Her medical history is also significant for gastroesophageal reflux disease and migraine headaches, and her medications include rizatriptan as needed and a multivitamin. Her temperature is 36.9C (98.4F), blood pressure is 116/78 mm Hg, pulse is 70/min, and respirations are 14/min. On examination, blood is dripping steadily from the right nostril. The nasal mucosa appears otherwise normal. The mouth and pharynx appear normal and without lesions. No blood is visible in the posterior throat. Cranial nerves are intact and the remainder of the neurologic exam is normal. Chest auscultation is unremarkable and heart sounds are normal. Her abdomen is nontender and nondistended. There is some splenomegaly. There is no evidence of purpura or petechiae. Laboratory evaluation reveals the following: CBC Hb: 13.2 g/dL Hct: 41.1 % MCV: 90 fl Platelet count: 38,000/cmm Leukocyte count: 3,900/cmm Segmented neutrophils: 79% Band neutrophils: 2% Lymphocytes: 10% Monocytes: 5% Eosinophils: 3% Basophils: 1%

Coagulation Studies Prothrombin time: 13 sec Partial thromboplastin time: 24 sec CD4 count: 430/mm3 Viral load: <1500 copies/mL Serum ANA: Normal (<1:40) Item 1 of 2 An icepack is applied to the woman?s nose and the bleeding soon stops. She receives a dose of intravenous immune globulin. Which of the following treatments should be recommended next? A. Corticosteroids B. Interferon alfa C. Zidovudine D. Dapsone E. Splenectomy Explanation: Although platelet abnormalities become more likely as immune function deteriorates, thrombocytopenia has been known to occur at all stages of HIV infection (whether during the asymptomatic period or end-stage AIDS). Primary HIV-associated thrombocytopenia (PHAT) affects approximately 40% of HIV-positive patients at some point. From a clinical standpoint, PHAT is highly similar to idiopathic thrombocytopenic purpura (ITP). Up to 20% of patients with PHAT will experience a spontaneous remission. However, most will require some intervention. The mainstay of therapy for PHAT is zidovudine (Choice C), which is known to also reduce the occurrence of opportunistic infections and Kaposis sarcoma. Even though it has the potential for suppression of myeloid and erythroid precursors, AZT increases platelet production HIV patients. Experts recommend the inclusion of zidovudine at a minimum dose of 600 mg/day in a multiagent antiretroviral regimen. If the thrombocytopenia does not begin to resolve within 4-8 weeks of therapy, the zidovudine dosage can be increased to 1,000-1,500 mg/day. Corticosteroid use (Choice A) is an effective means of treating PHAT, but the condition often returns once the steroids are tapered. In addition, steroid use in HIV-positive patients can accelerate the development of conditions such as Kaposi?s sarcoma and can increase the risk of opportunistic infections. It is therefore recommended that steroids be used only for brief periods in HIV-positive patients. Interferon alfa (Choice B) is a second-line treatment for patients with chronic PHAT who have not adequately responded to zidovudine. It is not an ideal choice because platelet counts return to baseline within a few weeks of therapy cessation and significant side effects (eg, fever, fatigue, headache, and depression) are observed in many patients. Dapsone (Choice D) is sometimes used as an adjuvant to zidovudine because it has been shown to elevate platelet counts in patients with PHAT. It has the potential to cause a slight reduction in hemoglobin secondary to hemolysis. Splenectomy (Choice E) may be recommended for those patients with persistent PHAT or dependence on repeated infusions of intravenous immune globulin. Educational Objective: Primary HIV-associated thrombocytopenia (PHAT) affects approximately 40% of HIV-positive patients at some point. The mainstay of therapy for PHAT is zidovudine, which is known to also reduce the occurrence of opportunistic infections and Kaposi?s sarcoma. Item 2 of 2

Appropriate action was taken. The patient decided to begin highly active antiretroviral therapy (HAART). Assuming she is able to adhere to the three-drug regimen, what is her expected HIV viral load after six months of treatment? A. 0-50 copies/mL B. 100-500 copies/mL C. 1,000-5,000 copies/mL D. 10,000-50,000 copies/mL E. 100,000-500,000 copies/mL Explanation: In patients who have been started on HAART, the HIV viral load should be measured at four weeks and then again at eight to twelve weeks after the initiation of treatment. Remeasuring should be performed every six to eight weeks until the viral load is undetectable (<50 copies/mL), at which point the viral load can be evaluated every three months. Patients who are treatment-na or who have better baseline immune function (ie, higher CD4 cell counts and lower viral loads) appear to have the best response to HAART, often achieving extremely low nadirs in viral load within 4-6 months after the initiation of therapy. In a treatment-na HIV-positive patient, it is expected that a three-drug regimen will decrease the viral load to <50 copies/mL within 16-24 weeks (Choice A). Virologic failure is defined as the inability to achieve this benchmark, and may occur secondary to the development of drug resistance or the failure of the drugs to reach the intended sites. Within 8-16 weeks after the initiation of HAART, it is expected that a three-drug regimen will decrease the viral load to <500 copies/mL (Choice B). Within 4 weeks after the initiation of HAART, it is expected that a three-drug regimen will decrease the viral load to <5000 copies/mL (Choice C). Higher levels of HIV (Choices D and E) are more commonly seen in patients who are not on HAART. Educational Objective: In a treatment-na HIV-positive patient, HAART should decrease the viral load to <50 copies/mL within 6-months.

A 69-year-old male comes to the emergency department. He is complaining of a severe pain in his abdomen which started 30 minutes ago, and is gradually getting worse. The pain is characterized as 10/10 in intensity, sharp, non-radiating, non-shifting, located mainly in the periumbilical region, and with no aggravating or relieving factors known to the patient. He currently feels nauseated, and has vomited ?yellowish-green stuff? once. His past medical history is significant for hypertension and recent MI, for which he underwent a coronary artery angiogram. This revealed a 100% left anterior descending artery occlusion, for which a stent was placed. Vital signs on examination are: blood pressure 110/75 mm Hg, pulse 100/min and irregular, respirations 18/min, and temperature 37.4C(99.4oF). Only a mild tenderness is noted in the umbilical region. His EKG reveals atrial fibrillation and an old anterior infarct. Plain x-ray and CT scan of the abdomen do not reveal any abnormalities. Labs show: Serum Na: 139 mEq/L Serum K: 4.1 mEq/L Chloride: 105 mEq/L Bicarbonate: 14 mEq/L BUN: 22 mg/dL Serum creatinine: 1.0 mg/dL Blood glucose: 118 mg/dL Amylase: 250 U/L Lipase: 15 U/L CBC Hb: 15g/dL Platelet count: 250,000/cmm Leukocyte count: 16,000/cmm What is the most likely diagnosis of this patient? A. Small bowel volvulus B. Acute mesenteric ischemia C. Acute appendicitis D. Perforated peptic ulcer E. Acute pancreatitis Explanation: Acute onset of severe abdominal pain wherein the physical examination findings do not match the severity of pain, along with metabolic acidosis, is generally mesenteric ischemia unless proven otherwise. This condition is frequently overlooked initially. All radiological studies may be initially normal. Laboratory data may show a mild to moderate metabolic acidosis. Leukocytes and elevated Hb and Hct% in the CBC may occur secondary to hemoconcentration. The ischemia may be due to an occlusion secondary to thrombosis, embolism and/or vasospasm. The most common cause is superior mesenteric artery (SMA) occlusion secondary to an embolism. The SMA is the most common site involved because it runs off at a minimal angle from the aorta and has a wide diameter. The origin of the embolus is commonly from the left atrium, left ventricle or cardiac valves. Risk factors are cardiac arrhythmias, recent MI and abdominal malignancy. The patient may later develop abdominal distention and signs of acute peritonitis. If not treated promptly, consequences may be very serious. Untreated mesenteric ischemia may cause bowel infarction, sepsis and death. Mortality in this case may be as high as 60%. (Choice A) The patient?s presentation, abdominal examination findings, and negative x-ray and CT scan results make the diagnosis of an intestinal obstruction unlikely. (Choice C and D) The negative CT scan and negative x-ray rules out volvulus, acute appendicitis (no signs of inflammation on CT scan) or perforated peptic ulcer (no signs of gas seen under diaphragm).

(Choice E) Serum amylase, lactic dehydrogenase (LDH), creatine phosphokinase, or alkaline phosphatase can be elevated in mesenteric ischemia. The normal lipase makes the diagnosis of pancreatitis unlikely. Educational Objective: Acute onset of severe abdominal pain wherein the physical examination findings do not match the severity of pain, along with metabolic acidosis, is generally mesenteric ischemia unless proven otherwise. Laboratory data may show elevated Hb and Hct% secondary to hemoconcentration and leukocytosis. The following vignette applies to the next 2 items A 63-year-old Caucasian male presents to his primary care physician for a new patient visit. He is a pleasant and talkative man who has no new complaints; he reports that he "has never felt healthier." He has come prepared with a list of regularly taken medications. Included on the list are atenolol, sildenafil, terbinafine, minoxidil (topical), aspirin, and saw palmetto. He has no known drug allergies and has a remote history of cholecystectomy. He drinks one ounce of bourbon before bed each night. He has a remote 15-pack-year history of smoking cigarettes and has never used recreational drugs. He is married and has three grown children, and was recently transferred to this city to assume direction of a local factory. Physical examination is remarkable for a soft mid-systolic murmur that is loudest in the second right intercostal space. There is some bony enlargement of the distal interphalangeal joints bilaterally. The patient wishes to discuss his medication list in more depth and indicates an increasing interest in alternative medicines. Item 1 of 2 Which of the following conditions is saw palmetto most commonly used to treat? A. Anxiety B. Osteoarthritis C. Benign prostatic hyperplasia D. Hypercholesterolemia E. Depression Explanation: Alternative medicine is enjoying an increasingly wide audience within the United States, with as many as 83 million adults reporting the usage of complementary therapies. Herbal preparations are not regulated by governmental agencies and are generally classified as food or dietary supplements, enabling manufacturers to avoid the scientific scrutiny exercised when prescription drugs are readied for market. The limited data that exists regarding herbal preparations is, unfortunately, often flawed. Despite these difficulties, health care practitioners should make an effort to recognize the primary indications and adverse effects of the more popular herbal remedies. Saw palmetto is one such herbal preparation, and men most often use it for the treatment of benign prostatic hyperplasia (Choice C). Saw palmetto has been shown to improve urinary symptom scores, nocturia, and peak urine flow. Its efficacy appears similar to that of finasteride, and adverse effects are unusual and mild. Anxiety (Choice A) and insomnia are sometimes treated with kava, an herbal preparation originally used in Polynesia. Multiple placebo-controlled trials demonstrated that kava has significant anxiolytic properties. Osteoarthritis (Choice B) is often treated with glucosamine and/or chondroitin, two preparations generally considered to be safe. Glucosamine may alter glucose regulation or sensitivity to insulin, and diabetic patients should be cautioned accordingly. Hypercholesterolemia (Choice D) is sometimes treated with preparations of garlic. There is limited data regarding its efficacy. St. John?s wort is frequently used in the treatment of depression (Choice E). The available clinical data suggests that it is significantly more effective than placebo for most patients. In vitro, it appears to inhibit the reuptake of serotonin, dopamine, and norepinephrine.

Educational Objective: Saw palmetto is a popular herbal preparation used in the treatment of benign prostatic hyperplasia. Kava is used for anxiety and insomnia. Garlic is used for hypercholesterolemia. St. John?s wort is frequently used in the treatment of depression. Glucosamine is used for osteoarthritis. *Extremely important question for USMLE step-3 Item 2 of 2 The patient is counseled appropriately regarding his use of alternative medicines. Which of the following side effects is associated most strongly with saw palmetto? A. Bleeding and platelet dysfunction B. Hepatotoxicity C. Hypertension D. Nephrotoxicity E. Photosensitization Explanation: The milder side effects observed with the use of saw palmetto are similar to those seen with placebo and include headache, nausea, and dizziness. Hypertension (Choice C) is the most serious adverse effect noted, and one study found it occurred in 3.1% of patients taking saw palmetto. Bleeding and platelet dysfunction (Choice A) are associated with usage of gingko biloba and garlic, two popular herbal preparations. Hepatotoxicity (Choice B) in the form of hepatitis, cirrhosis, and liver failure is associated with kava. Other herbal supplements known to cause liver toxicity include comfrey, borage leaf, coltsfoot, ephedra, chaparral, germander, and a Chinese medicine called jin bu huan. Nephrotoxicity (Choice D) in the form of fulminant tubulointerstitial renal disease has been observed with the use of some Chinese herbs, especially those containing artistolochic acid. This particular compound is included in many Chinese herbal weight loss regimens. The concurrent use of renal vasoconstrictors fenfluramine and diethylpropion may hasten the development of nephrotoxicity. The most common acute drug reactions to St. John?s wort (hypericum) include gastrointestinal distress, dizziness, fatigue, and dry mouth. Longer-term studies have found an increase in anorgasmia, urinary frequency, and swelling. Photosensitization (Choice E) can be seen with intravenous administration of hypericum in HIVpositive patients. Educational Objective: Hypertension is a side effect of saw palmetto. Bleeding and platelet dysfunction can be seen with Ginkgo and garlic (check bleeding time). Hepatotoxicity is seen with Kava. The following vignette applies to the next 2 items A 36-year-old African-American female is seen in the emergency department (ED) for the evaluation of a sudden onset, left lower chest pain and difficulty in breathing. Approximately one year ago, she was diagnosed with left lower extremity deep vein thrombosis secondary to smoking and oral contraceptive pill (OCP) use. At that time, her OCPs were discontinued, she quit smoking, and she was treated with six months of warfarin therapy. She does not have any other medical problems. Her mother and maternal aunt have had "problems with blood clots." After performing the appropriate ancillary procedures, results reveal the presence of multiple, left-sided pulmonary emboli. A diagnosis of an inherited thrombophilia is suspected. Item 1 of 2 Which of the following is most likely to be present in this patient? A. Factor V Leiden mutation B. Low protein C levels

C. Prothrombin gene mutation D. Antiphospholipid antibody E. Antithrombin III deficiency Explanation: The patient in the above scenario has suffered from a second episode of venous thromboembolism in a year. Factor V Leiden is the most common cause of inherited or heredity thrombophilia, accounting for approximately 40-50% of the inherited thrombophilias. These disorders predispose the patient to the development of venous thromboembolism. (Choices B, C, D and E) Other common causes of heredity thrombophilia include prothrombin gene mutation, hyperhomocysteinemia, protein C, protein S, and antithrombin III deficiency. Some patients may have more than one type of inherited disorder, and are at even greater risk for venous thromboembolism. Educational Objective: Factor V Leiden is the most common cause of inherited or heredity thrombophilia, accounting for approximately 40-50% of the inherited thrombophilias. Item 2 of 2 The results of the patient's hypercoagulation work-up are as follows: Factor V Leiden mutation: Negative Protein C and S levels: Normal Antithrombin III levels: Normal Antiphospholipid antibody: Negative Prothrombin gene mutation: Positive MTHFR (methyltetrahydrofolate reductase) gene mutation: Heterozygous Homocysteine levels: Normal Which of the following is the most appropriate next step in the long-term management of this patient? A. Prescribe warfarin for another one year B. Prescribe subcutaneous low-molecular-weight heparin for another one year C. Start the patient on high dose folic acid (3 mg) per day D. Prescribe life long warfarin therapy E. Start her on folic acid 1 mg per day Explanation: The patient in the above scenario has suffered from a second episode of venous thromboembolism in a year. Her initial laboratory evaluation for an underlying inherited thrombophilia has revealed the presence of prothrombin gene mutation. According to the current recommendations, all patients with a history of spontaneous venous thromboembolism in the presence of more than one inherited thrombophilia should have indefinite anticoagulation. Lifelong anticoagulation is also recommended for patients with hereditary thrombophilia who present with recurrent spontaneous thrombosis, life-threatening spontaneous venous thromboembolism (e.g. massive pulmonary emboli), or thrombosis at an unusual site (mesenteric or cerebral vein). (Choices A and B) All patients with recurrent thrombosis and evidence of a hereditary defect which predisposes them to venous thromboembolism should receive lifelong anticoagulation. (Choices C and E) Hyperhomocysteinemia is an independent risk factor for coronary, cerebrovascular, and venous thromboembolic disease. Therapy with daily folic acid supplementation has been shown to reduce the homocysteine levels in patients with hyperhomocysteinemia. The patient is heterozygous for MTHFR gene mutation and her homocysteine levels are normal; therefore, therapy with folic acid is not necessary.

Educational Objective: All high-risk patients with a history of spontaneous thrombosis and a predisposing heredity defect should receive indefinite anticoagulation to reduce the risk of recurrent venous thromboembolism. A 52-year-old gravida 3, para 3 Caucasian bookstore manager presents to the primary care physician for a routine follow-up examination. She was diagnosed with diabetes mellitus sixteen years ago, and controls the condition well with insulin. When she visited her gynecologist two weeks ago for her annual examination, she complained of frequent hot flashes and changes in her menstrual cycle. Over the past six months, she has had either no menses or irregular spotting, but last month she experienced a much heavier menstrual flow. She is divorced and has not been sexually active for several years. Her vital signs are normal. The physical examination is remarkable only for mild obesity. The gynecologist recommended a specific test to further evaluate her condition; however, she is unable to recall the name or purpose of this test, and asks you to clarify the management plan for her gynecologic concerns. Which of the following is the most appropriate response? A. We must obtain a pregnancy test before we can discuss these concerns further B. Your symptoms are suggestive of perimenopause. A vaginal ultrasound is indicated to evaluate your uterine lining C. Your symptoms are suggestive of perimenopause. Measurement of serum luteinizing hormone is needed for clinical confirmation D. Your symptoms are suggestive of menopause. No further testing is necessary E. Your symptoms are suggestive of menopause. Would you like to initiate hormone replacement therapy to minimize your hot flashes? Explanation: Perimenopause is the time extending from two to eight years preceding menopause until one year following the last menstrual period. This state is associated with normal ovulatory cycles interspersed with anovulatory cycles that vary in length. Because the hormone levels are inconsistent during perimenopause and estrogen is frequently unopposed, menses become irregular and heavy breakthrough bleeding may be reported. Endometrial hyperplasia can develop during lengthy intervals of anovulation. If a patient complains of an episode of heavy dysfunctional bleeding or of six or more months of irregular menses, then endometrial surveillance in the form of vaginal ultrasound (to ensure the endometrial thickness is <4mm) or endometrial biopsy is indicated. (Choice A) A pregnancy test would be an appropriate first step in the evaluation of amenorrhea in most women of childbearing age; however, a sexually inactive woman with clinical symptoms highly suggestive of perimenopause is not likely to be pregnant. (Choice C) If the diagnosis of menopause is uncertain, the pathognomonic finding is an elevated level of follicle-stimulating hormone (FSH). The measurement of serum luteinizing hormone (LH) is of less help because serum LH concentrations may be elevated at certain points in the normal menstrual cycle and cannot be readily distinguished from a typical menopausal serum LH value. (Choice D) Menopause is defined as the presence of amenorrhea for six to twelve months in conjunction with the symptoms of menopause (eg, hot flashes, vaginal dryness). If the clinical presentation is classic, then testing is unnecessary. Women with particularly severe symptoms may elect to initiate hormone replacement therapy, although this is usually recommended only for shortterm use. Educational Objective: If a perimenopausal patient complains of an episode of heavy dysfunctional bleeding or of six or more months of irregular menses, then endometrial surveillance in the form of vaginal ultrasound (to ensure the endometrial thickness is <4mm) or endometrial biopsy is indicated. *Extremely important question for USMLE step-3

A 68-year-old female on the surgical floor developed generalized seizures. She underwent an elective hernia repair one day ago. Postoperatively, she was placed on intravenous fluids containing 5% dextrose. She has mild bronchial asthma. She has never been intubated or received oral glucocorticoids for her bronchial asthma. On physical examination, her vital signs are stable. She is in an obtunded state secondary to her postictal status and administration of lorazepam. While the patient is being examined, the laboratory technician calls and reports that her serum sodium is 110 mEq/L. Her other lab tests are pending. What is the next best step in the management of this patient? A. Rapid infusion of 0.9% saline intravenously B. Infuse 3% saline intravenously to increase her serum sodium at the rate of 0.5 to 1 mEq/L per hour C. Infuse 3% hypertonic saline to increase her serum sodium to normal limits within 16 hours D. Check the urine and serum osmolality E. Infuse 3% hypertonic saline to increase her serum sodium to normal limits within eight hours Explanation: Postoperative hyponatremia is a common problem, and is caused by SIADH secondary to the use of anesthetic agents, postoperative pain and nausea. This patient has postoperative severe hyponatremia, which is manifested by her generalized seizure activity. She received hypotonic fluids in the form of 5% dextrose, which aggravated this problem and caused a precipitous fall in serum sodium levels. Severe hyponatremia is preferably treated by a slow infusion of 3% hypertonic saline (513 mEq/L of sodium). The initial goal is to increase the sodium level by 3 mEq/L in the first three hours. Subsequently, the rate of 3% hypertonic saline is adjusted to increase the serum sodium level by 0.5 to 1 mEq/L in the next 12-18 hours. Frequent monitoring of the serum sodium level is warranted to prevent a rapid increase in serum sodium, which may lead to central pontine myelinolysis. The increase in serum sodium level achieved by infusion of one liter of 3% hypertonic saline can be calculated with the following formula: (513 ? serum sodium)/(total body water + 1). For instance, the serum sodium level in this patient is 110 mEq/L and her weight is 60 Kg. Applying these numbers to the formula will give us: (513 ? 110)/(60 x 0.5 + 1) = 403/31 = 13 mEq/L. One liter of hypertonic saline in this patient will increase the serum sodium by 13 mEq/L. (Choice A) Infusion of normal saline is not indicated in patients with SIADH. (Choice D) Checking the urine and serum osmolality is generally used for the diagnosis of SIADH. Waiting for the results will be disastrous, and will not lead to any change in the management of this patient. (Choices C and E) A rapid infusion of 3% hypertonic saline to quickly increase the serum sodium level could be potentially dangerous because it may lead to central pontine myelinolysis. Educational Objective: 3% hypertonic saline is the preferred treatment for severe symptomatic hyponatremia. The serum sodium level should be gradually increased in hyponatremic patients to prevent central pontine myelinolysis. A 26-year-old Caucasian man is brought to the emergency department. He is an alcoholic, and has been drinking vodka for the last 48 hours. He states that he is unable to walk because of severe muscular pain, especially in his thighs. He has smoked one pack of cigarettes a day for the past 12 years, and occasionally uses marijuana or cocaine. He has no other medical problems. His family history is not significant. His temperature is 37.8?C (100?F), blood pressure is 100/60 mmHg, pulse is 116/min, and respirations are 20/min. Examination shows a young man with dry skin and mucous membranes. The lungs are clear. There is generalized muscular tenderness, especially in the thighs

and shoulders. Muscular strength is decreased globally. There are no other neurologic deficits. The patient?s laboratory tests show the following: Serum chemistry Serum Na: 147 mEq/L Serum K: 5.2 mEq/L Chloride: 107 mEq/L Bicarbonate: 20 mEq/L BUN: 58 mg/dL Serum creatinine: 2.3 mg/dL Calcium: 8.2 mg/dL Glucose: 72 mg/dL CK: 60,000 U/L Urinalysis results are pending. Which of the following is the most appropriate course of action? A. Start intravenous furosemide. B. Start intravenous fluids. C. Start sodium polystyrene sulfonate (Kayexalate). D. Start intravenous bicarbonate. E. Request a nephrology consultation. Explanation: This patient is developing rhabdomyolysis (RM). In this condition, the creatine kinase (CK) levels are usually greater than 10,000 U/L. Patients develop muscle tenderness, increased muscle tone, renal failure, hyperkalemia, hypocalcemia or hyperphosphatemia. The latter three characteristics are markers of cell breakdown. Cocaine or alcohol intoxication, as well as severe trauma or extreme exertion, can lead to RM. Neuroleptic malignant syndrome, hypothermia, hypothyroidism and polymyositis have also been associated with RM. Management involves intravenous isotonic saline solutions to ensure adequate hydration. Volume replacement must be started as soon as RM is diagnosed or highly suspected. Time should not be wasted while waiting to identify the etiology of the condition. (Choice D) When adequate hydration is achieved, the next objective will be to alkalinize the urine (to obtain a pH greater than 6). Bicarbonate solutions will be used at this point, but not earlier. (Choice A) Furosemide use will worsen the dehydrated status of the patient. (Choice E) A renal consultation will be necessary if the renal function does not improve after adequate hydration of the patient, since dialysis may be necessary at that point. (Choice C) An attempt to correct the patient?s mild hyperkalemia must first be made using adequate hydration. Restoring renal function by hydrating the patient may be all that is necessary. Concomitant EKG monitoring is mandatory to check for any arrhythmias. Kayexalate administration is not necessary. Educational Objective: Rhabdomyolysis is a serious medical condition, which can be complicated by hyperkalemia, renal failure and hyperphosphatemia. The mainstay of therapy is immediate adequate parenteral hydration, followed by alkalinization of the urine. A 26-year-old Asian-American daycare worker presents to the clinic after prolonged close contact with a child with known meningococcal meningitis. Her past medical history is significant for irritable bowel syndrome and migraine headaches. Her current medications include oral contraceptives, tegaserod, and ibuprofen. She is concerned about her risk for acquiring meningitis, and asks for your advice. In addressing this issue, which of the following is the most appropriate next step?

A. Provide reassurance and no medical treatment. B. Prescribe rifampin 600 mg PO bid for 4 doses. C. Prescribe ciprofloxacin 500 mg PO single dose. D. Prescribe penicillin 250 mg PO tid for 30 doses. E. Admit her to the hospital for quarantine. Explanation: Individuals who have been in prolonged close contact with a case of invasive meningococcal infection should be placed on prophylactic antibiotics to eradicate pharyngeal carriage of the organism. Note that unless there has been direct exposure to respiratory secretions, the majority of healthcare workers do not require chemoprophylaxis. Rifampin (600 mg PO bid for 4 doses) is the standard treatment, but ciprofloxacin (500 mg PO single dose) is an equally acceptable alternative for adult patients who cannot tolerate rifampin. Because studies have demonstrated reduced steroid levels in women who used oral contraceptives concomitantly with rifampin, this daycare worker should be placed on ciprofloxacin instead. (Choice A) Providing reassurance is acceptable only if the woman was not in prolonged close contact with the ill child. Since the woman is a daycare worker and was with the child for several hours each day, she should be treated. (Choice B) Rifampin would be the ideal treatment regimen if this woman were not on oral contraceptives. Whenever possible, it is preferable to prescribe new medications that do not alter the efficacy of current medications. (Choice D) Penicillin is an appropriate treatment for known cases of Group A Streptococcus infection. It is not effective in treating meningococcal meningitis. (Choice E) Admission to the hospital for quarantine is unnecessary. This woman can be treated on an outpatient basis. Educational Objective: A single oral dose of ciprofloxacin is acceptable chemoprophylaxis for Neisseria meningitidis in adults who cannot tolerate rifampin therapy.

A 29-year-old female is admitted to the hospital for an elective cesarean section. She is on her 38th week of pregnancy. The patient was diagnosed with type 1 diabetes three years ago. She is currently on NPH (32 units before breakfast, 26 units at bedtime) and regular insulin (22 units before breakfast, 22 units before supper). Before the admission, her glucose control was acceptable, and she denied hypoglycemic symptoms. She does not have any chronic diabetes complications. The procedure is scheduled on the next day of admission. Which statement correctly describes the proper preoperative management of diabetes in this patient? A. Half dose of NPH and regular insulin on the night before surgery, no insulin on the day of surgery, and insulin according to the sliding scale to keep the blood glucose below 250 mg/dl. B. No insulin one night prior to surgery or on the morning of surgery, and sliding scale to keep the blood glucose below 200 mg/dl. C. Normal insulin dose on the night before surgery, half dose of NPH and regular insulin on the morning of surgery, followed by sliding scale insulin to maintain blood glucose below 200 mg/dl. D. Normal insulin dose on the night before surgery, and start the patient on an insulin drip and infusion of D5 ? NS with 40 mEq of KCl to keep the blood glucose below 160 mg/dl. E. No change in insulin dosage. Explanation:

The management of diabetes during labor and cesarean section can be challenging. During pregnancy, the requirement of insulin increases, particularly during the second trimester, due to the "diabetogenic effects" of the placental hormones. Following a normal labor or cesarean section, there is a decrease in the level of these hormones, which drastically decreases the patient's insulin requirement. The patient has type 1 diabetes mellitus, and should be kept on insulin to prevent ketoacidosis, even if she is not eating. The patient should take her normal dose of insulin on the night before surgery. Since the patient will not be eating after midnight, the procedure should be scheduled during the early morning hours. Insulin infusion is the best way to treat a patient perioperatively, and the drip rate is adjusted to keep the blood glucose under 160 mg/dl. There are no set guidelines for the target glucose levels during and after surgery; however, studies have shown that a tight glucose control prevents perioperative complications in the ICU and post-CABG patients. Intravenous insulin has a half-life of approximately 5 minutes; and its hypoglycemic effects do not last more than 30 minutes. The amount of insulin infused during the last 24-hour period will therefore give a rough idea of the patient's most recent insulin requirements. (Choice C) The patient can be managed with half the dose of NPH and regular insulin before surgery, and sliding scale insulin thereafter; however, this regimen gives suboptimal control. This can be used if facilities for starting an insulin drip are not available; however, it is necessary to remember that the insulin requirement will drop following delivery of the placenta. Patients should be switched to scheduled insulin dosage as soon as they start tolerating food. Educational Objective: Understand the management of diabetes during delivery and labor. Insulin administration should not be stopped in a type 1 diabetic patient even if they are not eating. The insulin requirement significantly decreases following delivery of a pregnant diabetic. A 55-year-old man is brought to the emergency room by his neighbor for evaluation of crushing, substernal chest pain. The pain started two hours ago, and is accompanied by nausea and diaphoresis. The patient has a history of hypertension, diabetes mellitus and hyperlipidemia. His blood pressure is 124/72 mmHg and heart rate is 78/minute. An electrocardiogram done in the emergency room reveals the presence of 3 mm ST segment elevation in leads II, III and aVF. You immediately provide the initial care for an acute myocardial infarction, and call the cardiac catheterization team for an emergent coronary angiogram. When you return to the patient's room to reevaluate him, you find him standing next to his bed. He has taken off the electrodes and the cardiac monitor, and is getting dressed. Upon asking him, he tells you "I wish to go back home. My pain is much better now. I know that I am having a heart attack and I can die from it, but I do not want to undergo cardiac catheterization." Which of the following is the most appropriate next step in the management of this patient?

A. Discharge the patient home and prescribe medications. B. Call security and restrain him. C. Call the neighbor for consent to treat the patient. D. Call the hospital ethics committee. E. Obtain permission through the hospital lawyer to treat the patient. Explanation: When faced with a situation such as the one given above, the physician must always assess the patient's decision-making capacity by determining if the patient has the cognitive ability and emotional stability to make decisions about his healthcare. The patient should be able to understand and rationally evaluate the diagnosis, planned treatment, available alternative treatment, and risks of refusing treatment. Failure to recognize the lack or absence of decision-making capacity puts the patient at risk of harm if he refuses treatment. On the other hand, inappropriately coercing a patient into treatment deprives him of his fundamental right of autonomy and the right to make his own health care decisions. The patient in the above vignette appears to have a clear understanding of his medical condition and the risks (risk of death) of refusing treatment. He does not wish to undergo cardiac catheterization, and wants to go home. The best way to deal with this situation is to educate him regarding the risks and benefits of cardiac catheterization and alternative therapy. He should be questioned regarding the reason behind his decision. If he still does not want any treatment, then the physician has to discharge him with medications (and of course, document everything extensively). (Choice B) If there are any indications that other medical or psychological conditions may be impairing the patient's decision-making capacity, then he can be restrained and treated for these emergent conditions. (Choice C) The patient appears competent and has clearly expressed his wishes to the physician. There is no reason to contact the neighbor for decision-making or consent for procedures. (Choices D and E) The hospital ethics committee should be involved only if there are any doubts regarding the patient?s decision-making capacity. Hospital attorneys are usually needed to appoint a surrogate decision-maker if the patient is deemed incompetent. These approaches can only be used in non-emergent situations. Educational Objective: Every adult patient with a sound mind and understanding has a right to make his own healthcare decisions. A 52-year-old male has just been diagnosed with type 2 diabetes. His past medical history is insignificant. His father had type 2 diabetes and died of a heart attack at the age of 55 years. While you are explaining the importance of tight blood sugar control, he interrupts and says, "doc, I do not want pills for my diabetes because I feel fine, and I do not want to get addicted to the pills." If the patient?s blood glucose is kept under tight control, which of the following would he most likely have a decreased risk for? A. Retinopathy B. Peripheral vascular disease C. Myocardial infarction D. Stroke E. Hypoglycemia Explanation: Epidemiological studies (including the United Kingdom Prospective Diabetes Study, UKPDS) have shown that tight blood glucose control in patients with type 2 diabetes decreases the risk for the development of microvascular complications such as retinopathy, nephropathy, and neuropathy;

however, studies have failed to show any significant benefit of blood glucose control on macrovascular complications such as myocardial infarction. A similar study was performed for patients with type 1 diabetes (Diabetes Control and Complication Trial, DCCT), and this also showed that intensive blood glucose control decreases the risk of all microvascular complications. The decrease of macrovascular complications was not significantly different between the intensive and conventionally treated groups. Some possible explanations to account for this lack of statistical significance are the younger patient population in the DCCT and the low event rate for macrovascular complications. (Choices B, C, and D) As indicated above, the risk for macrovascular complications has not been shown to decrease after the intensive control of blood glucose levels in type 2 diabetes. (Choice E) The risk for hypoglycemia markedly increases with intensive blood glucose control. Patients therefore require careful monitoring of their blood glucose levels to prevent the occurrence of hypoglycemic episodes. Educational Objective: Tight blood glucose control clearly decreases the risk for the development of microvascular complications in type 1 and type 2 diabetes; however, the effect on the development of macrovascular complications with tight blood glucose control is unclear. There is a significant increase in the incidence of hypoglycemia with tight blood glucose control. A 46-year-old Caucasian female is hospitalized for upper gastrointestinal hemorrhage. She had two episodes of coffee ground colored vomiting during the past twelve hours. She has had intermittent episodes of passage of black stools over the last week. She visited a doctor two days ago and underwent an endoscopy that showed erosive gastritis and mild reflux esophagitis. Omeprazole therapy has been started. Her past medical history is significant for polycystic kidney disease and hypertension. Her most recent serum creatinine level is 2.9 mg/dL. Coagulation studies demonstrate the following results: APTT: 30 sec (N 25-40 sec) PT: 12 sec (N 11-15 sec) Bleeding time: 15 min (N 2 ? 7 min) Platelet count: 250,000/mm3 Which of the following is the best next step in the management of this patient? A. Observation and supportive therapy B. IV desmopressin C. Platelet transfusion D. Cryoprecipitate infusion E. Immediate surgery Explanation: Renal dysfunction is associated with an increased tendency to bleed. Impaired platelet function is the major contributor to this tendency, and this is reflected by a prolonged bleeding time. Other coagulation studies, such as prothrombin time (PT), partial thromboplastin time (PTT), and platelet count are usually normal. Correction of the platelet dysfunction is necessary in patients with active bleeding (such as the patient in this case) or those who are about to undergo a surgical procedure. Several available options to correct the abnormality include the following: correction of anemia, administration of desmopressin, dialysis, estrogen treatment, and cryoprecipitate infusion. Desmopressin represents the simplest and least toxic acute treatment of a prolonged bleeding time. It acts by increasing the release of factor VIII:von Willebrand factor multimers from endothelium. (Choice A) Asymptomatic patients do not need any specific therapy.

(Choice D) Cryoprecipitate infusion is associated with infectious complications. Its use is limited to patients who are not responding to blood transfusions and desmopressin. (Choice C) Platelet transfusion is not indicated in such patients because their platelet count is usually normal. (Choice E) Any surgery without preceding correction of platelet dysfunction is very dangerous because it can precipitate significant bleeding. Educational Objective: Platelet dysfunction is the major contributor of increased bleeding tendency in patients with renal failure. Desmopressin represents the simplest and least toxic acute treatment of prolonged bleeding in patients with renal dysfunction. A father brings his 3-year-old son to the emergency department because the child is febrile and has been refusing to walk for one day. There is no known history of trauma or recent illness. While lying down on the examination table, the child keeps his left hip externally rotated. He cries when his hips are internally rotated on examination. What is the preferred imaging modality to evaluate an effusion of the hip in this child? A. Bone scintigraphy B. Computed tomography without contrast C. Computed tomography with contrast D. Ultrasonography E. Magnetic resonance imaging Explanation: Transient synovitis (also known as toxic synovitis) is a common condition that causes pain in the hip, thigh, or knee in boys aged 3-10 years old. Up to 25% of children with transient synovitis will have bilateral effusions. The disorder may be preceded by a respiratory infection, although the erythrocyte sedimentation rate (ESR) and white blood count (WBC) are typically normal. If the clinical presentation suggests transient synovitis and plain radiographs are unremarkable, an ultrasound should be performed. Ultrasonography is the preferred technique for identifying small joint effusions, and may demonstrate widening of the joint space of the hip. This modality is also useful in guiding aspiration of the joint, a procedure that is warranted in a febrile child with hip effusion. (Choices A, B, C, and E) The other imaging modalities are considered less useful in evaluating joint effusions and should not be ordered. Educational Objective: Transient synovitis of the hip is a common, self-limited condition in young boys that is best evaluated by ultrasound. A 14-year-old Caucasian female presents to the pediatrician complaining of discoloration of her toenails, which makes it "impossible to wear sandals." She says that some months ago, she first noticed the big toes of both feet were affected, and that the yellowish brown discoloration began with the distal tips of her toenails and spread proximally to involve the entirety of the nail. On physical examination, the nail plates of all ten toes are heaped up and irregular. Brownish discoloration is evident. Dermatophytic hyphae and arthrospores are visible upon KOH examination of scrapings from the nail bed. What is most effective treatment regimen for this girl?s condition? A. Ciclopirox B. Fluconazole C. Ketoconazole D. Griseofulvin

E. Terbinafine Explanation: Onychomycosis is a fungal infection of the toenails or fingernails. The most common form of onychomycosis is distal subungual onychomycosis, which is caused by the dermatophyte Trichophyton rubrum. Because nail dystrophies caused by other diseases (e.g., psoriasis, lichen planus, eczematous conditions) can mimic the appearance of onychomycosis, many insurance companies require diagnostic confirmation of the condition before reimbursing treatment costs. The diagnosis is typically established with KOH examination of nail bed scrapings. If KOH examination is negative, then evaluation by culture is appropriate. Oral terbinafine (Choice E) is one of the newest medications available for treating this condition, and is considered superior in terms of efficacy. Infection of the fingernails requires 6 weeks of therapy with terbinafine, while infection of the toenails requires 12 weeks of therapy. Ciclopirox (Choice A) is a topical antifungal nail lacquer recently approved for the treatment of mild to moderate onychomycosis. Although ciclopirox is considered safe, studies show its efficacy to be minimal. Fluconazole (Choice B) is an option in the treatment of onychomycosis, and its weekly dosing may be particularly appealing for those patients with complex medication regimens. However, it is less effective and more expensive than terbinafine and itraconazole. Ketoconazole (Choice C) and griseofulvin (Choice D) are rarely used now that terbinafine and itraconazole are available. These older medications required lengthy treatment regimens (often up to 18 months) and frequent laboratory monitoring, and were associated with high relapse rates and numerous adverse effects. Educational Objective: Oral terbinafine or itraconazole are considered superior methods of treatment for confirmed cases of onychomycosis. Previously griseofulvin was the drug of choice. The following vignette applies to the next 2 items A 24-year-old gravida 3, para 2 Caucasian woman at 37 weeks of gestation was admitted to the labor and delivery unit after the premature rupture of membranes. She attempted to deliver vaginally but when she failed to progress, a cesarean section was performed without complications. She is breastfeeding her infant daughter without difficulty. It is now the second day after the cesarean section and the woman complains that she has some foul-smelling vaginal discharge. Temperature is 38.7C (101.6F), blood pressure is 110/70 mm Hg, pulse is 94/min, and respirations are 18/min. Physical examination is remarkable for significant uterine tenderness. There is no drainage from the incision and the sutures remain well aligned. Her WBC count is 14,000/cmm with 80% neutrophils and 7% band forms. Item 1 of 2 Which of the following is the most appropriate next step in management? A. Ceftriaxone and metronidazole B. Levofloxacin and metronidazole C. Clindamycin and gentamicin D. Ampicillin and gentamicin E. Vancomycin Explanation: Postpartum endometritis is a polymicrobial infection of the decidua (the pregnancy endometrium) characterized by fever, uterine tenderness, foul-smelling vaginal discharge, and leukocytosis. As the infection is often produced by both aerobes and anaerobes from the genital tract, any treatment regimen must include broad-spectrum antibiotics that also cover beta-lactamase producing

anaerobes. At this time, the gold standard of treatment for endometritis is clindamycin and gentamicin (Choice C). Ceftriaxone and metronidazole (Choice A) and levofloxacin and metronidazole (Choice B) both provide appropriate coverage for endometritis. However, since metronidazole is contraindicated in breastfeeding mothers, neither combination should be used. The combination of ampicillin and gentamicin (Choice D) does not adequately cover beta-lactamase producing anaerobes. Vancomycin (Choice E) is a narrow-spectrum antibiotic that is effective against gram-positive organisms, and is often employed in treating MRSA and MRSE. It would not be an appropriate treatment for endometritis. Educational Objective: Postpartum endometritis is a polymicrobial infection of the decidua (the pregnancy endometrium) characterized by fever, uterine tenderness, foul-smelling vaginal discharge, and leukocytosis. The gold standard of treatment for endometritis is clindamycin and gentamicin. Metronidazole is contraindicated in breastfeeding mothers. Item 2 of 2 What is the single most important risk factor in the development of postpartum endometritis? A. Cigarette smoking B. Low socioeconomic status C. Maternal diabetes mellitus D. Prolonged rupture of membranes E. Route of delivery Explanation: Endometritis occurs after 3% of vaginal births, but after 15-30% of cesarean deliveries (especially those performed after labor commences or after the rupture of membranes). Studies show that the most important risk factor in the development of endometritis is the route of delivery (Choice E). Several causes contribute to this finding, including contamination of the uterine cavity, prolonged rupture of membranes, and presence of sutures or other foreign objects. Cigarette smoking (Choice A), low socioeconomic status (Choice B), maternal diabetes mellitus (Choice C), and prolonged rupture of membranes (Choice D) are other proposed risk factors for endometritis. None is considered as significant as route of delivery, however. Educational Objective: The most important risk factor in the development of endometritis is the route of delivery. Endometritis occurs after 3% of vaginal births, but after 15-30% of cesarean deliveries. A 35-year-old Caucasian man is seen in the emergency department for the evaluation of fever, extreme weakness, generalized abdominal pain and frequent bloody stools. He has had ulcerative colitis for the past six years and has been in remission with a maintenance treatment of sulfasalazine. His flare up started four days ago, and rapidly progressed to multiple bloody bowel movements and severe abdominal pain. He has been a lifelong smoker, but he has recently been trying to quit. He has a toxic appearance. His temperature is 100F (37.8C), heart rate is 104/min, and blood pressure is 106/72 mmHg. His mucous membranes appear dry. His abdominal examination reveals the presence of hypoactive bowel sounds. There is generalized abdominal tenderness and distension without any rebound tenderness or guarding. The percussion note is tympanic. A flat plate, supine, abdominal xray reveals the presence of colonic dilatation with multiple air fluid levels. Which of the following is the most appropriate initial step in the management of this patient?

A. Measure thyroid stimulating antibodies B. Increase the dose of sulfasalazine C. Obtain a high resolution CT scan D. Start the patient on intravenous corticosteroids E. Route of delivery Explanation: The patient presents with the characteristic presentation of an acute severe or fulminant exacerbation of ulcerative colitis. He has a toxic appearance, and an acute history of fever, severe abdominal pain, anorexia, and multiple frequent bloody bowel movements. Although most of the flares of ulcerative colitis are idiopathic, smoking cessation is one of the few that has been associated with an exacerbation of ulcerative colitis. Patients with severe or fulminant ulcerative colitis should be immediately hospitalized. The mainstay of therapy includes intravenous hydration, nutritional support, bowel rest, pain control and intravenous corticosteroids. Since these patients are at high risk of developing toxic megacolon or bowel perforation, they should be closely followed for the worsening or development of any new symptoms. Patients with evidence of colon or intestinal dilatation by physical examination or imaging studies should have a nasogastric or rectal tube placed for gut decompression. (Choice A) All patients with severe ulcerative colitis should have a surgical consultation. Those with toxic megacolon refractory to medical treatment for 72 hours or those with signs and symptoms of bowel perforation should be referred for emergent surgical treatment (colectomy). (Choice B) Increasing the dose of sulfasalazine has no role in the management of severe ulcerative colitis. (Choice C) Further imaging with a CT scan is not indicated unless the patient?s clinical condition deteriorates. Educational Objective: Intravenous hydration, bowel rest and IV corticosteroids are the mainstay of therapy for patients with severe or fulminant ulcerative colitis. The following Vignette applies to the next 2 items A 25-year-old male presents to the office because of weight loss, palpitations, shortness of breath, sweating, heat intolerance, and tremors for the past two months. His symptoms are progressively getting worse. He is also complaining of prominence of his eyes, with some gritty sensation and watery eyes. He denies diplopia, decreased vision, or proximal weakness. His past medical history is unremarkable, and he is currently on no medications. He has a six-pack-year history of smoking, and continues to smoke. He drinks alcohol socially, but denies use of recreational drugs. His family history is positive for a mother with hypothyroidism. He denies any drug allergies. Examination reveals an anxious male with a thin build, and in no acute distress. His temperature is 36.7 C (98 F), blood pressure is 140/50 mmHg, pulse is 100/min, and respirations are 22/min. His eyes were measured by an exophthalmometer and showed 22 mm on the right side, and 18 mm on the left side (normal less than 20 mm). He has minimal lid lag and some conjunctival erythema. Extraocular movements and pupillary reflexes are intact. The thyroid is diffusely enlarged and moderately firm. Auscultation of the thyroid reveals a systolic bruit. Skin is warm and moist. Hand examination reveals tremors and onycholysis. Cardiac examination shows tachycardia and 2/6 ejection systolic murmur in the right sternal upper border. Nervous system examination shows brisk symmetrical reflexes, and is otherwise non-focal. Laboratory tests show normal CBC, basic chemistries, and urine analysis. Thyroid tests show: TSH: < 0.01 mU/mL (normal 0.35-5.0 mU/mL) Free T4: 3.5 ng/dL (normal 0.8-1.9 ng/dL) 60% is radioactive iodine uptake at 24 hours and scan shows diffuse uptake.

Item 1 of 2 What is the most appropriate course of action for his condition? A. Methimazole is preferred over propylthiouracil (PTU) B. Refer him for a subtotal thyroidectomy C. Radioactive iodine ablation alone D. Radioactive iodine ablation with concurrent prednisone administration E. Route of delivery Explanation: In the United States, the majority of patients with Graves' disease are treated with radioiodine ablation. Radioactive iodine ablation is contraindicated in large retrosternal goiters because postradioiodine inflammatory swelling of the thyroid gland may compromise the airways. Although mild thyroid ophthalmopathy is not a contraindication for radioactive iodine ablation alone (Choice C), it can worsen without concurrent use of glucocorticoids. Glucocorticoids are thus concurrently administered to prevent worsening of ophthalmopathy. In patients with moderate to severe ophthalmopathy, many endocrinologists defer radioactive iodine ablation. In some European countries and India, antithyroid drugs still remain the preferred modality. Antithyroid drugs are a reasonable alternative; however, they are associated with significant side effects. The dose of methimazole is lower (20-40 mg/d) in most patients with Graves' disease. Agranulocytosis in methimazole is dose related (i.e., the incidence of agranulocytosis increases with higher doses); whereas, agranulocytosis is not dose related in propylthiouracil (PTU). Other significant side effects are drug allergies, hepatic damage, and ANCA-associated vasculitis. Antithyroid drugs can be used in elderly patients with thyrotoxicosis, because radioiodine can temporarily worsen thyrotoxicosis. In younger patients, there is usually no need to "cool off" the thyroid gland. Antithyroid drugs should be discontinued a few days before radioiodine administration because these drugs cause a decrease in radioiodine uptake. (Choice B) Subtotal thyroidectomy is rarely indicated in the treatment of Graves' disease. (Choice A) Thyroid stimulating antibodies have no defined role in the routine management of Graves' disease patients. They might have some use in predicting neonatal hyperthyroidism for pregnant patients with Graves' disease. Educational Objective: In the United States, the majority of patients with Graves' disease are treated with radioiodine ablation. Understand that radioactive iodine treatment for Graves' disease is associated with worsening of ophthalmopathy, which can be prevented by concurrent administration of glucocorticoids. Item 2 of 2 All the treatment options were explained to the patient, after which, he was appropriately treated and sent home. Seven days after treatment, the patient was brought to the emergency department by the paramedics. The patient is now in an agitated state, and states that he has been feeling sick for the past two days. He is unable to tolerate food and has been persistently vomiting. He denies any significant alcohol intake in the last few days. He also denies cough, shortness of breath, chest pain, or abdominal pain. He is extremely anxious and restless. His temperature is 38.9 C (102 F), blood pressure is 120/76 mmHg, pulse is 140/min, and irregularly irregular. Eyes and thyroid examination are unchanged. He has minimal scleral icterus. He is oriented to place and person, but not to time. Otherwise, the nervous system examination is nonfocal. A clinical diagnosis of thyroid storm is made, and laboratory tests were ordered. Which of the following statements correctly describes the management of thyroid storm? A. Methimazole is preferred over propylthiouracil (PTU) B. Iodine should be administered first to inhibit the release of thyroid hormones C. Glucocorticoids are used because they inhibit the conversion of T4 to T3 D. Treatment should be started when results of the lab tests are available

E. Rapid cooling of body temperature can lead to hypotension Explanation: Radioactive iodine ablation can precipitate thyroid storm. Four drugs that are commonly used in thyroid storm are thionamide (PTU and methimazole), glucocorticoids, iodine, and non-cardioselective beta-blockers. PTU inhibits conversion of T4 (inactive) to T3 (active) in the circulation. Doses of antithyroid drugs are much higher in thyroid storm. Glucocorticoids can be used in the management of thyroid storm because they also inhibit the conversion of T4 to T3. Other drugs that prevent T4 to T3 conversion besides glucocorticoids and PTU are iopanoic acid (radioactive contrast agent), amiodarone, and non-cardioselective beta-blockers, like propranolol. (Choice D) The diagnosis of thyroid storm is made clinically. Treatment should be started as soon as possible, without waiting for the lab results. Delaying the treatment increases mortality in thyroid storm. (Choice A) Methimazole can be used in the treatment of thyroid storm, but is not preferred over PTU. (Choice B) Iodine should not be administered before antithyroid drugs because excess iodine can serve as a substrate for the formation of more thyroid hormones, and can worsen the clinical picture. Iodine is usually administered one hour after a dose of PTU or methimazole. (Choice E) Rapid cooling of a hyperthermic patient does not lead to hypotension in thyroid storm. Conversely, rapid warming can cause hypotension in myxedema coma. Educational Objective: The diagnosis of thyroid storm is made clinically. Glucocorticoids can be used in the management of thyroid storm because they also inhibit the conversion of T4 to T3. Iodine should not be administered before antithyroid drugs because excess iodine can serve as a substrate for the formation of more thyroid hormones, and can worsen the clinical picture. A 55-year-old male was involved in a motor vehicle accident. He was hypotensive at the scene and had to be resuscitated. He suffered multiple injuries to his lower extremities and required numerous surgeries and prolonged mechanical ventilation. He was started on a high concentration of enteral glucose feeds on a 24-hour protocol. After four weeks, he is still dependent on the mechanical ventilator. His chest x-ray remains clear. He has profound respiratory muscle weakness, and the MRI shows significant thinning of the diaphragm. The most likely cause of his muscle weakness is related to which of the following? A. Hypocalcemia B. Hypophosphatemia C. Zinc deficiency D. Hypernatremia E. Guillain Barre Syndrome Explanation: Hypophosphatemia is not very recognized in hospitalized patients. Continuous glucose infusions are the leading cause of hypophosphatemia in hospitalized patients. The patients are usually alcoholic or otherwise debilitated, and the nadir in serum phosphate appears in the first few days after admission. Hypophosphatemia can impair ATP generation (which is needed by the skeletal muscles to perform work), and muscle weakness can result. Respiratory muscle weakness has been reported; this is severe enough to prevent weaning from mechanical ventilation. In addition, phosphate deficiency reduces cardiac contractility, and chronic phosphate deficiency has been implicated as a cause of cardiomyopathy. Phosphate depletion is also associated with depletion of 2, 3 diphosphoglycerate,

and this causes a leftward shift of the oxyhemoglobin dissociation curve. As a result, the oxygen bound to hemoglobin is less readily released to the tissues. (Choice A) Hypocalcemia may be found in as many as two-thirds of patients in the ICU on admission. The clinical manifestations are those suggestive of neuromuscular excitability, such as hyperreflexia, tetany, and seizures. Cardiac effects include peripheral vasodilatation, hypotension, and a prolonged QT interval. The most common causes are sepsis and hypomagnesemia. Hypoparathyroidism is a leading cause of hypocalcemia in an outpatient who has had neck surgery. The treatment is calcium replacement. (Choice C) Zinc deficiency is common in the ICU because of the prevalence of predisposing factors, such as diarrhea, diuresis, malnutrition, chronic renal failure, burns, and prolonged illness. Zinc is involved in DNA synthesis and lymphocyte transformations. A deficiency leads to susceptibility to infections and a skin rash. Diagnosis requires a decrease in plasma zinc levels. Effective replacement on a daily basis prevents the complications. (Choice D) Severe hyponatremia is a serious condition; however, rapid correction can also produce a serious illness called central pontine myelinolysis, which is a demyelinating brainstem lesion that causes permanent neurological deficits and can be fatal. Current evidence suggests that this lesion is produced by rapid correction of hyponatremia to normal or supra normal levels. Hyponatremia does not lead to loss of muscle strength. (Choice E) Guillain Barr?yndrome is an idiopathic polyneuritis of unknown etiology. The peripheral neuropathy advances over a few days and initially involves the distal extremities. The ascending paralysis may be associated with paresthesias. Educational Objective: Hypophosphatemia is a major cause of respiratory muscle weakness and can lead to the failure of being able to wean a patient off the respirator. A previously healthy 68-year-old Caucasian woman comes to the emergency department because of palpitations, heat intolerance, and excessive sweating for the past month. She claims that her symptoms are progressively getting worse. She has no other past medical problems. She is a retired postal worker. She does not use tobacco, alcohol, or any other drugs. She does not take any prescription medications. Her temperature is 36.7 C (98 F), blood pressure is 150/80 mmHg, pulse is 112/min, and respirations are 16/min. Her physical examination shows a diffusely enlarged and nontender thyroid gland, clear lung fields, and irregularly irregular heart sounds. Lab studies are within normal limits, except for those shown below: Thyroid function tests: TSH: 0.2 ?U/mL Thyroxine (T4), serum Total: 20.0 ?g/dL Thyroxine (T4), serum Free: 4.7 ng/dL Her electrocardiogram shows atrial fibrillation with a rate of 110 /min. Which of the following is the most appropriate next step in the management of this patient? A. Cardioversion B. Radioactive iodine C. 2D-Echocardiogram D. Beta blocker and heparin E. Repeat thyroid function test to confirm results Explanation: The frequency of atrial fibrillation is increased in the presence of both overt and subclinical hyperthyroidism. The general principles of treatment of atrial fibrillation remain the same regardless

of its cause. The primary goals of treatment are to achieve rate/rhythm control and to prevent systemic embolization with anticoagulation. The patient in the above vignette has a heart rate of 110/min and a one-month history of hyperthyroid symptoms. One month is enough time for a blood clot to form in the left atrium; therefore, treatment with a beta-blocker for rate control and heparin for anticoagulation is the correct choice among all the above options. (Choice A) Electrical or pharmacological cardioversion to convert atrial fibrillation into sinus rhythm is not indicated at this time. The underlying cause of this patient's atrial fibrillation is hyperthyroidism, which is treatable. Furthermore, cardioversion without prior adequate anticoagulation is associated with a risk of systemic embolization and stroke. (Choice B) Radioactive iodine is a treatment option for hyperthyroidism, but this should not be used acutely in this setting. The cause of hyperthyroidism should be further investigated before prescribing any definite therapy. (Choice C) Transesophageal echocardiogram, not a 2D-echocardiogram, may have a role in further management of this patient. Rate control with beta-blockers and anticoagulation still remain the primary goal of therapy at this time. (Choice E) There is no need to repeat the thyroid function tests for the confirmation of the diagnosis of hyperthyroidism. Her clinical presentation is quite consistent with the disease. Educational Objective: Rate control with either a beta-blocker or calcium channel blocker and anticoagulation to prevent systemic embolization are the main goals of therapy in chronic atrial fibrillation. A 46-year-old Caucasian woman comes to your office for a routine follow up visit. She was previously diagnosed with depression and obesity. She has smoked one pack of cigarettes daily for the past 30 years. She drinks one ounce of alcohol daily, and takes fluoxetine. Examination shows no abnormalities. The patient tells you that she read an article in the Internet, which discussed a study involving a new Chinese herb that was very effective for depression. According to the article, eight patients took the herb for six weeks, and five were cured. She wants to try it, but she wants to hear your advice first. Which of the following is the most appropriate response to the patients inquiry? A. Do not use it. This herb does not work B. Do not use it. The results may be false C. Do not use it. There is not enough evidence D. You must use only the medications I prescribed you. E. Use it, but at your own risk. I do not recommend it Explanation: The physician is being asked about a new herbal medication, of which he has no information. Data from only a single study is available. Five of eight patients who agreed to take the treatment responded "well" to treatment. There are many aspects to this study which makes its data insufficient to be considered as scientific proof or evidence. The population used was small, so the possibilities of obtaining an effect simply by chance (random error) are high. Other information about the study is also lacking. How were the patients diagnosed? Were their cases of depression equally severe? What were the adverse effects of the herbal medication? What other medications were the patients taking concurrently? Were the patients randomized to therapy? Were the patients and the physicians blinded to therapy? How high was the placebo effect in these cases? Such questions should first be answered adequately before the physician accepts the data from the study as accurate and reliable. (Choice D) The end of paternalism in medicine has led to a new era, in which the physician has to honor his patient?s right to autonomy. A doctor cannot simply tell his patient to use only his

prescriptions. He has to state the reasons why he thinks the new drug or herbal medicine should not be used. (Choice E) It is the physician?s duty to protect the patient from a possibility of drug or herbal toxicity. "Primun non nocere" (first, to avoid harm), as Galen said. Patients must be counseled about the perils of a new, untested drug, and not be left on their own. (Choices A and B) There is not enough information from this case to state that the new medication does not work or that the study has false results. Educational Objective: Physicians must be cautious when questioned about new therapies that have not been adequately tested. They must counsel their patients to wait until further and more solid information is available, to avoid undesirable side effects. Their approach must avoid paternalism and unjustified prejudice against or in favor of particular therapies. A 45-year-old male was involved in an argument over a drug deal. He ended up being shot in the head and abdomen. He was immediately intubated and taken to the operating room for an exploratory laparotomy. Postoperatively, he remained on the ventilator and was started on total parenteral nutrition. Seven days later, he developed a sudden onset of glucose intolerance. What does this most likely indicate? A. Liver dysfunction B. Sepsis C. Hypophosphatemia D. Bowel atrophy E. Carbon dioxide retention Explanation: Numerous complications can occur from intravenous nutrition in critically ill patients. The most dangerous metabolic complication, hyperglycemia, is most commonly caused by too rapid initiation of the infusion. This complication is best prevented by initiation of the infusion at 40-60 ml per hour, and slowly increasing the rate at 20 cc/hour every 24 hours, while monitoring the patient's glucose levels. Patients with normal glucose tolerance may manifest glycosuria for the first 48 hours of parenteral nutrition. Before initiating insulin, one must verify that the glucose level is high, and that glycosuria is not secondary to a reduced renal threshold for glucose. The most common cause of sudden hyperglycemia is sepsis, and hyperglycemia may antedate other signs of sepsis within 24 hours. The sudden appearance of hyperglycemia should initiate a thorough search for the source of infection. The infection may be due to line sepsis, pneumonia, wound infection, or another process. Blood cultures and a thorough examination of the patient are required. (Choice A) A fatty liver can be seen when glucose infusions exceed the daily caloric requirements. This is because of the production of fatty acids from excess glucose and the impaired ability to mobilize the fat for energy needs. Fat accumulation eventually leads to abnormal liver enzyme elevations in the serum, particularly alkaline phosphatase. The long-term consequences of fatty liver are not clear, but at present, there seems to be no serious sequelae. (Choice C) Hypophosphatemia is reported to occur in 30% of patients receiving intravenous nutrition. The mechanism is enhanced phosphate uptake into cells associated with enhanced glucose uptake. The consequences of hypophosphatemia include respiratory weakness, hemolysis, and impaired oxygen release from hemoglobin. These may be particularly prominent when serum phosphate falls below 1.0 mg/dL. Phosphate supplementation in the diet will prevent these complications. (Choice D) Bowel rest leads to degenerative changes in the small bowel mucosa only after a few days, and intravenous nutrition does not prevent these atrophic changes. Bowel atrophy during intravenous nutrition is now considered a major risk, because disruption of the normal mucosal barrier might allow intestinal microbes to enter the systemic circulation.

(Choice E) Carbon dioxide retention can develop when excess glucose is given to patients with severe lung disease. Glucose metabolism produces a larger quantity of CO2 for each liter of O2 consumed than the other two nutrient substrates. When the ability to eliminate CO2 via alveolar ventilation is impaired, this enhanced CO2 production of carbon produces hypercapnia, and the impaired ability to wean from mechanical ventilation. Educational Objective: The sudden onset of hyperglycemia in a patient receiving total parenteral nutrition should lead to a suspicion of sepsis. A 41-year-old Caucasian man comes to the emergency department after suffering from a wasp sting on his right arm. He complains of generalized itching and mild local pain. He has no other medical problems. His social history is not significant. His blood pressure is 120/80 mmHg, pulse is 86/min, temperature is 36.7C (98F) and respirations are 14/min. His oxygen saturation is 96% on room air. Examination reveals a small, 2cm x 2 cm, erythematous, elevated, warm and mildly tender area on the right arm. Multiple wheals are visible on his trunk and extremities. His lungs are clear on auscultation. His heart sounds and the rest of the exam are normal. Which of the following is the most appropriate next step in the management of this patient? A. Oral diphenhydramine B. Subcutaneous epinephrine C. Intravenous methylprednisolone D. Intravenous epinephrine E. Topical triamcinolone Explanation: Anaphylaxis is a serious systemic reaction to an exogenous agent. It occurs in approximately one in every 200 cases of insect stings. Although there may be few exceptions, it is generally an immediate, IgE-mediated hypersensitivity reaction. Mild systemic reactions may manifest as skin involvement only (such as in this patient), and such cases are managed with subcutaneous epinephrine. On the other hand, severe anaphylaxis is characterized by associated respiratory (i.e., bronchospasm) and cardiovascular (i.e., hypotension,) manifestations; this condition requires the intravenous administration of epinephrine. All patients should receive H1/H2 histamine blockers. (Choice A) Antihistamines such as oral diphenhydramine, chlorpheniramine or cetirizine are effective in relieving pruritus when there is only a local lesion; however, when used alone, these are not effective in patients with systemic anaphylaxis. (Choice C) Corticosteroids do not help in the acute phase; however, these can prevent the development of late phase reactions or "protracted anaphylaxis." (Choice D) Intravenous epinephrine must be used for severe cases. Patients in such conditions present with angioedema, shock, severe bronchospasm or upper airway obstruction. (Choice E) Topical corticosteroids are only useful for the management of local allergic reactions. Educational Objective: Insect stings can lead to anaphylaxis or local reactions only. Single cutaneous lesions respond to histamine-blockers and topical steroids. Mild anaphylaxis is characterized by generalized rash and pruritus, and is managed with subcutaneous epinephrine and systemic antihistamines. Mild to moderate bronchospasm should receive the same management. Severe anaphylaxis must be readily treated with intravenous epinephrine. A 57-year-old postmenopausal female with a history of hypertension, obesity, and type II diabetes, presents with pain in her right first metatarsophalangeal (MTP) joint for the past four hours. She is currently on atenolol, hydrochlorothiazide, glyburide and metformin. Examination reveals a swollen,

inflamed, and extremely tender right first MTP joint. Acute monoarticular gout is suspected clinically. Which of the following statements is true regarding acute gouty arthritis? A. Allopurinol should be started concomitantly with the other agents used for treating acute gouty arthritis. B. Tight blood sugar control is likely to prevent recurrent gouty arthritis. C. Weight loss and discontinuation of hydrochlorothiazide is likely to prevent recurrent episodes of acute gouty arthritis. D. An increase in the serum uric acid level is diagnostic of acute gouty arthritis. E. Joint aspiration is contraindicated in acute inflammatory arthritis. Explanation: Acute gout usually presents as monoarticular arthritis involving the first metatarsophalangeal joint. It is common in persons who have hypertension, obesity, and insulin resistance. Hydrochlorothiazide use increases the risk of acute gouty arthritis because the drug can reduce uric acid secretion into the urine and increase serum uric acid levels. Weight loss and discontinuation of hydrochlorothiazide will most likely reduce insulin resistance and increase the urinary excretion of uric acid, respectively. Reduction of purine-rich foods (meats and fish) in the diet will also cause a significant reduction of serum uric acid. These activities may help prevent or lessen recurrent attacks of acute arthritis. (Choice A) Allopurinol is a xanthine oxidase inhibitor that reduces the production of uric acid. It is used as a prophylactic agent to prevent recurrent gouty arthritis. It is never started during an acute episode. (Choice B) Although gout is more common in patients who have insulin resistance, there is no clear relationship between blood sugar and uric acid levels. Tight blood sugar control has never been shown in systemic studies to prevent episodes of recurrent gouty arthritis. (Choice D) Serum uric acid can be normal during an episode of acute gouty arthritis, and is not used in diagnosing acute gout. (Choice E) There is no contraindication for joint aspiration in acute inflammatory arthritis. In fact, the cornerstone for the diagnosis of acute gout is joint aspiration and demonstration of negatively birefringent needle-shaped crystals. Educational Objective: Acute gouty arthritis usually presents as monoarticular arthritis of the first metatarsophalangeal joint. The cornerstone for its diagnosis is joint aspiration and demonstration of negatively birefringent needle-shaped crystals. Prophylactic therapy should not be started during an acute episode. Weight loss, a purine-restricted diet, and the discontinuation of uricosuric agents (e.g., hydrochlorothiazide) may lead to a decrease in serum uric acid and prevent further attacks. A 60-year-old Caucasian male was admitted to the hospital. He complained of difficulty swallowing which rapidly got worse within the last two weeks. The patient was in his usual state of health two weeks ago, although he had noticed a swelling in front of his neck that started growing rapidly for the past month. He has been unable to eat solid food for the past day, and has lost 3 lbs. for the past week. He also complained that his voice was getting hoarse. He denied other accompanying symtoms, such as: trouble breathing, cough, chest pain, pain in the neck swelling, headache, decreased vision, fever, abdominal pain, skin rashes, or urinary complaints. His past medical history is significant for mild hypertension treated with hydrochlorothiazide, as well as a small goiter for the past 10 years. He states that his blood tests for thyroid hormone levels were normal, and he never took thyroid hormone preparations in the past. His family history is positive for a mother and several other family members with hypothyroidism. He has a smoking history of 25-pack-years, but quit 15 years ago. He does not drink alcohol. Physical examination revealed a large, irregular, hard, fixed, nontender swelling in the thyroid area. The lower margin of this swelling could not be palpated. He developed facial plethora and venous engorgement of the neck veins after raising his arms above the shoulder for 20 seconds. CBC and basic chemistries were normal. TSH is 66.0 mU/ml (normal 0.35-5.0 mU/ml) and free T4 0.2 mg/dl (normal 0.8 to 1.8 mg/dl). Antithyroid peroxidase antibody was detected in very high titers. What is the most likely diagnosis in this patient?

A. Follicular thyroid cancer B. Thyroid lymphoma C. Anaplastic carcinoma of thyroid D. Papillary thyroid cancer E. Medullary thyroid cancer Explanation: The patient most likely has a thyroid lymphoma, which is a rare thyroid malignancy occurring in patients with Hashimoto?s thyroiditis. Patients generally present with a rapidly enlarging goiter with compressive symptoms. The patient may or may not give a history of preexisting goiter. The thyroid gland is usually very firm or hard, and retrosternal extension is common. Thyroid lymphoma is not a common thyroid malignancy, and should be strongly suspected in patients with a rapidly enlarging firm goiter. Occasionally, patients with thyroid lymphoma present with a rapidly enlarging thyroid nodule. Thyroid lymphoma is managed differently from other thyroid malignancies. The majority of patients, even those with compressive symptoms, are treated with radiation and chemotherapy. The response is generally good. (Choice C) Anaplastic carcinoma is a possible differential diagnosis, but in the setting of Hashimoto?s thyroiditis, thyroid lymphoma is more likely. (Choices A, D, and E) Differentiated thyroid cancers of follicular cell origin (follicular and papillary) and medullary thyroid cancer are usually slow growing. Educational Objective: Thyroid lymphoma should be suspected if a patient with longstanding Hashimoto?s thyroiditis presents with a rapidly enlarging, firm thyroid mass. A 72-year-old Caucasian female is brought to the emergency room because of a sudden onset of confusion and clumsiness. Her daughter tells you that she was watching television when the symptoms started. She has a past history of hypertension and atrial fibrillation. On physical examination, there is no evidence of any focal weakness of her extremities. Her speech is clear, and she is able to repeat the sentences after you. She is able to engage in an intelligent conversation, and provide her own past medical history. When you ask her to copy a picture of a matchstick, she is unable to do it. Which of the lobes of the brain is most likely affected in this patient? A. The probability of having the disease is influenced by the gender of the child B. Dominant parietal lobe C. Nondominant temporal lobe D. Dominant temporal lobe E. Medullary thyroid cancer Explanation: Almost 90% of left-handed patients and 60% of right-handed patients have a left hemispheric dominance for speech and language functions. The patient in the above scenario appears to be suffering from construction apraxia. This is seen with the involvement of parietal lobes, and is much more pronounced with lesions of the right parietal hemisphere (nondominant parietal lobe). Patients have marked difficulty in copying simple line drawings. They may also experience difficulty in wearing clothes, and appear to struggle while attempting to get into a coat or pants. This is known as dressing apraxia, which is seen with bilateral or right-sided (nondominant) parietal lobe lesions. Confusion is also seen in patients with lesions of the nondominant parietal lobe. (Choice B) Damage to the dominant parietal lobe, especially the inferior parietal lobe, presents as Gerstmann syndrome. The patients have difficulty in performing simple arithmetic tasks (acalculia), inability to name individual fingers (finger agnosia), impaired writing (agraphia) and right/left confusion (difficulty in identifying or distinguishing the right or left side of the body).

(Choice C) Lesions of the nondominant temporal lobe can cause visual disorders (homonymous upper quadrantanopia) and impaired perception of complex sounds (auditory agnosia). (Choice D) Dominant temporal lobe lesions, in addition to having homonymous upper quadrantanopia, almost always involve the language functions, and lead to aphasia. Wernicke?s aphasia is usually seen in patients with dominant temporal lobe lesions. It is characterized by the impairment in comprehension of spoken or written language. Patients have difficulty in expressing their thoughts in a meaningful manner. Apraxia is not seen in temporal lobe lesions. Educational Objective: Nondominant parietal lobe lesions present with constructional and dressing apraxia. typically seen in dominant temporal lobe lesions. Aphasia is

A 22-year-old Caucasian female presents to your office at 8 weeks? gestation for routine prenatal counseling. Her husband has hemophilia A. Her family history is insignificant. She is concerned about the possibility of her child having hemophilia. Which of the following is the best response in this case? A. The probability of having the disease is influenced by the gender of the child B. The probability of hemophilia for the child is close to 50% C. The probability of hemophilia for the child is close to 25% D. The probability of hemophilia for the child is close to that of the general population E. The risk is difficult to estimate because the inheritance is polygenetic Explanation: Hemophilia A is an X-linked recessive disorder that leads to a deficit in coagulation factor VIII. It is a monogenetic disease that affects mainly males; heterozygous females are silent carriers. In this case, the father can transmit the affected X chromosome only to a female child, not a male child. A female child will become a silent carrier. For these reasons, the probability of their child having the disease is close to that of the general population (i.e., none of their children will be affected by the disease). (Choice E) Hemophila A is a monogenetic (not polygenetic) disease. (Choice A) If the child was female, she would receive a second ?healthy? X chromosome from the mother, and she would be a silent carrier of the disease. (Choice B) For autosomal-dominant diseases, the probability of the disease for the child is close to 50% if one of the parents is affected. (Choice C) For autosomal recessive diseases, the probability of the disease for the child is close to 25% if both parents are silent carriers. Educational Objective: If the father is affected by an X-linked recessive disease and the mother is not a carrier, none of their children will be affected by the disease, although all their female children will be silent carriers. You are the resident on call, and the nurse calls you in the middle of the night to inform you about a patient who just had a generalized seizure. The patient is a 66-year-old Caucasian male who had a right hemicolectomy one day prior for colon carcinoma. Postoperatively, the patient had a hypotensive episode, which responded to aggressive fluid resuscitation with 0.45% normal saline. The patient was in significant pain postoperatively and was receiving high doses of morphine for the pain. On physical examination, the patient is extremely drowsy and lethargic. His temperature is 36.7 C (98 F), blood pressure is 110/60 mmHg, pulse is 68/min, and respirations are 12/min. His pulse oximetry reading is 96% at room air. You order a stat electrolyte panel, and the results are as follows: Serum Na: 114 mEq/L Serum K: 4.2 mEq/L Chloride: 90 mEq/L Bicarbonate: 22 mEq/L BUN: 24 mg/dL Serum Creatinine: 1.2 mg/dL Calcium: 9.4 mg/dL Blood Glucose: 120 mg/dL What is the most appropriate next step in the management of the patient? A. 3% normal saline B. 0.9% normal saline C. 0.4 mg of IV naloxone

D. Ringer lactate E. Water restriction to less than 1 liter/day Explanation: Hyponatremia is a relative excess of water in relation to sodium in the body. It is caused by excessive free water intake (primary polydipsia), endocrine disorders (e.g. , adrenal insufficiency, hypothyroidism) and impaired water excretion from advanced renal failure and excessive antidiuretic hormone (ADH) release. Excessive ADH release can be due to a decreased effective circulating volume (volume depletion, CHF, cirrhosis, and overuse of thiazide diuretics) and syndrome of inappropriate ADH secretion (SIADH). SIADH is seen in a variety of CNS disorders, tumors (especially small cell lung cancer), drugs (e.g., carbamazepine, cyclophosphamide, and selective serotonin reuptake inhibitors), pulmonary diseases, HIV infection, as well as in postoperative patients. Major abdominal or thoracic surgery is commonly associated with hypersecretion of ADH, which is probably mediated by pain afferents. Other factors that may contribute to the development of hyponatremia in the postoperative patients are nausea, hypotension, and excessive use of hypotonic intravenous fluids. The symptoms are primarily neurologic and are due to water movement into the brain cells, thereby causing cerebral edema. It occurs due to rapid reduction in the plasma sodium concentration, especially in postoperative patients receiving large quantities of hypotonic fluids. Nausea and malaise are the earliest symptoms, followed by headache, lethargy, obtundation, seizures, coma, and respiratory arrest if the sodium level falls below 115 to 120 mEq/L. I The treatment of hyponatremia depends on the cause, the plasma sodium concentration, and the severity of symptoms. In patients with symptomatic or severe hyponatremia (sodium level below 115 mEq/L), an aggressive initial correction at the rate of 1.5 to 2 mEq/L/hour for the first three to four hours is indicated. Hypertonic saline should be used initially in this setting for rapid correction of hyponatremia. The plasma sodium levels should be monitored frequently to avoid raising sodium levels more than 12 mEq/L in the first 24 hours. (Choices E and B) Most patients with hyponatremia are usually asymptomatic. These patients are best managed by fluid restriction or the use of isotonic saline. (Choice C) The patient does not have any signs of morphine overdose; therefore, the use of naloxone is not indicated in this setting. (Choice D) Ringer lactate should not be used in the initial management of symptomatic hyponatremia. Its role in the treatment of hyponatremia is not well defined. Educational Objective: Hyponatremia causing seizures or other severe neurologic abnormalities should be initially treated aggressively with hypertonic saline. A 52-year-old Hispanic woman brings her father to the emergency department for an evaluation. She reports that her father fell and hit his head while trying to get to the bathroom. He has a past medical history of cerebrovascular accident with residual weakness on the left side, atrial fibrillation, benign prostrate hypertrophy, hypertension, and mild cognitive impairment. He lives with her daughter who is his primary caregiver. Physical examination reveals a disheveled man with a depressed affect. You notice a bruise over his left forehead. You also notice some old bruise marks over his arms and back. On direct questioning, he responds only in very brief sentences and avoids eye contact. His daughter keeps interrupting the interview as well. You suspect elder abuse in this patient. Which of the following factor is most likely to be associated with elder abuse? A. Male gender B. Physical and mental impairment

C. Financial status D. Living in a inner city area E. Country of origin Explanation: Elder abuse is a very common problem. The different kinds of elder abuse are physical abuse, sexual abuse, financial abuse, psychological abuse, neglect, and abandonment. Physicians should ask direct and specific questions about elder abuse to all older patients, and report any kind of suspected abuse. It is important for a physician to recognize the risk factors associated with elder abuse because some of these can suggest its occurrence. Different sets of risk factors have been identified for the abused elderly and the abusers. The risk factors associated with the abused are female gender, physical and mental impairment from chronic medical conditions, and old age (especially 80 and above). On the other hand, the risk factors identified in abusers include young age, relationship with the victim (spouse or children), substance abuse, mental illness, dependence on the victim, and previous history of violence in the abuser. (Choices C, D and E) The patient?s financial status, country of origin, and living in an inner city area have not been associated with a significant risk of elder abuse. Educational Objective: Chronic physical and mental impairment is one of the most common risk factors for elder abuse, along with old age, and female gender. The following vignette applies to the next 3 items A full term neonate was born vaginally to a 30-year-old Hispanic woman a few hours ago. The pregnancy was uneventful. At birth, you noted that the infant has ambiguous genitalia. The patient's family emigrated from Mexico to the USA 2 years ago. The patient had a brother who died at 1 month of age due to dehydration four years ago. The patient also has a 10-year-old sister who is perfectly normal. A diagnosis of congenital adrenal hyperplasia is suspected. Item 1 of 3 A deficiency of which of the following is the most common cause of congenital adrenal hyperplasia? A. 11-hydroxylase B. 3-beta-hydroxysteroid dehydrogenase C. 21-hydroxylase D. 17 alpha-hydroxyprogesterone E. 17 alpha-hydroxypregnenolone Explanation: Congenital adrenal hyperplasia (CAH) is a group of disorders characterized by a deficiency in one of the enzymes involved in steroid synthesis. In 90% of cases, the deficiency concerns 21-hydroxylase, which is necessary in mineralocorticoid and glucocorticoid synthesis. In addition to a low production of mineralocorticoids and glucocorticoids, the deficiency of 21-hydroxylase results in an excess of its substrates. These excess substrates are shunted towards androgen synthesis, resulting in ambiguous genitalia and virilization in females, and precocious puberty in males. ACTH levels are also evidently elevated, and lead to hyperplasia of the adrenal glands, further contributing to the increased production of androgens. Salt wasting syndrome occurs in severe deficiencies. It presents in the first 2-4 weeks of life with emesis, dehydration, and shock. Laboratory work-up reveals hyponatremia and hyperkalemia (from lack of aldosterone), and hypoglycemia (from lack of cortisol). (Choice A) 11-hydroxylase deficiency is a far less frequent cause of CAH than 21-hydroxylase deficiency. It is characterized by: (1) decreased aldosterone and cortisol, and (2) increased androgens

and deoxycorticosterone. Patients with this condition will present with hypertension, hypernatremia, and hypokalemia. (Choice B) 3-beta-hydroxysteroid dehydrogenase is a rare cause of CAH. It involves: (1) a decrease in testosterone, mineralocorticoids and glucocorticoids, and (2) an increase in DHEA-S. Consequently, males fail to acquire normal external genitalia because of a lack of testosterone, and females are slightly masculinized because of DHEA-S excess. (Choices D and E) 17 alpha-hydroxyprogesterone and 17 alpha-hydroxypregnenolone are substrates of 21-hydroxylase and 3-beta-hydroxysteroid dehydrogenase, respectively. Educational Objective: The most common cause of CAH is 21-hydroxylase enzyme deficiency. Item 2 of 3 The diagnosis of congenital adrenal hyperplasia is confirmed by an elevation of which of the following? A. 17-hydroxylase B. Dehydroepiandrosterone C. Androstenedione D. 17 alpha-hydroxyprogesterone E. 17 alpha-hydroxypregnenolone Explanation: Diagnosis confirmation of 21-hydroxylase deficiency is carried out by documenting the elevation of 17 alpha-hydroxyprogesterone, a substrate of 21-hydroxylase. Other biologic disturbances occuring in 21-hydroxylase deficiency include elevated corticotropin and renin levels, and a serum aldosterone level that is inappropriately low for the renin level. The treatment principle for 21-hydroxylase deficiency is based on supplementation of deficient hormones (mineralocorticoids and glucocorticoids). This supplementation will decrease corticotropin (ACTH) production, and consequently correct androgen levels. (Choice A) 17-hydroxylase is involved in androgen genesis, and is not related to 21-hydroxylase deficiency. (Choice B and C) Although androgen and corticotropin levels are also elevated, they are not utilized to diagnose 21-hydroxylase deficiency; however, they are useful for monitoring the patient's response during treatment. (Choice E) 17 hydroxypregnenolone is elevated in 3-beta-hydroxysteroid dehydrogenase. Educational Objective: A diagnosis of 21-hydroxylase deficiency is confirmed by documenting elevated levels of 17 alphahydroxyprogesterone. Item 3 of 3 A key difference between 21-hydroxylase deficiency and 11-hydroxylase deficiency is that, unlike 21hydroxylase deficiency, 11-hydroxylase deficiency is associated with which of the following? A. Hypernatremia B. Hyponatremia C. Hypotension D. Cortisol overproduction E. No ambiguity of genitalia Explanation: A deficiency in 11-hydroxylase results in an excess in its substrate, 11-deoxycorticosterone. This substrate has mineralocorticoid activity. Patients with this condition present with hypertension, hypernatremia, and hypokalemia.

(Choice D) Both enzyme deficiencies have decreased cortisol production. (Choice E) Ambiguous genitalia or virilization may be present in both conditions since androgens are produced in excess by the same mechanism. Educational Objective: 11-hydroxylase deficiency is a far less common form of CAH. It is distinguished from 21-hydroxylase deficiency by the presence of hypertension, hypernatremia, and hypokalemia.

A 66-year-old Caucasian female is admitted to the hospital after she fell at home and broke her right hip. Her medical problems include a history of hypertension, hypothyroidism and osteoporosis. Her daily medications are daily aspirin, hydrochlorothiazide, amlodipine, levothyroxine, calcium and vitamin D supplements. The next day, she undergoes right total hip replacement without any complications. On her fourth post-operative day, she develops a sudden onset of right-sided chest pain and shortness of breath. Before the doctor could take the appropriate measures to assess and treat her, she goes into cardiopulmonary arrest and dies. Her family agrees to have an autopsy done to ascertain her cause of death. What will the autopsy report most likely show in this patient? A. Normal findings B. Massive infarction of the lung C. Wedge shaped infarct of the lung D. Hampton?s hump E. Left atrial thrombus Explanation: This patient's cardiopulmonary arrest and death was most likely due to a massive pulmonary embolism (PE). This diagnosis is based on her clinical presentation (i.e., acute onset of dyspnea and right-sided chest pain after a recent high-risk surgical procedure). Postoperative patients, especially those who have undergone lower extremity surgery (knee and hip replacement) and other intraabdominal and pelvic procedures, are at an extremely high risk of developing deep venous thrombosis and subsequent pulmonary embolism. A massive PE can cause an acute rise in pulmonary vascular resistance and right ventricular failure, which can lead to death within one to two hours of the event. Massive PE is usually a clinical diagnosis since patients are usually clinically unstable to undergo any routine diagnostic evaluation. Unfortunately, the diagnosis is more frequently confirmed with autopsy findings, which will demonstrate an infarction of the whole lung or part of the involved lung. (Choice A) The autopsy report is extremely unlikely to be normal in this patient. (Choices C and D) Wedge-shaped infarcts (Hampton?s hump) are small, and these are commonly seen in patients with sub-segmental or peripheral pulmonary embolism (PE), which is not usually fatal. (Choice E) Left atrial thrombus can cause systemic embolization; however, it does not produce an acute onset of chest pain with dyspnea and/or hypoxemia due to PE. Educational Objective: Untreated massive pulmonary embolism leads to infarction of the lung parenchyma and can be rapidly fatal. A 70-year-old white male with a past medical history of coronary artery disease, myocardial infarction, congestive heart failure, hypertension, and type II diabetes comes to the emergency department after passing out early this morning. He was sitting in his chair, watching TV, when he had a brief episode of loss of consciousness, followed by complete recovery. He denies any headaches, nausea, pallor, or diaphoresis. His friend who was with him at that time said that he did not observe any limb movements during the episode. There was no soiling of his clothes by urine or feces. His medications include furosemide, potassium, metformin, glipizide, atenolol, enalapril, simvastatin and aspirin. He lives alone in his house, and is a non-smoker and non-alcoholic. His temperature is 36.7C(98F), blood pressure is 130/86 mm Hg, pulse is 70/min, and respirations are 16/min. Neurological exam is nonfocal. There is no postural drop of blood pressure. Chest x-ray, EKG, CBC, serum chemistries and set of cardiac enzymes are within normal limits. Which of the following is the most appropriate next step in the management of this patient? A. Admit the patient and do continuous EKG monitoring for 24 hours. B. Admit the patient and do coronary angiography. C. Send the patient home on Holter monitor.

D. Do CT scan of head without contrast. E. Admit the patient and do EEG.

Explanation: Based on the patient's past medical history and presentation, cardiac arrhythmia is the most likely cause of his syncope. Inpatient continuous cardiac monitoring is the most appropriate next step in the management of this patient. An acute coronary event should be ruled out with serial cardiac enzymes, and an echocardiogram is needed to evaluate left ventricular function. Coronary revascularization is needed if evidence of coronary ischemia exists. (Choice B) This patient has a normal EKG, normal cardiac enzymes, and no chest pain. Immediate coronary angiography is not needed. (Choice C) The patient lives by himself. Sending him home without identifying the cause of his syncope is inappropriate because he will then be at high risk of falling and injuring himself. (Choice D) The patient does not give any history of headache or focal neurological signs; therefore, a CT scan of the head is also inappropriate. (Choice E) The patient does not have any abnormal movements, urinary or fecal incontinence or postictal state; therefore, the possibility that seizures could have caused the syncope is highly unlikely. Educational Objective: Patients with multiple risk factors for developing cardiac arrhythmias need proper evaluation for possible implantable cardiac defibrillator placement to prevent sudden cardiac death. A 65-year-old Caucasian female with a past medical history of essential hypertension, type-2 diabetes, peripheral vascular disease, and dyslipidemia presents with epigastric pain for the past two months. The pain is described as "crampy, dull, and sometimes radiates to the back." The pain is worse after eating. Four weeks of over-the-counter omeprazole didn?t relieve her pain. She also complains of a 10 lb weight loss. Her bowel habits are normal. Her medications include glipizide, metformin, simvastatin, pentoxifylline and verapamil. Her temperature is 36.7C (98F), blood pressure is 170/86 mm Hg, pulse is 90/min, and respirations are 16/min. Serum lipase is normal. Ultrasound of the abdomen shows a normal gall bladder without any stones. Fecal occult blood testing is negative. An upper endoscopy does not show any abnormalities. The CT scan of the abdomen is normal. A colonoscopy done eight years ago for routine screening was also normal. Which of the following is the most appropriate next step in the management of this patient? A. Splanchnic angiography B. Colonoscopy C. Barium enema D. Mesenteric duplex ultrasonography E. Endoscopic retrograde cholangiopancreatography Explanation: This patient is most likely suffering from chronic mesenteric ischemia. Positive clues in her history include severe peripheral vascular disease, postprandial pain, and a history of weight loss. Mesenteric duplex ultrasonography is an appropriate screening test for chronic mesenteric ischemia, especially since it has a high negative predictive value. (Choice A) Splanchnic angiography is the gold standard for the diagnosis of chronic mesenteric ischemia. It is performed after the initial screening test shows evidence of chronic mesenteric ischemia. (Choices B and C) Barium enema and colonoscopy have no role in the diagnosis of chronic mesenteric ischemia.

Educational Objective: Mesenteric duplex ultrasonography is an appropriate screening test for chronic mesenteric ischemia. It has a high negative predictive value.

A 27-year-old Caucasian male with severe dyspnea, fever and dry cough is admitted to the hospital. He is a known HIV-positive IV drug abuser, and his most recent CD4 count was 50/cmm. He is allergic to penicillin and sulfonamides. His oxygen saturation level is 85%. His temperature is 38.3C(101F), blood pressure is 120/76 mm Hg, pulse is 90/min, and respirations are 24/min. Bilateral rales are heard on auscultation. His laboratory studies at the time of admission are: CBC Hb: 11.5g/dL MCV: 88 fl Platelet count: 120,000/cmm Leukocyte count: 7,500/cmm Segmented neutrophils: 68% Bands: 1% Eosinophils: 1% Lymphocytes: 24% Monocytes: 6% Serum Serum Na: 137 mEq/L Serum K: 3.7 mEq/L Chloride: 110 mEq/L Bicarbonate: 20 mEq/L BUN: 16 mg/dL Serum creatinine: 1.0 mg/dL Calcium: 9.0 mg/dL Blood glucose: 118 mg/dL A chest x-ray is performed. Sputum specimens are obtained (via sputum induction) for immunofluorescent staining, after which the patient is started on IV pentamidine therapy. Several hours later, he develops an episode of generalized tonic-clonic seizure. Which of the following is the best next step in the management of this patient? A. Loading dose of phenytoin B. Electroencephalogram C. Finger stick blood test D. Head CT scan E. Lumbar puncture Explanation: IV pentamidine therapy is associated with a large number of adverse effects; their incidence may reach 70% of treated patients. Metabolic and electrolyte disturbances are typical and may include hyperkalemia, hypokalemia, hypocalcemia, hypoglycemia, and hyperglycemia. Hypoglycemia should be promptly detected/excluded by performing a finger stick blood glucose level test in this patient. (Choice A) Once hypo- and hyperglycemia have been ruled out, other metabolic abnormalities should be sought and subsequently treated . (Choices B, D and E) There is no need to proceed immediately with EEG, head CT, or lumbar puncture before the patient is stabilized and the metabolic disturbances are corrected.

Educational Objective: IV pentamidine therapy is associated with a number of metabolic and electrolyte disturbances including hyperkalemia, hypokalemia, hypocalcemia, hypoglycemia, and hyperglycemia.

A 24-year-old baseball pitcher arrives to the emergency department with excruciating left shoulder pain that occurred after falling on an outstretched arm during a game. He has no significant past medical history. There is no history of alcohol, tobacco, or illicit drug use. His temperature is 36.7 C (98 F), blood pressure is 120/66 mmHg, pulse is 100/min, and respirations are 18/min. A thorough evaluation reveals a compound fracture of the left clavicle. What is the most appropriate treatment to be offered to the patient? A. Placement of a figure-of-eight brace B. Closed reduction of the fracture C. Open reduction and internal fixation of the fracture D. Absolute rest and strong pain medication E. Sling placement Explanation: Certain fractures of the shoulder need to be treated in the operating room. The common ones are: compound fractures, distal comminuted fractures, multiple trauma, and severe displacement that jeopardizes the integrity of the skin. In the majority of cases, only pain medication, rest, and a sling or a figure-of-eight brace are needed (Choices A, B, D, and E); however, the presence of the compound fracture in this patient requires open reduction and internal fixation (ORIF) of the left clavicle. Educational Objective: Most shoulder fractures are treated with a sling or a figure-of-eight brace placement, along with rest and pain control; however, some lesions, such as compound, distal comminuted, or severely displaced fractures, need surgical intervention. A 46-year-old Caucasian woman comes to the office because of weight gain. She is also complaining of a lack of energy, constipation and memory deficits. She has no other medical problems. She denies the use of tobacco, alcohol or drugs. Her family history is not significant. She is not taking any medications. Physical examination shows an obese woman, with an increased abdominal girth. Relaxation phase of the ankle reflex is slow. Laboratory tests are ordered, and the patient is asked to return one week later. The results are the following: Thyroid tests TSH: 15 ?U/mL Free T4: 0.1 ng/dL Serum chemistry Serum Na: 126 mEq/L Serum KL 4.2 mEq/L Chloride: 90 mEq/L Bicarbonate: 25 mEq/L BUN: 4 mg/dL Serum creatinine: 0.8 mg/dL Calcium: 9.8 mg/dL Glucose: 90 mg/dL Which of the following is the most appropriate pharmacotherapy for this patient?s hyponatremia? A. Water restriction B. Hypertonic saline C. Free water administration D. Levothyroxine E. Demeclocycline Explanation:

Plasma osmolality is calculated as: 2 x Serum Na + Glucose / 18 + BUN / 2.8 Normal plasma osmolality is around 280-290 mOsm/L. In this case: (2 x 126) + 90/18 + 14/2.8 = 252 + 5 + 5 = 262 mOsm/L. This patient has euvolemic hypoosmolar hyponatremia, which has various etiologies, such as: hypothyroidism, adrenal insufficiency, and syndrome of inappropriate ADH secretion (SIADH). She presents with signs and symptoms characteristic of hypothyroidism, which is the most likely cause for the euvolemic hypoosmolar hyponatremia. She does not appear dehydrated or volume overloaded. Therapy for asymptomatic patients with chronic, mild hyponatremia (125-135mEq/L) is aimed at correcting the underlying cause; therefore, treatment with levothyroxine will control the symptoms of hypothyroidism and will correct hyponatremia. (Choice A) Patients with SIADH and mild or moderate (115-124mEq/L) hyponatremia can be managed with water restriction. (Choice B) Sodium replacement using hypertonic saline is reserved for symptomatic patients with severe hyponatremia (<115 mEq/L). (Choice C) Free water administration is part of hypernatremia management. (Choice E) Demeclocycline is helpful in the treatment of some chronic, persistent cases of SIADH or antipsychotic-induced hyponatremia. Educational Objective: Euvolemic hypoosmolar hyponatremia can be due to SIADH, hypothyroidism or adrenal insufficiency. If the patient is asymptomatic or the hyponatremia is mild, treatment is first directed at correcting the underlying disorder (e.g. using corticosteroids or levothyroxine). Moderate hyponatremia and SIADH respond to water restriction. Symptomatic patients and those with severe hyponatremia may need sodium replacement using hypertonic saline solution. A 74-year-old woman presents to the the emergency department (ED) with worsening shortness of breath, orthopnea, and cough productive of pinkish sputum. She is well known to you from her previous multiple admissions to the hospital. Her past medical history is significant for diabetes mellitus, hypertension, hypercholesterolemia, coronary artery disease, myocardial infarction, congestive heart failure, osteoarthritis, glaucoma, and peptic ulcer disease. Her temperature is 37.2C(99F), blood pressure is 140/90 mm Hg, pulse is 108/min, and respirations are 22/min. Physical examination reveals the presence of a third heart sound, bibasilar crackles and bilateral pitting pedal edema. Chest x-ray reveals bilateral haziness. EKG reveals evidence of an old anterior wall myocardial infarction. Her recent transthoracic echocardiogram showed an ejection fraction of 30%. She is admitted to the hospital and treated for exacerbation of congestive heart failure. At the time of discharge, her son requests you to persuade his mother to stay with him and his wife. He has made several attempts to convince her to stay with them in the past, but she always declines and says that she prefers to live alone. What would be the most appropriate step at this point? A. Convince her that she will be happier staying with her son and daughter-in-law. B. Explain to her that given her multiple health problems, it would not be a good idea for her to stay by herself. C. Ask her if there is a particular reason for her refusal to stay with her son. D. Discuss with her the advantages versus the disadvantages of staying with her son. E. Ask her to consider the advantages versus the disadvantages of staying with her son. Explanation: A physician?s foremost duty is to act as the patient?s advocate. It is unethical to simply comply with the son's request and not have a discussion with the patient, who seems capable of making sound decisions. If the patient's health is significantly poor, the physician must show his concern for her welfare by explaining her state of health and advising against living alone.

(Choice A) It is unethical for a physician to make assumptions on what will make the patient happier. (Choices C, D ane E) At this point, the physician should discuss the patient's health and advise against her living alone. The patient's reaction to the physician's advice will determine the latter's next plan of action. Unless she volunteers the reasons as to why she refuses to stay with her son, the physician cannot explore her decision to live alone; however, an attempt can be made to explain the potential advantages and disadvantages of staying with someone, such as her son. Asking her to consider these advantages and disadvantages may be appropriate, but this should only be done after discussing her health issues. Educational Objective: A physician?s foremost duty is to act as the patient?s advocate. If the patient's health is significantly poor, the physician must show his concern for her welfare by explaining her state of health and advising against living alone. The following vignette applies to the next 2 items An 8-year-old boy is brought to the emergency room by paramedics with a reported ingestion about four hours ago. The patient had an accidental ingestion of ?antifreeze? after mistaking it for a beverage in the garage. Item 1 of 2 Which of the following physical signs would be consistent with the history of ?antifreeze? ingestion? A. Dry skin and mucosal surfaces B. Irregular heart rate C. Excessive salivation D. Pupillary dilation E. Rapid and deep breathing Explanation: Ethylene glycol is one of the components of antifreeze and other solvents. It is one of the alcohols (along with ethanol, methanol and isopropanol) with fatal intoxications. It is important to recognize its toxicity early for instituting quick and definitive therapy. Ethylene glycol gets metabolized in the body by an enzyme, alcohol dehydrogenase (ADH), into a variety of toxic metabolites, including glycolic acid, glyoxylic acid, and oxalic acid. These toxic metabolites are responsible for most of the clinical symptoms and organ damage seen with antifreeze ingestion. Ethylene glycol ingestion leads to a severe anion gap metabolic acidosis. This causes a typical rapid and deep breathing pattern known as Kussmaul?s respiration. Some of the other early signs include nausea, vomiting, slurred speech, ataxia, nystagmus, and lethargy. Further toxicity may lead to tachypnea, agitation, confusion, flank pain, renal failure, pulmonary edema, changes in mental status, and eventually, progression to a coma. (Choices A and D) Dry skin and mucosal surfaces, as well as dilated pupils, are seen in overdoses of anticholinergic agents. The common agents implicated are tricyclic antidepressants, certain plants (Datura stramonium), and mushrooms. (Choice B) Ventricular tachyarrhythmias are seen with overdoses of tricyclic antidepressants and second-generation antihistamines such as astemizole or terfenadine. (Choice C) Excessive salivation is classically seen with organophosphate or carbamate poisoning. These are commonly the agents used in pesticides and insecticides. Educational Objective:

Ethylene glycol ingestion leads to a severe anion gap metabolic acidosis. This causes a typical rapid and deep breathing pattern known as Kussmaul?s respiration. Item 2 of 2 Which of the following is the most appropriate treatment of early ethylene glycol ingestion? A. Ethanol infusion B. Fomepizole infusion C. Ethanol and fomepizole infusions running together D. Ethanol infusion followed by fomepizole infusion E. Fomepizole infusion followed by ethanol infusion

Explanation: Early and prompt treatment of ethylene glycol ingestion is required to prevent permanent tissue damage and death. Ethylene glycol gets metabolized in the body by an enzyme, alcohol dehydrogenase (ADH), into a variety of toxic metabolites, including glycolic acid, glyoxylic acid, and oxalic acid. Fomepizole is a competitive inhibitor of ADH, and is the antidote of choice in cases of ethylene glycol intoxication (and also methanol intoxication). If used early, it prevents the formation of toxic metabolites, causes a dramatic improvement in acidemia, and prevents renal failure. It also prolongs the half-life of ethanol; therefore, simultaneous use with ethanol is not recommended (Choices C, D, and E). (Choice B) Ethanol can also be used for the treatment of patients with methanol or ethylene glycol intoxication. Alcohol dehydrogenase actually has a greater affinity for ethanol than the other alcohols; however, fomepizole inhibits ADH more potently than ethanol and is, therefore, the antidote of choice. Educational Objective: Fomepizole is the antidote of choice in cases of ethylene glycol and methanol intoxication. Simultaneous use with ethanol is not recommended. A 55-year-old Caucasian man comes to the physician with complaints of severe pain and swelling in his right big toe. He has a past medical history of hypertension, osteoarthritis, and peptic ulcer disease. His medications include hydrochlorothiazide, acetaminophen, naproxen, and cimetidine. On physical examination, his first metatarsophalangeal joint has diffuse swelling, and the skin surrounding the joint is erythematous. His joint is exquisitely tender to palpation, and has severe restrictions in movement. Aspiration of the joint fluid reveals the presence of multiple needle-shaped negatively birefringent crystals within the polymorphonuclear cells. You start the patient on colchicine. The next day, the patient says that the severity of the pain has markedly decreased. His uric acid level is 8.7 mg/dL, serum creatinine is 1.4 mg/dL, and his liver function tests are within normal limits. Which of the following should be done to reduce the recurrence of acute gouty attacks? A. Discontinue hydrochlorothiazide. B. Start the patient on allopurinol. C. Start the patient on metoprolol. D. Discontinue cimetidine. E. Start the patient on low-dose aspirin. Explanation: Gout is the most common form of crystal-induced arthropathies. It is characterized by recurrent attacks of acute inflammatory arthritis, and is diagnosed by the presence of negatively birefringent needle-shaped monosodium urate crystals within the polymorphonuclear cells in the synovial fluid. Gouty arthritis or inflammation of the first metatarsophalangeal joint, known as podagra, is the initial presentation in almost three-fourths of the patients. There are many conditions and risk factors which can predispose a patient to gout. These include, but are not limited to: hyperuricemia, obesity, hypertension, hyperlipidemia, alcohol ingestion, and drugs. The most common drugs associated with hyperuricemia include diuretics (thiazide and loop diuretics), cyclosporine, low-dose salicylates, and pyrazinamide. Diuretics cause an increased proximal tubular reabsorption and a decreased tubular secretion of uric acid in the kidneys. This leads to a decrease in urate excretion, causing hyperuricemia. Diuretic-induced hyperuricemia over time could predispose this patient to further attacks of gouty arthritis; therefore, discontinuing hydrochlorothiazide will help in preventing recurrences of acute gouty arthritis. (Choice B) Allopurinol decreases the serum uric acid levels by inhibiting a key enzyme, xanthine oxidase, in the process of uric acid synthesis. It is most useful as long-term prophylactic therapy in patients with hyperuricemia and recurrent gouty arthritis. It should not be initiated during or immediately after an acute attack of gout, since it can cause an acute flare or recurrence of gouty arthritis by causing fluctuations in serum uric acid levels.

(Choice C) Metoprolol does not affect or prevent future episodes of acute gouty arthritis. (Choice D) Cimetidine has no effect on uric acid secretion or excretion by the kidneys. It does not increase the frequency of gouty attacks and does not need to be discontinued. (Choice E) An episode of acute gouty arthritis can be treated with any of the nonsteroidal antiinflammatory agents; however, these do not have any role in the prophylaxis of gouty arthritis. Aspirin use is generally avoided in patients with gout because low-dose salicylates can cause uric acid retention in the kidneys. Educational Objective: Diuretics (thiazide and loop diuretics) can cause hyperuricemia by decreasing uric acid excretion, which can lead to recurrent gouty arthritis. These drugs should be substituted with a different antihypertensive agent in patients with gouty arthritis. A 35-year-old white woman was admitted to the hospital. She is 38-weeks pregnant, and apparently had premature rupture of membranes at home. The delivery is uneventful; however, hours later, the newborn looks ill, sleepy, and has a poor appetite. Extensive work-up is completed, and a diagnosis of neonatal sepsis is made. The patient is concerned because she will be discharged tomorrow, but her baby needs to stay for at least a couple of days or more, in order to receive parenteral antibiotic therapy. Her insurance only covers admission for two days in the hospital for a non-complicated vaginal delivery. She requests you to "do something about it" so she can stay with her baby until he is ready for discharge. Which of the following is the most appropriate course of action? A. Don?t worry, we will take care of that. I?ll call the insurance company. They?ll understand. B. Forget about the insurance. There will be no problem. C. I am sorry. Those are the insurance rules. We can try, but there is little hope. D. Be calm. I can write an order to admit you or lengthen your hospitalization E. Don?t bother about it, your baby will be fine. You can go home and come back whenever you want. Explanation: The physician must address the legitimate concern of the mother, explain the rules to her, and offer to help by attempting to talk to the insurance company about the rules on coverage; however, the physician must not make any guarantees to avoid any unrealistic expectations. (Choice D) Insurance rules are strict. If the physician tries to justify an extension of a hospitalization not based on clinical grounds, he can be accused of deceit or insurance fraud. (Choices A, B and E) It is not advisable to give any false reassurance or an optimistic superficial prognosis about the newborn?s disease to make the mother feel better. Doing so may later create feelings of anger and deception in the mother. Educational Objective: The physician always has a commitment to tell the truth not only to the patient, but also to the insurance company that pays for his services. No false hope is to be created in the patient, and insurance fraud is to be avoided. In cases like this, the physician and patient may attempt to talk to the insurance company about the rules on coverage; however, the patient must be advised to avoid unrealistic expectations. A 24-year-old Colombian-American woman is brought to the emergency room because of severe abdominal pain. She is 30 weeks pregnant, and has no other medical problems. The pain started 24 hours ago in the epigastrium, but the pain is now felt over the lower abdomen. She also feels nauseated, and has not eaten in the last 12 hours. Her temperature is 38.2 C (100.8 F), blood pressure is 100/70 mmHg, pulse is 100/min and respirations are 22/min. Examination reveals a gravid

uterus and increased fetal movements. There is severe tenderness of the right lower quadrant (RLQ) on palpation, which is also elicited through leg elevation and left lower quadrant pressure. The laboratory tests show the following: Hb: 12.86 g/dL Ht: 39% Platelet count: 260,000/cmm Leukocyte count: 16,000/cmm Segmented neutrophils: 75% Bands: 9% Lymphocytes: 16% Which of the following complications is more likely to occur if the patient is not adequately treated? A. Bacterial endocarditis B. Bacterial meningitis C. Pancreatitis D. Pylephlebitis E. Tubo-ovarian abscess Explanation: This patient has the characteristic signs and symptoms of acute appendicitis. McBurney?s sign (tenderness at McBurney?s point, the site two-thirds between the umbilicus and right anterior superior iliac spine), Rovsing?s sign (transmission of pain from the left to the right lower quadrant), and iliopsoas sign (RLQ pain on passive extension of the right hip) are all positive. There is a left shift in the white blood cell count (true for 95% of diagnosed cases). In a third trimester pregnant woman, there is a minimal risk of premature delivery; however, if rupture of the appendix occurs, peritonitis can lead to fetal death, abscess formation and pylephlebitis (infectious thrombosis of the portal veins). These complications are rare but feasible, especially if therapy is not offered within the first 24 hours of the initiation of symptoms. (Choice C) Pancreatitis is a common complication of biliary tract obstruction, not appendicitis. (Choices A and B) Bacterial endocarditis or meningitis can occur in extreme cases of sepsis, but their association with appendicitis is very uncommon. (Choice E) Appendicitis will not cause a tubo-ovarian abscess. Educational Objective: Around 14% of pregnant women in the second trimester can have premature delivery when affected by appendicitis, while around one third of those in the first trimester may experience abortion. During the third trimester, the main complication is appendix perforation with peritonitis and subsequent pylephlebitis. A 55-year-old Caucasian male is hospitalized after a massive hemorrhage from a duodenal ulcer. The hemorrhage was stopped with endoscopic manipulation. 2L of Ringer lactate was infused and 2 units of packed red blood cells (RBC) were ordered. Ten minutes after the transfusion of packed RBC began, the patient complains of tremors and feeling chilly. His temperature is 39.4 C (103 F), blood pressure is 120/76 mmHg, pulse is100/min, and respirations are 24/min. The transfusion is stopped, and acetaminophen is given. Direct anti-globulin test and plasma-free hemoglobin level are negative. Urinalysis is normal. The reaction abates three hours after the start of the transfusion. Which of the following actions could have prevented the patient's reaction to the transfusion? A. Warming the blood B. Using whole blood C. Careful cross matching of the blood D. Infusing calcium gluconate

E. Washing cells Explanation: The clinical scenario described is most consistent with febrile transfusion reaction. It is characterized by fever and chills that usually respond to NSAIDs and acetaminophen. Unlike more serious reactions (e.g., hemolytic reaction or bacterial contamination of the transfused blood), hemodynamic abnormalities and renal dysfunction are not usually present. The febrile reaction is caused by a reaction of the antibodies in the patients plasma to the donors leukocytes. The antibodies frequently have HLA specificity; therefore, leukocyte depletion techniques (although not commonly employed due to high cost) can reduce the probability of febrile transfusion reaction. These techniques are cell washing, use of frozen deglycerolized red cells, use of leukocyte depletion RBC filters, etc. (Choice B) Transfusion of whole blood carries an increased risk of febrile reaction because centrifugation removes up to 70% of leucocytes. (Choice C) Careful cross matching of the blood prevents hemolytic reactions. Acute hemolytic transfusion reactions (AHTRs) also present in a similar way. The most common complaints are fever and chills. The classic triad of fever, back pain, and red or pink color of the urine is rarely seen. In order to rule out a hemolytic reaction, the transfusion must be stopped and certain tests (direct antiglobulin/Coombs test and measurement of plasma-free hemoglobin level) must be performed. A urinalysis will also reveal the presence of hemoglobin. (Choice D) Calcium gluconate infusion is employed in rare cases of severe hypocalcaemia following massive blood transfusion. (Choice A) Warming the blood is recommended only during rapid, massive transfusions to prevent hypothermia. Educational Objective: Febrile reaction is a common transfusion reaction that is caused by a reaction of the antibodies in the patient?s plasma to the donor?s leukocytes. Leukocyte depletion techniques (e.g., cell washing) decrease the probability of febrile, nonhemolytic transfusion reaction. A 12-year-old girl comes to your office for the evaluation of her third episode of maxillary sinusitis during the past seven months. She complains of fever, headache, nasal congestion, purulent nasal discharge, and facial pain. Except for previous sinusitis episodes, her medical history is unremarkable. She has no known allergies. Her family history is unremarkable. Both her parents smoke cigarettes. On examination, there is tenderness on the maxillary sinuses. Respiratory, cardiac and abdominal examinations are normal. CT scan demonstrates mucosal thickening, opacification, and air-fluid levels in the maxillary sinuses. Which of the following would you say to the patient?s parents? A. Her sinusitis is caused by Wegener?s granulomatosis. B. Her sinusitis is due to a hereditary immune deficiency. C. Her sinusitis is caused by Kartagener syndrome. D. Cigarette smoking is probably associated with her disease. E. This may be caused by cocaine use. Explanation: This patient is having another episode of acute sinusitis. Recurrent sinusitis consists of repeated bouts of acute sinusitis without leaving any significant damage. In contrast, chronic sinusitis is characterized by more subtle and variable symptoms, and is associated with mucous tissue damage. Symptoms last for eight weeks or longer, or occur more than four times a year, and persist for more than 20 days.

Cigarette smoke and air pollution can damage the cilia responsible for moving mucus through the sinuses. The mucus then builds up, forms plugs, obstructs the sinuses, and causes inflammation. Other causes of chronic or recurrent sinusitis include untreated acute sinusitis, structural abnormalities of the nasal septum or palate, and chronic medical disorders that cause inflammation in the airways or persistent thickening of mucus. Some of these include diabetes, AIDS or other disorders of the immune system, cystic fibrosis, Kartagener syndrome, and Wegener's granulomatosis. (Choice A) Wegeners granulomatosis is a systemic necrotizing vasculitis marked by glomerulonephritis, chronic pneumonitis, and ulceration of the nasopharyngeal mucosa. Other clinical features include fever, neuropathy, skin rashes, and joint pain. This patient has none of these symptoms; therefore, this diagnosis is very unlikely. (Choice B) Hereditary immune deficiency is unlikely because the patient's family history is unremarkable. In addition, the sinusitis in hereditary immune deficiency is unlikely to be isolated; it would have to be associated with failure to thrive. (Choice C) Kartagener syndrome is an autosomal recessive disorder consisting of transposition of the viscera, chronic sinusitis, and airway disease leading to bronchiectasia. It is part of a group of respiratory disorders known as primary ciliary dyskinesia. This patient has recurrent sinusitis, but is otherwise normal; therefore, Kartagener syndrome is unlikely. (Choice E) Cocaine abuse may cause a constant runny nose; however, it is not known to result in chronic or recurrent sinusitis. Educational Objective: Cigarette smoke and air pollution cause recurrent or chronic sinusitis by damaging the cilia responsible for moving mucus through the sinuses. Other causes of chronic or recurrent sinusitis include untreated acute sinusitis, structural abnormalities of the nasal septum or palate, and chronic medical disorders that cause inflammation in the airways or persistent thickening of mucus. A 6-year-old African American girl is brought to the emergency department (ED) with progressive sleepiness and loss of appetite. She was diagnosed with sickle cell anemia two years ago, after an episode of severe pneumonia that required hospitalization and an exchange transfusion. She has had two episodes of painful crisis and one episode of acute otitis media in the last two years. All her vaccinations are up-to-date. She has no known drug allergies. Her blood pressure is 110/65 mmHg, pulse is 120/min, temperature is 36.7C (98F), and respirations are 25/min. Physical examination reveals pallor. Laboratory findings are as follows: Hemoglobin: 6.0 g/L Erythrocyte count: 2.2 mln/mm3 MCHC: 32% MCV: 85 fl Reticulocytes: 0.1% Platelets: 200,000/mm3 Leukocyte count: 4,500/mm3 Neutrophils: 56% Eosinophils: 1% Lymphocytes: 33% Monocytes: 10% Which of the following is the most likely diagnosis of this patient? A. Sequestration crisis B. Hyperhemolytic crisis C. Acute chest syndrome D. Bacteriemia E. Aplastic crisis

Explanation: Acute severe anemia may be superimposed on chronic anemia in patients with sickle cell disease. When this occurs, it is a life-threatening condition. Clinical manifestations include weakness, pallor and lethargy. The three typical causes of acute anemia are splenic sequestration crisis, aplastic crisis, and hyperhemolytic crisis. Aplastic crisis represents a transient failure of erythropoiesis with an abrupt reduction in the blood hemoglobin and the number of erythroid precursors in the bone marrow. A very important finding during a crisis is the virtual absence of reticulocytes. The typical cause is an infection; parvovirus B19 infection is the most common cause in children. (Choice A) Splenic sequestration crisis is explained by the splenic pooling of RBCs. It is more common in young children with sickle cell disease because the spleen undergoes scarring and atrophy with age. (Choice B) Hyperhemolytic crisis is a rare diagnosis. In some cases, it can be explained by an associated glucose-6-phosphate dehydrogenase deficiency. Most importantly, both splenic sequestration and hyperhemolytic crises are accompanied by high reticulocytosis due to increased hemolysis. (Choice D) Patients with sickle cell anemia are predisposed to bacteremia due to splenic fibrosis; however, fever, hypotension and leukocytosis usually accompany this disease. (Choice C) Acute chest syndrome is characterized by fever and chest pain. Educational Objective: Aplastic crisis represents a transient failure of erythropoiesis with an abrupt reduction in the blood hemoglobin and the number of erythroid precursors in the bone marrow. A very important finding during an aplastic crisis is the virtual absence of reticulocytes. The following vignette applies to the next 2 items A 36-year-old African-American woman comes to the emergency department (ED) and complains of a severe, throbbing headache that began several hours ago. She also developed blurred vision and nausea. She took acetaminophen, which did not provide any relief. She was diagnosed with Raynaud?s phenomena a few years ago. She recently started having heartburn, for which she takes over-the-counter ranitidine. Her temperature is 36.7C(98F), blood pressure is 200/110 mm Hg, and pulse rate is 94/min. Pulse oximetry showed 94% in room air. There is no abdominal bruit. The lab results are as follows: CBC Hb: 11.5g/dL MCV: 82 fl Platelet count: 280,000/cmm Leukocyte count: 7,500/cmm Serum Serum Na: 135 mEq/L Serum K: 4.2 mEq/L Chloride: 98 mEq/L Bicarbonate: 20 mEq/L BUN: 30 mg/dL Serum Creatinine: 2.0 mg/dL Calcium: 9.0 mg/dL Blood Glucose: 118 mg/dL Item 1 of 2

All her routine labs were completely normal six months ago. Her blood pressure during her previous visits was always less than 140/80 mm Hg. What is the most likely diagnosis of this patient? A. Malignant hypertension B. Renal artery stenosis C. Scleroderma D. Hemolytic uremic syndrome E. Pheochromocytoma Explanation: Based on the patient's past medical history (Raynaud's phenomena and GERD) and physical findings, she is most likely having scleroderma renal crisis which resulted in acute renal failure and severe hypertension. (Choice A) The patient does not have a prior history of hypertension. Malignant hypertension and malignant nephrosclerosis are therefore unlikely. (Choice B) Another possible cause for a sudden onset of renal crisis is renal artery stenosis; however, the absence of a renal bruit makes it an unlikely diagnosis. (Choice D) Sudden onset renal failure can be seen in HUS; however, it is predominantly manifested by hematological abnormalities, (i.e., thrombocytopenia and microangiopathic hemolytic anemia), and there is usually an accompanying history of prior diarrhea. (Choice E) Pheochromocytoma is usually associated with episodes of tachycardia, palpitations, headache and hypertension.

Item 2of 2 Which of the following is the most appropriate pharmacotherapy? A. Administer intravenous furosemide (Lasix) B. Start enalapril C. Administer calcium channel blockers D. Administer intravenous hydrocortisone E. Start intravenous sodium nitroprusside Explanation: Scleroderma (Systemic sclerosis) is characterized by the abnormal deposition of collagen in multiple organ systems, including the skin and blood vessels. Majority of the patients have underlying renal involvement, but overt symptoms of scleroderma renal disease are seen less commonly. Thickening of the vessel wall and narrowing of the vascular lumen result in ischemia, which activates the reninangiotensin system. This results in hypertension, which is often as severe as malignant hypertension, as seen in this patient. It is therefore important to monitor the blood pressure in scleroderma patients, and the presence of hypertension should alert the physician to the possibility of scleroderma renal disease. ACE inhibitors are the agents of choice in the treatment of hypertension since these reverse the angiotensin-induced vasoconstriction. These also alleviate the renal failure in majority of patients. Most patients respond favorably if these drugs are used promptly after the diagnosis. (Choices A, C, D and E) The other anti-hypertensive agents are not first-line agents for hypertension in scleroderma renal crisis. A sudden reduction of blood pressure is not favorable because this can further compromise renal perfusion and result in acute tubular necrosis. The use of potent parenteral agents such as nitroprusside and labetalol is therefore not indicated. Educational Objective: ACE inhibitors are the drugs of choice for the treatment of hypertension in scleroderma renal crisis. A 34-year-old diabetic woman undergoes emergency laparoscopic cholecystectomy for acute cholecystitis. On the third post-operative day, she has a fever of 38.0 C (100.3 F). The site of the intravenous cannula in the cephalic vein is erythematous and tender. A cord is palpated at this site. The cannula is removed, and treatment with nafcillin is started. On the sixth post-operative day, the patient's fever persists. Erythema and tenderness of the same site have increased. Auscultation of the lungs reveals clear breath sounds; the abdomen is quiet. Two blood cultures are positive for Staphylococcus epidermidis, which is susceptible to naficillin. Urine cultures are negative. The abdominal ultrasound is negative for fluid collections. What is the most appropriate next step in the management of this patient? A. Increase the dose of naficillin to her maximal B. Chest x-ray C. Abdominal CT scan with contrast D. Needle aspiration of the affected area of the cephalic vein E. Add intravenous heparin to the current treatment Explanation: This patient is persistently febrile, despite using susceptible antibiotics. This patient may have a suppurative superficial thrombophlebitis (collection of pus in the vein). The cephalic vein has to be explored and drained of any pus found. (Choice A) Antibiotic therapy alone would not cure this condition unless the involved vein is drained of pus. (Choice E) In addition to antibiotics, heparin is used for treatment of deep thrombophlebitis, most notably for phlebitis of the pelvic veins as an obstetric complication. Because of the minimal risk of pulmonary thromboembolism due to superficial thrombophlebitis, heparin is rarely used for this condition.

(Choices B, C and F) Post-operative fever can be due to pneumonia, lung abscess, or intraabdominal abscess. These possibilities must be checked if it is proven that the site of the infection is not the cephalic vein. Educational Objective: Superficial suppurative thrombophlebitis requires drainage if the antibiotic therapy alone is ineffective. A 19-month-old Caucasian boy is brought to the emergency department. According to his mother, he has been constantly crying, and this is apparently due to intermittent abdominal pain. From time to time, he draws up his legs and pulls them towards his abdomen. His symptoms began ten hours ago. He has vomited at least four times within the past few hours, and his last episode produced a bilious vomitus. He has not been able to eat since then. He has passed loose stools that were admixed with blood and mucus. He has no other medical problems. His temperature is 37.5? C (99.5?F) and pulse is 116/min. He appears dehydrated and apathetic. Examination reveals a distended abdomen with increased bowel sounds. On palpation, the abdomen is soft and very tender. There is no rebound tenderness, but a tender, sausage-shaped abdominal mass is palpated on the right side. Rectal examination reveals a small amount of bleeding. Plain x-ray of the abdomen reveals widespread distention of the bowel loops. The patient receives intravenous hydration. Which of the following is the most likely diagnosis? A. Inflammatory bowel disease (IBD) B. Meckel?s diverticulum C. Acute gastroenteritis D. Appendicitis E. Intussusception Explanation: Intussusception occurs when a part of the small bowel is telescoped into itself. The most frequent vicinity is the ileocecal region. Almost 60% of the cases occur in children younger than 1 year of age, while 80% present before 2 years of age. The classic presentation is a sudden onset of colicky abdominal pain followed by vomiting, and the vomitus rapidly changes from alimentary to bilious. The child usually draws up his legs towards the abdomen because of the pain. Hematochezia is present in 70% of the cases; 15% of the patients have the characteristic "currant jelly" stools. Initially, the child is irritable, but he may eventually become apathetic when he is no longer able to eat, and he gets dehydrated. Palpation of a sausage-shaped abdominal mass on the right side is characteristic. Abdominal x-ray reveals generalized distension of the bowel loops or another evidence of obstruction. An enhanced density that projects into the air level of the large bowel can be identified in 25% of the cases. This density represents the invaginated bowel loop, and is known as the "crescent sign." (Choice C) Acute gastroenteritis (AGE) presents with fever that precedes abdominal pain and diarrhea. AGE will not explain the presence of the abdominal mass and the massive bowel distention seen in the abdominal x-ray of this patient. (Choice B) Meckel?s diverticulum usually presents in toddlers and older children, generally between five and ten years of age, and in young adults. The most common location of pain is in the right lower quadrant. Hematochezia is also present, but the stool is non-mucoid, and there is no palpable abdominal mass. (Choice D) The early presence of the mass rules out appendicitis, which typically starts as epigastric pain and then localizes in the right lower quadrant. Furthermore, hematochezia is extremely uncommon in this condition. (Choice A) Inflammatory bowel disease (IBD) rarely affects children. When it does so, the patients are usually in the preschool or school age. As in the other conditions, the presence of the sausage-shaped mass in an acute setting rules out this entity. Educational Objective:

Intussusception is one of the most common causes of complicated abdominal pain in infants. The characteristic presentation is an acute onset of abdominal pain, progressive vomiting, dehydration, and diarrhea with bloody, mucoid stools (currant jelly). Pertinent physical findings are abdominal distention and palpation of a sausage-shaped abdominal mass on the right side. Abdominal x-ray may reveal generalized distention of the bowel loops, evidence of intestinal obstruction, or the "crescent sign."

A 63-year-old African-American male presents for the evaluation of urinary urgency, hesitancy, nocturia, and a weak urinary stream. These symptoms have progressively worsened over the past year, despite being on doxazosin. He denies any fever, abdominal pain, hematuria, malaise, or weight loss. He is a non-smoker and non-alcoholic. Rectal examination reveals a smooth, firm enlargement of the prostate without any induration or asymmetry. Neurological examination is normal. His PSA level is elevated, and you explain the option of transurethral resection of the prostate. He is concerned about the complications of the procedure. Which of the following is the most likely expected complication of this procedure? A. Retrograde ejaculation B. Urinary tract infection C. Urethral stricture D. Urinary incontinence E. Erectile dysfunction Explanation: Retrograde ejaculation is the most frequent complication of transurethral resection of the prostate (TURP), which is an invasive procedure used in the management of benign prostatic hyperplasia (BPH). (Choices B & C) UTIs and urethral strictures are also complications of TURP, but are less common. (Choices D and E) Urinary incontinence and erectile dysfunction are two common complications of radical prostatectomy, which is used for the treatment of early prostate cancer. These complications occur due to damages to the urethral sphincter and penile nerves during surgery. These complications do not occur with TURP. Educational Objective: Retrograde ejaculation is the most frequent complication of transurethral resection of the prostate, and all patients should be made aware of this. A 63-year-old afebrile man is admitted to your service for the evaluation of an episode of slurred speech and right, upper extremity weakness which resolved within one hour. The patient did not have any headaches or loss of consciousness. He denies any history of head trauma, heart disease, arteritis, vasculitis, connective tissue disease, or syphilis. His past medical history is significant for mild hypertension. He stopped smoking eight years ago, after a 60 pack year history of smoking. A bruit is heard on auscultation of the upper anterior cervical triangle on the left side of the neck. Carotid duplex ultrasonography reveals a greater than 70% left internal carotid artery (ICA) stenosis, and no stenosis in the right ICA or common carotid. These findings are confirmed by angiography. What is the most effective treatment strategy for this patient? A. Anticoagulation with low dose IV heparin and oral coumadin B. Prescription of high dose aspirin (650 to 1300 mg/day) or ticlopidine C. Extracranial-intracranial bypass between the left external carotid and left middle cerebral arteries D. Placement of a left carotid artery stent E. Left carotid endarterectomy Explanation: This patient's history is most consistent with a transient ischemic attack (TIA). The brief duration of focal neurological symptoms and absence of headaches make other cerebrovascular pathology and migraine unlikely. A simple partial (focal) seizure would generally begin with abnormal movements in an extremity before paresis, and is thus also unlikely. The probable cause of this patient's presentation is embolization of fragments from an ulcerated, atherosclerotic plaque in the left ICA into branches of the middle cerebral artery supplying the motor cortex and Broca's area. Randomized, controlled clinical trials in patients with any recent carotid TIA, and who have a documented

obstruction of > 70% in the ipsilateral carotid artery, have shown that an endarterectomy significantly reduces the (otherwise increased) risk of a subsequent stroke, when compared with medical therapy alone. For an obstruction of < 30%, medical therapy is preferred. For an obstruction between 30 and 70%, the best therapy remains controversial. (Choice A) Anticoagulation is not routinely indicated in the treatment of isolated transient ischemic attacks due to atherothrombosis. An exception may be a patient with recent, frequent, or daily TIA's. Anticoagulation is indicated in selected patients suffering from evolving or completed, small, ischemic and non-hemorrhagic infarcts of cardioembolic origin (e.g., due to emboli originating from mural thrombi in patients with atrial fibrillation). (Choice B) This describes a major component of medical therapy to prevent the progression of and/or recurrence of non-hemorrhagic strokes, namely platelet aggregation inhibitors. All patients with TIA's may benefit from such therapy. In this patient, the definitive treatment is endarterectomy; however, he may be given aspirin or another anti-platelet agent at least up until two weeks prior to surgery. (Choice C) This is a possible treatment regimen; however, its benefits are less proven compared to endarterectomy. It is generally reserved for a minority of patients who require immediate carotid occlusion, or who have inoperable ipsilateral occlusion with inadequate collateral flow and continued symptoms, despite anticoagulation. (Choice D) Carotid stenting may be a feasible, safe alternative to endarterectomy in selected patients; however, more randomized, controlled clinical trials are needed to compare the long-term results of these two procedures. Educational Objective: In a patient with a unilateral internal carotid artery (ICA) stenosis that is greater than 70%, carotid endarterectomy in the distribution of the affected ICA is indicated for the prevention of stroke after a first TIA. A 19-year-old Caucasian male is brought to the emergency department (ED) after a major motor vehicle accident. He was an unrestrained driver. He confessed to consuming a moderate amount of alcohol several hours ago. He did not lose his consciousness. His blood pressure is 150/95 mmHg, and heart rate is 110/min. His respiration is shallow and rapid. He is able to move his extremities and obey commands. His pupils are equal and reactive to light. Inspection reveals anterior chest bruises and peripheral cyanosis. Breaths sounds are heard over the lower lung lobes. Neck vein distention is not present. Which of the following is the most likely diagnosis in this patient? A. Tension pneumothorax B. Massive hemothorax C. Cardiac tamponade D. Flail chest E. Cardiac contusion Explanation: The patient has tachypnea, shallow breathing, tachycardia, anterior chest bruises and peripheral cyanosis. The clinical scenario described is most consistent with flail chest. Flail chest is usually the result of double rib fractures in more than one site, and is present in 10-20% of trauma admissions. The main pathophysiologic feature of flail chest is the increased work of breathing due to muscular spasm and pain. Hypoxia develops frequently due to associated pulmonary contusions. (Choices A and B) Pneumothorax and massive hemothorax are typically associated with hypotension and a unilateral decrease in breath sounds. (Choice C) Cardiac tamponade is characterized by hypotension and neck vein distension.

(Choice E) Cardiac contusion, if massive, may lead to hemodynamic instability; however, significant respiration changes (rapid shallow breathing) should not be present. Educational Objective: Flail chest is usually the result of double rib fractures in more than one site, and is present in 10-20% of trauma admissions.

A 25-year-old Caucasian female presents to her gynecologist and complains of amenorrhea for the past three weeks. Her past medical history is significant for diabetes mellitus type 1 and a wellcontrolled seizure disorder. Two months ago, she changed her method of contraception from condoms to birth control pills. Menarche was at the age of 12. Her menstrual cycle is normally 29 days long, and menstruation lasts for 3-4 days. The physical exam is normal. Her BMI is 21.8. What is the most likely cause of her amenorrhea? A. Noncompliance B. Pelvic inflammatory disease C. Polycystic ovary syndrome D. Decreased oral contraceptive efficacy secondary to antiseizure medication usage E. Decreased oral contraceptive efficacy secondary to insulin usage Explanation: It is likely that this patient is amenorrheic because she is pregnant. Numerous commonly used antiseizure medications (e.g., phenytoin, carbamazepine, ethosuximide, phenobarbital, topiramate) are known to decrease the efficacy of oral contraceptives through the induction of the cytochrome P450 system in the liver. Alternative antiseizure medications that do not decrease oral contraceptive efficacy include gabapentin and valproate. (Choice A) Noncompliance in this patient is less likely because of her history of seizure disorder that is well-controlled by medication. (Choice B) Symptoms of pelvic inflammatory disease may include lower abdominal pain, chills, fever, menstrual disturbances, purulent cervical discharge, and cervical and adnexal tenderness. This patient?s normal physical exam is inconsistent with a diagnosis of pelvic inflammatory disease. (Choice C) Polycystic ovary syndrome is a known cause of amenorrhea and is typically manifested by hirsutism, obesity, and virilization. This patient?s normal physical exam and BMI are inconsistent with a diagnosis of polycystic ovary syndrome. (Choice E) Insulin usage does not reduce the efficacy of oral contraceptives. Educational Objective: The most common cause of secondary amenorrhea is pregnancy. Drugs that inhibit or induce the cytochrome P450 system can reduce oral contraceptive efficacy. A 32-year-old homeless, Caucasian man is hospitalized because of fever, chills, diaphoresis, and altered mental status. He has had these symptoms for the past 24 hours. He is an intravenous drug abuser and has been using heroin for the past four years. He also drinks 4-5 bottles of beer daily, and has smoked one pack of cigarettes a day for the past 15 years. His temperature is 38.3 C (101 F), blood pressure is 100/60 mmHg, pulse is 118/min, and respirations are 16/min. Examination shows clear lung fields, normal first and second heart sounds, and a 2/6 systolic murmur in the apex. Mental status examination shows a drowsy, but easily aroused patient, with mild right-sided hemiparesis. Therapy is started with vancomycin and gentamycin. Eight hours later, the patient is vomiting and having abdominal cramps. On physical examination, his eyes are watery, and his pupils are dilated. There are no new neurologic findings. Which of the following is the most appropriate intervention at this moment? A. Administer naloxone B. Administer methadone C. Administer chlordiazepoxide D. Perform a lumbar puncture E. Order a head CT scan Explanation:

The patient is having bacterial endocarditis complicated by embolic stroke. This is the result of intravenous drug abuse. In the course of treatment, he develops mydriasis, nausea, vomiting, and watery eyes. These signs, as well as piloerection, rhinorrhea, and diarrhea, are indicators of opioid withdrawal, which can be readily treated with the administration of methadone or other opioids. (Choice C) Chlordiazepoxide is used in the management of alcohol withdrawal, which is characterized by tremors, hypertension, and when more advanced, hallucinations. (Choice A) Naloxone is used in the management of opioid overdose, which is characterized by miosis, and not mydriasis. (Choices D and E) There is no need for further studies at this point since the patient?s signs and symptoms can be explained by opioid withdrawal. Educational Objective: Intravenous drug abusers can develop multiple neurologic complications. Miosis is an indicator of opioid overdose; mydriasis is an indicator of opioid withdrawal, or cocaine, amphetamines, and tricyclics overdose. A 24-year-old male is brought to the emergency department by his friend because of disorientation and restlessness. This patient was hospitalized one month ago for opioid overdose with subsequent aspiration pneumonia. His blood pressure is 160/100 mmHg and heart rate is 120/min. He is not oriented in time and space, but recalls his name and date of birth. Bilateral vertical nystagmus is present. Which of the following is the best next step in the management of this patient? A. Diazepam B. Haloperidol C. Alkalization of urine D. Hemodialysis E. Low-sensory environment Explanation: This patient presents with symptoms and signs suggestive of phencyclidine (PCP) intoxication. Although high doses of many CNS depressants may also produce nystagmus, the scenario is typically associated with prominent sedation; nystagmus in awake or agitated patients is characteristic for PCP intoxication. This patient is disoriented and restless, but is not agitated or shows aggressive behavior. Placement in a low-sensory environment, along with metabolic and hemodynamic control, is the most reasonable approach. (Choice B) Haloperidol is typically required in patients who demonstrate aggressive behavior. (Choice A) Benzodiazepines are used to treat agitation and seizures in these patients. (Choice C) PCP is a weak base. Alkalization of the urine is not helpful for its removal. (Choice D) PCP has a large volume of distribution (good tissue penetration). Hemodialysis cannot effectively remove the drug from the body. Educational Objective: A low-sensory environment is ideal for patients with PCP intoxication. A 54-year-old African-American woman comes to the physician because of gross hematuria. This symptom began seven days ago, and is progressively getting worse. She denies any other symptoms. She has no other medical problems. She has smoked one pack of cigarettes daily for twenty years. She drinks 1-2 beers daily. Her family history is not significant. She takes no medication. Her vital signs are within normal limits. Examination shows no abnormalities. Which of the following is the most appropriate next step in the management of this patient?

A. CT scan of the abdomen and pelvis B. X-ray of kidney, ureter, and bladder C. Intravenous pyelogram D. Urinalysis E. Renal ultrasound

Explanation: The first step in the management of all patients with hematuria is to obtain a urinalysis. Urinalysis confirms the presence of hematuria, and differentiates extraglomerular hematuria from glomerular hematuria. Red cell casts, dysmorphic red cells, and associated proteinuria are features of glomerular bleeding. Centrifugation of urine allows distinction between hematuria, hemoglobinuria, and myoglobinuria. (Choice F) Patients with glomerular bleeding should be referred to a nephrologist, who will decide if a renal biopsy is required or not. (Choices A, C, and E) All patients with unexplained extraglomerular hematuria should undergo further radiological studies to localize the lesion of kidney, collecting system, ureter, or bladder. Intravenous pyelogram (IVP) is usually the first imaging modality of choice. It is superior to renal ultrasound for better detection of medullary sponge kidney and lesions in the renal pelvis and ureters. Young patients with normal IVP do not require further imaging; however, older patients with normal IVP should be followed by ultrasound or helical CT for the detection of possible malignancy. (Choice B) X-ray of the kidney, ureter, and bladder has a limited role in the diagnostic evaluation of hematuria. Educational Objective: The first step in the management of all patients with hematuria is to obtain a urinalysis. The following vignette applies to the next 2 items A 64-year-old Caucasian female presents to the emergency department due to several hours history of right-sided weakness in her arm and leg. She has slight slurring of speech, and denies nausea, vomiting, headache, visual problems or swallowing difficulty. She had an episode of arm weakness two days ago that lasted 15-20 minutes and resolved spontaneously. She takes medications for hypertension, diabetes, joint pains and occasional heartburn, but does not remember the names of the drugs. She does not smoke or consume alcohol. Her blood pressure is 150/90 mmHg and heart rate is 80/min. CT scan is negative for intracranial hemorrhage. Two days after the initial event, the patient is stable and shows no progression of neurological deficit. Telemetry monitoring shows paroxysmal atrial fibrillation. Warfarin therapy is started in this patient. Item 1 of 2 Which of the following is the biggest concern in this patient's therapy? A. Thrombocytopenia B. Skin necrosis C. Drug interaction D. Recurrent strokes E. GI bleeding Explanation: Warfarin therapy is a popular method of prevention of thrombotic complications. It can be used to prevent recurrent ischemic strokes, especially if the stroke is embolic. Atrial fibrillation is probably the most common indication for warfarin therapy. Drug interactions and concurrent medical problems are among the most common causes of suboptimal or excessive anticoagulation in patients on warfarin therapy. The list of drugs (both prescription and over-the-counter) that can potentially interact with warfarin is extensive. This patient is taking multiple medications; therefore, she is at a significant risk for dangerous drug interactions. (Choices D and E) A large study has shown that adequate anticoagulation with warfarin (INR 2.0 - 3.0) significantly decreases the risk of ischemic complications (including strokes), whereas the risk of major bleeding complications remains low. At the same time, excessive anticoagulation (INR > 4.0) increases the risk of bleeding complications substantially; therefore, careful monitoring of coagulation status is crucial.

(Choice B) Skin necrosis is a rare complication of warfarin therapy that usually occurs early during the therapy and is related to high doses of the drug. (Choice A) Thrombocytopenia is not a typical complication of warfarin therapy. Thrombocytopenia is seen with clopidogrel and heparin. Educational Objective: Drug interactions and concurrent medical problems are among the most common causes of suboptimal or excessive anticoagulation in patients on warfarin therapy. Item 2 of 2 Five days after the initial event, the patient demonstrates some improvement in the weakness of his extremities. She is cooperative and communicates her thoughts effectively. Her blood pressure is 130/80 mmHg and heart rate is 70/min. Power on her right extremities is 3/5 and 5/5 on her left. What is the most appropriate next step for this patient at this point? A. Permanent nursing home placement B. Recommend respiratory therapy C. Refer to local rehabilitation facility D. Recommend behavioral therapy E. Recommend massage therapy Explanation: Stroke is a disabling illness that affects many aspects of a patient?s life and places a substantial burden on family members and others. The goal of rehabilitation is to minimize the impact of the disability and increase the quality of life. Several studies suggest that the early initiation of rehabilitation therapy can improve the recovery process and improve the functional status. It has been estimated that out of 30-day stroke survivors, 10% demonstrate complete recovery, 10% do not respond to any therapy and 80% may potentially benefit from a complex rehabilitation process. Stroke survivors who do not undergo rehabilitation are more likely to be institutionalized. Several rehabilitation programs have been evaluated and are widely recommended; these include physical therapy, occupational therapy and speech therapy. Physical therapy stimulates specific skills acquisition and may aid in gait and arm retraining. It may include manual stimulation to evoke muscle/nerve responses and promote functional recovery. Occupational therapists develop competency in the activities of daily living. Speech therapy assists the patient to some degree of useful language communication. (Choice B) Respiratory therapy may be indicated in patients with weak respiratory muscles. Educational Objective: Several studies suggest that the early initiation of rehabilitation therapy after a stroke can improve the recovery process and functional state. A 27-year-old Caucasian man comes to see you in the office with complaints of painless sores on his glans penis. He first noted the lesions approximately two months ago. He adds that his whole body feels stiff, especially in the early morning hours, but this gets better with activity as the day advances. He also complains of generalized fatigue, achiness and lower back pain. He denies any other symptoms, including chest pain, palpitations, cough, shortness of breath, fever, abdominal pain, diarrhea, blood in the stool, or any skin rashes. His lab results are as follows: Hb: 14.4 g/dL Ht: 38% MCV: 92fl Platelet count: 280,000/cmm Leukocyte count: 8,000/cmm Segmented neutrophils: 70% Lymphocytes: 24% ESR (Westergren): 80 mm/h

Which of the following would be most likely seen on his diagnostic or physical examination? A. Prescribe oral trimethoprim and sulfamethoxazole for three days. B. Limitation of spine motion on lumbar flexion C. Presence of thickened lesions on his nails D. HLA-B51 positive E. Inflammatory arthritis Explanation: This patient has classic features of inflammatory spinal pain, which is one of the diagnostic criteria for spondyloarthropathy. Back pain is considered to be inflammatory in nature if it has an insidious onset, is associated with morning stiffness, improves with exercise or activities, and occurs before the age of 40 years. The presence of painless ulcers over his glans penis and inflammatory arthritis are suspicious for a diagnosis of Reiter?s syndrome. Reiter?s syndrome classically occurs as a triad of urethritis, uveitis, and arthritis. The clinical finding that is common to all patients with inflammatory spondyloarthropathy or spinal disease is the limitation of spine movement. This is tested by performing a Schober test. In this test, the spinous process of the fifth lumbar disc is identified and marked. A point 10 cm above the first mark is identified and marked as well. The patient is then asked to bend forward, causing lumbar spine flexion, and the distance between the two points is measured. A normal Schober test is a measured distance of 15 cm or more between the two marked points. All other inflammatory spondyloarthropathies without a specific known cause are usually classified into undifferentiated spondyloarthropathies. (Choice A) Arthritis or spondyloarthropathy can also occur as extraintestinal manifestations of various primary gastrointestinal disorders. These include disorders such as Crohn?s disease and ulcerative colitis, celiac sprue, or gluten-sensitive enteropathy, Whipple?s disease and Beh??s disease. Patients with Crohn?s disease can have oral aphthous ulcers, small bowel or colonic involvement associated with the spondyloarthropathy. They usually also have skip lesions identified on colonoscopy, with large areas of diseased colon separated by normal appearing colonic mucosa. Most of these patients are symptomatic, with complaints of fever, generalized fatigue and malaise, abdominal pain, diarrhea or blood in the stool. The patient in the above vignette does not have any typical gastrointestinal symptoms which could suggest a diagnosis of Crohn?s disease. (Choice C) Psoriatic arthritis usually presents as inflammatory arthritis, with pain and stiffness in the affected joints. The stiffness is worse in the morning and with prolonged immobility, and is relieved by physical activity. Patients with psoriatic arthritis also have the typical nail lesions (pitting or onycholysis) and skin lesions. Genital or penile ulcers are not typically seen in patients with psoriasis. (Choice D) HLA-B27 positive patients have a high incidence of developing spondyloarthropathies, especially with spinal involvement. HLA-B51 has not been associated with a high incidence of spondyloarthropathy. Educational Objective: Inflammatory spondyloarthropathy of the spine classically presents with back pain that is worse in the morning and is typically relieved by physical activity. There is limitation of motion on lumbar spine flexion (Schober test). A 70-year-old white female with an 80% right internal carotid artery stenosis is admitted to the hospital for carotid endarterectomy. She reports no symptoms. Her temperature is 36.7 C (98 F), blood pressure is 120/76 mmHg, pulse is 80/min., and respirations are 14/min. Physical examination is unremarkable. Her urine analysis showed 10-15 WBC/HPF. Urine culture showed > 100,000 colonies/mL of Escherichia coli, which is sensitive to trimethoprim and sulfamethoxazole combination.

Her BUN level is 20 mg/dL, and serum creatinine level is 1.2 mg/dL. Which of the following is the most appropriate next step in the management of this patient? A. Prescribe oral trimethoprim and sulfamethoxazole for three days. B. Prescribe oral trimethoprim and sulfamethoxazole for seven days. C. Reassurance and repeat urine culture in two months. D. Obtain blood cultures. E. Do regular straight catheterization to avoid stagnation of the urine. Explanation: This patient has asymptomatic bacteruria. Elderly patients are more likely to have asymptomatic bacteruria than frank urinary tract infection. Antibiotic therapy is usually not recommended if the urine WBC is < 20 WBC/HPF, and if the patient does not have any symptoms. Most of these patients' bacteruria will resolve without treatment over a period of time. Very few patients may develop urosepsis; hence, patients should be advised to report any symptoms of UTI, such as fever, dysuria, frequency, or suprapubic pain. Repeat urine culture should be performed in 2 months. (Choice D) Blood cultures are not indicated in this patient since the patient does not have fever or any other symptoms suggestive of febrile illness. (Choice E) Regular straight catheterization is not indicated at this point. Educational Objective: Asymptomatic bacteruria is common in elderly patients, and does not require oral antibiotic therapy if the urine WBC count is less than 20 WBC/HPF. A 6-year-old Caucasian boy is brought to the emergency department by his family. He was playing with their family dog in the backyard of their home, when the dog suddenly attacked the boy and bit his left hand. His father immediately washed the wound with copious amounts of water and brought him to the emergency department. On physical examination, there is a 1 cm x 1 cm laceration seen over the dorsum of his left hand. There is minimal amount of serous drainage from the wound. X-ray of the left hand does not reveal the presence of any foreign body in the wound. What should be done for wound closure in this patient? A. Close the wound now with nylon sutures. B. Leave the wound open to drain and observe closely. C. Close the wound with Vicryl after ensuring adequate saline irrigation. D. Do not close the wound with sutures, but put a nylon tape over the wound. E. Close the wound with a Dacron patch. Explanation: Animal bites, especially dog and cat bites, are common presenting problems in the emergency department. They are frequently caused by animals known to the victim. Animal bites can lead to a wide spectrum of injuries, ranging from minor scratches or abrasions to deep puncture wounds or lacerations. The extremities, especially the upper extremities, are common sites of injury in older children or adult patients. All animal bites, regardless of the site, should be thoroughly cleaned and irrigated with normal saline, and all devitalized tissues should be d?ided. A plain radiograph should be obtained if a patient is suspected of having a foreign body or has a bite occurring close to a bone. Most open lacerations can be closed primarily within a few hours of injury. This is especially true for injuries of the face where infections are less common due to a good vascular supply; however, bites involving the hands should not be sutured or closed primarily due to a high risk of subsequent wound infection. These should be left open to drain and examined frequently for signs of infection. Other situations where primary

closure is not recommended include puncture wounds, cat and human bites (high risk of infection), and patients presenting late after the bite. (Choices A, C, D, and E) Primary wound closure with nylon, Vicryl, or covering the wound with tape is not recommended in patients with bite wounds over the hands. These wounds should be left open to drain and examined closely for signs of infection. Educational Objective: Dog bite injuries of the hand and puncture wounds anywhere on the body should not be closed primarily due to a high risk of development of wound infection. A 10-year-old Hispanic male child without any significant past medical history is brought to the emergency department following an episode of generalized tonic-clonic seizures. His mother states that for the past year, his school performance has been deteriorating. He has been complaining of some muscle pain and cramps for the last two to three months. There is no history of trauma, fever, headache, vomiting, or cough. He has not seen a physician for the past five years. His vital signs are normal. Physical examination reveals a sedated child who is arousable on painful stimuli. He is prepubertal, without any dysmorphic features. His eye examination reveals pupils that are 3 mm in diameter and normally reactive to light. He has bilateral cataracts. His heart, lungs, and abdomen are unremarkable. He moves all his extremities upon painful stimulation. His reflexes are hypoactive; plantars are equivocal. Lab investigations reveal a calcium level of 6.0 mg/dL (albumin 3.8 g/dL), phosphorus level of 8.2 mg/dL and PTH level of 150 pg/mL (normal 10 to 65 pg/ mL). Other labs are within normal limits. Non-contrast CT scan of the head shows basal ganglion calcification. What is the most likely cause of this patient?s hypocalcemia? A. Vitamin D deficiency B. Hypoparathyroidism C. Pseudohypoparathyroidism D. Pseudopseudohypoparathyroidism (PPHP) E. Hyperphosphatemia Explanation: This patient most likely has pseudohypoparathyroidism. He has longstanding hypocalcemia with hyperphosphatemia, as evidenced by bilateral cataracts and calcification of his basal ganglia (Fahr?s syndrome). Pseudohypoparathyroidism is due to the resistance of PTH on its target tissue. There are multiple types of pseudohypoparathyroidism. Type 1A has features of Albright hereditary osteodystrophy in addition to hypoparathyroidism. Albright hereditary osteodystrophy (AHO) is characterized by short stature, round facies, short fourth and fifth metacarpals, and a short neck. This patient most likely has type 1B pseudohypoparathyroidism, which does not have features of AHO. (Choice D) Another variety of pseudohypoparathyroidism is called pseudopseudohypoparathyroidism (PPHP). PPHP patients do not have hypocalcemia and hyperphosphatemia because the resistance to PTH is mild; however, they present with features of Albright hereditary osteodystrophy. (Choice B) Patients with idiopathic or surgical hypoparathyroidism have a low calcium level, high phosphorous level, and low PTH level (remember that PTH is high in pseudohypoparathyroidism). (Choice A) V itamin D deficiency is characterized by a decrease in both calcium and phosphorus levels (hypoparathyroidism and pseudohypoparathyroidism have high phosphorus levels), along with increased PTH levels. (Choice E) Acute hyperphosphatemia can cause a decrease in calcium levels. This is classically seen in patients with seizures, tumor lysis, or acute renal failure. In the patient described above, increased phosphatase appears to be chronic, as evidenced by basal ganglion calcification and cataracts; therefore, it is unlikely that hyperphosphatemia is contributing significantly to hypocalcemia. Disorder Calcium Phosphorus PTH 25 OHD PHP Low High High Normal HP Low High Low Normal VDD Low Low High Low (PHP: pseudohypoparathyroidism; HP: hypoparathyroidism; VDD: vitamin D deficiency)

25 hydroxy vitamin D levels could be low if there is concomitant vitamin D deficiency, a common problem in subjects living in temperate regions. Educational Objective: Longstanding hypocalcemia with hyperphosphatemia, as evidenced by bilateral cataracts and calcification of basal ganglia and elevated PTH levels, indicates pseudohypoparathyroidism. This is due to the resistance of PTH on its target tissue. A 32-year-old intravenous drug abuser presented to the emergency room because of fever, dry cough, and progressive shortness of breath for the past 2 days. His oxygen saturation is 82% on room air. His initial chest x-ray showed bilateral interstitial infiltrates. His basic metabolic panel showed a BUN level of 22 mg/dL and a creatinine level of 1.1 mg/dL. His CBC showed a WBC count of 9,000/cmm, and a hemoglobin of 12.8. He was started on intravenous trimethoprim and sulfamethoxazole. After 3 days of treatment, he is symptomatically better, and his oxygen saturation is 93% on room air. His chest xray also improved. His repeat basic metabolic panel showed a BUN level of 24 mg/dL and a creatinine level of 2.3 mg/dL. His urine analysis obtained on the day of admission and on day three showed no protein, no blood, negative nitrite, negative esterase, 0-1 RBC/HPF, and 1-2 WBC/HPF. His urine output during the three days of hospitalization has averaged between 1,300-1,500 mL/24 hours. Which of the following is the most likely explanation of his abnormal basic metabolic panel? A. Acute glomerulonephritis from an underlying infection B. Acute tubular necrosis from sepsis C. Intratubular obstruction from crystallization of sulfonamides D. Allergic interstitial nephritis from sulfonamides E. Decreased clearance of the creatinine because of the trimethoprim Explanation: This patients abnormal basic metabolic panel is best explained by a decreased creatinine clearance induced by trimethoprim. Trimethoprim can inhibit tubular creatinine secretion, which results in an elevated serum creatinine level. (Choice A) This patient does not have acute glomerulonephritis. He does not have any hypertension, protein in the urine, RBC or RBC casts in the urine analysis. (Choice B) Acute tubular necrosis resulting from sepsis can cause an elevated creatinine level; however, this should also present with a decreased urine output, as well as muddy brown casts in the urine analysis. (Choice C) Sulfonamides can cause an intratubular obstruction secondary to the intratubular precipitation of sulfonamides; however distinct crystals should be seen in the urine analysis. This complication is very rarely seen with sulfamethoxazole. (Choice D) Acute interstitial allergic nephritis can result from sulfonamide therapy; however, it usually takes more than 7-10 days to develop, and is usually associated with fever, rash, joint pains, sterile pyuria, and eosinophiluria. Urine Hansel stain may be positive. Educational Objective: Remember that trimethoprim can decrease the creatinine secretion and can cause an elevated serum creatinine level without an increase in blood urea nitrogen level. The urine analysis will be normal. The following vignette applies to the next 2 items A 22-year-old Caucasian female presents to the emergency room because of a severe throbbing headache. The headache started four hours ago, was accompanied by nausea and vomiting, and was not relieved by high-dose acetaminophen, aspirin, and ibuprofen. She has had similar headaches frequently, but she says that "they had never been this severe". They are usually provoked by fatigue

and emotional stress, and can wake her up at night. She describes no aura. The headaches slowly progress to maximal intensity within an hour or two. Her mother had similar headaches that poorly responded to over-the-counter analgetics. Item 1 of 2 Which of the following features is most consistent with the diagnosis of a ?common? migraine? A. No aura B. Family history C. Night time awakening D. Poor response to NSAIDs E. Provocation by fatigue and emotional stress Explanation: The absence of an aura was formerly used to define a common migraine. Patients with an aura were diagnosed with a classic migraine. The contemporary equivalents of these terms are migraine without aura and migraine with aura respectively. Migraine without aura is more common, accounting for over 80% of migraine cases. It is not clear whether these two forms of disease have different pathophysiological mechanisms. (Choices B, C, D and E) The other listed characteristics (family history, night awakening, poor response to NSAIDs, and provocation by fatigue and emotional stress) have little value in classifying migraine headaches. Educational Objective: The presence or absence of an aura distinguishes the common migraine (migraine without aura) from the classic migraine (migraine with aura). Item 2 of 2 Which of the following is the most appropriate pharmacotherapy for this patient? A. Indomethacin B. Sumatriptan C. Methysergide D. Propranolol E. Amitriptyline Explanation: Serotonin receptor (5-HT1B/D) antagonists, such as sumatriptan and naratriptan, are currently widely employed to treat migraine attacks. They are believed to block neurogenic inflammation produced by stimulation of the trigeminal sensory C-fibers. They decrease dilatation of the blood vessels and plasma extravasation, and are effective agents for the treatment of acute migraine attacks. (Choice A) NSAIDs can be used to treat mild attacks, but would most probably be ineffective in this patient who has tried several over-the-counter agents. (Choice C) Methysergide is a serotonin antagonist that blocks 5-HT2 receptors. It is used as a prophylactic agent. (Choices D and E) Beta-blockers and tricyclic antidepressants are effective preventive agents, but have little value during an acute attack. Educational Objective: Serotonin antagonists such as sumatriptan and naratriptan act by blocking 5-HT1B/D receptors, and are currently widely employed to treat migraine attacks. A 52-year-old woman is hospitalized because of shortness of breath on exertion, abdominal distention, and a 25-pound weight loss in the last three months. The symptoms have been progressively getting worse over the last four weeks. Her other medical problems include chronic obstructive pulmonary disease, coronary artery disease, cholelithiasis, and hepatitis-C. She has smoked two packs of cigarettes daily for 20 years. She drinks 4-6 beers on a daily basis. Her mother has a history of breast

cancer. Her vital signs are within normal limits. Physical examination is unremarkable, except for the presence of massive ascites. The patient?s labs reveal: LFTs: Total bilirubin: 1.0 mg/dL Direct bilirubin: 0.8 mg/dL Alkaline phosphatase: 120 U/L Aspartate aminotransferase: 78 U/L Alanine aminotransferase: 55 U/L Amylase: 55 U/L Lipase: 123 U/L Total protein: 5.6 g/dL Serum albumin: 2.8 g/dL Ascitic fluid White cell count: 150 cells/cmm Albumin: 2.2 g/dL Amylase: 48 U/L Glucose: 32 mg/dL Which of the following is the most likely cause of ascites? A. Ovarian cancer B. Congestive heart failure C. Hepatic cirrhosis D. Budd Chiari syndrome E. Alcoholic hepatitis Explanation: Abdominal paracentesis and ascitic fluid analysis is the most reliable way to differentiate between the different causes of ascites. The most common cause of ascites in the United States is hepatic cirrhosis. Some of the other causes are cancer, congestive heart failure, peritoneal tuberculosis, nephrotic syndrome, and pancreatic disease. The differential diagnosis of ascites can be narrowed down by calculating the serum to ascitic fluid albumin gradient (SAAG). This is done by subtracting the ascitic fluid albumin value from the serum albumin value. SAAG value is useful in identifying the presence or absence of portal hypertension. SAAG value of > or = 1.1 g/dL indicates the presence of portal hypertension. Clinical conditions associated with high SAAG (> 1.1 g/dL), include cirrhosis (Choice C), congestive heart failure (Choice B), and alcoholic hepatitis (Choice E). Conditions associated with low (< 1.1 g/dL) albumin gradient; include peritoneal carcinomatosis, peritoneal tuberculosis, nephrotic syndrome, pancreatitis, and serositis. The patient in the above vignette has a SAAG of 0.6 g/dL. This, along with her clinical presentation, is consistent with a diagnosis of ovarian cancer with peritoneal spread. (Choice D) Budd Chiari syndrome is one of the postsinusoidal non-cirrhotic causes of portal hypertension. It is caused by thrombosis of the hepatic veins and/or suprahepatic inferior vena cava. The SAAG value in patients with Budd Chiari syndrome is > or = to 1.1 g/dL. Educational Objective: SAAG value of > or = 1.1 g/dL indicates the presence of portal hypertension. The following vignette applies to the next 3 items A 65-year-old male is brought to the emergency department by his wife with complaints of severe upper abdominal pain. The pain started suddenly four hours ago while they were watching their favorite television show. The pain radiates to his back and gets worse with any movement. He has had peptic ulcer disease for many years and has been on various antacids, H-2 blockers and more recently on proton pump inhibitors for symptom control. He also has hypertension, which is being controlled with a thiazide diuretic. He has an active lifestyle and walks five miles everyday. His diet mainly consists of milk and other dairy products, as they help relieve "burning" symptoms in his stomach. In the ED, his temperature is 37.8C (100F), blood pressure is 110/62 mmHg, heart rate is

110/min and respiratory rate is 22/min. He is lying flat and motionless on the bed. His mucous membranes are dry. On palpation, his abdomen is "rigid like a board." There is marked tenderness even on superficial palpation; he does not allow any further palpation of the abdomen. The lung and the heart examination are both within normal limits. Item 1 of 3 Which of the following is the diagnostic test of choice at this time? A. Electrocardiogram B. Chest X-ray C. CT scan of the abdomen D. Abdominal X-ray E. Urgent endoscopy Explanation: The patient in the above vignette has a classic presentation of perforated peptic ulcer. Even though the incidence of peptic ulcer disease has declined significantly, the incidence of ulcer complications (bleeding, perforation, penetration) has remained stable over the years. This is mainly attributed to the increased rate of nonsteroidal anti-inflammatory drug use, especially in the elderly population. Most patients have an excellent prognosis if the treatment is initiated within the first 6 hours of the onset of symptoms; however, their chances of survival decline rapidly with more than a 12-hour delay. Peptic ulcer perforation is a clinical diagnosis (i.e., largely based on the history and physical examination). Abdominal radiographs, including supine and erect films, are usually the initial test of choice in patients with suspected ulcer perforation. The presence of free air (pneumoperitoneum) on the erect abdominal film is highly indicative of perforated peptic ulcer. (Choices A and B) An ECG or chest radiograph is not required for the diagnosis of peptic ulcer perforation. (Choice C) CT scan is highly sensitive in detecting small amounts of free air; however, it can cause significant delays in the implementation of the definitive therapy, and is usually not necessary. (Choice E) Upper GI endoscopy has no role in the diagnosis & management of patients with perforated peptic ulcer. Educational Objective: Ulcer perforation should be suspected in all patients with a history of peptic ulcer disease presenting with a sudden onset of severe abdominal pain. The presence of free air on an upright abdominal radiograph is highly indicative of perforated peptic ulcer. Item 2 of 3 Which of the following is the most appropriate management for this patient? A. Admit the patient and perform an urgent endoscopy B. Admit the patient and start him on intravenous fluids C. Emergent laparotomy D. Admit the patient and start him on intravenous antibiotics E. Observe the patient in the emergency room for 24 hours Explanation: Peptic ulcer perforation is a potentially fatal complication of peptic ulcer disease. If left untreated, the patient can have a rapid clinical deterioration leading to death within 12-24 hours. On the other hand, with early diagnosis and treatment, the prognosis is generally excellent, with most patients going on to have a full recovery. Emergent exploratory laparotomy with either a simple patch closure or surgical repair of the leak is indicated in all patients with perforated peptic ulcer. This is especially important in elderly patients and patients with large leaks, as they are unlikely to respond to initial conservative therapy. (Choices B and D) Conservative management with intravenous fluids, nasogastric suction and antibiotics are useful adjunctive therapy for patients who are awaiting surgery; however, most patients should undergo emergent laparotomy with surgical repair of the perforated gut.

(Choice E) Observation in the ED alone is not enough, and the resulting delay can be potentially fatal. Educational Objective: Patients with suspected or confirmed peptic ulcer perforation should have emergent exploratory laparotomy with surgical repair of the perforation. Item 3 of 3 The appropriate step is taken for the patient, and he recovers well. Three years later, he comes to the emergency department with a one-day history of nausea, vomiting, upper abdominal pain and distension. An abdominal radiograph reveals the presence of multiple air fluid levels in the small intestine consistent with intestinal obstruction. Which of the following is the most likely mechanism of his intestinal obstruction? A. Intestinal fibrosis due to ulcer healing B. Adhesion formation in the peritoneal cavity C. Extension of the ulcer into the intestine D. Intestinal narrowing due to stricture formation E. Ogilvie?s syndrome Explanation: The patient in the vignette is presenting with signs and symptoms of small bowel obstruction. A complete obstruction of the intestinal lumen leads to dilatation of the stomach and small intestine proximal to the obstruction, thereby causing abdominal distension, nausea, vomiting and intermittent abdominal pain. Postoperative adhesion formation is the most common cause of small bowel obstruction in patients with a history of prior abdominal or pelvic surgery. Some of the less common causes include hernia, neoplasm, volvulus, intussusception, and stricture formation in patients with inflammatory bowel disease. (Choices A and D) Intestinal fibrosis or strictures can occur with ulcer healing; however, postoperative adhesions are still the most common cause of intestinal obstruction in such patients. (Choice C) Ulcer penetration generally does not lead to intestinal obstruction. (Choice E) Ogilvie?s syndrome (also known as acute colonic pseudoobstruction) is characterized by the dilation of the cecum and right colon in the absence of a mechanical obstruction to the flow of intestinal contents. It tends to involve the right side of the colon, and not the small intestine. Educational Objective: Postoperative adhesions are the most common cause of small intestinal obstruction in patients with a history of abdominal surgery. The following vignette applies to the next 2 items A 50-year-old white female was found to have a serum cholesterol level of 268 mg/dL in a local health fair. She has no disease and is physically very active. She had menopause one year ago. She has been on hormone replacement therapy for hot flashes for the last six months. She also takes adequate calcium and vitamin D supplements. Her mother has hyperlipidemia. She does not smoke or drink. Physical examination reveals a body mass index of 22 kg/m2, and is otherwise unremarkable. Item 1 of 2 Which of the following is the most appropriate next step in the management of this patient? A. Start atorvastatin. B. Increase dietary restrictions. C. Start ezetimibe. D. Discontinue hormone replacement therapy. E. Check the complete lipid profile.

Explanation: The National Cholesterol Education Program, Adult Treatment Panel III, recommends performing a complete lipid profile in a patient with a total serum cholesterol level of more than 200 mg/dL. The patient's high total cholesterol level can be due to the increase in HDL, VLDL, or LDL cholesterol fractions. Increased HDL levels are protective and do not require treatment. The primary treatments for increased VLDL cholesterol (increased triglycerides) and increased LDL cholesterol are different; therefore the exact cause of the elevated total serum cholesterol levels must be identified by checking the complete lipid profile in this patient. (Choices A, B, C, and D) Specific treatment of hypercholesteremia or hypertriglyceridemia can be instituted once a clear diagnosis of the specific hyperlipidemia is made. Until then, treatment is not warranted. Educational Objective: Obtaining a complete lipid profile is very useful in a patient with an isolated, elevated serum cholesterol level. Treatment is not started unless a diagnosis of the specific hyperlipidemia is made. Item 2 of 2 She missed the followup appointments and did not follow your recommendations. She showed up 3months later and further work-up showed: Total cholesterol: 280 mg/dL Triglycerides: 430 mg/dL HDL: 52 mg/dL In addition to advising the appropriate lifestyle and dietary changes, what is the next best step in the management of this patient? A. Start atorvastatin. B. Start gemfibrozil. C. Discontinue hormone replacement therapy. D. Start nicotinic acid. E. Ask her to drink two glasses of red wine every day. Explanation: In some patients, hormone replacement therapy can cause a significant increase in serum triglyceride levels. The increase in triglyceride levels in this patient is therefore most likely due to her hormone replacement therapy, which could also be aggravating her genetic hyperlipidemia. Discontinuation of hormone replacement therapy generally results in a significant decrease in triglyceride levels. Since there is a reciprocal relationship between serum triglyceride and HDL levels, the decrease in serum triglyceride levels is likely to cause further increase in her HDL levels. (Choice A) Atorvastatin is mainly used in patients with hypercholesterolemia. It is useful in decreasing triglyceride levels in patients who have elevated LDL levels. LDL is calculated after measuring total cholesterol, triglyceride and HDL levels, using a formula [LDL= total cholesterol ? (HDL + triglycerides/5)]; however, this formula is not very accurate when triglyceride levels are over 400 mg/dL. Since the primary abnormality in this patient is hypertriglyceridemia, atorvastatin may not be very useful in the management of hyperlipidemia in this patient. (Choice B) Gemfibrozil is not indicated at this point, since the discontinuation of hormone replacement therapy is likely to reduce her triglyceride levels. (Choice D) Nicotinic acid is useful for combined hyperlipidemia. It reduces LDL and triglyceride levels, and increases HDL levels. In this patient, nicotinic acid is likely to worsen her hot flashes; therefore, it is not the preferred agent.

(Choice E) Alcohol use is negatively correlated with coronary artery disease. This effect is likely to be secondary to the increased HDL levels and the effects of alcohol on blood coagulation. Alcohol intake could thus worsen this patient's hypertriglyceridemia. Educational Objective: Hormone replacement therapy can cause significant hypertriglyceridemia. Drug therapy is not indicated until triglycerides remain significantly elevated despite discontinuation of hormone replacement therapy. A 63-year-old Caucasian woman is hospitalized because of headaches, weakness, weight loss, heartburn, abdominal pain, abdominal distention, diarrhea, and black stools. She has had these symptoms for the past few weeks. Her other medical problems include hypertension, hypercholesterolemia, peripheral vascular disease, and bronchial asthma. Her father had prostate cancer, and her mother had colon cancer. Her medications include lisinopril, albuterol and ipratropium metered dose inhalers. She is allergic to cats. Her temperature is 37.2 C (99 F), blood pressure is 130/80 mmHg, pulse is 104/min, and respirations are 16/min. Examination shows no cardiovascular or respiratory abnormalities. The abdomen is soft, mildly tender and non-distended; bowel sounds are increased. There is no rebound tenderness or rigidity. There is no costovertebral angle tenderness. Neurologic examination is within normal limits. Fecal occult blood per rectum is positive. The patient? s labs reveal: CBC Hb: 9 g/dL Ht: 28% MCV: 76fl Platelet count: 600,000/cmm Leukocyte count: 13,000/cmm Segmented neutrophils: 76% Eosinophils: 9% Lymphocytes: 14% Monocytes: 1% Serum Chemistry Serum Na: 131 mEq/L Serum K: 4.6 mEq/L Chloride: 90 mEq/L Bicarbonate: 16 mEq/L BUN: 48 mg/dL Serum Creatinine: 1.7 mg/dL Calcium: 11.4 mg/dL Her chest-x ray shows no infiltrates, but there are some sclerotic lesions in the right humerus. Which of the following is the most likely diagnosis? A. Mastocytosis B. Whipple?s disease C. Scleroderma D. Primary hyperparathyroidism E. Multiple myeloma Explanation: The presence of sclerotic bone lesions, diarrhea, eosinophilia, and peptic ulcer disease or gastrointestinal bleeding, are essential features of systemic mastocytosis. (Choice C) Scleroderma usually doesn?t cause bone damage. (Choice D) Hyperparathyroidism would not explain the eosinophilia or diarrhea.

(Choice B) Whipple?s disease is not associated with sclerotic bone lesions. (Choice E) Multiple myeloma bone lesions are typically osteolytic, not sclerotic. Educational Objective: Systemic mastocytosis should be suspected in any patient with unexplained peptic disease, malabsorption, eosinophilia, unexplained flushing or anaphylaxis, and sclerotic bone lesions. H1 receptor antagonists, proton pump inhibitors, or cromolyn sodium are part of the initial management of this disease.

A 3-year-old boy is brought to the emergency department by his mother because he is lethargic and refuses to eat. His family recently arrived in the US as refugees from an African country. His mother hardly speaks English, and this has made history-taking very challenging. His temperature is 36.1C(97F), blood pressure is 90/60 mm Hg, pulse is 130/min, and respirations are 26/min. His height corresponds to the 38th percentile and his weight to the 5th percentile. He is lethargic and hypoactive. There is symmetrical edema of both feet. Painful, oral ulcers are observed. The patient is subsequently admitted to the hospital for rehydration and feeding. Which of the following is the best statement concerning the initial management of this patient? A. Establish IV line and start IV hydration/feeding B. Insert nasogastric tube and start hydration/feeding C. Start high-calorie feeding D. Start high-protein feeding E. Start iron supplementation Explanation: Severe malnutrition remains as one of the major health problems in developing countries. Migration of people from such regions into the United States is one of the relevant reasons for learning the basic concepts of the management of malnourished patients. In this case, the patient's history and presentation (lethargic, hypoactive, severe wasting, symmetrical lower extremity edema) suggest severe malnutrition, which is an indication for hospitalization. Initial treatment of the patient should address the following issues: temperature control (warming), possible infection, dehydration and malnutrition (feeding). Dehydration should be treated with oral rehydration whenever possible; however, insertion of a nasogastric tube (NGT) may be needed in patients with impaired consciousness, vomiting and painful oral ulcers. After the initial treatment, rehabilitation and follow-up of the patient is necessary. (Choice A) IV hydration is used in patients with severe diarrhea or shock. Its use in malnourished patients may lead to overhydration and heart failure. (Choices C and D) Feeding solutions should not initially contain excess calories and protein because the patient's gastrointestinal reserves (e.g., gastric acid production, intestinal motility, and pancreatic enzyme production) have decreased in response to his malnourished state. (Choice E) Early iron supplementation is not recommended because this may lead to gastrointestinal ulceration. Iron supplementation should begin in the rehabilitation phase. Educational Objective: The initial treatment of patients with severe malnutrition should address the following issues: temperature control (warming), possible infection, dehydration and malnutrition (feeding). Dehydration should be treated with oral rehydration whenever possible.

A 44-year-old Caucasian woman comes to your office two months after the death of her husband and complains of decreased sleep and a 2 lb weight loss. While speaking about her husband, she starts to cry. She says that she hears him calling her name during the night. She denies suicidal ideations. She still goes out with her close friends and enjoys doing so. Which of the following is the most likely diagnosis of this patient? A. Major depressive episode B. Delusional disorder C. Normal grief D. Pathologic grief E. Post-traumatic stress disorder Explanation: Normal grief reaction may include intense sadness, yearning for the deceased, and ?searching behaviors? such as auditory hallucinations. Grief usually comes in waves, and is provoked by reminders of the deceased (crying after speaking about her husband). Changes in appetite and somatic complaints are also common. These symptoms may persist several months after the event, and usually subside gradually. (Choice D) Pathologic grief occurs when the abovementioned symptoms persist after several months and do not subside. Only 6% of patients experience these symptoms thirteen months after the event. (Choice A) Depression may complicate grief reaction, and is characterized by hopelessness, helplessness, guilt, and suicidal ideations, in addition to the abovementioned symptoms. (Choice B) No delusion is present in this patient. (Choice E) Post-traumatic stress disorder develops after a life-threatening event (e.g. rape, combat experience) and is characterized by intrusive thoughts and flashbacks of this event. Educational Objective: Normal grief reaction may include intense sadness, yearning for the deceased, and ?searching behaviors? such as auditory hallucinations. Changes in appetite and somatic complaints are also common. A male infant is born at term to a 22-year-old African-American primigravida, blood group A, Rh (D)+. The prenatal course was complicated, with asymptomatic bacteriuria and mild hypertension without proteinuria in the third trimester. The newborn demonstrates yellow sclerae twelve hours after birth. He is blood group O, Rh (D)+. The total bilirubin level is 7 mg/dl, with the unconjugated bilirubin predominantly elevated. Coombs test is negative. The mother says that there is ?a kind of blood disease running in the family.? Which of the following is the most likely cause of this patient's jaundice? A. Sickle cell anemia B. HbS-HbC disorder C. Beta-thalassemia D. G-6-PD-deficiency anemia E. Physiologic jaundice Explanation: This infant presents with Coombs negative unconjugated hyperbilirubinemia in the first 24 hours of life. Hemolysis should be strongly suspected. G-6-PD deficiency is the most common red cell enzymopathy that can lead to hemolysis. It is an X-linked disorder, and should be suspected in a male infant of African, Mediterranean, or Asian descent. No triggering agent is usually present, although those infants who develop severe jaundice with G6PD deficiency usually have Gilbert's syndrome as well. (Choice E) Physiologic jaundice manifests 24 hours after birth.

(Choices A, B, and C) Other diseases producing hemolytic anemias, such as sickle cell anemia, HbSHbC disorder, and beta-thalassemia, affect the synthesis of the beta-chain of hemoglobin. They rarely manifest early in life because the predominant hemoglobin type in newborns is HbF (alpha2/gamma2), which lacks beta-chains. Educational Objective: Neonatal jaundice appearing in the first 24 hours of birth is always pathologic. Immune or non-immune hemolysis is frequently present in such patients. The following vignette applies to the next 2 items A 21-year-old Caucasian male presents to the emergency department with a history of progressive difficulty in walking for one day. He initially noticed paresthesias in his lower limbs, followed by a sense of fatigue and weakness. He can hardly walk today, and has noticed some weakness in his arms. He believes his symptoms developed while camping. He recalls taking a long hike through the woods. He denies any trauma or any recent illness (i.e., any history of headache or fever). His past medical history is insignificant. He denies the use of tobacco, alcohol, or any recreational drug use. His temperature is 36.7C(98F), blood pressure is 120/76 mm Hg, pulse is 90/min, and respirations are 16/min. Physical examination reveals prominent symmetrical paresis of the lower extremities and milder paresis of the arms. Deep tendon reflexes are decreased on both sides. A lumbar puncture is performed, and CSF analysis shows no abnormalities.

Item 1 of 2 Which of the following would be most helpful in discovering the cause of this patients condition? A. Urine drug screen B. Nerve conduction studies C. Tensilon (edrophonium) test D. Careful skin examination E. Test for Clostridium botulinum toxin Explanation: The clinical scenario described is most consistent with paralysis that is caused by an attachment of a neurotoxin-secreting tick. Although this disease has been reported worldwide, most cases occur in North America and Australia. The tick species that cause most cases in the United States and Canada are Dermacentor andersoni (the Rocky Mountain wood tick), and D. Variabilis (the American dog tick). Symptoms develop 5-6 days after a female tick attaches to the patient. The typical presentation is a progressive ascending paralysis that occurs over a matter of hours to days. Complete paralysis may develop in severe cases. Fever is typically not present in tick-borne paralysis. Pupillary abnormalities are uncommon in tick paralysis. If the patient has a history of fever or prodromal illness, the diagnosis is unlikely. Finding the attached tick on the skin is the most important diagnostic measure. The differential diagnoses for tick paralysis include Guillain-Barre syndrome (GBS), myasthenia gravis, and botulism. (Choices B and C) Although GBS and other spinal cord tumors also present with ascending progressive paralysis, this occurs in a matter of days to weeks. On the other hand, ascending paralysis is not typical for myasthenia gravis and botulism. Unlike Guillain-Barre syndrome or poliomyelitis CSF examination is normal in tick paralysis. (Choice E) Botulism usually usually causes a descending paralysis, cranial nerves are affected early in the course, and pupillary abnormalities are common. (Choice A) Toxicological screening will not be helpful in this case. Educational Objective: The first symptom of tick paralysis is usually an unsteady gait that progresses to ascending paralysis. The typical presentation is a progressive ascending paralysis that occurs over a matter of hours to days. Fever is typically not present in tick-borne paralysis. Pupillary abnormalities are uncommon in tick paralysis. Item 2 of 2 Which of the following is the best way to manage this patient? A. Remove the offending agent. B. Administer anti-toxin. C. Consider IV immunoglobulin. D. Consider acetylcholinesterase inhibitors. E. Consider plasma exchange . Explanation: After diagnosing a patient with tick paralysis, the physician should carefully search for the offending agent. Extra vigilance is required when searching the following areas: the scalp, the axillae, the ears, buttocks and interdigital spaces. In most cases, removal of the tick(s) will cause a substantial improvement of the paresis within several hours. Sometimes, the paralysis may worsen for 24-48 hours after the removal of the tick (typically I. Holocyclus). In such cases, careful observation and supportive therapy should be provided. (Choice B) Anti-toxin therapy is used for the treatment of botulism (Choices C and E) IV immunoglobulin and plasma exchange have been proven to be effective in patients with Guillain-Barre syndrome.

(Choice D) Acetylcholinesterase inhibitors are used in the treatment of patients with myasthenia gravis. Educational Objective: In most cases of tick paralysis, removal of the tick(s) will cause a substantial improvement of the paresis within several hours. The following vignette applies to the next 2 items A 40-year-old woman comes to the clinic after she noticed a small swelling in the front of her neck while dressing up. She denies any compressive symptoms. She also denies symptoms suggestive of hyper- or hypothyroidism. Physical examination reveals a 2.5 x 2.2 cm nodule in her left thyroid lobe. There is no history of neck irradiation when she was a child or family history of thyroid illness. She is currently on no medication. The rest of the physical examination is unremarkable. Item 1 of 2 Which of the following is the next best step? A. FNAC of the thyroid nodule B. Radionuclide scan C. TSH levels D. Chest x-ray E. CT scan of the neck Explanation: Measurement of the TSH level is the first best step in the management of nodular thyroid disease. This will assess the functional status of the thyroid gland. Suppression of ultra-sensitive TSH is suggestive of thyrotoxicosis that can be due to excessive secretion of thyroid hormones from nodule(s). In such a case, radionuclide thyroid scan is the next best step to evaluate the functional status of this nodule. If the nodule is hot (increased uptake in the nodule and suppressed uptake in rest of the thyroid gland), the chances of malignancy within this nodule are extremely low. An elevated TSH level is suggestive of hypothyroidism, which could be due to Hashimoto?s thyroiditis. Many patients with Hashimoto?s thyroiditis have irregularities in their thyroid gland due to focal accumulation of the inflammatory cells. Treatment with levothyroxine rapidly leads to a decrease in thyroid size and disappearance of this nodularity. (Choice A) FNAC is not generally required in hot nodules. If the TSH level is normal, then FNAC of the thyroid nodule is the most appropriate step. All thyroid nodules over 1 cm in diameter should be biopsied if the TSH level is within normal limits. The final management decision can be made depending on the results of FNA, cosmetic requirement and/or compressive symptoms. Majority of the thyroid nodules are benign, and even benign nodules grow with time. (Choice B) Radioactive iodine uptake (RAIU) is not routinely used in most centers in the United States; however, a few endocrinologists and internists still use RAIU and scan as the primary modality to evaluate thyroid nodular disease. All nodules that are cold (decreased uptake within the nodule and normal uptake in the rest of the thyroid gland) are biopsied. Hot nodules are not usually biopsied and investigated for thyrotoxicosis. (Choice D) Chest x-ray is usually not done, unless retrosternal extension of a large thyroid nodule is suspected. Chest x-ray can also be performed to evaluate for metastasis of thyroid cancer. (Choice E) CT scan of the neck is less sensitive than thyroid ultrasound to evaluate nodular thyroid disease. CT scan is used for retrosternal goiter and in patients with compressive symptoms. Educational Objective: Measurement of the ultra-sensitive TSH level is the first step in the evaluation of nodular thyroid disease. FNAC of nodules that are greater than 1 cm in diameter is performed in all patients with

normal TSH levels. Radioactive nuclide scan is done if TSH is suppressed; this is done to look for hot nodules. Ultrasonography is more accurate than CT scan for the evaluation of thyroid nodules. Item 2 of 2 After performing the appropriate investigation, a diagnosis of papillary thyroid cancer is made. What is the next best step in the management of this patient? A. Staging of the cancer B. Subtotal thyroidectomy C. Near total thyroidectomy D. Radioactive iodine E. High-dose thyroxin Explanation: Near total thyroidectomy (NTT) is the preferred treatment for patients with papillary thyroid cancer. By taking out most of the thyroid tissue, it becomes easier to destroy the residual thyroid tissue (small thyroid tissue left after surgery) by giving high-dose radioactive iodine. Since thyroglobulin is produced only by thyroid tissue, it is used as a tumor marker for following patients after NTT and radioactive iodine treatment. After destroying all the thyroid tissue by NTT and radioactive iodine, the serum thyroglobulin level should be undetectable. Detectable serum thyroglobulin following NTT and radioactive iodine treatment suggests residual disease. (Choice A) Staging of papillary thyroid cancer is not performed preoperatively. Even with the presence of metastasis, most papillary thyroid cancers are slow growing, and the prognosis is usually very good. Metastases usually take up radioactive iodine. Total body radioiodine uptake and scan is done following the administration of large dose radioactive iodine for destroying post NTT residual thyroid tissue. (Choice B) Subtotal thyroidectomy is not used in the treatment of papillary thyroid cancer. Thyroid lobectomy may occasionally be performed in low-risk patients with papillary cancer of less than 1 cm in diameter; however, it is not possible to follow thyroglobulin levels as a tumor marker when only part of the thyroid is removed, and a large amount of residual thyroid gland is intact. Furthermore, if a large amount of residual thyroid tissue is present after surgery, it is extremely difficult to destroy this residual tissue with radioactive iodine. (Choice D) Radioactive iodine is given once NTT has been performed. Radioactive iodine treatment is never used as a primary modality. Multiple high doses of radioactive iodine may be required to treat thyroid metastasis if the thyroglobulin level is high or if the radioactive iodine scan shows metastasis. (Choice E) Giving high-dose levothyroxine is used following thyroidectomy and radioactive iodine treatment of papillary and follicular thyroid cancers. The goal is to suppress TSH below the normal range, which creates a state of subclinical thyrotoxicosis. (TSH is a growth factor for papillary thyroid cancer; therefore, lowering the level of TSH below normal can inhibit the growth of thyroid cancer.) However, subclinical thyrotoxicosis is associated with an increased risk for atrial fibrillation and rapid bone loss. Educational Objective: NTT is the preferred treatment for most patients with papillary thyroid cancer. Radioactive iodine is given following total thyroidectomy to destroy the residual thyroid. Total body scan is performed following radioactive iodine treatment to look for uptake in the thyroid region and metastatic disease. Suppressive doses of levothyroxine are used to suppress the TSH levels to below the normal range, thereby helping to inhibit the growth of thyroid cancer. The following vignette applies to the next 2 items A 36-year-old Caucasian man comes to the emergency department and complains of severe pain in his left hand and arm. He is the captain of an ice hockey team and is concerned because he has an important game coming up next week. He also works as a carpenter, and recalls that he obtained a small puncture wound from a wood splinter yesterday while working in the workshop. He washed the

wound and put a Band-Aid on it at that time. The next morning, he woke up and felt severe pain in his arm. The pain has worsened since then. He denies any other medical problems and does not take any medications regularly. His temperature is 38.3?C (103F), heart rate is 96/min, respiratory rate is 16/min, and blood pressure is 90/60 mmHg. Physical examination of his left upper extremity reveals slight erythema and swelling of the forearm and distal half of the left upper arm with mild tenderness on palpation. There are two blisters on the posterior aspect of his proximal forearm. There is no crepitus on local palpation. Item 1 of 2 Which of the following is the most likely organism that is responsible for this condition? A. Group A streptococci B. Group C streptococci C. Staphylococcus aureus D. Staphylococcus epidermidis E. Clostridium perfringens Explanation: Necrotizing fasciitis is a fulminant infection of the subcutaneous tissue that spreads along the fascial planes and leads to extensive tissue necrosis and destruction. Two types of clinically distinct necrotizing fasciitis have been described. Type I necrotizing fasciitis is usually seen in patients with underlying diabetes and peripheral vascular disease. It is caused by a mixture of aerobic and anaerobic microorganisms. Some commonly isolated organisms include Staphylococcus aureus, Bacteroides fragilis, Escherichia coli, group A streptococci, Prevotella species, etc. In contrast, type II necrotizing fasciitis usually occurs in patients with no concurrent medical illness. Many patients report a history of laceration, blunt trauma, surgical procedure, or long history of intravenous drug use as a predisposing factor. It is typically caused by Group A streptococci (Streptococcus pyogenes). Severe pain in the absence of significant skin changes is the initial clinical presentation. The infection spreads rapidly along the fascial plane with erythema, blisters and bullae formation, fever, malaise, systemic toxicity and shock, developing in a rapid sequence. In some cases, the infection can lead to extensive tissue necrosis and destruction with minimal skin changes. (Choice B) Group C streptococci are one of the less frequent causes of cellulitis and septic arthritis. It is not associated with the development of necrotizing fasciitis. (Choice C) Staphylococcus aureus can be seen as a part of polymicrobial flora in diabetic patients with type I necrotizing fasciitis. It is not usually associated with the development of necrotizing fasciitis by itself. (Choice D) Staphylococcus epidermidis is not associated with necrotizing fasciitis. (Choice E) Clostridium perfringens cellulitis is usually preceded by a local skin disruption providing a port of entry for the Clostridium spore. The area typically has gas in the skin; however, muscle and deep fascia are spared. Educational Objective: Group A streptococci (GAS) is the causative agent for rapidly spreading cellulitis and necrotizing fasciitis in most healthy young patients. Item 2 of 2 Which of the following is the most appropriate next step in the management of this patient? A. Obtain a plain radiograph of the left upper limb B. Obtain a CT scan of the left upper limb C. Obtain blood cultures, start intravenous penicillin and observe D. Start the patient on intravenous clindamycin and prepare him for urgent surgical debridement E. Start the patient on intravenous vancomycin and prepare him for urgent surgical debridement Explanation:

Necrotizing fasciitis is a rapidly spreading infection of the subcutaneous tissue with extensive tissue necrosis and destruction. If left untreated, it rapidly causes systemic toxicity and hemodynamic collapse. A clinical suspicion of necrotizing fasciitis warrants urgent, aggressive surgical exploration and debridement of the involved tissue to prevent morbidity and mortality. These measures will also help to ascertain the diagnosis by providing tissue samples for diagnostic purposes. In addition to the surgical exploration, appropriate antibiotic therapy and hemodynamic support should be provided to all patients with necrotizing fasciitis. Clindamycin is the drug of choice for the treatment of necrotizing fasciitis/myonecrosis due to GAS. (Choices A and B) A plain radiograph and CT scan may be useful in detecting the presence of gas in the subcutaneous tissue; however, surgical therapy should not be delayed in order to obtain radiologic imaging. (Choice C) Early surgical exploration and debridement are the most critical measures in the management of patients with necrotizing fasciitis. Antibiotics alone are not recommended. Blood cultures are positive in around 60% of the patients. (Choice E) Most infections are due to group A streptococci, and these are typically treated with penicillin or clindamycin. Vancomycin is not the appropriate choice. Educational Objective: Surgical exploration and debridement, appropriate intravenous antibiotics, and hemodynamic support should be provided to all patients with necrotizing fasciitis. Therapy should not be delayed to obtain imaging or laboratory studies. Clindamycin is the drug of choice for the treatment of necrotizing fasciitis/myonecrosis due to GAS. A 25-year-old Caucasian woman, gravida 2 para 1, presents to your office at 20 weeks gestation for a routine prenatal visit. She is known to be D (-) while her husband is D (+). During her first pregnancy, she received a standard dose of anti-D immune globulin at 28 weeks gestation and immediately postpartum; she also had a significant intrapartum placental abruption, which did not require caesarian delivery. Her most recent anti-D antibody titers are 1:34. Which of the following is the most likely explanation of the positive antibody screen of this patient? A. No prophylaxis early in this pregnancy B. Too early administration of anti-D immune globulin postpartum C. Low dose of anti-D immune globulin at 28 weeks of her first pregnancy D. Low dose of anti-D immune globulin postpartum E. No prophylaxis between the pregnancies Explanation: A low dose of anti-D immune globulin postpartum is the most likely cause of anti-D immunization in this patient. Events that are associated with feto-maternal hemorrhage (such as placental abruption) may require adjustments in the dosage of anti-D immune globulin; therefore, the presence and the amount of feto-maternal transfusion should have been determined in this patient during her first pregnancy. The rosette test is a qualitative test that helps determine the presence of feto-maternal hemorrhage. If negative, the standard dose of anti-D immune globulin should be administered. If positive, the amount of hemorrhage can be evaluated using Kleihauer-Betke stain or fetal red cell stain using flow cytometry, and the dose of anti-D immune globulin should be corrected accordingly. (Choice C) A standard dose of anti-D immune globulin should be administered at 28 weeks of an uncomplicated pregnancy. (Choice A) There is no need for earlier prophylaxis because the risk of alloimmunization before 28 weeks is very low. (Choice E) Prophylaxis between pregnancies is rarely necessary (e.g., after accidental transfusion of Rh-positive blood). (Choice B) Too late, not too early administration of anti-D immune globulin postpartum may result in failure of anti-D immune globulin prophylaxis.

Educational Objective: Failure to adjust the dose of anti-D immune globulin after events that are associated with excessive feto-maternal hemorrhage (e.g., placental abruption) may result in maternal alloimmunization. A 23-year old Caucasian primipara gives birth to a full-term, female infant who has an isolated cleft lip. The mother says that her brother had a cleft lip and cleft palate which were surgically corrected. Her prenatal course was uncomplicated. She did not take any medications except multivitamins during the pregnancy. She does not smoke or consume alcohol. Careful inspection reveals an upper lip pit in the mother. Which of the following is the best statement concerning the infant?s condition? A. The condition is most likely X-linked in this family B. The expressivity of the condition in the family is close to 100% C. The recurrence risk in the following pregnancies is low D. Reconstruction is generally performed at 10 weeks of age E. Associated cardiac and renal abnormalities are common Explanation: Cleft lip with or without cleft palate is typically a multifactorial disorder. It has been associated with the use of teratogenic agents (commonly alcohol) during pregnancy. A careful and thorough history is important in making the correct diagnosis. Reconstruction of the cleft lip is generally performed at approximately three months of age, according to the rule of ?10:? 10 lbs of weight, 10 weeks of age, and 10 g of hemoglobin. (Choices A and B) The modes of inheritance can be autosomal dominant, autosomal recessive, and Xlinked. This family seems to have an autosomal dominant inheritance, called Van der Woude syndrome, with variable expressivity because the mother has a very limited expression of the disease ? an upper lip pit. (Choice C) The recurrence risk in the following pregnancies is close to 50%. (Choice E) Associated cardiac and renal abnormalities are not common. Educational Objective: Cleft lip with or without cleft palate is typically a multifactorial disorder. It has been associated with the use of teratogenic agents during pregnancy, and the modes of inheritance can be autosomal dominant, autosomal recessive, and X-linked. Reconstruction of the cleft lip is generally performed at approximately three months of age, according to the rule of 10 10 lbs of weight, 10 weeks of age, and 10 g of hemoglobin. A 46-year-old obese male presents to the emergency department with acute right flank pain and hematuria. He has type 2 diabetes mellitus, gastroesophageal reflux disease (GERD), and hypertension. He is currently on glyburide, metformin, acarbose, atenolol, ramipril, and cimetidine. Physical examination reveals a grossly overweight male in moderate distress. His blood pressure is 160/100 mmHg, pulse is 100/min, temperature is 36.7C(98F), and respirations are 24/min. He has significant right flank tenderness. The rest of the physical examination, including examination of ocular fundi, is within normal limits. His lab investigations revealed the following: CBC Hb: 13.4g/dL Hct: 42% WBC: 12,500/cmm Platelet count: 230,000/cmm Serum chemistry Serum Na: 138mEq/dL Serum K: 3.9mEq/dL BUN: 16mg/dL Creatinine: 1.9mg/dL

Calcium: 8.9mg/dL Glucose: 242mg/dL Uric acid: 8.5mg/dL Urinalysis Protein: Negative Blood: Trace Nitrite: Negative Esterase: Negative WBC: 5/hpf RBC: 2/hpf (normal shape) Casts: None Bacteria: None Spiral CT scan shows a 4 mm stone in the right uretero-pelvic junction. The kidneys and renal pelvis are normal bilaterally. What is the most likely cause of his increased serum creatinine? A. Obstructive uropathy B. Decreased tubular secretion of creatinine C. Diabetic nephropathy D. Hypertensive nephropathy E. Glomerulonephritis Explanation: This patient has an isolated elevation in serum creatinine, which may be due to the use of cimetidine. Cimetidine can cause a decrease in the tubular secretion of creatinine, which leads to decreased creatinine clearance. Other drugs that can cause a decrease in the tubular secretion of creatinine are probenecid and trimethoprim. (Choice A) The patient does not have obstructive uropathy because he has a single small stone in his right ureteropelvic junction without any features of obstructive uropathy on CT scan. (Choices C & D) The patient is unlikely to be suffering from diabetic or hypertensive nephropathy. Aside from the increased creatinine level, there is no other evidence of chronic complications of diabetes such as retinopathy or neuropathy. Isolated nephropathy as a diabetic complication is highly unlikely. Furthermore, azotemia due to diabetic or hypertensive nephropathy will present with the synchronous elevation of BUN and creatinine levels. The patient described above has isolated elevation of serum creatinine levels. (Choice E) Patients with glomerulonephritis have dysmorphic red blood cells (RBCs), and RBC casts are typically seen in their urine. This patient has normally appearing RBCs in his urine; therefore, a diagnosis of glomerulonephritis is unlikely. Educational Objective: Isolated elevation in serum creatinine can be caused by the decreased tubular secretion of creatinine, which is induced by certain drugs (i.e., cimetidine, probenecid, and trimethoprim). A 62-year-old Caucasian man comes to the office with complaints of a dull, non-throbbing headache, hearing loss on the left side, and persistent tinnitus. His symptoms have been getting progressively worse for the last two weeks. He has a history of small cell cancer of the lung, which was treated with a combination of chemotherapy and radiotherapy four months ago. His lung mass has reduced considerably in size after its treatment. A contrast-enhanced magnetic resonance imaging scan shows a 1 x 2 cm circumscribed mass in the left cerebellopontine angle compressing the eighth cranial nerve. There is another small 0.5 x 1 cm mass in the left frontal lobe cortex with significant edema surrounding the lesion. Which of the following is the most likely cause of this patient?s symptoms? A. Acoustic neuroma

B. Medulloblastoma C. Metastatic cancer D. Meningioma E. Astrocytoma Explanation: Brain tumors are classified into tumors that originate in the brain itself (primary brain tumor), and those that originate elsewhere in the body and metastasize to the brain from another primary site (secondary brain tumor). Secondary brain tumors are the more common type of brain tumors. The common primary sites of origin of brain metastasis, in the order of frequency, are: lung, breast, unknown primary, melanoma, and colon cancer. Small cell lung cancer has a predilection for early metastasis to the brain. The most common route of spread of the cancer is by hematogenous spread. The diagnostic imaging of choice to evaluate a patient with suspected metastatic disease is a contrast-enhanced MRI scan. The clues to the presence of a metastatic brain lesion (as compared to a primary brain tumor) on radiographic imaging are the presence of multiple, well-circumscribed lesions, and a relatively large amount of vasogenic edema as compared to the size of the lesion. In the above vignette, the patient's symptoms is most likely being caused by a metastatic lung cancer which has compressed the eighth cranial nerve. Educational Objective: Metastatic lesions to the brain are the most common cause of brain tumors. These account for more than half of all the brain tumors seen in adult patients. A 69-year-old African American man is diagnosed with benign prostatic hyperplasia after complaining of increased urinary frequency and urgency, hesitancy, incomplete bladder emptying, and decreased force of stream. A course of medical therapy is attempted, but the patient experiences no relief, and transurethral resection of the prostate is then recommended. The procedure is performed and the obstruction is successfully relieved; however, six hours later, the patient becomes nauseous, disoriented, and experiences occasional twitching. His temperature is 37.0C (98.6F), blood pressure is 108/58 mm Hg, pulse is 88/min, and respirations are 18/min. What abnormal finding is most likely to be noted on his laboratory panel? A. Hyperkalemia B. Hypernatremia C. Hypocalcemia D. Hypokalemia E. Hyponatremia Explanation: Transurethral resection of the prostate (TURP) is frequently associated with the use of 20-30 liters of isosmotic flushing solutions that lack sodium but contain glycine, sorbitol, or mannitol. Up to three liters of this fluid may enter the circulation by either direct entrance into the large prostatic veins or by leaking into the retroperitoneal space through the opened prostatic capsule. As a result, the plasma sodium concentration may fall below 100 mEq/L, causing nausea, confusion, disorientation, twitching, seizures, and hypotension. Hyponatremia can be confirmed by establishing the presence of an osmolal gap that can exceed 30-60 mosmol/kg. (Choice A) The major causes of hyperkalemia are typically classified as secondary to increased potassium release from cells (e.g., pseudohyperkalemia, metabolic acidosis, hyperglycemia, increased tissue catabolism, beta-blockade, exercise) or reduced urinary potassium excretion (e.g., hypoaldosteronism, renal failure, volume depletion, renal tubular acidosis type 1, ureterojejunostomy). (Choice B) The major causes of hypernatremia are typically classified as secondary to unreplaced water loss (e.g., insensible and sweat losses, gastrointestinal losses, diabetes insipidus, osmotic

diuresis, hypothalamic lesions), water loss into cells (e.g., severe exercise, seizures), or sodium overload (intake of hypertonic sodium solutions). (Choice C) The major causes of hypocalcemia are typically classified as secondary to loss of calcium from the circulation (e.g., hyperphosphatemia, acute pancreatitis, osteoblastic metastases, intravascular complexing with certain substances, acute respiratory alkalosis), hypoparathyroidism, magnesium metabolism disorders, vitamin D deficiencies, sepsis, or fluoride intoxication. (Choice D) The major causes of hypokalemia are typically classified as secondary to decreased potassium intake, increased entry into cells (e.g., extracellular pH increase, increased insulin, increased beta-adrenergic activity, hypothermia), increased gastrointestinal losses, increased urinary losses, increased sweat losses, dialysis, or plasmapheresis. Educational Objective: Transurethral resection of the prostate (TURP) is frequently associated with hyponatremia secondary to the addition of isosmotic flushing solutions to the circulation. A 30-year-old African-American woman is brought to the emergency department after a gunshot wound to her pelvis. She is screaming in severe pain. She is diaphoretic and anxious. There is very little external bleeding. Which of the following is the earliest sign of hypovolemia? A. Increased heart rate B. Decreased arterial blood pressure C. Dry mucous membranes D. Decreased skin turgor E. Decreased urine output Explanation: Tachycardia is the immediate physiologic response to volume depletion, and is the earliest sign of hypovolemia. (Choice B) In patients with hypovolemia, the arterial blood pressure is initially normal; postural hypertension will then develop, followed by persistently low blood pressure. (Choices C and D) Dry mucous membranes and decreased skin turgor are generally the features of interstitial fluid depletion; however, these signs do not develop rapidly. (Choice E) Decreased urine output is another sign of hypovolemia resulting from salt and water retention; however, it is not the earliest sign. Educational Objective: Tachycardia is the earliest sign of hypovolemia. A 32-year-old Caucasian man comes to see you in the office for smoking cessation counseling. He works as an executive in a multinational financial corporation, and has recently moved into your city. He exercises regularly. He does not have any medical issues. He has a 20-pack-year history of smoking, and currently smokes two packs of cigarettes daily. He got married two years ago, and ever since, he has been trying to quit smoking on his own without any success. He tried over-the-counter nicotine gum approximately one year ago, and was able to stop smoking for two months. He claims that the stress at his workplace became too much for him to handle during that time, and he had to start smoking again. He appears very determined this time, and says that he will do whatever it takes to quit smoking. Which of the following is the most appropriate next step in the management of this patient?

A. Prescribe bupropion for six to eight weeks. B. Ask the patient to join a group counseling program. C. Advise him to try quitting slowly by cutting back one cigarette a day. D. Provide him with intensive smoking cessation counseling along with a nicotine patch. E. Warn him that he may potentially die from the hazards of cigarette smoking. Explanation: Cigarette smoking is the most common cause of preventable deaths. Smoking cessation is associated with significant short and long-term health benefits, and should always be encouraged in all patients

during routine office visits. Because cigarette smoking is highly addictive, cessation causes significant withdrawal symptoms. These may include insomnia, irritability, anxiety, frustration, restlessness, mild depression, or weight gain. These withdrawal symptoms and associated intense craving for cigarettes make the attempts at quitting very difficult. Most smokers typically make several attempts to quit before they are able to quit successfully. Primary care physicians frequently come across patients who have already made several efforts to quit smoking without success. Current recommendations for patients with a history of relapse are to use a combination of intense behavioral counseling along with nicotine replacement therapy (nicotine gum or patch) or bupropion. These are based on studies which have demonstrated a higher rate of successful quitting when a combination approach involving intense behavioral counseling and pharmacotherapy is used. (Choices A and B) The combination of intensive behavioral or group counseling and nicotine replacement or bupropion has a higher likelihood of success than either modality used alone. (Choice C) Abrupt cessation (cold-turkey approach) of cigarette smoking is the preferred approach in most smoking cessation programs. (Choice E) The patient appears to be well aware of the adverse effects of cigarette smoking and does not need a reason to quit. He is seeking help to find a way to successfully quit. Educational Objective: A combined approach involving intensive behavioral counseling and pharmacotherapy (with nicotine replacement therapy or bupropion) should be offered to all patients who are motivated to quit smoking. A 63-year-old accountant is brought to the emergency department after suddenly collapsing at his desk at work. He is unconscious upon arrival, but regains consciousness within several minutes. His medical history is significant for stable angina, hypertension, and hypercholesterolemia. He has had no previous surgeries. His medications include atenolol, simvastatin, aspirin, and a multivitamin. His physical examination is remarkable for paralysis of the right upper and lower extremities. Vibration and position sense are absent on the right side. When the flat of the right foot is stroked with a pen, the right great toe goes up and the other toes fan out. The patient?s tongue deviates to the left upon protrusion. Given these findings, a lesion in which region of the brain is most likely? A. Lateral pons B. Medial pons C. Lateral medulla D. Medial medulla E. Central midbrain Explanation: The analysis of a patient?s signs and symptoms can allow for localization of lesions within the central nervous system, and some broad generalizations apply to this categorization process. Brainstem lesions typically involve the cranial nerves and sensory loss of one half of the face and the contralateral half of the body. Lesions of the thalamus or cortex are associated with sensory loss of one half of the face and the same half of the body. Cortical lesions are associated with findings specific to the cerebral cortex (e.g., aphasia, neglect, abnormal graphesthesia or stereognosis). (Choice D) An unusual condition, the medial medullary syndrome is typically caused by an occlusion of the vertebral artery or one of its branches. As a result, the patient will demonstrate contralateral paralysis of the arm and leg, contralateral loss of tactile, vibratory, and position sense, and tongue deviation to the injured side.

(Choice A) A lesion of the lateral pons can cause lateral midpontine syndrome, which results in impaired sensory and motor function of CN V (the trigeminal nerve) with accompanying limb ataxia. (Choice B) An ischemic lesion of the medial pons can cause medial midpontine syndrome. Common characteristics include ipsilateral limb ataxia and contralateral eye deviation and paralysis of the face, arm, and leg. Impairment of touch and position sense is variable. (Choice C) A lesion of the lateral medulla may result in well-known abnormalities such as Wallenberg syndrome. This syndrome typically involves an ipsilateral Horner syndrome, loss of pain and temperature sensation of the face, weakness of the palate, pharynx, and vocal cords, and cerebellar ataxia. There is also loss of pain and temperature sensation on the contralateral side of the body. (Choice E) Lesions in the central midbrain can cause a number of different syndromes, including Weber?s syndrome, Benedikt?s syndrome, Claude?s syndrome, Nothnagel?s syndrome, and Parinaud? s syndrome. Oculomotor paresis and other abnormalities of CN III function are common to all of these syndromes, and cerebellar ataxia and contralateral hemiplegia may be noted as well. Educational Objective: The medial medullary syndrome is typically associated with contralateral spastic hemiplegia, contralateral vibratory and proprioception loss, and tongue deviation to the injured side. *Extremely important question for USMLE step-3 A 58-year-old female is admitted for an elective left knee replacement. The patient has hypertension, chronic obstructive pulmonary disease, osteoporosis, and GERD. She is currently on ramipril, amlodipine, metformin, multivitamins, calcium carbonate, pantoprazole (Protonix), and raloxifene. She is also on daily salmeterol and ipratropium bromide as metered-dose inhalations. Aside from metformin, which of the following medications should be discontinued before subjecting this patient to surgery? A. Raloxifene B. Ramipril C. Amlodipine D. Protonix E. Calcium carbonate Explanation: Raloxifene is a selective estrogen receptor modulator (SERM) which selectively stimulates estrogen receptors on bone cells. It has been shown to improve bone mineral density at both the hip and lumbar spine. Studies have not shown that it reduces the risk for vertebral fractures. Side effects include hot flashes and an increased risk for deep venous thrombosis and pulmonary embolism. Raloxifene should therefore be discontinued at least 72 hours prior to an elective surgical procedure to prevent deep venous thrombosis. It can be restarted after the patient is fully ambulatory. (Choices B, C, D, and E) There is no reason to discontinue amlodipine, ramipril, Protonix, and calcium carbonate. Educational Objective: Raloxifene can predispose a patient to DVT and should be discontinued in immobilized patients. A 24-year-old female is brought to the emergency department (ED) after an episode of witnessed seizure activity in her home. Her mother says that she has a history of generalized tonic-clonic seizures. She has been seizure-free for the past six years. She stopped taking all her medications two years ago. Her only current daily medication includes a multivitamin and an oral contraceptive pill. At the ED, she is given a loading dose of IV fosphenytoin and started on maintenance doses of oral phenytoin. Over the next three days, she does not have any further seizures. On the fourth day, she has devloped a horizontal nystagmus. Her total phenytoin level on the same day is 20 mcg/mL (normal range is10-20 mcg/mL). Which of the following is the most appropriate next step?

A. Stop phenytoin immediately. B. Stop phenytoin and repeat the drug levels. C. Observation alone. D. Discontinue the oral contraceptive pill. E. Reduce the dose of phenytoin. Explanation: It is important to recognize the early signs and symptoms of phenytoin toxicity. The earliest sign is the presence of nystagmus on far lateral gaze. Some other effects include blurred vision, diplopia, ataxia, slurred speech, dizziness, drowsiness, lethargy, and decreased mentation, which progresses to coma. Systemic side effects and neurotoxicity is one of the major limitations to the use of phenytoin. Usual therapeutic range of phenytoin is between 10-20 mcg/mL and most patients will experience adverse dose related neurotoxic effects with levels greater than 20 mcg/mL. However, the serum levels associated with neurotoxicity vary from patient to patient. Some patients can experience toxic effects even when the measured levels are within the normal therapeutic range. The first step in the management of side effects due to higher drug levels is to reduce the dose or alter the treatment schedule to minimize the peak drug levels. Therefore, in this vignette, the dose of phenytoin should be reduced and patient should be observed for resolution of nystagmus. (Choices A and B) Abrupt cessation of an antiepileptic drug is not recommended. Any antiepileptic drug that needs to be withdrawn due to intolerable side effects should be tapered gradually (over days to weeks) to prevent seizure relapse. (Choice C) The dosage of the drug should be reduced to prevent further toxicity. Observation alone is not appropriate. (Choice D) Oral contraceptive pills do not affect the drug level of phenytoin. Conversely, chronic phenytoin therapy can cause failure in contraception by enhancing the metabolism of oral contraceptive pills through the induction of hepatic enzymes. Educational Objective: Patients with signs and symptoms of phenytoin toxicity should be initially managed by altering the drug dosage or treatment schedule to minimize drug peak levels. A 25-year-old male machinery operator is brought to the emergency department by an ambulance immediately after sustaining chemical burns to his hands and arms. He was working in a nearby industrial warehouse, when a carton containing an unknown chemical powder was knocked over onto him. Much of the powder remains on his hands and arms. He is in great pain, and is unable to answer any further questions. What is the appropriate initial intervention? A. Rinse the powder off immediately with copious cold water B. Rinse the powder off immediately with sterile saline C. Rinse the powder off immediately with sterile Ringer?s lactate D. Rinse the powder off immediately with neutralizing solution E. Brush the powder off immediately Explanation: When an incident involving exposure to hazardous materials occurs, it is important for emergency personnel to accomplish two goals: the treatment of exposed individuals, and the containment of any remaining hazardous materials. Unknown dry chemicals should always be brushed off first. (Choice A) Only after the remaining visible powder is removed should the area be irrigated with copious amounts of low-pressure water for 15-30 minutes. Unknown liquid chemicals, in contrast, are simply immediately washed off with water. The victim?s clothing should be promptly removed and stored in a plastic bag, although irrigation should begin before the victim is undressed. (Choices B and C) Rinsing the powder off with sterile saline or sterile Ringer?s lactate is not recommended. Cold water is the agent of choice once the powder has been brushed away from the skin. (Choice D) Rinsing the powder off with a neutralizing solution is inadvisable, even if the powder is known to be an acid or alkali. Animal studies have demonstrated that animals with acid or alkali

burns that were washed with water survived longer than those washed with neutralizing solution. It has been postulated that the additional damage caused by the heat of the exothermic neutralization reaction is responsible for this increased mortality. Educational Objective: Dry unknown chemicals should always be brushed off of the skin first. Once the remaining visible powder is removed, then the area should be irrigated with copious amounts of low-pressure water. The following Vignette applies to the next 3 items A 40-year-old male accountant presents to clinic for a follow-up visit regarding his recent exposure to HIV. While on a business trip two weeks ago, he had unprotected anal intercourse with a man who confided that he was HIV positive a few days later. Immediately after discovering this, the patient purchased an over-the-counter home kit for HIV testing at a local pharmacy. The kit uses a double ELISA test with a confirmatory Western Blot, and its sensitivity and specificity approach 100%. He was informed by telephone that the ELISA test was negative, but finds that he is still extremely anxious about the possibility of contracting the virus. His medical complaints at this time include fatigue, a sore throat, and some muscle aches, which he attributes to his long work hours and poor sleep. His medical history is significant for psoriasis and an episode of renal calculi. He does not take medications at this time. He does not smoke cigarettes and restricts his alcohol intake to social occasions. His temperature is 38.4C (101.1F), blood pressure is 122/82 mm Hg, pulse is 72/min, and respirations are 15/min. On physical examination, he appears anxious and tired. There is no erythema, exudate, or ulcer visible in the mouth or throat. There is some generalized lymphadenopathy. Chest auscultation is unremarkable and heart sounds are normal. His abdomen is nontender and nondistended and there is no palpable hepatosplenomegaly. Bowel sounds are normal. Rectal tone is normal and a small laceration is visible on the anus. The laceration is healing well and granulation tissue is present. Genital exam is normal. Item 1 of 3 Which of the following should be recommended at this time? A. No further testing is necessary B. Obtain biopsy of anal lesion C. Order IgM HIV antibody assay D. Order HIV RNA PCR assay E. Order Western Blot Explanation: Traditionally, when a known exposure to HIV has occurred, HIV antibody testing should be performed at the initial visit and then repeated at 6, 12, and 24 weeks. The standard HIV antibody test is called ELISA (Enzyme-Linked Immunosorbent Assay), which is an excellent screening tool. Specimens that are ELISA-positive are then tested with the more specific Western Blot for confirmation. In a patient with a clinical presentation suggestive of primary HIV infection and a negative or indeterminate HIV ELISA test (a scenario that can occur when the test is performed after exposure and before seroconversion), the diagnosis is definitively determined by either confirming a high viral load with an HIV RNA PCR assay (Choice D) or by confirming the presence of the p24 antigen with the test of the same name. Recommending no further testing (Choice A) is inadvisable. This patient has some clinical features suggestive of HIV infection (eg, fever, fatigue, myalgias, lymphadenopathy) and a known recent exposure to the virus. It is possible that he has not seroconverted yet since his exposure to HIV was so recent, which means that further testing is warranted. Obtaining a biopsy of a skin lesion (Choice B) is rarely of help in diagnosing primary HIV infection because the histopathologic changes seen in HIV infection are nonspecific. Since IgM-anti-HIV antibodies circulate in small numbers, assays for these antibodies are considered insensitive (Choice C) and are therefore of little use in diagnosing HIV infection. The Western Blot test (Choice E) is typically used to confirm a positive HIV ELISA test. The Western Blot is less sensitive but more specific than ELISA, and allows for a direct visualization of antibodies to various components of HIV. Since false negatives are common early in the course of the illness (before seroconversion has occurred), it would be better to diagnose acute HIV infection by either confirming a high viral load or the presence of a positive p24 antigen.

Educational Objective: In patients with clinical presentations suggestive of primary HIV infection and negative or indeterminate HIV ELISA tests, the diagnosis can be definitively confirmed with an HIV RNA PCR assay or a test for the p24 antigen. Item 2 of 3 Further testing reveals that the accountant is indeed HIV positive. His baseline viral load and CD4 count levels are measured on two separate occasions. The risks and benefits of antiretroviral therapy are discussed, and the decision is made to defer the initiation of therapy until a later date. He asks how frequently his immune status will be monitored. In light of the circumstances, which of the following is most appropriate? A. Evaluate his CD4 count and HIV load once every 6-9 months B. Evaluate his CD4 count and HIV load once every 3-4 months C. Evaluate his CD4 count and HIV load once every 12 months D. Evaluate his CD4 count and HIV load once every month E. Evaluate his CD4 count and HIV load once every 2 weeks Explanation: Guidelines on the treatment and management of HIV-positive patients have been issued by an expert panel at the United States Department of Health and Human Services. This panel recommends that the CD4 count and viral load be measured every three to four months (Choice B) to determine the optimal time to initiate medicating those patients not already on highly active antiretroviral therapy (HAART). A similar schedule is recommended for those patients on HAART to evaluate the efficacy of treatment. Evaluating the CD4 count and viral load less often (Choices A and C) may needlessly delay treatment initiation in a patient whose condition is worsening. Evaluating the CD4 count and viral load more often (Choices D, and E) is usually not necessary early in the course of the disease as precipitous changes in immune system function are uncommon. Educational Objective: HIV-positive patients not already on HAART should have their CD4 count and viral load measured every 3-4 months. Item 3 of 3 A schedule is established for the testing of this patient?s CD4 count and HIV load. Which of the following laboratory results is the strongest indication for the initiation of HAART in an asymptomatic HIV-positive patient with no history of AIDS-defining illness? A. CD4 count of 700 and HIV viral load of 80,000 B. CD4 count of 600 and HIV viral load of 60,000 C. CD4 count of 500 and HIV viral load of 50,000 D. CD4 count of 400 and HIV viral load of 20,000 E. CD4 count of 300 and HIV viral load of 60,000 Explanation: The expert panel at the United States Department of Health and Human Services agrees that antiretroviral therapy is clearly indicated for those patients with a history of AIDS-defining illness or with severe symptoms of HIV infection, as well as in those patients who are asymptomatic but have CD4 counts <200. In asymptomatic patients who have CD4 counts ranging from 201-350, providers should discuss the risks and benefits of HAART (Choice E). Patients with a high viral load (55,000 to 100,000 copies/mL) or a rapid decline in CD4 count (greater than 100 cells/?L per year) should also be encouraged to initiate HAART.

In those asymptomatic patients with CD4 counts >350 and a viral load of >100,000, most clinicians will recommend that therapy be deferred, though some will consider the initiation of antiretroviral therapy. Therapy is almost always deferred in those patients with CD4 counts of >350 and viral loads <100,000 (Choices A, B, C, and D). Educational Objective: Asymptomatic HIV-positive patients with CD4 counts <350 and plasma viral loads of > 55,000 copies/mL should be offered therapy with HAART. The role of HAART in patients with high viral load (55,000 to 100,000 copies/mL) is not clear (in general not recommended) if their CD4 count is more than 350.

The following vignette applies to the next 2 items A 54-year-old Caucasian man is brought to the emergency department. He complains of chest pain, nausea, vomiting and fever. His symptoms began eight hours ago, and are progressively getting worse. He was drinking alcohol with a group of friends, when he developed persistent nausea and repeated vomiting. As the nausea and vomiting ceased, he began to feel better. He was able to eat meals despite moderately severe chest pain, which he attributed to the vomiting. His chest pain then increased in severity, and the pain radiated to his right shoulder. His chest pain is aggravated by deep inspiration. He developed fever in the last couple of hours. He has no other medical problems. He drinks 3-4 bottles of beer daily. His father had a myocardial infarction at age 50. He takes no medications. His blood pressure is 140/90 mm Hg, heart rate is 118 beats/min, respiratory rate is 22/min, and temperature is 39.2 C (102.5F). On examination, he is alert and awake, but appears mildly dehydrated. Breath sounds are decreased in the left hemithorax, with dullness on percussion of the lower third region. The abdomen is mildly distended and tender, especially in the epigastrium. There is no rebound tenderness. Bowel sounds are present. There are no neurological abnormalities. Chest x-ray reveals a left pleural effusion and a small radiolucent band at the left side of the cardiac silhouette. Item 1 of 2 Which of the following is the most likely diagnosis? A. Acute myocardial infarction B. Esophageal perforation with mediastinitis C. Acute bacterial pneumonia D. Acute cholecystitis E. Acute pancreatitis Explanation: This patient?s presentation of chest pain after repeated episodes of vomiting is classic for esophageal perforation (Boerhaave?s syndrome), complicated with acute mediastinitis. Other associated symptoms are dyspnea, epigastric pain or shoulder pain. Most esophageal tears occur in the distal third of the esophagus, and this leads to pleural effusion. In seventy-five percent of the cases, pleural effusion will develop six hours after perforation. The effusion is located on the left side in 66% of the cases, and can be accompanied by pneumomediastinum, pneumothorax or both. The radiologic finding of a radiolucent band adjacent to the cardiac border is typical of pneumomediastinum. It takes a mean of more than four hours for fever to develop due to mediastinitis. Urgent management is needed since mediastinitis carries a mortality rate of more than 40% if not properly diagnosed within the first 24 hours. (Choice D) Patients with acute cholecystitis may also present with shoulder pain and fever; however, the presence of pneumomediastinum, pleural effusion and chest pain make this diagnosis unlikely. (Choice E) Acute pancreatitis can be accompanied by left pleural effusion, vomiting, tachycardia and fever; however, the presence of normal bowel sounds and pneumomediastinum makes this an unlikely diagnosis. (Choice C) The history of previous alcohol intake, subsequent repeated vomiting, fever, and the radiologic finding (left pleural effusion) raises the concern about the possibility of aspiration pneumonia; however, the absence of infiltrates in the chest x-ray makes this diagnosis unlikely. Pleural effusions always follow the development of the radiologic infiltrates. (Choice A) Myocardial infarction or ischemia is the main differential diagnosis of any episode of chest pain; however, the history of multiple episodes of vomiting prior to the chest pain, as well as the radiologic findings of left pleural effusion and pneumomediastinum, makes this possibility unlikely. Educational Objective: Esophageal perforation, also known as Boerhaave?s syndrome, usually presents as acute chest pain following episodes of repeated vomiting. Most tears occur in the distal third of the esophagus, which

leads to pleural effusion. Pneumomediastinum and pneumothorax can be part of the presentation. Urgent management is needed because of the risk of mediastinitis, which carries a mortality rate of more than 40% if not properly diagnosed within the first 24 hours.

Item 2 of 2 The patient?s fever persists. He looks more dyspneic. His blood pressure is 140/80 mmHg, heart rate is 124/min, respiratory rate is 24/min, and temperature is 39.4 C (102.9F). He is started on intravenous fluids. Urine and blood culture results are pending. His other laboratory tests reveal: CBC Hb: 11.4 g/dL Ht: 34% Platelet count: 450,000/cmm Leukocyte count: 15,000/cmm Segmented neutrophils: 80% Bands: 3% Lymphocytes: 17% Serum chemistry Serum Na: 139 mEq/L Serum K: 3.4 mEq/L Chloride: 96 mEq/L Bicarbonate: 32 mEq/L BUN: 34 mg/dL Serum creatinine: 1.2 mg/dL Calcium: 10 mg/dL Blood Glucose: 84 mg/dL Which of the following is the most appropriate test to confirm the diagnosis? A. Upper GI endoscopy B. Cardiac enzymes C. Esophagogram D. Amylase and lipase E. CT scan of the chest and abdomen Explanation: The best diagnostic test for esophageal perforation is an esophagogram with water-soluble contrast. This test provides a definite diagnosis in 90% of the cases. If the test is negative but the clinical suspicion is high, barium contrast can be used. Water-soluble contrast is preferred to barium because the latter can produce further mediastinal irritation and injury. (Choice A) Upper GI endoscopy has no role and should not be used in the evaluation of patients with spontaneous esophagal perforation. Both the endoscope and insufflation of air can cause extension of perforation and worsen the mediastinitis. (Choices B and D) As previously discussed, the possibilities of this patient having pancreatitis and myocardial infarction are low. (Choice E) An abdominal CT scan will only allow visualization of the esophagogastric junction, as well as part of the pleural effusion. A chest CT scan may provide a better visualization of the mediastinal compromise; however, it may fail to detect small tears or ruptures of the esophagus, and is therefore not the best diagnostic modality. Educational Objective:

The best diagnostic test for esophageal perforation is an esophagogram with water-soluble contrast. This test provides a definite diagnosis in 90% of the cases. CT scan of the chest is helpful, but may not detect small tears or ruptures. Upper GI endoscopy has no role and should not be used.

A 52-year-old white female gets bone mineral density testing done following her menopause six months ago. Her BMD (by DXA) reveals a T-score of ?2.6 at the lumbar spine and ?1.8 at the femoral neck. She has chronic constipation, and is otherwise asymptomatic. She denies any loss of height. Except for borderline hypertension "for many years", her past medical history is unremarkable. She is currently on calcium and vitamin D supplementation. She does not smoke or drink. Her mother has osteoporosis. Examination reveals a blood pressure of 160/100 mm Hg, and regular heart rate of 72/min. She is 5?4" (160 cm) tall and weighs 160 pounds (72.5 kg). The rest of the physical examination is unremarkable. Her complete blood count is normal. Basic serum chemistry reveals a serum calcium level of 10.8 mg/dL (normal 8.5 to 10.5 mg/dL, corresponding albumin 4.0 g/dl) and serum phosphorus level of 2.2 mg/dL (normal 2.5 to 5 mg/dL). The PTH level is 58 pg/mL (normal 10 to 65 pg/mL). Her 24-hour urinary calcium level is 300 mg, and 25-hydroxyvitamin D level is 26 mcg/L (normal 18 to 68 mcg/L). Her medical records reveal that all her prior calcium values have been within normal limits. What is the next best step in the management of this patient? A. Refer for parathyroidectomy B. Discontinue calcium/vitamin D supplementation C. Followup with repeat serum calcium and serum creatinine in six months and repeat bone mineral density measurement in one year D. Begin alendronate E. Begin hydrochlorothiazide Explanation: Many patients with primary hyperparathyroidism have PTH levels that are in the high normal range but are inappropriately elevated for the degree of hypercalcemia. In normal individuals, serum calcium levels inhibit the release of PTH by acting on calcium-sensing receptors on the parathyroid gland. This mechanism is defective in patients with hyperparathyroidism caused by either adenoma or hyperplasia, where the rise in calcium is ineffective in suppressing the release of PTH. Other causes of high serum calcium with high circulatory PTH levels in the presence of normal renal function are familial hypocalciuric hypercalcemia (FHH) and lithium toxicity. Both can be ruled out by the patient's normal calcium levels in prior studies, and the absence of any history of a past mood disorder or lithium use, respectively. Indications for parathyroidectomy in patients with primary hyperparathyroidism include: Bone mineral density of less than 2.5 standard deviation (T score of <?2.5) below the normal young adult at any site. Any complication of primary hyperparathyroidism (e.g., kidney stones or overt bone disease) Serum calcium more than 1 mg/dL above the upper limit of normal or history of life-threatening hypercalcemia. Daily urinary excretion of calcium more than 400 mg Thirty percent reduction in creatinine clearance Patients younger than 50 years of age A parathyroidectomy is indicated in this patient with primary hyperparathyroidism because of her low bone mineral density at the lumbar spine. (Choice C) Patients with primary hyperparathyroidism without any indications for parathyroidectomy do well on conservative management. In these patients, periodic measurement of serum calcium, serum creatinine, and bone mineral density is required. (Choice B) Although one would assume that calcium and vitamin D supplementation in hypercalcemic patients with primary hyperparathyroidism leads to an increase in serum calcium level, studies have shown that normal vitamin D and calcium intake in such patients does not lead to hypercalcemia. Normal calcium and vitamin D intake should be continued in patients with primary hyperparathyroidism, unless they are in hypercalcemic crisis. (Choice D) Alendronate can increase bone mineral density in patients with primary hyperparathyroidism; however, it is not the best treatment because it can increase the levels of PTH during the first few months of therapy. Bone mineral density improvement following parathyroidectomy is also likely to be much higher than with alendronate.

(Choice E) Hydrochlorothiazide is a useful antihypertensive drug; however, it might further increase the levels of calcium in hypercalcemic patients. This patient should be started on any other first-line antihypertensive agent. Educational Objective: The indication for surgical treatment of primary hyperparathyroidism in this patient is low bone mineral density (T score of ?2.6 at the lumbar spine). Other major indications for surgical treatment of primary hyperparathyroidism include renal dysfunction (30% reduction in serum creatinine), kidney stones, hypercalciuria (> 400 mg calcium excretion/d), young age, and serum calcium 1 mg/dl above the upper limit of normal range. A 26-year-old nulliparous woman at 39 weeks of gestation is admitted to the maternity ward because she is in labor. Her uterine contractions began six hours ago. The membranes ruptured before she was rushed to the hospital. The patient's current pregnancy was uneventful, and routine work-up revealed nothing abnormal. Third trimester US revealed a fetus in cephalic presentation and of average weight. Examination upon admission showed an anterior and effaced cervix, dilated at 3 cm. Dilation progressed thereafter to 6 cm. Her pelvis is of normal shape and proportions. Examination performed two and a half hours later demonstrates the same dilation and fetal descent at 0 station. The fetal heart tracing is reassuring. Which of the following is the most appropriate next step in the management of this patient? A. Folic acid B. Magnesium sulfate C. Vaginal delivery D. Caesarian section E. Begin hydrochlorothiazide Explanation: This patient has an arrest disorder in the active phase of labor. In nulliparous women, the most common cause of an arrest disorder is hypotonic uterine contractions. It is also the most likely diagnosis in this case, since all other parameters that may cause this abnormality are normal: the pelvis is clinically normal, the fetus is of average size, and there is no anomaly of fetal presentation. In addition, the fetal heart tracing is reassuring; therefore, labor may be allowed to proceed. Augmentation of labor using oxytocin infusion is the most appropriate next step in the management of this patient. (Choice B) There is no indication for magnesium sulfate in this case. MgSO4 is used as a CNS depressant, mainly in preeclampsia and eclampsia, or as an electrolyte replenisher for hypomagnesemia. (Choices C and D) At this point, we cannot yet determine if the patient should undergo a Caesaran section or vaginal delivery. The decision will depend on her response to the augmentation of labor and the fetus' well-being. Educational Objective: Hypotonic uterine contractions is the most common cause of arrest disorders in the active phase of labor. Patients with such conditions should undergo an augmentation of labor using amniotomy and/or oxytocin infusion. A 78-year-old healthy, Chinese-American man is brought to the emergency department because of hallucinations, delusions, and aggressive behavior, according to his nephew. He has had these symptoms for the past six days. He denies fever, chills, decreased vision, headaches, or abdominal pain. His other medical problems include hypertension, coronary artery disease, and osteoporosis. He does not use tobacco or drink alcohol. He is a strict vegetarian. He lives alone. His medications include aspirin, hydrochlorothiazide, enalapril, acetaminophen, and diclofenac. He has no known drug allergies. His blood pressure is 140/90 mmHg, pulse is 100/min, temperature is 37.2 C (99 F), and respirations are 14/min. Examination shows dry mucus membranes and a loud second heart sound. The abdomen is soft, nontender, and non-distended. Bowel sounds are mildly increased. There is no rebound tenderness or rigidity. The range of motion of the spine is normal and without pain. There is no scoliosis or excessive kyphosis of the back. There is no tenderness to palpation of the lower back.

The mental status examination reveals an agitated, elderly man with delusional ideation and bizarre behavior. There are no motor or sensory focal deficits, and no meningeal signs. The patient?s labs reveal: CBC Hb: 9.6 g/dL Ht: 38% MCV: 108 fl Platelet count: 120,000/cmm Leukocyte count: 4,000/cmm Segmented neutrophils: 50% Bands: 3% Eosinophils: 3% Lymphocytes: 34% Monocytes: 10% Serum chemistry Serum Na: 136 mEq/L Serum K: 4.4 mEq/L Chloride: 102 mEq/L Bicarbonate: 26 mEq/L BUN: 36 mg/dL Serum Creatinine: 1.4 mg/dL Calcium: 9.6 mg/dL Blood Glucose: 80 mg/dL LFT Total bilirubin: 1.9 mg/dL Direct bilirubin: 0.4 mg/dL Alkaline phosphatase: 100 U/L Aspartate aminotransferase: 35 U/L Alanine aminotransferase: 30 U/L His chest-x ray and urinalysis results are normal. pharmacotherapy? A. Folic acid B. Chemotherapy C. Vitamin B12 D. Thiamine E. Antibiotics Explanation: Vitamin B12 deficiency can present in a subtle manner in the elderly. Some patients do not even have anemia, but the majority have an increase in the microscopic corpuscular volume (MCV). Megaloblastic anemia can be secondary to folic acid or vitamin B12 deficiency, and is characterized by an increase in indirect bilirubin and reticulocytes, as well as pancytopenia or bicytopenia, as in this case. The neurologic presentation includes ataxia, dementia, and occasionally, delirium. (Choice A) Folic acid deficiency is not related to any neurologic abnormalities. (Choice D) Clinically, this is not a case of thiamine deficiency. Thiamine deficiency presents with a classical triad of encephalopathy, oculomotor dysfunction, and gait ataxia, and is usually seen in alcoholics. Furthermore, anemia and elevated MCV are not seen in thiamine deficiency. Which of the following is the most appropriate

(Choice E) Even though infection is one of the most common causes of delirium in the elderly, this patient has no other signs or symptoms of infection. He is afebrile. He has no left shift, and his white count is not elevated. (Choice B) Although the patient has pancytopenia, there is no evidence of malignancy. Even if there were malignancy, aggressive chemotherapy could be especially dangerous for a patient of this age. Educational Objective: It is important to recognize vitamin B12 deficiency as a potential cause of delirium or dementia in the elderly patient. Subtle laboratory findings, such as increased MCV and reticulocytes, or pancytopenia, can help to identify this condition. The following vignette applies to the next 2 items An 18-month-old girl with a history of low-grade fever and irritability is brought to the emergency department by her parents. Her mother complains that she is not as playful as before, and that she has been sleeping more for the past week. Her grandmother was diagnosed with pulmonary tuberculosis two months ago, and is presently undergoing antitubercular therapy. The initial physical findings highly suggest meningitis. A sample of her cerebrospinal fluid reveals the presence of low glucose, elevated proteins, and lymphocytic pleocytosis. Special staining and microscopic examination of the CSF fluid also reveals the presence of acid-fast organisms. Since these findings are all consistent with tuberculous meningitis, the patient is started on antituberculous therapy. Her mother asks you how long the antitubercular therapy will take. Item 1 of 2 Which of the following is the most appropriate response? A. She will need treatment for at least 12 months. B. She will need treatment for at least nine months. C. Adjunctive corticosteroid treatment can reduce the duration of treatment. D. She will need treatment for at least six months. E. The treatment duration can be reduced if she agrees to intrathecal drug therapy. Explanation: Tuberculous meningitis is one of the forms of extrapulmonary tuberculosis. It is associated with high morbidity and mortality rates. Patients usually present with an insidious onset of symptoms such as malaise, headache, and low-grade fever. If left untreated, these symptoms can rapidly progress to persistent headaches, vomiting, cranial nerve involvement, confusion, seizures, coma, and eventually, death within six to eight weeks of the onset of illness. All patients with suspected tuberculous meningitis (based on initial history and CSF examination) should be immediately started on empiric antituberculous therapy, pending the results of confirmatory tests. The prognosis of the patient with tuberculous meningitis greatly depends on the stage in which the treatment is initiated. Early diagnosis and treatment of the patients is critical to prevent adverse clinical outcomes. All patients with tuberculous meningitis should be initially treated with a combination of isoniazid, rifampin, and pyrazinamide (bactericidal agents) for the first two months, followed by treatment with INH and rifampin for the succeeding months. These three agents have good CSF penetration, and can achieve adequate therapeutic levels in the cerebrospinal fluid. Based on the current guidelines, all infants and children with drug-sensitive tuberculous meningitis should be treated for at least 12 months. In a patient with drug-resistant infection, the therapy may be extended to 18-24 months, depending on the clinical response and severity of the illness. (Choice C) There is evidence that adjunctive corticosteroid treatment in certain situations can significantly reduce the neurological complications and mortality rates in patients with tuberculous meningitis; however, this does not affect the duration of antitubercular therapy. (Choice E) Intrathecal therapy for tuberculous meningitis is not recommended since all the agents have good CSF penetration.

Educational Objective: Specific antituberculous therapy should be initiated early in all patients with clinically suspected tuberculous meningitis. Infants and children with tuberculous meningitis, miliary TB, and tuberculous osteomyelitis should receive 12 months of antituberculous therapy. Item 2 of 2 The mother is extremely anxious and asks if the grandmother (who has been on antituberculous therapy for the past two months) can still transmit her infection to other household members. Which of the following is the most appropriate response? A. She can be considered non-infectious after one week of antituberculous therapy. B. She can be considered non-infectious if her cough has resolved. C. She can be considered non-infectious if she has had three negative sputum acid-fast smears on three different occasions. D. She can be considered non-infectious after two months of antitubercular therapy. E. She can be considered non-infectious if the chest radiograph findings have resolved. Explanation: All patients with sputum-positive pulmonary or laryngeal tuberculosis can transmit the infection to other household contacts or healthcare workers via infectious aerosols containing Mycobacterium tuberculosis bacilli. The acts of coughing, sneezing, singing, and even speaking can all produce microscopic aerosols containing the organism. All such patients should be placed in respiratory isolation until they are confirmed to be non-infectious. Patients are rendered non-infectious if they are receiving effective antituberculous therapy and have had three consecutive negative results on sputum acid-fast smears performed on different occasions. (Choices A and D) Non-infectivity should be documented by serial negative acid-fast smears, and not by the duration of therapy. A patient can still be infectious after prolonged drug therapy if he has a drug-resistant infection. (Choice B) Many patients can continue to have cough even after they have been rendered noninfectious with effective antitubercular therapy. (Choice E) Resolution of chest x- ray findings in patients with pulmonary tuberculosis typically lag behind the clinical response. Chest radiograph changes can persist for longer periods of time after the patient has been rendered non-infectious, and may even become permanent. Educational Objective: Patients are rendered non-infectious if they are clinically improving on effective antituberculous therapy and have three negative results on sputum smears on different occasions. You are the on-call resident for the oncology service in your hospital. A nurse calls you late in the evening and tells you that one of the patients from the oncology floor has started bleeding from his nose, and that the bleeding has not stopped with local pressure. The patient was diagnosed with acute myelogenous leukemia six months ago, and has been receiving chemotherapy ever since. He was readmitted earlier during the day with a relapse of AML. Later during the day, while eating dinner, he started bleeding from his left nostril. You order a stat complete blood count, which reveals a hemoglobin level of 8.2 g/dL and platelet count of 10,000/microL. You decide to transfuse the patient with six units of pooled platelets, and the repeat platelet count one hour after the transfusion is 15,000/microL. Which of the following is the most likely explanation for these findings? A. This is a normal response B. You have measured the platelet counts too early C. The patient has antibodies against the platelets D. You have not given him enough platelets E. He has disseminated intravascular coagulation

Explanation: Platelets play a critical role in the normal hemostatic mechanism; hence, their deficiency or impaired function can cause significant or life-threatening hemorrhage. Platelet transfusions are commonly used to prevent or treat acute hemorrhage in patients with thrombocytopenia due to any cause. As a general rule, for an adult with a body surface area of 2.0 m2, a transfusion of 1 unit of platelets should immediately raise the post-transfusion platelet count by approximately 5,000/microL. (For this patient, six units should have raised the post-transfusion platelet count by at least 30,000/microL.) If there is a smaller than expected rise in the platelet count, refractoriness to platelet transfusions should be considered Platelet refractoriness is defined as an absolute platelet count increment of less than or equal to 2,000/microL per unit of platelet transfusion given to an average-sized adult. Alloimmunization is one of the common and treatable causes of platelet refractoriness. It results from the production of antibodies to HLA class 1 antigen on the transfused platelets. These alloantibodies can cause a rapid destruction of the transfused platelets and can cause refractoriness to platelet transfusion. (Choice A) This is not a normal response. A normal response to platelet transfusion is an immediate post-transfusion rise in the platelet count of at least 5,000/microL per unit of platelets transfused. (Choice B) The post-transfusion platelet count should be measured 10 - 60 minutes after the completion of transfusion. A subnormal response or increment in the platelet count suggests platelet refractoriness due to alloimmunization. (Choice E) A platelet count with a normal increment at one hour after transfusion, but with a rapid return to baseline within 24 hours indicates reduced platelet survival in the circulation. This is typically seen in patients with disseminated intravascular coagulation, sepsis, active bleeding, or in patients who use drugs that reduce platelet survival. (Choice D) The patient has received enough platelets. Six units of platelets should have raised the post-transfusion platelet count by at least 30,000/microL in this patient. Educational Objective: The post-transfusion platelet count should be measured 10 - 60 minutes after the completion of transfusion. Each unit of platelet transfused should cause an immediate posttransfusion rise in the platelet count by at least 5,000/microL. Platelet refractoriness due to alloimmunization should be considered if platelet transfusions fail to cause the expected increment in the platelet count. The following vignette applies to the next 2 items A 35-year-old HIV-positive woman comes to the emergency room with her 10-year-old HIV-negative daughter. She tells you that her daughter had a fight with her other 12-year-old daughter, who is HIVpositive. During the process, the HIV-positive daughter bit her sister on her forearm. On examination of the bite wound, there are two deep puncture wounds with a slight amount of oozing blood on her left forearm. There is slight warmth and swelling around the bite marks. You provide general wound care with thorough saline irrigation. The mother is concerned about the risk of transmission of HIV to her younger daughter. Item 1 of 2 Which of the following infections is the younger daughter at greatest risk for? A. HIV infection B. Necrotizing fascitis with Staphylococcus epidermidis C. Gas gangrene with Clostridium perfringens D. Osteomyelitis with Pasteurella multocida E. Soft tissue infection with Eikenella corrodens Explanation: Soft tissue or wound infections can occur after a human bite due to exposure to human mouth microbes. Human bites are more serious than animal bites, and can be limb threatening. These can involve the soft tissue, fascia, bone or joints, depending upon the extent and the depth of the bite. Most of the soft tissue infections caused by human bites are polymicrobial in nature. Infections with human bites are associated with alpha-hemolytic Streptococci, Staphylococcus aureus, Eikenella corrodens, Haemophilus species and anaerobic bacteria (bacteroides species, Peptostreptococcus,

Actinomyces species and fusobacterium). Eikenella corrodens is a gram-negative anaerobe commonly found in soft tissue infections due to human bites. (Choice A) HIV is present in very minimal quantities in the saliva of some AIDS patients; however, this does not mean that HIV can be transmitted by the saliva. Exposure or contact to saliva of patients with HIV has never been shown to result in the transmission of HIV infection. (Choice B) Staphylococcus aureus, not epidermidis, may be seen as part of the polymicrobial flora in infections due to human bites. (Choice C) Clostridium perfringens is not found in human oral flora. It causes gas gangrene or myonecrosis after deep, penetrating injuries or wounds, especially when there is a disruption in the blood supply to the traumatized tissue. (Choice D) Pasteurella multocida wound infections or osteomyelitis is seen after bite injuries with cats and dogs. Cats, especially with their sharp pointed teeth, can implant the bacteria under the periosteum, causing osteomyelitis. Educational Objective: Human bites can be complicated by a polymicrobial soft tissue infection. Eikenella corrodens, a gramnegative anaerobe, is often recovered from these infections. Item 2 of 2 Apart from local wound care, which of the following is the most appropriate therapy for this patient? A. Ampicillin ? sulbactam B. Cefazolin C. Vancomycin D. Ampicillin ? sulbactam with Zidovudine (AZT) for 3 weeks E. Ampicillin ? sulbactam with triple therapy with zidovudine (AZT), lamivudine (3TC) and Indinavir (protease inhibitor) Explanation: The patient in the above vignette has early signs of soft tissue infection around the bite wound. She presents with pain and swelling at the site of infection. Examination of the site reveals erythema, warmth and swelling, with purulent drainage from the site. Most of the soft tissue infections from human bites have a mixture of anaerobic and aerobic organisms. Treatment is therefore directed at these organisms. Ampicillin-sulbactam is the drug of choice for such infections because it provides broad-spectrum coverage for both the aerobes and anaerobes, including Eikenella corrodens. (Choices B and C) Cefazolin and vancomycin do not provide coverage against Eikenella corrodens and some other anaerobes. They are not appropriate as monotherapy for infections due to human bites. (Choices D and E) Contact with saliva, tears, or human sweat has never been shown to cause transmission of HIV. Severe tissue trauma with extensive tearing and damage in the presence of blood may lead to an increased risk of transmission; however, numerous reports in literature about human bites show that HIV has never been transmitted in this manner. There is no need to treat this patient with AZT or triple therapy for prophylaxis against HIV infection. Educational Objective: Ampicillin-sulbactam is the parenteral drug of choice for significant wound infections due to human bites (This is also the drug of choice for cat or dog bites). A 64-year-old widowed Caucasian woman presents with the chief complaint, "After sex, I?ve got a lot of pain and some bleeding, which I never had before." She also reports some new onset mild vulvar itching and burning. Earlier this month, she resumed having sexual relations for the first time in twenty years. She had been celibate since her husband?s death by myocardial infarction. She underwent menopause at age 55 and has not experienced any vaginal bleeding since then. Her past medical history is significant for generalized anxiety disorder, mild aortic regurgitation, and

hypertension. Current medications include paroxetine and hydrochlorothiazide. Pelvic examination reveals loss of labial fullness, pallor of the vaginal epithelium, and decreased vaginal secretions. What is the most likely cause of her vaginal bleeding? A. Vulvar cancer B. Cervical cancer C. Endometrial cancer D. Candidiasis E. Atrophic vaginitis Explanation: The most common cause of postmenopausal bleeding is atrophic vaginitis (50-60%). However, one should always rule out the most serious conditions, such as endometrial carcinoma (less than 10%) by obtaining endometrial biopsy. Atrophic vaginitis (Choice E) is characterized by vaginal dryness, burning, dyspareunia, reduced vaginal secretions, and vulvar pruritus. Occasionally, urinary symptoms such as dysuria, hematuria, and discomfort are also seen. Pelvic examination typically reveals loss of labial fullness, pallor of the vaginal epithelium, and decreased vaginal secretions. The condition commonly occurs secondary to declining estrogen levels in postmenopausal women. In nonmenopausal women, estrogen production can be countered by radiation therapy, chemotherapy, immunologic disorders, oophorectomy, and lactation. Atrophic vaginitis is treated with hormonal replacement therapy, transvaginal estrogen replacement (e.g., creams, hormone-releasing rings), or lubricants. Vulvar cancer (Choice A) most often occurs in women aged 65-75 and is characterized by a long history of pruritus. Vulvar bleeding, discharge, dysuria, and pain are reported less often. A raised vulvar lump or mass is apparent on pelvic examination, and may be fleshy, ulcerated, leukoplakic, or warty in appearance. Cervical cancer (Choice B) most often occurs in women aged 40-60. Women at increased risk for developing cervical cancer include smokers and those with a history of sexually transmitted diseases, human papillomavirus infection, low socioeconomic status, two or more lifetime sexual partners, or immunosuppression (e.g., AIDS). Cervical cancer is responsible for less than 1% cases of postmenopausal bleeding. Candidiasis (Choice D) is characterized by intense vaginal discomfort, odorless white or yellow vaginal discharge, pruritus, dyspareunia, or dysuria. Microscopic examination of vaginal secretions with 10% KOH may demonstrate hyphae. Educational Objective: The most common cause of postmenopausal bleeding is atrophic vaginitis (50-60%). However, one should always rule out the most serious conditions, such as endometrial carcinoma (less than 10%) by obtaining endometrial biopsy. Cervical cancer is responsible for less than 1% cases of postmenopausal bleeding. A 62-year-old Caucasian man is admitted to the hospital for coronary artery bypass grafting and aortic valve replacement. He has a past history of myocardial infarction, hypertension, hyperlipidemia, and chronic renal insufficiency. He is a retired financial advisor with a 50-pack year smoking history. He underwent the surgery without any complications, and was successfully extubated the same day. His post-operative medications include aspirin, metoprolol, lisinopril, atorvastatin, and morphine via patient-controlled analgesia (PCA) pump. On the next day, the nurse calls you with concerns about his mental status. He has been getting progressively drowsy, lethargic, and extremely difficult to arouse. His temperature is 36.1 C (97 F), blood pressure is 90/60 mmHg, respirations are erratic with a rate of 2-4/min, heart rate is 58/min, and oxygen saturation is 92% on 2L/min of oxygen. His pupils are equal and pinpoint. What is the most appropriate next step in the management of this patient? A. Order 2 mg of intravenous naloxone and repeat as necessary B. Raise the head of the bed to 60 degrees to prevent aspiration

C. Obtain a stat arterial blood gas analysis to rule out CO2 narcosis D. Check and secure his airway E. Administer intravenous doxapram (respiratory stimulant) Explanation: Morphine overdose is a common scenario in hospitalized patients, especially for postoperative patients receiving high doses of morphine for acute pain control. Opiate naive patients with renal insufficiency are at a much higher risk. The usual manifestations of overdose are lethargy, drowsiness, miosis, bradycardia, hypotension, respiratory depression, and apnea. The first and initial priority in the management of overdose is always to check for and maintain a patent airway. It is important to ensure adequate respiratory exchange at all times before instituting specific therapy. (Choice A) Intravenous naloxone is the specific antidote for reversing the effects of morphine overdose. It is usually given as a 2 mg IV dose and repeated as necessary up to a maximum dose of 10 mg. However, adequate control of the airway is still the first priority in all patients. (Choice B) The airway should be secured invasively in this setting to prevent aspiration. Raising the head end of the bed alone will not prevent aspiration. (Choice C) Obtaining laboratory studies will lead to an inappropriate delay in the treatment and adversely affect the outcomes. For instance, identifying CO2 retention will not change the initial management in this setting, and should be deferred. (Choice D) The use of respiratory stimulants has not been shown to improve patient outcomes, and is not recommended. Educational Objective: Remember the ABCs of basic life support. should be given after ensuring adequate gas exchange. Specific therapy with naloxone

A mother brings her 4-year-old son to the emergency department. She cannot speak or understand English. She can speak only Spanish. Neither you, nor any of your emergency room staff, knows Spanish. Which of the following is the most appropriate choice to communicate with this patient? A. Send them home until the translator is available. B. Ask the mom to bring a translator. C. Try to communicate with the child. D. Call the long distance company for a translator. E. Do the physical examination of the child and try to determine the reason for his visit. Explanation: Almost all the hospitals in the United States have translators available. As a backup, hospitals also obtain accounts with long distance companies who provide translation services. The doctor uses a speakerphone, and a long distance company translator, on the other end of the phone line, assists in obtaining an adequate medical history and physical examination from the patient. (Choices A and B) Sending the patient home or requesting the mother to bring a translator is inappropriate. Doing so may waste a lot of time, and can result to unnecessary delays in treatment. (Choice D) The child is too young to reliably communicate with, and it is unlikely that the child will be conversant with English at this age, especially since his mother speaks only Spanish. (Choice E) Examining the child to determine the problem without first obtaining a medical history is an incorrect approach to this patient?s management. Educational Objective: Almost all the hospitals in the United States have translators available. As a backup, hospitals also obtain accounts with long distance companies who provide translation services.

A 39-year-old Caucasian female presents to your office with a 4-month history of "intense" headaches. She describes the headaches as non-throbbing, bilateral, and occurring in episodes which last for several hours, 4-5 times a week, and usually on the weekdays. She does not have any nausea, vomiting, blurring of vision, or eye pain. She takes acetaminophen to relieve her headaches. There is no family history of headaches. The physical examination is normal. Which of the following is the most likely cause of this patients headache? A. Tension headache B. Migraine C. Cluster headache D. Brain tumor E. Glaucoma Explanation: Tension headache is the most common headache syndrome. According to one study, the lifetime prevalence of tension headache can be as high as 69% for men and 88% for women. This patient presents with the characteristic symptomatology of tension headaches, which is described as nonthrobbing with bilateral localization. There are no associated nausea and vision problems; physical examination is normal (no neurological abnormalities). The pattern of occurence of the patient's headaches are also characteristic. Tension headaches tend to occur on the weekdays, and this may be associated with stress and overstrain at work. (Choice B) Migraine headaches are also common, but these are usually accompanied by aura, nausea, vomiting & photophobia. The headaches are usually described as throbbing in character, and the family history is usually positive. (Choice C) Cluster headaches are always unilateral and typically localize in the periorbital region. (Choice D) A brain tumor is always a possibility in a patient who presents with tension-type or migraine-type headaches; however, this is not common. Furthermore, important warning signs (e.g., bilateral headache, but worse ipsilaterally; morning headaches; nausea and vomiting; headaches worsened by bending; night awakenings; and neurological findings) that may indicate the presence of a mass lesion are absent in this patient. (Choice E) The headaches that are associated with glaucoma are usually periorbital and accompanied by nausea, vomiting, and visual changes. Educational Objective: Tension headaches are characterized by a non-throbbing headache with bilateral localization. There are no associated nausea and vision problems, as well as no neurological abnormalities. A 76-year-old Caucasian female comes to the emergency department after sustaining a fracture in her right proximal femur in an accidental, non-syncopal fall. She is a heavy alcohol drinker. Her past medical history is insignificant. She is started on thiamine and folic acid. The next morning, she undergoes orthopedic surgery for internal fixation of her right femur. Her post-operative course is unremarkable, except for mild agitation, presumably secondary to alcohol withdrawal. She is started on low molecular weight heparin for prophylaxis of DVT after her surgery. On her third, post-operative day, she develops respiratory distress. Pulse oximetry drops to 85% on room air. She is extremely confused and agitated. Her temperature is 36.7C(98F), blood pressure is 110/70 mm Hg, pulse is 120/min, and respirations are 30/min. Auscultation of the heart and lungs is essentially normal. There is no JVD or peripheral edema. Chest x-ray is normal and EKG shows sinus tachycardia. Which of the following is the most appropriate next step in the management of this patient? A. V/Q scan of the lungs B. Echocardiography C. D-dimer testing D. Start IV furosemide E. CT scan of the head

Explanation: Pulmonary embolism may still occur in patients who are receiving DVT prophylaxis due to their high risk. The sudden respiratory failure, tachycardia and tachypnea in the above patient may be due to pulmonary embolism; therefore, a V/Q scan should be obtained. (Choice C) D-dimer testing is not very useful in this setting. Although it has a very high negative predictive value, results are almost always elevated in hospitalized patients, especially when the patient has undergone recent surgery. This test might be useful in a healthy patient who comes to the emergency department with a similar presentation. In such a case, a negative D-dimer test result almost always ascertains that the patient does not have PE. (Choice B) There is nothing in the patient's history, physical exam, and test findings which suggest cardiac dysfunction or failure. Echocardiography is not indicated. (Choice D) Giving IV furosemide to the patient is inappropriate. IV furosemide is useful in the setting of acute pulmonary edema, which can be ruled out since this patient has clear lungs on exam and on chest-x ray. (Choice E) Hypoxia can cause confusion. She is clearly in respiratory distress and has no focal neurologic signs. CT scan of the head is not indicated. Educational Objective: Pulmonary embolism can still occur in patients who are receiving heparin for DVT prophylaxis The following vignette applies to the next 2 items A 31-year-old diabetic male presents to the emergency department because of abdominal pain. The patient states that he has not been feeling well for the past two days, and has been vomiting intractably. He stopped taking his regular dose of insulin because he was unable to "hold anything down" and was not eating well. His initial labs show: CBC: Hematocrit: 37% WBC: 11,000/cmm, normal differential Platelets: 240,000/cmm Serum chemistry: Sodium: 129 mEq/dL Potassium: 3.4 mEq/dL Carbon dioxide: 8 mmol/L Chloride: 90 mmol/L BUN: 48 mg/dL Creatinine: 2.3 mg/dL Blood glucose: 560 mg/dL Ketones were positive in the urine and serum. In the emergency room, he was started on intravenous normal saline with potassium and insulin infusion (7 units/hr.). Seven hours after his admission to the intensive care unit, the patient's blood glucose is 210 mg/dl, and his clinical status has improved. Item 1 of 2 What is the next best step in the management of this patient? A. Decrease the insulin infusion rate, and change the fluid to D5 ? NS with potassium. B. Continue normal saline, and start administering insulin by subcutaneous route after an hour overlap. C. Stop intravenous fluids, start oral feeding and subcutaneous insulin. D. Continue the same regimen until blood sugar is below 120 mg/dl. E. Continue normal saline and decrease the insulin infusion rate Explanation:

Incorrect fluid management is the most frequent mistake in the management of diabetic ketoacidosis (DKA). Most patients with DKA have 5-8 liters of fluid deficit. The typically administered initial fluid is normal saline, which is continued until the blood sugar approaches 250 mg/dl. At this time, normal saline should be changed to a dextrose-containing fluid (typically D5%1/2 NS containing 20-40 mEq/l of potassium chloride) which is given at a rate of 100-125 cc/hr. Dextrose infusion is very important to decrease ketone levels. At the same time, the insulin infusion rate is decreased to 1-2 units/hour. Starting an insulin infusion will shift the potassium into the cells, which could cause dangerous hypokalemia. Since the total potassium deficit is approximately 300 mEq, the patient should start receiving potassium supplementation at the outset of the treatment, even if potassium levels are within normal range. (Choice C) Switching to oral feeding and subcutaneous insulin should be done at an appropriate time. Typically, it is done when patients are clinically stable, anion gap has normalized, bicarbonate is over 10 mEq/l, the patients are able to tolerate food, and precipitating factor(s) are reasonably under control. (Choice B, D, and E) A delay in starting dextrose infusion usually results in the prolongation of ketone clearance from the serum. Educational Objective: Understand that glucose infusion and potassium replacement are important components of DKA management, aside from insulin and normal saline infusion. Item 2 of 2 The patient was managed appropriately. His insulin requirement, in order to keep his blood glucose level between 110 to 160 mg/dl, ranged from 1-3 units/hour. A repeat serum chemistry showed: Sodium: 136 mEq/dL Potassium: 4.0 mEq/dL Carbon dioxide: 18 mmol/L Chloride: 108 mmol/L BUN: 20 mg/dL Creatinine: 1.3 mg/dL The patient stopped vomiting. His blood glucose dropped to 54 mg/dl. The insulin infusion was stopped, and D5% + NS with KCl was continued. The patient now feels hungry and wants to eat. He receives 10 units of regular insulin bolus, and was given some food in the next hour. He starts to vomit shortly after his meal. A third basic metabolic panel shows: Sodium: 130 mEq/dL Potassium: 3.8 mEq/dL Carbon dioxide: 14 mmol/L Chloride: 98 mmol/L BUN: 22 mg/dL Creatinine: 1.4 mg/dL Blood glucose: 324 mg/dL What is the most likely explanation of his current condition? A. Premature switch to oral feeding and subcutaneous insulin B. Insufficient dose of subcutaneous insulin C. Hypoglycemic episode D. Discontinuation of intravenous insulin before starting subcutaneous insulin E. Patient should have received subcutaneous NPH instead of regular insulin Explanation: Another common mistake in the management of DKA is stopping intravenous infusion of insulin without an overlapping dose of subcutaneous insulin. Generally, rapid acting insulin is started when the switch from infusion to subcutaneous insulin is made. Because subcutaneously administered

insulin takes time to be absorbed, it should be administered 30 to 60 minutes before insulin infusion is stopped. Failure to do so could result in rapid recurrence of DKA. (Choice A) The patient was switched to subcutaneous insulin at an appropriate time because the anion gap had normalized and the patient had clinically improved. (Choice B) The dose of regular insulin was appropriate. (Choice C) A hypoglycemic episode does not usually trigger DKA by itself. (Choice E) If there is uncertainty in the patient's insulin requirement and food intake, intermediate and long-acting insulin are generally not started immediately. Not giving NPH with regular insulin does not explain the recurrence of DKA. Patients can generally resume their home dosage of insulin fairly quickly. A 26-year-old Caucasian woman with Down syndrome is brought in to the gynecologist for a discussion of contraceptive options. Her mother, whom she lives with, accompanies her. The mother expresses concern because she recently discovered her daughter having consensual sexual intercourse with a coworker who also has Down syndrome. The daughter is not currently using any contraception, and based on her knowledge of the girl?s functionality, the mother expresses great doubt that the daughter would be able to use contraception correctly or consistently. The mother requests that her daughter be sterilized because, she says, "I absolutely cannot deal with taking care of any children she might have. I am already overwhelmed as it is." When questioned, the daughter says she definitely does not want to be sterilized because she may want to have children one day. What is the most appropriate way to handle this situation? A. Inform daughter that sterilization is necessary and schedule procedure B. Inform daughter that sterilization is reversible and schedule procedure C. Warn daughter that if she will not agree to sterilization, she will not be eligible to receive other care she may need (e.g., Papanicolaou test) D. Recommend that the mother remove the daughter from the workplace and watch her more closely at home to prevent future sexual contacts E. Recommend usage of long-term birth control such as medroxyprogesterone acetate Explanation: When deciding ethical issues such as this, it is important to focus on the principles of patient autonomy and beneficence. Patients have the right to seek, accept, or refuse care. With the mentally disabled, it may be impossible to obtain informed consent, but at the least simple assent should be strenuously pursued. If, as in this instance, sterilization is not wanted, it should not be performed against the patient?s will. Involuntary sterilization violates a woman?s right to privacy, her reproductive rights, and her bodily integrity. The best means of proceeding would be to explain available contracptive options to the daughter, focusing on those that provide long-term protection with little effort required on the part of the patient (e.g. medroxyprogestrone acetate) (Choice E). Informing the daughter that sterilization is necessary (Choice A) or assuring her that it is reversible (Choice B) are both inappropriate. Sterilization is occasionally reversible, but often it is not. Moreover, she has indicated that she does not want sterilization performed, and her wishes should be honored if at all possible. Pressuring the daughter to proceed by withholding other unrelated services (Choice C) is unfair and unethical. Recommending the mother remove the daughter from the workplace and watch her more closely (Choice D) places a significant burden on the mother, which she has already indicated she cannot handle. It also prevents the daughter from enjoying some measure of autonomy and socialization in the workplace. As long as sexual abuse or exploitation is not suspect, the daughter should be allowed

to partake in consensual relationships. A better option is to provide her with reversible contraceptive measures so she will be protected from pregnancy. Educational Objective: In the mentally disabled, it may be impossible to obtain informed consent, but at the least simple assent should be strenuously pursued. Involuntary sterilization violates a woman?s right to privacy, her reproductive rights, and her bodily integrity, and should be avoided.

A 64-year-old Caucasian female has recently undergone total hip replacement arthroplasty. Her early postoperative period was uncomplicated. On the third post-operative day, she complains of shortness of breath and a non-productive cough. She is receiving low molecular weight heparin for deep venous thrombosis (DVT) prophylaxis, and enalapril and atenolol for hypertension. Her temperature is 37.8C(100F), blood pressure is 110/70 mm Hg, pulse is 98/min, and respirations are 22/min. Oxygen saturation on pulse oxymetry is 88% on room air. No calf tenderness or edema is present on physical examination. ECG reveals a sinus rhythm with nonspecific ST segments and T wave abnormalities. Which of the following is the best next step in the management of this patient? A. Serum troponin I B. Chest x-ray C. Arterial blood gases D. V/Q scan E. Pulmonary angiography Explanation: Hip replacement procedures are accompanied by a high risk of post-operative pulmonary thromboembolism (PE). This risk can be significantly reduced by carefully planned DVT prophylaxis. Although the episode of acute shortness of breath suggests the possibility of PE, alternative diagnoses should not be ignored. A thorough evaluation should be performed, the components of which include a detailed history, physical examination, ECG, oxygen saturation and chest x-ray. A chest x-ray may suggest the presence of PE, and may help exclude other chest pathology. Also, if the baseline chest-x ray is abnormal V/Q scan is not indicated because of the difficulty in interpreting the V/Q scan. In those cases CT angiogram might be useful. (Choices D and E) A quick and careful evaluation of the patient helps to determine the pre-test possibility of PE before performing more specific tests that will determine or exclude the diagnosis of PE. (Choice A) Serum troponin I level would have been indicated if this patient had chest pain or EKG changes suggestive of coronary artery disease. (Choice C) ABG is non-specific. It is not indicated before the chest-x ray. Educational Objective: A chest x-ray should be obtained in all post-operative patients who complain of shortness of breath before performing more specific tests that will determine or exclude the diagnosis of PE. Chest x-ray findings may suggest the presence of PE and may help exclude other chest pathology. A 32-year-old Caucasian registered nurse is admitted to the hospital for rigors and high-grade fevers for the past two days. Her blood cultures on the next day reveal the presence of Staphylococcus aureus, group-B streptococci, and Peptostreptococcus. A review of her past medical records reveals that she has been admitted to the hospital twice in the past six months. During her last admission, her blood cultures yielded Enterobacter faecalis, Streptococcus species, and Bacteroides. During that admission, she gradually improved with intravenous antibiotics, and her repeat blood cultures were negative. She again had a relapse of her symptoms, and was found to have bacteremia with two

anaerobic organisms. Her current vital signs are as follows: temperature 38.9C(102F), blood pressure 110/64 mmHg, heart rate 102/min, and respiratory rate 18/min. The rest of her physical examination is unremarkable. Which of the following is the most likely explanation for her symptoms? A. The patient is injecting exogenous material parenterally. B. She has a polymicrobial subacute bacterial endocarditis. C. The patient is suffering from subacute appendicular abscess. D. She has a multidrug-resistant bacterial infection. E. Her symptoms can be explained by a dental abscess.

Explanation: The occurrence of repeated episodes of bacteremia and sepsis with multiple, different organisms in a young, female healthcare professional is highly suspicious for a factitious disorder. Factitious disorder is a psychiatric condition in which the patient feigns an illness to be a patient and to assume a sick role. The patient typically does not have an external incentive other than to be a patient, which differentiates this condition from malingering. It is usually seen in young women and healthcare professionals. It can present in a variety of ways, ranging from a mild exaggeration of symptoms to a much more dramatic form, with patients seeking multiple invasive procedures and operations (Munchausen?s syndrome). Patients may fake their illnesses by either lying about their history and symptoms, or by creating an illness by the ingestion of drugs or by the injection of exogenous materials such as sputum, urine, feces, or milk. The patient in the above vignette appears to be injecting exogenous materials such as sputum or feces. This can cause a polymicrobial bacteremia or sepsis with the organisms described above. (Choice B) Bacterial endocarditis can cause persistent bacteremia, but is typically associated with similar bacterial organisms. It is rarely polymicrobial. (Choices C and E) The patient does not have any signs or symptoms suggestive of an appendicular abscess or a dental abscess. Moreover, repeated acute episodes with different bacteria are not consistent with a localized abscess. (Choice D) Drug resistance can develop during the therapy for bacterial infections; however, polymicrobial bacteremia on different occasions after a complete response is not typical for drug resistance. Educational Objective: Repeated hospital admissions due to polymicrobial bacteremia in a healthcare professional should raise your suspicion for a factitious disorder. The following vignette applies to the next 2 items An 82-year-old Caucasian female comes to the emergency department for the evaluation of right ear pain and drainage for the past two days. She saw her primary care physician approximately one week ago for decreased hearing from the right ear, and he performed aural irrigation to remove impacted cerumen in her right ear. She has a history of hypertension, diabetes mellitus, and rheumatoid arthritis. She denies any history of smoking or alcohol abuse. Her medications include hydrochlorothiazide, enalapril, glyburide, and low-dose prednisone. On physical examination, her temperature is 37.2C (99F), blood pressure is 146/74 mmHg, heart rate is 100/min, and respiratory rate is 16/min. There is marked tenderness with motion of the right earlobe. Purulent discharge and granulation tissue is noted on the floor of the right external auditory canal at the osseocartilaginous junction. The tympanic membrane appears intact.

Item 1 of 2 Which of the following is the most likely cause of the above findings? A. Topical tobramycin B. Malignant otitis externa C. Ramsay Hunt syndrome D. Acute mastoiditis E. Her symptoms can be explained by a dental abscess. Explanation: This is a classic presentation of malignant otitis externa (also known as malignant external otitis), which is an invasive infection of the external auditory canal and the bones forming the skull base. The infection usually begins as external otitis and progresses rapidly to involve the adjacent bones at the base of the skull. Patients have marked pain (otalgia) and purulent drainage or discharge from the ear (otorrhea). The finding of granulation tissue at the floor of the bone-cartilage junction in the external auditory canal is pathognomonic of this condition. The tympanic membrane is usually intact. Malignant otitis externa is typically seen in elderly patients with diabetes mellitus. Patients with HIV disease and other immunocompromised states are also at a higher risk of having malignant otitis externa. A number of patients with malignant otitis externa usually have an associated history of aural irrigation for the removal of cerumen. Pseudomonas aeruginosa is the usual causative organism in these patients. An untreated infection can progress rapidly to involve the skull base, temporomandibular joint, and cranial nerves, causing osteomyelitis and cranial nerve palsies. (Choice A) Acute otitis media is associated with an inflamed, erythematous, bulging and immobile tympanic membrane due to the presence of fluid in the middle ear. (Choice C) Ramsay Hunt syndrome (also known as herpes zoster oticus) is an ear manifestation of a reactivated varicella zoster virus. It is characterized by a triad of ear pain, vesicles in the external auditory canal, and ipsilateral facial paralysis. (Choice D) Acute mastoiditis is usually seen as a complication of acute otitis media. Educational Objective: Malignant otitis externa is typically seen in elderly, diabetic patients, and is characterized by severe pain and the presence of granulation tissue on the floor of the external auditory canal at the osseocartilaginous junction. Item 2 of 2 Which of the following is the most appropriate treatment for this patient?s condition? A. Topical tobramycin B. Oral amoxicillin C. Intravenous acyclovir D. Intravenous ciprofloxacin E. Acetic acid drops Explanation: Pseudomonas aeruginosa is the usual causative organism in almost all the cases of malignant otitis externa. Anti-pseudomonal antibiotic therapy is therefore the treatment of choice. Fluoroquinolones (ciprofloxacin), anti-pseudomonal penicillins (piperacillin, ticarcillin) with or without aminoglycosides, and third generation cephalosporins (ceftazidime) are all effective in the treatment of malignant external otitis. All patients should be treated with intravenous antibiotics initially, and then switched to oral antibiotics (depending on the clinical response), to complete 6-8 weeks of antibiotic therapy. (Choice A) Topical antibiotics have no role in the treatment of malignant external otitis. (Choice B) Oral amoxicillin is not effective against Pseudomonas aeruginosa, and should not be used. (Choice C) Intravenous acyclovir is used in the management of patients with Ramsay Hunt syndrome. (Choice E) Topical acetic acid (and other acidifying agents) is generally used in patients with mild external otitis. It has no role in the treatment of malignant otitis externa.

Educational Objective: Systemic anti-pseudomonal antibiotics should be used in the treatment of malignant external otitis. A 55-year-old Caucasian male comes to your office for a regular follow-up visit. He had an allograft liver transplantation two years ago for chronic hepatitis C. He is currently receiving immunosuppressive therapy with cyclosporine and tacrolimus. His most recent laboratory test results are as follows: Total bilirubin: 1.0 mg/dl AST: 200 IU/L ALT: 290 IU/L Alkaline phosphatase: 200 IU/L Serum albumin: 3.6 g/dl PT: 12 sec A liver biopsy is done, and this reveals portal tract inflammation with piecemeal necrosis and scattered lobular inflammation. What is the most likely diagnosis in this patient? A. Acute liver failure B. Cytomegalovirus (CMV) hepatitis C. Recurrent chronic hepatitis C D. Toxicity from the immunosuppressant treatment E. Hepatic artery thrombosis Explanation: Abnormalities of liver enzymes are common findings after liver transplantation. Predominant aminotransferase elevations may be due to recurrent hepatitis C or hepatitis B infection, CMV hepatitis, ischemic reperfusion injury, drugs, or allograft rejection. In contrast, predominant elevations of alkaline phosphatase and GGT levels are indicative of biliary complication or hepatic artery thrombosis. Almost all patients with Hepatitis C infection who undergo transplantation have a documented recurrence of hepatitis C infection in the transplanted liver; however, the severity of hepatitis C infection varies, and clinical consequences are limited to the first five years. Histological studies show the presence of moderate to severe chronic hepatitis in more than half of all transplant patients, and bridging fibrosis or cirrhosis in approximately 10%. (Choice A) This patient has normal prothrombin time, normal serum albumin levels, and no symptoms suggestive of acute liver failure. (Choice B) CMV hepatitis is characterized by flu-like symptoms, unexplained fever, constitutional complaints, leukopenia, and an increase in hepatocellular enzyme levels. CMV hepatitis depends on the immune suppression of the patient. (Choice E) Hepatic artery thrombosis typically presents with features of cholangitis such as fever, elevated liver enzyme levels, and jaundice. (Choice D) Cyclosporine and tacrolimus are the most commonly used agents in immunosuppressive therapy for liver transplantation. The side effects of these drugs include nephrotoxicity, neurotoxicity, hypertension, hirsutism, gingival hyperplasia, and susceptibility to opportunistic infection and malignancy. Educational Objective:

Almost all patients with hepatitis C infection who undergo transplantation have a documented recurrence of hepatitis C infection in the transplant liver. The following vignette applies to the next 2 items A 40-year-old Caucasian female is brought to the emergency department (ED) by the paramedics after she was involved in a motor vehicle accident. She was the driver of the vehicle and was wearing a seatbelt. Her car was hit by another car coming from the opposite side, and then went off the road and hit a sideline tree. When the medics arrived at the scene of the accident, she was alert, oriented and complained of mild chest pain and shortness of breath. In the ED, she tells you that she has had a dry cough for the past week, but denies any associated fever, chills, difficulty breathing or expectoration with the cough. She does not smoke or drink alcohol. Her temperature is 36.7 C (98F), blood pressure is 126/82 mmHg, respirations are 22/min, and heart rate is 106/min. Physical examination reveals an area of tenderness over the left chest wall where the seatbelt was placed. Her lung examination reveals decreased breath sounds over the left side. An initial EKG reveals sinus tachycardia. The chest radiograph shows a bilateral homogenous opacification of the upper zones of the lung, more pronounced on the left side. Item 1 of 2 What is the most likely diagnosis based on the patients clinical presentation? A. Pulmonary edema B. Pneumonia C. Pulmonary contusion D. Pulmonary infarction E. Pleural effusion Explanation: Based on the clinical presentation, this patient most likely suffered from a pulmonary contusion during the accident. It is the most common lung parenchymal injury in patients with blunt chest trauma from any cause. The forces associated with blunt trauma are transmitted to the lung parenchyma and results in contusion with hemorrhage into the lung tissue. The clinical presentation depends on the extent of lung injury. The patients usually present with varying degrees of dyspnea, tachypnea, hypoxemia, and hemoptysis. Physical examination reveals decreased breath sounds over the affected area of the lungs. The diagnosis of pulmonary contusion may be missed initially, as it may take several hours for clinical and radiographic abnormalities to develop after the initial trauma. Chest radiography generally reveals homogenous opacification of the lung fields that do not confirm to a specific anatomic segment of the lung. Treatment is mainly supportive and the resolution of symptoms generally occurs in three to five days in mild cases. (Choice A) Patients with pulmonary contusion are prone to develop pulmonary edema with overly aggressive fluid resuscitation; however, the patient's clinical presentation is not consistent with pulmonary edema at this point. (Choices B, D & E) The clinical and radiographic presentation in this patient is not consistent with a diagnosis of pneumonia, pulmonary infarction or pleural effusion. Educational Objective: Pulmonary contusion is the most common lung parenchymal injury seen in patients with blunt chest trauma. Chest radiography generally reveals homogenous opacification of the lung fields that do not confirm to a specific anatomic segment of the lung. Item 2 of 2 Which of the following is the most appropriate management for this patient? A. Admit her to the hospital and start intravenous antibiotics B. Admit her to the hospital and monitor for 24 to 48 hours C. Obtain a V/Q scan D. Admit her to the hospital and start diuretics and fluid restriction E. Admit her to the hospital, perform a thoracentesis and start her on antibiotics

Explanation: The full clinical and radiographic features of pulmonary contusion may take several hours to develop after the initial blunt injury. Since the patients are at risk for late clinical deterioration, they should be admitted to the hospital after the initial injury and monitored for 24 to 48 hours for any signs of clinical deterioration. The symptoms of mild pulmonary contusion usually resolve within three to five days. In the initial phase, management is generally conservative with aggressive pulmonary toilet, supplemental oxygen, pain control and careful fluid management to prevent worsening edema in the injured lung. In patients with significant lung parenchymal damage, endotracheal intubation and mechanical ventilation may be necessary to provide respiratory support until the resolution of parenchymal injury. (Choice C) The patient should be admitted to the hospital for continuous monitoring; however, there is no indication to start the patient on intravenous antibiotics or obtain a V/Q scan. (Choice D) Diuretic and fluids restriction have no role in the management of patients with pulmonary contusion. (Choice A) Thoracentesis is indicated only in patients with traumatic hemothorax or pneumothorax causing respiratory compromise. It has no role in the management of patients with pulmonary contusion. Educational Objective: All patients with significant blunt chest trauma and pulmonary contusion should be admitted to the hospital and monitored for signs of clinical deterioration. At his fathers insistence, a 17-year-old boy presents to the pediatricians office with a complaint of severe depression. He states that after his girlfriend ended their relationship last month, he has felt unable to complete his homework and uninterested in playing the sports he previously enjoyed. He spends most of his time in his room, refuses to come out for meals, and contends that "its too much effort to actually get out of bed." Upon further questioning, he admits that he is seriously considering committing suicide with a handgun his father keeps in an unlocked drawer. The boy states that he has not spoken to anyone else about his suicidal ideation. Although he has no close relatives or friends who have succeeded in committing suicide, his younger sister attempted to commit suicide last year. He refuses to contract for safety, but also insists that the contents of his discussion not be revealed to any other parties. What is the next best step in managing this situation? A. Maintain confidentiality; refer to psychiatrist for appointment tomorrow B. Maintain confidentiality; prescribe antidepressant with scheduled follow-up in two days C. Refuse to maintain confidentiality, and inform parents of situation with instructions watch son closely D. Refuse to maintain confidentiality, inform parents of situation, and prescribe antidepressant with scheduled follow-up in two days E. Refuse to maintain confidentiality; hospitalize patient immediately Explanation: Teenaged and adult patients should be informed that their private disclosures about psychiatric concerns will be kept confidential unless the patient is subsequently deemed to be a risk to himself or others. This is an extremely important legally mandated exception, and should be made clear at the beginning of any such discussion. Despite being a teenager, the boy in this case would be entitled to complete confidentiality if he were not suicidal but instead simply wanted to discuss his depression. Since he is suicidal, has weak social support and access to lethal means, and refuses to contract for safety, this boy must be hospitalized immediately because he is at great risk for committing suicide (Choice E). Maintaining confidentiality (Choices A and B) would be a poor choice because the boy is at high risk for committing suicide. His parents need to be informed of the problem and the boy must be hospitalized in an acute psychiatric setting for stabilization of his condition. An antidepressant may be helpful eventually, but cannot be expected to improve his mental state for some weeks.

Giving the parents responsibility for their imminently suicidal son (Choices C and D) is dangerous and inappropriate. They have sought professional medical advice precisely because the situation is already beyond their abilities. Their son requires immediate hospitalization and help from psychiatrists trained in addressing suicidal ideation. Educational Objective: Teenagers with serious suicidal ideation must be hospitalized and their parents informed of the situation. A 66-year-old man comes to the physician for the evaluation of left-sided shoulder pain, hoarseness of voice, and weight loss for the past two months. His past medical history includes hypertension, degenerative joint disease, and emphysema. He has an extensive history of smoking - he has smoked two packs of cigarettes a day for the last 40 years. On physical examination, his vital signs are all within normal limits. He has evidence of ptosis, and his left pupil is smaller than the right pupil. You admit the patient to the hospital. An initial CT scan confirms your suspicion of a Pancoast tumor. There is extensive local invasion of the tumor into the brachial plexus, vertebral bodies, and mediastinal structures. A bone scan done the next day is also positive for distant metastasis. Which of the following is the most appropriate treatment of choice for palliation of his symptoms? A. Urgent surgery consultation B. Chemotherapy C. Surgical resection D. Observation E. Refuse to maintain confidentiality; hospitalize patient immediately Explanation: The patient has a locally advanced and metastatic (stage IV) left-sided superior sulcus pulmonary or Pancoast tumor. He also has Horner?s syndrome, which is characterized by the presences of miosis, ptosis, anhydrosis, and enophthalmos in the ipsilateral eye. The presence of Horner?s syndrome and hoarseness of voice are consistent with local invasion of the paravertebral sympathetic chain and the recurrent laryngeal nerve, respectively. Most of these tumors are non-small cell lung cancer, and arise from the superior sulcus of the lungs. Radiation therapy with surgical resection is the most common treatment for Pancoast tumor; however, radiation therapy alone should be used in patients with distant metastatic disease for palliation of symptoms. (Choice C) Surgical resection is contraindicated in patients with extensive invasion of the brachial plexus, vertebral bodies, or mediastinal nodal involvement by the tumor. Educational Objective: Radiotherapy should be used as a primary treatment modality for symptom relief in patients with locally advanced and metastatic Pancoast tumor. A 64-year-old Caucasian man is brought to the emergency department because of nausea, vomiting, and abdominal distention. He denies any abdominal pain. His symptoms have been present for the past two days, and are progressively getting worse. His other medical problems include hypertension, hypercholesterolemia, coronary artery disease, and congestive heart failure, with an ejection fraction of 25-30%. He drinks 1-2 beers daily. His father had a myocardial infarction at age 68. His medications include aspirin, furosemide, metoprolol, and simvastatin. His temperature is 37.2 F (99 F), blood pressure is 130/70 mmHg, and heart rate is 98 beats/min. The abdomen is soft and nontender, but distended. Bowel sounds are decreased. There is no rebound tenderness or rigidity. The patients laboratory tests reveal: CBC Hb: 13 g/dL Ht: 38% Platelet count: 300,000/cmm Leukocyte count: 11,000/cmm Segmented neutrophils: 70% Bands: 3%

Lymphocytes: 27% Serum Chemistry Serum Na: 132 mEq/L Serum K: 2.7 mEq/L Chloride: 104 mEq/L Bicarbonates: 24 mEq/L BUN: 32 mg/dL Serum Creatinine: 1.2 mg/dL Calcium: 10 mg/dL Blood Glucose: 80 mg/dL Which of the following is the most appropriate next step in the management of this patient? A. Urgent surgery consultation B. Start IV potassium replacement C. Start antibiotic therapy D. Order an abdominal CT scan immediately E. Order an abdominal x-ray immediately Explanation: This patient has an acute abdomen syndrome. Findings from the physical examination and laboratory tests point towards hypokalemic paralytic ileus as the most probable etiology. This patient has been taking a loop-diuretic, which can produce hypokalemia and hyponatremia. Parenteral potassium replacement is necessary to treat the ileus, as well as to prevent any untoward cardiac complications associated with hypokalemia. (Choice C) There is no evidence of infection; therefore, there are no indications to begin antibiotic therapy. (Choice A) A surgery evaluation is not urgent. An underlying cause (hypokalemia) of the paralytic ileus has been identified, and should be addressed first in the management of this patient. (Choice E) Abdominal x-rays can confirm the ileus, showing dilatation of the gastric chamber, small bowel, and colon; however, potassium replacement and an EKG recording are more important at this moment. (Choice D) An abdominal CT scan is not very helpful in the management of paralytic ileus. Educational Objective: Diuretic-induced hypokalemia is a common complication of antihypertensive/congestive heart failure therapy, especially when loop-diuretics are used. Paralytic ileus is one of the indications for prompt parenteral potassium replacement.

The following Vignette applies to the next 2 items A 56-year-old Caucasian male presents to your office for a routine check-up. He has no present complaints. He has long standing hypertension, which is controlled with amlodipine. Last year, he was hospitalized due to right-sided pneumonia. Three months ago, he was diagnosed with acute bronchitis. He smokes one pack of cigarettes daily and consumes alcohol occasionally. He denies any recreational drug use. His blood pressure is 130/80 mmHg, pulse is 90/min, temperature is 36.7C (98F), and respirations are 18/min. Hemoglobin: 20.2 g/L Erythrocyte count: 6.8 mln/mm3 MCV: 90 fl Reticulocytes: 1.5% Platelets: 300,000/mm3 Leukocyte count: 5,000/mm3 Neutrophils: 58% Eosinophils: 2% Lymphocytes: 33% Monocytes: 7% Item 1 of 2 Which of the following is the most likely diagnosis in this patient? A. Polycythemia vera B. Relative polycythemia C. Secondary polycythemia D. Drug-induced polycythemia E. Thalassemia trait Explanation: Differential diagnosis of polycythemia may be difficult and may require sophisticated laboratory evaluation, such as measurement of serum erythropoietin levels and blood volume estimation. At the same time, a thorough history, physical examination and basic labs provide important clues to the correct diagnosis. Patients with secondary polycythemia present with appropriately high serum erythropoietin levels. Their erythropoietin production increases to compensate for hypoxemia which may be due to COPD, right-to-left cardiac shunt or high altitude residence. (Choices A and B) Unlike polycythemia vera or relative polycythemia, there is no increase in white blood cell count or platelet count in secondary polycythemia. (Choice D) Although some drugs (i.e., androgens and anabolic steroids) may result in polycythemia, Ca antagonists are not known to cause this blood disorder. (Choice E) Thalassemia trait may increase the RBC count; however, hemoglobin levels and MCV are decreased with this disease. Educational Objective: Erythropoietin production in patients with secondary polycythemia increases to compensate for hypoxemia. Unlike polycythemia vera, there is usually no increase in white blood cell count or platelet count in secondary polycythemia. Item 2 of 2 Which of the following is the best next step in the management of this patient? A. Oxygen saturation measurement B. Liver function tests and urinalysis C. Chest x-ray D. Serum erythropoietin level E. Serum erythropoietin level

Explanation: Polycythemia secondary to pulmonary diseases is the most common cause of polycythemia. A thorough history and physical examination may suggest the correct diagnosis. Low oxygen saturation on pulse oximetry, a very simple test, confirms the diagnosis. Sometimes, patients will not present with reduced oxygen saturation at rest or during the daytime. This is why pulse oximetry is obtained after minimal exertion in all patients. If suggested by the history, sleep studies may be necessary to determine the presence of nocturnal desaturation. (Choice C) A chest x-ray may help identify a pulmonary disease, but it does not confirm arterial desaturation, which actually leads to polycythemia. (Choice D) Serum erythropoietin levels are commonly measured in patients with polycythemia because it is an important diagnostic tool; however, this is not the first test to run in these patients. (Choice B) Liver function tests and urinalysis are employed when tumor polycythemia is suspected. (Choice E) Blood volume measurement is helpful in demonstrating relative polycythemia. Educational Objective: Pulse oximetry should be obtained after minimal exertion in patients with suspected secondary polycythemia. Sleep studies may sometimes be necessary to determine the presence of nocturnal desaturation

You might also like